You are on page 1of 404

Planche no 1.

Logique
* trs facile ** facile *** difficult moyenne **** difficile
I : Incontournable T : pour travailler et mmoriser le cours
Exercice no 1 (**IT)
Exprimer laide de quantificateurs les phrases suivantes puis donner leur ngation.
1) (dans cette question, f dsigne une fonction de R dans R)
a) f est la fonction nulle.
b) Lquation f(x) = 0 a une solution.
c) Lquation f(x) = 0 a exactement une solution.
d) Le dnominateur D de f sannule au moins une fois sur R.
e) f est lidentit de R (cest--dire la fonction qui, chaque rel, associe lui-mme).
f ) Le graphe de f coupe la droite dquation y = x.
g) f est croissante sur R.
h) Lquation sin x = x a une solution dans R.
2) (dans cette question, (un )nN est une suite relle)
a) La suite (un )nN est borne.
b) La suite (un )nN est croissante.
c) La suite (un )nN est monotone.
3) (dans cette question, f est une fonction du plan dans lui-mme)
a) f est lidentit du plan.
b) f a au moins un point invariant (on dit aussi point fixe).
4) Pour tout point M du plan P, M est sur le cercle C de centre et de rayon R si et seulement si la distance de M
vaut R.
Exercice no 2 (*IT) Donner la ngation des phrases suivantes
1) x > 3
2) 0 < x 6 2.
Exercice no 3 (**IT)
Les phrases suivantes sont-elles quivalentes ?
1) x R, (f(x) = 0 et g(x) = 0) et (x R, f(x) = 0) et (x R, g(x) = 0) .
2) x R, (f(x) = 0 ou g(x) = 0) et (x R, f(x) = 0) ou (x R, g(x) = 0) .
Donner un exemple de fonctions f et g de R dans R, toutes deux non nulles et dont le produit est nul.
Exercice no 4 (**IT)
Dans chacun des cas suivants, dire si la proposition est vraie ou fausse puis le dmontrer.
1) x R/ sin(x) = x.
2) x R, x2 + 1 6= 0.

3) x C, x2 + 1 6= 0.
Exercice no 5. (**IT)
1) Montrer que la fonction sin nest pas nulle.
2) Montrer que la fonction valeur absolue nest pas drivable sur R.
Exercice no 6. (**IT)
1) Montrer que la proposition : (x R/ cos x = 0) et (x R/ sin x = 0) est vraie.

2) Montrer que la proposition : (x R/ cos x = 0 et sin x = 0) est fausse.


Exercice no 7. (***IT)

Montrer que 2 est irrationnel.


http ://www.maths-france.fr

c Jean-Louis Rouget, 2014. Tous droits rservs.


Exercice no 8. (***IT)
Soient a et b deux entiers naturels non nuls. Montrer que (k N/ b = ka et k N/ a = kb) a = b.
Exercice no 9. (**IT)
Ecrire avec des quantificateurs les propositions suivantes puis dans chaque cas dire si la proposition est vraie ou fausse.
1) Tout entier naturel est pair ou impair.
2) Tout entier naturel est pair ou tout entier naturel est impair.
3) Pour chaque entier, on peut trouver un entier strictement plus grand.
4) Il y a un entier plus grand que tous les entiers.
Exercice no 10. (**IT)
Ecrire avec des quantificateurs les propositions suivantes :
1) f est constante sur R (o f est une fonction de R dans R).
2) f nest pas constante sur R.

http ://www.maths-france.fr

c Jean-Louis Rouget, 2014. Tous droits rservs.


Planche no 1. Logique : corrig


Exercice no 1
1) a) (f = 0 x R, f(x) = 0) et (f 6= 0 x R/ f(x) 6= 0).

b) (Lquation f(x) = 0 a (au moins) une solution si et seulement si x R/ f(x) = 0) et (lquation f(x) = 0 na pas de
solution si et seulement si x R, f(x) 6= 0).
c) (Lquation f(x) = 0 a exactement une solution si et seulement si ! x R/ f(x) = 0) et (lquation f(x) = 0 na pas
exactement une solution si et seulement si x R/ f(x) 6= 0 ou (x, x ) R2 / (x 6= x et f(x) = f(x ) = 0).
d) (Le dnominateur D de f sannule au moins une fois sur R si et seulement si x R/ D(x) = 0) et
(Le dnominateur D de f ne sannule pas sur R si et seulement si x R, D(x) 6= 0).
e) (f est lidentit de R si et seulement si x R, f(x) = x) et (f nest pas lidentit de R si et seulement si x R/ f(x) 6=
x).
f ) (Le graphe de f coupe la droite dquation y = x si et seulement si x R/ f(x) = x) et (Le graphe de f ne coupe pas
la droite dquation y = x si et seulement si x R/ f(x) 6= x).

g) (f est croissante sur R si et seulement si (x, x ) R2 , x 6 x f(x) 6 f(x )) et (f nest pas croissante sur R si et
seulement si (x, x ) R2 / x 6 x et f(x) > f(x )).

h) (Lquation sin x = x a une solution dans R si et seulement si x R/ sin x = x) et (Lquation sin x = x na pas de
solution dans R si et seulement si x R, sin x 6= x).
2) a) ((un )nN borne M R/ n N, |un | 6 M) et ((un )nN non borne M R/ n N, |un | > M).
b) ((un )nN croissante n N/ un+1 > un ) et ((un )nN non croissante n N/ un+1 < un ).

c) ((un )nN monotone (n N/ un+1 > un ) ou (n N/ un+1 6 un )) et


((un )nN non monotone ((n N/ un+1 < un ) et (n N/ un+1 > un )).

3) a) (f = IdP M P, f(M) = M) et (f 6= IdP M P/ f(M) 6= M).

b) (f a au moins un point fixe M P/ f(M) = M) et (f na pas de point fixe M P, f(M) 6= M).

Constatez que les phrases f(M) = M ou f(M) 6= M nont aucun sens si elles ne sont pas accompagnes de quantificateurs.
Exercice no 2
Le contraire de x > 3 est x < 3. Le contraire de 0 < x 6 2 est ((x 6 0) ou x > 2).
Exercice no 3
1) Oui. Dans les deux cas, chaque fois que lon se donne un rel x0 , f(x0 ) et g(x0 ) sont tous deux nuls.
2) Non. La deuxime affirmation implique la premire mais la premire nimplique pas la deuxime. La premire phrase
est la traduction avec des quantificateurs de lgalit fg = 0. La deuxime phrase est la traduction avec quantificateurs de
(f = 0 ou g = 0).
Voici un exemple de fonctions f et g toutes deux non nulles dont le produit est nul.
Soient f : R
R
et g : R
R
.
0 si x < 0
0 si x > 0
x 7
x 7
x si x > 0
x si x 6 0
Pour chaque valeur de x, on a soit f(x) = 0 (quand x 6 0), soit g(x) = 0 (quand x > 0). On a donc : x R, (f(x) =
0 ou g(x) = 0) ou encore x R, f(x)g(x) = 0 ou enfin, fg = 0. Cependant, f(1) = 1 6= 0 et donc f 6= 0, et g(1) = 1 6= 0
et donc g 6= 0. On a donc pas (f = 0 ou g = 0) ou encore, on na pas ((x R, f(x) = 0) ou (x R, g(x) = 0)).
Exercice no 4
1) La proposition x R/ sin(x) = x est vraie. En effet, soit x0 = 0. Alors sin (x0 ) = x0 .

2) La proposition x R, x2 + 1 6= 0 est vraie. En effet, soit x un rel. x2 + 1 > 1 et en particulier x2 + 1 6= 0. On a


ainsi montr que pour tout rel x, x2 + 1 6= 0.

3) La proposition x C, x2 + 1 6= 0 est fausse. Pour le dmontrer, on dmontre que sa ngation est vraie. La
ngation de cette proposition est x C, x2 + 1 = 0 . Cette proposition est effectivement vraie car le complexe x0 = i
vrifie x20 + 1 = 0.
Exercice no 5.

1) Puisque sin
6= 0, il existe une rel x tel que sin(x) 6= 0. Donc, la fonction sin nest pas nulle.
2
http ://www.maths-france.fr

c Jean-Louis Rouget, 2014. Tous droits rservs.


2) Une fonction est drivable sur R si et seulement si cette fonction est drivable en chaque rel. Donc, une fonction nest
pas drivable sur R si et seulement si cette fonction nest pas drivable en au moins un rel.
La fonction valeur absolue nest pas drivable en 0 et donc la fonction valeur absolue nest pas drivable sur R.
Exercice no 6.

1) Soit x0 = . Alors cos (x0 ) = 0. Donc la proposition x R/ cos x = 0 est vraie.


2
Soit x1 = 0. Alors sin (x1 ) = 0. Donc la proposition x R/ sin x = 0 est vraie.
Puisque les deux propositions x R/ cos x = 0 et x R/ sin x = 0 sont vraies, la proposition : (x
R/ cos x = 0) et (x R/ sin x = 0) est vraie.
2) Supposons par labsurde que la proposition : (x R/ cos x = 0 et sin x = 0) soit vraie. Soit x0 un rel tel que
cos (x0 ) = sin (x0 ) = 0. Alors, cos2 (x0 ) + sin2 (x0 ) = 0. Ceci contredit le fait que pour tout rel x, cos2 (x) + sin2 (x) = 1.
Donc, la proposition : (x R/ cos x = 0 et sin x = 0) est fausse.
Exercice no 7.
Supposons par labsurde que

a
ou
2 soit rationnel. Il existe alors deux entiers naturels non nuls a et b tel que 2 =
b

encore tels que a2 = 2b2 .

On peut poser a = 2 3 5 ... et b = 2 3 5 ... o , , . . . sont des entiers naturels.

Lgalit a2 = 2b2 scrit encore 22 32 52 ... = 22 +1 32 52 ... Ceux qui ont pris lenseignement de spcialit en
Terminale savent que cette galit implique que lexposant 2 est gal lexposant 2 + 1. Cette galit est impossible
car 2 est un entier pair et 2 +
1 est un entier impair.

Il tait donc absurde de supposer 2 rationnel. On a montr que 2 est irrationnel.


Exercice no 8.

Soient k0 et k1 deux entiers naturels tels que b = k0 a et a = k1 b. Alors, b = k0 k1 b puis k0 k1 = 1 car b nest pas nul.
Supposons par labsurde que lun des deux entiers naturels k0 ou k1 ne soit pas gal 1. Alors, (k0 > 2 et k1 > 1) ou
(k0 > 1 et k1 > 2). Dans les deux cas, on a k0 k1 > 2 et en particulier k0 k1 6= 1.
On a montr par labsurde que k0 = k1 = 1. Mais alors, a = b.
Exercice no 9.
1) nN, ((k N/ n = 2k) ou (k N/ n = 2k + 1)). Cette proposition est vraie car pour chaque n, lune des deux
propositions n est pair ou n est impair est vraie.
2) (nN, k N/ n = 2k) ou (nN, k N/ n = 2k + 1)). Cette proposition est fausse car chacune des deux propositions
tout entier naturel n est pair et tout entier naturel n est impair est fausse.
3) n N, m N/ n < m. Cette proposition est vraie. En effet, si n est un entier naturel, lentier m = n + 1 est
strictement plus grand que n.
4) m N/ n N, n < m. Cette proposition est fausse.
Exercice no 10.
Ecrire avec des quantificateurs les propositions suivantes :
1) f est constante sur R si et seulement si C R/ x R, f(x) = C.
On peut donner une dfinition plus simple. f est constante sur R si et seulement si x R, f(x) = f(0).
2) f nest pas constante sur R si et seulement si x R, f(x) 6= f(0).

http ://www.maths-france.fr

c Jean-Louis Rouget, 2014. Tous droits rservs.


Planche no 2. Raisonnement par rcurrence


* trs facile ** facile *** difficult moyenne **** difficile
I : Incontournable T : pour travailler et mmoriser le cours
Exercice no 1 (**T)
Montrer par rcurrence que, pour tout n N, 2n > n.
Exercice no 2 (**T)
Montrer par rcurrence que, pour tout n > 4, n! > n2 (o n! = 1 2 . . . n).
Exercice no 3 (***)
Montrer par rcurrence que, pour tout entier n > 2 est divisible par au moins un nombre premier.
Exercice no 4 (**T)
Soit (un )nN la suite dfinie par :
u0 = 2, u1 = 1 et pour tout entier naturel n, un+2 un+1 6un = 0.
Montrer par rcurrence que, pour tout n N, un = (2)n + 3n .
Exercice no 5 (***I)
1) Montrer par rcurrence que, pour tout naturel non nul n,

n
X
k=1

k=

n(n + 1)
. En calculant la diffrence (k + 1)2 k2 ,
2

trouver une dmonstration directe de ce rsultat.


n
n
n
X
X
X
2) Calculer de mme les sommes
k2 ,
k3 et
k4 (et mmoriser les rsultats). On donne les identits remarquables
k=1

k=1

k=1

(a + b)3 = a3 + 3a2 b + 3ab2 + b3 ,


(a + b)4 = a4 + 4a3 b + 6a2 b2 + 4ab3 + b4 et
(a + b)5 = a5 + 5a4 b + 10a3 b2 + 10a2 b3 + 5ab4 + b5 .
Exercice no 6 (**T)
1) Montrer par rcurrence que, pour tout n N ,

n
X
k=1

2) Montrer par rcurrence que, pour tout n N ,

n
1
=
. Trouver une dmonstration directe.
k(k + 1)
n+1

n
X
k=1

1
n(n + 3)
=
. Trouver une dmonstration
k(k + 1)(k + 2)
4(n + 1)(n + 2)

directe.
Exercice no 7 (****)
n
X
1
Pour n 1, on pose Hn =
. Montrer que, pour n > 2, Hn nest jamais un entier (indication : montrer par rcurrence
k
k=1
que Hn est le quotient dun entier impair par un entier pair en distinguant les cas o n est pair et n est impair).

http ://www.maths-france.fr

c Jean-Louis Rouget, 2014. Tous droits rservs.


Planche no 2. Raisonnement par rcurrence : corrig


Exercice no 1
Montrons par rcurrence que : n N, 2n > n.
Pour n = 0, 20 = 1 > 0. Lingalit dmontrer est donc vraie quand n = 0.
Soit n > 0. Supposons que 2n > n et montrons que 2n+1 > n + 1.
2n+1 = 2 2n
> 2(n + 1) (par hypothse de rcurrence)
=n+1+n+1
> n + 1.
On a montr par rcurrence que :
n N, 2n > n.
Exercice no 2
Montrons par rcurrence que : n > 4, n! > n2 .
Pour n = 4, 4! = 4 3 2 1 = 24 et 42 = 16. Puisque 24 > 16, lingalit dmontrer est donc vraie quand n = 4.
Soit n > 4. Supposons que n! > n2 et montrons que (n + 1)! > (n + 1)2 .
(n + 1)! = (n + 1) n!
> (n + 1) n2 (par hypothse de rcurrence).
Or, (n + 1) n2 (n + 1)2 = (n + 1)(n2 n 1) = (n + 1)(n(n 1) 1) > 5 (4 3 1) = 55 > 0 et donc
(n + 1) n2 > (n + 1)2 puis (n + 1)! > (n + 1)2 .
On a montr par rcurrence que :
n > 4, n! > n2 .
Exercice no 3
Montrons par rcurrence que : n > 2, n est divisible par au moins un nombre premier.
2 est divisible par 2 qui est un nombre premier. La proprit dmontrer est donc vraie quand n = 2.
Soit n > 2. Supposons que pour tout k J2, nK, k est divisible par au moins un nombre premier et montrons que n + 1
est divisible par au moins un nombre premier.
Si n + 1 est un nombre premier, n + 1 admet au moins un diviseur premier savoir lui-mme. Sinon, n + 1 nest pas
premier. Dans ce cas, il existe deux entiers a et b lments de J2, nK tels que n = a b. Par hypothse de rcurrence,
lentier a est divisible par au moins un nombre premier p. Lentier p divise lentier a et lentier a divise lentier n + 1.
Donc lentier p divise lentier n + 1.
Dans tous les cas, lentier n + 1 est divisible par au moins un nombre premier.
On a montr par rcurrence que tout entier suprieur ou gal 2 est divisible par au moins un nombre premier.
Exercice no 4
Montrons par rcurrence que : n N, un = (2)n + 3n .
(2)0 + 30 = 2 = u0 et (2)1 + 31 = 1 = u1 . Lgalit dmontrer est donc vraie quand n = 0 et n = 1.
Soit n > 0. Supposons que un = (2)n + 3n et que un+1 = (2)n+1 + 3n+1 et montrons que un+2 = (2)n+2 + 3n+2 .
un+2 = un+1 + 6un

= (2)n+1 + 3n+1 + 6 ((2)n + 3n ) (par hypothse de rcurrence)

= (2 + 6) (2)n+2 + (3 + 6) 3n = 4 (2)n+2 + 9 3n
= (2)2 (2)n+2 + 32 3n = (2)n+2 + 3n+2 .
On a montr par rcurrence que :
http ://www.maths-france.fr

c Jean-Louis Rouget, 2014. Tous droits rservs.


n N, (2)n + 3n .
Exercice no 5
n
X

1) Montrons par rcurrence que : n > 1,

k=

k=1

Pour n = 1,

1
X

k=1=

k=1

1 (1 + 1)
.
2

Soit n > 1. Supposons que

n
X

k=

k=1
n+1
X

n(n + 1)
.
2

k=

k=1

n
X

n+1
X
n(n + 1)
(n + 1)(n + 2)
et montrons que
k=
.
2
2
k=1

k + (n + 1) =

k=1

= (n + 1)
On a montr par rcurrence que :

n(n + 1)
+ (n + 1) (par hypothse de rcurrence)
2

 (n + 1)(n + 2)
+1 =
.
2
2

n

n > 1,

n
X

k=

k=1

n(n + 1)
.
2

On peut donner plusieurs dmonstrations directes.


1re demonstration. Pour k > 1, (k + 1)2 k2 = 2k + 1 et donc

n
X
k=1

(n + 1) 1 = 2

n
X

k + n ou encore 2

k=1

n
X

k = n + n ou enfin

k=1

n
X
k=1

n
n
X
X

k+
1 ce qui scrit
(k + 1)2 k2 = 2
k=1

k=1

n(n + 1)
k=
.
2

2me demonstration. On crit


1 +
n +

2
+
(n 1) +

+
+

3
(n 2)

... +
... +

(n 1) +
2
+

n =
1 =

S
S

et en additionnant (verticalement), on obtient 2S = (n + 1) + (n + 1) + . . . + (n + 1) = n(n + 1) do le rsultat. La mme


{z
}
|
n termes

dmonstration scrit avec le symbole sigma :


2S =

n
X
k=1

k+

n
X

(n + 1 k) =

k=1

n
X

(k + n + 1 k) =

k=1

n
X

(n + 1) = n(n + 1).

k=1

3me demonstration. On compte le nombre de points dun rectangle ayant n points de large et n + 1 points de long.
Il y en a n(n + 1). Ce rectangle se dcompose en deux triangles isocles contenant chacun 1 + 2 + ... + n points. Do le
rsultat.

..
.
..
.

..
.

...

...
..
.

..

..

..
...

...

..
.

4me dmonstration. Dans le triangle de Pascal, on sait que pour n et p entiers naturels donns,


n+1
. Donc, pour n > 2 (le rsultat est clair pour n = 1),
p+1
http ://www.maths-france.fr

  

n
n
+
=
p
p+1

c Jean-Louis Rouget, 2014. Tous droits rservs.


n
X

1 + 2 + ... + n = 1 +

C1k = 1 +

k=2

n
X

(C2k+1 C2k ) = 1 + (C2n+1 1) =

k=2

n(n + 1)
.
2

2) Pour k > 1, (k + 1)3 k3 = 3k2 + 3k + 1. Donc, pour n > 1 :


3

n
X

k2 + 3

k=1

n
X

k+

k=1

n
X

n
X

1=

k=1


(k + 1)3 k3 = (n + 1)3 1.

k=1

Do,
n
X

1
k =
3
2

k=1



n(n + 1)
1
1
3
(n + 1) 1 3
n = (2(n + 1)3 3n(n + 1) 2(n + 1)) = (n + 1)(2n2 + n),
2
6
6

et donc
n
X

n > 1,

k2 =

k=1

n(n + 1)(2n + 1)
.
6

Pour k 1, (k + 1)4 k4 = 4k3 + 6k2 + 4k + 1. Donc, pour n 1, on a


4

n
X

k3 + 6

k=1

n
X

n
X

k2 + 4

k=1

n
X

k+

k=1

1=

k=1

n
X
k=1

Do :
n
X
k=1


(k + 1)4 k4 = (n + 1)4 1.

 1
1
(n + 1)4 1 n(n + 1)(2n + 1) 2n(n + 1) n = ((n + 1)4 (n + 1)(n(2n + 1) + 2n + 1)
4
4

2
2
 (n + 1) (n + 1) (2n + 1)
n2 (n + 1)2
1
4
2
(n + 1) (n + 1) (2n + 1) =
=
=
4
4
4
!
2
n
n
X
X
n2 (n + 1)2
n > 1,
k3 =
=
k .
4

k3 =

k=1

k=1

Pour k > 1, (k + 1)5 k5 = 5k4 + 10k3 + 10k2 + 5k + 1. Donc, pour n > 1,


5

n
X

k4 + 10

k=1

n
X
k=1

k3 + 10

n
X

k2 + 5

k=1

n
X

n
X

k+

k=1

k=1

1=

n
X

((k + 1)5 k5 ) = (n + 1)5 1.

k=1

Do :
n
X
k=1

1
k =
5
4



5 2
5
5
5
2
(n + 1) 1 n (n + 1) n(n + 1)(2n + 1) n(n + 1) n
2
3
2

1
(6(n + 1)5 15n2 (n + 1)2 10n(n + 1)(2n + 1) 15n(n + 1) 6(n + 1))
30
n(n + 1)(6n3 + 9n2 + n 1)
1
(n + 1)(6n4 + 9n3 + n2 n) =
=
30
30
=

Finalement,
n N ,

n
X

k=

k=1
n
X

n N ,

k2 =

k=1

n N ,

n
X
k=1

n N ,

n
X
k=1

http ://www.maths-france.fr

n(n + 1)
2

n(n + 1)(2n + 1)
6

n2 (n + 1)2
=
k =
4
3

k4 =

n
X
k=1
2

!2

n(n + 1)(6n3 + 9n + n 1)
.
30

c Jean-Louis Rouget, 2014. Tous droits rservs.


Exercice no 6
n
X

1) Montrons par rcurrence que n > 1,

k=1

Pour n = 1,

1
X
k=1

n
1
=
.
k(k + 1)
n+1

1
1
1
= =
et la formule propose est vraie pour n = 1.
k(k + 1)
2
1+1

Soit n > 1. Supposons que

n
X
k=1

n+1
X
k=1

n+1
X
1
n
n+1
1
=
et montrons que
=
.
k(k + 1)
n+1
k(k + 1)
n+2
k=1

n
X

1
=
k(k + 1)

k=1

1
k(k + 1)

1
(n + 1)(n + 2)

1
n
+
(par hypothse de rcurrence)
n + 1 (n + 1)(n + 2)
n(n + 2) + 1
n2 + 2n + 1
=
=
(n + 1)(n + 2)
(n + 1)(n + 2)
n+1
(n + 1)2
=
=
(n + 1)(n + 2)
n+2

On a montr par rcurrence que :


n
X

n > 1,

k=1

1
n
=
.
k(k + 1)
n+1

Dmonstration directe. Pour k > 1,


(k + 1) k
1
1
1
=
=
,
k(k + 1)
k(k + 1)
k (k + 1)
et donc,
n
X
k=1

n
n
n
n+1
X
X
1 X
1
1 X1
1
=

k(k + 1)
k
(k + 1)
k
k
k=1

k=1

k=1

k=2

n
1
=
.
=1
n+1
n+1
2) Montrons par rcurrence que n > 1,

n
X
k=1

Pour n = 1,

1
X
k=1

1
1
1 (1 + 3)
= =
et la formule propose est vraie pour n = 1.
k(k + 1)(k + 2)
6
4 (1 + 1)(1 + 2)

Soit n > 1. Supposons que

n
X
k=1

n+1
X
k=1

n(n + 3)
1
=
.
k(k + 1)(k + 2)
4(n + 1)(n + 2)

n+1
X
1
n(n + 3)
(n + 1)(n + 4)
1
=
et montrons que
=
.
k(k + 1)(k + 2)
4(n + 1)(n + 2)
k(k + 1)(k + 2)
4(n + 2)(n + 3)
k=1

n
X

1
=
k(k + 1)(k + 2)
=

k=1

1
k(k + 1)(k + 2)

1
(n + 1)(n + 2)(n + 3)

1
n(n + 3)
+
(par hypothse de rcurrence)
4(n + 1)(n + 2) (n + 1)(n + 2)(n + 3)

n3 + 6n2 + 9n + 4
n(n + 3)2 + 4
=
4(n + 1)(n + 2)(n + 3)
4(n + 1)(n + 2)(n + 3)
2
(n + 1)(n + 4)
(n + 1)(n + 5n + 4)
=
=
4(n + 1)(n + 2)(n + 3)
4(n + 2)(n + 3)
=

On a montr par rcurrence que :


http ://www.maths-france.fr

c Jean-Louis Rouget, 2014. Tous droits rservs.


n > 1,

n
X
k=1

n(n + 3)
1
=
.
k(k + 1)(k + 2)
4(n + 1)(n + 2)

Dmonstration directe. Pour k > 1,


1
1 (k + 2) k
1
=
=
k(k + 1)(k + 2)
2 k(k + 1)(k + 2)
2

1
1

k(k + 1) (k + 1)(k + 2)

et donc,
n
X
k=1

!
!
n
n
n+1
X
X
1 X
1
1
1
1
=

k(k + 1)
(k + 1)(k + 2)
2
k(k + 1)
k(k + 1)
k=1
k=1
k=1
k=2


1 1
n2 + 3n
1
n(n + 3)
=
=

=
.
2 2 (n + 1)(n + 2)
4(n + 1)(n + 2)
4(n + 1)(n + 2)

1
1
=
k(k + 1)(k + 2)
2

n
X

Exercice no 7
pn
o qn est un entier pair et pn est un entier
qn
impair (la fraction prcdente ntant pas ncessairement irrductible mais coup sr pas un entier).
3
Pour n = 2, H2 = et H2 est bien du type annonc.
2
pk
o pk est un entier impair et qk est
Soit n > 2. Supposons que pour tout entier k tel que 2 6 k 6 n, on ait Hk =
qk
pn+1
un entier pair et montrons que Hn+1 =
o pn+1 est un entier impair et qn+1 est un entier pair.
qn+1
Montrons par rcurrence que, pour n > 2, Hn peut scrire sous la forme

pn
1
(n + 1)pn + qn
+
ne marche coup sur que si (n + 1)pn + qn est impair ce qui
=
qn n + 1
(n + 1)qn
est assur si n + 1 est impair et donc si n est pair).

(Recherche. Lide Hn+1 =

1
(2k + 1)pn + qn
pn
+
. (2k + 1)
=
qn
2k + 1
(2k + 1)qn
est pn sont impairs et donc (2k + 1)pn est impair puis (2k + 1)pn + qn est impair car qn est pair. Dautre part, qn est
pair et donc (2k + 1)qn est pair. Hn+1 est bien le quotient dun entier impair par un entier pair.

1er cas. Si n est pair, on peut poser n = 2k o k N . Dans ce cas, Hn+1 =

2me cas. Si n est impair, on pose n = 2k 1 o k > 2 (de sorte que 2k 1 > 3).

Hn+1 =

2k
X
1
i=1

k
k1
X
X 1
1
+
2i
2i + 1
i=1

i=0

(en sparant les fractions de dnominateurs pairs des fractions de dnominateurs impairs)
=

k1

k1

i=1

i=0

i=0

X 1
1
1X1 X 1
+
= Hk +
.
2
i
2i + 1
2
2i + 1

Maintenant, en rduisant au mme dnominateur et puisque un produit de nombres impairs est impair, on voit que
k1
X 1
K
est du type
o K et K sont des entiers. Ensuite, puisque 2 6 k 6 2k 1 = n, par hypothse de
2i + 1
2K + 1
i=0
pk
o pk est un entier impair et qk un entier pair. Aprs rduction au mme dnominateur, on obtient
rcurrence, Hk =
qk
K
(2K + 1)pk + 2Kqk
pk
+
=
.
Hn+1 =

2qk
2K + 1
2qk (2K + 1)
2Kqk est un entier pair et (2K + 1)pk est un entier impair en tant que produit de deux nombres impairs. Donc le
numrateur est bien un entier impair et puisque 2qk(2K + 1) est un entier pair, Hn+1 est encore une fois de la forme
dsire.
On a montr par rcurrence que pour tout naturel n > 2, Hn est le quotient dun entier impair par un entier pair et en
particulier nest pas un entier.

http ://www.maths-france.fr

c Jean-Louis Rouget, 2014. Tous droits rservs.


Planche no 3. Ensembles. Relations. Applications


* trs facile ** facile *** difficult moyenne **** difficile
I : Incontournable T : pour travailler et mmoriser le cours
Exercice no 1 (*T)
Soient E et F deux ensembles. Montrer que P(E) = P(F) E = F.
Exercice no 2 (**T)
A et B sont des parties dun ensemble E. Montrer que :
1) (AB = A B) (A = B = ).
2) (A B) (B C) (C A) = (A B) (B C) (C A).
3) AB = BA.
4) (AB)C = A(BC).
5) AB = A = B.
6) AC = BC A = B.
Exercice no 3 (**IT)
Soit R la relation dfinie dans R par :
(x, y) R2 , xRy xey = yex .
Montrer que R est une relation dquivalence sur R. Pour chaque rel x, prciser le nombre dlments de la classe
dquivalence de x.
Exercice no 4 (***)
Soit P lensemble des nombres complexes dont la partie imaginaire est strictement positive. On dfinit sur P la relation R
par
(z, z ) P2 , zRz R/ z =

z cos + sin
.
z sin + cos

Montrer que R est une relation dquivalence sur P.


Exercice no 5 (**IT)
Soit E un ensemble. Montrer que la relation dinclusion est une relation dordre sur P(E). Cette relation dordre est-elle
partielle ou totale ?
Exercice no 6 (***IT)
Dans chacun des cas suivants, dterminer f(I) puis vrifier que f ralise une bijection de I sur J = f(I) puis prciser f1 :
1) f(x) = x2 4x + 3, I =] , 2].
2x 1
, I =] 2, +[.
x+2



3
3) f(x) = 2x + 3 1, I = , + .
2
x
, I = R.
4) f(x) =
1 + |x|

2) f(x) =

Exercice no 7 (***IT)
Pour z C \ {i}, on pose f(z) =

z+i
. Montrer que f ralise une bijection de D = {z C/ |z| < 1} sur P = {z C/ Re(z) <
zi

0}. Prciser f1 .

http ://www.maths-france.fr

c Jean-Louis Rouget, 2014. Tous droits rservs.


Exercice no 8 (***IT)
Soient E un ensemble puis A une partie de E. Pour tout X P(E), on pose A (X) = X A et A (X) = X A. Montrer
que
1) A injective A surjective A = E.
2) A injective A surjective A = .
Exercice no 9 (*IT)
Soient f une application dun ensemble E vers un ensemble F et g une application de F vers un ensemble G.
Montrer que : (g f injective f injective) et (g f surjective g surjective).
Exercice no 10 (**I)
Soit f une application dun ensemble non vide E dans lui-mme telle que ff = f. Montrer que f est injective si et seulement
si f est surjective.
Exercice no 11 (**)
Parmi f g h, g h f et h f g deux sont injectives et une est surjective. Montrer que f, g et h sont bijectives.
Exercice no 12 (***IT)
f est une application dun ensemble E dans lui-mme. Montrer que :
1) a) f est injective X P(E), f1 (f(X)) = X.
b) f est injective (X, Y) P(E)2 , f(X Y) = f(X) f(Y).

2) f est surjective X P(E), f f1 (X) = X.
Exercice no 13 (***I) Thorme de Cantor :

1) Montrer quil existe une injection de E dans P(E).


2) En considrant la partie A = {x E/ x
/ f(x)}, montrer quil nexiste pas de bijection f de E sur P(E).

Exercice no 14 (****) (Une bijection entre N2 et N)


Soit f :

N2

N
. Montrer que f est une bijection. Prciser, pour n N donn, le couple
(x + y)(x + y + 1)
(x; y) 7 y +
2
(x, y) dont il est limage.

http ://www.maths-france.fr

c Jean-Louis Rouget, 2014. Tous droits rservs.


Planche no 3. Ensembles, relations, applications : corrig


Exercice no 1
Si E = F, alors P(E) = P(F).
Rciproquement, supposons que P(E) = P(F). F est un lment de P(F) et donc F est un lment P(E). Mais alors
F E. En changeant les rles de E et F on a aussi E F et finalement E = F.
Exercice no 2
1) Si A = B = alors AB = = A B.
Si AB = A B, supposons par exemple A 6= .
Soit x A. Si x B, x A B et donc x AB ce qui est absurde et si x
/ B, x AB et donc x A B ce qui est
absurde. Donc A = B = .
Finalement, AB = A B A = B = .
2) Par distributivit de sur ,

(A B) (B C) (C A) = ((A B) (A C) (B B) (B C)) (C A)

= ((A C) B) (C A) (car B B = B et A B B et B C B)
= (A C C) (A C A) (B C) (B A)

= (A B) (B C) (A C) (C A)
= (A B) (B C) (C A)
3) AB = (A \ B) (B \ A) = (B \ A) (A \ B) = BA.
4)

x (AB)C x est dans AB ou dans C mais pas dans les deux


((x A et x
/ B et x
/ C) ou (x B et x
/ A et x
/ C) ou (x C et x
/ AB)
x est dans une et une seule des trois parties ou dans les trois.

Par symtrie des rles de A, B et C, A(BC) est galement lensemble des lments qui sont dans une et une seule des
trois parties A, B ou C ou dans les trois. Donc (AB)C = A(BC). Ces deux ensembles peuvent donc se noter une
bonne fois pour toutes ABC.
5) A = B A \ B = et B \ A = AB = .
A 6= B x E/ ((x A et x
/ B) ou (x
/ A et x B)) x E/ x (A \ B) (B \ A) = AB AB 6= .

6) / Immdiat.

] Si A et B sont vides, alors A = B. Sinon, lune au moins des deux parties A ou B nest pas vide. Supposons sans perte
de gnralit que A nest pas vide. Soit x un lment de A.
Si x
/ C alors x AC = BC et donc x B car x
/ C.
Si x C alors x
/ AC = BC. Puis x
/ BC et x C et donc x B. Dans tous les cas, x est dans B. Tout lment de
A est dans B et donc A B.
En changeant les rles de A et B, on a aussi B A et finalement A = B.
Exercice no 3
Rflexivit. Pour tout rel x, on a xex = xex et donc, pour tout rel x, on a xRx. Par suite, la relation R est rflexive.
Symtrie. Soient x et y deux rels tels que xRy. On a donc xey = yex puis yex = xey et donc yRx. On a montr que
pour tous rels x et y, si xRy alors yRx. Par suite, la relation R est symtrique.
Transitivit. Soient x, y et z trois rels tels que xRy et yRz. On a donc xey = yex et yez = zey . On en dduit que
xez = xey ey ez = yex ey ez = yez ey ex = zey ey ex = zex
et donc xRz. On a montr que pour tous rels x, y et z, si xRy et yRz, alors xRz. Par suite, la relation R est transitive.
Finalement, la relation R est rflexive, symtrique et transitive. Par suite, la relation R est une relation dquivalence sur
R.
Soit x un rel. Dterminons le nombre dlments de la classe dquivalence de x.
Pour t R, posons f(t) = tet . f est drivable sur R et pour tout rel t, f (t) = (1 t)et . f est strictement croissante
sur ] , 1] et strictement dcroissante sur [1, +[, tend vers en et tend vers 0 en +. Le graphe de f est
http ://www.maths-france.fr

c Jean-Louis Rouget, 2014. Tous droits rservs.


1
2
3
4
5
Pour tout rel y,
xRy xey = yex xex = yey f(x) = f(y).

Ltude de f montre que si x ] , 0] {1}, la classe de x est un singleton et si x ]0, 1[]1, +[, la classe de x est
constitue de deux lments distincts.
Exercice no 4

z cos(0) + sin(0)
z cos() + sin()
, il existe un rel tel que z =
et
z sin(0) + cos(0)
z sin() + cos()
donc zRz. Ceci montre que R est un lment de P.
Rflexivit. Soit z un lment de P. Puisque z =

z cos() + sin()
. On en
z sin() + cos()
dduit que z (z sin() + cos()) = z cos() + sin() puis que z(cos() + z sin()) = z cos() sin().
Symtrie. Soient z et z deux lments de P tels que zRz . Il existe un rel tel que z =

Supposons cos() + z sin() = 0. On ne peut avoir sin() = 0 car alors cos() = 0 ce qui est absurde, les fonctions sin et
cos()
et en particulier z est un rel. Ceci est absurde
cos ne sannulant pas simultanment. Donc, sin() 6= 0 puis z =
sin()
car la partie imaginaire de z nest pas nulle et donc cos() + z sin() 6= 0.
On peut alors crire
z=
Le rel = est tel que z =

z cos() sin()
z cos() + sin()
=
.

cos() + z sin()
z sin() + cos()

z cos( ) sin( )
et donc z Rz.
z sin( ) + cos( )

On a montr que pour tous lments z et z de P, si zRz alors z Rz. Par suite, la relation R est symtrique.
Transitivit. Soient z, z et z trois lments de P tels que zRz et z Rz . Il existe donc des rels et tels que
z cos() + sin()
z cos( ) + sin( )
z =
et z =
. On en dduit que
z sin() + cos()
z sin( ) + cos( )
z cos() + sin()
cos( ) + sin( )

z
cos(
)
+
sin(
)
z
sin() + cos()

=
z =
z cos() + sin()
z sin( ) + cos( )

sin( ) + cos( )
z sin() + cos()
(z cos() + sin()) cos( ) + (z sin() + cos()) sin( )
=
(z cos() + sin()) sin( ) + (z sin() + cos()) cos( )
z(cos() cos( ) sin() sin( ) + sin() cos( ) + cos() sin( )
=
z(sin() cos( ) + cos() sin( )) + cos() cos( ) sin() sin( )
z cos( + ) + sin( + )
.
=
z sin( + ) + cos( + )

http ://www.maths-france.fr

c Jean-Louis Rouget, 2014. Tous droits rservs.


z cos( ) sin( )
et donc zRz . On a montr que pour tous lments z, z et z
z sin( ) + cos( )
de P, si zRz et z Rz , alors zRz . Par suite, la relation R est transitive.
Le rel = + est tel que z =

Finalement, la relation R est rflexive, symtrique et transitive. Par suite, R est une relation dquivalence sur P.
On peut montrer que les classes dquivalences pour la relation R sont des cercles centrs sur laxe des ordonnes.
Exercice no 5
Rflexivit. Pour tout lment A de P(E), on a A A. Par suite, la relation est rflexive.
Anti-symtrie. Soient A et B deux lments de P(E) tels que A B et B A. Alors A = B. Par suite, la relation
est anti-symtrique.
Transitivit. Soient A, B et C trois lments de P(E) tels que A B et B C. Alors A C. On en dduit que la
relation est transitive.
Finalement, la relation est rflexive, anti-symtrique et transitive. Par suite, la relation est une relation dordre sur
P(E).
Si E contient au moins deux lments distincts x et y, posons A = {x} et B = {y}. On a A 6 B et B 6 A. Donc, P(E)
contient au moins deux lments non comparables ou encore la relation est une relation dordre partielle.
Si E est vide ou un singleton, est une relation dordre totale sur P(E).
Exercice no 6
1) f est drivable et donc continue sur I =] , 2], et pour x ] , 2[, f (x) = 2x 4 < 0. f est ainsi continue et
strictement dcroissante sur ] , 2].


f ralise donc une bijection de ] , 2] sur f(] , 2]) = f(2), lim f(x) = [1, +[= J. On note g lapplication
x

de I dans J qui, x associe x 4x + 3(= f(x)). g est bijective et admet donc une rciproque. Dterminons g1 . Soit
y [1, +[ et x ] , 2].
2

y = g(x) y = x2 4x + 3 x2 4x + 3 y = 0.

= 4 (3 y) = y + 1 > 0. Donc, x = 2 + y + 1 ou x = 2 y + 1. Enfin, x ] , 2] et donc, x = 2 y + 1. En


rsum,
p
x ] , 2], y [1, +[, y = g(x) x = 2 y + 1.
On vient de trouver g1 :

x [1, +[, g1 (x) = 2

x+1

2) Je vous laisse vrifier que f ralise une bijection de ] 2, +[ sur ] , 2[, note g. Soient alors x ] 2, +[ et
y ] , 2[.
y = g(x) y =

2x 1
2y + 1
y(x + 2) = 2x 1 x(y + 2) = 2y + 1 x =
.
x+2
y + 2

2y + 1
, mais il nest pas ncessaire de vrifier que cette expression
y + 2
est bien dfinie et lment de ] 2, +[ car on sait lavance que y admet au moins un antcdent dans ] 2, +[), et
cest donc ncessairement le bon). En rsum,

(on a ainsi trouv au plus une valeur pour x savoir x =

On vient de trouver g1 :

x ] 2, +[, y ] , 2[, y = g(x) x =


x ] , 2[, g1 (x) =

2y + 1
.
y + 2

2x + 1
x + 2

.



3
3
3) f est continue et strictement croissante sur , + et donc bijective de , + sur
2
2

   

3
3
, lim f(x) = [1, +[.
= f
f , +
2
2 x


http ://www.maths-france.fr

c Jean-Louis Rouget, 2014. Tous droits rservs.






3
3
Notons g lapplication de , + dans [1, +[ qui x associe 2x + 3 1. Soient alors x , + et y
2
2
[1, +[.


y2
1
2x + 3 1 = y x = (3 + (y + 1)2 ) x =
+ y 1.
2
2


3
Comme g est une bijection, le rel x obtenu convient ncessairement. En rsum, x , + , y [1, +[, y =
2
y2
+ y 1. On vient de trouver g1 :
g(x) x =
2
g(x) = y

x [1, +[, g1 (x) =

x2
+ x 1.
2

4) f est dfinie sur R.


1+x
x
<
= 1. Donc, f([0, +[) [0, 1[.
1+x
1+x
x
x1
Pour x ] , 0], 1 x > 0 et donc 0 > f(x) =
>
= 1. Donc, f(] , 0]) ] 1, 0].
1x
1x
Finalement, f(R) ] 1, 1[.
Pour x [0, +[, 0 6 f(x) =

Vrifions alors que f ralise une bijection de R sur ] 1, 1[.


Soit y [0, 1[ et x R. Lgalit f(x) = y impose x dtre dans [0, +[. Mais alors
f(x) = y

x
y
= y x = y(1 + x) x(1 y) = y x =
.
1+x
1y

Le rel x obtenu est bien dfini, car y 6= 1, et positif, car y [0, 1[. On a montr que :
y [0, 1[, !x R/ y = f(x) ( savoir x =

y
).
1y

Soit y ] 1, 0[ et x R. Lgalit f(x) = y impose x dtre dans ] , 0[. Mais alors


f(x) = y

x
y
= y x = y(1 x) x =
.
1x
1+y

Le rel x obtenu est bien dfini, car y 6= 1, et strictement ngatif, car y ] 1, 0[. On a montr que :
y ] 1, 0[, !x R/ y = f(x) ( savoir x =

y
).
1+y

Finalement,
y ] 1, 1[, !x R/ y = f(x),

y
ce qui montre que f ralise une bijection de R sur ] 1, 1[. De plus, pour y ] 1, 1[ donn, f1 (y) =
si y > 0 et
1y
y
y
f1 (y) =
si y < 0. Dans tous les cas, on a f1 (y) =
.
1+y
1 |y|
x
En notant encore f lapplication de R dans ] 1, 1[ qui x associe
, on a donc
1 + |x|
x ] 1, 1[, f1 (x) =

x
.
1 |x|

Exercice no 7
1) Montrons que la restriction de f D, note g, est bien une application de D dans P.
Soit z D. On a |z| < 1 et en particulier z 6= i. Donc, f(z) existe. De plus,


1 z+i zi
1 2zz 2
|z|2 1
1
=
=
< 0.
+
Re(f(z)) = (f(z) + f(z)) =
2
2 zi zi
2 (z i)(z i)
|z i|2
Donc, f(z) est lment de P. g est donc une application de D dans P.
2) Montrons que g est injective. Soit (z, z ) D2 .
http ://www.maths-france.fr

c Jean-Louis Rouget, 2014. Tous droits rservs.


g(z) = g(z )

Donc g est injective.

z+i
z + i
=
zz + iz iz + 1 = zz + iz iz + 1 2i(z z) = 0 z = z .
zi
z i

3) Montrons que g est surjective. Soient z D et Z P.


g(z) = Z

i(Z + 1)
z+i
= Z z + i = zZ iZ z(Z 1) = i(Z + 1) z =
,
zi
Z1

(ce qui montre que Z admet au plus un antcdent dans D, savoir z =


Il reste cependant vrifier que

i(Z + 1)
(mais on le sait dj car g est injective).
Z1

i(Z + 1)
est dfini et est effectivement dans D).
Z1

i(Z + 1)
Rciproquement, le nombre
est bien dfini puisque Z est dans P et donc Z 6= 1. De plus, puisque Re(Z) < 0,
Z1


i(Z + 1)
i(Z + 1)
|Z + 1|


Z 1 = |Z 1| < 1 (Z tant strictement plus proche de 1 que de 1) et donc Z 1 D. Finalement g est une
bijection de D sur P, et :
z P, g1 (z) =

i(z + 1)
.
z1

Exercice no 8
1) Si A = E, pour tout X de P(E), A (X) = X E = X et donc A = IdP(E) . Dans ce cas, A est injective et surjective.
, Soit A une partie de E, distincte de E. Vrifions que A nest ni injective, ni surjective.
Puisque A 6= E, il existe un lment x0 de E qui nest pas dans A. Soient B = et C = {x0 }. On a
A (B) = B A = = C A = A (C)
mais B 6= C. Donc, A nest pas injective. Dautre part, pour tout X de P(E), A X est contenue dans A et en particulier
ne peut tre gale E. Donc, E na pas dantcdent par A . Ceci montre que A nest pas surjective.
En rsum, si A = E, A est injective et surjective et si A 6= E, A nest ni injective, ni surjective. On a donc montr
que : A injective A surjective A = E.

2) Si A = , pour tout X de P(E), A (X) = X = X et donc A = IdP(E) . Dans ce cas, A est injective et surjective.
, Soit A une partie de E, distincte de . Vrifions que A nest ni injective, ni surjective.
Puisque A 6= , il existe un lment x0 de A. Soient B = et C = {x0 }. Puisque x0 est dans A, on a
A (B) = B A = A = C A = A (C)
mais B 6= C. Donc, A nest pas injective. Dautre part, pour tout X de P(E), A X contient A et en particulier ne peut
tre gale . Donc, na pas dantcdent par A . Ceci montre que A nest pas surjective.

En rsum, si A = , A est injective et surjective et si A 6= , A nest ni injective, ni surjective. On a donc montr


que : A injective A surjective A = .

Exercice no 9

1) Soit (x1 , x2 ) E2 .

f(x1 ) = f(x2 ) g(f(x1 )) = g(f(x2 )) (car g est une application)


x1 = x2 (car g f est injective).

On a montr que (x1 , x2 ) E2 , f(x1 ) = f(x2 ) x1 = x2 , et donc f est injective.

2) Soit y H. Puisque g f est surjective, il existe un lment x dans E tel que g(f(x)) = y. En posant z = f(x), z est un
lment de F tel que g(z) = y. On a montr : y G, z F/ g(z) = y, et donc g est surjective.
Exercice no 10
Supposons f injective. Soit x un lment de E. Par hypothse, f(f(x)) = f(x). Puisque f est injective, on en dduit que
f(x) = x.
http ://www.maths-france.fr

c Jean-Louis Rouget, 2014. Tous droits rservs.


Ainsi, pour tout x de E, f(x) = x et donc f = IdE . En particulier, f est bijective et en particulier, f est surjective.
Supposons f surjective. Soit x1 et x2 deux lments de E. Puisque f est surjective, il existe deux lments y1 et y2 de E
tels que x1 = f(y1 ) et x2 = f(y2 ).
f(x1 ) = f(x2 ) f(f(y1 )) = f(f(y2 )) f(y1 ) = f(y2 ) (car f f = f)
x1 = x2 .

Donc, f est injective puis f est bijective. On note de nouveau que puisque f est injective, ncessairement f = IdE .
Remarque. Si on sait que f est bijective, on peut crire
f f = f f f f1 = f f1 f = IdE .

Exercice no 11

On peut supposer sans perte de gnralit que f g h et g h f sont injectives et que h f g est surjective. Daprs
le no 9, puisque f g h = (f g) h est injective, h est injective et puisque h f g = h (f g) est surjective, h est
surjective.
Dj h est bijective. Mais alors, h1 est surjective et donc f g = h1 (h f g) est surjective en tant que compose de
surjections. Puis h1 est injective et donc f g = (f g h) h1 est injective. f g est donc bijective. f g est surjective
donc f est surjective. g h f est injective donc f est injective. Donc f est bijective. Enfin g = f1 (f g) est bijective en
tant que compose de bijections.
Exercice no 12
1) a) Supposons f injective.
Soit X P(E). On a toujours X f1 (f(X)). (x X f(x) f(X) x f1 (f(X))).
Rciproquement, soit x E.
x f1 (f(X)) f(x) f(X) x X/ f(x) = f(x ) x X/ x = x (puisque f est injective)
x X.

Finalement, f1 (f(X)) X et donc f1 (f(X)) = X.

Supposons que pour tout X de P(E), f1 (f(X)) = X. Soit x X. Par hypothse, f1 {f(x)} = f1 (f({x})) = {x} ce
qui signifie que f(x) a un et un seul antcdent savoir x. Par suite, tout lment de lensemble darrive a au plus un
antcdent par f et f est injective.
b) Supposons f injective. Soit (X, Y) (P(E))2 . On a toujours f(X Y) f(X) f(Y) (X Y X f(X Y) f(X) et
de mme, f(X Y) f(Y) et finalement, f(X Y) f(X) f(Y)).
Rciproquement, soit y F. y f(X) f(Y) (x, x ) X Y/ y = f(x) = f(x ). Mais alors, puisque f est injective,
x = x X Y puis y = f(x) f(X Y). Finalement, f(X Y) = f(X) f(Y).

Supposons que pour tout (X, Y) (P(E))2 , on a f(X Y) = f(X) f(Y).


Soit (x1 , x2 ) E2 tel que f(x1 ) = f(x2 ). Posons X = {x1 } et Y = {x2 }. Par hypothse f(X Y) = f(X) f(Y) ce qui fournit
f ({x1 } {x2 }) = f({x1 }) f({x2 }) = {f(x1 )} {f(x2 )} = {f(x1 )}.

En particulier, f({x1 } {x2 }) 6= ce qui impose {x1 } {x2 } 6= puis x1 = x2 . Donc f est injective.

2) Supposons f surjective. Soit X P(E). On a toujours f f1 (X) X (limage dun antcdent dlment de X est
dans X).
Rciproquement, soit y un lment de X. Puisque f est surjective, y a un antcdent
x par f qui est par dfinition

 un
lment de f1 (X). Mais alors, y qui est limage de x appartient f f1 (X) . On a montr que X f f1 (X) est
finalement que f f1 (X) = X


Supposons que pour tout X P(E), f f1 (X) = X. Soient y un lment de E puis X = {y}. Par hypothse, f f1 ({y}) =
{y}. y est donc limage dun lment de f1 ({y}) et en particulier y a un antcdent par f. On a montr que tout lment
y de E a un antcdent par f dans E et donc f est surjective.
Exercice no 13
1) Il y a linjection triviale f :

E P(E) .
x 7
{x}

2) Soit f une application quelconque de E dans P(E). Montrons que f ne peut tre surjective.
http ://www.maths-france.fr

c Jean-Louis Rouget, 2014. Tous droits rservs.


Soit A = {x E/ x
/ f(x)}. Montrons que A na pas dantcdent par f. Supposons par labsurde que A a un antcdent
a. Dans ce cas, o est a ?
aAa
/ f(a) = A,

ce qui est absurde et

a
/ A a f(a) = A,

ce qui est absurde. Finalement, A na pas dantcdent et f nest pas surjective. On a montr le thorme de Cantor : pour
tout ensemble E (vide, fini ou infini), il nexiste pas de bijection de E sur P(E).
Exercice no 14 f est bien une application de N2 dans N car, pour tout couple (x, y) dentiers naturels, lun des deux
(x + y)(x + y + 1)
est bien un entier naturel (on peut aussi constater que
entiers x + y ou x + y + 1 est pair et donc,
2
(x + y)(x + y + 1)
= 1 + 2 + ... + (x + y) est entier pour x + y > 1).
2
Remarque. La numrotation de N2 a t effectue de la faon suivante :
0
1
2
3
..
.
y
..
.

0
0
2
5
9

1
1
4
8

2
3
7

3
6

...

...

Sur une parallle la droite dquation y = x, la somme x + y est constante. Il en est de mme de lexpression
(x + y)(x + y + 1)
et quand on descend de 1 en y, on avance de 1 dans la numrotation.
2
(p + 1)(p + 2)
p(p + 1)
6n<
.
Lemme. n N, !p N/
2
2
Dmonstration.
Pour dmontrer ce lemme, on pourrait se contenter de constater que la suite des nombres triangulaires


p(p + 1)
est strictement croissante. Nanmoins, on va faire mieux et fournir explicitement p en fonction de n.
2
p>0
Soient n et p deux entiers naturels.
p(p + 1)
(p + 1)(p + 2)
6n<
p2 + p 2n 6 0 et p2 + 3p + 2 2n > 0
2
2

1 + 8n + 1
3 + 8n + 1
1 + 8n + 1
p6
et p >
= 1 +
2
2
2



1 + 8n + 1
1 + 8n + 1
.
<p+1 p=E
p6
2
2



1 + 8n + 1
Le lemme est dmontr car E
est un entier naturel.
2
Montrons que f est surjective (et au passage, dterminons lantcdent dun entier n donn).




x+y=p
1 + 8n + 1
(p est un entier naturel). On pose
Soient n un entier naturel et p = E
p(p + 1) ou encore
2
y=n
2

y = n p(p + 1)
(x + y)(x + y + 1)
p(p + 1) p(p + 1)
2
. Tout dabord, y +
= n
+
= n. Mais il reste encore
p(p
+
3)

2
2
2
x=py=
n
2
p(p + 1)
vrifier que x et y ainsi dfinis (qui sont lvidence des entiers relatifs) sont bien des entiers naturels. Puisque
2
p(p + 1)
p(p + 3)
p(p + 1)
est un entier naturel et que n >
, y est bien un entier naturel. Ensuite,
=
+ p est aussi un
2
2
2
entier naturel et de plus,
http ://www.maths-france.fr

c Jean-Louis Rouget, 2014. Tous droits rservs.



(p + 1)(p + 2)
1 = 0,
2



1 + 8n + 1
p(p + 3)
et x est bien un entier naturel. Ainsi, pour n naturel donn, en posant p = E
puis x =
n et
2
2
p(p + 1)
y =n
, x et y sont des entiers naturels tels que f((x, y)) = n. f est donc surjective.
2
Montrons que f est injective.
(x + y)(x + y + 1)
= n, alors ncessairement,
Pour cela, on montre que si x et y sont des entiers naturels vrifiant y +
2
p(p + 1)
p(p + 3)
x + y = p (et donc y = n
puis x =
n).
2
2
Soient donc x et y deux entiers naturels. On a :
p(p + 3)
p(p + 3)
n>

2
2

(x + y)(x + y + 1)
(x + y)(x + y + 1)
(x + y)(x + y + 1)
(x + y + 1)(x + y + 2)
6
+y=n<
+ (x + y + 1) =
,
2
2
2
2
et le lemme montre que x+y = p. Lunicit du couple (x, y) est donc dmontre. f est une application injective
et surjective



8n
+
1
1
+
1
2
.
et donc f est bijective. Sa rciproque est f
: N 
N
 o p = E
2
p(p + 1)
p(p + 3)
,n
n 7
2
2

http ://www.maths-france.fr

c Jean-Louis Rouget, 2014. Tous droits rservs.


Planche no 4. Les symboles et


* trs facile ** facile *** difficult moyenne **** difficile
I : Incontournable T : pour travailler et mmoriser le cours
Exercice no 1. (IT)
(Cet exercice est consacr aux sommes de termes conscutifs dune suite arithmtique ou dune suite gomtrique.)
n
X

1) (*) Calculer

i, n N \ {0, 1, 2},

i=3

n
X

(2i 1), n N , et

i=1

n+1
X

(3k + 7), n N \ {0, 1, 2}.

k=4

lim 0, 99...9
2) (*) Calculer le nombre 1, 1111... = lim 1, |11...1
| {z }.
{z } et le nombre 0, 9999... = n+
n+
n

n1

3) (*) Calculer 1 1 + 1 ... + (1)


|
{z
n

,nN .
}
n

X 1
1 1 1
.
+ + + ... = lim
n+
2 4 8
2k

4) (*) Calculer

k=1

5) (**) Calculer

n
X

cos

k=0

k
, n N.
2

6) (***) Soient n N et R. Calculer

n
X

cos(k) et

k=0

n
X

sin(k). (Indication : calculer

k=0

7) (***) Pour x [0, 1] et n N , on pose Sn =

n
X

(1)k1

k=1

n
X
k=0

cos(k) + i

n
X

sin(k).)

k=0

xk
. Dterminer lim Sn .
n+
k

8) (**) On pose u0 = 1 et, pour n N, un+1 = 2un 3.


a) Calculer la suite (un 3)nN .
n
X
b) Calculer
uk .
k=0

Exercice no 2. (IT)
(Cet exercice est consacr aux sommes tlescopiques.)
Calculer les sommes suivantes :
n
n
X
X
1
1
1) (**)
et
k(k + 1)
k(k + 1)(k + 2)
k=1

2) (**)

n
X

k=1

k k! et

k=0

n
X
k=1

3) (***) Calculer Sp =

k
(k + 1)!

n
X

kp pour n N et p {1, 2, 3, 4} (dans chaque cas, chercher un polynme Pp de degr p + 1

k=1

tel que Pp (x + 1) Pp (x) = xp ).


4) (***) Soient n N et R. Calculer Cn =

n
X

cos(k). (Indication : calculer 2 sin

k=0

 

Cn (on donne 2 sin a cos b =


2

sin(a + b) + sin(a b)).)


Exercice no 3. (IT)
Calculer les sommes suivantes :
X
1) (**)
1.
16i<j6n

2) (**)

X
16i,j6n

j et

j.

16i<j6n

http ://www.maths-france.fr

c Jean-Louis Rouget, 2014. Tous droits rservs.


3) (*)

ij.

16i,j6n

4) (***) Pour n N , on pose un =

n
n
1 XX
(5h4 18h2 k2 + 5k4 ). Dterminer lim un .
n+
n5
k=1 h=1

Exercice n 4. (IT)

n 
Y
1
1+
, n N .
1) (*) Calculer
k
o

k=1

2) (***) Calculer

n
Y
k=1

cos

a
, a ]0, 2[, n N (indication : on sait que pour tout rel x, sin(2x) = 2 sin(x) cos(x).)
2k

http ://www.maths-france.fr

c Jean-Louis Rouget, 2014. Tous droits rservs.


Planche no 4. Les symboles et : corrig


Exercice no 1.
1) Soit n > 3.
n
X

i=

i=3

(n 2)(n + 3)
(3 + n)(n 2)
=
.
2
2

Soit n N .
n
X
i=1

(2i 1) =

(1 + (2n 1))n
= n2
2

et
n+1
X

(3k + 7) =

k=4

(19 + 3n + 10)(n 2)
1
1
= (3n + 29)(n 2) = (3n2 + 23n 58).
2
2
2

2) Soit n N . Posons un = 1, 11...1


| {z }. On a
n

k X
k 1 1
n 
n 
X
1
1
1
10n+1 = 10
.
=
=
un = 1 +
1
10
10
9
9 10n
k=0
k=1
1
10
10
1
tend vers 0, et donc un tend vers
.
Quand n tend vers +,
9 10n
9
1, 11111.... =

10
.
9

Soit n N . Posons un = 0, 99...9


| {z }. On a
n

1
n
X
1
9
9 1 10n
un =
= 1 n.
=
1
10k
10
10
k=1
1
10

Quand n tend vers +,

1
tend vers 0, et donc un tend vers 1.
10n
0, 9999.... = 1.

3) Soit n N . Posons un = 1 1 + 1 ... + (1)n1 . On a


{z
}
|
n

un =

n1
X
k=0

(1)k =

1
1 (1)n
= (1 (1)n ) =
1 (1)
2

0 si n est pair
.
1 si n est impair

1
n
1 n
X
1
1
2 = 1 1 . Quand n tend vers +, on obtient
4) Soit n N .
=
k
1
2
2
2n
k=1
1
2
1 1 1
+ + + ... = 1.
2 4 8

http ://www.maths-france.fr

c Jean-Louis Rouget, 2014. Tous droits rservs.


5) Soit n N.
n
X

k=0

n
X

k
= Re
cos
2
= Re

ik/2

k=0

n
X

(= Re

k=0

1 e(n+1)i/2
1 ei/2

)
(n+1)

ei(n+1)/4 2i sin 4
2i sin 4
ei/4

= Re

1
(2n + 1) 1
= sin
+ =
4
2
2

n
(n + 1)
cos
2 sin
4
4

1 si n 4N (4N + 1)
0 si n (4N + 2) (4N + 3)

= 1 + 0 1 + 0 = 0, on a
En fait, on peut constater beaucoup plus simplement que cos 0 + cos + cos + cos
2
2
immdiatement S4n = S0 = 1, S4n+1 = S1 = 1, S4n+2 = S2 = 0 et S4n+3 = S3 = 0.
6) Soient n N et R. Posons Cn =

n
X

cos(k) et Sn =

k=0
n
X

Cn + iSn =

n
X

sin(k). Alors, daprs la formule de Moivre,

k=0

(cos(k) + i sin(k)) =

k=0

n
X

eik =

k=0

n
X

ei

k=0

- 1er cas. Si
/ 2Z, alors ei 6= 1. Par suite,
(n+1)

Cn + iSn =

2i sin 2
1 ei(n+1)
= ei(n+11)/2
1 ei
2i sin 2

= ein/2

k

sin (n+1)
2
sin 2

Par suite,
n
(n + 1)
(n + 1)
n
sin
sin
sin
2
2
2
2
et Sn = Im(Cn + iSn ) =
.
Cn = Re(Cn + iSn ) =

sin
sin
2
2
- 2me cas. Si 2Z, on a immdiatement Cn = n + 1 et Sn = 0.
cos

Finalement,

n
(n + 1)

cos
sin

2
2
X
si
/ 2Z

n N,
cos(k) =
sin

k=0
2

n + 1 si 2Z

n
(n + 1)

sin
sin

2
2
X
si
/ 2Z

et
sin(k) =
.
sin

k=0
2

0 si 2Z

7) Soient x [0, 1] et n N . Puisque x 6= 1, on a


Sn (x) =

n
X

(1)k1 xk1 =

k=1

n1
X

(x)k =

k=0

1
1 (x)n
=
(1 (x)n ).
1 (x)
1+x

Par suite,
Sn (x) = Sn (0) +

Zx
0

Sn (t) dt =

Zx
0

1 (t)n
dt =
1+t

Zx
0

1
dt
1+t

Zx
0

(t)n
dt = ln(1 + x)
1+t

Zx
0

(t)n
dt.
1+t

Mais alors,

Comme


Z x
Zx
Zx n
Zx
(t)n
(t)n
t
xn+1
1
n




dt
=
|Sn (x) ln(1 + x)| =
dt
dt
6
t
dt
=
6
.
1+t
1
+
t
1
+
t
n
+
1
n
+
1
0
0
0
0

1
tend vers 0 quand n tend vers +, on en dduit que
n+1
x [0, 1],

http ://www.maths-france.fr

lim

n+

n
X

(1)k1 xk1 = ln(1 + x).

k=1

c Jean-Louis Rouget, 2014. Tous droits rservs.


En particulier,


(1)n1
1 1
ln 2 = lim
1 + ... +
n+
2 3
n

8) a) Soit n N. un+1 3 = 2un 6 = 2(un 3). La suite (un 3)nN est donc une suite gomtrique, de raison q = 2
et de premier terme u0 3 = 2. On en dduit que, pour tout entier naturel n, un 3 = 2 2n et donc que
n N, un = 3 2n+1 .
b) Soit n N.
n
X

k=0

uk =

n
X

32

k=0

n
X

2k = 3(n + 1) 2

k=0

n N,

n
X

2n+1 1
= 2n+2 + 3n + 5.
21

uk = 2n+2 + 3n + 5.

k=0

Exercice no 2.
1
(k + 1) k
1
1
=
=
, et donc
k(k + 1)
k(k + 1)
k k+1

n 
n
X
X
1
1
n
1
1
=1
=

=
.
k(k + 1)
k k+1
n+1
n+1
k=1
k=1


1
1
1 (k + 2) k
1
1
Pour tout naturel non nul k, on a
et donc
=
=

k(k + 1)(k + 2)
2 k(k + 1)(k + 2)
2 k(k + 1) (k + 1)(k + 2)
1) Pour tout naturel non nul k, on a

n
X

k=1




n 
1 1
n(n + 3)
1
1X
1
1
1
=
=
=

.
k(k + 1)(k + 2)
2
k(k + 1) (k + 1)(k + 2)
2 2 (n + 1)(n + 2)
4(n + 1)(n + 2)
k=1

2) Soit n N. Pour tout entier naturel k, k k! = (k + 1 1) k! = (k + 1) k! k! = (k + 1)! k! puis


n
X

k=0

n
X

k k! =

((k + 1)! k!) = (n + 1)! 1.

k=0

Soit n N. Pour tout entier naturel k,


(k + 1 1)
k+1
1
1
1
k
=
=

(k + 1)!
(k + 1)!
(k + 1)! (k + 1)!
k! (k + 1)!
puis
n
X

k=0


n 
X
1
1
1
k
=1
=

.
(k + 1)!
k! (k + 1)!
(n + 1)!
k=0

3) Soit n N .

- Calcul de S1 . Posons P1 = aX2 + bX + c. On a


P1 (X + 1) P1 (X) = a((X + 1)2 X2 ) + b((X + 1) X) = 2aX + (a + b).

Par suite,
1
1
et b =
2
2
X
X(X 1)
X2
P1 =
.
P1 =
2
2
2

P1 (X + 1) P1 (X) = X 2a = 1 et a + b = 0 a =

http ://www.maths-france.fr

c Jean-Louis Rouget, 2014. Tous droits rservs.


Mais alors,
n
X

n
X

k=

k=1

(P1 (k + 1) P1 (k)) = P1 (n + 1) P1 (1) =

k=1

n(n + 1)
.
2

- Calcul de S2 . Posons P2 = aX3 + bX2 + cX + d. On a


P2 (X + 1) P2 (X) = a((X + 1)3 X3 ) + b((X + 1)2 X2 ) + c((X + 1) X) = 3aX2 + (3a + 2b)X + a + b + c.
Par suite,
P2 (X + 1) P2 (X) = X2 3a = 1 et 3a + 2b = 0 et a + b + c = 0 a =
P2 =

Mais alors,
n
X

k2 =

k=1

n
X

1
1
1
et b = et c =
3
2
6

X3 X2 X
X(X 1)(2X 1)

+ P2 =
.
3
2
6
6

(P2 (k + 1) P2 (k)) = P2 (n + 1) P2 (1) =

k=1

n(n + 1)(2n + 1)
.
6

- Calcul de S3 . Posons P3 = aX4 + bX3 + cX2 + dX + e. On a


P3 (X + 1) P3 (X) = a((X + 1)4 X4 ) + b((X + 1)3 X3 ) + c((X + 1)2 X2 ) + d((X + 1) X)
= 4aX3 + (6a + 3b)X2 + (4a + 3b + 2c)X + a + b + c + d.
Par suite,

Mais alors,

P3 (X + 1) P3 (X) = X3 4a = 1, 6a + 3b = 0, 4a + 3b + 2c = 0 et a + b + c + d = 0
1
1
1
a = , b = , c = et d = 0
4
2
4
3
2
4
X
X
X2 (X 1)2
X

+
P3 =
.
P3 =
4
2
4
4
n
X

k3 =

k=1

n
X

(P3 (k + 1) P3 (k)) = P3 (n + 1) P3 (1) =

k=1

n2 (n + 1)2
.
4

- Calcul de S4 . Posons P4 = aX5 + bX4 + cX3 + dX2 + eX + f. On a


P4 (X + 1) P4 (X) = a((X + 1)5 X5 ) + b((X + 1)4 X4 ) + c((X + 1)3 X3 ) + d((X + 1)2 X2 )
+ e((X + 1) X)
= 5aX4 + (10a + 4b)X3 + (10a + 6b + 3c)X2 + (5a + 4b + 3c + 2d)X + a + b + c + d + e.
Par suite,
P4 (X + 1) P4 (X) = X4 5a = 1, 10a + 4b = 0, 10a + 6b + 3c = 0, 5a + 4b + 3c + 2d = 0

et a + b + c + d + e = 0
1
1
1
1
a = , b = , c = , d = 0 et e =
5
2
3
30
X4 X3
X
X(X 1)(6X3 9X2 + X + 1)
X5

P4 =
.
P4 =
5
2
3
30
30

Mais alors,
n
X

k=1

k4 =

n
X

(P4 (k + 1) P4 (k)) = P4 (n + 1) P4 (1) =

k=1

http ://www.maths-france.fr

n(n + 1)(6n3 + 9n2 + n 1)


.
30
c Jean-Louis Rouget, 2014. Tous droits rservs.

n N ,

n
n
n(n + 1) X 2 n(n + 1)(2n + 1) X 3 n2 (n + 1)2
,
k =
,
k =
=
k=
2
6
4
k=1
k=1
k=1
n
X
n(n + 1)(6n3 + 9n2 + n 1)
et
k4 =
.
30
n
X

n
X

!2

k=1

k=1

4) Soient n N et R.

 

 
 
n 
n
X
X

1
1

2 sin Cn =
cos(k) =
sin
k+
+ sin
k +

2 sin
2
2
2
2
k=0
k=0

 

 
n 
X
1
1
=
sin
(k + 1)
sin
k+

2
2
k=0

 
 
1

sin
= sin
(n + 1)
2
2


(2n + 1)
 
sin

2
 
.
- 1er cas. Si
/ 2Z, alors
6= 0. On peut alors crire Cn = +
/ Z puis sin

2
2
2
2 sin
2
- 2me cas. Si 2Z, on a immdiatement Cn = n + 1.
Exercice no 3.
1) Soit n un entier suprieur ou gal 2.
!
j1
n
n
n1
X
X
X
X
X
n(n 1)
1 re solution.
1=
1 =
(j 1) =
j=
.
2
16i<j6n

2 me solution.

j=2

i=1

j=2

k=1

1 est le nombre de couples (i, j) dentiers compris au sens large entre 1 et n tels que i < j. Il y

16i<j6n

a n2 couples (i, j) dentiers compris au sens large entre 1 et n.


Parmi ces n2 couples, il y en a n tels que i = j et donc n2 n = n(n 1) tels que 1 6 i, j 6 n et i 6= j. Comme il y a
n(n 1)
couples (i, j) tels que 1 6 i < j 6 n.
autant de couples (i, j) tels que i > j que de couples (i, j) tels que i < j, il y a
2
Finalement,
X

1=

16i<j6n

n(n 1)
.
2

2) Soit n N .
X

j=

16i,j6n

i=1

n
X

n
n
X
X
j=1

n
X

nj = n

j=1

n
X
j=1

j=n

n2 (n + 1)
n(n + 1)
=
.
2
2

Soit n un entier suprieur ou gal 2.


X

j=

16i<j6n

j1
n
X
X
j=2

=
=

i=1

(j 1)j =

j=2

n
X
j=2

j2

n
X

j=2


 


n(n + 1) 2n + 1
n(n + 1)
n(n + 1)(2n + 1)
1
1 =
1
6
2
2
3

n(n + 1)2
.
3

3) Soit n N . Daprs lexercice no 2, question3), on a


http ://www.maths-france.fr

c Jean-Louis Rouget, 2014. Tous droits rservs.


16i,j6n

ij =

16i6n

n
X

16j6n

n2 (n + 1)2
.
4

n(n + 1)(2n + 1)
6

j =

4) Soit n N . Daprs
X

2 2

h k =

16h,k6n

Comme dautre part,

n
X

h4 =

h=1

16h,k6n

k4 =

h=1

k=1

2

n(n + 1)(6n3 + 9n2 + n 1)


daprs lexercice no 2, question3), on a
30
!
n
n
n
X
X
X
n2 (n + 1)(6n3 + 9n2 + n 1)
4
=
nh4 = n
h4 =
,
h
30

k4 =

k=1
n
X

h4 =

16h,k6n

et bien sr

n
X

n
X

h=1

h=1

k=1

h=1

n2 (n + 1)(6n3 + 9n2 + n 1)
. Par suite,
30

1
un = 5
n
1
n5
1
= 5
n



n2 (n + 1)(6n3 + 9n2 + n + 14)
n2 (n + 1)2 (2n + 1)2
25
18
30
36

 2
3
2
2
2
n (n + 1)(6n + 9n + n + 14) n (n + 1) (2n + 1)2

3
2




15 12
6
6
5
+ termes de degr au plus 4
2n 2n + n

3
2

= 1 + termes tendant vers 0


Par suite,

lim un = 1.

n+

Exercice no 4.
1) Soit n N .
 Y
n
n 
Y
k+1
n+1
1
=
=
= n + 1 (produit tlescopique).
1+
k
k
1

k=1

k=1

a
a
a
6 < et donc sin k 6= 0.
2k
2
2
On sait alors que pour tout rel x, sin(2x) = 2 sin x cos x. Par suite, pour tout naturel k,


sin a/2k1
a
a
a
a
.
sin 2 k = 2 sin k cos k et donc cos k =
2
2
2
2
2 sin (a/2k )

2) Soit a ]0, 2[ et n N . Alors, pour tout naturel non nul k, on a 0 <

Mais alors,

n
Y

sin(a/2k1 )
n
k1
Y
a
sin a
1
sin(a/2
)
cos k =
= n
= n k=1
(produit tlescopique).
n
2
2 sin(a/2k )
2 Y
2
sin
(a/2n )
k
k=1
k=1
sin(a/2 )
n
Y

k=1

http ://www.maths-france.fr

c Jean-Louis Rouget, 2014. Tous droits rservs.


Planche no 5. Le binme de Newton


* trs facile ** facile *** difficult moyenne **** difficile
I : Incontournable T : pour travailler et mmoriser le cours
Exercice no 1. (IT)
Identits combinatoires (la difficult va en augmentant graduellement de facile assez difficile sans tre insurmontable).
   
 
n
n
n
1) Calculer
+
+ ... +
.
0
1
n
     
     
n
n
n
n
n
n
2) Montrer que
+
+
+ ... =
+
+
+ ... et trouver la valeur commune des deux sommes.
0
2
4
1
3
5
3) a) On pose j = e2i/3 . Montrer
+ j
+ j2 = 0 et que j3 = 1.
  que
 1
n
n
n
b) Calculer la somme
+
+
+ ...
0
3
     6
n
n
n
c) Calculer la somme
+
+
+ ....
0
4
8
 


n
n1
4) Montrer que n N , k J1, nK, k
=n
.
k
k1
 2  2
 2  
n
n
n
2n
5) Montrer que
+
+ ... +
=
(utiliser le polynme (1 + x)2n ).
0
1
n
n
 
   
n
n
n
 
 
 
n
n
n
1
n
0
+
+ ... +
(considrer dans chaque cas
6) Calculer les sommes 0
+1
+ ... + n
et
1
2
n+1
0
1
n
un certain polynme astucieusement choisi).
  

  

p
p+1
n
n+1
7) Montrer que
+
... +
=
o 0 6 p 6 n. Interprtation dans le triangle de Pascal ?
p
p
p
p+1
Exercice no 2. (**)
Quel est le coefficient de a4 b2 c3 dans le dveloppement de (a b + 2c)9 .
Exercice no 3. (**I)
Dvelopper (a + b + c + d)2 et (a + b + c)3 .
Exercice no 4. (***)
Soit (n, a, b) N ]0, +[]0, +[. Quel est le plus grand terme du dveloppement de (a + b)n ?
Exercice no 5. (**)
Rsoudre dans N* lquation

     
n
n
n
+
+
= 5n.
1
2
3

Exercice no 5. (**)
Soient n un entier naturel et un rel. Calculer

n  
X
n

k=0

http ://www.maths-france.fr

cos(k).

c Jean-Louis Rouget, 2014. Tous droits rservs.


Planche no 5. Le binme de Newton : corrig


Exercice no 1.
1) Soit n N. Daprs la formule du binme de Newton,

n  
X
n

k=0

n N,

n  
X
n

k=0

n  
X
n

k=0

1k 1nk = (1 + 1)n = 2n .

= 2n .

E(n/2) 



E((n1)/2) 
X
n
n
2) Soit n un entier naturel non nul. Posons S1 =
et S2 =
. Alors
2k
2k + 1
k=0
k=0
 
n
X
k n
= (1 1)n = 0 (car n > 1),
(1)
S1 S2 =
k
X

k=0

et donc S1 = S2 . Puis S1 + S2 =

n  
X
n

k=0

= 2n , et donc S1 = S2 = 2n1 .

     
     
n
n
n
n
n
n
n N ,
+
+
+ ... =
+
+
+ . . . = 2n1 .
0
2
4
1
3
5

3) a) Posons j = e2i/3 .
3

2i
j3 = e2i/3 = e 3 3 = e2i = cos(2) + i sin(2).

Ensuite, puisque j 6= 1,

1 + j + j2 =

11
1 j3
=
= 0.
1j
1j

b) Soit n un entier naturel.


n  
X
n

k=0

= (1 + 1)n = 2n ,

n  
n  
X
X
n k
n 2k
j = (1 + j)n et
j = (1 + j2 )n .
k
k

k=0

k=0

En additionnant ces trois galits, on obtient


n  
X
n
(1 + jk + j2k ) = 2n + (1 + j)n + (1 + j2 )n .
k

k=0

Maintenant,
- si k 3N, il existe p N tel que k = 3p et 1 + jk + j2k = 1 + (j3 )p + (j3 )2p = 3 car j3 = 1.
- si k 3N + 1, il existe p N tel que k = 3p + 1 et 1 + jk + j2k = 1 + j(j3 )p + j2 (j3 )2p = 1 + j + j2 = 0
- si k 3N + 2, il existe p N tel que k = 3p + 2 et 1 + jk + j2k = 1 + j2 (j3 )p + j4 (j3 )2p = 1 + j2 + j = 0.

E(n/3) 
n  
X
X
n
n
k
2k
Finalement,
(1 + j + j ) = 3
. Par suite,
k
3k
k=0

k=0

E(n/3) 

k=0

http ://www.maths-france.fr


 1
1 n
n
2 + (1 + j)n + (1 + j2 )n = (2n + 2 Re((1 + j)n ))
=
3
3
3k

n 
1 n
1
2 + 2 cos
= (2n + 2 Re((j2 )n )) =
3
3
3
     
n 
n
n
1 n
n
n N,
2 + 2 cos
.
+
+
+ ... =
3
3
0
3
6

c Jean-Louis Rouget, 2014. Tous droits rservs.


4) Pour 1 6 k 6 n, on a


 
(n 1)!
n1
n!
n
=n
=k
.
k
=k
k!(n k)!
(k 1)!((n 1) (k 1))!
k1
k
 
2n
5)
est le coefficient de xn dans le dveloppement de (1 + x)2n . Mais dautre part ,
n
! n   !
n  
X
X n
n k
2n
n
n
(1 + x) = (1 + x) (1 + x) =
x
xk .
k
k
k=0

k=0

Dans le dveloppement de cette dernire expression, le coefficient de xn vaut

n
X

k=0

polynmes sont gaux si et seulement si ils ont mmes coefficients et donc

 

n  2
X
n
n
n
ou encore
. Deux
k
nk
k
k=0

  X
n  2
2n
n
=
.
n
k
k=0

6) a) 1re solution. Pour x rel, posons P(x) =

n
X

k=1

Pour tout x rel,

 
n k1
k
x
.
k

!
n  
X
n k

x
= ((1 + x)n ) = n(1 + x)n1 .
k

P(x) =

k=0

En particulier, pour x = 1, on obtient :


 
n
X
n
k
= n(1 + 1)n1 = n2n1 .
k

k=1

 
n
X
n
k
= n2n1 .
n N ,
k

k=1

2me solution. Daprs 4),


  X


n
n
n1
X
X n 1
n
n1
k
=
n
=n
= n(1 + 1)n1 = n2n1 .
k
k1
k

k=1

k=1

k=0

b) 1re solution. Pour x rel, posons P(x) =

n   k+1
X
n x

. On a
k k+1
n  
X
n k
P (x) =
x = (1 + x)n ,
k
k=0

k=0

et donc, pour x rel,


P(x) = P(0) +

Zx
0

P (t) dt =

Zx

(1 + t)n dt =

En particulier, pour x = 1, on obtient


1
(1 + x)n+1 1 .
n+1

 
n
n
X
2n+1 1
k
=
.
n N,
k+1
n+1
k=0

 


n
n+1
2me solution. Daprs 4), (n + 1)
= (k + 1)
et donc
k
k+1


 
n+1
n

n
n+1 
n
X k+1
X k
1 X n+1
2n+1 1
1
=
=
((1 + 1)n+1 1) =
.
=
k+1
n+1
n+1
n+1
n+1
k
k=0

http ://www.maths-france.fr

k=0

k=1

c Jean-Louis Rouget, 2014. Tous droits rservs.



 
 



p+k
p+k+1
p+k
p
7) Pour 1 6 k 6 np,
=

(ce qui reste vrai pour k = 0 en tenant compte de


= 0).
p
p+1
p+1
p+1
Par suite,
np
X
k=0


np
X p + k + 1 p + k
p+k
=1+

p
p+1
p+1
k=1


n+1
=1+
1 (somme tlescopique)
p+1


n+1
=
.
p+1

Interprtation
 dans le triangle de Pascal. Quand on descend dans le triangle de Pascal, le long de la colonne p, du


p
p
coefficient
(ligne p) au coefficient n
(ligne n), et que lon additionne ces coefficients, on trouve n+1
qui se trouve
p+1
p
une ligne plus bas et une colonne plus loin.
Exercice no 2. La formule du binme de Newton fournit
 
9  
X
9
9
k
9k
9
(a b + 2c) =
(a b) (2c)
= (a b) + ... +
(a b)6 (2c)3 + ... + (2c)9 .
k
6
9

k=0

Ensuite,
(a b)6 =

6  
X
6

k=0

ak (b)6k = a6 ... +

 
6 4
a (b)2 .. + (b)6 .
4

Le coefficient cherch est donc


  
9 6 3 987 65

23 = 3 4 7 3 5 8 = 10080.
2 =
32
2
6 4
Exercice no 3.
(a + b + c + d)2 = a2 + b2 + c2 + d2 + 2(ab + ac + ad + bc + bd + cd)
et
(a + b + c)3 = a3 + b3 + c3 + 3(a2 b + ab2 + a2 c + ac2 + b2 c + bc2 ) + 6abc.
Exercice no 4.
 
n k nk
Soit n un entier naturel non nul. Le terme gnral du dveloppement de (a + b) est uk =
a b
, 0 6 k 6 n. Pour
k
0 6 k 6 n 1, on a :


n
ak+1 bnk1
uk+1
nk a
k+1
 
=
.
=
n k nk
uk
k+1
b
a b
k
n

Par suite,
uk+1
nk a
na b
>1
> 1 (n k)a > (k + 1)b k <
.
uk
k+1
b
a+b

a
na b
> n 1 (ce qui quivaut n < ), alors la suite (uk )06k6n est strictement croissante et le plus grand
a+b
b
terme est le dernier : an .
na b
b
2me cas. Si
6 0 (ce qui quivaut n 6 ), alors la suite (uk )06k6n est strictement dcroissante et le plus
a+b
a
grand terme est le premier : bn .
1er cas. Si

na b
6 n 1. Dans ce cas, la suite est strictement croissante puis ventuellement momentanment
a+b
na b
constante, suivant que
soit un entier ou non, puis strictement dcroissante (on dit que la suite u est unimodale).
a+b

3me cas. Si 0 <

http ://www.maths-france.fr

c Jean-Louis Rouget, 2014. Tous droits rservs.




na b
na b
Si
+ 1, la suite u croit strictement jusqu ce rang puis dcroit strictement. Le

/ N, on pose k0 = E
a+b
a+b
plus grand des termes est celui dindice k, atteint une et une seule fois.
na b
N, le plus grand des termes est atteint deux fois lindice k0 1 et lindice k0 .
a+b
Exercice no 5.

Si

Pour n > 3,
     
n
n(n 1) n(n 1)(n 2)
n
n
+
+
= 5n n +
+
= 5n
1
2
3
2
6

n(24 + 3(n 1) + (n 1)(n 2)) = 0 n2 25 = 0


n = 5.

Dautre part, 1 + 0 + 0 6= 5 et 2 + 1 + 0 6= 10 et donc 1 et 2 ne sont pas solution de lquation. Lquation propose admet
une solution et une seule dans N savoir 5.
Exercice no 6.
Soient n un entier naturel et un rel.
!
!
n  
n  
X
X

n ik
n
i k nk
cos(k) = Re
= Re
e
e
1
k
k
k
k=0
k=0
n 



= Re (ei + 1)n = Re ei/2 ei/2 + ei/2


 n

ein/2
= Re
2 cos
2
 
 
n

cos
.
= 2n cosn
2
2

n  
X
n

k=0

http ://www.maths-france.fr

c Jean-Louis Rouget, 2014. Tous droits rservs.


Planche no 6. Systmes dquations linaires


* trs facile ** facile *** difficult moyenne **** difficile
I : Incontournable T : pour travailler et mmoriser le cours
Exercice no 1 : (**T)
Rsoudre dans R3

1)

4)

les systmes suivants :


2x + 3y z = 1
4x + y + 2z = 6
x 3y + z = 2
2x + y = 1
x+y=0
(dans R3 )
3x + 4y = 1

2x + y + z = 7
x + 2y + z = 8
2)

x + y + 2z = 9
x+y+z+t = 3
x+y+zt = 3
5)
(dans R4 )

x y z t = 1

x+y+z=2
3x y = 1
3)

4x + z = 4

Exercice no 2 : (**T)
Lespace est rapport un repre orthonorm
respectives x + y + 2z = 1 et 2x y + z = 2.




O, i , j . Dterminer lintersection des plans P et P dquations

Exercice no 3 : (**)
Rsoudre dans R3 ou R4 suivant le cas, en discutant en fonction du paramtre m, les systmes suivants :

2x + 3y + z = 4
2x + my + z = 3m
x + my + 2z = 5
x (2m + 1)y + 2z = 4
1)
2)

7x + 3y + (m 5)z = 7
5x y + 4z = 3m 2
Exercice no 4 : (***I)
Dans le plan, on donne n points A1 , ... , An . Existe-t-il n points M1 ,..., Mn tels que A1 soit le milieu de [M1 , M2 ], A2
soit le milieu de [M2 , M3 ],..., An1 soit le milieu de [Mn1 , Mn ] et An soit le milieu de [Mn , M1 ].
no 5 : (**T)
1) Dterminer tous les polynmes P de degr 2 tels que P(1) = 1, P (1) = 1 et P(1) = 0.
2) Dterminer tous les polynmes P de degr 3 tels que P(1) = 1, P(1) = 0 et P(2) = 1.

http ://www.maths-france.fr

c Jean-Louis Rouget, 2014. Tous droits rservs.


Planche no 6. Systmes dquations linaires : corrig


Exercice no 1.
1) Soit (x, y, z) R3 .

2x + 3y z = 1
z = 2x + 3y 1
z = 2x + 3y 1
x=1
4x + y + 2z = 6
4x + y + 2(2x + 3y 1) = 6
8x + 7y = 8
8 + 7y = 8

x 3y + z = 2
x 3y + (2x + 3y 1) = 2
3x = 3
z = 2x + 3y 1

x=1

y=0

z=1

Lensemble des solutions du systme propos est {(1, 0, 1)}.


2) Soit (x, y, z) R3 .

2x + y + z = 7

x + 2y + z = 8

x + y + 2z = 9

2x + y + z = 7
2x + y + z = 7

x + 2y + z = 8
x + 2y + z = 8

x + y + 2z = 9
x
+ y + 2z = 9

(2x + y + z) + (x + 2y + z) + (x + y + 2z) = 7 + 8 + 9
x+y+z =6

(2x + y + z) (x + y + z) = 7 6
x=1

x=1
(x + 2y + z) (x + y + z) = 8 6
y=2
y=2 .

(x + y + 2z) (x + y + z) = 9 6
z=3

z=3

x+y+z=6
x+y+z= 6

Lensemble des solutions du systme propos est {(1, 2, 3)}.


3) Soit (x, y, z) R3 .

x+y+z =2
y = 3x 1
0x=5
3x y = 1
z = 4x + 4
y = 3x 1 .

4x + z = 4
x + (3x 1) + (4x + 4) = 2
z = 4x + 4

Le systme propos na pas de solution.


4) Soit (x, y, z) R3 .

2x + y = 1
y = x
y = x
x+y=0

2x x = 1

x=1

3x + 4y = 1
3x 4x = 1
x=1

x=1

y = 1

Lensemble des solutions du systme propos est {(1, 1, z), z R}.


5)

x+y+z+t=3
2x = 2 (L1 L1 + L3 )
x=1
x+y+zt=3
x+y+zt=3
1+y+zt=3

x y z t = 1
x y z t = 1
1 y z t = 1

x=1
x=1
y+zt = 2
2t = 0 (L2 L2 + L3 )

y z t = 2
y z t = 2

x=1

t=0

z = y + 2

Lensemble des solutions du systme propos est {(1, y, 0, y + 2), y R}.


http ://www.maths-france.fr

c Jean-Louis Rouget, 2014. Tous droits rservs.


Exercice no 2.
Soit M(x, y, z) un point de lespace.


z = 2x + y + 2
3x + 3y = 3

x + y + 2(2x + y + 2) = 1
z = 2x + y + 2


y=x1
y =x1

z = 2x + (x 1) + 2
z = x + 1

x=
y =1 .
R/

z = + 1

x=
y = 1 , R. Il revient au mme de dire que lensemble
P P est la droite de reprsentation paramtrique

z
= + 1

x + y + 2z = 1
est {(x, x 1, x + 1), x R}.
des solutions du systme
2x y + z = 2
M P P

x + y + 2z = 1

2x y + z = 2

Exercice no 3. m est un paramtre rel.


1) Soit (x, y, z) R3 .

2x + 3y + z = 4
z = 4 2x 3y
z = 4 2x 3y
x + my + 2z = 5
x + my + 2 (4 2x 3y) = 5
5x + (m 6)y = 3

7x + 3y + (m 5)z = 7
7x + 3y + (m 5) (4 2x 3y) = 7
(2m + 17)x + (3m + 18)y = 4m + 27

Le dterminant du systme form par les deux dernires quations est




5
=
2m + 17


m 6
= 5(3m + 18) (2m + 17)(m 6) = (m 6)(15 (2m + 17))
3m + 18

= 2(m 1)(m 6).

Le systme form par les deux dernires quations est de Cramer si et seulement si m
/ {1, 6}.
Si m
/ {1, 6}, les formules de Cramer fournissent alors :


1
3
m6

x
=


4m + 27 3m + 18

2(m

1)(m

6)
5x + (m 6)y = 3


(2m + 17)x + (3m + 18)y = 4m + 27

5
3


y=
2(m 1)(m 6) 2m + 17 4m + 27

4m2 42m + 108

x=
2(m 1)(m 6)

14m 84

y=
2(m 1)(m 6)

2m 9
2(2m 9)(m 6)

=
x=
2(m

1)(m

6)
m1

14(m

6)
7

=
y=
2(m 1)(m 6)
m1

2m 9

x=
m1 .

y=
m1

Mais alors,

http ://www.maths-france.fr

c Jean-Louis Rouget, 2014. Tous droits rservs.


2x
+
3y
+
z
=
4

x + my + 2z = 5

7x + 3y + (m 5)z = 7

2m 9
m1
7
y=
m1
2m 9
7
z=42
3
m1
m1
2m 9
x=
m1
7
y=
m1
4m 7
z=
m1


2m 9
7
4m 7
Si m
/ {1, 6}, lensemble des solutions du systme propos est
.
,
,
m1 m1 m1

z = 4 2x 3y

z = 4 2x 3y
2x + 3y + z = 4
3
x+y=
5x 5y = 3 ou encore
x + y + 2z = 5 ou aussi
. Dans ce
Si m = 1, le systme scrit
5

15x + 15y = 23
7x + 3y 4z = 7

23

x+y=
.
15
cas, le systme na pas de solution.

x= 3
z = 4 2x 3y
x= 3
5
5
ou enfin
.
Si m = 6, le systme scrit
5x = 3
ou encore
3
14

3y
3y
z
=
4

z
=
5x = 3
5
5



14
3
Si m = 6, lensemble des solutions est
, y,
3y , y R .
5
5
x=

2) Soit (x, y, z) R3 .

Ensuite,

2x + my + z = 3m
x = (2m + 1)y 2z + 4
x (2m + 1)y + 2z = 4
2((2m + 1)y 2z + 4) + my + z = 3m

5x y + 4z = 3m 2
5((2m + 1)y 2z + 4) y + 4z = 3m 2

x = (2m + 1)y 2z + 4
x = (2m + 1)y 2z + 4
(5m + 2)y 3z = 3m 8
(5m + 2)y 3z = 3m 8

(5m + 2)y 3z = 3 m 11.


(10m + 4)y 6z = 3m 22
2
3m 8 =

Si m 6= 2, le systme na pas de solution.

3
3
m 11 m = 3 m = 2.
2
2

z = 14 8y
x = 3y 2z + 4
3
ou enfin
Si m = 2, le systme scrit
ou encore
14 8y
8y 3z = 14

x = 3y 2
+4
3



16 + 7y
14 8y
Dans ce cas, lensemble des solutions est
, yR .
, y,
3
3

z = 14 8y
3
16 + 7y .

x=
3

Exercice no 4.
Si zk est laffixe complexe de Mk et ak est laffixe complexe de Ak , le problme pos quivaut au systme :
k {1, ..., n 1}, zk + zk+1 = 2ak et zn + z1 = 2an ,
Rsolvons le systme constitu des n 1 premires quations

http ://www.maths-france.fr

c Jean-Louis Rouget, 2014. Tous droits rservs.


z1 + z2 = 2a1
z2 = 2a1 z1

z
z
+
z
=
2a

3 = 2a2 2a1 + z1
2
3
2

z3 + z4 = 2a3
z4 = 2a3 2a2 + 2a1 z1

..
..

.
.

z
+
z
=
2a
z
=
2an2 2an3 + . . . + (1)n3 a1 + (1)n2 z1

n2
n1
n2
n1

zn1 + zn = 2an1
zn = 2an1 2an2 + 2an3 + . . . + 2(1)n2 a1 + (1)n1 z1

La dernire quation scrit alors 2an1 2an2 + 2an3 + . . . + 2(1)n2 a1 + (1)n1 z1 + z1 = 2an ou encore
(1 (1)n ) z1 = 2an 2an1 + 2an2 2an3 + . . . + 2(1)n1 a1
Si n est impair, cette dernire quations est quivalente
z1 = an an1 + an2 an3 + . . . + (1)n1 a1
En reportant cette expression dans les n 1 premires quations, on obtient le fait que le problme pos a une solution
et une seule.
Si n est pair, la dernire quation scrit
0 z1 = an an1 + an2 an3 + . . . a1
Si an an1 + an2 an3 + . . . a1 6= 0, le problme pos na pas de solution.
Si an an1 + an2 an3 + . . . a1 = 0, la dernire quation est vrifie par tout nombre complexe z1 . Le systme se
rduit alors aux n 1 premires quations. Le problme pos a dans ce cas une infinit de solutions.
Remarque. Si n est pair, on peut poser n = 2p, p N . La condition a2p a2p1 + a2p2 a2p3 + . . . + a2 a1 = 0
scrit encore

A1 A2 + A3 A4 + . . . + A2p1 A2p = 0 .

Exercice no 5.

1) Soient a, b et c trois rels. Pour tout rel x, posons P(x) = ax2 + bx + c.

b = 2a + 1
b = 2a + 1
a+b+c =1
P(1) = 1
a + c = 0
a
+
(2a
+
1)
+
c
=
1
2a + b = 1
P (1) = 1

3a + c = 1
a (2a + 1) + c = 0
ab+c =0
P(1) = 0

a=

b = 2a + 1

1
c=a

b=

3a + a = 1

1
c=
4
1 2 1
1
1
x R, P(x) = x + x + x R, P(x) = (x + 1)2 .
4
2
4
4
Il existe un et un seul polynme P de degr 2 tels que P(1) = 1, P (1) = 1 et P(1) = 0 savoir le polynme x 7

1
(x + 1)2 .
4

2) Soient a, b, c et d quatre rels. Pour tout rel x, posons P(x) = ax3 + bx2 + cx + d.

http ://www.maths-france.fr

c Jean-Louis Rouget, 2014. Tous droits rservs.


P(1) = 1
a + b c + d = 1
d = a b c
P(1) = 0
a
+
b
+
c
+
d
=
0
a + b c + (a b c) = 1

P(2) = 1
8a + 4b + 2c + d = 1
8a + 4b + 2c + (a b c) = 1

d = a b c

d = a b c
c = a
2a 2c = 1

2


7a + 3b + c = 1

7a + 3b + a
=1
2

c = a

b = 2a +

d = 2a




1
1
x R, P(x) = ax3 + 2a +
x2 + a
x + 2a.
2
2

Les polynmes P de degr 3 tels que P(1) = 1, P(1) = 0 et P(2) = 1 sont les polynmes de la forme




1
1
x 7 ax3 + 2a +
x2 + a
x + 2a, a R.
2
2

http ://www.maths-france.fr

c Jean-Louis Rouget, 2014. Tous droits rservs.


Planche no 7. Nombres complexes


* trs facile ** facile *** difficult moyenne **** difficile
I : Incontournable T : pour travailler et mmoriser le cours
Exercice no 1 (**IT)
Calculer de deux faons les racines carres de 1 + i et en dduire les valeurs exactes de cos


8

et sin


.
8

Exercice no 2 (**T)
Rsoudre dans C les quations suivantes :
1)
2)
3)
4)
5)

z2 + z + 1 = 0
2z2 + 2z + 1 = 0
z2 2z cos + 1 = 0, rel donn.
z2 (6 + i)z + (11 + 13i) = 0
2z2 (7 + 3i)z + (2 + 4i) = 0.

Exercice no 3 (**IT) (Une construction du pentagone rgulier la rgle et au compas).


2
3
1) On pose z = e2i/5 puis a = z + z4 et b =
une
dont les solutions sont a
z +
 z . Dterminer
 
 quation

 dusecond degr

2
2
4
4

et b et en dduire les valeurs exactes de cos
, sin
, cos
, sin
, cos
et sin
.
5
5
5
5
5
5

1
2) Le cercle de centre daffixe passant par le point M daffixe i recoupe (Ox) en deux points I et J. Montrer que
2
xI + xJ = xI xJ = 1 et en dduire une construction la rgle et au compas, du pentagone rgulier inscrit dans le cercle
de centre O et de rayon 1 dont un des sommets est le point daffixe 1.
3) La diagonale [AC] dun pentagone rgulier (ABCDE) est recoupe par deux autres diagonales en deux points F et G.
AF
FG
Calculer les rapports
et
.
AC
AF
Exercice no 4 (***)
3

1 + i tan
1 + iz

=
.
Soit ] , [ donn. Rsoudre dans C lquation
2 2
1 iz
1 i tan
Exercice no 5 (***I)
Soient A, B et C trois points du plan, deux deux distincts, daffixes respectives a, b et c. Montrer que :
(ABC) quilatral j ou j2 est racine de lquation az2 + bz + c = 0
1
1
1
a2 + b2 + c2 = ab + ac + bc
+
+
= 0.
bc ca ab

Exercice no 6 (**T)

Rsoudre dans C lquation z4 (5 14i)z2 2(5i + 12) = 0.


Exercice no 7 (**I)
Dterminer les complexes z tels que z,

1
et z 1 aient mme module.
z

Exercice no 8 (**IT)
On note U lensemble des nombres complexes de module 1. Montrer que :
z C, (z U \ {1} x R/ z =

Exercice no 9 (**IT)
Forme trigonomtrique de

1 + ix
).
1 ix

1 + cos i sin
1 + ei
.
et de
1 cos + i sin
1 ei

Exercice no 10 (*IT)

Calculer (1 + i 3)9 .
http ://www.maths-france.fr

c Jean-Louis Rouget, 2014. Tous droits rservs.


Exercice no 11 (**T)

8 2
.
Dterminer les racines quatrimes de i et les racines cubiques de
1+i
Exercice no 12 (***)
Montrer que les solutions de lquation 1 + z + z2 + ... + zn1 nzn = 0 sont de module infrieur ou gal 1.
Exercice no 13 (**T)
Pour z C \ {1}, on pose Z =
1)
2)
3)
4)

1+z
. Dterminer et construire lensemble des points M daffixes z tels que
1z

|Z| = 1.
|Z| = 2.
Z R.
Z iR.

Exercice no 14 (*T)
Nature et lments caractristiques de la transformation dexpression complexe :
1)
2)
3)
4)

z
z
z
z

=z+3i
= 2z + 3
= iz + 1
= (1 i)z + 2 + i

Exercice no 15 (**I)
On considre lquation (E) : (z 1)n (z + 1)n = 0 o n est un entier naturel suprieur ou gal 2 donn
1) Montrer que les solutions de (E) sont imaginaires pures.
2) Montrer que les solutions de (E) sont deux deux opposes.
3) Rsoudre (E).
Exercice no 16 (***T) (ESIM 1993)
1 z
1
sh z
(e + ez ), sh z = (ez ez ) et th z =
.
2
2
ch z
Quels sont les nombres complexes z pour lesquels th z existe ?
Rsoudre dans C lquation th
 z = 0.

| Im z| <
Rsoudre dans C le systme
2 .
| th z| < 1


Montrer que la fonction th ralise une bijection de = z C/ | Im z| <
sur U = {z C/ |z| < 1}.
4

Pour z C, on pose ch z =
1)
2)
3)
4)

http ://www.maths-france.fr

c Jean-Louis Rouget, 2014. Tous droits rservs.


Planche no 7. Nombres complexes : corrig


Exercice no 1

On a 1 + i = 2ei/4 . Les racines carres de 1 + i dans C sont donc 4 2ei/8 et 4 2ei/8 .


On a aussi, pour (x, y) R2 ,

2
s

s
x2 = ( 2 + 1)

x y = 1

2
+
1
2

1
.
1

,
(x + iy)2 = 1 + i

(x,
y)

x2 + y2 = 2
2
y = ( 2 1)

2
2

2
xy > 0

xy > 0

!
r
r
2+1
21

Les racines carres de 1 + i sont donc aussi


. Puisque Re(ei/8 ) = cos > 0, on obtient
+i
2
2
8
r
r

2+1
21
4
+i
, ou encore
2ei/8 =
2
2
s
s

q
q

2
+
1
21
1
i/8
+i

e
=
2+ 2+i 2 2
=
2
2 2
2 2
et donc, par identification des parties relles et imaginaires,
cos

1p
1p
2 + 2 et sin =
2 2.
=
8
2
8
2

Exercice no 2

1
1
3
3
= j ou z = i
= j2 .
1) z + z + 1 = 0 z = + i
2
2
2
2
2

1
2) = 12 2 = 1 = i2 . Lquation a donc deux solutions non relles et conjugues, savoir z1 = (1 + i) et
2
1
z2 = (1 i).
2
3) Soit R. Pour tout complexe z, on a
z2 2z cos + 1 = (z cos )2 + 1 cos2 = (z cos )2 + sin2 = (z cos )2 (i sin )2
= (z cos i sin )(z cos + i sin ) = (z ei )(z ei )
Lquation propose a donc deux solutions (pas ncessairement distinctes) z1 = ei et z2 = ei .
De plus, = cos2 1 = sin2 et ces solutions sont distinctes si et seulement si
/ Z.

4) Soit (E) lquation z2 (6 + i)z + (11 + 13i) = 0. Son discriminant est = (6 + i)2 4(11 + 13i) = 9 40i. Comme
40 = 2 20 = 2 (4 5) et que 42 52 = 16 25 = 9, on est en droit de deviner que = (4 5i)2 . Lquation (E) a
6 + i + 4 5i
6 + i 4 + 5i
deux solutions distinctes dans C savoir z1 =
= 5 2i et z2 =
= 1 + 3i.
2
2

5) Soit (E) lquation 2z2 (7 + 3i)z + (2 + 4i) = 0. Son discriminant est = (7 + 3i)2 8(2 + 4i) = 24 + 10i. Comme
10 = 2 5 = 2 (5 1) et que 52 12 = 24, on est en droit de deviner que = (5 + i)2 . Lquation propose a deux
7 + 3i + 5 + i
7 + 3i 5 i
1
solutions distinctes dans C savoir z1 =
= 3 + i et z2 =
= (1 + i).
4
4
2
Exercice no 3
 
 
4
2
4i/5
6i/5
4i/5
4i/5
2i/5
8i/5
2i/5
2i/5
et b = e
+e
=e
+e
= 2 cos
.
1) On a a = e
+e
=e
+e
= 2 cos
5
5
2
3
4
2
3
4
1, z, z , z et z sont les cinq racines cinquimes de 1 dans C. Par suite, 1 + z + z + z + z = 0. Mais alors
a + b = z + z2 + z3 + z4 = 1
et
ab = (z + z4 )(z2 + z3 ) = z3 + z4 + z6 + z7 = z + z2 + z3 + z4 = 1 (car z5 = 1).

http ://www.maths-france.fr

c Jean-Louis Rouget, 2014. Tous droits rservs.


1 5
1 + 5
et
. Enfin, puisque
a et b sont donc les solutions de lquation X + X 1 = 0 dont les racines sont
2
2 

 
 
h
i
1 + 5
1 5
2

4
2
2
=
=
> 0 et donc,
0, , on a a > 0. Par suite, cos
et cos
. Dautre part, sin
5
2
5
4
5
4
5
v
s
!2
 
  u
q
u

2
2
1 + 5
1
t
2
sin
10 + 2 5.
= + 1 cos
= 1
=
5
5
4
4
2

cos

2
5

 

51
1p
2
10 + 2 5.
=
=
et sin
4
5
4

 
 



4
1 5
1p

4
De mme, en remplaant 5 par 5, cos
10 2 5. Enfin, cos
=
=
= cos
=
et sin
5
4
5
4
5
5

 





4
1+ 5

1p
4
cos
10 2 5.
=
= sin
= sin
=
et sin
5
4
5
5
5
4

2) Le rayon du grand cercle vaut, daprs le thorme de Pythagore :

p
5
2
2
R = O + OM =
.
2

 
1 + 5
1 5
2
Donc xI = x + R =
et
et xJ = x R =
. Par suite, xI + xJ = xI xJ = 1 puis xI = 2 cos
2
2
5
 
4
. Ceci montre que les mdiatrices des segments [O, I] et [O, J] coupent le cercle de centre O et de rayon 1
xJ = 2 cos
5
en quatre des cinq sommets du pentagone.
M

3) Posons x =

AF
. Daprs le thorme de Thales (je vous laisse vrifier les paralllismes),
AC

AF
HK
FG
AC 2AF
1 2x
=
=
=
=
.
AC
HC
FC
AC AF
1x

3 5
2
. Puis
Donc x 3x + 1 = 0 et puisque x < 1, x =
2

AC AF
1 + 5
FG
AC 2AF
1
2
1 + 5
3+ 5
AG
2=
=
=1x=
et
=
= 2=
2=
.
AC
AC
2
AF
AF
x
2
2
3 5
x=

http ://www.maths-france.fr

c Jean-Louis Rouget, 2014. Tous droits rservs.


B
C
G
F
K

A
H

D
E
Dfinition du nombre dor.
A

AC
petit
moyen
BC
=
(
=
) cest--dire, en posant
On veut que C partage le segment [A, B] de telle sorte que
AC
AB
moyen
grand
 x 2 x
ax
x
x
1 + 5
x
+ 1 = 0 et donc, puisque > 0, =
ou encore
.
a = AB et x = AC, =
a
x
a
a
a
a
2
1 +
Le nombre dor (ou proportion dore) est le nombre
2
On peut aussi prendre pour le nombre dor le rapport

= 0, 618...

a
1+ 5
=
= 1, 618...
x
2

Exercice no 4
i h 1 + i tan
cos + i sin
, .
=
= e2i . Donc,
Soit
2 2 1 i tan
cos i sin


1 + iz
1 iz

Maintenant, pour k {1, 0, 1},

3

2k
1 + i tan
1 + iz
2
k {1, 0, 1}/
= ei( 3 + 3 ) = k
1 i tan
1 iz

k {1, 0, 1}/ i(k + 1)z = k 1.

2 2k
3
k = 1
+
+ 2Z k +
+ 3Z,
3
3
2
i h
ce qui est exclu pour , . Donc,
2 2


1 + iz
1 iz

3

1 + i tan
k 1
k {1, 0, 1}/ z =
1 i tan
i(k + 1)

k {1, 0, 1}/ z =

k {1, 0, 1}/ z =

ei( 3 +
e

2i sin(
3
i(2 cos( 3

k {1, 0, 1}/ z = tan

http ://www.maths-france.fr

k
3 )

k
i(
3+ 3 )

ei( 3 +
i(e

+
+

k
3 )

k
i(
3+ 3 )

ei( 3 +

k
3 )

k
i(
3+ 3 )

+e

k
3 )
k
3 ))

k
+
3
3

c Jean-Louis Rouget, 2014. Tous droits rservs.


Exercice no 5

(A, B, C) quilatral C = rA, 3 (B) ou C = rA, 3 (B) c a = (j2 )(b a) ou c a = (j)(b a)

(1 j2 )a + j2 b + c = 0 ou (1 j)a + jb + c = 0 ja + j2 b + c = 0 ou j2 a + jb + c = 0
2
j2 a + j2 b + c = 0 ou j2 a + jb + c = 0 j ou j2 sont solutions de lquation az2 + bz + c = 0

(A, B, C) quilatral ja + j2 b + c = 0 ou j2 a + jb + c = 0

(ja + j2 b + c)(j2 a + jb + c) = 0 a2 + b2 + c2 + (j + j2 )(ab + ac + bc) = 0


a2 + b2 + c2 = ab + ac + bc

(A, B, C) quilatral a2 + b2 + c2 ab ac bc = 0

a2 + ab + ac bc b2 + bc + ba ac c2 + ca + cb ab = 0
(c a)(a b) + (a b)(b c) + (b c)(c a) = 0
(c a)(a b) + (a b)(b c) + (b c)(c a)
=0
(b c)(c a)(a b)
1
1
1
+
+
= 0.

bc ca ab

Exercice no 6

Le discriminant de lquation Z2 (5 14i)Z 2(5i + 12) = 0 vaut


(5 14i)2 + 8(5i + 12) = 75 100i = 25(3 4i) = (5(1 2i))2 .
Lquation Z2 (5 14i)Z 2(5i + 12) = 0 admet donc les deux solutions Z1 =
5 14i 5 + 10i
= 2i. Ensuite,
Z2 =
2

5 14i + 5 10i
= 5 12i et
2

z est solution de lquation propose z2 est solution de lquation Z2 (5 14i)Z 2(5i + 12) = 0
z2 = 5 12i = (3 2i)2 ou z2 = 2i = (1 i)2

z = 3 2i ou z = 3 + 2i ou z = 1 i ou z = 1 + i.

Exercice no 7

Soient z un complexe non nul, M le point daffixe z et A le point daffixe 1.



1
1
|z| = |z| =
|z|2 = 1 |z| = 1 OM = 1,
z
|z|

et

Donc,

Exercice no 8

|z| = |z 1| OM = AM M med[OA] xM =

1
1
Re(z) = .
2
2


1
1
1
3
|z| = = |z 1| |z| = 1 et Re(z) = z = i
z = j ou z = j2 .
z
2
2
2

|1 + ix|
|1 + ix|
1 + ix
= 1.
. Puisque 1 ix 6= 0, z est bien dfini et |z| =
=
1 + ix
1 ix
|1 ix|
1 + ix + 2
2
Enfin, z =
= 1 +
6= 1. On a montr que :
1 ix
1 ix
Soient x R et z =

x R,
http ://www.maths-france.fr

1 + ix
U \ {1}.
1 ix
4

c Jean-Louis Rouget, 2014. Tous droits rservs.


Rciproquement, soit z U \ {1}. Il existe un rel


/ + 2Z tel que z = ei . Mais alors,
z = ei =
=

cos 2 (1 + i tan 2 )
cos 2 + i sin 2
ei/2
=
=

i/2
e
cos 2 i sin 2
cos 2 (1 i tan 2 )

1 + i tan 2
1 i tan

(cos

6= 0 car
/ + Z),
2
2
2

et z est bien sous la forme voulue avec x = tan R.


2
Exercice no 9
Soit R.

Donc,

1 + cos + i sin = 0 cos = 1 et sin = 0 + 2Z.

1 + cos i sin
existe pour
/ + 2Z. Pour un tel ,
1 cos + i sin

cos 2
2 cos2 2 2i sin 2 cos 2
cos 2 ei/2
1 + cos i sin
cos(/2) i sin(/2)
=
=
=

1 cos + i sin
sin(/2) + i cos(/2)
sin 2
sin 2 ei()/2
2 sin2 2 + 2i sin 2 cos 2

= i cotan( ).
2
h
[i
[

- 1er cas. cotan > 0


]2k, + 2k[.
k, + k
2
2
2
kZ
kZ
 
 
1 + cos i sin

i/2
Dans ce cas, la forme trigonomtrique de
e
(module= cotan
et argument=
est cotan
1 cos + i sin
2
2

(2)).
2

 


1 + cos i sin
= cotan
,
.
1 cos + i sin
2
2
[

- 2me cas. cotan < 0


] + 2k, 2(k + 1)[.
2
kZ
Dans ce cas,

 
 

i/2

1 + cos i sin
i/2

e
= cotan
e
,
= cotan
1 cos + i sin
2
2
et donc,



1 + cos i sin

.
= cotan( ),
1 cos + i sin
2 2
- 3me cas. cotan
Pour
/ 2Z, on a

1 + cos i sin
= 0 + 2Z. Dans ce cas, on a
= 0.
2
1 cos + i sin

2 cos 2
ei/2 (ei/2 + ei/2 )
1 + ei

= i/2 i/2
= i cotan .
=
i
1e
2
e
(e
ei/2 )
2i sin 2


[
1 + ei

Si
]2k, + 2k[,
.
= cotan ,
i
1e
2 2
kZ


[

1 + ei
.
=

cotan
,

Si
] + 2k, 2(k + 1)[,
1 ei
2
2
kZ

Si + 2Z,

1 + ei
= 0.
1 ei

http ://www.maths-france.fr

c Jean-Louis Rouget, 2014. Tous droits rservs.


Exercice no 10

9
1 + i 3 = (2ei/3 )9 = 29 e3i = 512.
La forme algbrique dun complexe est particulirement bien adapte laddition.
La forme trigonomtrique dun complexe est particulirement bien adapte la multiplication.
Exercice no 11

i = ei/2 et les racines quatrimes de i sont donc les ei( 8 + 2 ) , k {0, 1, 2, 3}. Ensuite,

8
8 2
= i/4 = 8ei/4 = 2e3i/4 .

1+i
e

8 2
2k

sont donc les nombres 2ei( 4 + 3 ) , k {1, 0, 1}, ou encore les trois nombres
Les racines cubiques de
1
+
i

2ei 4 = 2(1 + i), 2ei/12 et 2e11i/12 .


Exercice no 12
Soit z C tel que |z| > 1.
|1 + z + ... + zn1 | 6 1 + |z| + |z|2 + ... + |z|n1 < |z|n + |z|n + ... + |z|n = n|z|n = |nzn |,
et en particulier, 1 + z + ... + zn1 6= nzn . Donc, si 1 + z + ... + zn1 nzn = 0, alors |z| 6 1.
Exercice no 13
A- Solutions algbriques. Pour z C \ {1}, posons z = x + iy o (x, y) R2 .
1) Soit z C \ {1}.
|Z| = 1

|1 + z|2
= 1 (1 + x)2 + y2 = (1 x)2 + y2 4x = 0 x = 0.
|1 z|2

Lensemble cherch est la droite (Oy) (car le point daffixe 1 nappartient pas (Oy)).
2) Soit z C \ {1}.

10
|Z| = 2 (1 + x)2 + y2 = 4((1 x)2 + y2 ) 3x2 + 3y2 10x + 3 = 0 x2 + y2 x + 1 = 0
3

2
5
16
x
+ y2 =
3
9


4
5
, 0 et de rayon (car le point daffixe 1 nappartient pas ce cercle).
Lensemble cherch est le cercle de centre
3
3
3) Soit z C \ {1}.
ZRZ=Z

1+z
1+z
=
(1 + z)(1 z) = (1 z)(1 + z) z z = z z z = z z R.
1z
1z

Lensemble cherch est la droite (Ox) priv du point de coordonnes (1, 0).
4) Soit z C \ {1}.

1+z
1+z
=
(1 + z)(1 z) = (1 z)(1 + z)
1z
1z
1 zz = 1 + zz |z|2 = 1 |z| = 1.

Z iR Z = Z

Lensemble cherch est le cercle de centre O et de rayon 1, priv du point de coordonnes (1, 0).

B- Solutions gomtriques (pour 1), 3) et 4)). Soient A et B les points daffixes respectives 1 et 1, M le point
daffixe z et E lensemble cherch.
1)
http ://www.maths-france.fr

c Jean-Louis Rouget, 2014. Tous droits rservs.


3) Soit M 6= B.

M E |z + 1| = |z 1| AM = BM M med[AB] = (Oy).

1+z
= 0 ()
1z

M = A ou M 6= A et (BM, AM) = 0 ()

M E z = 1 ou z 6= 1 et arg

M (AB) \ {B}.

et on retrouve la droite (Ox) prive du point B(1, 0).


4) Soit M 6= B.


1+z

= ()
1z
2

M = A ou M 6= A et (BM, AM) = ()
2
M est sur le cercle de diamtre [AB] priv de B.

M E z = 1 ou z 6= 1 et arg

et on retrouve le cercle de centre O et de rayon 1 priv du point B(1, 0).


Exercice no 14
Soit f la transformation considre.

1) f est la translation de vecteur


u (3, 1).

2) = 2 + 3 = 3. f est lhomothtie de rapport 2 et de centre (3, 0).


1

1 1
3) = i + 1 = (1 + i). Comme i = ei/2 , f est la rotation dangle
et de centre ( , ).
2
2
2
2

4) = (1 i) + 2 + i = 1 2i. Comme 1 i = 2ei/4 , f est la similitude de centre (1, 2), de rapport 2 et

dangle .
4
Exercice no 15
1) Soit z C. Soient M, A et B les points daffixes respectives z, 1 et 1.
z solution de (E) (z 1)n = (z + 1)n |(z 1)n | = |(z + 1)n | |z 1|n = |z + 1|n
|z 1| = |z + 1| (car |z 1| et |z + 1| sont des rels positifs)
AM = BM M med[AB] M (Oy) z iR.

2) Soit z C.

(z 1)n (z + 1)n = (1)n ((z + 1)n (z 1)n ) = (1)n ((z 1)n (z + 1)n ).
Par suite,
z solution de (E) (z 1)n (z + 1)n = 0 (z 1)n (z + 1)n = 0 z solution de (E).

3) Soit z C.

z solution de (E) (z 1)n = (z + 1)n k J0, n 1K/ z + 1 = e2ik/n (z 1)

e2ik/n 1
eik/n eik/n
k J1, n 1K/ z = ik/n
2ik/n
e
+1
e
+ eik/n
k
2i sin n
k
.
k J1, n 1K/ z = i cotan
k J1, n 1K/ z =
n
2 cos k
n

k J1, n 1K/ z =

Les solutions de lquation (E) sont les nombres de la forme i cotan

http ://www.maths-france.fr

k
, 1 6 k 6 n 1.
n

c Jean-Louis Rouget, 2014. Tous droits rservs.


Exercice no 16
1) Soit z C. sh z et ch z sont dfinis et donc, th z existe si et seulement si ch z 6= 0. Or,


ch z = 0 ez + ez = 0 e2z = 1 e2z+i = 1 2z + i 2iZ z i + Z .
2




th z existe si et seulement si z
/ i + Z ou encore z
/i
+ Z .
2
2


+ Z .
2) Soit z
/i
2

th z = 0 sh z = 0 ez = ez e2z = 1 2z 2iZ z iZ.



Comme i
+ Z iZ = , th z = 0 si et seulement si z iZ.
2


+ Z . Posons z = x + iy o (x, y) R2 .
3) Soit z
/i
2



2
2




| th z| < 1 ez ez < ez + ez ez ez ez ez < ez + ez ez + ez
ezz e(zz) < ezz + e(zz) 2(e2iy + e2iy ) > 0
cos(2y) > 0

Par suite,

2
| th z| < 1
| Im z| <

|y| < z .
2
4
cos(2y) > 0

|y| <

4) Soit z . Daprs 1), th z existe et daprs 3), | th z| < 1. Donc z th z U. Ainsi, th est une application de
dans U.
Soit alors Z U et z .

Puisque Z 6= 1,

Puisque |Z| < 1,

th z = Z

e2z 1
1+Z
= Z e2z =
.
2z
e +1
1Z

1+Z
1+Z
6= 0 et on peut poser
= rei o r R+ et ] , ].
1Z
1Z

2Re
i h
et donc , .
2 2

1+Z
1Z


2 1 |Z|2
1+Z 1+Z
=
=
>0
+
1Z 1Z
|1 Z|2

En posant z = x + iy o (x, y) R2 .

1+Z
e2z = rei e2x = r et 2y + 2Z
1Z
1

x = ln r et y + Z
2
2

1
k Z/ z = ln r + + k.
2
2
i h
h
i


Puisque , , on a ,
puis
2 2
2
4 4
i h

k = 0.
+ k ,
2
4 4
Mais alors,


x = ln r
2
th z = Z
.

y=
2
e2z =

http ://www.maths-france.fr

c Jean-Louis Rouget, 2014. Tous droits rservs.








1 1 + Z i
1+Z
1+Z
Ainsi, tout lment Z de U a un et un seul antcdent z dans ( savoir z = ln
, Arg
+ Arg
2
1 Z 2
1Z
1Z
1+Z
dsignant largument de
qui est dans ] , [).
1Z
th ralise donc une bijection de sur U.

http ://www.maths-france.fr

c Jean-Louis Rouget, 2014. Tous droits rservs.


Planche no 8. Gnralits sur les fonctions


* trs facile ** facile *** difficult moyenne **** difficile
I : Incontournable T : pour travailler et mmoriser le cours
Exercice no 1 (**T)
Etudier la parit des fonctions suivantes :
x5 x
x2 1
5) f5 (x) = cos(2x) + tan2 (x)

1) f1 (x) = 3x4 5x2 + 1


ex 1
4) f4 (x) = x
e +1

7) f7 (x) = x2 + 1

2) f2 (x) =

8) f8 (x) =

x
x6 x4 31x2 + 3

x5 x
x3 1
6) f6 (x) = sin(x) x
3) f3 (x) =

9) f9 (x) = cos(x) + sin(x).

Exercice no 2 (***I)
Soit f une application de R dans R. Montrer que f scrit de manire unique comme la somme dune fonction paire et
dune fonction impaire.
Exercice no 3 (**I)
Soit f une application de R dans R, drivable sur R. Montrer que si f est paire, sa drive est impaire et si f est impaire,
alors sa drive est paire.
Gnraliser f , f(3) , . . . , f(n) , n > 2.
A-t-on des rsultats analogues pour les primitives ?
Exercice no 4 (**T)
1) Montrer que la droite dquation x =
f : x 7 x2 3x + 2.

3
est un axe de symtrie du graphe dans un repre orthonorm de la fonction
2

2x + 1
2) Montrer que le point I de coordonnes (1, 2) est centre de symtrie du graphe de la fonction f : x 7
.
x1


ex
1
est centre de symtrie du graphe de la fonction f : x 7 x
3) Montrer que le point I de coordonnes 0,
.
2
e +1
4) Etudier les symtries de la courbe reprsentative de la fonction f : x 7 cos(x) + cos(3x).

Exercice no 5

(**T)

Etudier la priodicit des fonctions suivantes :


1) f1 (x) = E(x) x
4) f4 (x) = cos (4x)
no 6

2) f2 (x) = E(2x)
2x
 
3x
5) f5 (x) = cos
2

3) f3 (x) = cos(2x)

 sin(4x)
2x
6) f6 (x) = cos
.
3

(**T)

x2
est borne sur R.
x2 + 1
x
est borne sur R.
2) Montrer que la fonction f : x 7 2
x +1
Exercice no 7 (**T)
1) Montrer que la fonction f : x 7

Etudier le sens de variation des fonctions suivantes sur le domaine considr.




5
1
1) x 7 3(x + 1)2 + 1, I = R
2) x 7 3
, I = ,
2x + 1
2


Zx
1
1
4) x 7 exp 1 2
, I = R 5) x 7
dt
2
ln |x| + 1
0 1+t

http ://www.maths-france.fr

3) x 7 ln(1 + ex ), I = R
Z1
6) x 7 ext dt
0

c Jean-Louis Rouget, 2014. Tous droits rservs.


Exercice no 8 (**T)
Etudier le sens de variation des fonctions suivantes sur lintervalle considr.


h i

3x2 1
2) x 7 x ln x, I = [1, +[ 3) x 7 e
cos
,I=R
1) x 7 sin x cos x, I = 0,
4
2(x2 + 1)
4
2x + 3
x 1
4) x 7
, I =]1, +[
5) x 7 4
,I=R
6) x 7 x7 + x4 + x2 + 3, I =] , 0]
x1
x +1
Exercice no 9 (*T)

1) Montrer que, si x est un rel tel que 1 6 x 6 2, on a

3
2x + 1
5
6
6 .
11
4x + 3
7

3
2x + 1
=
. Que constatez-vous ?
4x + 3
11
2x + 1
pour x [1, 2].
3) Encadrer au mieux
4x + 3
Exercice no 10 (*T)
2) Rsoudre lquation

Encadrer au mieux les expressions suivantes sur le domaine D considr :


1) x2 , D = [1, 2]

2) x2 3x + 2, D = [0, 4]

5) cos x, D = [ , ]
4

6)

5x + 1
, D = [0, 4]
13x + 2

1
, D = [2, 1]
x
2n + 3
7)
,nN
2n 9

1
, D = [1, 1] \ {0}
x2
4n + 1
8)
,nN
3n + 7

3)

4)

2x2 7x + 1
6 10x
x+3

3) x 6 0,

Exercice no 11 (**T)
Dmontrer les ingalits suivantes :
1) x [1, 3], 2x2 5x + 3 6 3x 3

2) x [1, +[,

x2 x + 1 > x

Exercice no 12 (*I)

Etudier le signe de x2 + 1 x et x2 + 1 + x suivant les valeurs de x.


Exercice no 13 (**I)
Dmontrer (et mmoriser) les ingalits classique suivantes :
1) x > 0, sin x 6 x

http ://www.maths-france.fr

2) x R, ex > 1 + x

3) x ] 1, +[, ln(1 + x) 6 x

c Jean-Louis Rouget, 2014. Tous droits rservs.


Planche no 8. Gnralits sur les fonctions : corrig


Exercice no 1
1) f1 est dfinie sur R qui est symtrique par rapport 0. De plus, pour tout rel x
f1 (x) = 3(x)4 5(x)2 + 1 = 3x4 5x2 + 1 = f1 (x).
La fonction f1 est paire.
2) f2 est dfinie sur R \ {1, 1} qui est symtrique par rapport 0. De plus, pour tout x de R \ {1, 1}
f2 (x) =

(x)5 (x)
x5 x
= 2
= f2 (x).
2
(x) 1
x 1

La fonction f2 est impaire.


3) f3 (1) existe mais f3 (1) nexiste pas. Le domaine de dfinition de la fontion f3 nest pas symtrique par rapport 0
et donc, la fonction f3 nest ni paire, ni impaire.
4) Pour tout rel x, ex + 1 > 1, en particulier pour tout rel x, ex + 1 6= 0. f4 est dfinie sur R qui est symtrique par
rapport 0. De plus, pour tout x de R
1
1
1 ex
ex 1
x
ex 1
e
=
=

=
= f4 (x).
f4 (x) = x
1
e +1
1 + ex
ex + 1
+
1
ex
La fonction f4 est impaire.




5) f2 est dfinie sur R \
+ Z qui est symtrique par rapport 0. De plus, pour tout x de R \
+ Z
2
2
f5 (x) = cos(2x) + tan2 (x) = cos(2x) + tan2 (x) = f5 (x).
La fonction f5 est paire.
6) f2 est dfinie sur R qui est symtrique par rapport 0. De plus, pour tout x de R
f6 (x) = sin(x) (x) = (sin(x) x) = f6 (x).
La fonction f6 est impaire.
7) Pour tout rel x, x2 + 1 > 0. Donc f7 est dfinie sur R qui est symtrique par rapport 0. De plus, pour tout x de R
q
p
f7 (x) = (x)2 + 1 = x2 + 1 = f7 (x).
La fonction f7 est paire.

8) On ne peut pas dterminer le domaine de dfinition de la fonction f8 . Mais comme (x)6 (x)4 31(x)2 + 3 =
x6 x4 31x2 + 3, pour tout rel x, f8 (x) existe si et seulement si f8 (x) existe. Donc f8 est dfinie sur un domaine D
qui est symtrique par rapport 0. De plus, pour tout x de D
f8 (x) =

(x)6

x
x
= 6
= f8 (x).
4
2
4
(x) 31(x) + 3
x x 31x2 + 3

La fonction f8 est impaire.



 
= 1 6= 1 = f9
. Donc, f9 nest pas paire.
9) f9 (0) = 1 6= 0 et donc f9 nest pas impaire. f9
2
2
La fonction f9 nest ni paire, ni impaire.
Exercice no 2
Soit f une application de R dans R.
Unicit. Supposons quil existe deux fonctions g et h telles que g est paire, h est impaire et f = g + h. Ncessairement
pour tout rel x,

f(x) = g(x) + h(x)
.
f(x) = g(x) h(x)
En additionnant et en retranchant membre membre ces deux galits, on obtient pour tout rel x
http ://www.maths-france.fr

c Jean-Louis Rouget, 2014. Tous droits rservs.


1
1
(f(x) + f(x)) et h(x) = (f(x) f(x)).
2
2
Ceci montre lunicit dun couple (g, h) tel que g est paire, h est impaire et f = g + h.
g(x) =

Existence. Rciproquement, posons pour tout rel x


1
1
(f(x) + f(x)) et h(x) = (f(x) f(x)).
2
2
1
1
Pour tout rel x, g(x) + h(x) = (f(x) + f(x)) + (f(x) f(x)) = f(x) et donc f = g + h.
2
2
1
Pour tout rel x, g(x) = (f(x) + f(x)) = g(x) et donc la fonction g est paire.
2
1
1
Pour tout rel x, h(x) = (f(x) f(x)) = (f(x) f(x)) = h(x) et donc la fonction h est impaire.
2
2
Ainsi, les fonctions g et h conviennent ce qui montre lexistence dun couple (g, h) tel que g est paire, h est impaire et
f = g + h.
g(x) =

Exercice no 3
Soit n un entier naturel. Supposons que f est n fois drivable sur R. Pour tout rel x, posons g(x) = f(x). Alors g est n
fois drivable sur R et pour tout rel x,
g(n) (x) = (1)n f(n) (x).
Si de plus f est paire, alors g = f et donc pour tout rel x, f(n) (x) = (1)n f(n) (x) ou encore
pour tout rel x, f(n) (x) = (1)n f(n) (x).
Si n est un entier pair, posons n = 2p o p est un entier. Lgalit prcdente scrit
pour tout rel x, f(2p) (x) = f(2p) (x)
et donc f(2p) est paire. Ainsi, les fonctions f, f , f(4) , f(6) . . . sont paires.
Si n est un entier pair, posons n = 2p o p est un entier. Lgalit prcdente scrit
pour tout rel x, f(2p+1) (x) = f(2p) (x)
et donc f(2p+1) est impaire. Ainsi, les fonctions f , f(3) , f(5) . . . sont impaires. On rsume ces rsultats en disant que
si f est paire, f(n) a la parit de n.
Si f est impaire, alors f est paire et on peut lui appliquer les rsultats prcdents. Donc,
si f est impaire, f(n) a la parit contraire de celle de n.
Une primitive de fonction impaire est automatiquement une fonction paire. Dmontrons-le.
Soit f une fonction continue sur R et impaire. Soit F une primitive de f. Pour tout rel x, posons G(x) = F(x) F(x). G
est drivable sur R et pour tout rel x
G (x) = F (x) + F (x) = f(x) + f(x) = 0.
Donc, la fonction G est constante sur R puis, pour tout rel x,
G(x) = G(0) = F(0) F(0) = 0.
On en dduit que pour tout rel x, F(x) = F(x) et donc que F est paire.
Le rsultat analogue est faux si on suppose que f est paire car une fonction constante est paire. Voici une succession de
primitives pour exemple :
x 7 1,

http ://www.maths-france.fr

x 7 x + 4, x 7

x2
x3
5
+ 4x 5, x 7
+ 2x2 x + 1 . . .
2
6
2

c Jean-Louis Rouget, 2014. Tous droits rservs.


Exercice no 4
1) La fonction f est dfinie sur R qui est symtrique par rapport

3
. De plus, pour tout rel x,
2

f(3 x) = (3 x)2 3(3 x) + 2 = x2 3x + 2 = f(x).


3
est un axe de symtrie du graphe dans un repre orthonorm de la fonction f.
2
2) La fonction f est dfinie sur R \ {1} qui est symtrique par rapport 1. De plus, pour tout rel x de R \ {1}
Donc, la droite dquation x =

f (2xI x) + f(x) =

2(2 x) + 1 2x + 1
2x 5 2x + 1
4x 4
4(x 1)
+
=
+
=
=
= 4 = 2yI .
(2 x) 1
x1
x1
x1
x1
x1

Donc, le point I de coordonnes (1, 2) est centre de symtrie du graphe de la fonction f.


3) La fonction f est dfinie sur R qui est symtrique par rapport 0. De plus, pour tout rel x,
1
ex
x
ex
1
ex
ex + 1
ex
+ x
+ x
= e
x1= x
+ x
= x
= 1 = 2yI .
f (2xI x) + f(x) = x
1
e +1 e +1
e +1
e +1 e +1
e +1
+
1
ex


1
Donc, le point I de coordonnes 0,
est centre de symtrie du graphe de la fonction f.
2
4) La fonction f est paire et donc laxe des ordonnes est un axe de symtrie du graphe de f. Plus gnralement, pour
tout entier relatif k et pour tout rel x,
f(2k x) = cos(2k x) + cos(6k 3x) = cos(x) + cos(3x) = cos(x) + cos(3x) = f(x).
On en dduit que pour tout entier relatif k, la droite dquation x = k est un axe de symtrie du graphe de f.
Pour tout rel x,
f( x) = cos( x) + cos(3 3x) = cos( x) + cos( 3x) = cos(x) cos(3x) = f(x),
 
, 0 est un centre de symtrie du
et donc pour tout rel x, f( x) + f(x) = 0. On en dduit que le point de coordonnes
 2

graphe de f. Plus gnralement, pour tout entier relatif k, le point de coordonnes
+ k, 0 est un centre de symtrie
2
du graphe de f.
Exercice no 5
Chacune des fonctions considres est dfinie sur R.
1) Pour tout rel x, f1 (x + 1) = E(x + 1) (x + 1) = E(x) + 1 x 1 = E(x) x = f1 (x) et donc la fonction f1 est
1-priodique. Plus gnralement, pour tout entier relatif k, f1 est k-priodique.


1
1
2) Pour tout rel x, f2 x +
= E(2x + 1) (2x + 1) = E(2x) 2x = f2 (x) et donc la fonction f2 est -priodique. Plus
2
2
k
gnralement, pour tout entier relatif k, f2 est -priodique.
2
3) Pour tout rel x, f3 (x + ) = cos(2x + 2) sin(4x + 4) = cos(2x) sin(4x) = f3 (x) et donc la fonction f3 est
-priodique. Plus gnralement, pour tout entier relatif k, f3 est k-priodique.



4) Pour tout rel x, f4 x +


= cos(4x+2) = cos(4x) = f4 (x) et donc la fonction f4 est -priodique. Plus gnralement,
2
2
k
-priodique.
pour tout entier relatif k, f4 est
2




 
4
4
3x
3x
5) Pour tout rel x, f5 x +
= cos
= f5 (x) et donc la fonction f5 est
+ 2 = cos
-priodique. Plus
3
2
2
3
4k
-priodique.
gnralement, pour tout entier relatif k, f5 est
3

 

2x
2x
= f6 (x) et donc la fonction f6 est 3-priodique. Plus
+ 2 = cos
6) Pour tout rel x, f6 (x + 3) = cos
3
3
gnralement, pour tout entier relatif k, f6 est 3k-priodique.

http ://www.maths-france.fr

c Jean-Louis Rouget, 2014. Tous droits rservs.


Exercice no 6
Pour tout rel x, x2 + 1 > 1 et en particulier, pour tout rel x, x2 + 1 6= 0. Donc, les deux fonctions sont dfinies sur R.
1) Pour tout rel x,
x2
x2 + 1
6 2
= 1.
+1
x +1
x2
est borne sur R.
Donc, pour tout rel x, 0 6 f(x) 6 1. La fonction f : x 7 2
x +1
06

x2

2) Pour tout rel x,

Donc, pour tout rel x,


Exercice no 7

1
|x|
(|x| 1)2 > 0 x2 2|x| + 1 > 0 x2 + 1 > 2|x| > 2
2
x
+1


x 1
6 1 6 f(x) 6 1 .
2
x + 1 2
2
2

1
x
1
6 f(x) 6 . La fonction f : x 7 2
est borne sur R.
2
2
x +1

1) Pour tout rel x, f(x) = k(h(g(x))) o g : x 7 x + 1, h : y 7 y2 et k : z 7 z + 1.


La fonction g est strictement croissante sur [1, +[ valeurs dans [0, +[, la fonction h est strictement croissante sur
[0, +[ valeurs dans R et la fonction k est strictement dcroissante sur R. Donc la fonction f est strictement dcroissante
sur [1, +[.
La fonction g est strictement croissante sur ], 1] valeurs dans ], 0], la fonction h est strictement dcroissante sur
] , 0] valeurs dans R et la fonction k est strictement dcroissante sur R. Donc la fonction f est strictement croissante
sur ] , 1].

1
2) Pour tout rel x, f(x) = k(h(g(x))) o g : x 7 2x + 1, h : y 7 et k : z 7 3 5z.
y


1
valeurs dans ] , 0[, la fonction h est strictement dcroissante sur
La fonction g est strictement croissante sur ,
2
] ,
dans R et la fonction k est strictement dcroissante sur R. Donc la fonction f est strictement croissante
 0[ valeurs

1
sur , .
2

3) La fonction x 7 1 + ex est strictement croissante sur R, valeurs dans ]0, +[ et la fonction X 7 ln X est strictement
croissante sur ]0, +[. Donc, x 7 est strictement croisante sur R.


1
4) La fonction x 7 exp 1 2
est dfinie sur R et paire.
ln |x| + 1
La fonction x 7 ln x est croissante et ngative sur ]0, 1]. Donc, la fonction x 7 1 + ln2 x est dcroissante et stric1
tement positive sur ]0, 1]. Par suite, la fonction x 7 1 2
est dcroissante sur ]0, 1] et enfin, la fonction
ln x + 1




1
1
est dcroissante sur ]0, 1]. De mme, la fonction x 7 exp 1 2
est croissante
x 7 exp 1 2
ln |x| + 1
ln |x| + 1


1
sur [1, +[. Par parit, la fonction x 7 exp 1 2
est dcroissante sur ] , 1] et croissante sur [1, 0[.
ln |x| + 1

1
0 et donc, f5 (x )f5 (x) =
5) Soient x et x deux rels tels que x 6 x . Alors, pour tout rel t de lintervalle [x, x ],
1
+
t2
Z x
1
dt > 0 (par positivit de lintgrale). f5 est croissante sur R.
2
x 1+t

6) Soient x et x deux rels tels que x 6 x . Alors, pour tout rel t de lintervalle [0, 1], on a xt 6 x t, puis pour tout rel

t de [0, 1], ext 6 ex t . Par croissance de lintgrale, on a alors f6 (x) 6 f6 (x ). f6 est croissante sur R.
Exercice no 8
h i
1) La fonction x 7 sin x est strictement croissante sur 0, . La fonction x 7 cos x est strictement dcroissante sur
4
h i
h i
h i
0,
et donc la fonction x 7 cos x est strictement croissante sur 0, . f1 est strictement croissante sur 0,
en
4
4
4
h i
tant que somme de deux fonctions strictement croissantes sur 0, .
4
http ://www.maths-france.fr

c Jean-Louis Rouget, 2014. Tous droits rservs.


2) Les deux fonctions x 7 x et x 7 ln x sont croissantes et positives sur [1, +[. Donc, f2 est croissante sur [1, +[ en
tant que produit de fonctions croissantes et positives sur [1, +[.
h i

+
et la fonction X 7 cos X est dcroissante sur
est
dcroissante
sur
R

valeurs
dans
0,
3) f3 est paire. x 7
2(x2 + 1) 
2


[0, ]. Donc, la fonction x 7 cos


est croissante sur R+ , et de plus positive sur R+ . f3 est croissante sur R+
2
2
2(x + 1)
en tant que produit de fonctions croissantes et positives sur R+ . Par parit, f3 est dcroissante sur R .
4) 2 (1) 1 (3) = 5 < 0. Donc, f4 est strictement dcroissante sur ] , 1[ et strictement dcroissante sur ]1, +[.

5) f5 est paire. La fonction x 7 x4 est croissante sur R+ valeurs dans R+ . Puisque 1 1 1 (1) = 2 > 0, la fonction
X1
est croissante sur ] 1, +[ et en particulier sur R+ . f5 est croissante sur R+ . Par parit, f5 est dcroissante
X 7
X+1
sur R .
6) Les fonctions x 7 x7 et x 7 x4 + x2 + 3 sont strictement dcroissantes sur ] , 0]. Donc, f6 est strictement
dcroissante sur ] , 0] en tant que somme de fonctions strictement dcroissantes sur ] , 0].

Exercice no 9

1) Si 1 6 x 6 2, 3 6 2x + 1 6 5 et 0 < 7 6 4x + 3 6 11. Par suite, 0 < 3 6 2x + 1 6 5 et 0 <

1
1
1
6
6 , puis
11
4x + 3
7

2x + 1
5
3
6
6 .
11
4x + 3
7
2x + 1
3
1
2)
=
11(2x + 1) = 3(4x + 3) 10x = 2 x = . La solution obtenue nest pas dans lintervalle [1, 2],
4x + 3
11
5
2x + 1
3
ou encore lquation
=
na pas de solution dans lintervalle [1, 2]. Ceci signifie que lencadrement fourni au 1)
4x + 3
11
est trop large et peut donc srement tre amlior.
2x + 1
est croissante sur [1, 2]. Par suite, si x [1, 2],
4x + 3
2x + 1
5
3
= f(1) 6
= f(x) 6 f(2) =
.
7
4x + 3
11

3) Puisque 2 3 1 4 = 2 > 0, la fonction f : x 7


Exercice no 10

1) f1 est dcroissante sur [1, 0] et croissante sur [0, 2]. f1 admet donc un minimum gal f1 (0) = 0 et un maximum gal
Max{f1 (1), f1 (2)} = 4. Pour x [1, 2], 0 6 x2 6 4.
 
1
3
2) Pour x [0, 4], = = f2
6 f2 (x) 6 f2 (4) = 6.
4
2
1
1
6 .
x
2
4) f4 est paire, dcroissante sur ]0, 1]. f4 nest pas majore sur ]0, 1] et admet un minimum gal f4 (1) = 1.
1
Pour x [1, 1] \ {0}, 2 > 1.
x
h i
h i

1
5) f5 est dcroissante sur
, . Pour x
, , 1 = cos 6 cos x 6 cos = .
4
4
4
2
3
5 2 13 1
2
=
< 0. f6 est donc

/ [0, 4]) et pour x [0, 4], f6 (x) =


6) f6 est drivable sur [0, 4] (car
13
(13x + 2)2
(13x + 2)2
7
5x + 1
1
dcroissante sur [0, 4] et pour x [0, 4],
= f6 (4) 6 f6 (x) =
6 f6 (0) = .
18
13x + 2
2


9
2x + 3
9
et aussi sur
7) Pour x 6= , posons f7 (x) =
. Puisque 2 (9) 3/times2 < 0, f7 est dcroissante sur 0,
2
2x 9
2


2p + 3
2n + 3
25+3
9
, + . Par suite, pour n > 5, 1 = limp+
6
6
= 13 et pour 0 6 n 6 4,
2
2p 9
2n 9
259
24+3
2n + 3
20+3
1
2n + 3
11 =
6

= . La plus petite valeur de


est 11 (obtenue pour n = 4) et la plus
249
2n 9
209
3
2n 9
2n + 3
est 13 (obtenue pour n = 5).
grande valeur de
2n 9
3) f3 est dcroissante sur [2, 1]. Donc, pour x [2, 1], f2 (1) 6 f2 (x) 6 f2 (2), ou encore 1 6

http ://www.maths-france.fr

c Jean-Louis Rouget, 2014. Tous droits rservs.


28
4
(3n + 7)
+1
4
4n + 1
3
3
=
=
8) Pour n N,
3n + 7
3n + 7
3
que pour tout entier naturel n,

1
4n + 1
4
6
< .
7
3n + 7
3

25
7
9(n + )
3

. La suite

4n + 1
3n + 7

est donc croissante. On en dduit

nN

Exercice no 11
1) Soit x [1, 3]. (2x2 5x + 3) (3x 3) = 2x2 8x + 6 = 2(x2 4x + 3) = 2(x 1)(x 3) 6 0. Donc, pour x [1, 3],
2x2 5x + 3 6 3x 3.
2x2 + 7x + 1
2x2 + 7x2 + x2
2x2 7x + 1
6
6
= 10x.
x+3
x+3
x
2


3
1
+ > 0 et donc x2 x + 1 existe pour tout rel x. De plus, pour tout rel
3) Pour tout rel x, x2 x + 1 = x
2
4

x, x2 x + 1 > 0.

Pour x 6 0, x2 x + 1 x > 0 et donc x2 x + 1 + x a le mme signe que


 p

p
x2 x + 1 + x
x2 x + 1 x = x + 1 > 0.

Ainsi, pour tout rel x 6 0, x2 x + 1 + x > 0 et donc pour tout rel x 6 0, x2 x + 1 > x.
2) Pour x > 1,

Exercice no 12

Pour tout rel x, x2 + 1 > 0 et donc, pour tout rel x, x2 + 1 existe. Ensuite, pour tout rel x,
p

x2 + 1 > x2 = |x|.

2
2
2
Par suite,
pour tout rel x, x + 1 > |x| > x et donc x + 1 x > 0 et aussi, pour tout rel x, x + 1 > |x| > x et
donc x2 + 1 + x > 0.
Exercice no 13
1) Pour x > 0, posons f(x) = sin x x. f est drivable sur [0, +[ et pour x > 0, on a f (x) = cos x 1 6 0. f est donc
dcroissante sur [0, +[. Mais f(0) = 0 et f est ngative sur [0, +[ ce qui dmontre lingalit de lnonc.

Commentaire.
h i La dmonstration prcdente ne tient pas debout car la formule (sin) = cos a t tablie en sachant que
pour x 0, , on a sin(x) 6 x. Nanmoins, cest ce qui est attendu dans la pratique (et il se trouve quon est capable
2
de dmontrer que (sin) = cos sans utiliser lingalit sin(x) 6 x).

2) Pour x R, posons f(x) = ex 1 x. f est drivable sur R et pour x R, on a f (x) = ex 1. f est ngative sur R
et positive sur R+ . f admet donc un minimum en 0 gal f(0) = 0. f est ngative sur R ce qui dmontre lingalit de
lnonc.
1
x
3) Pour x > 1, posons f(x) = ln(1+ x)x. f est drivable sur ]1, +[ et pour x > 1, on a f (x) =
1 =
.
1+x
1+x

+
f est positive sur ] 1, 0] et ngative sur R . f admet donc un maximum en 0 gal f(0) = 0.
On en dduit que f est ngative sur ] 1, +[ ce qui dmontre lingalit de lnonc.

http ://www.maths-france.fr

c Jean-Louis Rouget, 2014. Tous droits rservs.


Planche no 9. Valeur absolue. Partie entire. Ingalits


* trs facile ** facile *** difficult moyenne **** difficile
I : Incontournable T : pour travailler et mmoriser le cours
Exercice no 1. (**I) (Moyennes arithmtique, gomtrique et harmonique)
x+y

(moyenne arithmtique), g = xy (moyenne gomtrique)


Soient x et y deux rels tels que 0 < x 6 y. On pose m =
2


1
1 1
1
et =
(moyenne harmonique). Montrer que x 6 h 6 g 6 m 6 y.
+
h
2 x y
Exercice no 2. (*I) (Ingalit de Bernoulli)
Montrer que, pour a rel positif et n entier naturel donns, (1 + a)n > 1 + na.
Exercice no 3. (***)
On veut montrer de manire lmentaire (cest--dire

nen se passant du logarithme nprien et en ne travaillant quavec les
1
deux oprations + et ) que pour n N , 1 +
< 3.
n
Pour cela dvelopper, puis majorer uk =

Ckn
uk+1
1
en commenant par majorer vk =
par .
nk
uk
2

Exercice no 4. (***I)
Soient n N et a1 , a2 ,..., an , nrels strictement
 positifs.
1
1
1
> n2 (dvelopper et penser f(x) = x + ).
Montrer que (a1 + a2 + ... + an )
+ ... +
a1
an
x
Exercice no 5. (***I) (Ingalit de Cauchy-Schwarz)
Soient n N et a1 , a2 ,..., an , b1 , b2 ,..., bn , 2n rels. Montrer que
v
v
n

u n
u n
n
X
X
uX 2 uX


t
a k bk 6
|ak | |bk | 6
ak t
b2k .



k=1

(Indication. Considrer le polynme f(x) =

k=1

n
X

k=1

k=1

(ak + bk x)2 , dvelopper puis ordonner suivant les puissances dcroissantes

k=1

puis utiliser, dans le cas gnral, les connaissances sur le second degr). Retrouver alors le rsultat du no 4.
Exercice no 6. (***)
Soient a, b et c trois rels positifs. Montrer que lun au moins des trois rels a(1 b), b(1 c), c(1 a) est infrieur ou
1
gal .
4
Exercice no 7. (**IT)
1) Montrer que : x R, E(x + 1) = E(x) + 1.

2) Montrer que : (x, y) R2 , E(x) + E(y) 6 E(x + y).

3) Montrer que : (x, y) R2 , E(x) + E(y) + E(x + y) 6 E(2x) + E(2y).

Exercice no 8. (**I)
Tout entier naturel non nul n scrit de manire unique sous la forme
n = a0 + 10a1 + ... + 10p ap ,
o p est un entier naturel et les ai sont des entiers lments de J0, 9K, ap tant non nul. Dterminer p en fonction de n.
Dterminer le nombre de chiffres de n en base 10.
Exercice no 9. (**I)
Soit x un rel. Dterminer

lim

n+

http ://www.maths-france.fr

E(x) + E(2x) + ... + E(nx)


.
n2

c Jean-Louis Rouget, 2014. Tous droits rservs.


Exercice no 10. (**IT)


Soient n un entier naturel et x un rel positif.
1) Combien y a-t-il dentiers naturels entre 1 et n ? entre 1 et x ?
2) Combien y a-t-il dentiers naturels entre 0 et n ? entre 0 et x ?
3) Combien y a-t-il dentiers naturels pairs entre 0 et x ? Combien y a-t-il dentiers naturels impairs entre 0 et x ?
4) Combien y a-t-il de multiples de 3 entre 0 et x ?
5) Combien lquation x + 2y = n, n entier naturel donn et x et y entiers naturels inconnus, a-t-elle de couples solutions ?
6) De combien de faons peut-on payer 10 euros avec des pices de 10 et 20 centimes deuros ?
Exercice no 11. (****)
Montrer que : n N , x R,

n1
X
k=0



k
E x+
= E(nx) (poser la division euclidienne de E(nx) par n).
n

Exercice n 12. (****)


o

Montrer que pour n N, E



 
 n 
8n + 24
1
n+2E
=E
.
3
25
25

Exercice no 13. (****I)


Soit n N .


n

1) Montrer quil existe (an , bn ) (N )2 tel que 2 + 3


= an + bn 3, puis que 3b2n = a2n 1.


n 
n
2) Montrer que E 2 + 3
est un entier impair (penser 2 3 )).
Exercice no 14. (***)
Montrer que n N , x R, E

E(nx)
n

= E(x).

Exercice no 15. (***I)

Montrer que n N, (n > 3 n < n n!).


(commencer par vrifier que pour k = 2, 3, ..., n, on a : (n k + 1)k > n).
Exercice no 16. (***)
Montrer que n N ,

n
X

| cos k| >

k=1

n
(remarquer que si x [0; 1], x2 6 x).
4

Exercice n 17. (**I)


o

Montrer que n N, x R, | sin(nx)| 6 n| sin x|.

http ://www.maths-france.fr

c Jean-Louis Rouget, 2014. Tous droits rservs.


Planche no 9. Valeur absolue, partie entire, ingalits : corrig


Exercice no 1.
Soient x et y deux rels tels que 0 < x 6 y.
x+y
y+y
x+x
6
=m6
= y et donc
1) On a dj x =
2
2
2
x 6 m 6 y.
x+y
yx
(on peut aussi crire : m x =
x=
> 0).
2
2

2) On a ensuite x = x x 6 x y = g 6 y y = y et donc
x 6 g 6 y.
3) m g =

2
x+y
1  2

2  1
=
x 2 xy +
y
y x > 0 et donc,
xy =
2
2
2
x 6 g 6 m 6 y.

4) Daprs 1), la moyenne arithmtique de

1
1
1
1
1
1
1
et
est comprise entre et , ce qui fournit 6 6 , ou encore
x
y
x
y
y
h
x
x 6 h 6 y.

1
1
5) Daprs 3), la moyenne gomtrique des deux rels et
est infrieure ou gale leur moyenne arithmtique. Ceci
x
y
r


1
1
1 1
1
1
1
ou encore 6 et finalement
6
+
fournit
x y
2 x y
g
h


x+y
1 1
1
1

, g = xy et m =
+
.
x 6 h 6 g 6 m 6 y o =
h
2 x y
2
Remarque 1. On a h =
de

1
2xy
, mais cette expression ne permet pas de comprendre que
est la moyenne arithmtique
x+y
h

1
1
et .
x
y

Remarque 2. On peut visualiser lingalit entre moyenne arithmtique et gomtrique.


Si (ABC) est un triangle rectangle en A et A est le pied de la hauteur issue de A, on sait que AA 2 = A B A C. On se
sert de cette remarque pour construire g et la comparer graphiquement m.
On accole deux segments de longueurs respectives x et y. On construit alors un triangle rectangle dhypothnuse ce segment
(de longueur x + y) not [BC], tel que le troisime sommet A ait une projection orthogonale A sur (BC) vrifiant BA = x
et CA = y.
A

http ://www.maths-france.fr

m
y

x
A

x+y

c Jean-Louis Rouget, 2014. Tous droits rservs.


x+y

La moyenne arithmtique de x et y est m =


le rayon du cercle, et la moyenne gomtrique de x et y est g = xy =
2

A B.A C = AA , la hauteur issue de A du triangle (ABC).


Exercice no 2.
Soient n un entier naturel non nul et a un rel positif. Daprs la formule du binme de Newton,
(1 + a)n = 1 + na + ... > 1 + na.
Exercice no 3.
 
 
n
n
n

n
X
uk+1
1
k
k
=
. Pour k J0, nK, posons uk =
puis pour k J0, n 1K, vk =
. Pour
Pour n N , 1 +
n
nk
nk
uk
k=0
k J1, n 1K, on a alors



n
nk
n!k!(n k)!
nk
(n + 1) (k + 1)
1
n+1
1
k+1
vk =  
=
=
= +
=
n
n
n!(k
+
1)!(n

1)!
n(k
+
1)
n(k
+
1)
n
n(k
+ 1)
nk+1
k
1
n+1
6 +
(car k > 1)
n
2n
1
1
1
<
=
2 2n
2
Ainsi, pour k J1, n 1K, uk+1 6

1
uk et donc,
2

1
1 n
1
u1 = k1 = k1 .
2k1
2
n
2
En tenant compte de u0 = 1, on a alors pour n N ,
uk 6

1

n X


n
n
1 n
X
1
1
1
1
2
= 1 + 2 1 n = 3 n1 < 3.
=
uk 6 1 +
=1+
1+
1
n
2k1
2
2
k=0
k=1
1
2
Exercice no 4.
Soient n N et a1 , a2 ,..., an , n rels strictement positifs.
n
X
i=1

! n
X 1
=
ai
aj
j=1

X
16i,j6n

ai X ai
=
+
aj
ai
i=1

X
16i<j6n

ai aj
+
aj ai

=n+

16i<j6n

ai aj
+
aj
ai

(x 1)(x + 1)
1
1
. f est
. f est drivable sur ]0, +[ et pour x > 0, f (x) = 1 2 =
x
x
x2
donc strictement dcroissante sur ]0, 1] et strictement croissante sur [1, +[. f admet ainsi un minimum en 1. Par suite,
Pour x > 0, posons alors f(x) = x +

1
= 2.
1
(Remarque. Lingalit entre moyenne gomtrique et arithmtique permet aussi dobtenir le rsultat :
r
1
1
1
(x + ) > x = 1.)
2
x
x
x > 0, f(x) > f(1) = 1 +

On en dduit alors que


n
X
i=1

ai

n
X
1
>n+
aj
j=1

X
16i<j6n

2=n+2

n2 n
= n2 .
2

Exercice n 5.
o

http ://www.maths-france.fr

c Jean-Louis Rouget, 2014. Tous droits rservs.


Pour x rel, posons f(x) =

n
X

(ak + bk x) . On remarque que pour tout rel x, f(x) > 0. En dveloppant les n carrs, on

k=1

obtient,
f(x) =

n
X

b2k x2 + 2ak bk x + ak =

k=1

1er cas. Si

n
X
k=1

n
X


2

b2k

k=1

x2 + 2

n
X

a k bk

k=1

x+

n
X
k=1

a2k

b2k 6= 0, f est un trinme du second degr de signe constant sur R. Son discriminant rduit est alors ngatif

ou nul. Ceci fournit


0 =

n
X

a k bk

k=1

!2

n
X

k=1

b2k

n
X

a2k

k=1

et donc
v
v

u n
n
n
u
X
X
uX 2
u

2t
t
a k bk
ak
bk .



k=1

2me cas. Si

n
X

k=1

k=1

b2k = 0, alors tous les bk sont nuls et lingalit est immdiate.

k=1

Finalement, dans tous les cas,


v

v
uX
n
n
X
u
X
u n 2

u
2t
t
ak
bk .
a k bk 6



k=1

k=1

k=1

Cette ingalit est encore valable en remplaant les ak et les bk par leurs valeurs absolues, ce qui fournit les ingalits
intermdiaires.
Retrouvons alors lingalit du no 4. Puisque les ak sont strictement positifs, on peut crire :
n
X

ai

i=1

n
X
1
ai
i=1

n
X

i=1

ai

n
X
i=1

1
ai

2!

>

n
X

ai
i=1

1
ai

!2

= n2 .

Exercice n 6.
o

Si lun des rels a, b ou c est strictement plus grand que 1, alors lun au moins des trois rels a(1 b), b(1 c), c(1 a)
1
est ngatif (puisque a, b et c sont positifs) et donc infrieur ou gal .
4
Sinon, les trois rels a, b et c sont dans [0, 1]. Le produit des trois rels a(1 b), b(1 c) et c(1 a) vaut
a(1 a)b(1 b)c(1 c).
2

1
1
1
+ 6 . Par suite,
Mais, pour x [0, 1], x(1 x) est positif et dautre part, x(1 x) = x
2
4
4
1
a(1 a)b(1 b)c(1 c) 6 3 .
4
1
Il est alors impossible que les trois rels a(1 b), b(1 c) et c(1 a) soient strictement plus grand que , leur produit
4
1
tant dans ce cas strictement plus grand que 3 .
4
1
On a montr dans tous les cas que lun au moins des trois rels a(1 b), b(1 c) et c(1 a) est infrieur ou gal .
4
Exercice no 7.
1) Soit x R. Alors, E(x) x < E(x) + 1 puis E(x) + 1 6 x + 1 < (E(x) + 1) + 1. Comme E(x) + 1 Z, on a bien
E(x + 1) = E(x) + 1.

http ://www.maths-france.fr

c Jean-Louis Rouget, 2014. Tous droits rservs.


2) Soient (x, y) R2 . On a E(x) + E(y) 6 x + y. Ainsi, E(x) + E(y) est un entier relatif infrieur ou gal x + y. Comme
E(x + y) est le plus grand entier relatif infrieur ou gal x + y, on a donc E(x) + E(y) 6 E(x + y).
Amliorons. E(x) 6 x < E(x) + 1 et E(y) 6 y < E(y) + 1 fournit E(x) + E(y) 6 x + y < E(x) + E(y) + 2 et donc E(x + y)
vaut, suivant le cas, E(x) + E(y) ou E(x) + E(y) + 1 (et est dans tous les cas suprieur ou gal E(x) + E(y)).
3) Soit (x, y) R2 . Posons k = E(x) et l = E(y).




1
1
et y l, l + , alors x+y [k+l, k+l+1[ et donc E(x+y) = k+l, puis E(x)+E(y)+E(x+y) =
1er cas. Si x k, k +
2
2
k + l + k + l = 2k + 2l. Dautre part, 2x [2k, 2k + 1[ et 2y [2l, 2l + 1[. Par suite, E(2x) + E(2y) = 2k + 2l. Dans ce cas,
E(x) + E(y) + E(x + y) = E(2x) + E(2y).






1
1
1
3
2me cas. Si x k + , k + 1 et y l, l + , alors x + y k + l + , k + l +
et donc E(x + y) = k + l ou
2
2
2
2
k + l + 1,puis E(x) + E(y) + E(x + y) = 2k + 2l ou 2k + 2l + 1. Dautre part, 2x [2k + 1, 2k + 2[ et 2y [2l, 2l + 1[. Par
suite, E(2x) + E(2y) = 2k + 2l + 1. Dans ce cas, E(x) + E(y) + E(x + y) 6 E(2x) + E(2y).




1
1
3me cas. Si x k, k +
et y l + , l + 1 , on a de mme E(x) + E(y) + E(x + y) 6 E(2x) + E(2y).
2
2




1
1
4me cas. Si x k + , k + 1 et y l + , l + 1 , on a E(x) + E(y) + E(x + y) = 2k + 2l + 2 = E(2x) + E(2y).
2
2
Finalement, on a dans tous les cas E(x) + E(y) + E(x + y) 6 E(2x) + E(2y).
Exercice no 8.
p est dtermin par lencadrement : 10p 6 n < 10p+1 qui scrit encore p 6 log10 (n) < p + 1. Par suite,
p = E (log10 (n)) .
On en dduit que
le nombre de chiffres dun entier n en base 10 est E (log10 (n)) + 1.
Exercice no 9.
Soient x R et n N . Pour 1 6 k 6 n, on a
kx 1 < E(kx) 6 kx.
En sommant ces ingalits, on obtient
E(x) + E(2x) + ... + E(nx)
x + 2x + ... + nx
n(n + 1)x
(n + 1)x
6
=
=
,
2
2
2
n
n
2n
2n
et aussi,
E(x) + E(2x) + ... + E(nx)
(x 1) + (2x 1) + ... + (nx 1)
>
=
n2
n2
Finalement, pour tout naturel non nul,

n(n + 1)
xn
(n + 1)x
1
2
=
.
n2
2n
n

(n + 1)x
1
E(x) + E(2x) + ... + E(nx)
(n + 1)x
6
<
.
2n
n
n2
2n
x
Les deux membres extrmes de cet encadrement tendent vers quand n tend vers +. Daprs le thorme des gendarmes,
2
on peut affirmer que
x R,

http ://www.maths-france.fr

x
E(x) + E(2x) + ... + E(nx)
= .
2
n+
n
2
lim

c Jean-Louis Rouget, 2014. Tous droits rservs.


Exercice no 10.
1) Par dfinition dun entier, il y a n entiers entre 1 et n. Ensuite, pour tout entier naturel k, on a
1 6 k 6 x 1 6 k 6 E(x).

Il y a donc E(x) entiers entre 1 et x.

2) Il y a n + 1 entiers entre 0 et n et E(x) + 1 entiers entre 0 et x.


3) Les entiers naturels pairs sont les entiers de la forme 2k, k N. Or,
0 6 2k 6 x 0 6 k 6

x
.
2

x
Le nombre des entiers pairs compris entre 0 et x est encore le nombre des entiers k compris au sens large entre 0 et .
2
x
+ 1 entiers pairs entre 0 et x.
Daprs 2), il y a E
2
Soit k un entier naturel.


1
x1
x1
0 6 2k + 1 6 x 6 k 6
.
06k6E
2
2
2


x1
x+1
Si x [0, 1[, il nay pas dentier impair compris entre 0 et x et si x > 1, il y a E
+ 1 = E(
) entiers impairs
2
2
entre 0 et x.
x
+ 1 multiples de 3 entre 0 et x.
4) Il y a E
3
5) Soient n N et (x, y) N2 . On a

x + 2y = n x = n 2y.

Le nombre de couples (x, y) solutions est encore


 n le nombre dentiers naturels y tels que n 2y N ou encore le nombre
dentiers y tels que0 6 2y 6 n. Il y a donc E
+ 1 couples solutions.
2
6) Si x et y sont respectivement le nombre de pices de 10 centimes deuros et le nombre de pices de 20 centimes deuros,
le nombre cherch est le nombre de
 dentiers naturels solutions de lquation 10x + 20y = 1000 qui scrit encore
 couples
100
+ 1 = 51 faons de payer 10 euros avec des pices de 10 et 20 centimes deuros.
x + 2y = 100. Daprs 5), il y a E
2
Exercice no 11.
Soient x R et n N . Notons p la partie entire de nx. Pour k J0, n 1K, on a alors
p = E(nx) p 6 nx < p + 1

p+k
k
p+k+1
6x+ <
n
n
n

().

La division euclidienne de p par n fournit (q, r) Z2 tel que p = qn + r et 0 6 r 6 n 1. () scrit alors


q+

k+r
k
k+r+1
6x+ <q+
.
n
n
n

Si r = 0, alors pour k J0, n 1K,


k
k
k+1
n1+1
6x+ < q+
6 q+
= q + 1.
n
n
n
n


k
= q et donc
Dans ce cas, pour tout k lment de J0, n 1K, E x +
n
 n1
n1
X 
X
k
E x+
=
q = nq = p = E(nx).
n
q6 q+

k=0

k=0

Si 1 6 r 6 n 1, alors pour 0 6 k 6 n r 1, on a
q6q+
et dans ce cas, E(x +

k
k+r+1
nr1+r+1
k+r
6x+ < q+
6q+
= q + 1,
n
n
n
n

k
) = q et si n r 6 k 6 n 1, on a
n

http ://www.maths-france.fr

c Jean-Louis Rouget, 2014. Tous droits rservs.


nr+r
k+r
k
k+r+1
n1+r+1
n+n
q+1 = q+
6 q+
6x+ <q+
6 q+
<q+
= q + 2,
n
n
n
n
n
n


k
= q + 1. Ainsi,
et dans ce cas, E x +
n
n1
X
k=0


 nr1
n1
n1
n1
X
X
X
X
k
E x+
=
q+
(q + 1) =
q+
1 = nq + ((n 1) (n r 1)) = nq + r = p = E(nx).
n
k=0

k=nr

k=0

k=nr

On a montr que
x R, n N ,

n1
X
k=0



k
= E(nx).
E x+
n

Exercice no 12.
Soit n N. La division euclidienne de n par 25 fournit un quotient entier q et et un reste r lment de J0, 24K tels que
n = 25q + r. On a alors
 






 n 
1
r+2
25q + r + 2 q
r+2
E
n+2E
=E
= E 8q +
= 8q + E
,
3
25
3
3
3
et


8r + 24
.
25




8r + 24
r+2
=E
, 0 6 r 6 24, (), ce qui
Pour montrer lgalit de lnonc, il reste donc vrifier les 25 galits E
3
25
peut dj se vrifier la main .
E

8n + 24
25

=E

8(25q + r) + 24
25

= 8q + E

Diminuons encore le nombre de vrifications. La division euclidienne de r par 3 scrit r = 3k + l avec 0 6 l 6 2. Mais
alors,










l+2
8r + 24
25k k + 8l + 24
k + 8l + 24
r+2
=k+E
et E
=E
=k+E
.
E
3
3
25
25
25
k + 24
k + 8l + 24
24
Si l = 0, k varie de 0 8 et dans ce cas, 0 6
=
6
< 1. Par suite,
25
25
25




l+2
k + 8l + 24
=0=E
.
E
25
3
On a ainsi vrifi () quand r {0, 3, 6, 9, 12, 15, 18, 21, 24}.
Si l = 1 ou l = 2, E(

l+2
) = 1 et dautre part, k varie de 0 7. Dans ce cas,
3
7 + 8 + 24
k + 8l + 24
16 + 24
1=
6
6
<2
25
25
25

et donc
E

k + 8l + 24
25

=1=E

l+2
3

On a ainsi vrifi () pour les autres valeurs de r. Finalement, on a montr que


n N, E

 


 n 
1
8n + 24
n+2E
=E
.
3
25
25

Exercice no 13.
Soit n N .
1) La formule du binme de Newton permet dcrire
http ://www.maths-france.fr

c Jean-Louis Rouget, 2014. Tous droits rservs.


2+



E(n/2) 
E((n1)/2) 
n  
X
X
n X
2k+1 n2k1
k
n
n 2k n2k
n
3
3 2nk =
3 2
+
3
2
=
k
2k
2k + 1
k=0

k=0

k=0

k=0



E((n1)/2) 
X
n
n k n2k
3k 2n2k1 .
3 2
+ 3
2k + 1
2k

E(n/2) 

k=0

E(n/2) 



E((n1)/2) 
X
n k n2k
n
3 2
et bn =
3k 2n2k1 , an et bn sont des entiers tels que
2k
2k + 1
k=0
k=0


n
= an + bn 3. En remplaant 3 par 3 (cest--dire par un calcul conjugu), on a aussi 2 3
=

Ainsi, en posant an =

n
3)

an bn 3. Mais alors,

(2 +


n 
n
 

2 3
= (4 3)n = 1.
a2n 3b2n = an + bn 3 an bn 3 = 2 + 3
2) On note que (2 +

3) + (2 3)n = (an + bn 3) + (an bn 3) = 2an . Mais,


0 < (2

n
3) < 1.

Par suite,


ou encore

2+


n 
n 
n
n
3
< 2 + 3 + 2 3 = 2an < 2 + 3 + 1,


n
2an 1 < 2 + 3 < 2an .


n 
n 
On en dduit que E 2 + 3
= 2an 1 et donc queE 2 + 3
est un entier impair.
Exercice no 14.

Soient n N et x R.
E(x) 6 x < E(x) + 1 nE(x) 6 nx < nE(x) + n

nE(x) 6 E(nx) < nE(x) + n (car nE(x) et nE(x) + n sont des entiers)

E(nx)
E(x) 6
< E(x) + 1
n


E(nx)
E
= E(x).
n
Exercice no 15.
Soit n N \ {0, 1, 2}.
n!2 =

n
Y

(n + 1 k)

k=1

n
Y
k=1

k=

n
Y

k(n + 1 k).

k=1



n+1
et strictement dcroissante sur
Maintenant, la fonction x
7
x(n + 1 x) est strictement croissante sur 0,
2


n+1
, n + 1 . Puisque f(1) = f(n) = n, on en dduit que pour x [2, n 1], f(x) > n. Puisque n > 3, on a n 1 > 2
2
et on peut crire
n!2 = n2

n1
Y

k(n + 1 k) > n2

k=2

n1
Y

n = nn ,

k=2

et donc,

n
n! = (n!2 )1/(2n) > (nn )1/2n = n.
http ://www.maths-france.fr

c Jean-Louis Rouget, 2014. Tous droits rservs.


Exercice no 16.
Soit n N . Puisque, pour tout entier k, | cos k | [0, 1], on a alors
!


n
n
X
X
1
n 1
n 1
1 e2in
2ik
| cos k | >
cos k =
= + Re e2i
(1 + cos(2k)) = + Re
e
2
2
2
2
2
1 e2i
k=1
k=1
k=1
k=1


n cos(n + 1) sin n
sin n
n
1
n 1
= +
>
.
= + Re ei(n1+2)
2
2
sin 1
2
2 sin 1
2
2 sin 1
n
X

n
X

1
1
3
n
= 0, 594.... Par suite, pour n > 3,
6 0, 75 = 6 , et donc
2 sin 1
2 sin 1
4
4
1
n n
n
n

> = .
2
2 sin 1
2
4
4
1
2
Enfin, si n = 1, | cos 1| = 0, 5... > 0, 25 = et si n = 2, | cos 1| + | cos 2| = 0.9... > 0, 5 = . Finalement,
4
4

Maintenant,

n N ,

n
X

| cos k| >

k=1

n
.
4

Exercice no 17.
Soit x R. Montrons par rcurrence que : n N, | sin(nx)| 6 n| sin x|.
Lingalit est vraie quand n = 0.
Soit n > 0. Supposons que | sin(nx)| 6 n| sin x| et montrons que | sin((n + 1)x)| (n + 1)| sin x|. Alors,
| sin(n + 1)x| = | sin nx cos x + cos nx sin x| 6 | sin nx| | cos x| + | cos nx| | sin x|
6 | sin nx| + | sin x|
n| sin x| + | sin x| (par hypothse de rcurrence)
= (n + 1)| sin x|.
On a montr par rcurrence que
n N, x R, | sin(nx)| n| sin x|.

http ://www.maths-france.fr

c Jean-Louis Rouget, 2014. Tous droits rservs.


Planche no 10. Exponentielles et logarithmes


* trs facile ** facile *** difficult moyenne **** difficile
I : Incontournable T : pour travailler et mmoriser le cours
Exercice no 1 (**)
Trouver la plus grande valeur de

n
n, n N .

Exercice no 2 (**I)

n
1
Dterminer lim
1+
.
n+
n
Exercice no 3 (**I)
1) Etudier brivement la fonction x 7

ln x
et tracer son graphe.
x

2) Trouver tous les couples (a, b) dentiers naturels non nuls et distincts vrifiant ab = ba .
Exercice no 4
Rsoudre dans R les quations ou inquations suivantes :
1) (**) ln |x + 1| ln |2x + 1| 6 ln 2
3) (***) lnx (10) + 2ln10x (10) + 3ln100x (10) = 0

x
2) (**) x x = x
1
1
4) (**) 22x 3x 2 = 3x+ 2 22x1

Exercice no 5 (***)
(xx )x
x .
x+ x(x )
Exercice no 6

Trouver lim

Construire le graphe des fonctions suivantes :


x

1
( tudier sur ]0, +[).
1) (***I) f1 (x) = 1 +
x


2) (**) f2 (x) = log2 1 log 1 x2 5x + 6 .
2

Exercice n 7 (**)
o

Montrer que x ]0, 1[, xx (1 x)1x >

http ://www.maths-france.fr

1
.
2

c Jean-Louis Rouget, 2014. Tous droits rservs.


Planche no 10. Exponentielles et logarithmes : corrig


Exercice no 1
Pour n N , posons un =
nul n, on a un = f(n).

n
n puis, pour x rel strictement positif, f(x) = x1/x de sorte que pour tout entier naturel non

f est dfinie sur ]0, +[ et pour x > 0, f(x) = e

ln x
x

. f est drivable sur ]0, +[ et pour x > 0,

1 ln x ln x
e x .
x2
Pour x > 0, f (x) est du signe de 1 ln x et donc f est strictement positive sur ]0, e[ et strictement ngative sur ]e, +[.
Par suite, f est strictement croissante sur ]0, e] et strictement dcroissante sur [e, +[. En particulier, pour n > 3,

3
un = f(n) 6 f(3) = u3 = 3.

2, 3 3 . Enfin, ( 2)6 = 8 < 9 = ( 3 3)6 et donc


Comme u2 = 2 > 1 = u1 , on a donc Max {un , n N } = Max

2 < 3 3 (par stricte croissance de la fonction x 7 x6 sur [0, +[). Finalement,


f (x) =

Max



n
n, n N = 3 3 = 1, 44..

Exercice no 2
1
Pour tout entier naturel non nul n, 1 +
existe et est strictement positif. Donc, pour tout entier naturel non nul n,
n

n
1
1+
existe. De plus, pour tout entier naturel non nul n,
n


1
n

ln 1 + n
1
1
.
= en ln(1+ n ) = exp
1+

1
n
n
lim

n+

1
= 0 et donc
n

1
ln 1 +
n
lim
1
n+
n
puis

lim

n+

1+

1
n

n

ln (1 + x)
= 1,
x0
x

= lim

= e1 = e.
n

1
1+
= e.
n+
n
lim

Exercice no 3
ln x
1 ln x
. f est donc
. f est dfinie et drivable sur ]0, +[ et, pour x > 0, f (x) =
x
x2
strictement croissante sur ]0, e] et strictement dcroissante sur [e, +[. Le graphe de f sen dduit facilement :

1) Pour x > 0, posons f(x) =

http ://www.maths-france.fr

c Jean-Louis Rouget, 2014. Tous droits rservs.


1
1
e
1

4
2) Soient a et b deux entiers naturels non nuls tels que a < b.
ln b
ln a
=
f(a) = f(b).
a
b
Si a > 3, puisque f est strictement dcroissante sur [e, +[, on a alors f(a) > f(b) et en particulier, f(a) 6= f(b). a nest
alors pas solution.
ab = ba ln(ab ) = ln(ba ) b ln a = a ln b

a = 1 nest videmment pas solution. Par exemple, ab = ba 1b = b1 b = 1 = a ce qui est exclu.

Donc, ncessairement a = 2 et b est un entier suprieur ou gal 3, et donc e, vrifiant f(b) = f(2). Comme f
est strictement dcroissante sur [e, +[, lquation f(b) = f(2) a au plus une solution dans [e, +[. Enfin, comme
24 = 16 = 42 , on a montr que :
il existe un et un seul couple (a, b) dentiers naturels non nuls tel que a < b et ab = ba , savoir (2, 4).
Exercice no 4
1) Soit x R,




x+1


6 ln 2 x + 1 6 2 et x 6= 1
ln |x + 1| ln |2x + 1| 6 ln 2 ln


2x + 1
2x + 1
x+1
x+1
x+1
2 6
6 2 et x 6= 1
+ 2 > 0 et
2 6 0 et x 6= 1
2x + 1
2x + 1
2x + 1
5x + 3
3x 1

> 0 et
6 0 et x 6= 1
2x + 1
2x + 1


 
 



3
1
1
1
x , , +
et
, , +
et x 6= 1
5
2
2
3
 


1
3
x ] , 1[ 1, , + .
5
3
 

1
3
S =] , 1[ 1, , + .
5
3


2) Soit x > 0.
x

http ://www.maths-france.fr


x

x
x x ln x = x ln x ln x
x
=0
2


ln x x 2 x = 0 x = 1 ou x = 4.
2

c Jean-Louis Rouget, 2014. Tous droits rservs.


S = {1, 4}.

3) Pour x ]0, +[\


1 1
, ,1 ,
100 10
ln(10)
ln(10)
ln(10)
+2
+3
=0
ln x
ln(10x)
ln(100x)
(ln x + ln(10))(ln x + 2 ln(10)) + 2 ln x(ln x + 2 ln(10)) + 3 ln x(ln x + ln(10))
=0

ln x(ln x + ln(10))(ln x + 2 ln(10))

lnx (10) + 2ln10x (10) + 3ln100x (10) = 0

6 ln2 x + 10 ln(10) ln x + 2 ln2 (10) = 0


q
q

5 ln(10) + 13 ln2 (10) 5 ln(10) 13 ln2 (10)


,
ln x

6
6

x e((5 13)/6) ln(10) , e((5+ 13)/6) ln(10)


x 10(5 13)/6 , 10(5+ 13)/6 .

S = 10(5 13)/6 , 10(5+ 13)/6 .

4) Soit x R.
1

22x 3x 2 = 3x+ 2 22x1 22x + 22x1 = 3x+ 2 + 3x 2


1

22x1 (2 + 1) = 3x 2 (3 + 1) 3 22x1 = 4 3x 2


3
3
2x3
x 2
(2x 3) ln 2 = x
ln 3
2
=3
2
3
3 ln 2 ln 3
3
2
x=
x= .
2 ln 2 ln 3
2
S =


3
.
2

Exercice no 5
x

Pour x > 0, (xx )x = ex ln(x

= ex

ln x

et x(x

= ex

x > 0,

ln x

. Par suite,


(xx )x
2
x
.
x ) = exp ln x x x
(x
x


Or, x2 xx = xx 1 x2x = ex ln x (1e(2x) ln x ). Quand x tend vers +, (2x) ln x tend vers . Donc, 1e(2x) ln x
(xx )x
tend vers 1 puis x2 xx tend vers . Mais alors, ln x(x2 xx ) tend vers , puis (xx ) = exp(ln x(x2 xx )) tend
x
vers 0.
x

(xx )
= 0.
x
x+ x(x )
lim

Exercice no 6
On notera Ci le graphe de fi .
1
1
1) Soit x > 0. x nest pas nul donc existe puis 1 + > 0 et f1 (x) existe.
x
x




1
1
= x ln x + x ln(1 + x). Par suite, x ln 1 +
tend vers 0 quand x tend vers 0
Etude en 0. Pour x > 0, x ln 1 +
x
x



1
par valeurs suprieures et donc f1 (x) = exp x ln 1 +
tend vers 1. Ainsi, limx0 f1 (x) = 1.
x
x>0
http ://www.maths-france.fr

c Jean-Louis Rouget, 2014. Tous droits rservs.


Posons encore f1 (0) = 1 et tudions la drivabilit de f1 en 0. Pour x > 0,








 exp x ln 1 +
1
1
f1 (x) f1 (0)

=
exp x ln 1 +
1 =
x0
x
x
x ln 1 +


1

1 
1
x

.
ln 1 +
1
x
x



1
Or, x ln 1 +
tend vers 0 quand x tend vers 0 (daprs plus haut) et donc
x



1
1
exp x ln 1 +
ey 1
x


= lim
lim
= 1.
y0
x0
1
y
x>0
x ln 1 +
x


1
tend vers + quand x tend vers 0 par valeurs suprieures. Finalement,
Dautre part, ln 1 +
x
f1 (x) f1 (0)
lim
= +.
x0
x0
x>0

Ainsi, f1 nest pas drivable en 0 mais C1 admet laxe des ordonnes pour tangente en (0, f1 (0)) = (0, 1).


1
 ln 1 +



ln(1 + y)
1
1
x
=
= lim
= 1. Par suite,
Etude en +. Pour x > 0, x ln 1 +
et donc lim x ln 1 +
1
x+
y0
x
x
y
x
lim f1 (x) = e.
x+



1
. Par suite, pour x > 0,
Etude des variations de f1 . Pour x > 0, f1 (x) > 0 puis ln (f1 (x)) = x ln 1 +
x



1
 x 2


1
x
= f1 (x)g(x),
f1 (x) = f1 (x) ln(f1 ) (x) = f1 (x)
ln 1 + x +
1
1+
x


1
1
o g(x) = ln 1 +

. Sur ]0, +[, f1 est du signe de g.


x
1+x
Pour dterminer le signe de g, tudions dabord les variations de g sur ]0, +[. g est drivable sur ]0, +[ et pour x > 0,
1
1
1
1
1
2
x
=
=
< 0.
+
+
g (x) =
2
2
1
(x + 1)
x(x + 1) (x + 1)
x(x + 1)2
1+
x
g est donc strictement dcroissante sur ]0, +[ et, puisque lim g(x) = 0, g est strictement positive sur ]0, +[. Il en

x+

est de mme de f1 . f1 est strictement croissante sur ]0, +[.


On en dduit C1 .
e

http ://www.maths-france.fr

c Jean-Louis Rouget, 2014. Tous droits rservs.


2) Domaine de dfinition de f2 . Soit x R.


f2 (x) existe x2 5x + 6 > 0 et 1 log 1 (x2 5x + 6) > 0 x2 5x + 6 > 0 et
2

ln(x2 5x + 6)
<1
ln 21

1
1
x2 5x + 6 > 0 et ln(x2 5x + 6) > ln x2 5x + 6 >
2
#
" 2

" #
5+ 3
5 3
11

> 0 x ,
, +
x2 5x +
2
2
2

"
" #

5 3
5+ 3
f2 est dfinie sur D = ,

, + .
2
2
#



5
et strictement croissante
Variations de f2 . La fonction x 7 x 5x + 6 est strictement dcroissante sur ,
2



5
5+ 3
5
5 3
5
sur
, + . Comme
>
et que
< , la fonction x 7 x2 5x + 6 est strictement dcroissante
2
2
2
2
2
#
"
"
#

5 3
5+ 3
sur ,
et strictement croissante sur
, + , valeurs dans ]0, +[, intervalle sur lequel la fonction
2
2
2

ln(x2 5x + 6)
a le mme sens de variations et
logarithme nprien est strictement croissante. La fonction x 7 1 +
ln 2
"
#
"

#
5+ 3
5 3
et strictement croissante sur
finalement f1 est strictement dcroissante sur ,
, + .
2
2

2


5
5
5
Axe de symtrie Soit x R. x D x D et de plus,
x 5
x + 6 = x2 5x + 6. Par suite,
2
2
2


5
x = f1 (x).
x D, f1
2
5
pour axe de symtrie.
2
Le calcul des limites est facile et on en dduit C2 .

C1 admet donc la droite dquation x =

3
2
1

1
2
Exercice no 7
Pour tout x ]0, 1[, x et 1 x sont strictement positifs et donc xx (1 x)1x existe.
Par stricte croissance de la fonction ln sur ]0, +[, il est quivalent de dmontrer que
x ]0, 1[, x ln(x) + (1 x) ln(1 x) > ln(2).
Pour x ]0, 1[, posons f(x) = x ln(x) + (1 x) ln(1 x). f est drivable sur ]0, 1[ et pour tout x de ]0, 1[,
f (x) = ln(x) + x

1
1
ln(1 x) + (1 x)
= ln(x) ln(1 x).
x
1x

Pour x ]0, 1[,


http ://www.maths-france.fr

c Jean-Louis Rouget, 2014. Tous droits rservs.


1
f (x) > 0 ln(x) > ln(1 x) x > 1 x x > .
2




1
1
Ainsi, la fonction f est strictement positive sur
, 1 et ngative sur 0, . La fonction f admet donc un minimum en
2
2
1
et ce minimum est gal
2
 

  
1
1
1
1
+ 1
ln 1
= ln(2).
ln
2
2
2
2
Ceci montre que x ]0, 1[, x ln(x) + (1 x) ln(1 x) > ln(2) et donc que
x ]0, 1[, xx (1 x)1x >

http ://www.maths-france.fr

1
.
2

c Jean-Louis Rouget, 2014. Tous droits rservs.


Planche no 11. Fonctions puissances


* trs facile ** facile *** difficult moyenne **** difficile
I : Incontournable T : pour travailler et mmoriser le cours
Exercice no 1 (**T)
Dterminer le domaine de dfinition des fonctions suivantes puis tudier leur drivabilit :

1) f1 : x 7 x2 + 1

2) f2 : x 7 3 x3 + 1.

2) f3 : x 7 x3 x4 .
Exercice no 2 (*T)
Donner la drive des fonctions suivantes :
1)

x2 + 1
x

4)
x2 + 1

2)

3
x3 + 1

5)

x1
.
x+1

1
3) 
3
4
x2 + x + 1

Exercice no 3 (**T)
Dterminer les limites suivantes :

p

p
x2 + x + 1 + x et lim
x2 + x + 1 + x
1) lim
x
x+

p
3
x3 + 1 x
2) lim
x+

3) lim

x1

2x + 7 3
2x + 5 1
et lim
.
x2
x1
3x + 15 3

Exercice no 4 (***)
r

x3
. On tudiera en particulier la drivabilit de f en 0 gauche. Dautre
x1
1
part, on montrera que la droite dquation y = x + est asymptote la courbe de f en + et que la droite dquation
2
1
y = x est asymptote la courbe de f en .
2
Exercice no 5 (**)

Etudier le signe de x2 + 1 x et x2 + 1 + x.
Etude complte de la fonction f : x 7

http ://www.maths-france.fr

c Jean-Louis Rouget, 2014. Tous droits rservs.


Planche no 11. Fonctions puissances : corrig


Exercice no 1

u1 est dfinie sur R.

: x
7 x2 + 1 est drivable sur R et strictement positive sur R. Donc, la fonction f1 = u1 est drivable

1) La fonction u1 : x 7 x2 + 1 est dfinie sur R et positive sur R. Donc, la fonction f1 =

La fonction u1
sur R.

2) La fonction f2 est dfinie sur R.


La fonction u2 : x 7 x3 + 1 est drivable sur R et ne sannule pas sur R \ {1}. Donc, la fonction f2 =
sur R \ {1}.

3
u2 est drivable

Etudions la drivabilit de la fonction f2 en 1. Pour x 6= 1,


p

3
3
(x + 1) (x2 x + 1)
f2 (x) f2 (1)
x2 x + 1
=
=
2 .
3
x (1)
x+1
x+1

Par suite, lim

x1

f2 (x) f2 (1)
= +. La fonction f2 nest pas drivable en 1.
x (1)

En rsum, la fonction f2 est dfinie sur R, drivable sur R \ {1} et pas drivable en 1.
3) Soit x R. f3 (x) existe x3 x4 > 0. Or, pour tout rel x,



sgn x3 x4 = sgn x3 (1 x) = sgn (x(1 x)) .

Donc, pour tout rel x, f(x) existe si et seulement si x [0, 1]. Le domaine de dfinition de la fonction f3 est [0, 1].

La fonction u3 : x 7 x3 x4 est drivable sur ]0, 1[ et strictement positive sur ]0, 1[. Donc, la fonction f3 = u3 est
drivable sur ]0, 1[.
Drivabilit en 0 ( droite). Pour x ]0, 1],

p
x3 (1 x)
x3 1 x
=
(car x3 > 0 et 1 x > 0)
x
x

x3/2
=
1 x = x 1 x.
x

f3 (x) f3 (0)
=
x0

f3 (x) f3 (0)
= 0. La fonction f3 est drivable en 0.
x0
x0
Drivabilit en 1 ( gauche). Pour x [0, 1[,

Donc, lim


p
x3 (1 x)
x3 1 x
=
x1
1x

3
x
= p
.
(1 x)

f3 (x) f3 (1)
=
x1

f3 (x) f3 (1)
= . La fonction f3 nest pas drivable en 1.
x1
En rsum, la fonction f3 est dfinie sur [0, 1], drivable sur [0, 1[ et pas drivable en 1.

Donc, lim

x0

Exercice no 2
1) Pour tout rel x, f1 (x) =

x2 + 1 puis
2x
x
f1 (x) =
=
.
2 x2 + 1
x2 + 1

1/3
puis, pour tout rel x diffrent de 1,
2) Pour tout rel x, f2 (x) = x3 + 1
f2 (x) =

http ://www.maths-france.fr

2/3
x
1
(3x2 ) x3 + 1
= 
2 .
3
3
x3 + 1
1

c Jean-Louis Rouget, 2014. Tous droits rservs.


3/4
3) Pour tout rel x, f3 (x) = x2 + x + 1
puis pour tout rel x

7/4
3
3(2x + 1)
f3 (x) = (2x + 1) x2 + x + 1
= 
7 .
4
4 4 x2 + x + 1

1/2
4) Pour tout rel x, f4 (x) = x x2 + 1
puis, pour tout rel x
f4 (x)

 
3/2
1/2
1
x2
1

(2x) x2 + 1
=

=1 x +1
+x
2
x2 + 1 (x2 + 1) x2 + 1
1
(x2 + 1) x2

=
.
=
2
2
2
(x + 1) x + 1
(x + 1) x2 + 1
2

5) f5 est dfinie sur ] , 1[[1, +[, drivable sur ] , 1[]1, +[ et pour x ] , 1[]1, +[,

si x > 1

3/2
(x + 1) (x 1)1/2
2
1
r
f5 (x) =
=
.

1
(x + 1)2
x1


si x < 1
2
(x + 1)3/2 (x 1)1/2
x+1
Exercice no 3

p
p

x2 + x + 1 = lim
X = + et lim x = +. En additionnant, on obtient lim
x2 + x + 1 + x =
1) lim
x+
x+
x+
X+
+.
Pour tout x < 0,
 


x2 + x + 1 + x
x2 + x + 1 x
x2 + x + 1 x2
x+1

=
=
2
2
2
x +x+1x
x +x+1x
x +x+1x
x+1
x+1
r
r
=
=

1
1
1
1
x2 1 + + 2 x
x 1 + + 2 x
x x
x x


1
1
x 1+
1+
x
x
! = r
=
.
r
1
1
1
1
1+ + 2 1
x 1+ + 2 1
x x
x x

p
x2 + x + 1 + x =


p
1
x2 + x + 1 + x = .
x
2

On en dduit que lim


2) Pour x > 0,

p
3
x3 + 1 x =



 
2

3
3
x3 + 1 x
x3 + 1 + x 3 x3 + 1 + x2
2


3
x3 + 1 + x 3 x3 + 1 + x2

1
= 
.
2

3
x3 + 1 + x 3 x3 + 1 + x2

3
x3 + 1 x3
= 
2

3
x3 + 1 + x 3 x3 + 1 + x2
3

Le dnominateur de cette fraction tend vers + quand x tend vers + et donc lim

x+

p

3
x3 + 1 x = 0.

7
3) 1 re solution. Pour x > et x 6= 1,
2




2x + 7 3
2x + 7 + 3
2x 2
2
2x + 7 3

=
=
=
x1
(x 1) 2x + 7 + 3
(x 1) 2x + 7 + 3
2x + 7 + 3

2x + 7 3
1
et donc lim
= .
x1
x1
3
http ://www.maths-france.fr

c Jean-Louis Rouget, 2014. Tous droits rservs.



7
7
2 me solution. Pour x > , posons f(x) = 2x + 7. Pour x > et x 6= 1,
2
2

2x + 7 3
f(x) f(1)
=
.
x1
x1

2
1
2x + 7 3
2
= f (1) =
f est drivable en 1 et donc lim
= = .
x1
x1
6
3
2 21+7
1 re solution. Pour x >

5
et x 6= 2,
2






2x + 5 1
2x + 5 + 1
3x + 15 + 3
(2x + 4) 3x + 15 + 3
2x + 5 1


=

=
3x + 15 3
3x + 15 3
3x + 15 + 3
2x + 5 + 1
(3x + 6) 2x + 5 + 1


2 3x + 15 + 3


=
3 2x + 5 + 1

2x + 5 1
26
= 2.
et donc lim
=
x2
32
3x + 15 3

5
5
2 me solution. Pour x > , posons f(x) = 2x + 5 et g(x) = 3x + 15. Pour x > et x 6= 2,
2
2

2x + 5 1
2x + 5 1
f(x) f(2)
x+2
x (2)

=
=

.
x+2
x (2)
g(x) g(2)
3x + 15 3
3x + 15 3

2
p

2 2 (2) + 5 1
2x + 5 1
f (2)
=
= 2.
=
Donc lim
3
x2
g
(2)
1/2
3x + 15 3
p
2 3 (2) + 15
Exercice no 4

Domaine de dfinition. Soit x un rel. f(x) existe si et seulement si x 6= 1 et


Pour x 6= 1,

x3
> 0.
x1

x3
x3
a le mme signe que x(x 1). Donc pour x 6= 1,
> 0 x ] , 0]]1, +[.
x1
x1
f est dfinie sur D =] , 0]]1, +[.

Drivabilit en 0 gauche. Soit x < 0.

r
r
r
1 x2 x
x3
x2
x
x
=
=
=
.
x1
x
x1
x
x1
x1
r
f(x) f(0)
x
tend vers 0 et donc lim
= 0.
Quand x tend vers 0 par valeurs infrieures,
x0
x1
x0
1
f(x) f(0)
=
x0
x

f est donc drivable ( gauche) en 0 et f (0) = 0.


Etude en +. Pour x > 1,

lim

x+

v
u
x
x3
u
= r
f(x) = u 
.
t
1
1
x 1
1
x
x

1
= 1 et donc lim f(x) = +. Ensuite, pour x > 1,
x+
x

http ://www.maths-france.fr

c Jean-Louis Rouget, 2014. Tous droits rservs.


r 1
r 1
1
+ 1

1
1
1

x
1
x
x
r
f(x) x = r
x = x
1
=x

1
1
1
r
+1
1
1
1
x
x
1
x
1/x
1
1
1
1
1
1
1
1
x
x
=x
=
.
=x
1
1
1
1
r
r
+1
+1
+1
1 r
x
1
1
1
1
1
1
x
x
x


1
1
tend vers 0 quand x tend vers +.
Lexpression prcdente tend vers quand x tend vers + et donc f(x) x +
2
2
1
On en dduit que la droite dquation y = x + est asymptote au graphe de f en +.
2
Etude en . Pour x < 0,
v

u
x
x3
x2
u
 =r
f(x) = u 
= r
.
t
1
1
1
x 1
1
1
x
x
x

Donc lim f(x) = +. Ensuite, pour x < 0,


x

1
.
1
1
1
r
+
1
1

1
x
1
x
1
x


1
1
tend vers 0 quand x tend vers
Lexpression prcdente tend vers quand x tend vers + et donc f(x) x
2
2
1
. On en dduit que la droite dquation y = x est asymptote au graphe de f en .
2
Drive et variations. Pour x ] , 0[]1, +[,

 3 

1
1
x
|x|3
1
= ln
= (3 ln(|x|) ln(|x 1|))
ln(f(x)) = ln
2
x1
2
|x 1|
2
f(x) + x = r

+x=

puis
f (x)
1
=
f(x)
2

3
1

x x1

4x 3
.
2x(x 1)

Pour x D, f(x) > 0 et x(x 1) > 0, donc pour tout x de D, f(x) est du signe de 4x 3. On en dduit le tableau de
variations de f.
x

f (x)

4
3

+
+
+

f
8
3

0
Graphe.

http ://www.maths-france.fr

c Jean-Louis Rouget, 2014. Tous droits rservs.


6
5
4
3
2
1

http ://www.maths-france.fr

c Jean-Louis Rouget, 2014. Tous droits rservs.


Planche no 12. Trigonomtrie circulaire


* trs facile ** facile *** difficult moyenne **** difficile
I : Incontournable T : pour travailler et mmoriser le cours
Exercice no 1 (*IT)
Rsoudre dans R puis dans [0, 2] les quations suivantes :
1) sin x = 0
6) cos x = 0

2) sin x = 1
7) tan x = 0

3) sin x = 1
8) tan x = 1.

4) cos x = 1

5) cos x = 1

Exercice no 2 (*IT)
Rsoudre dans R puis dans [0, 2] les quations suivantes :
1
1) sin x =
2

1
2) sin x =
2

1
3) tan x = 1 4) tan x =
3

5) cos x =

3
2

1
6) cos x = .
2

Exercice no 3 (**IT)
Rsoudre dans R puis dans I les quations suivantes :
1
, I = [0, 2]
2
4) cos(2x) = cos2 x, I = [0, 2]
7) | cos(nx)| = 1
10) sin x = tan x, I = [0, 2]

x

1
= , I = [0, 4]
2
5) 2 cos2 x 3 cos x + 1 = 0, I = [0, 2]
8) sin(nx) = 0
11) sin(2x) + sin x = 0, I = [0, 2]

1) sin(2x) =

2) sin

3) tan(5x) = 1, I = [0, ]
6) cos(nx) = 0 (n N )
9) | sin(nx)| = 1
12) 12 cos2 x 8 sin2 x = 2, I = [, ].

Exercice no 4 (**IT)
Rsoudre dans I les inquations suivantes :
1) cos x 6

1
, I = [, ]
2

4) cos2 x > cos(2x), I = [, ]

1
2) sin x > , I = R
2
1
5) cos2 x 6 , I = [0, 2]
2

x
3) cos x > cos , I = [0, 2]
2
x
x
6) cos 6 sin , I = [0, 2].
3
3

Exercice no 5 (*I)

Calculer cos et sin .


8
8
o
Exercice n 6 (*I)

Calculer cos
et sin .
12
12
Exercice no 7 (***)
X
Montrer que
cos(a1 a2 ... an ) = 2n cos a1 cos a2 ... cos an (la somme comporte 2n termes).
Exercice no 8 (***I)
n
a
Y
1) Calculer
cos k pour a lment donn de ]0, 2[ (penser sin(2x) = 2 sin x cos x).
2
k=1

2) Pour a ]0, [, dterminer

lim

n+

n
X

k=1


 a 
ln cos k .
2

Exercice n 9 (**)
o

Rsoudre dans R lquation 24 cos

x+1

+ 16 24 sin

x3

= 20.

Exercice n 10 (***)
o

1
1
Soit a un rel distinct de et .
3
3
1) Calculer tan(3) en fonction de tan .
http ://www.maths-france.fr

c Jean-Louis Rouget, 2014. Tous droits rservs.


2) Rsoudre dans R lquation :


3a a3
3x x3
=
.
1 3x2
1 3a2
On trouvera deux mthodes, lune algbrique et lautre utilisant la formule de trigonomtrie tablie en 1).
Exercice no 11 (***) Combien lquation
tan x + tan(2x) + tan(3x) + tan(4x) = 0,
possde-t-elle de solutions dans [0, ] ?
Exercice no 12 (**I)
Calculer une primitive de chacune des fonctions suivantes :
1) x 7 cos2 x
6) x
7 cos x sin6 x

2) x 7 cos4 x
7) x
7 cos5 x sin2 x

3) x 7 sin4 x
8) x
7 cos3 x.

4) x 7 cos2 x sin2 x

5) x 7 sin6 x

Exercice no 13 (***I)
Z /3
Z /3
Calculer I =
cos4 x sin6 x dx et J =
cos4 x sin7 x dx.
/6

/6

Exercice n 14 (**)
o

Dmontrer les identits suivantes, en prcisant chaque fois leur domaine de validit :




1 cos x
2
x
2
1)
+ sin x + sin x +
=0
= tan
2) sin x
sin x
2
3
3




2
1
2
+ x + tan
x =
4)
tan x =
.
3) tan
4
4
cos(2x)
tan x
tan(2x)
Exercice no 15 (***)
Soit k un rel distinct de 1 et de 1.
1) Etudier les variations de fk : x 7
2) Calculer

sin x
1 2k cos x + k2

fk (x) dx.

Exercice no 16 (***I)
Calculer les sommes suivantes :
n
n
X
X
1)
cos(kx) et
sin(kx), (x R et n N donns).
k=0

2)

k=0

n
X

cos2 (kx) et

n
X

Ckn cos(kx) et

k=0

3)

n
X

k=0

k=0

sin2 (kx), (x R et n N donns).

n
X

k=0

Ckn sin(kx), (x R et n N donns).

Exercice n 17 (***)

cos a + cos b + cos c = 0
Rsoudre le systme
o a, b et c sont trois rels.
sin a + sin b + sin c = 0
o

Exercice no 18 (**)

3
5
7
3
+ cos4
+ cos4
+ cos4
= .
8
8
8
8
2
Exercice no 19 (***)

Montrer que cos4

1) Rsoudre dans R lquation cos(3x) = sin(2x).


2) En dduire les valeurs de sin x et cos x pour x lment de
http ://www.maths-france.fr


3
.
, ,
10 5 10
c Jean-Louis Rouget, 2014. Tous droits rservs.

Exercice no 20 (***IT)
Etude complte et graphe des fonctions suivantes :
1) f1 : x 7 2 cos(x) + cos(2x)

3) f3 : x 7 |tan(x)| + cos(x)

http ://www.maths-france.fr

sin(x)
2 cos(x)
2 sin(x) + 1
: x 7
2 cos(x) + 1

2) f2 : x 7

2) f4

c Jean-Louis Rouget, 2014. Tous droits rservs.


Planche no 12. Trigonomtrie circulaire : corrig


Exercice no 1
1) sin x = 0 x Z. De plus, S[0,2] = {0, , 2}.

.
2) sin x = 1 x + 2Z. De plus, S[0,2] =
2
2

3

.
3) sin x = 1 x + 2Z. De plus, S[0,2] =
2
2
4) cos x = 1 x 2Z. De plus, S[0,2] = {0, 2}.

5) cos x = 1 x + 2Z. De plus, S[0,2] = {}.



3
6) cos x = 0 x + Z. De plus, S[0,2] =
.
,
2
2 2

7) tan x = 0 x Z. De plus, S[0,2] = {0, , 2}.




5

.
,
8) tan x = 1 x + Z. De plus, S[0,2] =
4
4 4

Exercice no 2




  5
1
5
1) sin x = x
.
+ 2Z
+ 2Z . De plus, S[0,2] =
,
2
6
6
6 6



  3

1
5 7
2) sin x = x + 2Z
.
+ 2Z . De plus, S[0,2] =
,
4
4
4 4
2



3 7
3

et S[0,2] =
.
,
3) tan x = 1 x + Z. De plus, S[0,] =
4
4
4 4


1
7

et S[0,2] =
.
4) tan x = x + Z. De plus, S[0,] =
,
6
6
6 6
3




 

11
3
.
x + 2Z
+ 2Z . De plus, S[0,2] =
,
5) cos x =
2
6
6
6 6

 



1
3
3 5
3
.
6) cos x = x
+ 2Z
+ 2Z . De plus, S[0,2] =
,
4
4
4 4
2
Exercice no 3





  5

  5
1
5 13 17
1) sin(2x) = 2x
.
+ 2Z
+ 2Z x
+ Z
+ Z . De plus, S[0,2] =
,
,
,
2
6
6
12
12
12 12 12 12


 


 


x
1
5
7
5
7
5 7
x
2) sin =
.
+ 2Z
+ 2Z x
+ 4Z
+ 4Z . De plus, S[0,4] =
,
2
2
4
4
2
2
2 2
2

9 13 17
.
3) tan(5x) = 1 5x + Z x
+ Z. De plus, S[0,] =
, ,
,
,
4
20
5
20 4 20 20 20
1
(1 + cos(2x)) cos(2x) = 1 2x 2Z x Z. De plus, S[0,2] = {0, , 2}.
2

 

1
5) 2 cos2 x3 cos x+1 = 0 (2 cos x1)(cos x1) = 0 cos x = ou cos x = 1 x + 2Z
+ 2Z 2Z.
2
3
3


5
De plus, S[0,2] = 0, ,
, 2 .
3 3

+ Z.
6) cos(nx) = 0 nx + Z x
2
2n n

7) | cos(nx)| = 1 nx Z x Z.
n

8) sin(nx) = 0 nx Z x Z.
n

9) | sin(nx)| = 1 nx + Z x
+ Z.
2
2n n
cos x 1
= 0 sin x = 0 ou cos x = 1 x Z. De plus, S[0,2] = {0, , 2}.
10) sin x = tan x sin x
cos x
4) cos(2x) = cos2 x cos(2x) =

http ://www.maths-france.fr

c Jean-Louis Rouget, 2014. Tous droits rservs.


11) 1re solution.


sin(2x) + sin x = 0 sin(2x) = sin(x + ) (k Z/ 2x = x + + 2k) ou (k Z/ 2x = (x + ) + 2k)
2k
)
(k Z/ x = + 2k) ou (k Z/ x =
3


2
4
De plus, S[0,2] = 0,
, ,
, 2 .
3
3
2me solution.

sin(2x) + sin x = 0 2 sin(x) cos(x) + sin(x) = 0 sin(x)(2 cos(x) + 1) = 0 sin(x) = 0 ou cos(x) =


(k Z/ x = k) ou (k Z/ x =

12)

+ 2k) ou (k Z/ x = + 2k)
3
3

1
2

1
1
1
12 cos2 x 8 sin2 x = 2 6 cos2 x 4(1 cos2 x) = 1 cos2 x = cos x = ou cos =
2
2
2
 



+ Z x + Z.
x + Z
4
4
4
2


3 3
.
De plus, S[,] = , , ,
4
4 4 4
Exercice no 4
h
i h i
1

x ,
, .
2
3
3

[
1
5
2) Pour x R, sin x > x
+ 2k .
+ 2k,
4
4
2
1) Pour x [, ], cos x 6

kZ

3) Pour x [0, 2],


cos x > cos




x
x
x
x
x
x
x
2 cos2 cos 1 > 0 2 cos + 1 cos 1 > 0 2 cos + 1 < 0 et cos 6= 1
2
2
2
2
2
2
2

[  2
x
1
x
4
cos <
+ 2k,
+ 2k
2
2
2
3
3
kZ



[  4
4
8
x
+ 4k,
+ 4k x
, 2 .
3
3
3
kZ

1
(1 + cos(2x)) > cos(2x) cos(2x) 6 1 x [, ].
2
 


1
5 7
1
1
3
5) Pour x [0, 2], cos2 x 6 6 cos x 6 x

.
,
,
2
4 4
4 4
2
2
4) Pour x [, ], cos2 x > cos(2x)
6) Pour x [0, 2],
cos

x 
x
x
1
1
x
x
x
> 0 k Z/ 2k 6 6 + 2k
6 sin sin cos > 0 sin

3
3
3
3
3
4
3
4
2
2
3
3
3
k Z/
+ 6k 6 x 6 3 +
+ 6k
6 x 6 2
4
4
4

Exercice no 5

 1
1

1 + cos 2
=
cos2 =
8
2
8
2

!
2+ 2

2
1+
=
et donc, puisque cos > 0,
2
4
8

cos =
8

http ://www.maths-france.fr

2+ 2
.
2
2

c Jean-Louis Rouget, 2014. Tous droits rservs.




 2 2
1

1 cos 2
=
=
et donc, puisque sin > 0,
De mme, sin
8
2
8
4
8
2

sin =
8

2 2
.
2

Exercice no 6
 

cos
= cos cos + sin sin =
= cos

12
3
4
3
4
3
4

De mme,

sin

 

= sin cos sin sin =


= sin

12
3
4
3
4
3
4

cos
=
12

6+ 2
.
4

6 2
.
4

6+ 2
6 2

et sin
=
.
4
12
4

Exercice no 7
Pour n entier naturel non nul, on pose Sn =

ei(a1 ...an ) .

S1 = eia1 + eia1 = 2 cos a1


Soit n > 1. Supposons que Sn = 2n cos a1 ... cos an et montrons que Sn+1 = 2n+1 cos a1 ... cos an cos an+1 .
Sn+1 =

ei(a1 ...an+1) = eian+1

= 2 cos an+1 Sn

ei(a1 ...an ) + eian+1

ei(a1 ...an )

= 2n+1 cos a1 ... cos an+1 (par hypothse de rcurrence).


On a montr par rcurrence que : n > 1, Sn = 2n cos a1 ... cos an .
X
X
Ensuite, pour n > 1,
cos(a1 ... an ) = Re(Sn ) = 2n cos a1 ... cos an (on obtient aussi
sin(a1 ... an ) =
Im(Sn ) = 0).

Exercice no 8
a
a
1) Si a est dans ]0, 2[ alors, pour tout entier naturel non nul k, k est dans ]0, [ et donc sin k 6= 0. De plus, puisque
2
2
 a
a
a
sin 2 k = 2 sin k cos k , on a :
2
2
2
 a 
 a 
n sin
n
a Y
sin
Y
2k1 = 1
2k1
cos k =
a
a
n
2
2
k=1 2 sin
k=1
k=1 2 sin
2k
2k
sin a
 a  (produit tlescopique).
=
n
2 sin n
2
n
Y

2) k N , cos

i h
a
a
>
0
car
est
dans
0, .
2k
2k
2

n
X

k=1

 a 
ln cos k
= ln
2


n
Y

k=1

a
cos k
2

a
sin n
sin x
2
lim
Maintenant, lim
= 1 et donc,
a = x0
n+
x
n
2
http ://www.maths-france.fr

 a 



sin n
sin
a
sin
a
2 .
 a  = ln
ln
= ln
a
a
n
2 sin n
2n
2

c Jean-Louis Rouget, 2014. Tous droits rservs.


a




 a 
sin
sin a
sin a
n
lim
ln cos k
ln a2 = ln
.
= lim ln
n+
n+
2
a
a
k=1
2n
n
X

Exercice no 9

Soit x R.
2

24 cos

x+1

+ 16 24 sin

x3

x+1

+ 16 214 cos x = 20 24 cos x 10 + 16 24 cos x = 0


2
2
2
16
24 cos x 10 + 4 cos2 x = 0 (24 cos x )2 10 24 cos x + 16 = 0
2
2
24 cos x est solution de lquation X2 10X + 16 = 0

= 20 24 cos

24 cos

= 8 4 cos2 x = 1 ou 4 cos2 x = 3

1
1
3
3
cos x = ou cos x = ou cos x =
ou cos x =
2
2
 2   2 
x
+ Z
+ Z .
6
2
3
2

Exercice no 10

= 2 ou 24 cos

1) Tout dabord, daprs la formule de Moivre,


cos(3) + i sin(3) = (cos + i sin )3 = (cos3 3 cos sin2 ) + i(3 cos2 sin sin3 ),
et par identification des parties relles et imaginaires,
R, cos(3) = cos3 3 cos sin2 et sin(3) = 3 cos2 sin sin3 .
Ensuite, tan(3) et tan existent 3
/
Soit donc
/


+ Z.
6
3

tan(3) =


+ Z et
/ + Z 3
/ + Z
/ + Z.
2
2
2
6
3

3 cos2 sin sin3


sin(3)
3 tan tan3
=
=
,
cos(3)
1 3 tan2
cos3 3 cos sin2

aprs division du numrateur et du dnominateur par le rel non nul cos3 .


R \


6


3 tan tan3
.
Z , tan(3) =
3
1 3 tan2

1
2) Soit a 6= .
3
1re mthode. a est bien sr racine de lquation propose, ce qui permet dcrire :




1
3x x3
3a a3
=
3x x3 1 3a2 = 1 3x2 3a a3 (car ne sont pas solution de lquation)
1 3x2
1 3a2
3



3a2 1 x3 3 a3 3a x2 3 3a2 1 x + a3 3a = 0


(x a) 3a2 1 x2 + 8ax a2 + 3 = 0.

Le discriminant rduit du trinme 3a2 1 x2 + 8ax a2 + 3 vaut :

= 16a2 (3a2 1)(a2 + 3) = 3a4 + 6a2 + 3 = ( 3(a2 + 1))2 > 0.

Lquation propose a donc trois racines relles :


S =

http ://www.maths-france.fr

4a 3(a2 + 1) 4a + 3(a2 + 1)
.
,
a,
1 3a2
1 3a2
4

c Jean-Louis Rouget, 2014. Tous droits rservs.


i h

2me mthode. Il existe un unique rel ,
\ ,
tel que a = tan et de mme, si x est un rel distinct
2
2
6 6
h

i

1

\ ,
tel que x = tan ( savoir = Arctan a et = arctan x).
de , il existe un unique rel ,
2 2
6 6
3
1
Comme ne sont pas solution de lquation propose, on a :
3
3x x3
3a a3
3 tan tan3
3 tan tan3
=

=
tan(3) = tan(3)
1 3x2
1 3a2
1 3 tan2
1 3 tan2

3 3 + Z + Z.
3

Ceci refournit les solutions x = tan = a, puis






tan

+
tan
(a + 3)(1 + 3a)
4a + 3(a2 + 1)

a+ 3
3


=
=
,
x = tan +
= 1 3a =
3
1 3a2
1 3a2
1 tan tan
3

2
4a 3(a + 1)

.
et x = tan( ) =
3
1 3a2
Exercice no 11
Pour x [0, ], posons f(x) = tan x + tan(2x) + tan(3x) + tan(4x).
f(x) existe tan x, tan(2x), tan(3x) et tan(4x) existent

 
 




x
/ + Z , 2x
/ + Z , 3x
/ + Z et 4x
/ + Z
2
2
2
2
 


 



/ + Z , x
/ + Z et x
/ + Z
x
/ + Z , x
4
2
3
8
4
6
2
3 5 3 5 7
.
, , ,
, ,
,
,
,
x
/
8 6 4 8 2 8 4 6 8

f est dfinie et continue sur

h h i h i h  3   3   5   5 3   3 5   5 7   7 
0,

,
,
,
,
,
,
,
,
, .
8
8 6
6 4
4 8
8 2
2 8
8 4
4 6
6 8
8
Sur chacun des dix intervalles prcdents, f est dfinie, continue et strictement croissante en tant que somme de fonctions
strictement croissantes. La restriction de f chacun de ces dix intervalles est donc bijective de lintervalle considr sur
lintervalle image, ce qui montre dj que lquation propose, que lon note dornavant (E), a au plus une solution par
intervalle et donc au plus dix solutions dans [0, ].


h h
7
Sur I = 0,
ou I =
, , puisque f(0) = f() = 0, (E) a exactement une solution dans I. Ensuite, dans lexpression
8
8
de somme f, une et une seule des quatre fonctions est un infiniment grand en chacun des nombres considrs ci-dessus,

lexception de . En chacun de ses nombres, f est un infiniment grand. Limage par f de chacun des six intervalles ouverts
2

nayant pas pour borne est donc ] , +[ et (E) admet exactement une solution dans chacun de ces intervalles. Ceci
2
porte le total 6 + 2 = 8 solutions.

, tan x et tan(3x) tendent vers + tandis que tan(2x) et tan(4x) tendent vers 0. f tend donc vers + en
, et de
En
2
2
+
mme f tend vers en
. Limage par f de chacun des deux derniers intervalles est donc encore une fois ] , +[
2
et finalement,
Lquation (E) admet exactement dix solutions dans [0, ].
Exercice no 12


1
1
1
2
2
1) cos x = (1 + cos(2x)) et une primitive de x 7 cos x sur R est x 7
x + sin(2x) .
2
2
2

2) Daprs les formules dEuler,


http ://www.maths-france.fr

c Jean-Louis Rouget, 2014. Tous droits rservs.



 4
1 ix
1 4ix
ix
cos x =
=
e +e
(e
+ 4e2ix + 6 + 4e2ix + e4ix )
2
16
1
1
=
(2 cos(4x) + 8 cos(2x) + 6) = (cos(4x) + 4 cos(2x) + 3)
16
8


1 1
sin(4x) + 2 sin(2x) + 3x .
Donc, une primitive de x 7 cos4 x sur R est x 7
8 4
4

3) Daprs les formules dEuler,


 4
1 4ix
1 ix
=
e eix
(e
4e2ix + 6 4e2ix + e4ix )
2i
16
1
1
=
(2 cos(4x) 8 cos(2x) + 6) = (cos(4x) 4 cos(2x) + 3)
16
8


1
1
4
Donc, une primitive de x 7 sin x sur R est x 7
sin(4x) 2 sin(2x) + 3x .
8 4
sin4 x =

1
1
1
4) cos x sin x = sin2 (2x) = (1 cos(4x)) et une primitive de x 7 cos2 x sin2 x sur R est x 7
4
8
8
2



1
x sin(4x) .
4

5) Daprs les formules dEuler,


 6

1 6ix
1 ix
=
e eix
e 6e4ix + 15e2ix 20 + 15e2ix 6e4ix + e6ix
2i
64
1
1
= (2 cos(6x) 12 cos(4x) + 30 cos(2x) 20) =
( cos(6x) + 6 cos(4x) 15 cos(2x) + 10)
64
32


1
3
15
1
sin(6x) + sin(4x)
sin(2x) + 10x .
Donc, une primitive de x 7 sin6 x sur R est x 7
32
6
2
2
sin6 x =

6) cos x sin6 x = sin x sin6 x et une primitive de x 7 cos x sin6 x sur R est x 7

1
sin7 x.
7

7) cos5 x sin2 x = cos x(1 sin2 x)2 sin2 x = sin x sin2 x 2 sin x sin4 x + sin x sin6 x et une primitive de x 7 cos5 x sin2 x
2
1
1
sur R est x 7 sin3 x sin5 x + sin7 x.
3
5
7
1
8) cos3 x = sin x sin x sin2 x et une primitive de x 7 cos3 x est x 7 sin x sin3 x.
3
Exercice no 13
1) Pour x rel , on a :
 

 4 1 ix
 6
1 ix
ix
ix
cos x sin x =
e +e
e e
2
2i
1
= 10 (e4ix + 4e2ix + 6 + 4e2ix + e4ix )(e6ix 6e4ix + 15e2ix 20 + 15e2ix 6e4ix + e6ix )
2
1
= 10 (e10ix 2e8ix 3e6ix + 8e4ix + 2e2ix 12 + 2e2ix + 8e4ix 3e6ix 2e8ix + e10ix )
2
1
= 9 (cos 10x 2 cos 8x 3 cos 6x + 8 cos 4x + 2 cos 2x 6)
2
1
(cos 10x 2 cos 8x 3 cos 6x + 8 cos 4x + 2 cos 2x 6).
=
512
(Remarque. La fonction propose tait paire et labsence de sinus tait donc prvisible. Cette remarque guidait aussi les
calculs intermdiaires : les coefficients de e2ix , e4ix ,... taient les mmes que ceux de e2ix , e4ix ,...) Par suite,
4

sin 10x sin 8x sin 6x

+ 2 sin 4x + sin 2x
10
4
2




1
1
9 3 + 4
=

.
3 + 2( 3) =
512
4
2048

1
I=
512

http ://www.maths-france.fr

/3

6
3
6
/6


c Jean-Louis Rouget, 2014. Tous droits rservs.


2) Pour x rel, on a
cos4 x sin7 x = cos4 x sin6 x sin x = cos4 x(1 cos2 x)3 sin x
= cos4 x sin x 3 cos6 x sin x + 3 cos8 x sin x cos10 x sin x.
Par suite,
/3

cos5 x 3 cos7 x cos9 x cos11 x
+

+
J=
5
7
3
11
/6

3
1
1
1
1
1
1
1
(1 27 3)
(1 81 3) +

(1 243 3)
= (1 9 3) +
5 32
7 128
3 512
11 2048

1
= 11
(14784(1 9 3) + 7920(1 27 3) 1540(1 81 3) + 105(1 243 3))
2 3 5 7 11

1
=
(8284 + 18441 3).
365 440
Exercice no 14
x
1 cos x
1) tan existe si et seulement si x
/ + 2Z et
existe si et seulement si x
/ Z. Pour x
/ Z,
2
sin x
x
2 sin2
x
1 cos x
 x  2  x  = tan .
=
sin x
2
cos
2 sin
2
2
2) Pour tout rel x,
sin(x

2
1
1
3
3
2
) + sin x + sin(x +
) = sin x
cos x + sin x sin x +
cos x = 0,
3
3
2
2
2
2

ou, bien mieux,









2
2
2
2
+ sin x + sin x +
= Im ei(x 3 ) + eix + ei(x+ 3 ) = Im eix j2 + 1 + j = 0.
sin x
3
3



2

x , tan
+ x et
existent si et seulement si x, + x et 2x ne sont pas dans + Z, ce qui
4
4
cos(2x)
4
4
2


quivaut x
/ + Z. Donc, pour x
/ + Z,
4
2
4
2

3) tan

tan

4) Pour x
/



 1 tan x 1 + tan x
cos x sin x cos x + sin x
x + tan
+x =
+
=
+
4
4
1 + tan x 1 tan x
cos x + sin x cos x sin x
(cos x sin x)2 + (cos x + sin x)2
2(cos2 x + sin2 x)
2
=
=
=
.
2
2
cos(2x)
cos(2x)
cos x sin x

Z,
4
cos x sin x
cos2 x sin2 x
2 cos(2x)
2
1
tan x =

=
=
=
.
tan x
sin x cos x
sin x cos x
sin(2x)
tan(2x)

Exercice no 15
1) Pour tout rel x, 1 2k cos x + k2 = (k cos x)2 + sin2 x 0. De plus,
1 2k cos x + k2 = 0 k cos x = sin x = 0 x Z et k = cos x k {1, 1},

ce qui est exclu. Donc,

k R \ {1, 1}, x R, 1 2k cos x + k2 > 0.


fk est donc dfinie sur R, drivable sur R en vertu de thormes gnraux, impaire et 2-priodique. On ltudie dornavant
sur [0, ]. Pour x [0, ], on a :
http ://www.maths-france.fr

c Jean-Louis Rouget, 2014. Tous droits rservs.


1
sin x(2k sin x)(1 2k cos x + k2 )3/2
2
= (1 2k cos x + k2 )3/2 (cos x(1 2k cos x + k2 ) k sin2 x)

fk (x) = cos x(1 2k cos x + k2 )1/2

= (1 2k cos x + k2 )3/2 (k cos2 x + (1 + k2 ) cos x k)


= (1 2k cos x + k2 )3/2 (k cos x 1)(k cos x)
x R, fk (x) =

(k cos x 1)(k cos x)


.
(1 2k cos x + k2 )3/2

1er cas : |k| < 1 et k 6= 0 (f0 (x) = sin x). Pour tout rel x, (1 2k cos x + k2 )3/2 (k cos x 1) < 0 (car |k cos x| < 1) et
fk (x) est du signe de cos x k.
x

Arccos k

f (x)

0
1

f
0

1 k2
car fk (Arccos k) =
= 1).
1 2k2 + k2
2me cas : k > 1. Pour tout rel x, (1 2k cos x + k2 )3/2 (k cos x) > 0 et fk (x) est du signe de k cos x 1.
x

Arccos

f (x)

1
k

0
1
k

1


1 2
1
1
k
car fk Arccos
=
= .
2
k
k
12+k
3me cas : k < 1. Pour tout rel x, (1 2k cos x + k2 )3/2 (k cos x) < 0 et fk (x) est du signe de 1 k cos x.
x

Arccos

f (x)

0
1

f
0

1
k

1


1 2
1
1
k
car fk Arccos
=
= .
2
k
k
12+k
Z
2) Pour k R \ {1, 1}, posons Ik =
fk (x) dx.
Si k = 0, Ik =

sin x dx = 2. Sinon,

Z
i
1 hp
2k sin x
1

dx =
1 2k cos x + k2
k 0 2 1 2k cos x + k2
k
0
p
1
1 p
= ( 1 + 2k + k2 1 2k + k2 ) = (|k + 1| |k 1|).
k
k

Ik =

http ://www.maths-france.fr

c Jean-Louis Rouget, 2014. Tous droits rservs.


Plus prcisment, si k
1
((1 + k) (k 1)) =
k

1
] 1, 1[\{0}, Ik = ((1 + k) (1 k)) = 2, ce qui reste vrai pour k = 0. Si k > 1, Ik =
k
2
2
, et enfin, si k < 1, Ik =
. En rsum,
k
k
Si k ] 1, 1[, Ik = 2 et si k ] , 1[]1, +[, Ik =

2
.
|k|

Exercice no 16
1) Soient n N et x R. Posons Sn =
1re solution.

n
X

cos(kx) et Sn =

k=0

Sn + iSn =

n
X

sin(kx).

k=0

n
X

(cos(kx) + i sin(kx)) =

k=0
ix

Maintenant, e

n
X

eikx =

k=0

n
X

(eix )k .

k=0

= 1 x 2Z. Donc,

1er cas. Si x 2Z, on a immdiatement Sn = n + 1 et Sn = 0.

2me cas. Si x
/ 2Z, alors eix 6= 1 et

(n+1)x

Sn + iSn =

2i sin 2
1 ei(n+1)x
ei(n+1)x/2 ei(n+1)x/2 ei(n+1)x/2
= einx/2
=
ix
ix/2
i(n+1)x/2
i(n+1)x/2
1e
2i sin x2
e
e
+e

= einx/2

sin (n+1)x
2
sin x2

Par identification des parties relles et imaginaires, on obtient





 nx 
(n + 1)x

cos
sin

2
2
X
x
si x
/ 2Z
cos(kx) =

sin

k=0

n + 1 si x 2Z




nx 
(n + 1)x

sin
sin

2
2
X
x
si x
/ 2Z
et
sin(kx) =

sin

k=0

0 si x 2Z

2me solution.
 

 

n 
n
n
X
X
xX
x
1
1
2 sin
cos(kx) =
2 sin cos(kx) =
x sin
k
x
sin
k+
2
2
2
2
k=0
k=0
k=0
  X

 !

n
n
X
1
1
x
sin
k
x
=
sin
(k + 1)
2
2
k=0
k=0

 


1
x
= sin
n+
x sin
(somme tlescopique)
2
2
(2n + 1)x
x
(n + 1)x
nx
= sin
+ sin = 2 sin
cos
2
2
2
2
et donc, si x
/ 2Z,...
2) Soient n N et x R. Posons Sn =

n
X

cos2 (kx) et Sn =

k=0

Sn + Sn =

n
X

sin2 (kx). On a :

k=0
n
X

(cos2 (kx) + sin2 (kx)) =

k=0

et
Sn Sn =

n
X

1 = n + 1,

k=0

(cos2 (kx) sin2 (kx)) =

k=0

http ://www.maths-france.fr

n
X

n
X

cos(2kx).

k=0

c Jean-Louis Rouget, 2014. Tous droits rservs.


Daprs 1), si x Z, on trouve immdiatement,


n
X

cos2 (kx) = n + 1 et

k=0

et si x
/ Z,

n
X

sin2 (kx) = 0,

k=0

Sn + Sn = n + 1 et Sn Sn =

cos(nx) sin(n + 1)x


,
sin x

de sorte que
1
Sn =
2





cos(nx) sin(n + 1)x
1
cos(nx) sin(n + 1)x

n+1+
et Sn =
n+1
.
sin x
2
sin x

3) Soient n N et x R. Daprs la formule du binme de Newton


n
X

Ckn

cos(kx)) + i

k=0

n
X

Ckn

sin(kx)

k=0

n
X

Ckn eikx =

k=0

n
X

Ckn eix

k=0


ix n

= 1+e

k

1nk


n
x
nx 
nx
= eix/2 + eix/2 einx/2 = 2n cosn
+ i sin
cos
.
2
2
2

Par identification des parties relles et imaginaires, on obtient alors


n N, x R,

n
X

Ckn cos(kx) = 2n cosn

n
X
nx
x
nx
x
cos
et
Ckn sin(kx) = 2n cosn sin
.
2
2
2
2
k=0

k=0

Exercice no 17

cos a + cos b + cos c = 0


(cos a + cos b + cos c) + i(sin a + sin b + sin c) = 0 eia + eib + eic = 0
sin a + sin b + sin c = 0







eia + eib = | eic | = 1 eia/2 eib/2 ei(ab)/2 + ei(ab)/2 = 1



a b 1

cos
=
2 2

 

 

2
2
a b 

+ Z + Z a b
+ 2Z
+ 2Z

2
3
3
3
3
2
k Z, {1, 1}/ b = a +
+ 2k.
3

Par suite, ncessairement, eib = jeia ou eib = j2 eia . Rciproquement, si eib = jeia ou encore b = a +
eia + eib + eic = 0 eic = (eia + eib ) = (1 + j)eia = j2 eia k Z/ c = a

et si eib = j2 eia ou encore b = a

2
+ 2k,
3

2
+ 2k ,
3

2
+ 2k,
3

2
+ 2k .
3



2
2
S =
a, a +
+ 2k, a
+ 2k , a R, {1, 1}, (k, k ) Z2 .
3
3

eia + eib + eic = 0 eic = (eia + eib ) = (1 + j2 )eia = jeia k Z/ c = a +

Exercice no 18

http ://www.maths-france.fr

10

c Jean-Louis Rouget, 2014. Tous droits rservs.


cos4





3
5
7

3

= 2 cos4 + sin4
+ cos4
+ cos4
+ cos4
= 2 cos4 + cos4
8
8
8
8
8
8
8
8




2
1

2
2
2
2
2
= 2 1 sin
2 cos
+ sin
sin
=2
cos
8
8
8
8
2
4


1
3
=2 1
= .
4
2

Exercice no 19
1) Soit x R.







cos(3x) = sin(2x) cos(3x) = cos


2x k Z/ 3x = 2x + 2k ou k Z/ 3x = + 2x + 2k
2
2
2





2k
ou k Z/ x = + 2k .
k Z/ x =
+
10
5
2
S[0,2] =

9 13 3 17
, ,
,
,
,
10 2 10 10 2 10

2) cos(3x) = Re(e3ix ) = Re((cos x + i sin x)3 ) = cos3 x 3 cos x sin2 x = cos3 x 3 cos x(1 cos2 x) = 4 cos3 x 3 cos x.
x R, cos(3x) = 4 cos3 x 3 cos x.
Par suite,
cos(3x) = sin(2x) 4 cos3 x 3 cos x = 2 sin x cos x cos x(4 cos2 x 3 2 sin x) = 0

cos x(4 sin2 x 2 sin x + 1) = 0 (cos x = 0) ou (4 sin2 x + 2 sin x 1 = 0).

13

et
(car, dans chacun des deux
10
10

et
cas, cos x 6= 0), ou encore, lquation 4X2 + 2X 1 = 0 admet pour solutions les deux nombres distincts X1 = sin
10
13
, qui sont donc les deux solutions de cette quation. Puisque X1 > 0 et que X2 < 0, on obtient
X2 = sin
10

1 + 5
1 5
X1 =
et X2 =
.
4
4
13
3
Donc, (puisque sin
= sin
),
10
10
Daprs 1), lquation 4 sin2 x + 2 sin x 1 = 0 admet entre autres pour solutions

1 +
sin
=
10
4
3
Ensuite, sin
= cos
10

2
10

= cos

3
1+ 5
et sin
=
.
10
4

, et donc
5

1+ 5

.
cos =
5
4

=
Enfin, cos
10

1p
1p

3
=
.
1 sin2
10 + 2 5 et de mme sin =
10 2 5 = cos
10
4
5
4
10

Exercice no 20

1) La fonction f1 est dfinie sur R, 2-priodique et paire. On ltudie sur [0, ].


La fonction f1 est drivable sur [0, ] et pour tout x de [0, ]
http ://www.maths-france.fr

11

c Jean-Louis Rouget, 2014. Tous droits rservs.


f1 (x) = 2 sin(x) 2 sin(2x) = 2 sin(x) 4 sin(x) cos(x) = 2 sin(x)(1 + 2cos(x)).


La fonction sinus sannule en 0 et et est strictement positive sur ]0, [. Donc la fonction f1 est du signe de 1 2 cos(x)
sur ]0, [. Ensuite, pour x ]0, [,
1 2 cos(x) = 0 cos(x) =

et

2
1
x=
,
2
3

1
2
x>
(par stricte dcroissance de la fonction cos sur [0, ].)
2
3




2
2
2
, strictement positive sur
Ainsi, la fonction f1 est strictement ngative sur 0,
, et sannule en 0,
et .
3
3
3
On en dduit le tableau de variations de la fonction f1 :
1 2 cos(x) > 0 cos(x) <

f1 (x)

0
3

2
3

0
1

f1

3
2

Graphe de f1 .
3
2
1

1
2
3

2) Pour tout rel x, 2 cos(x) 6= 0 et donc, la fonction f2 est dfinie sur R, 2-priodique et impaire. On ltudie sur [0, ].
La fonction f2 est drivable sur [0, ] et pour tout x de [0, ]
f2 (x) =

cos(x)(2 cos(x)) sin(x)(sin(x))


2 cos(x) 1
=
.
2
(2 cos(x))
(2 cos(x))2

La fonction f2 est du signe de 2 cos(x) 1 sur [0, ]. Ensuite, pour x [0, ],


2 cos(x) 1 = 0 cos(x) =

et

x= ,
2
3

x < (par stricte dcroissance de la fonction cos sur [0, ].)


2
3
i i
h h

Ainsi, la fonction f2 est strictement positive sur 0, , strictement ngative sur


, et sannule en . On note que
3
3
3
2 cos(x) 1 > 0 cos(x) >

http ://www.maths-france.fr

12

c Jean-Louis Rouget, 2014. Tous droits rservs.


f2


3

3
2


=
= 0, 57 . . .
=
1
3
2
2

On en dduit le tableau de variations de la fonction f2 :


x

2
3

f1 (x)

3
3

f1
0

Graphe de f2 .
1

1


3) f3 est dfinie sur D = R \
+ Z , paire et 2-priodique. f3 est continue sur D en vertu de thormes gnraux.
h h
i i
h h i i2
, . Si x 0, , f3 (x) = tan x + cos x et si x
, , f3 (x) = tan x + cos x.

On tudie f3 sur 0,
2
2
2
2

Etude en . lim | tan x| = + et lim cos x = 0. Donc, lim f(x) = +. La courbe reprsentative de la fonction f3
2 x/2
x/2
x/2

admet la droite dquation x = pour droite asymptote.


2
h h
h h i i

, en vertu de thormes gnraux et pour x 0, ,


Drivabilit et drive. f3 est drivable sur 0,
2
2
2
i i
1
1

f3 (x) =
sin x et pour x
, , f3 (x) = 2 sin x.
cos2 x
2
cos x
f3 est drivable droite en 0 et (f3 )d (0) = 1. Par symtrie, f3 est drivable gauche en 0 et (f3 )g (0) = 1. f3 nest pas
drivable en 0.
De mme, f2 est drivable gauche et droite en avec (f3 )g () = 1 et (f3 )d () = 1, et nest donc pas drivable en .

Variations. f3 est strictement dcroissante sur ] , ] en tant que somme de deux fonctions strictement dcroissantes sur
2

] , ]. Puis, pour x lment de ]0, [,


2
2
1
f3 (x) =
sin x > 1 1 = 0.
cos2 x

La fonction f3 est strictement positive sur ]0, [ et donc f3 est strictement croissante sur [0, [.
2
2
Graphe.

http ://www.maths-france.fr

13

c Jean-Louis Rouget, 2014. Tous droits rservs.


y = f3 (x)
6
5
4
3
2
1

1
4) La fonction f4 est 2-priodique. On ltudie sur [, ]. Pour x [, ],
2
2
1
x=
ou x =
.
2
3
3


2
2
2
Pour x [, ], f4 (x) existe si et seulement si x 6=
et x 6=
. On tudie la fonction f4 sur D = ,

3
3
3

 

2 2
2
,

, .
3 3
3
2 cos(x) + 1 = 0 cos(x) =

2
2
. Quand x tend vers
par valeurs infrieures, 2 cos(x) + 1 tend vers 0 par valeurs suprieures et quand x
Etude en
3
3
2
2
par valeurs suprieures, 2 cos(x) + 1 tend vers 0 par valeurs infrieures. Dautre part, quand x tend vers
,
tend vers
3
3

2 sin(x) + 1 tend vers 3 + 1 qui est strictement positif. On en dduit que


lim f4 (x) = + et

x 2
3

lim + f4 (x) = .

x 2
3

2
2
Etude en . Quand x tend vers
par valeurs infrieures, 2 cos(x) + 1 tend vers 0 par valeurs infrieures et quand
3
3
2
par valeurs suprieures, 2 cos(x) + 1 tend vers 0 par valeurs suprieures. Dautre part, quand x tend vers
x tend vers
3

2
, 2 sin(x) + 1 tend vers 3 + 1 qui est strictement ngatif. On en dduit que
3
lim

x 2
3

f4 (x) = + et

lim

x 2
3

f4 (x) = .

Drive. La fonction f4 est drivable sur D et pour tout x de D,


(2 cos(x))(2 cos(x) + 1) (2 sin(x) + 1)(2 sin(x))
4 + 2 cos(x) + 2 sin(x)
=
(2 cos(x) + 1)2
(2 cos(x) + 1)2




1
1


4 + 2 2 cos(x) + sin(x)
4 + 2 2 sin x +
2
2
4 .
=
=
(2 cos(x) + 1)2
(2 cos(x) + 1)2



> 4 2 2 > 0 et donc la fonction f4 est strictement positive sur D. La fonction f4
Pour tout x de D, 4 + 2 2 sin x +
4




 




2 2
2 2
2
2
2
et sur ,
et sur
,

, (mais pas sur ,


est donc strictement croissante sur ,
3
3 3
3
3
3 3


2
, ).
3
f4 (x) =

http ://www.maths-france.fr

14

c Jean-Louis Rouget, 2014. Tous droits rservs.


y = f4 (x)

Graphe.
4
3
2
1

1
2
3
4

http ://www.maths-france.fr

15

c Jean-Louis Rouget, 2014. Tous droits rservs.


Planche no 13. Fonctions circulaires rciproques


* trs facile ** facile *** difficult moyenne **** difficile
I : Incontournable T : pour travailler et mmoriser le cours
Exercice no 1 (***IT)
Domaine de dfinition et calcul des fonctions suivantes :
sin(Arcsin x), Arcsin(sin x), cos(Arccos x), Arccos(cos x), tan(Arctan x), Arctan(tan x).
Exercice no 2 (IT)
1) (**) Calculer Arccos x + Arcsin x pour x lment de [1; 1].
1
pour x rel non nul.
x
3) (**) Calculer cos(Arctan a) et sin(Arctan a) pour a rel donn.

2) (**) Calculer Arctan x + Arctan

a+b
(on tudiera
1 ab
dabord cos(Arctan a + Arctan b) et on distinguera les cas ab < 1, ab > 1 et a > 0, ab > 1 et a < 0).

4) (***) Calculer, pour a et b rels tels que ab 6= 1, Arctan a + Arctan b en fonction de Arctan
Exercice no 3 (***I)
Z sin2 x
Z cos2 x

Arcsin t dt +
Existence et calcul de
Arccos t dt.
0

Exercice no 4 (***)
Simplifier les expressions suivantes :


x
1) f1 (x) = Arcsin
.
1 + x2


1 x2
.
2) f2 (x) = Arccos
1 + x2

3) f3 (x) = Arcsin 1 x2 Arctan


4) f4 (x) = Arctan

!
1x
.
1+x

x
x1
1
Arctan
+ Arctan
.
2x2
x+1
x

Exercice no 5 (**)
1
1
1
+ Arctan + Arctan .
2
5
8
Exercice no 6 (***I)
Calculer Arctan

Calculer un = Arctan
(Utiliser le n 2.4))

2
2
2
+ Arctan 2 + ... + Arctan 2 pour n entier naturel non nul donn puis dterminer
2
1
2
n

lim un .

n+

Exercice no 7 (**) (Mines de DOUAI 1984)


On considre la fonction numrique f telle que :
f(x) = (x2 1) Arctan

1
,
2x 1

et on appelle (C ) sa courbe reprsentative dans un repre orthonorm.


1) Quel est lensemble de dfinition D de f ?
2) Exprimer, sur D \ {0}, la drive de f sous la forme : f (x) = 2xg(x).
3) Montrer que : x R, 2x4 4x3 + 9x2 4x + 1 > 0 et en dduire le tableau de variation de g.
4) Dresser le tableau de variation de f.

http ://www.maths-france.fr

c Jean-Louis Rouget, 2014. Tous droits rservs.


Exercice no 8 (**)
Simplifier les expressions suivantes
2

1) sin(2 Arcsin x) 2) cos(2 Arccos x) 3) sin

Arccos x
2

Exercice no 9
Rsoudre dans R les quations suivantes :
1
3
3) (*) tan(x) = 3

1) (*) cos x =

5) (***) Arcsin(2x) = Arcsin x + Arcsin(x 2)


7) (***) Arctan(x 1) + Arctan(x) + Arctan(x + 1) =

http ://www.maths-france.fr

1
4
x
4) (***) Arcsin(x) + Arcsin
=
2 4 
6) (***) 2 Arcsin x = Arcsin 2x 1 x2
2) (*) sin(2x) =

.
2

c Jean-Louis Rouget, 2014. Tous droits rservs.


Planche no 13. Fonctions circulaires rciproques : corrig


Exercice no 1
1) Arcsin x existe si et seulement si x est dans [1, 1]. Donc, sin(Arcsin x) existe si et seulement si x est dans [1, 1] et
pour tout x de [1, 1], sin(Arcsin x) = x.
h i
2) Arcsin(sin x) existe pour tout rel x mais ne vaut x que si x est dans , .
2 2

Sil existe un entier relatif k tel que + 2k 6 x < + 2k, alors 6 x 2k < et donc
2
2
2
2
Arcsin(sin x) = Arcsin(sin(x 2k)) = x 2k.


1
1
1
x
x
puis
+ < k + et donc k = E
+
De plus, on a k 6
2 4
2
2 4


1
x
.
+
Arcsin(sin x) = x 2 E
2 4
Sil existe un entier relatif k tel que

+ 2k 6 x <
+ 2k, alors < x + 2k 6 et donc
2
2
2
2

Arcsin(sin x) = Arcsin(sin( x + 2k)) = x + 2k.




x
1
1
1
x
De plus, k 6
puis
< k + et donc k = E

2 4
2
2 4


1
x
.

Arcsin(sin x) = x + 2 E
2 4
3) Arccos x existe si et seulement si x est dans [1, 1]. Donc, cos(Arccos x) existe si et seulement si x est dans [1, 1] et
pour tout x dans [1, 1], cos(Arccos x) = x.
4) Arccos(cos x) existe pour tout rel x mais ne vaut x que si x est dans [0, ].
 x 
.
Sil existe un entier relatif k tel que 2k 6 x < + 2k, alors Arccos(cos x) = x 2k avec k = E
2
Sil 
existe un
 entier relatif k tel que + 2k 6 x < 2k alors Arccos(cos x) = Arccos(cos(2k x)) = 2k x avec
x+
.
k=E
2
5) Pour tout rel x, tan(Arctan x) = x.

+ Z et pour ces x, il existe un entier relatif k tel que


2



x 1
+ k < x < + k. Dans ce cas, Arctan(tan x) = Arctan(tan(x k)) = x k avec k = E
.
+
2
2
2
6) Arctan(tan x) existe si et seulement si x nest pas dans

Exercice no 2
1) 1re solution. Posons f(x) = Arccos x + Arcsin x pour x dans [1, 1].
f est dfinie et continue sur [1, 1], drivable sur ] 1, 1[. De plus, pour x dans ] 1, 1[,
f (x) =

1
1

= 0.
2
1x
1 x2

Donc f est constante sur ] 1, 1[ puis sur [1, 1] par continuit de f en 1 et en 1. Pour tout x de [1, 1], f(x) = f(0) =
x [1, 1], Arccos x + Arcsin x =

.
2

.
2

2me solution. Il existe un unique rel dans [0, ] tel que x = cos , savoir = Arccos x. Mais alors,

 

=+ =
Arccos x + Arcsin x = Arccos(cos ) + Arcsin sin
2
2
2

h i
 

= car est dans , .)


(Arccos(cos ) = car [0, ] et Arcsin sin
2
2
2
2 2
http ://www.maths-france.fr

c Jean-Louis Rouget, 2014. Tous droits rservs.


1
2) 1re solution. Pour x rel non nul, posons f(x) = Arctan x + Arctan . Notons que f est impaire.
x
1
1
1
f est drivable sur R et pour x non nul, f (x) =
= 0. f est donc constante sur ] , 0[ et sur ]0, +[

1
1 + x2 x2
1+ 2
x

(mais pas ncessairement sur R ). Donc, pour x > 0, f(x) = f(1) = 2 Arctan 1 = , et puisque f est impaire, pour x < 0,
2

f(x) = f(x) = . Donc,


2

1 2 si x > 0

x R , Arctan x + Arctan =
= sgn(x).
x si x < 0
2
2

i h
2me solution Pour x rel strictement positif donn, il existe un unique rel dans 0,
tel que x = tan savoir
2
= Arctan x. Mais alors,



1
= Arctan(tan ) + Arctan(cotan ) = Arctan(tan ) + Arctan tan

x
2

=+ = .
2
2
i
h

(car et sont lments de 0, .)


2
2
i h
1
1
.
De
plus
,
Arctan
a
est
dans
,
et donc cos(Arctan a) > 0. On
3) cos2 (Arctan a) =
=
1 + a2
2 2
1 + tan2 (Arctan a)
1
en dduit que pour tout rel a, cos(Arctan a) =
. Ensuite,
1 + a2
a
.
sin(Arctan a) = cos(Arctan a) tan(Arctan a) =
1 + a2
Arctan x + Arctan

a R, cos(Arctan a) =

a
1
.
et sin(Arctan a) =
1 + a2
1 + a2

4) Daprs 3),
1 ab

,
cos(Arctan a + Arctan b) = cos(Arctan a) cos(Arctan b) sin(Arctan a) sin(Arctan b) =
1 + a 2 1 + b2
ce qui montre dj , puisque ab 6= 1, que cos(Arctan a + Arctan b) 6= 0 et donc que tan(Arctan a + Arctan b) a un sens.
Immdiatement,
a+b
.
1 ab
i
h i h i h
Maintenant, Arctan a + Arctan b est dans ,
, .
,

2
2 2
2
tan (Arctan a + Arctan b) =


1er cas. Si ab < 1 alors cos(Arctan a + Arctan b) > 0 et donc Arctan a + Arctan b est dans ] , [. Dans ce cas,
2 2


a+b
Arctan a + Arctan b = Arctan
.
1 ab
i
h i h
2me cas. Si ab > 1 alors cos(Arctan a + Arctan b) < 0 et donc Arctan a + Arctan b est dans ,

, . Si de
2
2
i h

plus a > 0, Arctan a + Arctan b > et donc Arctan a + Arctan b est dans
, . Dans ce cas, Arctan a + Arctan b
2
2
a+b
a+b

. Donc, Arctan a + Arctan b = Arctan
+ . Si a < 0, on
est dans ] , [ et a mme tangente que Arctan
2 2
1 ab
1 ab
a+b
trouve de mme Arctan a + Arctan b = Arctan
.
1 ab
En rsum,
http ://www.maths-france.fr

c Jean-Louis Rouget, 2014. Tous droits rservs.



a+b

si ab < 1
Arctan

1
ab

a+b
Arctan a + Arctan b =
Arctan
+ si ab > 1 et a > 0 .

1
ab

a
+
b

Arctan
si ab > 1 et a < 0
1 ab
Exercice no 3
Pour x rel, on pose f(x) =

Z sin2 x
0

Z cos

Arcsin t dt +

Arccos t dt.

Zy

Arcsin t dt est dfinie et drivable sur [0, 1].


0
Z sin2 x

2
Arcsin t dt est
De plus, x 7 sin x est dfinie et drivable sur R valeurs dans [0, 1]. Finalement, la fonction x 7

La fonction t 7 Arcsin t est continue sur [0, 1]. Donc, la fonction y 7

dfinie et drivable sur R.

De mme, la fonction t 7 Arccos t est continue sur [0, 1]. Donc, la fonction y 7

Zy

Arccos t dt est dfinie et drivable

sur [0, 1]. De plus, la fonction x 7 cos2 x est dfinie et drivable sur R, valeurs dans [0, 1]. Finalement, la fonction
Z cos2 x

x 7
Arccos t dt est dfinie et drivable sur R.
0

Donc, f est dfinie et drivable sur R et, pour tout rel x,

f (x) = 2 sin x cos x Arcsin( sin2 x) 2 sin x cos x Arccos( cos2 x)

= 2 sin x cos x (Arcsin(| sin x|) Arccos(| cos x|)) .


h i
On note alors que f est -priodique et paire. Pour x lment de 0, , f (x) = 2 sin x cos x(xx) = 0. f est donc constante
2
Z 1/2
Z 1/2
Z 1/2
h i
h i


sur 0,
et pour x lment de 0, , f(x) = f
=
Arcsin t dt +
dt = . Mais
Arccos tdt =
2
2
4
2
4
0
0
0
alors, par parit et -priodicit,
x R,
Exercice no 4
1) 1re solution. Pour tout rel x,
R, impaire, et pour tout rel x,

Z sin2 x
0

Z cos

Arcsin t dt +

Arccos t dt = .
4

x
< 1. Ainsi f1 est dfinie et drivable sur
x2 + 1 > x2 = |x| et donc 1 <
2
x +1

2x
x2 + 1 x
p
1
1
2 x2 + 1 r

f1 (x) =
x2 + 1
=
x2 + 1
(1 + x2 ) x2 + 1
x2
1
1 + x2
1
=
= Arctan (x).
1 + x2
Donc il existe une constante relle C telle que pour tout rel x, f1 (x) = Arctan x + C.
x = 0 fournit C = 0 et donc,


x
= Arctan x.
x R, Arcsin
x2 + 1
i h
et x = tan .
2me solution. Pour x rel donn, posons = Arctan x. est dans ,
2 2

tan
= cos2 tan = cos tan (car cos > 0)
= p
x2 + 1
1 + tan2
= sin ,

http ://www.maths-france.fr

c Jean-Louis Rouget, 2014. Tous droits rservs.


et donc
i h
f1 (x) = Arcsin(sin ) = (car est dans , )
2 2
= Arctan x.
2
1 x2
=
6 1 + 2 = 1 avec galit si et seulement si x = 0. f2 est
1 + x2
1 + x2

donc dfinie et continue sur R, drivable sur R . De plus, f2 est paire. Pour tout rel x non nul,

2) 1re solution. Pour tout rel x, 1 < 1 +

f2 (x) =

1
1
2
4x
2x(1 + x2 ) 2x(1 x2 )
s

=
=
2
2
2


2
2
(1 + x )
1+x
1 + x2
4x
1 x2
1
1 + x2

o est le signe de x. Donc il existe une constante relle C telle que pour tout rel positif x, f2 (x) = 2 Arctan x + C (y
compris x = 0 puisque f est continue en 0).
x = 0 fournit C = 0 et donc, pour tout rel positif x, f2 (x) = 2 Arctan x. Par parit,
x R, Arccos

1 x2
1 + x2

= 2 Arctan |x|.

i h
2me solution. Soit = Arctan x pour x rel donn. est dans ,
et x = tan .
2 2
1 x2
1 tan2
= cos2 (1 tan2 ) = cos2 sin2 = cos(2).
=
1 + x2
1 + tan2
Donc

h
h


2 si 0,
2 Arctan x si x > 0
2 Arctan x si x > 0
2
i
i
f2 (x) = Arccos(cos(2)) =
=
=
2 Arctan x si x 6 0
2 Arctan(x) si x 6 0
2 si , 0
2
= 2 Arctan |x|.

3) La fonction x 7 Arcsin 1 x2 est dfinie et continue sur [1, 1], drivable sur [1, 1] \ {0} car pour x lment de
[1, 1], 1 x2 est lment de [0, 1] et vaut 1 si et seulement si x vaut 0.
1x
est dfini et positif si et seulement si x est dans ] 1, 1], et nul si et seulement si x = 1. f3 est donc dfinie et
1+x
continue sur ] 1, 1], drivable sur ] 1, 0[]0, 1[. Pour x dans ] 1, 0[]0, 1[, on note le signe de x et on a :
1
x

1
1
1
1
(1 + x) (1 x)
p
r
f3 (x) =
=
+
.

2
2
2
2
1

x
(1
+
x)
2
1x 1+
1x
1x
1 x2
1 (1 x )
2
1+x
1+x
1
1
1
= ( Arcsin) (x). Donc, il existe un rel C tel que, pour tout x de [0, 1] (par
Si x est dans ]0, 1[, f3 (x) =
2 1 x2
2

1
continuit en 0 et en 1) f3 (x) = Arcsin x + C. x = 1 fournit C = . Donc, pour tout x de 50, 1]
2
4
 1
1
1 
Arcsin x =
Arcsin x = Arccos x.
3 (x) =
4 2
2 2
2
x [0, 1], f3 (x) =

1
Arccos x.
2

3
1
3

= ( Arcsin) (x). Donc il existe un rel C tel que, pour tout x de ] 1, 0] (par
2 1 x2
2

3
continuit) f3 (x) = Arcsin x + C . x = 0 fournit = C . Donc,
2
2
4

Si x est dans ] 1, 0[, f3 (x) =

x ] 1, 0], f3 (x) =
http ://www.maths-france.fr

3
Arcsin x + .
2
4
c Jean-Louis Rouget, 2014. Tous droits rservs.

4) f4 est drivable sur D = R \ {1; 0} et pour x lment de D, on a :


(x + 1) x
1
1
x (x 1)

+
1
(x + 1)2
x2
x2
(x 1)2
1+ 4
1+
1+
4x
(x + 1)2
x2
1
1
4x
4x
4x

+
= 4
+
= 0.
= 4
4x + 1 2x2 + 1 + 2x 2x2 + 1 2x
4x + 1 (2x2 + 1)2 4x2

f4 (x) =

1
x3

f4 est donc constante sur chacun des trois intervalles ] , 1[, ] 1, 0[ et ]0, +[. Pour x > 0, f(x) = f(1) = 0. Pour
1


1 < x < 0, f(x) = lim f(t) = Arctan ( ) + Arctan 2 = + = .
t1
2
2
2
2
t>1

Pour x < 1, f(x) = lim f(t) = + = 0.
t
4
4

0 si x ] , 1[]0, +[
x R \ {1; 0}, f4 (x) =
.
si x ] 1, 0[
Exercice no 5
1
1

0 6 Arctan + Arctan < Arctan 1 + Arctan 1 = et


2
5
2
1 1


+
1
1
7
= 2 5 = .
tan Arctan + Arctan
11
2
5
9
1
25
1
1

1
1
7
7
1 h h
Comme Arctan + Arctan [0, [, on a donc Arctan + Arctan = Arctan . De mme, Arctan + Arctan 0,
2
5
2
2
5
9
9
8
2
et
7 1


+
1
7
65
= 9 8 =
= 1,
tan Arctan + Arctan
71
9
8
65
1
98
7
1

et donc Arctan + Arctan = Arctan 1 = . Finalement,


9
8
4
Arctan

1
1
1

+ Arctan + Arctan = .
2
5
8
4

Exercice no 6
o
(On va retrouver lei rsultat
de lexercice
et b hdeux rels positifs.
i nh 2 dans un cas particulier) Soienti a

h
et donc, Arctan a Arctan b , . De plus,
Alors, Arctan a 0, , Arctan b 0,
2
2
2 2
tan(Arctan a) tan(Arctan b)
ab
tan(Arctan a Arctan b) =
=
,
1 + tan(Arctan a) tan(Arctan b)
1 + ab
i h
et donc, puisque Arctan a Arctan b , ,
2 2


ab
.
a 0, b 0, Arctan a Arctan b = Arctan
1 + ab

2
(k + 1) (k 1)
= Arctan
= Arctan(k + 1) Arctan(k 1) (puisque
k2
1 + (k 1)(k + 1)
k 1 et k + 1 sont positifs). Par suite, si n est un entier naturel non nul donn,

Soit alors k un entier naturel non nul. Arctan

n1
n
n+1
X
X
X
2
=
(Arctan(k
+
1)

Arctan(k

1))
=
Arctan
k

Arctan k
k2
k=1
k=2
k=0
k=1

= Arctan(n + 1) + Arctan n .
4

un =

n
X

Arctan

http ://www.maths-france.fr

c Jean-Louis Rouget, 2014. Tous droits rservs.


La limite de un vaut donc


3
+ =
.
2
2
4
4
lim

n+

n
X

Arctan

k=1

2
3
=
.
2
k
4

Exercice no 7


1
.
1) f est dfinie et drivable sur D = R \
2
2) Pour x lment de D,
f (x) = 2x Arctan

1
2
+ (x2 1)
2x 1
(2x 1)2

1
1+

De plus, pour x non nul : f (x) = 2xg(x) o g(x) = Arctan

1
(2x 1)2

= 2x Arctan

1
x2 1
2
.
2x 1 2x 2x + 1

1
x2 1
1

.
2x 1 2x 2x2 2x + 1

3) Pour x lment de D \ {0},


1 2x(2x3 2x2 + x) (x2 1)(6x2 4x + 1)
1

2x2 2x + 1 2
x2 (2x2 2x + 1)2
2
2
4
2
2x (2x 2x + 1) + 2x 7x + 4x 1
2x4 4x3 + 9x2 4x + 1
=
=

.
2x2 (2x2 2x + 1)2
2x2 (x2 2x + 1)2

g (x) =

Maintenant,
2

3
2
+ > 0.
2x4 4x3 + 9x2 4x + 1 = 2x2 (x 1)2 + 7x2 4x + 1 = 2x2 (x 1)2 + 7 x
7
7




1
1
et sur
, + . En +, g(x) tend vers 0. Donc g est
Donc, g est strictement dcroissante sur ] , 0[, sur 0,
2
2


1
1
3
strictement positive sur
, + . Quand x tend vers par valeurs infrieures, g tend vers + < 0 et quand x tend
2
2
2 2


1
en un
vers 0 par valeurs suprieures, g(x) tend vers +. Donc g sannule une et une seule fois sur lintervalle 0,
2




1
1
certain rel x0 de 0, . g est de plus strictement ngative sur x0 ,
et strictement positive sur ]0, x0 [. Quand x tend
2
2
vers , g(x) tend vers 0. Donc g est strictement ngative sur ] , 0[.

4) Enfin, puisque f (x) = 2xg(x) pour x 6= 0, on a les rsultats suivants :




1
1
sur ] , 0[, f > 0, sur ]0, x0 [, f > 0, sur x0 , , f < 0, sur ] , +[, f > 0. Comme f (0) = 1 > 0, on a donc : sur
2
2
1
1
1
] , x0 [, f > 0, sur ]x0 , [, f < 0 et sur ] , +[, f > 0. f est strictement croissante sur ] , x0 ] et sur ] , +[ et
2
2
2
1
est strictement dcroissante sur [x0 , [.
2
o
Exercice n 8

1) Pour tout rel x de [1, 1], sin(2 Arcsin x) = 2 sin(Arcsin x) cos(Arcsin x) = 2x 1 x2 .


2) Pour tout rel x de [1, 1], cos(2 Arccos x) = 2 cos2 (Arccos x) 1 = 2x2 1.


Arccos x
1
1x
2
3) Pour tout rel x de [1, 1], sin
= (1 cos(Arccos x)) =
.
2
2
2
Exercice no 9
1) Soit x R.
1
cos x = k Z/ x = Arccos
3
http ://www.maths-france.fr

 
 
1
1
+ 2k ou k Z/ x = Arccos
+ 2k.
3
3
6

c Jean-Louis Rouget, 2014. Tous droits rservs.



 
 
 

1
1
S = Arccos
+ 2Z Arccos
+ 2Z .
3
3
2) Soit x R.

3) Soit x R.

 
 
1
1
1
+ 2k ou k Z/ x = + Arcsin
+ 2k
sin(2x) = k Z/ 2x = Arcsin
4
4
4
 
 
1
1
1
1
k Z/ x = Arcsin
+ k ou k Z/ x = + Arcsin
+ k.
2
4
2 2
4

tan(x) = 3 k Z/ x = Arctan(3) + k.

4) Une solution est ncessairement dans


 x [1, 1] et mme dans [0, 1].
La fonction x 7 Arcsin(x) + Arcsin
est continue et strictement croissante sur [0, 1] en tant que somme de deux
2
x
ralise donc une bijection
fonctions continues et strictement croissantes sur [0, 1]. La fonction x 7 Arcsin(x) + Arcsin
2




2

2
de [0, 1] sur 0,
. Comme 0,
, lquation propose a une solution et une seule et cette solution est dans [0, 1].
3
4
3
x

 x 
1
Si Arcsin(x) + Arcsin
= alors sin Arcsin(x) + Arcsin
= .
2
4
2
2
 


x
2
1
2
6 Arcsin(1)+Arcsin
=
,
Rciproquement, puisque x [0, 1], 0 6 Arcsin(x)+Arcsin
. Dans lintervalle 0,
2
2
3
3
1

il y a un nombre et un seule dont le sinus vaut savoir . Donc, pour x dans [0, 1],
4
2
Arcsin(x) + Arcsin

x
2

r

 x 

1
x2 x p
1
= sin Arcsin(x) + Arcsin
= x 1
+
1 x2 =
4
2
4
2
2
2
s



x2
1
x2
x2
+ (1 x2 ) + x2
(1 x2 ) =
1
x2 1
4
4
4
2

(car le premier membre de lquation initiale est positif)


s

1 5x2 x4
x2
2
x
(1 x2 ) =
+
1
4
2
4
2


2
2
x
(1 x2 ) = 2x4 5x2 + 2 et 2x4 5x2 + 2 > 0
16x4 1
4


1
4x8 20x6 + 16x4 = 4x8 20x6 + 33x4 20x2 + 4 et x2
/
,2
2







1
10 32 10 + 32
1
2
4
2
2
et x2
/
,2 x
,
,2
17x 20x + 4 = 0 et x
/
2
17
17
2

10 32
x2 =
s 17
10 4 2
(car x > 0).
x=
17
S =

r


10 4 2
.
17





1 1
1 1
5) Une solution est ncessairement dans , . Soit donc x un rel de , .
2 2
2 2

http ://www.maths-france.fr

c Jean-Louis Rouget, 2014. Tous droits rservs.


 

 
Arcsin(2x) = Arcsin x + Arcsin x 2 sin(Arcsin(2x)) = sin Arcsin x + Arcsin x 2
r
 2
p
2x = x 1 x 2 + x 2 1 x2
p
p
x = 0 ou 1 2x2 + 2 2x2 = 2
q
x = 0 ou 1 2x2 + 2 2x2 + 2 (1 2x2 ) (2 2x2 ) = 4
q
x = 0 ou 2 (1 2x2 ) (2 2x2 ) = 1 + 4x2
x = 0 ou 4(4x4 6x2 + 2) = (4x2 + 1)2
r
r
7
7
2
x = 0 ou 32x = 7 x = 0 ou x =
ou x =
32
32



1 1
Rciproquement, pour chacun des ces trois nombres x, la seule implication crite est une quivalence si x est dans ,
2 2
r !2
 2
i
h

14
16
1
7

=
) et de plus Arcsin x + Arcsin(x 2) est dans , . Or,
6
=
(ce qui est le cas puisque
32
64
64
2
2 2
0 6 Arcsin
r

7
+ Arcsin
32

7
+ Arcsin
32

!
r
r
r

7
7
7
8
2 = Arcsin
+ Arcsin
6 2 Arcsin
= 2 Arcsin =
32
32
16
16
2
2

7
2
32

et donc Arcsin
h i
, 0 ce qui achve la rsolution.
2

r
h i
7
0, . De mme, par parit, Arcsin
2
32

S =

+ Arcsin

7
2
32


14
14
.
,
0,
8
8

6) Soit x R. Arcsin x existe si et seulement si x [1, 1]. Ensuite,


Arcsin(2x

p
p
1 x2 ) existe x [1, 1] et 2x 1 x2 [1, 1]

x [1, 1] et 4x2 (1 x2 ) [0, 1] x [1, 1] et 4x2 (1 x2 ) 6 1

x [1, 1] et 4x4 4x2 + 1 > 0 x [1, 1] et (2x2 1)2 > 0


x [1, 1]

Pour x [1, 1], sin(2hArcsin(x))


= 2 sin(Arcsin x) cos(Arcsin x) = 2x 1 x2 = sin(Arcsin(2x 1 x2 )), et de plus,
i


Arcsin(2x 1 x2 ) , . Par suite,
2 2
h i
x solution x [1, 1] et 2 Arcsin(x) ,
2 2


h i
1
1
x , .
x [1, 1] et Arcsin(x) ,
4 4
2
2


1
1
S = , .
2
2

7) Par croissance de la fonction arctangente sur R, si x 6 0, Arctan(x 1) + Arctan(x) + Arctan(x + 1) 6 Arctan(1) +

Arctan(0) + Arctan(1) = 0. En particulier, Arctan(x 1) + Arctan(x) + Arctan(x + 1) 6= . Une solution est donc
2
ncessairement strictement positive.
http ://www.maths-france.fr

c Jean-Louis Rouget, 2014. Tous droits rservs.


Soit donc x un rel strictement positif.

Arctan(x 1) + Arctan(x + 1) = Arctan(x)


2
2
 
1
Arctan(x 1) + Arctan(x + 1) = Arctan
x
i h
1
tan (Arctan(x 1) + Arctan(x + 1)) = et Arctan(x 1) + Arctan(x + 1) ,
x
2 2
i h
1
(x 1) + (x + 1)
= et Arctan(x 1) + Arctan(x + 1) ,

1 (x 1)(x + 1)
x
2 2
h
i
1

2x
= et Arctan(x 1) + Arctan(x + 1) ,

2 x2
x
2 2


i h
2
2
et x
/ 0, 2
2x = 2 x et Arctan(x 1) + Arctan(x + 1) ,
2 2
r
i h
2
x=
et Arctan(x 1) + Arctan(x + 1) ,
3
2 2
!
!
r
r
r
i h
2
2
2
(car Arctan
1 +
+ 1 = 0, 8 . . . , ).
x=
3
3
3
2 2

Arctan(x 1) + Arctan(x) + Arctan(x + 1) =

r 
2
S =
.
3

http ://www.maths-france.fr

c Jean-Louis Rouget, 2014. Tous droits rservs.


Planche no 14. Trigonomtrie hyperbolique


* trs facile ** facile *** difficult moyenne **** difficile
I : Incontournable T : pour travailler et mmoriser le cours
Exercice no 1 (*IT)
Etablir pour ch, sh et th les formules daddition, de duplication et de linarisation.
Exercice no 2 (**)
Etudier f : x 7 ln(ch x) x. Montrer en particulier que la droite D dquation y = 2x ln 2 est asymptote au
graphe de f en (on dit que la droite dquation y = ax + b est asymptote au graphe de f en si et seulement si
lim f(x) (ax + b) = 0. Construire le graphe de f et la droite D.
x

Exercice no 3 (**)
Rsoudre dans R lquation sh(2 + x) + sh(2 + 2x) + ... + sh(2 + 100x) = 0.
Exercice no 4 (**I)
1
2

.
th(2x) th x



2) En dduire la valeur de un = 20 th 20 x + 21 th 21 x + ... + 2n1 th 2n1 x pour n entier naturel non nul et x rel
non nul donns puis calculer la limite de la suite (un ).
1) Montrer que pour tout rel x non nul, on a : th x =

Exercice no 5 (***I) (dfinition de argsh, argch et argth)


1) a) Montrer que sh est une bijection de R sur R. On note argsh la fonction rciproque (argument sinus hyperbolique).
b) Construire le graphe de argsh.
c) Dterminer une expression simple de largument sinus hyperbolique dun nombre (ou encore rsoudre lquation
argsh x = y dinconnue x et de paramtre y).
d) Etudier la drivabilit de argsh et dterminer sa drive.
2) a) Montrer que ch ralise une bijection de [0, +[ sur un intervalle prciser. On note argch la fonction rciproque
(argument cosinus hyperbolique).
b) Construire le graphe de argch.
c) Dterminer une expression simple de largument cosinus hyperbolique dun nombre.
d) Etudier la drivabilit de argch et dterminer sa drive.
3) a) Montrer que th ralise une bijection de R sur un intervalle prciser. On note argth la fonction rciproque (argument
tangente hyperbolique).
b) Construire le graphe de argth.
c) Dterminer une expression simple de largument tangente hyperbolique dun nombre.
d) Etudier la drivabilit de argth et dterminer sa drive.
Exercice no 6 (**)
Simplifier les expressions suivantes



1) ln
x2 + 1 + x) + ln( x2 + 1 x .
2)

ch(ln x) + sh(ln x)
.
x

3) sh2 x cos2 y + ch2 x sin2 y.


Exercice no 7 (**)
Rsoudre dans R les quations suivantes :
1) ch x = 2
2) ch x =

1
.
2

http ://www.maths-france.fr

c Jean-Louis Rouget, 2014. Tous droits rservs.


3) th x = a, a rel donn.
Exercice no 8 (**)
Calculer

n
X
k=0

ch(ak + b), ((a, b) R2 , n N).

Exercice no 9 (***)
Rsoudre dans R lquation a ch x + b sh x = c en discutant en fonction des paramtres rels a, b et c.

http ://www.maths-france.fr

c Jean-Louis Rouget, 2014. Tous droits rservs.


Planche no 14. Trigonomtrie hyperbolique : corrig


Exercice no 1
ch(a + b) = ch a ch b + sh a sh b
sh(a + b) = sh a ch b + ch a sh b
th a + th b
th(a + b) =
1 + th a th b

et ch(a b) = ch a ch b sh a sh b,
et sh(a b) = sh a ch b sh b ch a
th a th b
et th(a b) =
.
1 th a th b

Deux dmonstrations :

ch a ch b + sh a sh b =

1 a
1
((e + ea )(eb + eb ) + (ea ea )(eb eb )) = (ea+b + eab ) = ch(a + b).
4
2
th(a + b) =

sh(a + b)
sh a ch b + sh b ch a
th a + th b
=
=
ch(a + b)
ch a ch b + sh a sh b
1 + th a th b

aprs division du numrateur et du dnominateur par le nombre non nul ch a ch b.


En appliquant a = b = x, on obtient :
x R, ch(2x) = ch2 x + sh2 x = 2ch2 x 1 = 2sh2 x + 1, sh(2x) = 2 sh x ch x et th(2x) =

2 th x
.
1 + th2 x

En additionnant entre elles les formules daddition, on obtient les formules de linarisation :

ch a ch b =

1
1
1
(ch(a + b) + ch(a b)), sh a sh b = (ch(a + b) ch(a b)) et sh a ch b = (sh(a + b) + sh(a b)),
2
2
2

et en particulier
ch2 x =

ch(2x) + 1
ch(2x) 1
et sh2 x =
.
2
2

Exercice no 2
Drive et variations. Pour tout rel x, ch x > 0.Donc f est dfinie, continue et drivable sur R. Pour tout rel x,
f (x) =

sh x
1 = th x 1 < 0.
ch x

f est donc strictement dcroissante sur R.


Etude en . lim ch x = + et donc lim ln (ch x) = + puis lim f(x) = +. De plus, pour tout rel x,
x

f(x) = ln

x+

ex + ex
x = ln(ex + ex ) ln 2 x = ln(ex ) x ln 2 + ln(1 + e2x ) = 2x ln 2 + ln(1 + e2x ).
2

Donc, pour tout rel x, f(x) (2x ln 2) = ln(1 + e2x ). Or, dune part lim ln(1 + e2x ) = ln 1 = 0 et donc la droite
x

D dquation y = 2x ln 2 est asymptote la courbe reprsentative de f en et dautre part, pour tout rel x,
ln(1 + e2x ) > 0 et la courbe reprsentative de f est strictement au dessus de D sur R.
Etude en +. Pour tout rel x,
f(x) = ln

ex + ex
x = ln(ex + ex ) ln 2 x = ln(ex ) x ln 2 + ln(1 + e2x ) = ln 2 + ln(1 + e2x )
2

et f tend vers ln 2 quand x tend vers +. On en dduit que la droite dquation y = ln 2 est asymptote au graphe de
f en +.

http ://www.maths-france.fr

c Jean-Louis Rouget, 2014. Tous droits rservs.


Graphe.

Cf
2

ln 2

Exercice no 3
Soit x un rel.
S=

100
X

k=1

1
sh(2 + kx) =
2

100
X

kx

k=1

100
X

k=1

kx

Si x = 0 alors directement S = 100 sh 2 6= 0. Si x 6= 0 alors ex 6= 1 et ex 6= 1. Dans ce cas,






100x
100x
100x
100x
1
1
2 x 1 e
2 1 e
2 x1e
2 x1e
S=
e e
+e
e e
e e
=
.
2
1 ex
1 ex
2
1 ex
1 ex

aprs multiplication du numrateur et du dnominateur de la deuxime fraction par le rel non nul ex . Pour x 6= 0, on a
donc :




S = 0 ex+2 (1 e100x + e2 1 e100x = 0 ex+2 1 e100x + e2100x e100x 1 = 0


1 e100x ex+2 e100x2 = 0 ex+2 = e100x2 (car x 6= 0)
4
.
x + 2 = 100x 2 x =
101


4
S =
.
101
Exercice no 4
2 th x
1 + th2 x
2
ce
qui
scrit
pour
x
non
nul
:
=
2
th
x
th(2x)
1 + th x

On a vu dans lexercice no 1 que pour tout rel x, th(2x) =


ou encore th x +

2
1
=
ou finalement
th x
th(2x)
x R , th x =

1
2

.
th(2x) th x

Soit n un entier naturel non nul et x un rel non nul. Daprs ce qui prcde,
un =

n
X

2 th(2 x) =

k=0

http ://www.maths-france.fr

n 
X

k=0

2k+1
2k

th(2k+1 x) th(2k x)
2

2n+1
1

(somme tlescopique).
th(2n+1 x) th x
c Jean-Louis Rouget, 2014. Tous droits rservs.

Ensuite, pour x > 0, th(2n+1 x) tend vers 1 quand n tend vers linfini. Donc un tend vers + quand n tend vers + si
x > 0 et vers quand n tend vers + si x < 0.
Exercice no 5
1) a)
et strictement croissante sur R. La fonction sh ralise donc une bijection de ] , +[
 La fonction sh est continue

sur lim sh x, lim sh x =] , +[. sh est donc une bijection de R sur R.
x

x+

b) Graphe de argsh.

3
2
1

1
2
3

c) Soient x et y deux rels.



1 y
e ey ey 2x ey = 0
2
(ey )2 2xey 1 = 0 (aprs multiplication des deux membres par le rel non nul ex )

y = argsh x x = sh y x =

ey est solution de lquation X2 2xX 1 = 0.

Le discriminant rduit de lquation X2 2xX 1 = 0 est = x2 + 1. Ce discriminant est toujours


strictement positif
et

donc lquation X2 2xX 1 = 0 admet deux solutions relles distinctes savoir X1 = x + x2 + 1 et X2 = x x2 + 1.


Le produit de ces deux deux nombres est gal 1. Donc, lun de ces deux nombres est
strictement positif et lautre est
strictement ngatif. Le positif est le plus grand de ces deux nombres savoir X1 = x + x2 + 1. Donc
p
p
p
y = argsh x ey = x + x2 + 1 ou ey = x x2 + 1 ey = x + x2 + 1


p
y = ln x + x2 + 1 .



x R, argsh(x) = ln x + x2 + 1 .

d) sh est drivable sur R et sa drive savoir sh = ch ne sannule pas sur R. On sait alors que argsh est drivable sur
sh(R) = R. De plus, pour tout rel x,



1
x + x2 + 1
1
1
x

=
.
argsh (x) = 1 +
2
2
2
2
2
x +1 x+ x +1
x +1
x+ x +1
x +1
1
x R, argsh (x) =
.
2
x +1
2) a)
et strictement croissante sur R+ . La fonction ch ralise donc une bijection de ] , +[
 La fonction ch est continue

sur

lim sh x, lim sh x =] , +[. sh est donc une bijection de R sur R.

x+

http ://www.maths-france.fr

c Jean-Louis Rouget, 2014. Tous droits rservs.


b) Graphe de argch.
3
2
1

1
2
3

c) Soient x > 1 et y R.

1 y
e + ey ey 2x + ey = 0
2
2
(ey ) 2xey + 1 = 0 (aprs multiplication des deux membres par le rel non nul ex )

y = argch x x = ch y x =

ey est solution de lquation X2 2xX + 1 = 0.

Le discriminant rduit de lquation X2 2xX 1 = 0 est = x2 1 >. Ce discriminant est toujours et donclquation
X2 2xX
+ 1 = 0 admet deux solutions relles (ventuellement confondues si x = 1) savoir X1 = x + x2 1 et
X2 = x x2 1. Le produit de ces deux deux nombres est gal 1 et leur somme est gale 2x > 0. Donc, ces deux
nombres sont strictement positifs. Par suite,
p
p
y = argch x ey = x + x2 1 ou ey = x x2 1




p
p
y = ln x + x2 1 ou y = ln x x2 1 .

Comme le produit X1 X2 est gal 1 et que X1 et X2 sont strictement


positifs, lun des deux
savoir X1 est plus

 nombres,

2
2
grand que 1 et lautre, savoir X2 , est dans ]0, 1]. Mais alors, ln x + x 1 > 1 et ln x x 1 6 0 (avec galit
0 si et seulement si x = 1 et dans ce cas, X1 = X2 = 0).



Comme on ne veut retenir que la solution positive, il ne reste que y = ln x + x2 1 .





x > 1, argch(x) = ln x + x2 1 .

d) ch est drivable sur [0, +[ et sa drive savoir ch = sh ne sannule pas sur ]0, +[. On sait alors que argch est
drivable sur ch(]0, +[) =]1, +[ et nest pas drivable en ch(0) = 1. De plus, pour tout rel x > 1,



x
1
x + x2 1
1
1

argch (x) = 1 +
=

=
.
2
2
2
2
2
x 1 x+ x 1
x 1
x+ x 1
x 1
1
.
x > 1, argch (x) =
2
x 1

http ://www.maths-france.fr

c Jean-Louis Rouget, 2014. Tous droits rservs.


3) a)
et strictement croissante sur R. La fonction th ralise donc une bijection de ] , +[
 La fonction th est continue

sur lim th x, lim th x =] 1, 1[. th ralise donc une bijection de R sur ] 1, 1[.
x

x+

b) Graphe de argth.
3
2
1

1
2
3

c) Soient x ] 1, 1[ et y R.
y = argth x x = th y

(ey ey ) /2
= x ey ey = xey + xey
(ey + ey ) /2

e2y 1 = xe2y + x (aprs multiplication des deux membres par le rel non nul ey )
1+x
(1 x)e2y = 1 + x e2y =
1x


1
1+x
y = ln
.
2
1x
1
x R, argth(x) = ln
2

d) th est drivable sur [0, +[ et sa drive savoir th =


1+x
.
1x

1
ne sannule pas sur R. On sait alors que argth est drivable
ch2

sur th(R) =] 1, 1[. De plus, pour tout rel x de ] 1, 1[,




1
1
1
1

=
.

argth (x) =
2 1+x 1x
1 x2
x ] 1, 1[, argth (x) =
Exercice no 6
1) Soit x R. x2 + 1 > 0 et donc
Donc,

1
.
1 x2

x2 + 1 existe puis
p

x2 + 1 > x2 = |x| = Max{x, x}.

x2 + 1 + x > 0 et x2 + 1 x > 0. Lexpression propose existe pour tout rel x. De plus,


p
 p

p
p

ln( x2 + 1 + x) + ln( x2 + 1 x) = ln
x2 + 1 + x
x2 + 1 x = ln x2 + 1 x2 = ln 1 = 0.

2) Pour x > 0,

ch(ln x) + sh(ln x)
1
1
1
=
(x + + x ) = 1.
x
2x
x
x
http ://www.maths-france.fr

c Jean-Louis Rouget, 2014. Tous droits rservs.


3) Soit x et y deux rels.


sh2 x cos2 y + ch2 x sin2 y = sh2 x cos2 y + (1 + sh2 x) sin2 y = sh2 x + sin2 y.
Exercice no 7
1) Soit x R.
ch x = 2


1 x
e + ex = 2 ex 4 + ex = 0
2
(ex )2 4ex + 1 = 0 (aprs multiplication par le rel non nul ex )
ex est solution de lquation X2 4X + 1 = 0.

2
2
Le discriminant rduit de cette
quationest = (2) 1 = 3. Lquation X 4X + 1 = 0 admet donc deux solutions
relles distinctes savoir 2 + 3 et 2 3. Ces deux nombres sont strictement positifs et donc





ch x = 2 ex = 2 + 3 ou ex = 2 3 x = ln 2 + 3 ou x = ln 2 3 .

Remarque. La fonctionch est paire


et donc
les deux nombres obtenus sont ncessairement opposs lun de lautre. Cest


effectivement le cas car 2 + 3 2 3 = 4 3 = 1 et donc






1

= ln 2 + 3 .
ln 2 3 = ln
2+ 3
1
2) Pour tout rel x, ch x > 1. En particulier, pour tout rel x, ch x 6= . Lquation propose na pas de solution dans R.
2
Exercice no 8

Soient a et b deux rels et n un entier naturel.


n
X

k=0

1er cas. Si a = 0,

1
ch(ak + b) =
2

n
X

ch(ak + b) =

k=0

n
X

ak+b

k=0
n
X

n
X

akb

k=0

1
=
2

n
X

a k

(e ) + e

k=0

n
X

k=0


a k

ch(b) = (n + 1) ch(b).

k=0

2me cas. Si a 6= 0, alors ea 6= 1 et ea 6= 1 puis


n
X



(n+1)a
e(n+1)a 1
b 1 e
eb
+
e
ea 1
1 ea


(n+1)a/2
(n+1)a
a e
e(n+1)a/2
e(n+1)a/2 e(n+1)a/2
1
b (n+1)a
+a
2
2
eb+ 2 2
+
e
=
2
ea/2 ea/2
ea/2 ea/2


 


(n + 1)a
na
(n + 1)a
sh
ch
sh
+b
na 
1 b+ na
2
2
2
2 + eb 2
a
a
=
.
=
e
2
sh
sh
2
2

ch(ak + b) =

k=0

Exercice no 9

1
2

Soient a, b et c trois rels. Soit x un rel.




a ch x + b sh x = c a ex + ex + b ex ex = 2c (a + b)ex 2c + (a b)ex = 0
2

(a + b) (ex ) 2cex + (a b) = 0 (aprs multiplication des deux membres par le rel non nul ex

ex solution de lquation (a + b)X2 2cX + (a b) = 0.

1er cas. Si b = a, lquation scrit 2cex + 2a = 0 ou encore cex = a.


- Si c = a = 0(= b), tout rel est solution.
- Si c = 0 et a = b 6= 0, lquation na pas de solution.
a
- Si c 6= 0 et 6 0, lquation na pas de solution.
c
http ://www.maths-france.fr

c Jean-Louis Rouget, 2014. Tous droits rservs.


- Si c 6= 0 et

c
a
.
> 0, lquation a une solution et une seule savoir ln
c
a

2me cas. Si b 6= a, lquation (a + b)X2 2cX + (a b) = 0 est du second degr. Son discriminant rduit est
= c2 (a + b)(a b) = c2 + b2 a2 .
- Si c2 + b2 a2 < 0, lquation (a + b)X2 2cX + (a b) = 0 na pas de solution dans R et donc lquation
a ch x + b sh x = c na pas de solution dans R.
c
.
- Si c2 + b2 a2 = 0, lquation (a + b)X2 2cX + (a b) = 0 admet une solution double savoir
a+b
c
6 0, lquation a ch x + b sh x = c na pas de solution dans R.
Si
a+b


c
c
Si
.
> 0, lquation a ch x + b sh x = c a une solution et une seule dans R savoir ln
a+b
a+b
- Si c2 + b2 a2 > 0, lquation (a + b)X2 2cX + (a b) = 0 admet deux solutions relles distinctes dont le produit
ab
2c
est gal
et la somme est gale
.
a+b
a+b
Si a2 b2 < 0, lquation (a + b)X2 2cX + (a b) = 0 a une solution strictement ngative et une solution
strictement positive. Dans ce cas, lquation a ch x + b sh x = c a une solution et une seule.
Si a2 b2 > 0, lquation (a + b)X2 2cX + (a b) = 0 a deux solutions non nulles distinctes et de mme signe.
Si c(a + b) < 0, lquation (a + b)X2 2cX + (a b) = 0 a deux solutions strictement ngatives et dans
ce cas lquation a ch x + b sh x = c na pas de solution.
Si c(a + b) > 0, lquation (a + b)X2 2cX + (a b) = 0 a deux solutions strictement positives et dans
ce cas lquation a ch x + b sh x = c a deux solutions distinctes.
c
. Dans ce cas,
Si a2 b2 = 0, lquation (a + b)X2 2cX + (a b) = 0 a une solution gale 0 et lautre
a+b
lquation a ch x + b sh x = c a une solution et une seule si c(a + b) > 0 et pas de solution si c(a + b) 6 0.

http ://www.maths-france.fr

c Jean-Louis Rouget, 2014. Tous droits rservs.


Planche no 15. Calculs de primitives et dintgrales


* trs facile ** facile *** difficult moyenne **** difficile
I : Incontournable T : pour travailler et mmoriser le cours
Exercice no 1. (T) (utilisation dun formulaire de primitives)
Calculer les primitives des fonctions suivantes sans se soucier de lintervalle :

1
2
1
1
4
1
1) 3x3 7x 3 x + 3 x 7 + 2 + 3 + 4
4
x
4x
4x
x x x
x
x
3)
3
(x2 1)
2x + 1
5) 
2
3
x2 + x + 1
1
7)
1 + 4x2
1
9) 2
x +x+1
1
11)
1 + ex
13)

sin2 (x/2)
3

(x sin x)

2) (x 1)ex

2x

4) (x2 + 3) x3 + 9x 5
2
6)
1 4x2
1
4 + x2
1
10)
x ln x
sin2 (x/2)
12)
x sin x
 x x
ln x
14)
e
8)

Exercice no 2. (T) (intgration par parties pour trouver des primitives)


Calculer les primitives des fonctions suivantes sans se soucier de lintervalle :
1) ln x
6) Arccos x

2) x ln x
7) xex

11) eArccos x

12) cos x ln(1 + cos x)

15) eax cos(x) ((a, ) (R )2 )

16) sin(ln x) et cos(ln x)

3) ln(x + 1)

8) x2 3x + 1 ex
xex
13)
(x + 1)2
17) x2 ex sin x

4) Arcsin x
9) (1 x)e2x

5) Arctan x
10) ln(1 + x2 )

14) xn ln x (n N)

18) 1 x2

Exercice no 3. (T) (primitives de fonctions du type x 7 1/(ax2 + bx + c))

Calculer les primitives des fonctions suivantes sans se soucier de lintervalle :


1)

1
2x2 + 5x + 2

2)

1
4x2 4x + 1

3)

1
x2 + 2x + 2

4)

1
x2 + x + 1

5)

1
,
/ Z.
x2 2x cos + 1

Exercice no 4. (T) (fractions rationnelles en sinus, cosinus et tangente)


Calculer les primitives des fonctions suivantes sans se soucier de lintervalle :
1
sin x
cos(3x)
5)
sin x + sin(3x)

1)

2)
6)

1
cos x
cos4

3)

1
x + sin4 x

1
tan x

4)

1
2 + sin2 x

Exercice no 5. (I)
Z /2
Z /2
cos x
cos x
dx et J =
dx. Calculer I et J.
On pose I =
cos
x
+
sin
x
cos
x
+ sin x
0
0
Exercice no 6. (T) (ch, sh et th ...)
Calculer les primitives des fonctions suivantes sans se soucier de lintervalle :
1)

1
ch x

5) ch4 x
9)
http ://www.maths-france.fr

1
sh x
ch3 x
6)
1 + sh x
2)

1
th x

7) ch x 1

3)

4) ch3 x
8)

th x
1 + ch x

1
1 ch x
1

c Jean-Louis Rouget, 2014. Tous droits rservs.


Exercice no 7. (T) (avec des racines)


Calculer les primitives des fonctions suivantes sans se soucier de lintervalle :
1
1)
(transformation canonique).
2x x2

(quantit conjugue puis poser u = 1 + x et v = 1 x).


2)
1+x+ 1x

1 + x6
(poser u = x6 et v = 1 + u).
3)
x
Exercice no 8. (I)
Calculer les intgrales suivantes (a, b rels donns, p et q entiers naturels donns)
Za
ln x
1)
dx (a > 0)
2+1
x
Z1/a
Z
Z

2)
2 cos(px) cos(qx) dx et
2 cos(px) sin(qx) dx et
2 sin(px) sin(qx) dx ((p, q) N2 ).
0
0
0
Zb p
3)
(x a)(b x) dx
Za2
4)
(|x 1| + |x| + |x + 1| + |x + 2|) dx

Z2
2 
1
1 + 2 Arctan x dx
5)
x
Z1/2

x sin x
6)
2
0 1 + cos x
Exercice no 9.
Calculer f(x) =

Z1

Max(x, t) dt. Reprsenter graphiquement la fonction f.

Exercice no 10. (I) (Intgrales de Wallis)


Z /2
Pour n entier naturel, on pose Wn =
sinn x dx.
0

1) Calculer W0 et W1 .
2) Dterminer une relation entre Wn et Wn+2 .
3) En dduire W2n et W2n+1 en fonction de n.
Exercice no 11. (I)
Pour n entier naturel, on pose In =

Z /4

tann x dx.

1) Calculer I0 et I1 . Trouver une relation entre In et In+2 . En dduire In en fonction de n.


2) Montrer que In tend vers 0 quand n tend vers +, et en dduire les limites des suites (un ) et (vn ) dfinies par :
un =

n
n
X
X
(1)k1
(1)k1
(n N ) et vn =
.
k
2k 1

k=1

http ://www.maths-france.fr

k=1

c Jean-Louis Rouget, 2014. Tous droits rservs.


Planche no 15. Calculs de primitives et dintgrales : corrig


Exercice no 1.
1) Pour tout rel x de ]0, +[,

1
2
1
1
1
4
1
3x3 7x 3 x + 3 x 7 + 2 + 3 + 4 = 3x3 4x4/3 + 3x1/2 4x4/7
4
x
x
4x
4x
x
x
x
x
+ 2x3/2

1
1
1
+ 3 + 4,
2
x
4x
4x

et donc les primitives sur ]0, +[ de la fonction considre sont les fonctions de la forme

ou encore

x 7
x 7

3x4
1
4x7/3
3x3/2
x3/7
2x1/2
1
1
+ C, C R,

+
ln x
+
+ 2
4
7/3
3/2
3/7
1/2
x 8x
12x3

7

7 3x
1
3x4
1
12x2 3 x
4
1
+ C, C R.

+ 2x x ln x
+ 2
4
7
3
12x3
x x 8x

1
2
2
(2x 2)ex 2x et donc les primitives sur R de la fonction x 7 (x 1)ex 2x sont
2
1 2
les fonctions de la forme x 7 ex 2x + C, C R.
2
2

2) Pour tout rel x, (x 1)ex

2x

3) Soit I un intervalle sur lequel x2 1 ne sannule pas.


1
x
2x
x
=
et donc les primitives sur I de la fonction x 7
sont les fonctions
Pour tout rel x de I,
2
2
2
2 (x2 1)
(x2 1)
(x2 1)
1
+ C, C R.
de la forme x 7
2 (x2 1)

4) Soit I un intervalle sur lequel x3 + 9x 5 est positif.


1/2

1
et donc les primitives sur I de la fonction
Pour tout rel x de I, x2 + 3 x3 + 9x 5 = (3x2 + 9) x3 + 9x 5
3

3/2
3/2

1 x3 + 9x 5
2 3
x + 9x 5
+ C, C R.
x 7 x2 + 3 x3 + 9x 5 sont les fonctions de la forme x 7
+C=
3
3/2
9
2/3
2x + 1
2
5) Pour tout rel x, 
et donc les primitives sur R de la fonction
2 = (2x + 1) x + x + 1
3
x2 + x + 1

1/3
2x + 1
1
x2 + x + 1
+ C = 3 3 x2 + x + 1 + C, C R.
x 7 
2 sont les fonctions de la forme x 7
1/3
3
x2 + x + 1




2
2
2
1 1
1 1

p
de la fonction x 7
=
6) Pour tout rel x de , ,
et donc les primitives sur ,
2
2
2 2
2
2
1 4x
1 4x2
1 (2x)
sont les fonctions de la forme x 7 Arcsin(2x) + C, C R.

7) Pour tout rel x,

1
1
2
1
=
et donc les primitives sur R de la fonction x 7
sont les fonctions
2
2
1 + 4x
2 1 + (2x)
1 + 4x2

1
Arctan(2x) + C, C R.
2
1
1
1
= 2
et donc les primitives sur R de la fonction x 7
sont les fonctions de la forme
8) Pour tout rel x,
2
2
4+x
2 +x
4 + x2


1
x
x 7 Arctan
+ C, C R.
2
2
1
1
1
9) Pour tout rel x, 2
= 
!2 et donc les primitives sur R de la fonction x 7 x2 + x + 1 sont
2
x +x+1
3
1
+
x+
2
2

1
x
+
1

Arctan 2 + C, C R, ou encore les fonctions de la forme


les fonctions de la forme x 7
3/2
3/2
de la forme x 7

http ://www.maths-france.fr

c Jean-Louis Rouget, 2014. Tous droits rservs.


2
x 7 Arctan
3

2x + 1

+ C, C R.

1/x
1
=
et donc les primitives sur R de la
10) I est lun des deux intervalles ]0, 1[ ou ]1, +[. Pour tout rel x de I,
x ln x
ln x
1
sont les fonctions de la forme x 7 ln |ln(x)| + C, C R.
fonction x 7
x ln x
1
ex
1
1
11) Pour tout rel x de R,
= x
=
sont les
et donc les primitives sur R de la fonction x 7
x
1
1+e
e +1
1 + ex
1+ x
e
fonctions de la forme x 7 ln (1 + ex ) + C, C R.

12) I dsigne un intervalle sur lequel x sin x ne sannule pas. Pour tout rel de I,
primitives sur I de la fonction x 7

sin2 (x/2)
1
sont les fonctions de la forme x 7 ln |x sin x| + C, C R.
x sin x
2

13) I dsigne un intervalle sur lequel x sin x ne sannule pas. Pour tout rel de I,
les primitives sur I de la fonction x 7
14)

sin2 (x/2)
1 1 cos x
=
et donc les
x sin x
2 x sin x

sin2 (x/2)
3

(x sin x)

sin2 (x/2)

(x sin x)3

sont les fonctions de la forme x 7

1
1 cos x
et donc

2 (x sin x)3

1
2

4 (x sin x)

, C R.

Z  x
Z
 x x
x
ln x dx = (x ln x x) ex ln xx dx = ex ln xx + C =
, C R.
e
e

Exercice no 2.
Z
Z
Z
1
1) ln x dx = x ln x x dx = x ln x 1 dx = x ln x x + C, C R.
x
Z
Z 2
Z
2
x
x
1
x2
1
x2
x2
2) x ln x dx =
ln x
dx =
ln x
ln x
+ C, C R.
x dx =
2
2
x
2
2
2
4
Z
Z
Z
1
dx = (x + 1) ln(x + 1) 1 dx = (x + 1) ln(x + 1) x + C, C R.
3) ln(x + 1) dx = (x + 1) ln(x + 1) (x + 1)
x+1
Z
Z
p
x
4) Arcsin x dx = x Arcsin x
dx = x Arcsin x + 1 x2 + C, C R.
1 x2
Z
Z
x
1
5) Arctan x dx = x Arctan x
dx = x Arctan x ln(1 + x2 ) + C, C R.
1 + x2
2
Z
Z
p
x
dx = x Arccos x 1 x2 + C, C R.
6) Arccos x dx = x Arccos x +
1 x2
Z
Z
7) xex dx = xex + ex dx = xex ex + C = (x + 1)ex + C, C R.
8)
Z

Z
Z
 x
 x
 x
2
x
2
x
x 3x + 1 e dx = x 3x + 1 e (2x 3)e dx = x 3x + 1 e (2x 3)e + 2 ex dx


= x2 3x + 1 ex (2x 3)ex + 2ex + C = x2 5x + 6 ex + C, C R.
2

Z
e2x 1
1
1
1
9) (1 x)e
dx = (1 x)

e2x dx = (x 1)e2x + e2x + C = (2x 1)e2x + C, C R.


2
2
2
4
4
Z 2
Z


x +11
2
2
10) ln 1 + x dx = x ln 1 + x 2
dx = x ln(1 + x2 ) 2x + 2 Arctan x + C, C R.
x2 + 1
2x

11)

et donc,

eArccos x

eArccos x dx
1 x2
Zp
p
1
Arccos x
Arccos x
2
+
1 x2
eArccos x dx
= xe
1x e
1 x2

p
1
x 1 x2 eArccos x + C, C R.
dx =
2
eArccos x dx = xeArccos x +

http ://www.maths-france.fr

c Jean-Louis Rouget, 2014. Tous droits rservs.


12)
Z
cos2 x 1
sin x
dx = sin x ln(1 + cos x)
dx
cos x ln(1 + cos x) dx = sin x ln(1 + cos x) sin x
1 + cos x
cos x + 1
Z
= sin x ln(1 + cos x) (cos x 1) dx = sin x ln(1 + cos x) sin x + x + C, C R.



Z
x
xex
x+11 x
1 x
ex
1
1 x
x
x
13)
e
=
e
=
e
=
dx
=
e

e
+ C, C R.
et
donc
(x + 1)2
(x + 1)2
x+1
(x + 1)2
x+1
(x + 1)2
x+1
Z
Z
xn+1
1
xn+1
xn+1
n
14) x ln x dx =
+ C, C R.
ln x
ln x
xn dx =
n+1
n+1
n+1
(n + 1)2
15) 1re solution.
Z

et donc

2
1+ 2
a

Z

ax

1
cos(x) dx = eax cos(x) +
a
1
= eax cos(x) +
a

eax cos(x) dx =
Z

eax sin(x) dx
a
Z
2
ax
e sin(x) 2 eax cos(x) dx
a2
a

1
(a cos(x) + sin(x)) eax + C, C R puis
a2

eax cos(x) dx =

a2

1
(a cos(x) + sin(x)) eax + C, C R.
+ 2

2me solution.
Z

eax cos(x) dx = Re
=

16)

Z

e(a+i)x dx

= Re

e(a+i)x
a + i

+C=

eax
Re((a i)(cos(x) + i sin(x)) + C
+ 2

a2

eax (a cos(x) + sin(x))


+ C, C R.
a2 + 2
Z

sin(ln x) dx = x sin(ln x) cos(ln x) dx = x sin(ln x) x cos(ln x) sin(ln x) dx et donc


Z

17)

x2 ex sin x dx = Im
Z

Par suite,

2 (1+i)x

x e

Z

sin(ln x) dx =

x
(sin(ln x) cos(ln x)) + C, C R.
2


x2 e(1+i)x dx . Or,
(1+i)x
2e

1+i
1+i

(1+i)x
2e

1+i
1+i

(1+i)x

xe
dx = x
dx = x

 2
2
2x
x
e(1+ix) + C
+

=
2
1 + i (1 + i)
(1 + i)3


1i 2
1+i
=
e(1+ix) + C
x + ix
2
2

1 2
=
x 1 + i(x2 + 2x 1)) (cos x + i sin x)ex + C.
2
Z

x2 ex sin x dx =

http ://www.maths-france.fr

 (1+i)x

Z
e
1
(1+i)x
x

e
dx
1+i
1+i


1
(x2 1) cos x + (x2 2x + 1) sin x ex + C, C R.
2

c Jean-Louis Rouget, 2014. Tous droits rservs.


18) Sur ] 1, 1[,


Z
Zp
p
1 x2 dx = x 1 x2
Z
p
= x 1 x2 +

Z
p
x2
2x
x
dx = x 1 x2 +
dx
2 1 x2
1 x2
Z
Z
p
x2 1 + 1
1 x2
1

dx = x 1 x2
dx +
dx
2
2
1x
1x
1 x2
Z
p
p
1 x2 dx.
= x 1 x2 + Arcsin x

Les primitives sur ] 1, 1[ de la fonction x 7


C R.

1
x 1 x2 + Arcsin x + C,
1 x2 sont les fonctions de la forme x 7
2

Exercice no 3.
1) Soit I un intervalle ne contenant ni

2x2

1
ni 2. Dterminons deux rels a et b tels que pour tout rel x de I,
2

1
=
+ 5x + 2

1


2(x + 2) x +

1
2

 =

a
+
x+2

b
x+

1
2

Pour tout rel x de I,


a
+
x+2

b
1
x+
2

2b
a(2x + 1) + 2b(x + 2)
2(a + b)x + (a + 4b)
a
+
=
=
.
x + 2 2x + 1
2x2 + 5x + 2
2x2 + 5x + 2

1
1
On choisit a et b tels que 2(a + b) = 0 et a + 4b = 1 cest--dire a = et b = . Pour tout x de I, on a
3
3

1
1
1
=
+
2x2 + 5x + 2
3
x+2

.
1
x+
2

1
1
Les primitives sur I de la fonction x 7 2
sont les fonctions de la forme x
7
2x + 5x + 2
3
C R.
1
2) Soit I un intervalle ne contenant pas . Pour tout rel x de I,
2
1
1
.
=
2
4x 4x + 1
(2x 1)2
Les primitives sur I de la fonction x 7

4x2

3) Pour tout rel x,



ln x +



1
ln |x + 2| + C,
2

1
1
sont les fonctions de la forme x 7
+ C, C R.
4x + 1
2(2x 1)
1
1
=
.
x2 + 2x + 2
(x + 1)2 + 1

Les primitives sur R de la fonction x 7

4) Pour tout rel x,

x2

1
sont les fonctions de la forme x 7 Arctan(x + 1) + C, C R.
+ 2x + 2
x2

1
1
=

2
+x+1
1
+
x+
2

!2 .
3
2

1
x
+

1
1
2
+ C, C R
Arctan
sont les fonctions de la forme x 7
Les primitives sur R de la fonction x 7 2

x +x+1
3/2
3
2


2
2x + 1

+ C, C R.
ou encore x 7 Arctan
3
3

http ://www.maths-france.fr

c Jean-Louis Rouget, 2014. Tous droits rservs.


5) Pour tout rel x,


1
1
=
.
x2 2x cos + 1
(x cos )2 + (sin )2
Les primitives sur R de la fonction x 7

C R.

1
1
sont les fonctions de la forme x 7
Arctan
x2 2x cos + 1
sin


x cos
+ C,
sin

Exercice no 4.
2dt
x
,
1) 1re solution. En posant t = tan et donc dx =
2
1 + t2
Z
Z
Z

1
1 + t2 2dt
1
x

dx =
=
dt
=
ln
|t|
+
C
=
ln
tan
+ C, C R.
sin x
2t 1 + t2
t
2
2 me solution.

sin x
sin x
1
=
=
. En posant t = cos x et donc dt = sin x dx
2
sin x
1

cos2 x
sin x
Z

Z
Z
1
sin x dx
1
dt =
=
dt
2
2
2
1 cos x
1t
t 1

Z
1
1
1
1
dt = (ln |t 1| ln |t + 1|) + C
=

2
t1 t+1
2


2 x



1 1 cos x
x
1
1 2 sin 2
= ln
+ C = ln tan2 + C
+ C = ln


x
2
1 + cos x
2 2 cos2
2
2
2

x

= ln tan + C, C R.
2

1
dx =
sin x

2) 1re solution. On pose t = tan


Z

x
2dt
.
et donc dx =
2
1 + t2

Z
2
1 + t2 2dt
=
dt
2
2
1t 1+t
1 t2


Z
1
1
dt = ln |1 + t| ln |1 t| + C
=
+
1t 1+t




tan + tan x
1 + t

4
2 + C
+ C = ln
= ln
x

1t
1 tan tan
4
2

 x 


= ln tan
+
+ C.
2
4

1
dx =
cos x

2me solution. En posant t = sin x


Z



Z
1 + sin x
cos x
1
+ C...

dx =
dx = ... = ln
cos x
1 sin x
1 sin2 x

3me solution. 2n posant u = x + ,


2
Z
Z
Z


 x 
1
1
1
u





+
C
=
ln
dx =
du
=
ln
+
du
=

+ C.
tan
tan

cos x
sin
u
2
2
4
cos u
2
Z
Z
dx
cos x
3)
=
dx = ln | sin x| + C, C R.
tan x
sin x
Z
Z
Z
Z
1
dx
1
1
dx
dx
1


dx
=

=
=
.
4)
2
2
2
2x
2x
2x
2
cos
cos
cos
2
1
+
tan
x
+
tan
x
2
+
3
tan
x
2 + sin2 x
2
+ tan x
cos2 x
En posant u = tan x, on obtient
!
r
r
r !
Z
Z
1
3
3
1 3
1
1
dx =
du =
Arctan
u + C = Arctan
tan x ) + C, C R.
2 + 3u2
3 2
2
2
2 + sin2 x
6
http ://www.maths-france.fr

c Jean-Louis Rouget, 2014. Tous droits rservs.


5)
1 4 cos3 x 3 cos x
1 4 cos3 x 3 cos x
4 cos3 x 3 cos x
cos(3x)
=
=
dx =
3
2
sin x + sin(3x)
4
4
sin x cos2 x
4 sin x 4 sin x
sin x(1 sin x)


cos x 3
3
1
1 4 cos x
=


.
=
4
sin x
sin x cos x
sin x 2 sin(2x)
Par suite,
Z

3
cos(3x)
dx = ln | sin x| ln | tan x| + C, C R.
sin x + sin(3x)
4

5) cos4 x + sin4 x = (cos2 x + sin2 x)2 2 sin2 x cos2 x = 1


Z

1
dx =
4
cos x + sin4 x

dx =

1
1 sin2 (2x)
2
Z
Z
1
du
=
=
1 + cos2 u

1
sin2 (2x), et donc
2
1
du (en posant u = 2x)
2 sin2 u
1

dv
(en posant v = tan u)
1
1 + v2
1+
1 + v2




Z
dv
1
1
v
tan(2x)

=
= Arctan
+ C = Arctan
+ C, C R.
v2 + 2
2
2
2
2
Exercice no 5.
h i
h i
Pour tout x de 0, , cos x + sin x > 0 avec galit si et seulement si cos x = sin x = 0. Donc, pour tout x de 0, ,
2
2
h i
cos x + sin x > 0 et en particulier, pour tout x de 0, , cos x + sin x 6= 0.
2
h i
sin x
cos x
en tant que quotient de fonctions
et x 7
sont continues sur 0,
Ainsi, les deux fonctions x 7
cos x + sin x
h cos
i x + sin x
h i 2
continues sur le segment 0,
dont le dnominateur ne sannule pas sur 0, . On en dduit que les intgrales I et J
2
2
existent.
Z /2
Z /2

sin x + cos x
/2
dx = . et I J =
I+J=
dx = [ln | cos x + sin x|]0 = 0. Donc
2
cos
x
+
sin
x
0
0
I=J=

.
2

Exercice no 6.
dt
1) En posant t = ex et donc x = ln t puis dx =
,
t
Z
Z
Z
1
2 dt
1
dt = 2 Arctan(ex ) + C,
dx =
=2
1 t
ch x
1 + t2
t+
t
ou bien
Z
Z
ch x
1
dx =
dx = Arctan(sh x) + C.
2
ch x
sh x + 1
Remarque. Les deux fonctions x 7 2 Arctan (ex ) et x 7 Arctan (sh x) sont deux primitives sur R de la fonction
1
. Elles diffrent donc dune constante. Comme ces deux primitives prennent la mme valeur en 0, on a donc
x 7
ch x
x R, 2 Arctan(ex ) = Arctan(sh x).

2) En posant t = ex et donc x = ln t puis dx =

http ://www.maths-france.fr

dt
,
t

c Jean-Louis Rouget, 2014. Tous droits rservs.


1
dx =
sh x

dt
=2
1 t
t
t

1
dt =
2
t 1

Z

1
1

t1 t+1

dt


x/2
x

e
e 1
ex/2



= ln |t 1| ln |t + 1| + C = ln x
+ C = ln x/2
+C
e + 1
e
+ ex/2
x


= ln th + C, C R.
2

3)

1
dx =
th x

4)

ch3 dx =

Z

1
sh2 x + 1 ch x dx = sh3 x + sh x + C, C R.
3

ch x
dx = ln |sh x| + C, C R.
sh x

 1
1 x
1 4x
4
(e + ex ) =
e + 4e2x + 6 + 4e2x + e4x = (ch(4x) + 4 ch(2x) + 3) et donc
16
16
8
Z
1
ch4 x dx =
(sh(4x) + 8 sh(2x) + 12x) + C, C R.
32

5) Pour tout rel x, ch4 x =

6)
Z

ch3 x
dx =
1 + sh x

1 + sh2 x
ch x dx
1 + sh x
Z 2
u +1
=
du (en posant u = sh x)
u+1
Z 2
u 1+2
=
du (en posant u = sh x)
u+1
Z
u2
2
) du =
u + 2 ln |u + 1| + C
= (u 1 +
u+1
2
=

sh2 x
sh x + 2 ln |1 + sh x| + C, C R.
2

7) On peut poser u = ex mais il y a mieux.


Z

Z p
Z p 2
Z
(ch x 1)(ch x + 1)
sh x
sh x

ch x 1 dx =
dx =
dx = sgn(x)
dx
ch x + 1
ch x + 1
ch x + 1

= 2sgn(x) ch x + 1 + C.

8)
Z

th x
dx =
ch x + 1

1
sh x dx
ch x(ch x + 1)
Z
1
=
du (en posant u = ch x)
u(u + 1)

Z
ch x
1
1
du = ln

+ C, C R.
=
u u+1
ch x + 1

9)
Z

http ://www.maths-france.fr

1 + ch x
dx =
1 ch2 x
1
= cothx +
+ C.
sh x

1
dx =
1 ch x

1
dx
sh2 x

ch x
dx
sh2 x

c Jean-Louis Rouget, 2014. Tous droits rservs.


Exercice no 7.
1) En posant u = x 1,
Z
Z
Z
1
1
1

dx = p
du = Arcsin u + C = Arcsin(x 1) + C, C R.
dx =
2
2
2x x
1 u2
1 (x 1)

2)

Z

Z
Z
1
1+x 1x
1+x
1x
dx =
dx
dx
(1 + x) (1 x)
2
x
x
Z

Z

1
v
u
=
2v dv (en posant u = 1 + x et v = 1 x)
2u du +
2
2
2
u 1
1v
Z 2
Z 2
u 1+1
v 1+1
=
dv
du +
u2 1
1 v2




Z
Z
1
1
1
1
1
1
1
du +
dv

1+
=
2 u1 u+1
2 v1 v+1




1 u


1
+ ln 1 + v + C
= uv+
ln


2
1+u
1 v






1 1 + x


1
+ ln 1 + 1 x + C.

ln
= 1+x 1x+


2
1+ 1+x
1 1 x

dx =
1+x+ 1x

3) On pose u = x6 puis v =
Z

1 + u (ou directement u = 1 + x6 ) et on obtient :

Exercice no 8.
1) On pose t =

Z
Z
1 + x6 5
1+u
1
x dx =
du
6
x
6
u
Z 

 
Z
Z
1
1
v
v2
1
1
1
1
1
+
dv
2v
dv
=
dv
=

=
6 v2 1
3 v2 1
3
2 v1 v+1




1
1 v 1
=
v + ln
+C
3
2
v + 1

!
1 + x6 1
1
1 p


=
1 + x6 + ln
+ C, C R.
3
2 1 + x6 + 1

1 + x6
dx =
x

1
1
1
et donc x = et dx = 2 dt. On obtient
x
t
t
Za
Z 1/a
Za
ln t
ln(1/t) 1
ln x
dt =
dx =
dt = I,
I=
2+1
2
2+1
1
x
t
t
1/a
a
1/a
+
1
t2

et donc, I = 0.
1
(cos(p + q)x + cos(p q)x) et donc,
2
Premier cas. Si p 6= q,


Z
1 sin(p + q)x sin(p q)x
cos(px) cos(qx) dx =
+
= 0.
2
p+q
pq
0
0

2) cos(px) cos(qx) =

Deuxime cas. Si p = q 6= 0,
Z
0

Troisime cas. Si p = q = 0.

Z
0

http ://www.maths-france.fr

1
2

cos(px) cos(qx) dx =

cos(px) cos(qx) dx =

(1 + cos(2px)) dx =

1
2

Z
0

dx =

.
2

dx = .

c Jean-Louis Rouget, 2014. Tous droits rservs.


La dmarche est identique pour les deux autres et on trouve


Z
sin(px) cos(qx) dx = 0 pour tout choix de p et q.

Z
0

sin(px) sin(qx) dx = 0 si p 6= q et

si p = q 6= 0 puis
2

3) La courbe dquation y =

p
(x a)(b x) ou encore

x2 + y2 (a + b)x + ab = 0
est le demi-cercle de diamtre
y>0

[AB] o A(a, 0) et B(b, 0).


(b a)2
(b a)2
R2
=
et si a > b, I =
.
Par suite, si a 6 b, I =
2
8
8
4) Lintgrale propose est somme de quatre intgrales. Chacune delles est la somme des aires de deux triangles rectangles
1
isocles. Ainsi, I = ((12 + 32 ) + (22 + 22 ) + (32 + 12 ) + 42 ) = 22.
2
1
5) On pose u = . On obtient
x
Z2 


 
Z 1/2
Z2

1
1 du
1 
2
1 + 2 Arctan x dx =
I=
(1 + u ) Arctan
=
(1
+
)

Arctan
u
du
x
u
u2
u2
2
1/2
2
1/2

 

1
1

2
I.
=
2
2
2
3
3
I et donc I =
.
2
4
7) En posant x = u, on obtient
Par suite, I =

Z0

( u) sin( u)
du =
2
1 + cos ( u)
0
2

I,
= [Arctan(cos u)]0 I =
2

I=

et donc, I =

x sin x
dx =
1 + cos2 x

Z
0

sin u
du
1 + cos2 u

Z
0

u sin u
du
1 + cos2 u

2
.
4

1
Exercice no 8. Soit x R. La fonction t 7 Max(x, t) = (x + t + |x t|) est continue sur [0, 1] en vertu de thormes
2
Z1
Max(x, t) dt existe.
gnraux. Par suite,
0

Si x 6 0, alors t [0, 1], x 6 t et donc Max(x, t) = t. Par suite, f(x) =

Z1

Si x > 1, alors t [0, 1], t 6 x et donc Max(x, t) = x. Par suite, f(x) =

Si 0 < x < 1,

f(x) =

Zx
0

x dt +

Z1
x

Z01

t dt =

1
.
2

x dt = x.

1
1
t dt = x2 + (1 x2 ) = (1 + x2 ).
2
2

si x 6 0

2
1
En rsum, x R, f(x) =
(1 + x2 ) si 0 < x < 1 .

x si x > 1

http ://www.maths-france.fr

c Jean-Louis Rouget, 2014. Tous droits rservs.


Graphe de f.

y = f(x)

Exercice no 9.
Z /2
Z /2

sin x dx = 1.
1) W0 =
dx = et W1 =
2
0
0
h i
2) Soit n N. Les deux fonctions x 7 sinn+1 x et x 7 cos x sont de classe C1 sur le segment 0, . On peut donc
2
effectuer une intgration par parties et on obtient
Wn+2 =

Z /2
0


/2
sin x sinn+1 x dx = cos x sinn+1 x 0

= 0 + (n + 1)

Z /2

Z /2

( cos x)(n + 1) cos x cosn x dx

cos2 x sinn x dx (sinn+1 (0) = 0 car n + 1 > 0)

= (n + 1)

Z /2

Z /2

(1 sin x) sin x dx = (n + 1)

sin x dx

Z /2

n+2

sin

x dx

= (n + 1) (Wn Wn+2 )
et donc (n + 2)Wn+2 = (n + 1)Wn .
n N, Wn+2 =

n+1
Wn .
n+2

3) 1er cas. Si n est un entier naturel non nul et pair, on peut poser n = 2p o p N .
W2p =
=

2p 1 2p 3
3 1

. . . W0
2p
2p 2
4 2
(2p) (2p 1) (2p 2) (2p 3) . . . 3 2 1

((2p) (2p 2) (2p 4) . . . 4 2)


(2p)!
(2p)!

=
=
.
2
2
2
2
(2p p!)
22p (p!)

ce qui reste vrai quand p = 0 et donc


p N, W2p =

(2p)!
22p

(p!)

.
2

2me cas. Si n est un entier naturel impair suprieur ou gal 3, on peut poser n = 2p + 1 o p N .

http ://www.maths-france.fr

10

c Jean-Louis Rouget, 2014. Tous droits rservs.


W2p+1 =

2p 2
4 2
2p

. . . W1
2p + 1 2p 1
5 3
2

((2p) (2p 2) (2p 4) . . . 4 2)


(2p + 1) (2p) (2p 1) . . . 3 2 1
2

22p (p!)
.
(2p + 1)!

ce qui reste vrai quand p = 0 et donc


2

p N, W2p+1 =

22p (p!)
.
(2p + 1)!

Exercice no 10.
Z /4
Z /4

ln 2
sin x
/4
1) I0 =
dx = et I1 =
dx = [ ln | cos x|]0 =
.
4
cos
x
2
0
0
Soit n N.
In + In+2 =

Z /4

n+2

(tan x + tan

x) dx =

Z /4
0

tann+1 x
tan x(1 + tan x) dx =
n+1
n

/4
0

1
.
n+1

Soit n N .
n
n
n
n
X
X
X
X
(1)k1
(1)k1 I2k2 +
(1)k1 I2k
=
(1)k1 (I2k2 + I2k ) =
2k 1

k=1

k=1

k=1

n1
X

(1)k I2k

k=0

Ainsi, n N , I2n = (1)

n
X

k=1

(1)k I2k = I0 (1)n I2n .

k=1

!
n
X (1)k1
.

4
2k 1
k=1

!
n
X
(1)k1
(1)n
(1)
n

ln 2
.
= I1 (1) I2n+1 et donc, n N , I2n+1 =
De mme,
2k
2
k
k=1
k=1
i h
2) Soient 0,
et n N .
2
Z /4/2
Z /4
 

+ .

0 6 In =
tann x dx +
tann x dx 6 tann
4
4
2
2
0
/4/2
 
 
< 1 et donc lim tann
= 0. Par suite, il existe n0 N tel que, pour n > n0 ,

Maintenant, 0 < tan


n+
4
2
4 2
 
0 6 tann

< . Pour n n0 , on a alors 0 6 In < .


4 2
2
Ainsi, In tend vers 0 quand n tend vers +. On en dduit immdiatement que
n
X

k1

(1)k1
= ln 2 et
n+
k
lim

http ://www.maths-france.fr

11

(1)k1
= .
n+ 2k 1
4
lim

c Jean-Louis Rouget, 2014. Tous droits rservs.


Planche no 16. Equations diffrentielles linaires


* trs facile ** facile *** difficult moyenne **** difficile
I : Incontournable T : pour travailler et mmoriser le cours
Exercice no 1 (**IT)
Rsoudre sur lintervalle I de R propos les quations diffrentielles suivantes :
1
I =]0, +[
1 + x2

4) x(xy + y x) = 1, I =] , 0[
6) y + 2y = x2 3x, I = R

1) x ln x y + y = x, I =]1, +[

2) xy + 3y =

3) (1 x)2 y = (2 x)y I =] , 1[
5) 2xy + y = x4 , I =] , 0[
1
7) y + y =
, I=R
1 + 2ex

8) y sin x y cos x + 1 = 0, I =]0, [

Exercice no 2 (**IT)
1) Dterminer la solution sur R de lquation diffrentielle y + y th x = 0 prenant la valeur 1 en 0.
2) Dterminer la solution sur R de lquation diffrentielle y + y th x = x th x prenant la valeur 0 en 0.
Exercice no 3 (***I)
Rsoudre lquation diffrentielle (1 x2 )y 2xy = x2 sur chacun des intervalles I suivants : I =]1, +[, I =] 1, 1[,
I =] 1, +[, I = R.
Exercice no 4 (***)
Rsoudre sur ] , 0[ et sur ]0, +[ lquation diffrentielle : |x|y + (x 1)y = x3 .
Exercice no 5 (**)
Rsoudre sur R les quations diffrentielles :
1) y 2y + 2y = cos x ch x
2) y + 6y + 9y = e2x

2
x
4) y 2ky + (1 + k )y = e sin x, k R \ {1}.

3) y 2y + y = ch x

Exercice no 6 (**IT) (daprs Mines dAls 2005)


On note (E1 ) lquation diffrentielle :
x2 z + xz = z2 .
On cherche les solutions de (E1 ) sur ]1, +[ qui ne sannulent pas sur I =]1, +[.
1
. Vrifier que y est solution sur I dune quation diffrentielle linaire du premier ordre note (E2 ).
z
2) Rsoudre (E2 ) sur I puis dterminer les solutions de (E1 ) sur ]1, +[ qui ne sannulent pas sur I =]1, +[.

1) On pose y =

Exercice no 7 (**IT) (***)


On considre lquation diffrentielle (E) : ax2 y + bxy + cy = 0 (a, b, c rels, a 6= 0) pour x ]0, +[.

1) Soit y une fonction deux fois drivable sur ]0, +[. Pour t R, on pose z(t) = y(et ). Vrifier que y est deux fois
drivable sur ]0, +[ si et seulement si z est deux fois drivable sur R.
2) Effectuer le changement dinconnue prcdent dans lquation diffrentielle (E) et vrifier que la rsolution de (E) se
ramne la rsolution dune quation linaire du second ordre coefficients constants.
3) Rsoudre sur ]0, +[, lquation diffrentielle x2 y xy + y = 0.
Exercice no 8 (***)
Soit a un rel non nul. Soit f continue sur R et priodique de priode T 6= 0. Montrer que lquation diffrentielle y +ay = f
admet une et une seule solution priodique sur R, de priode T .

http ://www.maths-france.fr

c Jean-Louis Rouget, 2014. Tous droits rservs.


Exercice no 9 (**IT) (quelques quations diffrentielles en physique)


Rsoudre :
1) Lquation classique du premier ordre (activit radioactive, cintique chimique du 1er ordre, freinage avec frottement
fluide, circuits RL, ...)

x+

x
x
=
avec x(0) = x0

v
v
=
avec v(0) = v0 ).

2) Equation de la charge dun condensateur

o x et x0 sont deux rels (ou v +

RC

dU
+ U = E avec U(0) = U0 .
dt

3) Oscillateur harmonique
r

k
.
> 0 avec x(t0 ) = x0 et x(t0 ) = v0 .
m
r
k
..
.
2
b) x + 0 x = A o 0 =
> 0 avec x(t0 ) = x0 et x(t0 ) = v0 .
m
a) x + 20 x = 0 o 0 =

..

4) Circuits RLC
q + 2q + 20 q =

..

propre).

E
R
1
.
avec q(0) = 0 et q(0) = 0 ( =
est le coefficient damortissement et 0 =
est la pulsation
L
2L
LC

http ://www.maths-france.fr

c Jean-Louis Rouget, 2014. Tous droits rservs.


Planche no 16. Equations diffrentielles linaires : corrig


Exercice no 1
Les quations diffrentielles rsoudre dans cet exercice sont toutes linaires du premier ordre. On note (E) lquation
diffrentielle propose et (Eh ) lquation homogne associe.
1
1
y=
.
1) Sur I, lquation (E) est quivalente lquation y +
x ln x
ln x
1
1
et x 7
sont continues sur I et on sait que les solutions de (E) sur I sont de la forme f0 + f1
Les fonctions x 7
x ln x
ln x
o f0 est une solution particulire de (E) sur I et f1 est une solution particulire non nulle de (Eh ) sur I.
Soit f une fonction drivable sur I.

1
f solution de (E) sur I x I, x ln x f (x) + f(x) = x x I, ln x f (x) + f(x) = 1
x
x I, (f ln) (x) = 1 R/ x I, f(x) ln x = x +
x+
R/ x I, f(x) =
ln x


x+
, R .
S]1,+[ = x 7
ln x

1
3
.
2) Sur I, lquation (E) est quivalente lquation y + y =
x
x(1 + x2 )
3
1
Les fonctions x 7 et x 7
sont continues sur I et on sait que les solutions de (E) sur I sont de la forme f0 + f1
x
x(1 + x2 )
o f0 est une solution particulire de (E) sur I et f1 est une solution particulire non nulle de (Eh ) sur I.

Soit f une fonction drivable sur I.

x2
1
3
2

I,
x
f
(x)
+
3x
f(x)
=
1 + x2
1 + x2
1
x I, (x3 f) (x) = 1
R/ x I, x3 f(x) = x Arctan x +
1 + x2
x Arctan x +
R/ x I, f(x) =
x3


x Arctan x +
, R .
S]0,+[ = x 7
x3

f solution de (E) sur I x I, xf (x) + 3f(x) =

3) Sur I, lquation (E) est quivalente lquation y

2x
y = 0.
(1 x)2

2x
est continue sur I et on sait que les solutions de (E) sur I sont de la forme f1 o f1 est une
(1 x)2
solution particulire non nulle de (E) sur I.

La fonction x 7

Soit f une fonction drivable sur I.


2x
f(x) = 0
f solution de (E) sur I x I, (1 x)2 f (x) (2 x)f(x) = 0 x I, f (x)
(1 x)2


1+1x
1
1

x I, f (x)
f(x) = 0
f(x) = 0 x I, f (x) +

(1 x)2
(1 x)2 1 x


1
1
1
1
x I, e x1 +ln |1x| f (x) +
e x1 +ln |1x| f(x) = 0

2
(1 x)
1x


1
x I, (1 x)e x1 f (x) = 0
1

R/ x I, (1 x)e x1 f(x) =
1

http ://www.maths-france.fr

e 1x
.
R/ x I, f(x) =
1x
1

c Jean-Louis Rouget, 2014. Tous droits rservs.


S],1[ =


1
e 1x
, R .
x 7
1x

1
1
4) Sur I, lquation (E) est quivalente lquation y + y = 1 + 2 .
x
x
1
1
Les fonctions x 7 et x 7 1 + 2 sont continues sur I et on sait que les solutions de (E) sur I sont de la forme f0 + f1
x
x
o f0 est une solution particulire de (E) et f1 est une solution particulire non nulle de (Eh ).

Soit f une fonction drivable sur I.

f solution de (E) sur I x I, x(xf (x) + f(x) x) = 1 x I, (xf) (x) = x +

1
x

x ln(x) +
x2
+ ln(x) + R/ x I, f(x) = +
.
2
2
x


x ln(x)
= x 7 +
+ , R .
2
x
x

R/ x I, xf(x) =
S],0[

5) Sur I, lquation (E) est quivalente lquation y +

x3
1
y=
.
2x
2

1
x3
et x 7
sont continues sur I =] , 0[ et on sait que les solutions de (E) sur I sont de la forme
2x
2
f0 + f1 o f0 est une solution particulire de (E) et f1 est une solution particulire non nulle de (Eh ).

Les fonctions x 7

Soit f une fonction drivable sur I.

1
x3
f(x) =
2x
2


x3 ln |x|/2
1 ln |x|/2
1
ln |x|/2
f(x) =
e
x I,
x I, e
f (x) + e
x f (x) = (x)7/2
2x
2
2

1
9/2
+
R/ x I, x f(x) = (x)
9
x4

R/ x I, f(x) =
+
9
x

f solution de (E) sur I x I, f (x) +

S],0[ =



x ln(x)
x 7 +
+ , R .
2
x
x

6) Les fonctions x 7 2 et x 7 x2 3x sont continues sur R et on sait que les solutions de (E) sur R sont de la forme
f0 + f1 o f0 est une solution particulire de (E) et f1 est une solution particulire non nulle de (Eh ).
1re solution. Les solutions sur R de (Eh ) sont les fonctions de la forme x 7 e2x .
Dterminons une solution particulire de (E) sur R de la forme x 7 ax2 + bx + c.

puis



ax2 + bx + c + 2 ax2 + bx + c = 2ax + b + 2ax2 + 2bx + 2c = 2ax2 + 2(a + b)x + b + 2c.



x R, ax2 + bx + c + 2 ax2 + bx + c = x2 3x x R, 2ax2 + 2(a + b)x + b + 2c = x2 3x

a=
2a = 1
2
2(a + b) = 3

b = 2 .

b + 2c = 0
c=1

Les solutions de (E) sur R sont donc les fonctions de la forme x 7

x2
2x + 1 + e2x , R.
2

2me solution. Soit f une fonction drivable sur R.


http ://www.maths-france.fr

c Jean-Louis Rouget, 2014. Tous droits rservs.


f solution de (E) sur R x R, f (x) + 2f(x) = x2 3x


x R, e2x f (x) + 2e2x f(x) = (x2 3x)e2x x R, e2x f (x) = (x2 3x)e2x

Recherche dune primitive sur R de la fonction x 7 (x2 3x)e2x .


1re mthode. Deux intgrations par parties fournissent :
Z

Z

1 2
1
x 3x e2x
(2x 3)e2x dx
2
2
Z
 2x 1
1
1 2
2x
x 3x e (2x 3)e +
e2x dx
=
2
4
2


1
1
1 2
=
2x2 8x + 3 e2x + e2x + =
x 4x + 2 e2x +
4
4
2

(x2 3x)e2x dx =

2me mthode. Cherchons les primitives de x 7 (x2 3x)e2x sous la forme x 7 (ax2 + bx + c)e2x .
Donc,






ax2 + bx + c e2x = 2 ax2 + bx + c + (2ax + b) e2x = 2ax2 + 2(a + b)x + b + 2c e2x .

x R,




ax2 + bx + c e2x = x2 3x e2x

Rsolution de (E) sur R.


f solution de (E) sur R x R, e2x f

2a = 1
a=
2

2(a + b) = 3
b = 2 .

b + 2c = 0
c=1


(x) = x2 3x e2x

 2
x
2x
2x + 1 e2x +
R/ x R, e f(x) =
2

x2
2x + 1 + e2x .
2


x2
2x
2x + 1 + e
, R .
SR = x 7
2
R/ x R, f(x) =

1
sont continues sur R et on sait que les solutions de (E) sur R sont de la forme
1 + 2ex
f0 + f1 o f0 est une solution particulire de (E) et f1 est une solution particulire non nulle de (Eh ).

7) Les fonctions x 7 1 et x 7

Soit f une fonction drivable sur R.


ex
1
x
x

R,
e
f
(x)
+
e
f(x)
=
1 + 2ex
1 + 2ex
1
R/ x R, ex f(x) = ln (1 + 2ex ) +
2


1
x
R/ x R, f(x) =
ln (1 + 2e ) + ex
2

f solution de (E) sur R x R, f (x) + f(x) =

SR =

x 7



1
x
x
ln (1 + 2e ) + e , R .
2

cos x
1
8) Sur I, lquation (E) est quivalente lquation y
y=
.
sin x
sin x
1
cos x
et x 7
sont continues sur I =]0, [ et on sait que les solutions de (E) sur R sont de la
Les fonctions x 7
sin x
sin x
forme f0 + f1 o f0 est une solution particulire de (E) et f1 est une solution particulire non nulle de (Eh ).
Mais x 7 sin x est une solution non nulle de (Eh ) sur I et x 7 cos x est une solution de (E) sur ]0, [. Donc

http ://www.maths-france.fr

c Jean-Louis Rouget, 2014. Tous droits rservs.


SR = {x 7 sin x + cos x, R}.

Exercice no 2

1) La fonction x 7 th x est continue sur R et et on sait que les solutions de (E) sur R sont de la forme f0 o f0 est une
solution particulire non nulle de (E).

Soit f une fonction drivable sur R.

f solution de (E) sur R x R, f (x) + f(x) th x = 0 x R, ch xf (x) + sh xf(x) = 0 x R, (ch f) (x) = 0


R/ x R, ch xf(x) =

R/ x R, f(x) =
.
ch x

Les solutions de (E) sur R sont les fonctions de la forme x 7

, R.
ch x

= 1 = 1. La solution sur R de lquation diffrentielle y + y th x = 0


ch 0
1
.
prenant la valeur 1 en 0 est la fonction x 7
ch x
2) Les fonctions x 7 th x et x 7 x th x sont continues sur R et et on sait que les solutions de (E) sur R sont de la forme
f0 + f1 o f0 est une solution particulire de (E) et f1 est une solution particulire non nulle de (Eh ).
Soit f une telle fonction. f(0) = 1

1
. Dterminons une solution particulire de (E) sur R par la mthode de variation
ch x
de la constante. Il existe une solution particulire de (E) sur R de la forme f0 : (x)f1 (x) o est drivable sur R et
vrifie f0 = x th x.

Daprs 1), on peut prendre f1 : x 7

Or,

x R, (x)f0 (x) = x th x x R,

x sh x dx = x ch x

Z ch

(x)
= x th x x R, (x) = x sh x.
ch x

x dx = x ch x sh x + C, C R. On peut donc prendre : x 7 x ch x sh x puis

f0 : x 7 x th x. Les solutions de (E) sur R sont les fonctions de la forme x 7 x th x +

, R.
ch x

Soit f une telle fonction. f(0) = 0 0 +


= 0 = 0. La solution sur R de lquation diffrentielle y + y th x = x th x
ch 0
prenant la valeur 0 en 0 est la fonction x 7 x th x.

Exercice no 3

Lquation diffrentielle rsoudre dans cet exercice est linaire du premier ordre. On note (E) lquation diffrentielle
propose et (Eh ) lquation homogne associe.
x2
2x
et
x

7
sont continues sur I et on sait
1 x2
1 x2
que les solutions de (E) sur I sont de la forme f0 + f1 o f0 est une solution particulire de (E) et f1 est une solution
particulire non nulle de (Eh ).

Soit I lun des deux intervalles ] 1, 1[ ou ]1, +[. Les fonctions x 7


Rsolution de (E) sur I. Soit f une fonction drivable sur I.

f solution de (E) sur I x I, (1 x2 )f (x) 2xf(x) = x2

x I, ((1 x2 )f) (x) = x2 R/ x I, (1 x2 )f(x) =


R/ x I, f(x) =

x3
+
3

x3 +
,
3(1 x2 )

(en renommant la constante 3 ( dcrit R si et seulement si 3 dcrit R car lapplication t 7 3t est une bijection de R
sur lui-mme)).
Rsolution de (E) sur I =] 1, +[. Soit f une ventuelle solution de (E) sur I. Les restrictions de f ] 1, 1[ et ]1, +[
sont encore solution de (E) et donc de la forme prcdente. Par suite, ncessairement, il existe deux constantes 1 et2
x 3 + 2
1
x 3 + 1
et pour x > 1, f(x) =
. Enfin, lquation impose f(1) = .
telles que, pour 1 < x < 1, f(x) =
2
3(1 x )
3(1 x2 )
2

http ://www.maths-france.fr

c Jean-Louis Rouget, 2014. Tous droits rservs.


En rsum, une ventuelle solution de (E) sur I est ncessairement de la forme :

x 3 + 1

si 1 < x < 1

3(1 x2 )

1
.
x > 1, f(x) =
si x = 1

x 3 + 2

si x > 1

3(1 x2 )

Rciproquement, f ainsi dfinie, est drivable sur ] 1, 1[ et solution de (E) sur ] 1, 1[, drivable sur ]1, +[ et solution
de (E) sur ]1, +[ et, si f est drivable en 1, f vrifie encore (E) pour x = 1. Donc, f est solution de (E) sur ] 1, +[ si
et seulement si f est drivable en 1.
Pour 1 < x < 1,
1
x 3 + 1
+
2x3 + 21 + 3(1 x2 )
f(x) f(1)
3(1 x2 ) 2
=
=
x1
x1
6(1 x2 )(x 1)

Quand x tend vers 1 par valeurs infrieures, le dnominateur de la fraction tend vers 0 et le numrateur tend vers 2(1+ 1 ).
Donc, si 1 6= 1, f nest pas drivable gauche en 1. De mme, si 2 nest pas 1, f nest pas drivable droite en 1.
Ainsi, si f est solution de (E) sur I, ncessairement 1 = 2 = 1. Dans ce cas, pour x ] 1, +[\{1},
f(x) =

x3 1
(x 1)(x2 + x + 1)
x2 + x + 1
=
=

,
3(1 x2 )
3(1 x)(1 + x)
3(x + 1)

ce qui reste vrai pour x = 1. Ainsi, si f est une solution de (E) sur ] 1, +[, ncessairement pour x > 1, f(x) =
x2 + x + 1
. Rciproquement, f ainsi dfinie est drivable sur ] 1, +[ et en particulier en 1. f est donc solution de (E)

3(x + 1)
sur ] 1, +[.
Sur ] 1, +[, (E) admet une et une seule solution savoir la fonction x 7

x2 + x + 1
.
3(x + 1)

Rsolution de (E) sur R. Soit f une ventuelle solution de (E) sur R. La restriction de f ] 1, +[ est ncessairement
la fonction prcdente. Mais cette fonction tend vers quand x tend vers 1 par valeurs suprieures. Donc f ne peut
tre continue sur R. Lquation (E) na pas de solution sur R.
Exercice no 4
Rsolution de (E) sur ]0, +[.
Soit f une fonction drivable sur ]0, +[.
f solution de (E) sur ]0, +[ x ]0, +[, |x|f (x) + (x 1)f(x) = x3

x ]0, +[, xf (x) + (x 1)f(x) = x3




1

x ]0, +[, f (x) + 1


f(x) = x2
x


1
xln x
x ]0, +[, e
f (x) + 1
exln x f(x) = exln x x2
x
 x 
e
f (x) = xex
x ]0, +[,
x
 x 
e
f (x) = ((x 1)ex )
x ]0, +[,
x
R/ x ]0, +[, f(x) = xex ((x 1)ex + )

R/ x ]0, +[, f(x) = x2 x + xex

Les solutions de (E) sur ]0, +[ sont les fonctions de la forme x 7 x2 x + xex , R.

Rsolution de (E) sur ] , 0[.

Soit f une fonction drivable sur ] , 0[.


http ://www.maths-france.fr

c Jean-Louis Rouget, 2014. Tous droits rservs.


f solution de (E) sur ] , 0[ x ]0, +[, xf (x) + (x 1)f(x) = x3




1

f(x) = x2
x ] , 0[, f (x) + 1 +
x
1
x ] , 0[, ex+ln |x| f (x) + (1 + )ex+ln |x| f(x) = ex+ln |x| x2
x
x ] , 0[, (xex y) = x3 ex ()

Dterminons une primitive de la fonction x 7 x3 ex de la forme (ax3 + bx2 + cx + d)ex .

((ax3 + bx2 + cx + d)ex ) = ((ax3 + bx2 + cx + d) + (3ax2 + 2bx + c))ex


= (ax3 + (3a b)x2 + (2b c)x + c d)ex ,

et

((ax3 + bx2 + cx + d)ex ) = x3 ex

Par suite,

a = 1

a = 1
3a b = 0

b = 3
.
2b c = 0

c = 6 = d

cd=0

() R/ x ] , 0[, xex f(x) = (x3 + 3x2 + 6x + 6)ex +


ex + 6
.
R/ x ] , 0[, f(x) = x2 + 3x + 6 +
x

ex + 6
, R.
x
On peut montrer quil existe une solution et une seule sur R mais on manque encore doutils pour le prouver.

Les solutions de (E) sur ] , 0[ sont les fonctions de la forme x 7 x2 + 3x + 6 +


Exercice no 5

1) Lquation caractristique de lquation homogne y 2y + 2y = 0 est r2 2r + 2 = 0 dont les racines sont 1 i et


1 + i. Les solutions de lquation homogne sont les fonctions de la forme x 7 ex ( cos x + sin x), (, ) R2 . Lquation
avec second membre scrit
y 2y + 2y =


1  (1+i)x
e
+ e(1+i)x + e(1i)x + e(1i)x .
4

On applique alors le principe de superposition des solutions.


Recherche dune solution particulire de lquation y 2y + 2y = e(1+i)x .
1 + i est racine simple de lquation caractristique et donc lquation prcdente admet une solution particulire de la
forme f : x 7 (ax)e(1+i)x . Daprs la formule de Leibniz, pour tout rel x


f (x) 2f (x) + 2f(x) = (1 + i)2 (ax) + 2(1 + i)(a) 2 ((1 + i)(ax) + a) + 2(ax) e(1+i)x
= [2(1 + i)a 2a)] e(1+i)x
= 2iae(1+i)x .
puis,
i
x R, f (x) 2f (x) + 2f(x) = e(1+i)x 2ia = 1 a = .
2
ix
Une solution particulire de lquation y 2y + 2y = e(1+i)x est x 7 e(1+i)x . Par conjugaison, une solution
2
ix (1i)x

(1i)x
particulire de lquation y 2y + 2y = e
est x 7 e
.
2
http ://www.maths-france.fr

c Jean-Louis Rouget, 2014. Tous droits rservs.


Recherche dune solution particulire de lquation y 2y + 2y = e(1+i)x .


1 + i nest pas racine de lquation caractristique et donc lquation prcdente admet une solution particulire de la
forme f : x 7 ae(1+i)x . Pour tout rel x,
f (x) 2f (x) + 2f(x) = a((1 + i)2 2(1 + i) + 2)e(1+i)x = 4ae(1+i)x

puis, x R, f (x) 2f (x) + 2f(x) = e(1+i)x 4a = 1 a =

1
.
4

1
Une solution particulire de lquation y 2y + 2y = e(1+i)x est x 7 e(1+i)x . Par conjugaison, une solution
4
1
particulire de lquation y 2y + 2y = e(1i)x est x 7 e(1i)x .
4
Une solution particulire de lquation y 2y + 2y = cos x ch x est donc
1
2Re
4

ix (1+i)x 1 (1+i)x
e
+ e
2
4

 1

1
Re 2ix(cos x + i sin x)ex + (cos x + i sin x)ex =
2x sin xex + cos xex .
8
8

Les solutions sur R de lquation propose sont les fonctions de la forme x 7


(, ) R2 .

1
(2x sin xex + cos xex )+( cos x + sin x) ex ,
8

2) Lquation caractristique de lquation homogne y + 6y + 9y = 0 est r2 + 6r + 9 = 0 qui admet la racine double


r = 3. Les solutions de lquation homogne sont les fonctions de la forme x 7 e3x (x + ), (, ) R2 .
2 nest pas racine de lquation caractristique et donc lquation propose admet une solution particulire de la forme
f : x 7 ae2x . Pour tout rel x, f (x) + 6f (x) + 9f(x) = 25ae2x puis,
x R, f (x) + 6f (x) + 9f(x) = e2x 25a = 1 a =

1
.
25

1 2x
e .
25
1 2x
e + (x + )e3x , (, ) R2 .
Les solutions sur R de lquation propose sont les fonctions de la forme x 7
25

Une solution particulire de lquation y + 6y + 9y = e2x est x 7

3) Lquation caractristique de lquation homogne y 2y + y = 0 est r2 2r + 1 = 0 qui admet la racine double


r = 1. Les solutions de lquation homogne sont les fonctions de la forme x 7 ex (x + ), (, ) R2 .

1 x
(e + ex ). Appliquons le principe de superposition des solutions.
2
Recherche dune solution particulire de lquation y 2y + y = ex .
Le second membre scrit

1 est racine double de lquation caractristique et donc lquation propose admet une solution particulire de la forme
f : x
7 ax2 ex . Daprs la formule de Leibniz, pour tout rel x,
f (x) 2f (x) + f(x) = ((ax2 + 2(2ax) + 2a) 2(ax2 + (2ax)) + ax2 )e2x = 2aex ,

puis,
x R, f (x) 2f (x) + f(x) = ex 2a = 1 a =

Une solution particulire de lquation y 2y + y = ex est x 7

1
.
2

x2 x
e .
2

Recherche dune solution particulire de lquation y 2y + y = ex .

1 nest pas racine de lquation caractristique et donc lquation propose admet une solution particulire de la forme
f : x 7 aex . Pour tout rel x,
f (x) 2f (x) + f(x) = (a + 2a + a)ex = 4aex ,

puis,
http ://www.maths-france.fr

c Jean-Louis Rouget, 2014. Tous droits rservs.


x R, f (x) 2f (x) + f(x) = ex a =

Une solution particulire de lquation y 2y + y = ex est x 7

1
.
4

1 x
e .
4

Les solutions sur R de lquation propose sont les fonctions de la forme x 7


1
x2
+ x + ex + ex , (, ) R2 .
4
8

4) Soit k R. Lquation caractristique de lquation homogne y 2ky + (1 + k2 )y = 0 est r2 2kr + 1 + k2 = 0 dont


le discriminant rduit vaut 1 = i2 . Cette quation admet donc pour racines k + i et k i. Les solutions de lquation
homogne sont les fonctions de la forme x 7 ekx ( cos x + sin x), (, ) R2 .

Le second membre scrit Im e(1+i)x . Rsolvons donc lquation y 2y + y = e(1+i)x .

Puisque k 6= 1, 1+i nest pas racine de lquation caractristique et donc lquation propose admet une solution particulire
de la forme f : x 7 ae(1+i)x . Or, pour tout rel x

f (x) 2kf (x) + (1 + k2 )f(x) = a (1 + i)2 2k(1 + i) + 1 + k2 e(1+i)x = ((k 1)2 2(k 1)i)ae(1+i)x

et donc,

1
k 1 + 2i
a=
.
(k 1)(k 1 2i)
(k 1)(k2 2k + 5)
k 1 2i
Une solution particulire de lquation y 2y + y = e(1+i)x est x 7
e(1+i)x et une solution
(k 1)(k2 2k + 5)
particulire de lquation y 2y + y = ex sin x est
x R, f (x) 2kf (x) + (1 + k2 )f(x) = e(1+i)x a =

1
1
Im ((k 1 2i)(cos x + i sin x)ex ) =
(2 cos x + (k 1) sin x)ex .
(k 1)(k2 2k + 5)
(k 1)(k2 2k + 5)
Les solutions de lquation propose sont les fonctions de la forme

Exercice no 6

x 7

1
(k

1)(k2

2k + 5)

(2 cos x + (k 1) sin x)ex + ( cos x + sin x)ekx , (, ) R.

1
1) Soit z une fonction dfinie sur ]1, +[ qui ne sannule pas sur ]1, +[. Soit y = . Alors y ne sannule pas sur ]1, +[
z
1
et z = .
y
Puisque z ne sannule pas sur ]1, +[, z est drivable sur ]1, +[ si et seulement si y est drivable sur ]1, +[. De plus
Soient donc z une fonction drivable sur ]1, +[ qui ne sannule pas sur ]1, +[ puis y =

1
.
z

z solution de (E1 ) sur ]1, +[ x ]1, +[, x2 z (x) + xz(x) = z2 (x)


 
1
x
1
x ]1, +[, x2
(x) +
= 2
y
y(x)
y (x)

x
1
y
(x)
+
= 2
x ]1, +[, x2 2
y (x) y(x)
y (x)
x ]1, +[, x2 y (x) + xy(x) = 1.

z est solution de (E1 ) sur ]1, +[ si et seulement si y est une solution de (E2 ) : x2 y (x) + xy(x) = 1 sur ]1, +[ et ne
sannulant pas sur ]1, +[.
2) Soit y une fonction drivable sur ]1, +[.
1
1
x ]1, +[, (xy) (x) =
x
x
R/ x ]1, +[, xy (x) = ln(x) +

x ]1, +[, x2 y (x) + xy(x) = 1 x ]1, +[, xy (x) + y(x) =

http ://www.maths-france.fr

R/ x ]1, +[, y (x) =


8

ln(x) +
.
x

c Jean-Louis Rouget, 2014. Tous droits rservs.


De plus, comme ln(x) + = 0 e = e , y ne sannule pas sur ]1, +[ si et seulement si > 0.

Les solutions de de (E1 ) sur ]1, +[ qui ne sannulent pas sur I =]1, +[ sont donc les fonctions de la forme x 7

x
,
ln(x) +

> 0, ou encore (en posant a = e de sorte que a > 1 et = ln a) les fonctions de la forme
x 7

Exercice no 7

x
, a > 1.
ln(ax)

1) Supposons y deux fois drivable sur ]0, +[. La fonction : t 7 et est deux fois drivable sur R valeurs dans
]0, +[ et la fonction x 7 y(x) est deux fois drivable sur ]0, +[. Donc, la fonction z = y (de sorte que pour tout
rel t, z(t) = y (et )) est deux fois drivable sur R.

Rciproquement, supposons que z est deux fois drivable sur R. La fonction 1 : x 7 ln x est deux fois drivable sur
]0, +[ valeurs dans R et la fonction t 7 z(t) est deux fois drivable sur R. Donc, la fonction y = z 1 (de sorte que
pour tout rel x > 0, y(x) = z(ln x)) est deux fois drivable sur ]0, +[.

Finalement y est deux fois drivable sur ]0, +[ si et seulement si z est deux fois drivable sur R.
2) Pour tout rel t, posons donc x = et puis, z(t) = y(x) = y(et ).
Alors,
z (t) = et y (et ) = xy (x)
puis
z (t) = et y (et ) + (et )2 y (et ) = xy (x) + x2 y (x).
Donc, xy (x) = z (t) et x2 y (x) = z (t) xy (x) = z (t) z (t) et

ax2 y (x) + bxy (x) + cy(x) = a(z (t) z (t)) + bz (t) + cz(t) = az (t) + (b a)z (t) + cz(t).
Donc,
x > 0, ax2 y (x) + bxy (x) + cy(x) = 0 t R, az (t) + (b a)z (t) + cz(t) = 0.

3) On applique le 2) avec a = 1, b = 1 et c = 1. Lquation rsoudre sur R est alors z 2z + z = 0. Les solutions


de cette quation sur R sont les fonctions de la forme t 7 (t + )et , (, ) R2 . Les solutions sur ]0, +[ de lquation
initiale sont donc les fonctions de la forme x 7 x ln x + x, (, ) R2 .
Exercice no 8

On sait que les solutions sur R de lquation propose sont les fonctions de la forme :
Zx
g : x 7 eax + eax eat f(t) dt, R.
0

Dans ce cas, pour x R, g(x + T ) = ea(x+T ) + ea(x+T )


Z x+T

at

e f(t) dt =

ZT

0
ZT
0

at

e f(t) dt +

Z x+T
T

eat f(t) dt + eaT

Z x+T

eat f(t) dt. Or,

at

e f(t) dt =

Zx

ZT

at

e f(t) dt +

Zx

ea(u+T ) f(u + T ) du

eau f(u)du.

Donc,

g(x + T ) = ea(x+T ) + ea(x+T )

ZT
0

= ea(x+T ) + ea(x+T )

ZT

eat f(t) dt + eax

Zx

eau f(u) du

eat f(t) dt + g(x) eax .

Par suite,

http ://www.maths-france.fr

c Jean-Louis Rouget, 2014. Tous droits rservs.


g est T -priodique x R, g(x + T ) g(x) = 0


ZT


x R, ea(x+T ) eax + ea(x+T ) eat f(t) dt = 0
0
!
ZT

x R, eax eaT 1 + eaT
eat f(t) dt = 0
0

(1 eaT ) = eaT
=

eaT
1 eaT

ZT
0

Do lexistence et lunicit dune solution T -priodique :


x R, g(x) =

eaT
1 eaT

ZT

ZT
0

eat f(t) dt (car x R, eax 6= 0)

eat f(t) dt (a 6= 0 et T 6= 0 eat 6= 1).


!

e f(t) dt eax + eax


at

Zx

eat f(t) dt.

Exercice no 9

. x x
1) On rsout (E ) : x + =
avec x(0) = x0 . Cette quation est du type y + ay = b o a et b sont deux constantes

relles.
t

1re solution. La fonction constante t


7 x est une solution particulire de (E ) sur R et la fonction t 7 e est une
t
solution non nulle de (Eh ) sur R. Donc les solutions de (E ) sur R sont les fonctions de la forme x : t 7 x + e ,
R.
2me solution.

t R, x(t) +

t .
x(t)
1 t
x t
x
=
t R, e x(t) + e x(t) =
e

t
e
d  t 
t R,
e x (t) = x
dt

t
t
R/ t R, e x(t) = x e +
t

R/ t R, x(t) = x + e .
t

Les solutions de lquation sont les fonctions de la forme x : t 7 x + e , R.

Ensuite x(0) = x0 x + = x0 = x0 x . La solution au problme pos est la fonction


t

x : t 7 x + (x0 x ) e .

Remarque. Pour tout condition initiale x0 , la fonction x tend vers x quand t tend vers +.
dU
1
E
dU
+ U = E qui scrit encore
+
U=
est du type y + ay = b o a et b sont deux
2) Lquation (E ) : RC
dt
dt
RC
RC
constantes relles.
t

1re solution. La fonction constante t


7 E est une solution particulire de (E ) sur R et la fonction t 7 e RC est une
t
solution non nulle de (Eh ) sur R. Donc les solutions de (E ) sur R sont les fonctions de la forme U : t 7 E + e RC ,
R.
2me solution.

t > 0, RC

dU
E t
dU
1
E
t dU
1 t
(t) + U(t) = E t > 0
(t) +
U(t) =
t > 0, e RC
(t) +
e RC U(t) =
e RC
dt
dt
RC
RC
dt
RC
RC
t
e RC
d  t 
e RC U (t) = E
t > 0,
dt
RC
t
t
R/ t > 0, e RC U(t) = Ee RC +
t

http ://www.maths-france.fr

R/ t > 0, U(t) = E + e RC .
10

c Jean-Louis Rouget, 2014. Tous droits rservs.


Les solutions de lquation sont les fonctions de la forme t 7 E + e RC , R.

Ensuite U(0) = U0 E + = U0 = U0 E. La solution au problme pos est la fonction


t

U : t 7 E + (U0 E) e RC .

Remarque. Si U0 > E alors U0 E > 0. La fonction U est dcroissante et tend vers U = E. Le condensateur se dcharge.
Si U0 < E alors U0 E < 0. La fonction U est croissante et tend vers U = E. Le condensateur se charge.
Si U0 = E, la fonction U est constante.
Les quations qui suivent sont du type ay + by + cy = f(t) o a, b et c sont des constantes relles. La solution gnrale
est somme dune solution particulire et de la solution gnrale de lquation associe.
3) Lquation caractristique associe lquation diffrentielle x + 20 x = 0 est z2 + 20 = 0 dont les solutions sont non
relles et conjugues savoir z1 = i0 et z2 = i0 .

..

2
On sait que les solutions de lquation homogne sont les fonctions
de la forme t 7 B cos (0 t) + C sin (0 t), (B, C) R
2
2
ou encore t 7 A cos (0 t + ), (A, ) [0, +[R avec A = B + C .

a) Les condition initiales x(t0 ) = x0 et x(t0 ) = v0 fournissent



B cos (0 t0 ) + C sin (0 t0 ) = x0
.
B0 sin (0 t0 ) + C0 cos (0 t0 ) = v0



cos (0 t0 )
sin (0 t0 )

Le dterminant du systme est
= 0 . Les formules de Cramer fournissent
0 sin (0 t0 ) 0 cos (0 t0 )


1
1 x0
sin (0 t0 )
=
(0 cos (0 t0 ) x0 sin (0 t0 ) v0 ) ,
B=


v

cos
(
t
)
0
0
0
0
0 0

et

C=



1
1
cos (0 t0 )
x0
=
(cos (0 t0 ) v0 + 0 sin (0 t0 ) x0 ) .
0 0 sin (0 t0 ) v0 0

Une situation courante est v0 = 0 et on obtient la solution

x : t 7 x0 (cos (0 t) cos (0 t0 ) + sin (0 t) sin (0 t0 )) = x0 cos (0 (t t0 )) .

b) La fonction constante t 7
fonctions de la forme t 7
a).

A
est une solution particulire de lquation et donc les solutions de lquation sont les
20

A
+ B cos (0 t) + C sin (0 t), (B, C) R2 . Le calcul de B et C est analogue au calcul fait en
20

4) circuits RLC

Dans les deux cas, lquation caractristique de lquation homogne associe est (Ec ) : z2 + 2z + 20 = 0. Le discriminant
rduit de cette quation est
= 2 20 .
est un rel positif et 0 est un rel strictement positif. On a trois cas :
1er cas. Si > 0 , alors > 0 et donc (Ec ) admet deux solutions relles distinctes r1 = +
q
2 20 .

q
2 20 et r2 =

On note que r1 r2 = 20 > 0 et donc r1 et r2 sont deux rels non nuls et de mme signe puis que r1 + r2 = 2 < 0 et
donc r1 et r2 sont strictement ngatifs.

On sait que les solutions sur R de lquation homogne sont les fonctions de la forme
q : t 7 Aer1 t + Ber2 t , (A, B) R2 .

2me cas. Si = 0 , alors = 0 et (Ec ) admet une solution relle double r = < 0.
On sait que les solutions sur R de lquation homogne sont les fonctions de la forme
http ://www.maths-france.fr

11

c Jean-Louis Rouget, 2014. Tous droits rservs.


q : t 7 (At + B) et , (A, B) R2 .

3me cas. Si < 0 , alors < 0 et donc (Ec ) admet deux solutions non relles conjugues r1 = + i
q
r2 = i 20 2 . On note que 2Re (r1 ) = r1 + r1 = r1 + r2 = 2 < 0.

q
20 2 et

On sait que les solutions sur R de lquation homogne sont les fonctions de la forme

q : t 7 (A cos(t) + B sin(t)) et , (A, B) R2 ,


q
o = 20 2 est la pseudo-pulsation.
a) Dans tous les cas, la fonction constante t 7

E
= EC est une solution particulire de lquation. Donc,
L20

1er cas. Si > 0 , les solutions de lquation sont les fonctions de la forme
q : t 7 EC + Aer1 t + Ber2 t , (A, B) R2 ,
q
q
.
avec r1 = + 2 20 et r2 = 2 20 . Les conditions initiales q(0) = 0 et q(0) = 0 fournissent

A + B = EC
r1 A r2 B = 0
q
Le dterminant du systme est r1 r2 = 2 2 20 6= 0. Les formules de Cramer fournissent
ECr2
A= q
=
2 20




q
q
2
2
2
2
0 EC
0 EC
r1 EC
q
q
et B = q
.
2 20
2 20
2 20

On note que la solution tend vers q = EC.

2me cas. Si = 0 , les solutions de lquation sont les fonctions de la forme

q : t 7 EC + (At + B) et , (A, B) R2 .

Les conditions initiales q(0) = 0 et q(0) = 0 fournissent



B = EC
,
A B = 0

et donc

A = EC et B = EC.

La solution scrit q : t 7 EC 1 (t + 1) et . On note que la solution tend vers q = EC quand t tend vers +
daprs un thorme de croissances compares.
3me cas. Si < 0 , les solutions sont les fonctions de la forme

q : t 7 EC + (A cos(t) + B sin(t)) et , (A, B) R2 ,


q
o = 20 2 est la pseudo-pulsation.

Les conditions initiales q(0) = 0 et q(0) = 0 fournissent



A = EC
,
A B = 0

et donc

EC
.






La solution scrit q : t 7 EC 1 cos (t) + sin (t) et . On note que la solution tend vers q = EC quand
C
t tend vers +.
A = EC et B =

http ://www.maths-france.fr

12

c Jean-Louis Rouget, 2014. Tous droits rservs.


Planche no 17. Rationnels, rels


* trs facile ** facile *** difficult moyenne **** difficile
I : Incontournable T : pour travailler et mmoriser le cours
Exercice no 1 (I)
Montrer que les nombres suivants sont irrationnels.

1) (**) 2 et plus gnralement n m o n est un entier suprieur ou gal 2 et m est un entier naturel suprieur ou
gal 2, qui nest pas une puissance n-ime parfaite.
2) (**) log 2.
3) (***) e (Hermite a dmontr en 1873 que e est transcendant. Cest historiquement le premier vrai nombre dont
on a russi dmontrer la transcendance).
Z1
n
X
(1 t)n t
1
+
e dt, puis que pour tout entier naturel non
Pour cela, tablir que pour tout entier naturel n, e =
k!
n!
0
k=0
n
X
3
1
nul n, 0 < e
<
. Raisonner alors par labsurde.
k!
(n + 1)!
k=0
 
2
2
,
4) (***) cos( ). Pour cela trouver une quation du troisime degr coefficients entiers dont les solutions sont cos
7
7
 
 
4
6
cos
et cos
, puis vrifier que cette quation na pas de racine rationnelle (supposer par labsurde quil y a une
7
7
p
racine rationnelle avec p Z , q N et PGCD(p, q) = 1 et montrer que p divise 1 et q divise 8). (On rappelle ou on
q
admet le thorme de Gauss : soient a, b et c trois entiers relatifs tous non nuls. Si a divise bc et a et b sont premiers
entre eux, alors a divise c).

5) (***) 2 + 3 + 5.
Pour le nombre , il faudra attendre davoir quelques rsultats supplmentaires sur les suites et les polynmes ...
Exercice no 2 (**IT)
Soient A et B deux parties de R, non vides et bornes. Montrer que sup A, sup B, sup(A + B), inf A, inf B, inf (A + B)
existent et que lon a sup (A + B) = sup A + sup B et inf (A + B) = inf A + inf B. (A + B dsigne lensemble des sommes
dun lment de A et dun lment de B).
Exercice no 3 (**)


1
Soit A =
+ (1)n , n N . Dterminer sup A et inf A.
n
Exercice no 4 (**IT)
Soit A une partie non vide et borne de R. Montrer que sup{|x y|, (x, y) A2 } = sup A inf A.
Exercice no 5 (***IT)
Soient A et B deux parties non vides et majores de R. Que dire de sup(A B), sup(A B), sup(A + B) et sup(AB) ? (A + B
(resp. AB) dsigne lensemble des sommes (resp. des produits) dun lment de A et dun lment de B).
Exercice no 6 (**)
(Identit de Catalan) Montrer que pour tout entier naturel non nul n,

2n1
X
k=0

http ://www.maths-france.fr

2n
X
(1)k
1
=
.
k+1
k
k=n+1

c Jean-Louis Rouget, 2014. Tous droits rservs.


Exercice no 7 (**I) (Ingalits de Cauchy-Schwarz et de Minkowski)


Soient a1 ,..., an , b1 ,..., bn des nombres rels.
1) En considrant la fonction f : x 7

n
X

(ak + xbk )2 , montrer que |

k=1

n
X

k=1

Cauchy-Schwarz).

v
v
u n
u n
uX 2 uX
ak t
b2k (ingalit de
a k bk | 6 t
k=1

k=1

v
v
v
uX
uX
uX
n
u n
u
u n 2
2
2
t
t
(ak + bk ) 6
ak + t
bk .
2) En dduire lingalit de Minkowski :
k=1

k=1

k=1

(lingalit de Cauchy-Schwarz affirme que le produit scalaire de deux vecteurs est infrieur ou gal au produit de leurs
normes et lingalit de Minkowski est lingalit triangulaire).
Exercice no 8 (**)
Rsoudre dans R lquation
Exercice no 9 (**)

p
p

x + 2 x 1 + x 2 x 1 = 1.

Montrer que {r3 , r Q} est dense dans R.

http ://www.maths-france.fr

c Jean-Louis Rouget, 2014. Tous droits rservs.


Planche no 17. Rationnels, rels : corrig


Exercice no 1.
1) Soient m et n deux entiers naturels suprieurs 2.

a
n
m Q (a, b) (N )2 / n m = (a, b) (N )2 / an = m bn .
b
Tout dabord, si b = 1, m = an et m est une puissance n-ime parfaite. Ensuite, a = 1 est impossible car m bn > 2.

Supposons alors que a et b sont des entiers suprieurs 2 (et que an = m bn ). Lexposant de tout facteur premier
de an ou de bn est un multiple de n et parunicit de la dcomposition en facteurs premiers, il en est de mme de tout
facteur premier de m. Ceci montre que, si n m est rationnel, m est une puissance n-ime parfaite.

Rciproquement, si m est une puissance n-ime parfaite, n m est un entier et en particulier un rationnel. En rsum :

n
m Q n m N m est une puissance n -ime parfaite.

Par suite, si m nest pas une puissance n-ime parfaite, n m est irrationnel. Par exemple, 2 ou 3 7 sont des irrationnels.
2)
a
(a, b) (N )2 / 10a/b = 2 (a, b) (N )2 / 10a = 2b
b
(a, b) (N )2 / 5a = 2ba .

log 2 Q (a, b) (N )2 / log 2 =

On a ncessairement b a > 0 car sinon 5a > 1 et 2ba 6 1 ce qui contredit lgalit 5a = 2ba .
Lgalit 5a = 2ba est alors impossible par unicit de la dcomposition en facteurs premiers dun entier naturel suprieur
ou gal 2.
On a montr par labsurde que log 2 est irrationnel.
3) Montrons par rcurrence que : n N, e =

Z1
n
X
1
(1 t)n t
+
e dt.
k!
n!
0

k=0

Z1

Z1

(1 t)n t
e dt =
et dt = e 1 et donc, e = 1 +
n!
0
0
dmontrer est donc vraie quand n = 0.
Z1
n
X
1
(1 t)n t
Soit n > 0. Supposons que e =
+
e dt.
k!
n!
0

Pour n = 0,

Z1

et dt =

Z1
0
X
1
(1 t)0 t
+
e dt. La formule
k!
0!
0

k=0

k=0

(1 t)n+1
Les deux fonctions t 7
et t 7 et sont de classe C1 sur le segment [0, 1]. On peut donc effectuer une
(n + 1)!
intgration par parties qui fournit :
Z1
0

et donc,

1 Z 1

Z1
(1 t)n+1 t
1
(1 t)n+1 t
(1 t)n+1 t
(1 t)n t
e dt =
e
e dt =
+
e dt,
+
n!
(n + 1) n!
(n + 1)!
0 (n + 1)!
0 (n + 1)!
0

e=

Z1
Z1
n+1
n
X 1
X
1
(1 t)n+1 t
(1 t)n+1 t
1
+
+
e dt =
+
e dt.
k! (n + 1)!
k!
0 (n + 1)!
0 (n + 1)!
k=0

k=0

Le rsultat est ainsi dmontr par rcurrence.


Soit n un entier naturel non nul. Daprs ce qui prcde,
0<e

Z1
Z1
n
X
1
e
3
(1 t)n t
(1 t)n
=
e dt < e
dt =
<
.
k!
n!
n!
(n + 1)!
(n + 1)!
0
0

k=0

a
Supposons alors par labsurde que e soit rationnel. Alors, il existe (a, b) (N )2 / e = .
b
Soit n un entier naturel non nul quelconque. Daprs ce qui prcde, on a
n

0<

a X 1
3

<
,
b
k!
(n + 1)!
k=0

http ://www.maths-france.fr

c Jean-Louis Rouget, 2014. Tous droits rservs.


ce qui scrit encore


0 < an! b

n
X
3b
n!
<
.
k!
n+1

k=0

3b
X
3b
(3b)!
(3b)!
<
< 1. Mais ceci est impossible car an! b
En particulier, pour n = 3b, on a 0 < a(3b)! b
k!
3b + 1
k!
k=0
k=0
est un entier relatif. Il est donc absurde de supposer que e est rationnel. Finalement, e est irrationnel.
3b
X

4
6
2
, cos
et cos
est
4) Une quation du troisime degr dont les solutions sont cos
7
7
7




2
4
6
X cos
X cos
X cos
= 0,
7
7
7
ou encore




2
2
4
6
4
2
6
4
6
4
6
2
2
X + cos
X cos
+ cos
+ cos
cos
+ cos
cos
+ cos
cos
cos
cos
= 0.
X cos
7
7
7
7
7
7
7
7
7
7
7
7
3

Calculons alors ces trois coefficients.


Soit = e2i/7 . Puisque 7 = 1 et que + 2 + 3 + 4 + 5 + 6 = 1, on a :

cos
puis,

cos


1
4
6
1
2
+ 6 + 2 + 5 + 3 + 4 = ,
+ cos
+ cos
=
7
7
7
2
2







2
4
2
6
4
6
1
+ 6 2 + 5 + + 6 3 + 4 + 2 + 5 3 + 4
cos
+ cos
cos
+ cos
cos
=
7
7
7
7
7
7
4



1
3
6
4
=
+ + + + 4 + 5 + 2 + 3 + 5 + 6 + + 2
4
1
2
= (1) = ,
4
2

et enfin,

cos




4
6
1
2
+ 6 2 + 5 3 + 4
cos
cos
=
7
7
7
8
 1


1
=
3 + 6 + + 4 3 + 4 =
6 + 1 + 2 + 3 + 4 + 5 + 1 +
8
8
1
= .
8

Les trois nombres cos

4
6
1
1
1
2
, cos
et cos
sont solution de lquation X3 + X2 X = 0 ou encore de lquation
7
7
7
2
2
8
8X3 + 4X2 4X 1 = 0.

Montrons que cette quation nadmet pas de racine rationnelle. Dans le cas contraire, si, pour p entier relatif non nul
p
et q entier naturel non nul tels que p et q sont premiers entre eux, le nombre r =
est racine de cette quation, alors
q
8p3 + 4p2 q 4pq2 q3 = 0. Ceci peut encore scrire 8p3 = q(4p2 + 4pq + q2 ) ce qui montre que q divise 8p3 . Comme
q est premier avec p et donc avec p3 , on en dduit, daprs le thorme de Gauss que q divise 8. De mme, lgalit
q3 = p(8p2 + 4pq 4q2 ) montre que p divise 1.


1 1 1 1 1 1
Ainsi, ncessairement p {1, 1} et q {1, 2, 4, 8} ou encore r 1, 1, , , , , , . On vrifie aisment
2 2 4 4 8 8
quaucun de ces nombres nest racine de lquation considre et donc cette quation na pas de racine rationnelle. En
2
est irrationnel.
particulier, cos
7

5) On sait que 2, 3 et 5 sont irrationnels mais ceci nimpose rien la somme 2 + 3 + 5.


http ://www.maths-france.fr

c Jean-Louis Rouget, 2014. Tous droits rservs.


Posons =

2 + 3 + 5.

2 
2

2+ 3+ 5 2 =
3 + 5 2 2 2 + 2 = 8 + 2 15

2

2 2 2 6 = 60

4 + 82 + 36 4 23 122 + 24 2 36 = 0

4 42 24 = 4 2(2 6)

Si maintenant, on suppose que est rationnel, puisque 2 est irrationnel, on a ncessairement (2 6) = 0 (dans le cas

4 + 82 24
Q). Mais nest ni 0, ni 6, ni 6 (car 2 > 2 + 3 + 5 = 10 > 6). Donc 2 + 3 + 5
contraire, 2 =
2
4( 6)
est irrationnel.
=

Exercice no 2.
A et B sont deux parties non vides et majores de R et admettent donc des bornes suprieures notes respectivement
et .
Pour tout (a, b) A B. On a a + b 6 + . Ceci montre que A + B est une partie non vide et majore de R, et donc
que sup(A + B) existe dans R. De plus, puisque + est un majorant de A + B, on a sup(A + B) 6 + .
Soit alors > 0.

Il existe a0 A et b0 B tels que < a0 6 et < b0 6 , et donc tels que + < a0 + b0 6 + .


2
2
En rsum,
1) (a, b) A B, a + b 6 + et
2) > 0, (a, b) A B/ a + b > + .
On en dduit que sup(A + B) = + = sup A + sup B.
La dmarche et le rsultat sont analogues pour les bornes infrieures.
Exercice no 3.
Posons pour n entier naturel non nul un =

1
+(1)n de sorte que A = {un , n N } =
n



1
1
1
1
0, + 1, 1, + 1, 1, ... .
2
3
4
5

1
3
. Donc n N , 1 < u2n 6 .
2n
2
1

. Donc n N , 1 < u2n1 6 0.


Pour n > 1, u2n1 = 1 +
2n 1
3
3
Par suite, n N , 1 < un = . Donc, sup A et inf A existent dans R et de plus 1 6 inf A 6 sup A 6 .
2
2
3
3
Ensuite, = u2 A. Donc, sup A = max A = .
2
2
1
Enfin, pour chaque entier naturel non nul n, on a 1 6 inf A 6 u2n1 = 1 +
. On fait tendre n tend vers linfini
2n 1
dans cet encadrement, on obtient inf A = 1 (cette borne infrieure nest pas un minimum).
Pour n > 1, u2n = 1 +

inf A = 1 et sup A = max A =

3
.
2

Exercice no 4.


Posons B = |y x| , (x, y) A2 .

A est une partie non vide et borne de R, et donc m = inf A et M = sup A existent dans R.
Pour (x, y) A2 , on a m 6 x 6 M et m 6 y 6 M, et donc y x 6 M m et x y 6 M m ou encore |y x| 6 M m.
Par suite, B est une partie non vide et majore de R. B admet donc une borne suprieure.

Soit > 0. Il existe (x0 , y0 ) A2 tel que x0 > sup A et y0 < inf A + .
2
2
Ces deux lments x0 et y0 vrifient,


 
inf A +
= sup A inf A .
|y0 x0 | | > x0 y0 > sup A
2
2
http ://www.maths-france.fr

c Jean-Louis Rouget, 2014. Tous droits rservs.


En rsum,
1) (x, y) A2 , |y x| sup A inf A et
2) > 0, (x, y) A2 / |y x| > sup A inf A .
Donc, sup B = sup A inf A.
Exercice no 5.
1) A B peut tre vide et on na rien dire. Supposons donc A B non vide. Pour x A B, on a x 6 sup A et x 6 sup B
et donc x 6 min {sup A, sup B}.
Dans ce cas, sup(A B) existe et sup(A B) 6 min {sup A, sup B}.
On ne peut pas amliorer. Par exemple, soit A = [0, 1] Q et B = ([0, 1] (R \ Q)) {0}. On a sup A = 1, sup B = 1,
A B = {0} et donc sup(A B) = 0 < 1 = min{sup A, sup B}.
2) Pour x A B, on a x 6 max{sup A, sup B}.
Donc sup(A B) existe dans R et sup(A B) 6 max{sup A, sup B}.
Inversement, supposons par exemple sup A > sup B de sorte que max{sup A, sup B} = sup A.
Soit alors > 0. Il existe a A tel que sup A < a 6 sup A. De plus, a est dans A et donc dans A B.
En rsum,
1)x (A B), x 6 max{sup A, sup B} et
2) > 0, x (A B)/ max{sup A, sup B} < x.
Finalement, sup(A B) = max{sup A, sup B}.
3) Daprs lexerccie no 2, sup(A + B) = sup A + sup B.
4) Pour sup(AB), tout est possible. Par exemple, si A = B =] , 0] alors sup A = sup B = 0, mais AB = [0, +[ et
sup(AB) nexiste pas dans R.
Exercice no 6.

Montrons par rcurrence que n > 1,


Pour n = 1,

2n1
X
k=0

2n1
X
k=0

2n
X
(1)k
1
=
.
k+1
k
k=n+1

2n
X
(1)k
1
1
1
1
= 1 = et
= . Lidentit propose est donc vraie pour n = 1.
k+1
2
2
k
2
k=n+1

2n1
X

Soit n > 1. Supposons que

k=0

On a alors
2(n+1)1

k=0

2n
X
(1)k
1
=
.
k+1
k
k=n+1

2n1
X (1)k
(1)k
1
1
=
+

k+1
k+1
2n + 1 2n + 2
k=0

2n
X
1
1
1
+

(par hypothse de rcurrence)


k 2n + 1 2(n + 1)

k=n+1

2(n+1)
2n+1
2n+1
X 1
X 1
X 1
1
1
1
+

=
+
=
.
n+1
k 2(n + 1)
k 2(n + 1)
k
k=n+2

On a montr par rcurrence que n > 1,

2n1
X
k=0

k=n+2

k=n+2

2n
X
(1)k
1
=
(identit de Catalan).
k+1
k
k=n+1

Exercice n 7.
o

1) Si les bk sont tous nuls, lingalit est claire.


n
X
Sinon, pour tout rel x, f(x) =
(ak + xbk )2 =
k=1

Puisque lun au moins des bk nest pas nul, on a

n
X

k=1

n
X

b2k

x +2

n
X

a k bk

k=1

b2k > 0 et en particulier

k=1

http ://www.maths-france.fr

x+

n
X

k=1

n
X

a2k .

k=1

b2k 6= 0.

c Jean-Louis Rouget, 2014. Tous droits rservs.


f est donc un trinme du second degr, de signe constant sur R. Son discriminant rduit est donc ngatif ou nul ce qui
fournit :

0> =

n
X

a k bk

k=1

!2

n
X

a2k

k=1

n
X

k=1

b2k

v
v

uX
n
n
u
X
X
u n 2
u
t

2t
ak
bk , qui est lingalit de Cauchy-Schwarz.
ou encore
a k bk 6


k=1

k=1

k=1

2)

n
X

(ak + bk ) =

k=1

n
X

a2k

+2

k=1

n
X

k=1

n
X

a k bk +

k=1

n
X

k=1

b2k

n
X

a2k

k=1

k=1

v
v
u n
u n
n
X uX
X
u
2
2t
t
ak + 2
ak
b2k +
b2k
k=1

k=1

n

n
X
X


a k bk +
+ 2
b2k


k=1

(Cauchy-Schwarz)

k=1

v
2
v
u n
u n
uX 2 uX
= t
ak + t
b2k
k=1

k=1

v
v
v
uX
uX
uX
n
u n
u
u n 2
et donc, t
(ak + bk )2 6 t
a2k + t
bk , qui est lingalit de Minkowski.
k=1

k=1

k=1

Exercice no 8.

2

2

x 1 + 1 > 0. De mme, x 2 x 1 =
x 1 1 > 0.
Pour x > 1, x + 2 x 1 = x 1 + 2 x 1 + 1 =
p
p

Donc, si on pose f(x) = x + 2 x 1 + x 2 x 1, f(x) existe si et seulement x 1 et pour x > 1,


r
r




2
2


x1+1 +
x 1 11 = x 1 + 1 + x 1 1 .
f(x) =
Par suite,

f(x) = 1

x 1 + | x 1 1| = 0 x 1 = 0 et x 1 1 = 0 x 1 = 0 et x 1 = 1,

ce qui est impossible. Lquation propose na pas de solution.


Exercice no 9.

Soient xun rel et un rel strictement positif. On a 3 x < 3 x + . Puisque Q est dense dans R, il existe un rationnel r
tel que 3 x < r < 3 x + et donc tel que x < r3 < x + , par stricte croissance de la fonction t 7 t3 sur R.


Donc, r3 , r Q est dense dans R.

http ://www.maths-france.fr

c Jean-Louis Rouget, 2014. Tous droits rservs.


Planche no 18. Suites


* trs facile ** facile *** difficult moyenne **** difficile
I : Incontournable T : pour travailler et mmoriser le cours
Exercice no 1 (**IT)
Soient (un )nN une suite relle et (vn )nN la suite dfinie par :
n N, vn =

u0 + u1 + ... + un
.
n+1

1) Montrer que si la suite (un )nN converge vers un rel , la suite (vn )nN converge et a pour limite . Rciproque ?
2) Montrer que si la suite (un )nN est borne, la suite (vn )nN est borne. Rciproque ?
3) Montrer que si la suite (un )nN est croissante alors la suite (vn )nN lest aussi.
Exercice no 2 (***)
Soit (un )nN une suite relle. Montrer que si la suite (un )nN converge au sens de Csaro et est monotone, alors la suite
(un )nN converge.
Exercice no 3 (**IT)
n
X
1
Pour n entier naturel non nul, on pose Hn =
(srie harmonique).
k
k=1

1) Montrer que : n N , ln(n + 1) < Hn < 1 + ln(n) et en dduire

lim Hn .

n+

2) Pour n entier naturel non nul, on pose un = Hn ln(n) et vn = Hn ln(n + 1). Montrer que les suites (un ) et (vn )
1
, 1 ( est appele la constante dEuler). Donner une valeur approche de 102 prs.
convergent vers un rel
2
Exercice no 4 (**)
Soit (un )nN une suite arithmtique ne sannulant pas. Montrer que pour tout entier naturel n, on a
n
X
k=0

n+1
1
=
.
uk uk+1
u0 un+1

Exercice n 5 (***)
o

n
X

1
(on sera amen dterminer trois rels a, b et c tels que pour tout entier naturel non
12 + 22 + ... + k2
k=1
n
X
6
1
nul k,
et on utilisera lexercice no 3 : il existe une suite (n ) tendant vers 0 telle que
= ln n++n ).
k(k + 1)(2k + 1)
k
Calculer lim

n+

k=1

Exercice n 6 (**I) (moyenne arithmtico-gomtrique)


o

Soient a et b deux rels tels que 0 < a < b. On pose u0 = a et v0 = b puis, pour tout entier naturel n,
un+1 =

un + vn

un vn et vn+1 =
.
2

Montrer que les suites (un ) et (vn ) convergent vers une limite commune que lon ne cherchera pas calculer (cette limite
sappelle la moyenne arithmtico-gomtrique des nombres a et b).
Exercice no 7 (***)
Soient a et b deux rels tels que 0 < a < b. On pose u0 = a et v0 = b puis, pour tout entier naturel n,
un+1 =

un + vn

et vn+1 = un+1 vn .
2


 a 
b sin Arccos
 a b .
Montrer que les suites (un ) et (vn ) sont adjacentes et que leur limite commune est gale
Arccos
b
http ://www.maths-france.fr

c Jean-Louis Rouget, 2014. Tous droits rservs.


Exercice no 8 (**I)


un+1
tend vers un rel lment de [0, 1[ quand n tend

Soit (un )nN une suite relle ne sannulant pas. Montrer que si
un
vers +, alors un tend vers 0 quand n tend vers +.
Exercice no 9 (**)

Limite quand n tend vers + de

1)

sin n
n

n

1
2) 1 +
n

3)

n!
nn


2 !
1
n+
E
2
4)
2 !

1
E
n
2

n
5) n2

6) n + 1 n 7)

n
X

k2

k=1
n3

8)

n
Y

2k

2k/2 .

k=1

Exercice no 10 (**)
Etudier la suite (un ) dfinie par

1
.
n+1 n =
2 n + un

Exercice no 11 (**T) (Rcurrences homographiques).


Dterminer un en fonction de n quand la suite u vrifie :
un
3 2un

1) n N, un+1 =

2) un+1 =

4(un 1)
un

(ne pas se poser de questions dexistence).


Exercice no 12 (**)
Soient (un ) et (vn ) les suites dfinies par la donne de u0 et v0 et les relations de rcurrence
un + 2vn
2un + vn
et vn+1 =
.
3
3
Etudier les suites u et v puis dterminer un et vn en fonction de n en recherchant des combinaisons linaires intressantes
de u et v. En dduire lim un et lim vn .
un+1 =

n+

n+

Exercice n 13 (**)
o

Mme exercice avec un+1 =

un + wn
un + vn
vn + wn
et vn+1 =
et wn+1 =
.
2
2
2

Exercice no 14 (***)
Soit u une suite complexe et v la suite dfinie par vn = |un |.

On suppose que la suite ( n vn ) converge vers un rel positif .


Montrer que si 0 6 < 1, la suite (un ) converge vers 0 et si > 1, la suite (vn ) tend vers +.
Montrer que si = 1, tout est possible.
Exercice no 15 (***I)
1) Soit u une suite de rels strictement positifs. Montrer que si la suite
converge et a mme limite.

un+1
un

converge vers un rel , alors ( n un )

2) Etudier la rciproque.
p
1
n
3) Application : limites de a) n Cn
c) 2
2n b)
n
n
n!

r
n

(3n)!
.
n!

Exercice no 16 (*)

Soient u et v deux suites de rels de [0, 1] telles que

lim un vn = 1. Montrer que (un ) et (vn ) convergent vers 1.

n+

Exercice no 17 (**)
Montrer que si les suites (u2n ) et (u3n ) convergent alors (un ) converge.

http ://www.maths-france.fr

c Jean-Louis Rouget, 2014. Tous droits rservs.


Exercice no 18 (***T)
Etudier les deux suites un =

1+

1
n

n

et vn =

n+1

1
1+
.
n

Exercice no 19 (**T)
Mme exercice avec un =

n
X
1
1
et vn = un +
.
k!
n n!

k=0

Exercice n 20 (**T)
o

Mme exercice avec un =

n
X
1

k
k=1

2 n + 1 et vn =

n
X
1

k
k=1

2 n.

Exercice no 21 (**T)
Dterminer un en fonction de n et de ses premiers termes dans chacun des cas suivants :
1) n N, 4un+2 = 4un+1 + 3un .
2) n N, 4un+2 = un .
3) n N, 4un+2 = 4un+1 + 3un + 12.
4) n N,

un+2

un+1

1
.
un

5) n > 2, un = 3un1 2un2 + 2n .


Exercice no 22 (***)
Montrer que, pour n > 2,
  1q
  1q
p
p

2 + 2 + ... + 2 (n 1 radicaux) et sin n =


2 2 + ... + 2 (n 1 radicaux).
cos n =
2
2
2
2
r
q

n
En dduire lim 2
2 2 + ... + 2 (n radicaux).
n+

Exercice no 23 (***)

1) Montrer que pour x rel strictement positif, on a : ln(1 + x) < x < (1 + x) ln(1 + x).

k
k+1
n 
n 
n
Y
Y
n!
1
1
n
2) Montrer que
1+
<e <
1+
et en dduire la limite quand n tend vers + de
.
k
k
n
k=1

k=1

Exercice n 24 (****)
 
pn
avec pn Z et qn N , une suite de rationnels convergeant vers un irrationnel x. Montrer que les
Soit (un ) =
qn
suites (|pn |) et (qn ) tendent vers + quand n tend vers +.
o

Exercice no 25 (**)
Trouver un exemple de suite (un ) divergente, telle que k N \ {1}, la suite (ukn ) converge.
Exercice no 26 (***I)
Soit f une application injective de N dans N. Montrer que

lim f(n) = +.

n+

Exercice no 27 (***I)
Soit un lunique racine positive de lquation xn + x 1 = 0. Etudier la suite (un ).
Exercice no 28 (***)
Etude des suites (un ) = (cos na) et (vn ) = (sin na) o a est un rel donn.
a
1) Montrer que si
est rationnel, les suites u et v sont priodiques et montrer dans ce cas que (un ) et (vn ) convergent
2
si et seulement si a 2Z.

http ://www.maths-france.fr

c Jean-Louis Rouget, 2014. Tous droits rservs.


a
est irrationnel .
2
a) Montrer que (un ) converge si et seulement si (vn ) converge .
b) En utilisant diffrentes formules de trigonomtrie fournissant des relations entre un et vn , montrer par labsurde
que (un ) et (vn ) divergent.

2) On suppose dans cette question que

Exercice no 29 (***)
Calculer inf]0,[ (supnN (| sin(n)|)).
Exercice no 30 (**I)
Soit (un ) une suite relle non majore. Montrer quil existe une suite extraite de (un ) tendant vers +.
Exercice no 31 (***)
Soit (un ) une suite de rels lments de ]0, 1[ telle que n N, (1 un )un+1 >

http ://www.maths-france.fr

1
1
. Montrer que (un ) converge vers .
4
2

c Jean-Louis Rouget, 2014. Tous droits rservs.


Planche no 18. Suites : corrig


Exercice no 1

1) Soit > 0. Il existe un rang n0 tel que, si n > n0 alors |un | < . Soit n un entier naturel strictement suprieur
2
n0 .



n
n
1 X

1 X



uk =
(uk )
|vn | =
n + 1

n + 1
k=0

1
n+1

1
n+1

1
n+1

Maintenant,

n0
X

n
X

k=0
n0
X

k=0
n0
X

k=0

|uk | =

|uk | +
|uk | +

k=0

1
n+1

1
n+1

n0
X

|uk | +

k=0
n
X

k=n0 +1

n
X

1
n+1

6
2
n+1

|uk |

k=n0 +1
n0
X

|uk | +

k=0

n
1 X
n+1
2
k=0

|uk | est une expression constante quand n varie et donc,

n0
1 X
|uk | = 0. Par suite, il
n+ n + 1

lim

k=0

k=0

n0
1 X

existe un entier n1 > n0 tel que pour n > n1 ,


|uk | < .
n+1
2
k=0

Pour n > n1 , on a alors |vn | < + = .
2 2
On a montr que > 0, n1 N/ (n N)(n > n1 |vn | < ). La suite (vn ) est donc convergente et lim vn = .
n+

La rciproque est fausse. Pour n dans N, posons un = (1) . La suite (un ) est divergente. Dautre part, pour n dans N,
n
X
1
. Par suite, la suite (vn ) converge et
(1)k vaut 0 ou 1 suivant la parit de n et donc, dans tous les cas, |vn | 6
n+1
k=0
lim vn = 0.
n+

2) Si u est borne, il existe un rel M tel que, pour tout naturel n, |un | 6 M. Pour tout entier naturel n, on a alors
|vn | 6

n
n
1 X
1 X
1
(n + 1)M = M.
|uk | 6
M=
n+1
n+1
n+1
k=0

La suite v est donc borne.

k=0

n

p si n = 2p, p N
.
p si n = 2p + 1, p N
2
u nest pas borne car la suite extraite (u2p ) tend vers + quand p tend vers +. Mais, si n est impair, vn = 0, et si n
1 n
1 n
1 n+1
1
1
un =
, et dans tous les cas |vn | 6
6
= et la suite v est borne.
est pair, vn =
n+1
n+1 2
n+1 2
n+1 2
2
La rciproque est fausse. Soit u la suite dfinie par : n N, un = (1)n E

3) Si u est croissante, pour tout entier naturel n, on a :

n+1
X

k=0

k=0

n+1
n
1 X
1 X
1
vn+1 vn =
uk
uk =
n+2
n+1
(n + 1)(n + 2)
k=0

(n + 1)

k=0

X
1
1
((n + 1)un+1
uk ) =
(n + 1)(n + 2)
(n + 1)(n + 2)

k=0
n
X

uk (n + 2)

n
X

uk

k=0

(un+1 uk ) > 0.

La suite v est donc croissante.


Exercice no 2
Supposons sans perte de gnralit u croissante (quite remplacer u par u).
Dans ce cas, ou bien u converge, ou bien u tend vers +.
Supposons que u tende vers +, et montrons quil en est de mme pour la suite v.
Soit A R. Il existe un rang n0 tel que pour n naturel suprieur ou gal n0 , un > 2A. Pour n > n0 + 1, on a alors,
http ://www.maths-france.fr

c Jean-Louis Rouget, 2014. Tous droits rservs.


1
vn =
n+1

Maintenant, quand n tend vers +,

n0
X

n
X

uk +

uk

k=n0 +1

k=0

>

n0
1 X
(n n0 )2A
uk +
n+1
n+1
k=0

n0
(n n0 )2A
1 X
tend vers 2A et donc, il existe un rang n1 partir duquel
uk +
n+1
n+1
k=0

vn >

n0
1 X
(n n0 )2A
> A.
uk +
n+1
n+1
k=0

On a montr que : n N, n1 N/ (n N), (n > n1 vn > A). Par suite,

lim vn = +. Par contraposition, si

n+

v ne tend pas vers +, la suite u ne tend pas vers + et donc converge, daprs la remarque initiale.
Exercice no 3

1
est continue et dcroissante sur ]0; +[ et donc, pour k entier naturel non nul donn, on a :
x
Z k+1
1
1
1
1
1
= (k + 1 k)
6
dx =6 (k + 1 k) = .
k+1
k+1
x
k
k
k
Z k+1
Zk
1
1
1
1
dx et, pour k > 2, 6
dx.
Donc, pour k > 1, >
k
x
k
x
k
k1

1) La fonction x 7

En sommant ces ingalits, on obtient pour n > 1,

Z n+1
n Z k+1
n
X
X
1
1
1
>
dx =
dx = ln(n + 1),
Hn =
k
x
x
k
1
k=1

et pour n > 2,

k=1

Zn
n
n Zk
X
X
1
1
1
Hn = 1 +
61+
dx = 1 +
dx = 1 + ln n,
k
k1 x
1 x
k=2

k=2

cette ingalit restant vraie quand n = 1. Donc,

n N , ln(n + 1) 6 Hn 6 1 + ln n.
En particulier, pour n > 1, Hn > ln(n + 1). Puisque
2) Soit n un entier naturel non nul.
un+1 un =

lim ln(n + 1) + , on en dduit que

n+

1
1
ln(n + 1) + ln n =

n+1
n+1

Z n+1
n

1
dx =
x

Z n+1 
n

lim Hn = +

n+

1
1

n+1 x

dx 6 0

1
1
1
dcroit sur [n, n + 1] et donc, pour tout x de [n, n + 1],
6 0. De mme,
x
n+1 x

Z n+2
Z n+2 
1
1
1
1
1
vn+1 vn =
dx 0
ln(n + 2) + ln(n + 1) =

dx =

n+1
n+1
n+1 x
n+1 x
n+1

car la fonction x 7
car la fonction x 7

1
dcroit sur [n + 1, n + 2]. Enfin,
x


1
un vn = ln(n + 1) ln n = ln 1 +
n

et donc la suite u v tend vers 0 quand n tend vers +.


En rsum, la suite u dcrot, la suite v crot et la suite uv tend vers 0. On en dduit que les suites u et v sont adjacentes,
et en particulier convergentes et de mme limite. Notons cette limite.
Pour tout entier naturel non nul n, on a vn 6 6 un , et en particulier, v3 6 6 u1 avec v3 = 0, 5... et u1 = 1. Donc,
1
,1 .

2
Pour n entier naturel non nul donn, on a :

http ://www.maths-france.fr

c Jean-Louis Rouget, 2014. Tous droits rservs.


0 6 un vn 6 10



1
1
1
6 102 6 e0,01 1 n > 0,01
= 99, 5... n > 100.
ln 1 +
n
n
e
1

Donc 0 6 v100 6 102 . On trouve = 0, 57 102 prs par dfaut. Plus prcisment, = 0, 5772156649... ( est la
constante dEuler).
Exercice no 4
Soit r la raison de la suite u. Pour tout entier naturel k, on a
obtient :

n
X

k=0

1
1
1

= r
. En sommant ces galits, on
uk
uk+1
uk uk+1


n 
X
1
1
1
=

uk uk+1
uk uk+1
k=0

1
1

(somme tlescopique)
u0 un+1
un+1 u0
(n + 1)r
=
=
.
u0 un+1
u0 un+1
=

Si r 6= 0, on obtient
Exercice no 5

n
X

k=0

(n + 1)
1
=
, et si r = 0, u est constante et le rsultat est immdiat.
uk uk+1
u0 un+1

Soit k un entier naturel non nul. On sait que

k
X

i2 =

i=1

pour entier naturel non nul k,

k(k + 1)(2k + 1)
. Dterminons alors trois rels a, b et c tels que,
6

a
b
c
6
= +
+
().
k(k + 1)(2k + 1)
k k + 1 2k + 1
Pour k entier naturel non nul donn,
a
b
c
a(k + 1)(2k + 1) + bk(2k + 1) + ck(k + 1)
+
+
=
k k + 1 2k + 1
k(k + 1)(2k + 1)
=

(2a + 2b + c)k2 + (3a + b + c)k + a


.
k(k + 1)(2k + 1)

Par suite,

2a + 2b + c = 0
a=6
3a + b + c = 0
b=6
()
,

a=6
c = 24

et donc,

n N ,

n
X

k=1

6
=6
k(k + 1)(2k + 1)

n
n
n
X
X
1
1 X 1
+
4
k
k+1
2k + 1

k=1

k=1

k=1

Ensuite, daprs lexercice no 3, il existe une suite (n ) tendant vers 0 quand n tend vers + telle que, pour tout entier
n
X
1
naturel non nul n,
= ln n + + n . Pour tout entier naturel non nul n, on a
k
k=1

n
X

k=1

et donc

n+1
X1
1
1
1
=
= Hn 1 +
= ln n + 1 +
+ n ,
k+1
k
n+1
n+1
k=2

n
n
X
1
1 X 1
+
= 2 ln n + 2 1 +
+ 2n .
k
k+1
n+1

k=1

Dautre part,
http ://www.maths-france.fr

k=1

c Jean-Louis Rouget, 2014. Tous droits rservs.


n
X

k=1

2n+1
n
X 1 X
1
1
1
= 1 +

= 1 + H2n+1 Hn
2k + 1
k
2k
2
k=1

k=1

1
1
= ln(2n + 1) + (ln n + ) 1 + 2n+1 n
2
2


1
1
1
1
+ ln n 1 + 2n+1 n
= ln 2 + ln n + ln 1 +
2n
2
2
2


1
1
1
1
= ln n + ln 2 + 1 + ln 1 +
+ 2n+1 n
2
2
2n
2

et finalement,
n
X





1
1
1
1
= 6 (2 ln n + 2 1) 24
ln n + ln 2 + 1 24 ln 1 +
242n+1
12 + 22 + ... + k2
2
2
2n
k=1


1
242n+1 .
= 6(3 4 ln 2) 24 ln 1 +
2n
Donc,
lim

n+

n
X

k=1

1
= 6(3 4 ln 2).
12 + 22 + ... + k2

Exercice no 6
Puisque a et b sont positifs, par rcurrence, pour tout entier naturel n, un et vn existent et sont positifs.
Pour tout entier naturel non nul n,

vn un =

1
1
1

2
(un1 + vn1 ) un1 vn1 = (un1 2 un1 vn1 + vn1 ) = ( vn1 un1 ) > 0.
2
2
2

Ceci reste vrai quand n = 0 car a < b et donc, pour tout entier naturel n, un 6 vn .
Ensuite, pour tout entier naturel n,

un+1 un = un vn un = un ( vn un ) > 0 ;
1
1
vn+1 vn = (un + vn ) vn = (un vn ) 6 0.
2
2
Ainsi, la suite (un ) est croissante et la suite (vn ) est dcroissante.
Dautre part, pour tout entier naturel n, un 6 vn 6 v0 et donc la suite (un ) est majore par v0 . De mme la suite (vn )
est minore par u0 . On en dduit que les suites (un ) et (vn ) convergent vers des rels positifs.
1
Notons et les limites respectives des suites (un ) et (vn ). On fait tendre n vers + dans lgalit (un + vn ) = un+1
2
1
et on obtient ( + ) = ou encore = .
2
Exercice no 7
i h
a
a
< 1 et est lment de 0, . De plus, a = b cos . Enfin, pour tout entier
Posons = Arccos . existe car 0 <
bh
b
2
i

naturel n, n 0,
et donc, cos n > 0.
2
2
2
r
1

2
On a u0 = b cos et v0 = b puis u1 = (u0 + v0 ) = (1 + cos ) = b cos
et v1 = u1 v0 = b cos2 b = b cos
2
2
2
2
2
r

b
1 + cos
= b cos cos2 2 puis v2 = b cos cos2 2 b cos = b cos cos 2 .
puis u2 = cos
2
2
2
2
2
2
2
2
2
2
Montrons par rcurrence que pour tout entier naturel non nul n, vn = b

n
Y

k=1

http ://www.maths-france.fr

cos

et un = vn cos n .
k
2
2

c Jean-Louis Rouget, 2014. Tous droits rservs.


un+1
puis
vn+1 =
et donc vn+1 = b

n+1
Y

cos

k=1

n
Y

et un = vn cos n alors,
2k
2
k=1


1
vn cos n + vn = vn cos2 n+1
=
2
2
2

Cest vrai pour n = 1 et si pour n > 1, on a vn = b

cos

un+1 vn = vn cos

(car cos n+1 > 0),


2n+1
2

puis un+1 = vn+1 cos n+1 .


2k
2

On a montr par rcurrence que

n N , vn = b

n
Y

cos

k=1

et un = vn cos n .
2k
2

vn+1

Pour tout entier naturel non nul n, on a vn > 0 et


= cos n+1 < 1. La suite v est donc strictement dcroissante.
vn
2
Ensuite, pour tout entier naturel non nul n, on a un > 0 et

2
cos
cos
1 1
vn+1
un+1
2n+1 =
2n+1 = 1 1 +
=
> (1 + 1) = 1.

un
vn cos
2
2
cos
cos
2n
2n
2n
La suite u est strictement croissante. Maintenant, pour n N ,

n sin
Y

k1
2
vn = b
cos k = b

2
k=1
k=1 2 sin k
2
b sin
1
b sin
  .
=

=

2n sin n
sin n / n
2
2
2
n
Y

b sin
1
b sin

b sin
lim
=
, puis un = vn cos n
.
X0 sin X/X

b sin
b2 a 2
Ainsi, les suites u et v sont adjacentes de limite commune
=
a.

Arccos
b
Exercice no 8


un+1
1
un+1
6 + 1 =
[0, 1[ et donc
= , il existe un rang n0 tel que pour tout n > n0 ,
> 0. Puisque lim
n+ un
2
un
2
1+
.
2
Soit n > n0 + 1.
Donc,

lim vn =

n+

|un | = |un0 |

n1
Y

k=n0

|un0 |

n1
Y

k=n0



uk+1


uk (produit tlescopique)

1+
= |un0 |
2

1+
2

nn0

1+1
1+
<
= 1. Par suite, quand n tend vers +, |un0 |
Puisque [0, 1[,
2
2
lim un = 0.

1+
2

nn0

tend vers 0 et donc

n+

Exercice no 9


sin n

6 1 . Comme 1
1) Pour n N ,
n n
n

http ://www.maths-france.fr

n+

0, on en dduit que
5

sin
n

n+

0.

c Jean-Louis Rouget, 2014. Tous droits rservs.




1
n 

 ln 1 +

1
1
n
= n ln 1 +
=
2) ln
1+
1
n
n
n

n
1
1+
= en ln(1+1/n) tend vers e1 = e.
n

n+

1. Donc, ln

n 

1
tend vers 1 puis
1+
n

n!
. Pour n entier naturel non nul, on a
nn

n 
n 
n
n+1
n
1
(n + 1)!
nn
un+1
=
= 1+
.
=

=
un
n!
(n + 1)n+1
n+1
n
n


1


ln 1 +
1
ln (1 + X)
n
lim n ln 1 +
= lim
= 1 et donc
= lim
1
n+
n+
X0
n
X
n

3) Pour n N , posons un =

un+1
= lim en ln(1+1/n) = e1 .
n+
n+ un
lim

Ainsi,

un+1
1
tend vers = 0, 36... < 1. On sait alors que
un
e

lim un = 0 (voir exercice no 8).

n+

4) Pour n > 1,
2
2


1
1
1
n+
n+
2
2
.
2 6 un 6 
2

1
1
1
n
n
2
2
2
2


1
1
1
n+
n+
2
2
tendent vers 1 quand n tend vers + et donc, daprs le thorme de la limite par
Or, 
2 et 
2
1
1
1
n
n
2
2
encadrement, la suite u converge et a pour limite 1.

2
1
2 ln n
n
n
5) n2 = e n ln(n ) = e n . Donc quand n tend vers +, n2 tend vers e0 = 1.

1
6) n + 1 n =
0.
n+1+ n
7)

n
1 X 2 n(n + 1)(2n + 1)
2
1
= .
k =
n3
6n3
6
3
k=1

8)

n
Y

2k/2 = 2 2

Pn

k
k=1 2k1

. Pour x rel, posons f(x) =

k=1

f(x) =

n
X

k=1

Pour x 6= 1, on a donc
f(x) =

En particulier,

n
X

k=1

k
2k1

xn+1 1
x1

(x) =

n
X

kxk1 . Pour tout rel x,

!k=1

(x) =

n
X

k=0

(x).

(n + 1)xn (x 1) (xn+1 1)
nxn+1 (n + 1)xn + 1
=
.
(x 1)2
(x 1)2

n
n+1
 
n +1
n+1
1
2
2
=
=f
4 (daprs un thorme de croissances compares). Finalement,

2
2
1
1
2
n
Y
k
1
2k/2 2 2 4 = 4.
k=1

http ://www.maths-france.fr

c Jean-Louis Rouget, 2014. Tous droits rservs.


Exercice no 10
Soit n N.

1
1

= n + 1 n 2 n + un =
2 n + un = n + 1 + n
2 n + un
n+1 n


p
2
1
4 (n + un ) =
n + 1 + n un = n +
2n + 1 + 2 n(n + 1)
4

p
1
2n + 1 + 2 n(n + 1) .
un =
4

Par suite, quand n tend vers +,


!
r
1
1 n
1/n
1
1 n
1 1
1 1
1
1+ 1 = + r
un = +
= + r
+ = .
4
2
n
4
2
4 2
4 4
2
1
1
1+ +1
1+ +1
n
n
La suite (un ) converge et a pour limite

1
.
2

Exercice no 11
x
= x 2x2 2x = 0 x = 0 ou x = 1.
3 2x
Pour n entier naturel donn, on a alors

1) Calcul formel de un . Soit x R.

un
1
3un 3
un 1
un+1 1
3 2un
=
=
=3
.
un
un+1
un
un
3 2un

Par suite, pour tout entier naturel n,

un 1
u0 1
u0
= 3n
, puis un =
.
un
u0
u0 3n (u0 1)

4(x 1)
2) Calcul formel de un . Soit x R.
= x x2 4x + 4 = 0 x = 2.
x
Pour n entier naturel donn, on a alors
1

un+1 2

Par suite, pour tout entier naturel n,

un
1
un 2 + 2
1
1
=
=
= +
.
4(un 1)
2(un 2)
2(un 2)
2 un 2
2
un

1
n
1
2(u0 2)
= +
puis un = 2 +
.
un 2
2
u0 2
(u0 2)n + 2

Exercice no 12

un+1 un = (vn un )

1
Pour tout entier naturel n, on a
vn+1 vn = (vn un ) .

vn+1 un+1 = 1 (vn un )


3
La dernire galit montre que la suite v u garde un signe constant puis, puisque pour tout naturel n,
sgn(un+1 un ) = sgn(vn un ) et sgn(vn+1 vn ) = sgn(vn un ),
les suites u et v sont monotones de sens de variation opposs.
Si par exemple u0 6 v0 , alors, pour tout naturel n, on a :
u0 6 un un+1 6 vn+1 6 vn 6 v0 .
Dans ce cas, la suite u est croissante et majore par v0 et donc converge vers un certain rel . De mme, la suite v est
dcroissante et minore par u0 et donc converge vers un certain rel . Enfin, puisque pour tout entier naturel n, on a
2 +
2un + vn
, on obtient par passage la limite quand n tend vers linfini, =
et donc = . Les suites u
un+1 =
3
3
et v sont donc adjacentes. Si u0 > v0 , il suffit dchanger les rles de u et v.

http ://www.maths-france.fr

c Jean-Louis Rouget, 2014. Tous droits rservs.


1
Calcul des suites u et v. Pour n entier naturel donn, on a vn+1 un + 1 = (vn un ). La suite v u est gomtrique
3
1
1
de raison . Pour tout naturel n, on a donc vn un = n (v0 u0 ).
3
3
Dautre part, pour n entier naturel donn, vn+1 + un+1 = vn + un . La suite v + u est constante et donc, pour tout entier
naturel n, on a vn + un = v0 + u0 .
En additionnant et en retranchant les deux galits prcdentes, on obtient pour tout entier naturel n :




1
1
1
1
un =
v0 + u0 n (v0 u0 ) et vn =
v0 + u0 + n (v0 u0 ) .
2
3
2
3
En particulier, = =

u0 + v0
.
2

Exercice no 13
1
Pour tout entier naturel n, on a un+1 vn+1 = (un vn ) et donc, pour tout entier naturel n,
2
 n
1
(u0 v0 ).
un vn =
2
 n
 n
1
1
(v0 w0 ) et wn un =
(w0 v0 ) (attention,
De mme, en changeant les rles de u, v et w, vn wn =
2
2
cette dernire galit nest autre que la somme des deux premires et il manque encore une quation).
On a aussi, un+1 + vn+1 + wn+1 = un + vn + wn et donc, pour tout naturel n, un + vn + wn = u0 + v0 + w0 .
Ainsi, un , vn et wn sont solutions du systme


n
1

u
=

(v0 u0 )
n
n

 n
1
.

wn un =
(w0 u0 )

un + vn + wn = u0 + v0 + w0

Par suite, pour tout entier naturel n, on a


 n


un = 1 (u0 + v0 + w0 ) + 1
(2u

w
)

0
0
0

3

 2n


1
1
.
(u0 + v0 + w0 ) +
vn =
(u0 + 2v0 w0 )

3 
2 




1
1

wn =
(u0 v0 + 2w0 )
(u0 + v0 + w0 ) +
3
2
En particulier, les suites u, v et w convergent vers

u0 + v0 + w0
.
3

Exercice no 14

Supposons que la suite ( n vn ) tende vers le rel positif .


1
. est un rel strictement positif et donc,
2

1
1+
n0 N/ n N, (n > n0 n vn < +
=
).
2
2

n
1+
1+
1+1
Pour n > n0 , par croissance de la fonction t 7 tn sur R+ , on obtient |un | <
. Or, 0 <
<
= 1 et
2
2
2
n

1+
tend vers 0 quand n tend vers +. Il en rsulte que un tend vers 0 quand n tend vers +.
donc
2

1er cas. Supposons que 0 6 < 1. Soit =

2me cas. Supposons que > 1.

1
1+
n
vn >
=
).
2
2
n
n


1+
1+1
1+
1+
. Or,
tend vers + quand n tend vers
>
= 1, et donc
Mais alors, pour n > n0 , |un | >
2
2
2
2
+. Il en rsulte que |un | tend vers + quand n tend vers +.
n0 N/ n N, (n > n0

http ://www.maths-france.fr

c Jean-Louis Rouget, 2014. Tous droits rservs.



ln n
Soit, pour rel et n entier naturel non nul, un = n . n un = e n tend vers 1 quand n tend vers +, et ceci pour
toute valeur de . Mais, si < 0, un tend vers 0, si = 0, un tend vers 1 et si > 0, un tend vers +. Donc, si = 1,
on ne peut rien conclure.
Exercice no 15
1) Supposons > 0. Soit un rel strictement positif, lment de ]0, 2[.
n0 N/ n N, (n > n0

un+1

< + ).
<
2
un
2

n1
Y



nn0
nn0
uk+1
6 un 6 un0 +
, et donc
, on a un0
uk
2
2
k=n0



n0 /n 


n0 /n 

6 n un 6 (un0 )1/n +
+
.
(un0 )1/n
2
2
2
2

Maintenant, le membre de gauche de cet encadrement tend vers , et le membre de droite rend vers + . Par
2
2

suite, on peut trouver un entier naturel n1 > n0 tel que, pour n > n1 , (un0 )1/n ( )n0 /n ( ) > , et
2
2

(un0 )1/n ( + )n0 /n ( + ) < + . Pour n > n1 , on a alors < n un < + .


2
2

On a montr que > 0, n1 N/ (n N), (n > n1 < n un < + ) et donc, n un tend vers .
un+1

Soit un rel strictement positif. Si = 0, il existe un rang n0 tel que pour n > n0 , 0 6
< . Pour n > n0 ,
un
2
 1n0 /n

1/n
0 6 n un 6 (un0 )
.
2

Quand n tend vers +, le membre de droite tend vers est est donc strictement plus petit que partir dun certain
2
rang n1 .
Pour n > n0 , puisque un = un0

2) Soient a et b deux rels tels que 0 < a < b. Soit u la suite dfinie par
p N, u2p = ap bp et u2p+1 = ap+1 bp .

(on part de 1 puis on multiplie alternativement par a ou b).

p
p+1

Alors, 2p u2p = ab et 2p+1 u2p+1 = a 2p+1 b 2p+1 ab.

Donc, n un tend vers ab (et en particulier converge).


u2p+1
u2p+2
un+1
On a bien sr
= a et
= b. La suite (
) admet donc deux suites extraites convergentes de limites
u2p
u2p+1
un
distinctes et est ainsi divergente. La rciproque du 1) est donc fausse.
 
2n
3) a) Pour n entier naturel donn, posons un =
.
n
un+1
(2n + 2)!
(2n + 2)(2n + 1)
4n + 2
n!2
=
=
=

.
2
2
un
(2n)!
(n + 1)!
(n + 1)
n+1
s 
2n
un+1
tend vers 4 quand n tend vers +, et donc n
tend vers 4 quand n tend vers +.
Ainsi,
un
n
nn
.
n!

n
(n + 1)n+1
n!
(n + 1)n
1
un+1
.
=

=
= 1+
un
nn
(n + 1)!
nn
n
n

tend vers e quand n tend vers +, et donc n un =


tend vers e quand n tend vers +.
n
n!

b) Pour n entier naturel donn, posons un =

Ainsi,

un+1
un

c) Pour n entier naturel donn, posons un =

(3n)!
.
n2n n!

un+1
(3n + 3)!
n!
n2n
(3n + 3)(3n + 2)(3n + 1)
=

=
un
(3n)!
(n + 1)2n+2
(n + 1)!
(n + 1)2 (n + 1)
2n

1
3(3n + 2)(3n + 1)
1+
=
.
(n + 1)2
n
http ://www.maths-france.fr

n
n+1

2n

c Jean-Louis Rouget, 2014. Tous droits rservs.



2n
1
un+1
Maintenant, 1 +
= e2n ln(1+1/n) = e2 ln(1+1/n)/(1/n) tend vers e2 , et donc
tend vers 27e2 . Par suite,
n
u
n
r
27
1 n (3n)!
tend vers 2 .
2
n
n!
e
Exercice no 16
Daprs le thorme de la limite par encadrement :
0 6 un vn 6 un 6 1 u converge et tend vers 1.

Il en est de mme pour v en changeant les rles de u et v


Exercice no 17

p
|u2n | 0 et donc un 0.
 3
un
2
Si un 6= 0, alors (un ) =
converge.
u2n
Remarque. Lexercice
pna dintrt que si lapsuite u est une suite complexe, car si u est une suite relle, on crit
immdiatement un = 3 u3n (et non pas un = u2n ).
Si u2n 0, alors |un | =

Exercice no 18

Les suites u et v sont dfinies partir du rang 1 et strictement positives.


Pour tout naturel non nul n, on a :
un+1
=
un

n+2
n+1

n+1 

n
n+1

n

= e(n+1) ln(n+2)+n ln n(2n+1) ln(n+1) .

Pour x rel strictement positif, posons alors f(x) = (x + 1) ln(x + 2) + x ln x (2x + 1) ln(x + 1).
f est drivable sur ]0, +[ et pour x > 0,

f (x) =

2x + 1
1
1
x+1
+ ln(x + 2) + 1 + ln x
2 ln(x + 1) =
+
+ ln x + ln(x + 2) 2 ln(x + 1).
x+2
x+1
x+2 x+1

De mme, f est drivable sur ]0, +[ et pour x > 0,

f (x) =

1
1
1
1
2

+ +

(x + 2)2
(x + 1)2
x x+2 x+1

x(x + 1)2 x(x + 2)2 + (x + 1)2 (x + 2)2 + x(x + 1)2 (x + 2) 2x(x + 1)(x + 2)2
x(x + 1)2 (x + 2)2
2
2
2
2x 3x + (x + 2x + 1)(x + 4x + 4) + (x2 + 2x)(x2 + 2x + 1) 2(x2 + x)(x2 + 4x + 4)
=
x(x + 1)2 (x + 2)2
3x + 4
> 0.
=
x(x + 1)2 (x + 2)2

f est strictement croissante sur ]0, +[ et donc, pour x > 0,




1
1
t(t + 2)
= 0.
+
+ ln
f (x) < lim f (t) = lim
t+
t+
t+2 t+1
(t + 1)2
Donc, f est strictement dcroissante sur ]0, +[. Or, pour x > 0,
f(x) = (x + 1) ln(x + 2) + x ln x (2x + 1) ln(x + 1)




1
2
(2x + 1) ln 1 +
= (x + (x + 1) (2x + 1)) ln x + (x + 1) ln 1 +
x
x




2
1




ln 1 +
ln 1 +
2
1
x
x
= ln 1 +
ln 1 +
+2
2
.
2
1
x
x
x
x

http ://www.maths-france.fr

10

c Jean-Louis Rouget, 2014. Tous droits rservs.


ln(1 + u)
= 1, et donc, quand x tend vers +, f(x) tend vers 0 0 + 2 2 = 0. Comme f est strictement
u
dcroissante sur ]0, +[, pour tout rel x > 0, on a f(x) > lim f(t) = 0.
t+
un+1
= ef(n) > 1. La suite u est
f est donc strictement positive sur ]0, +[. En particulier, n N , f(n) > 0 et donc
un
strictement croissante.
x

1
sur ]0, +[.
Remarque. On pouvait aussi tudier directement la fonction x 7 1 +
x
On sait que lim

u0

Je vous laisse
de manire analogue que la suite v est strictement dcroissante. Enfin, puisque un tend vers e, et
 montrer

1
que vn = 1 +
un tend vers e, les suites u et v sont adjacentes.
n
n
n+1


1
1
< e < 1+
. Par exemple, pour
Remarque. En consquence, pour tout entier naturel non nul n, 1 +
n
n
n = 10, on obtient


11
10

10

<e<

11
10

11

et donc, 2, 59... < e < 2, 85... et pour n = 100, on obtient 1, 01100 < e < 1, 01101 et donc 2, 70... < e < 2, 73... Ces deux
suites convergent vers e lentement.
Exercice no 19
Il est immdiat que u croit strictement et que v u est strictement positive et tend vers 0. De plus, pour n entier naturel
non nul donn,

vn+1 vn =

1
1
n(n + 1) + n (n + 1)2
1
1
+

=
=
< 0,
(n + 1)! (n + 1) (n + 1)! n n!
n(n + 1) (n + 1)!
n(n + 1) (n + 1)!

et v est strictement dcroissante. Les suites u et v sont donc adjacentes et convergent vers une limite commune ( savoir
e).
Remarque. Dans ce cas, la convergence est trs rapide. On a pour tout entier naturel non nul n,
n
n
X
X
1
1
1
<e<
+
k!
k! n n!

k=0

k=0

et n = 7 fournit par exemple 2, 71825... < e < 2, 71828...).


Exercice no 20
Pour n entier naturel non nul donn, on a

1
2
1

un+1 un =
2 n+2+2 n+1 =

n+1
n+1
n+1+ n+2
2
1

= 0.
>
n+1
n+1+ n+1
De mme,

1
1
2
2 n+1+2 n =
vn+1 vn =

n+1
n+1
n+1+ n
1
2

<

= 0.
n+1
n+1+ n+1
La suite u est strictement croissante et la suite v est strictement dcroissante. Enfin,

,
vn un = 2 n + 1 2 n =
n+ n+1
et la suite v u converge vers 0. Les suites u et v sont ainsi adjacentes et donc convergentes, de mme limite .

http ://www.maths-france.fr

11

c Jean-Louis Rouget, 2014. Tous droits rservs.


Exercice no 21
3
1
1) Lquation caractristique est 4z2 4z3 = 0. Ses solutions sont et . Les suites cherches sont les suites de la forme
2
2
  n
 n 
1
3
(un ) =
o et sont deux rels (ou deux complexes si on cherche toutes les suites complexes).
+
2
2

+ = u0
Si u0 et u1 sont les deux premiers termes de la suite u, et sont les solutions du systme
et donc
3
+
= u1
2
2
1
1
= (3u0 2u1 ) et = (u0 + 2u1 ) daprs les formules de Cramer.
4
4
n

 n
1
1
1
3
+ (u0 + 2u1 )
.
n N, un = (3u0 2u1 )
4
2
4
2


u0
u1
1 1 + (1)n
1 (1)n
2) Clairement u2n = n et u2n+1 = n et donc un =
u0 + 2
u1 .
4
4
2
2n
2n
 n
 n
1
3
3) Les solutions de lquation homogne associe sont les suites de la forme
+
.
2
2
Une solution particulire de lquation propose est une constante a telle
3a +12 et 
donc a = 4.
 que 4a= 4a+
n
n
3
1
o et sont les
+
Les solutions de lquation propose sont donc les suites de la forme 4 +
2
2

+ = 4 + u0
1
1
solutions du systme
et donc = (4 + 3u0 2u1 ) et = (12 + u0 + 2u1 ).
3
+
= 4 + u1
4
4
2
2
n

 n
1
1
3
1
+ (12 + u0 + 2u1 )
.
n N, un = 4 + (4 + 3u0 2u1 )
4
2
4
2
!n !
!n
1
1i 7
1+i 7
4) La suite v = est solution de la rcurrence 2vn+2 = vn+1 vn et donc, (vn ) est de la forme
+
u
4
4
1
et donc un =
!n
!n .
1+i 7
1i 7

+
4
4
5) Les solutions de lquation homogne associe sont les suites de la forme ( + 2n ), (, ) R2 .
2 est racine simple de lquation caractristique et donc il existe une solution particulire de lquation propose de la
forme un = an2n . Pour n > 2, on a
un 3un1 + 2un2 = a(n + 2)2n+2 3a(n + 1)2n+1 + 2an2n
= (n(4a 6a + 2a) + (8a 6a)) 2n 2a 2n .
Donc

u est solution 2a = 1 a =

Les suites cherches sont les suites de la forme n2n1 + + 2n


Exercice no 22

nN

1
.
2

, (, ) R2 .

Lgalit propose est vraie pour n = 2 car cos 2 = cos =


.
2
4
2
  1q
p

2 + 2 + ... 2 (n 1 radicaux).
Soit n > 2. Supposons que cos n =
2
2
i h
 

Alors, puisque cos n+1 > 0 (car n+1 est dans 0, ),


2
2
2
v
 v
u
r
r
!
u
q
q
  u
u1

t 1 + cos 2n
1
1
t
cos n+1 =
1+
2 + 2 + ... 2 =
2 + 2 + ... 2, (n radicaux).
=
2
2
2
2
2
  1q
p

On a montr par rcurrence que, pour n > 2, cos n =


2 + 2 + ... 2 (n 1 radicaux).
2
2
http ://www.maths-france.fr

12

c Jean-Louis Rouget, 2014. Tous droits rservs.


Ensuite, pour n > 2,

Enfin,

  r1
1

(1 cos( n1 ) =
sin n =
2
2
2
2

sin x
= 1. Donc,
x0 x
Exercice no 23
car lim

2 2 + ... 2 (n 1 radicaux)

lim 2n

n+

2 2 + ... 2 = 2n 2 sin

sin

2n+1

2n+1 ,

2n+1

2 2 + ... 2 = .

1) Pour x rel positif, posons f(x) = x ln(1 + x) et g(x) = (x + 1) ln(x + 1) x. f et g sont drivables sur [0, +[ et pour
x > 0, on a
f (x) = 1

1
x
=
> 0,
x+1
x+1

et
g (x) = ln(x + 1) + 1 1 = ln(x + 1) > 0.
f et g sont donc strictement croissantes sur [0, +[ et en particulier, pour x > 0, f(x) > f(0) = 0 et de mme, g(x) >
g(0) = 0. Finalement, f et g sont strictement positives sur ]0, +[ ou encore,
x > 0, ln(1 + x) < x < (1 + x) ln(1 + x).

2) Soit k un entier
non nul.

 






 naturel
1
1
1
1
1
1
<
ln 1 +
, ce qui fournit k ln 1 +
< 1 < (k + 1) ln 1 +
, puis, par
< 1+
Daprs 1), ln 1 +
k
k
k
k
k
k
stricte croissance de la fonction exponentielle sur R,
k
k+1


1
1
<e< 1+
.
k N , 0 < 1 +
k
k

En multipliant membre membre ces encadrements, on obtient pour tout naturel non nul n :
k
k+1
n 
n 
Y
Y
1
1
n
<e <
.
1+
1+
k
k
k=1

k=1

Maintenant,

n 
Y

k=1

1
1+
k

k

k
n 
Y
k+1
k

k=1

n+1
Y

kk1

k=2
n
Y

=
kk

(n + 1)n
.
n!

k=1

De mme,

n+1
Y
kk


n
k+1
Y
(n + 1)n+1
1
k=2
= n
.
=
1+
Y
k
n!
k+1
k=1
k
k=1

On a montr que n N ,

(n + 1)
n!

n+1

< en <

(n + 1)
n!

et donc

n
n!
1n+1
1n+1
<
<
(n + 1)1/n .
n N ,
e n
n
e n
n+1
tend vers 1 quand n tend vers linfini de mme que
Daprs le thorme de la limite par encadrements, comme
n

n
1
n!
(n + 1)1/n = eln(n+1)/n , on a montr que
tend vers quand n tend vers +.
n
e
http ://www.maths-france.fr

13

c Jean-Louis Rouget, 2014. Tous droits rservs.


Exercice no 24
Soit x un irrationnel et
la fraction

pn
qn

une suite de rationnels tendant vers x (pn entier relatif et qn entier naturel non nul,

nN

pn
ntant pas ncessairement irrductible). Supposons que la suite (qn )nN ne tende pas vers +. Donc :
qn
A > 0/ (n0 N)(n > n0 / qn > A)

ou encore, il existe une suite extraite (q (n))nN de la suite (qn )nN qui est borne.
La suite (q (n))nN est une suite dentiers naturels qui est borne, et donc cette suite ne prend quun nombre fini de
valeurs. Lune au moins de ces valeurs est prise un infinit de fois car sinon la suite (q (n))nN naurait quun nombre
fini de termes. Mais alors, on peut extraire de la suite (q (n))nN et donc de la suite (qn )nN une suite (q(n) )nN qui
est constante et en particulier convergente.


p(n)
La suite (p(n) )nN =
(q(n) )nN est aussi une suite dentiers relatifs convergente et est donc constante
q(n) nN
partir dun certain rang.

Ainsi, on peut extraire de la suite p(n) nN et donc de la suite (pn )nN une suite (p(n) )nN constante. La suite
((q
)
est galement constante car extraite de la suite constante (q(n) )nN et finalement, on a extrait de la suite

 (n)
 nN

p(n)
pn
constante.
une sous suite
qn nN
q(n) nN
 




p(n)
p(n)
pn
Mais la suite
tend vers x. Puisque
est constante,
tend vers x et donc la suite extraite
qn nN
q(n) nN
q(n) nN
p(n)
= x et donc x est rationnel. Contradiction .
on a n N,
q(n)


pn
pn


Donc la suite (qn )nN tend vers +. Enfin, puisque |pn | = qn et que tend vers |x| > 0 (car x est irrationnel)
qn
qn
et qn tend vers +, la suite (|pn |)nN tend vers +.
Exercice no 25

On pose u0 = 0, u1 = 0, u2 = 1, u3 = 1, u4 = 0, u5 = 1, u6 = 0, u7 = 1, u8 = 0, u9 = 0, u10 = 0, u11 = 1... et plus


gnralement

0 si n nest pas premier
n > 2, un =
.
1 si n est premier
Soit k un entier naturel suprieur ou gal 2. Pour n > 2, lentier kn est compos et donc, pour n > 2, ukn = 0. En
particulier, la suite (ukn )nN converge et a pour limite 0. Maintenant, lensemble des nombres premiers est infini et si
pn est le n-ime nombre premier, la suite (pn )nN est strictement croissante. La suite (upn )nN est extraite de (un )nN
et est constante gale 1. En particulier, la suite (upn )nN tend vers 1. Ainsi la suite (un )nN admet au moins deux
suites extraites convergentes de limites distinctes et donc la suite (un )nN diverge bien que toutes les suites (ukn )nN
convergent vers 0 pour k > 2.
Exercice no 26
Soit f une application de N dans lui-mme, injective. Montrons que

lim f(n) = +.

n+

Soient A un rel puis m = Max(0, 1 + E(A)).


 m est dans tous les cas un entier naturel strictement suprieur A. Puisque
f est injective, on a card f1 ({0, 1, ..., m} 6 m + 1. En particulier, f1 ({0, 1, ..., m}) est une partie finie (ventuellement
vide) de N.

0 si f1 ({0, 1, ..., m}) =
Posons n0 = 1+
. Par dfinition de n0 , si n > n0 , n nest pas lment de f1 ({0, 1, ..., m})
Max f1 ({0, 1, ..., m}) sinon
et donc f(n) > m > A.
On a montr que A R, n0 N/ (n N), (n > n0 f(n) > A) ou encore

lim f(n) = +.

n+

Exercice no 27

Pour n naturel non nul et x rel positif, posons fn (x) = xn + x 1.


1
1
Pour x > 0, f1 (x) = 0 x = et donc u1 = .
2
2

Pour n > 2, fn est drivable sur R+ et pour x > 0, fn (x) = nxn1 + 1 > 0.

http ://www.maths-france.fr

14

c Jean-Louis Rouget, 2014. Tous droits rservs.


fn est ainsi continue et strictement croissante sur R+ et donc bijective de R+ sur





+
fn R = f(0), lim fn (x) = [1, +[.
x+

En particulier,

!x [0, +[/ fn (x) = 0.


Soit un ce nombre. Puisque fn (0) = 1 < 0 et que fn (1) = 1 > 0, par stricte croissance de fn sur [0, +[, on a :
n N, 0 < un < 1.
La suite u est donc borne.
Ensuite, pour n entier naturel donn et puisque 0 < un < 1 :
n+1
fn+1 (un ) = un
+ un 1 < un
n + un 1 = fn (un ) = 0 = fn+1 (un+1 ),

et donc fn+1 (un ) < fn+1 (un+1 ) puis, par stricte croissance de fn+1 sur R+ , on obtient :
n N, un < un+1 .
La suite u est croissante et majore par 1. Donc, la suite u converge vers un rel . De plus, lencadrement 0 < un < 1
fournit par passage la limite : [0, 1].

1+
1
=
. Mais alors, pour n > n0 , on a
Si 0 6 < 1, il existe un rang n0 tel que pour n > n0 , on a : un 6 +
2
2


n
1+
et quand n tend vers vers +, on obtient 1 6 0 ce qui est en contradiction avec 0 6 < 1.
1 un = un
n 6
2
Donc
lim un = 1.

n+

Exercice no 28
2p
1) Posons a =
o p Z, q N et PGCD(p, q) = 1. Pour tout entier naturel n, on a
q




2p
2p
un+q = cos (n + q)
= cos n
+ 2p = cos(na) = un .
q
q
La suite u est donc q-priodique et de mme la suite v est q-priodique. Maintenant, une suite priodique converge si et
seulement si elle est constante. En effet, soient T une priode strictement positive de u et la limite de u. Soit k J0, T 1K.
Pour tout entier naturel n, on a |uk u0 | = |uk+nT unT |. En faisant tendre n vers +, puisque |uk+nT unT | || = 0,
on obtient uk = u0 .
Par priodicit, on a alors n N, un = u0 et donc u est constante.
p
2p
o p Z, q N , PGCD(p, q) = 1 et

/ Z, alors u1 6= u0 et la suite u nest pas constante et donc


q
q
diverge, et si a 2Z, la suite u est constante et donc converge.

Or, si a =

2) Pour tout entier naturel n,


vn+1 = sin((n + 1)a) = sin(na) cos a + cos(na) sin a = un sin a + vn cos a.
a
vn+1 vn cos a

/ Z, sin a 6= 0 et donc un =
. Par suite, si v converge alors u converge. De mme, partir de
2
sin a
cos((n + 1)a) = cos(na) cos a sin(na) sin a, on voit que si u converge alors v converge (car cos a 6= 0). Les suites u et v
sont donc simultanment convergentes ou divergentes.

Puisque

Supposons que la suite u converge, alors la suite v converge. Soient et les limites respectives de u et v. Daprs ce qui
prcde, et sont solutions du systme :


sin a + cos a =
sin a + (cos a 1) = 0

cos a sin a = .
(cos a 1) sin a = 0.

Le dterminant de ce systme vaut sin2 a (cos a 1)2 < 0 car a


/ 2Z. Ce systme admet donc lunique solution
2
= = 0 ce qui contredit lgalit 2 + = 1. Donc, les suites u et v divergent.
http ://www.maths-france.fr

15

c Jean-Louis Rouget, 2014. Tous droits rservs.


Exercice no 29
Pour ]0, [, posons f() = supnN (| sin(n)|). {| sin(n)|, n N} est une partie non vide et majore (par 1) de R.
Donc, pour tout rel de ]0, [, f() existe dans R.


2
Si est dans
,
,
3 3


3
=f
.
f() = supnN (| sin(n)|) > sin >
2
3
i i

Si est dans 0, . Soit n0 lentier naturel tel que (n0 1) < 6 n0 (n0 existe car la suite (n)nN est strictement
3
3
croissante). Alors,

6 n0 = (n0 1) + < + 6 + =
.
3
3
3
3
3
Mais alors,
f() = supnN (| sin(n)|) > | sin(n0 )| >
Si est dans


2
, , on note que
3


3
.
=f
2
3

f() = supnN (| sin(n)|) = supnN (| sin(n( )|) = f( ) > f


car est dans ]0,

].
3

On a montr que ]0, [, f() > f


3


3

3
=
. Donc, inf]0,[ (supnN (| sin(n)|)) existe dans R et
2

inf]0,[ (supnN (| sin(n)|)) = Min]0,[ (supnN (| sin(n)|)) = f


3

3
.
2

Exercice no 30
La suite u nest pas majore. Donc, M R, n N/ un > M. En particulier, n0 N/ un0 > 0.
Soit k > 0. Supposons avoir construit des entiers n0 , n1 ,..., nk tels que n0 < n1 < ... < nk et i J0, kK, uni > i.
On ne peut avoir : n > nk , un < k + 1 car sinon la suite u est majore par le nombre Max{u0 , u1 , ..., unk , k + 1}. Par
suite, nk+1 > nk / unk+1 > k + 1.
On vient de construire par rcurrence une suite (unk )kN extraite de la suite u telle que k N, unk > k et en particulier
telle que lim unk = +.
k+

Exercice no 31
1
Si u converge vers un rel , alors [0, 1] puis, par passage la limite quand n tend vers +, (1 ) > , et donc
4
2

1
1
1
6 0 et finalement = . Par suite, si u converge, lim un = .

n+
2
2
2
De plus, puisque la suite u est valeurs dans ]0, 1[, pour n naturel donn, on a :
un (1 un ) =

1
un
2

2

et puisque 1 un > 0, on a donc n N, un < un+1 .


u est croissante et majore par 1. Donc u converge et

http ://www.maths-france.fr

lim un =

n+

16

1
< un+1 (1 un ),
4
1
(amusant).
2

c Jean-Louis Rouget, 2014. Tous droits rservs.


Planche no 19. Comparaison des suites en linfini


* trs facile ** facile *** difficult moyenne **** difficile
I : Incontournable T : pour travailler et mmoriser le cours
Exercice no 1 (***I)
Dterminer un quivalent le plus simple possible de chacune des suites suivantes quand n tend vers +.


n1
1) Arccos
n
n
6) 1 + n

 
1
2) Arccos
n
 


1
1
ln sin
7) ln cos
n
n


3) ch n

8)

 3/5
2

n
1
4) 1 +
n
s
(1)n
9) 1 +
1
n


(Arctan n)3/5

ln(n + n2 + 1
5)
n4 + n2 1

Exercice no 2 (**I)
Montrer que

n
X

k! n!.
+

k=0

Exercice no 3 (***I)
1) Soient u et v deux suites relles strictement positives. Pour n N, on pose Un =
si un vn et si
+

n
X

uk et Vn =

k=0

n
X

vk . Montrer que

k=0

lim Vn = +, alors Un Vn .

n+

2) Application. Trouver un quivalent de

n
X

n
X
1
et
ln(k).
k
k=1
k=1

Exercice no 4 (****)
Soit (un ) une suite relle de limite nulle. Montrer que si un + u2n

A-t-on : si un + un+1

2
1
, alors un
?
+ n
n

1
3
, alors un
.
+ n
2n

Exercice n 5 (***I)
o

et, n N, un+1 = sin(un ).


2
1) Montrer que la suite u est strictement positive, dcroissante de limite nulle.
Soit u la suite dfinie par u0 =

2) On admet que si u est une suite de limite nulle, alors, quand n tend vers +, sin(un ) = un
+


u3n
+ o u3n .
6

Dterminer un rel tel que la suite vn = u


n+1 un ait une limite relle non nulle.
En appliquant le lemme de Csaro la suite (vn ), dterminer un quivalent simple de un quand n tend vers +.

Exercice no 6 (**I)
On admet que

n
X
1
= ln n + + o(1) o est la constante dEuler.
k +

k=1

Pour n N , on pose un =

n
X
k=1

http ://www.maths-france.fr

1
. Dterminer
n+k

lim un .

n+

c Jean-Louis Rouget, 2014. Tous droits rservs.


Planche no 19. Comparaison des suites en linfini : corrig


Exercice no 1
n1
n1
existe et est lment de [1, 1]. Donc, Arccos
existe pour tout entier naturel
1) Tout dabord, pour n > 1,
n
n
non nul n.
n1
n1
Quand n tend vers +,
tend vers 1 et donc Arccos
tend vers 0. Mais alors,
n
n
s



2

n1
n1
n1
2n 1
2
= 1
=
sin Arccos
.
Arccos
+
+
n
n
n
n
n

1
tend vers 6= 0 et donc Arccos

.
n
2
n + 2

 1  n
1
3) ch n =
e + e n e n .
+ 2
2


n
n


1
1
1
1
n ln(1+1/n)
4) n ln 1 +
n = 1 et donc, 1 +
=e
tend vers e. Par suite, 1 +
e.
+
n +
n
n
n



5) Pour tout entier naturel n, n2 + 1 > 0 et donc n2 + 1 existe puis n + n2 + 1 > 0 et donc ln n + n2 + 1 existe.



ln n + n2 + 1
existe pour n > 1.
Ensuite, pour n > 1, n4 + n2 1 > n4 > 0,
n4 + n2 1


p
ln n + n2 + 1 ln(n + n) = ln(2n) = ln n + ln 2 ln n.
2) Arccos

argch n
ln n
ln n
Donc,
= 2 .

n
n4 + n2 1 + n4
6)





1
n ln n + 1 = n ln n n ln 1 +
n





1
1
= n ln n n + o
+
n
n


= n ln n 1 + o(1),
+

et donc

n
1 1
= e n ln( n)1+o(1) e n ln( n)1 = n .
n)
+
+
e n





  

1
1
1
ln n
1
1
ln sin
cos 1 ln
2 ( ln n) =
.
7) ln cos
n
n +
n
n +
2n
2n2
3/5

 3/5
 

 3/5 
 3/5 
2 1
6
1
1
1

3/5
1
1
, et
=
=
Arctan
+o
+o
8) (Arctan n)
=
+
+
2
n
2
n
n
2
5n
n
donc
   
 3/5
 3/5 
6
3/5 6
1
3/5
11+

(Arctan n)
=
+o
2
2
5n
n
2
5n

(1 +

s


(1)n
1
(1)n
(1)n


> 0, puis 1 +
1 existe. Ensuite,
9) Tout dabord, pour n > 1, = 6 1, et donc 1 +
n
n
n
n
s
(1)n
(1)n
.
1+
1
+ 2 n
n

http ://www.maths-france.fr

c Jean-Louis Rouget, 2014. Tous droits rservs.


Exercice no 2
Pour n > 2, on a
n2
n
1 X k!
1 X
.
k! = 1 + +
n!
n
n!
k=0

k=0

Pour 0 6 k 6 n 2,

k!
1
1
=
6
(le produit contenant au moins les deux premiers facteurs). Par
n!
n(n 1)...(k + 1)
n(n 1)

suite,
06

n2
X
k=0

On en dduit que

n2
X
k=0

k!
n2
6
.
n!
n(n 1)

1
k!
tend vers 0 quand n tend vers +. Comme
tend aussi vers 0 quand n tend vers +, on en
n!
n

n
1 X
dduit que
k! tend vers 1 et donc que
n!
k=0

n
X
k=0

k! n!.
+

Exercice no 3
1) Soit > 0. Les suites u et v sont quivalentes et la suite v est strictement positive. Donc, il existe un rang n0 tel que,

pour n > n0 , |un vn | < vn . Soit n > n0 .


2
!


n0
n
n
X
X
X
|Un Vn |
Un
1
1



|uk vk | 6
6
|uk vk | +
vk
Vn 1 =
Vn
Vn
Vn
2
k=0
k=0
k=n0 +1
!
n0
n0
X
1 X
1

6
|uk vk | +
|uk vk | + Vn =
Vn
2
Vn
2
k=0

k=0

Maintenant, lexpression
+. On en dduit que

n0
X

|uk vk | est constante quand n varie et dautre part, Vn tend vers + quand n tend vers

k=0
n0
X

1
Vn

|uk vk | tend vers 0 quand n tend vers +. Par suite, il existe un rang n1 > n0 tel que,

k=0

n0
1 X

|uk vk | < .
Vn
2
k=0



Un
1 < + = . On a montr que
Pour n > n1 , on a alors
Vn
2 2





Un
1 < .
> 0, n1 N/ n N, n > n1
Vn


Un
Ainsi, la suite
tend vers 1 quand n tend vers + et donc Un Vn .
+
Vn

pour n > n1 ,

2) Equivalent de

n
X
1
.
k
k=1

2
De plus,



1
2
2
n+1 n =
= .
n
n + 1 + n + 2 n
n

X


k + 1 k = 2 n + 1 2 1.
2

k=1

Cette dernire expression tend vers + quand n tend vers +.

http ://www.maths-france.fr

c Jean-Louis Rouget, 2014. Tous droits rservs.



1
1
En rsum, pour n > 1, > 0, 2( n + 1 n) > 0, de plus quand n tend vers +, 2 n + 1 n et enfin,
n
n +
n

X

k + 1 k tend vers + quand n tend vers +. Daprs 1),
2

k=1

n
n

X
X


1

2
k + 1 k = 2 n + 1 2 1 2 n.
+
k k=1
k=1

Equivalent de

n
X

ln(k).

k=1

1
(n + 1) ln(n + 1) n ln n = (n + 1 n) ln n + (n + 1) ln 1 +
n
Comme

n
X

= ln n + 1 + o(1) ln n.
+

((k + 1) ln(k + 1) k ln k) = (n + 1) ln(n + 1) tend vers + et que les suites considres sont positives et

k=1

quivalentes, on en dduit que


ln(n!) =

n
X
k=1

ln k

n
X

((k + 1) ln(k + 1) k ln k) = (n + 1) ln(n + 1) n ln n.


+

k=1

Exercice no 4
Pour n > 1, posons un =

1
(1)n
+ . On a alors
ln n
n





2
n
(1)n n(ln(n + 1) ln n)
1
1
n
n un + un+1
=1+
=
2 + n(1)

+ o(1)
n
n+1
ln n ln(n + 1) +
ln n ln(n + 1)
(1)n n ln(1 + 1/n)
(1)n (1 + o(1))
=
+ o(1) =
+ o(1) = o(1).
+
+ ln n ln(n + 1)
+
ln n ln(n + 1)


2
2
1
2
= o(1), ou encore un + un+1 =
Donc, n un + un+1
+ o( ), ou enfin, un + un+1 . Pourtant, un est
n +
n
n
n
(1)n
1
n
quivalent
et pas du tout
(|nun | =
+).
ln n
n
ln n
1
3
et montrons que un
.
Supposons maintenant que un + u2n
+
+ 2n
n
 
 
1
1
1
On pose vn = un . Il sagit maintenant de montrer que vn = o
sous lhypothse vn + v2n = o
.
+
+
n
n
n

Soit > 0. Il existe n0 N tel que, pour n > n0 , n |vn + v2n | < . Soient n > n0 et p N.
4
|vn | = |vn + v2n v2n v4n + ... + (1)p (v2p n + v2p+1 n ) + (1)p+1 v2p+1 n |
1
!
p
p
X
X 1
1 2p+1
6
|v2k n + v2k+1 n | + |v2p+1 n | <
+ |v2p+1 n |
+ |v2p+1 n | =
1
4
2k n
4n
k=0
k=0
1
2

+ |v2p+1 n |
6
2n
Maintenant, la suite u tend vers 0, et il en est de mme de la suite v. Par suite, pour chaque n > n0 , il est possible de

.
choisir p tel que |v2p+1 n | <
2n

En rsum, si n est un entier donn suprieur ou gal n0 , n|vn | < + = . On a montr que
2 2
> 0, n0 N/ n N, (n > n0 |nvn | < ).
 
 
1
1
1
1
et donc un =
, ou encore un
+o
.
Par suite, vn = o
+
+
+
n
n
n
n
Exercice no 5
i i
1) Il est immdiat par rcurrence que la suite u est dfinie et valeurs dans 0, .
2
http ://www.maths-france.fr

c Jean-Louis Rouget, 2014. Tous droits rservs.


i i
i i
On sait que pour tout rel x 0, , on a sin x < x. Soit n N. Puisque un 0, , on a un+1 = sin(un ) < un . La
2
2
suite u est donc strictement dcroissante. Puisque la suite u est dautre part minore par 0, la suite u converge vers un

rel not . Puisque pour tout n N, 0 < un 6 , on a 0 6 6 . Mais alors, par continuit de la fonction x 7 sin x sur
2
2
h i
0,
et en particulier en , on a
2


= lim un+1 = lim sin(un ) = sin
lim un = sin().
n+

n+

n+

i i
Or, si x 0, , sin x < x et en particulier sin x 6= x. Donc, = 0.
2
La suite u est strictement positive, strictement dcroissante, de limite nulle.
2) Soit R. Puisque un tend vers 0 quand n tend vers +,










u2n
u2n
u3

2
2
= u
=
u
1

+
o
u
+
o
u
un n + o u3n
n
n
+
+ n
+ n
6
6
6

u
=
n+1 = (sin(un ))

= u
n

u2+
n
+ o(u2+
).
n
6

et donc, u
n+1 un =
+

u2+
n
+ o(u2+
). En prenant = 2, on obtient alors
n
6
1
1
1
vn = 2
+ o(1).
2 =
un + 3
un+1

Daprs le lemme de Csaro,

n1
1 X
1
vk tend galement vers . Mais,
n
3
k=0



n1
n1 
1 X
1
1
1
1
1 X
1

=
.

vk =
n
n
n u2n u20
u2k+1 u2k
k=0
k=0



1
1
1
1
1
n
n
3
1
1
n

+ o(1) puis, 2 =
+ 2 + o(n) =
+ o(n). Donc, 2
, puis u2n
et enfin,
=
+ n
+ 3
n u2n u20 + 3
un + 3
un + 3
u0
puisque la suite u est strictement positive,

Ainsi,

un

3
.
n

Exercice no 6
Soit n > 1.
un =

n
X
k=1

2n
2n
n
X
X
1
1
1 X1
=
=

.
n+k
k
k
k
k=n+1

k=1

k=1

Mais alors
un = (ln(2n) + ) (ln(n) + ) + o(1) (ln(2n) + ) ln 2 + o(1),
+

et donc
lim

n+

http ://www.maths-france.fr

n
X
k=1

1
= ln 2.
n+k

c Jean-Louis Rouget, 2014. Tous droits rservs.


Planche no 20. Limites. Continuit en un point


* trs facile ** facile *** difficult moyenne **** difficile
I : Incontournable T : pour travailler et mmoriser le cours
Exercice no 1 (**T)
Montrer en revenant la dfinition que f(x) =

3x 1
est continue en tout point de R \ {5}.
x5

Exercice no 2 (*I)
Soient f et g deux fonctions continues en x0 R. Montrer que les fonction Min{f, g} et Max{f, g} sont continues en x0 .
Exercice no 3 (**IT)
Montrer que la fonction caractristique de Q est discontinue en chacun de ses points.
Exercice no 4 (**IT)
1) Montrer que la fonction x 7 sin x na pas de limite en +.
 
1
2) Montrer que la fonction x 7 sin
na pas de limite en 0.
x
 
1
3) Montrer que la fonction x 7 x sin
est prolongeante par continuit en 0.
x
Exercice no 5 (**T)

a ex1 1

si x > 1

x1
o (a, b) R2 .
Pour x 6= 1, on pose f(x) =

6x

si x > 1
x1

f est-elle prolongeable par continuit en 1 (discuter en fonction de a et b) ?


Exercice no 6 (**)
Trouver f bijective de [0, 1] sur lui-mme et discontinue en chacun de ses points.
Exercice no 7 (**IT)
Etudier en chaque point de R lexistence dune limite droite, gauche, la continuit de la fonction f dfinie par

 
1

xE
si x 6= 0
f(x) =
.
x

1 si x = 0.
Exercice no 8 (*)
Soit f une fonction dfinie sur R valeurs dans R et priodique, admettant une limite relle quand x tend vers +.
Montrer que f est constante.
Exercice no 9 (****)
x0

f(2x) f(x)
f(x)
= 0. Montrer que lim
= 0.
x0
x0 x
x

c Jean-Louis Rouget, 2014. Tous droits rservs.


Soit f une fonction dfinie sur un voisinage de 0 telle que lim f(x) = 0 et lim
f(2x) f(x)
.)
(Indication. Considrer g(x) =
x

http ://www.maths-france.fr

Planche no 20. Limites. Continuit en un point : corrig


Exercice no 1
Soit x0 R \ {5}. Pour x 6= 5,


3x 1 3x0 1
14|x x0 |
=

.
|f(x) f (x0 ))| =

x5
x0 5
|x 5| |x0 5|


|x0 5|
|x0 5|
|x0 5|
|x0 5|
ou encore si |x x0 | <
, x0 +
, on a |x 5| >
(faire un dessin).
Ensuite, pour x x0
2
2
2
2
Ainsi,


|x0 5|
28
|x0 5|
x x0
, |f(x) f(x0 )| 6
|x x0 |.
, x0 +
2
2
(x0 5)2


|x0 5| (x0 5)2
(> 0). On a
,
Soit > 0. Soit = Min
2
28 2
|x x0 | < |f(x) f(x0 )| 6

28
|x0 5|
|x x0 | (car |x x0 | <
)
(x0 5)2
2

(x0 5)2
(x0 5)2
28

(car |x x0 | <
)
2
(x0 5)
28 2
28 2

= < .
2
<

On a mont que > 0, > 0/ (x R \ {5}), (|x x0 | < |f(x) f(x0 )| < ). f est donc continue en chaque point
de R \ {5}.

Exercice no 2
1
1
Min(f, g) = (f + g |f g|) et Max(f, g) = (f g + |f g|) sont continues en x0 en vertu de thormes gnraux.
2
2
Exercice no 3
Notons Q la fonction caractristique de Q. Soit x0 un rel. On note que

1
2

/ Q.
x0 Q n N , x0 + Q n N , x0 +
n
n
!
!




1
1
2
2
existe, lim Q x0 +
existe et lim x0 +
6= lim x0 +
(lune des
Donc, lim Q x0 +
n+
n+
n+
n+
n
n
n
n

1
2
deux limites valant 1 et lautre valant 0) bien que lim x0 + = lim x0 +
= x0 . Ainsi, pour tout rel x0 R, la
n+
n n+
n
fonction caractristique de Q na pas de limite en x0 et est donc discontinue en x0 .

Exercice no 4

+ 2n. On a lim un = + = lim vn mais lim sin (un ) = 0 et


n+
n+
n+
2
lim sin (vn ) = 1. Puisque 1 6= 0, on en dduit que la fonction x 7 sin x na pas de limite quand x tend vers +.

1) Pour n N, posons un = 2n et vn =
n+

2) Pour n N , posons un =

1
1
et vn =
. On a
2n
+ 2n
2

lim un = 0 =

n+

lim vn mais

n+

lim sin (un ) = 0 et

n+

 
1
na pas de limite quand x tend vers 0.
lim sin (vn ) = 1. Puisque 1 6= 0, on en dduit que la fonction x 7 sin
n+
x

 
 


1
1

3) Pour tout rel non nul x, x sin
= |x| sin
6 |x|. Puisque |x| tend vers 0 quand x tend vers 0, on en dduit

x
x
 
1
tend vers 0 quand x tend vers 0.
que x sin
x
 
 
1
1
Ainsi, la fonction x 7 x sin
a une limite relle quand x tend vers 0 ou encore la fonction x 7 x sin
est
x
x
prolongeante par continuit en 0.

http ://www.maths-france.fr

c Jean-Louis Rouget, 2014. Tous droits rservs.


Exercice no 5
Si x > 1, 6x 5 > 0 et donc f(x) existe. Dautre part si x < 1, f(x) existe et finalement f est dfinie sur ] , 1[]1, +[.

Quand x tend vers 1 par valeurs infrieures, x 1 tend vers 0 et 6x 5 b tend vers 1 b. Donc, si b 6= 1, la fonction
na pas de limite relle en 1 droite. Si b = 1, pour x > 1,

6x 5 1
6x 5 1
6

=
=
f(x) =
,
x1
(x 1) 6x 5 + 1
6x 5 + 1
et donc lim f(x) =
x1
x<1

6
= 3. Finalement, f a une limite gauche en 1 si et seulement si b = 1. Dornavant, b est gal 1.
2

eX 1
ex1 1
= lim
= 1 et donc lim f(x) = a.
X0
x1
x1
x1
X

lim

x>1

X>0

x>1

Donc, si a 6= 3, lim f(x) 6= lim f(x) et f na pas de limite quand x tend vers 1. Si par contre a = 3, f a une limite relle
x1
x>1

x1
x<1

quand x tend vers 1 ou encore f est prolongeante par continuit en 1.


En rsum, f est prolongeante par continuit en 1 si et seulement si a = 3 et b = 1. Le prolongement encore not f est la
fonction



3 ex1 1

3 ex1 1
si x > 1

x1
si x > 1

x1
3
si
x
=
1
f : x 7
=
.

si x > 1

6x 5 1

6x 5 + 1
si x > 1
x1

Exercice no 6

x
si
x

(Q

[0,
1])
\
0,

2
.
Pour x [0, 1], posons f(x) =
1

x
si
x

(R
\
Q)

[0,
1]

1
1

0 si x = et si x = 0
2
2
1
f est bien une application dfinie sur [0, 1] valeurs dans [0, 1]. De plus, si x (Q [0, 1]) \ {0, }, alors f(f(x)) = f(x) = x.
2
Si x (R \ Q) [0,
x (R
\ Q) [0, 1] et donc f(f(x)) = f(1 x) = 1 (1 x) = x.
1],alors 1 
1
1
1
= 0 et f f
= f(0) = .
Enfin, f(f(0)) = f
2
2
2
Finalement, f f = Id[0,1] et f, tant une involution de [0, 1], fest une permutation de [0, 1].
Soit a un rel de [0, 1]. On note que

lim

xa, x(R\Q)

f(x) = 1 a et

lim

xa, xQ

f(x) = a. Donc, si f a une limite en a,

1
1
. Ceci montre dj que f est discontinue en tout rel de [0, 1] distinct de . Dautre
2
2
 
1
1
1
part, si a = ,
et donc f est discontinue en tout point de [0, 1].
lim
f(x) = a = 6= 0 = f
2 xa, xQ, x6=a
2
2

ncessairement 1 a = a et donc a =

Exercice no 7

Donnons tout dabord une expression plus explicite de f(x) pour chaque rel x.
1
Si x > 1, alors ]0, 1[ et donc, f(x) = 0.
x

1
1
Si p N / x
,
, f(x) = px.
p+1 p
 
1
f(0) = 1 et plus gnralement, p Z , f
= 1.
p
1
Si x 6 1, alors [1, 0[ et donc, f(x) = x.
x


1
1
1
1
\
, alors
,
< x 6 (< 0) fournit, par dcroissance de la fonction
Enfin, si p Z {1} tel que x
p+1 p
p+1
p
1
1
x 7 sur ] , 0[, p 6 < p + 1(< 0) et donc f(x) = px.
x
x
http ://www.maths-france.fr

c Jean-Louis Rouget, 2014. Tous droits rservs.


1
Etude en 0. x R , 1 < E
x

 
1
1
6 et donc 1 x < f(x) 6 1 si x > 0 et 1 6 f(x) < 1 x si x < 0. Par suite,
x
x
x R, |f(x) 1 6 |x|,

et lim f(x) = 1. f est donc continue en 0.


x0


1
1
pour p lment de Z\{1, 0} et donc est continue sur ces intervalles
f est affine sur chaque intervalle de la forme
,
p+1 p
1
et en particulier continue gauche en chaque . f est affine sur ] , 1] et aussi sur ]1, +[ et est donc continue sur
p
1
ces intervalles. Il reste donc analyser la continuit droite en , pour p entier relatif non nul donn. Mais,
p
 
 +
1
1
1
.
=
lim (x(p 1)) = 1 6= 1 = f
f
1
1
p
p
p
x p , x> p


f est donc discontinue droite en tout

1
o p est un entier relatif non nul donn.
p

Graphe de f :
b

Exercice no 8
Soit T une priode strictement positive de f. On note la limite de f en +.
Soit x0 un rel. n N, f (x0 ) = f (x0 + nT ) et quand n tend vers +, puisque T > 0, on obtient :
f(x) = lim f(x + nT ) = lim f(X) = .
n+

X+

Ainsi, x R, f(x) = et donc f est constante sur R.


Exercice no 9
f(2x) f(x)
. f est dfinie sur un voisinage de 0 et donc il existe a > 0 tel que ] a, a[ Df .
Pour x 6= 0, posons g(x) =
x
i a ah
Mais alors, ,
\ {0} Dg .
2 2
i a ah
Soit x ,
\ {0} et n N .
2 2
f(x) =

n1
X
k=0

n1
X
k=0

http ://www.maths-france.fr

 x 
 x 
x

f
+
f
(somme tlescopique)
2k
2k+1
2n

x
2k+1

 x 
 x 
+f n .
k+1
2
2
3

c Jean-Louis Rouget, 2014. Tous droits rservs.


i a ah
Par suite, pour x ,
\ {0} et n N , on a :
2 2




X 1  x  f (x/2n )
f(x) n1

6
.

g k+1 +
x

2k+1
2
x
k=0

Soit > 0. Puisque par hypothse, g tend vers 0 quand x tend vers 0,
i ah

/ X ] , [, |g(X)| < .
0,
2
2
x
Or, pour x ] , [\{0} et pour k dans N , k est dans ] , [\{0} et par suite,
2
1


n1
X 1  x  n1
X 1

1 1 2n
1

=
< ,
=

6
g

k+1
k+1
k+1
n
1
2
2
2
2
2 2
2
2
2
k=0
k=0
1
2




f(x) f (x/2n )
6 +
. On a ainsi montr que
et donc,

x 2
x




f(x) f(x/2n )
6 +
.
x ] , [\{0}, n N ,

x 2
x


f (x/2n )
f (x/2n )

< , ce que lon
tend vers 0 quand n tend vers +. Donc, on peut choisir n tel que
Mais, x fix,
2
x
x
f(x)
< + = . On a montr que
fait. On a alors
x 2 2




f(x)


< ,
> 0, > 0/ x Df , 0 < |x| <
x
ce qui montre que (f est drivable en 0 et que) lim

x0

http ://www.maths-france.fr

f(x)
= 0.
x

c Jean-Louis Rouget, 2014. Tous droits rservs.


Planche no 21. Continuit


* trs facile ** facile *** difficult moyenne **** difficile
I : Incontournable T : pour travailler et mmoriser le cours
Exercice no 1 (**I)
Soit A une partie non vide de R. Pour x rel, on pose f(x) = d(x, A) = Inf{|yx|, y A}. Montrer que f est Lipschitzienne.
Exercice no 2 (**I)
Soit f continue sur [a, b] valeurs dans [a, b]. Montrer que f a un point fixe.
Exercice no 3 (**I)
Soit f dfinie sur [0, +[ valeurs dans [0, +[, continue sur [0, +[ telle que
tend vers +. Montrer que f a un point fixe.

f(x)
a une limite relle [0, 1[ quand x
x

Exercice no 4 (****)
Soit f croissante de [a, b] dans lui-mme. Montrer que f a un point fixe.
Exercice no 5 (****)
Soit f croissante sur [a, b] telle que f([a, b]) = [f(a), f(b)]. Montrer que f est continue sur [a, b].
Exercice no 6 (***)
Soit f continue sur R+ telle que, pour tout rel positif x, on ait f(x2 ) = f(x). Montrer que f est constante sur R+ . Trouver
un exemple o f nest pas constante.
Exercice no 7 (**IT)
Pour tout rel x de [0, +[ on pose f(x) =

x.

1) Montrer directement sans utiliser le thorme de Heine que la fonction f est uniformment continue sur [0, 1].
2) Montrer que la fonction f est uniformment continue sur [1, +[.
3) Montrer que la fonction f est uniformment continue sur [0, +[.
Exercice no 8 (**IT)

Montrer que la fonction f : x 7 sin x2 nest pas uniformment continue sur [0, +[.
Exercice no 9 (***IT)
Soit f continue sur R+ valeurs dans R admettant une limite relle quand x tend vers +. Montrer que f est uniformment
continue sur R+ .
Exercice no 10 (***)
Soit f priodique et continue sur R. Montrer que f est borne et uniformment continue sur R.
Exercice no 11 (***I)
Trouver toutes les applications f de R dans R, continues sur R, vrifiant
(x, y) R2 , f(x + y) = f(x) + f(y).
Exercice no 12 (**)
Soit f de [0, 1] dans lui-mme telle que (x, y) ([0, 1])2 , |f(y) f(x)| > |x y|. Montrer que f = Id ou f = 1 Id.
Exercice no 13 (****)
Trouver les fonctions bijectives de [0, 1] sur lui-mme vrifiant x [0, 1], f(2x f(x)) = x.
Exercice no 14 (***I)
Soit f une application de [0, 1] dans R, continue sur [0, 1] et vrifiant f(0) = f(1).
1
. Montrer que lquation f(x + a) = f(x) admet au moins une solution.
n
2) Montrer (en fournissant une fonction prcise) que, si a est un rel de ]0, 1[ qui nest pas de la forme prcdente, il est
possible que lquation f(x + a) = f(x) nait pas de solution.

1) Soit n un entier naturel non nul et soit a =

http ://www.maths-france.fr

c Jean-Louis Rouget, 2014. Tous droits rservs.


3) Application. Un cycliste parcourt 20 km en une heure.


a) Montrer quil existe au moins un intervalle de temps de dure une demi-heure pendant lequel il a parcouru 10 km.
b) Montrer quil existe au moins un intervalle de temps de dure 3 min pendant lequel il a parcouru 1 km.
c) Montrer quil nexiste pas ncessairement un intervalle de temps de dure 45 min pendant lequel il a parcouru
15 km.

http ://www.maths-france.fr

c Jean-Louis Rouget, 2014. Tous droits rservs.


Planche no 21. Continuit : corrig


Exercice no 1
Soit (x, y) R2 et z A. |x z| 6 |x y| + |y z| ou encore |y z| > |x z| |x y|. Comme d(x, A) est un minorant de
{|x z|, z A}, on en dduit que |y z| > d(x, A) |x y|.
Ainsi, z A, |y z| > d(x, A) |x y| et donc d(x, A) |x y| est un minorant de {|y z|, z A}. Puisque d(y, A) est
le plus grand de ces minorants, on en dduit que d(x, A) |x y| 6 d(y, A). On a montr que
(x, y) R2 , d(x, A) d(y, A) 6 |y x|.

En appliquant ce rsultat y et x, on a aussi montr que (x, y) R2 , d(y, A) d(x, A) 6 |y x|.


Finalement, (x, y) R2 , kf(y) f(x)| 6 |y x|. Ainsi, f est donc 1-Lipschitzienne et en particulier continue sur R.
Exercice no 2
Pour x [a, b], posons g(x) = f(x) x. La fonction g est continue sur [a, b] puisque f lest. De plus, g(a) = f(a) a > 0
et g(b) = f(b) b 6 0. Daprs le thorme des valeurs intermdiaires, g sannule au moins une fois sur [a, b] ou encore,
lquation f(x) = x admet au moins une solution dans [a, b].
Exercice no 3
f(x)
+1
f(x)
tend vers [0, 1[, il existe A > 0 tel que pour x > A,
6
< 1. Ainsi, f(A) 6 A et f(0) > 0.
Puisque
x
x
2
La fonction g : x 7 f(x) x est donc continue sur [0, A] et change de signe sur [0, A]. Daprs le thorme des valeurs
intermdiaires, lquation g(x) = 0 admet une solution dans [0, A] et donc dans [0, +[ ou encore lquation f(x) = x
admet une solution dans [0, +[.
Exercice no 4
Puisque f est croissante sur [a, b], f admet en tout rel x de ]a, b[ une limite droite et une limite gauche vrifiant
< f(x ) 6 f(x) 6 f(x+ ) < +. De mme, f admet une limite droite en a et une limite gauche en b vrifiant
f(a) 6 f(a+ ) < + et < f(b ) 6 f(b).
Soit E = {x [a, b]/ f(x) > x}. E est une partie non vide de R (car a est dans E) et majore (par b). Donc, E admet une
borne suprieure c vrifiant a 6 c 6 b.
Montrons que f(c) = c.
1
Si c = b, alors n N , xn E/ b < xn 6 b. Puisque f est valeurs dans [a, b] et que les xn sont dans E, pour
n
tout entier naturel non nul n, on a
xn 6 f(xn ) 6 b ().
Quand n tend vers +, la suite (xn ) tend vers b (thorme des gendarmes) et donc, f tant croissante sur [a, b], la
suite (f(xn )) tend vers f(b ) ou vers f(b). Par passage la limite quand n tend vers + dans (), on obtient alors
b 6 f(b ) 6 f(b) 6 b ou directement b 6 f(b) 6 b. Dans tous les cas, f(b) = b. Finalement, dans ce cas, b est un point
fixe de f.
Si c [a, b[, par dfinition de c, pour x dans ]c, b], f(x) < x (car x nest pas dans E) et par passage la limite quand x tend
vers c par valeurs suprieures et daprs les proprits usuelles des fonctions croissantes, on obtient : f(c)(6 f(c+)) 6 c.
1
< xn 6 c. xn tant dans E, on a f(xn ) > xn . Quand n tend vers +, on
Dautre part, n N , xn E/ c
n

obtient : f(c) > f(c ) > c. Finalement, f(c) = c et dans tous les cas, f admet au moins un point fixe.
Exercice no 5
Puisque f est croissante sur [a, b], on sait que f admet en tout point x0 de ]a, b[ une limite gauche et une limite droite
+
relles vrifiant f(x
0 ) 6 f(x0 ) 6 f(x0 ) puis une limite droite en a lment de [f(a), +[ et une limite gauche en b
lment de ], f(b)].
+

Si f est discontinue en un x0 de ]a, b[, alors on a f(x


0 ) < f(x0 ) ou f(x0 ) < f(x0 ). Mais, si par exemple f(x0 ) < f(x0 ) alors,

x [a,





 6 f(x0 ) et x [x0 , b], f(x) > f(x0 ).

 x0 [ (6= ), f(x)

,
f
(x
)
f([a, b]) = ce qui est exclu puisque dautre part f x
Donc f x
0
0 , f (x0 ) 6= et f x0 , f (x0 )
0
[f(a), f(b)] (la dmarche est identique si f(x+
0 ) > f(x0 )). Donc, f est continue sur ]a, b[. Par une dmarche analogue, f est
aussi continue en a ou b et donc sur [a, b].

http ://www.maths-france.fr

c Jean-Louis Rouget, 2014. Tous droits rservs.


Exercice no 6

n
Soit x > 0. Pour tout naturel n, f(x) = f( x) = f(x1/4 ) = ... = f(x1/2 ). Or, x fix,

lim x1/2 = lim e/2 = 1 et,

n+

n+

f tant continue en 1, on a :


n
x > 0, f(x) = lim f x1/2 = f(1).
n+

f est donc constante sur ]0, +[, puis sur [0, +[ par continuit de f en 0.

Pour x > 0, posons f(x) = 0 si x 6= 1 et f(x) = 1 si x = 1. Pour x > 0, on a x2 = 1 x = 1.


f vrifie donc : x > 0, f(x2 ) = f(x), mais f nest pas constante sur R+ .
Exercice no 7
1) Soit > 0. Soient x et y deux rels de [0, 1]. Supposons que x 6 y.

2
x y = x + y 2 xy 6 x + y 2 x2 = y x = |x y| .

En changeant les rles de x et y, lgalit prcdente est encore valable si x > y. On a donc dmontr que pour tous rels

p
2
x y 6 |y x| ou encore x y 6 |x y|.
x et y de [0, 1],
Soit alors = 2 > 0. Soient x et y deux rels de [0, 1] tels que |x y| < . On a


p
y x 6 |y x| < 2 = .

On a montr que




> 0, > 0/ (x, y) [0, 1]2 , |x y| < y x < ,

et donc que la fonction x 7 x est uniformment continue sur [0, 1].

Remarque. Puisque la fonctionx 7 x est continue sur le segment [0, 1], le thorme de Heine permet daffirmer
directement que la fonction x 7 x est uniformment continue sur le segment [0, 1].
2) Soient x et y deux rels de [1, +[.

Donc la fonction x 7

sur [1, +[.



|y x|
x|
1
y x = |y
= |y x|.
6
2
x+ y
1+ 1

1
x est -Lipschitzienne sur [1, +[ et en particulier la fonction x 7 x est uniformment continue
2

3) Soit > 0.


.
Puisque f est uniformment continue sur [0, 1], 1 > 0/ (x, y) [0, 1]2 , |x y| < 1 |f(x) f(y)| <
2


Puisque f est uniformment continue sur [1, +[, 2 > 0/ (x, y) [1, +[2 , |x y| < 2 |f(x) f(y)| <
.
2

Soit = Min {1 , 2 } > 0. Soit (x, y) [0, +[2 tel que |x y| < .
Si x et y sont dans [0, 1],

|x y| < |x y| < 1 (car 6 1 )

|f(x) f(y)| <


2

|f(x) f(y)| < (car 6 ).


2
Si x et y sont dans [1, +[,
|x y| < |x y| < 2 (car 6 2 )

|f(x) f(y)| <


2

|f(x) f(y)| < (car 6 ).


2
Si par exemple, 0 6 x 6 1 6 y, alors |x 1| 6 |x y| < 6 1 et donc |f(x) f(1)| <
http ://www.maths-france.fr

et |y 1| 6 |x y| < 6 2
2

c Jean-Louis Rouget, 2014. Tous droits rservs.


et donc |f(y) f(1)| <

. Mais alors
2
|f(x) f(y)| 6 |f(x) f(1)| + |f(1) f(y)| <


+ = .
2 2

On a montr que > 0, > 0/ (x, y) [0, +[2 , (|x y| < |f(x) f(y)| < ) et donc que la fonction x 7
est uniformment continue sur [0, +[.

Exercice no 8

2n et yn =

+ 2n.
2
r

/2
et donc
+ 2n 2n = r
Pour tout entier naturel n, yn xn =

+ 2n + 2n
2
Pour tout entier naturel n, f (yn ) f (xn ) = 1 et donc lim (f (yn ) f (xn )) = 1 6= 0.

Pour n N, posons xn =

lim (yn xn ) = 0.

n+

n+

On a trouv deux suites de rels positifs (xn )nN et (yn )nN telles que xn yn tend vers 0 quand
n tend vers + et

2
f (xn ) f (yn ) tend vers 0 quand n tend vers +. On sait alors que la fonction f : x 7 sin x nest pas uniformment
continue sur [0, +[.
Exercice no 9
Posons = lim f(x).
x+

Soit > 0. A > 0/ x R+ ,



x > A |f(x) | <
.
3

2
Soit (x, y) [A, +[2 . Alors, |f(x) f(y)| 6 |f(x) | + | f(y)| <
. Dautre part, f est continue sur le segment [0, A]
3
et donc est uniformment continue sur ce segment daprs le thorme de Heine.

Donc, > 0/ (x, y) [0, A]2 , |x y| < |f(x) f(y)| < .


3
Rsumons. > 0 tant ainsi fourni, soient x et y deux rels de [0, +[ vrifiant |x y| < .

Si (x, y) [0, A]2 , on a |f(x) f(y)| < < .


3
2
< .
Si (x, y) [A, +[2 , on a |f(x) f(y)| <
3

Si enfin on a 0 6 x 6 A 6 y alors, puisque |A x| 6 |x y| < , on a |f(x) f(A)| < et puisque A et y sont dans
3
2
. Mais alors,
[A, +[, on a |f(y) f(A)| <
3
2
|f(x) f(y)| 6 |f(x) f(A)| + |f(y) f(A)| < +
= .
3
3
On a montr que > 0, > 0/ (x, y) [0, +[2 , (|x y| < |f(x) f(y)| < ). f est donc uniformment continue
sur [0, +[.
Exercice no 10
Soit T une priode strictement positive de f. f est continue sur le segment [0, T ] et donc est borne sur ce segment. Soit
M un majorant de |f| sur [0, T ]. M est encore un majorant de |f| sur R par T -priodicit et donc f est borne sur R.
Soit > 0.
f est continue sur le segment [0, T ] . Daprs le thorme de Heine, f est uniformment continue sur ce segment. Donc,


]0, T [/ (x, y) [0, T ]2 , |x y| < |f(x) f(y)| <
.
2
Soient x et y deux rels tels que |x y| < .
Sil existe un entier naturel k tel que (x, y) [kT, (k + 1)T ]2 , alors x kT [0, T ], y kT [0, T ], puis

|(x kT ) (y kT )| = |y x| < et donc |f(x) f(y)| < < .


2
Sinon, en supposant par exemple que x 6 y, puisque lon a choisi < T ,
k Z/ (k 1)T 6 x 6 kT 6 y 6 (k + 1)T.
Mais alors, |x kT | 6 |y x| < et |y kT | 6 |y x| < . Par suite,
|f(x) f(y)| 6 |f(x) f(kT )| + |f(y) f(kT )| <
http ://www.maths-france.fr


+ = .
2 2

c Jean-Louis Rouget, 2014. Tous droits rservs.


Dans tous les cas, si |x y| < , alors |f(x) f(y)| < . On a montr que
> 0, > 0/ (x, y) R2 , (|x y| < |f(x) f(y)| < ).

f est donc uniformment continue sur R.


Exercice no 11

Soit f une application de R dans R vrifiant


(x, y) R2 , f(x + y) = f(x) + f(y).
Puisque f(0) = f(0 + 0) = f(0) + f(0), on a f(0) = 0. Puis, pour x rel donn, f(x) + f(x) = f(x + x) = f(0) = 0 et donc,
pour tout rel x, f(x) = f(x) (f est donc impaire). On a aussi pour n N et x R,
f(nx) = f(x) + ... + f(x) = nf(x).
De ce qui prcde, on dduit :
x R, n Z, f(nx) = nf(x).
Soit a = f(1). Daprs 
ce qui
n Z, f(n) = f(n 1) = nf(1) = 
an.
 prcde,

1
1
1
1
Puis, pour n N , nf
=f n
= f(1) = a et donc n N , f
=a .
n
n
n
  n 
1
1
p
p

= pf
= pa = a . Finalement,
Puis, pour p Z et q N , f
q
q
q
q
r Q, f(r) = ar.
Si on na pas lhypothse de continuit, on ne peut aller plus loin. Supposons de plus que f soit continue sur R.
Soit x un rel. Puisque Q est dense dans R, il existe une suite (rn )nN de rationnels, convergente de limite x.
f tant continue en x, on a :


f(x) = f
lim rn = lim f (rn ) = lim arn = ax.
n+

n+

n+

f est donc une application linaire de R dans R. Rciproquement, les applications linaires conviennent.
Exercice no 12
On a 0 6 f(0) 6 1 et 0 6 f(1) 6 1. Donc |f(1) f(0)| 6 1. Mais, par hypothse, |f(1) f(0)| > 1. Par suite, |f(1) f(0)| = 1
et ncessairement, (f(0), f(1)) {(0, 1), (1, 0)}.
Supposons que f(0) = 0 et f(1) = 1 et montrons que x [0, 1], f(x) = x.
Soit x [0, 1]. On a |f(x) f(0)| > |x 0| ce qui fournit f(x) > x. On a aussi |f(x) f(1)| > |x 1| ce qui fournit
1 f(x) > 1 x et donc f(x) 6 x. Finalement, x [0, 1], f(x) = x et f = Id.
Si f(0) = 1 et f(1) = 0, posons pour x [0, 1], g(x) = 1 f(x). Alors, g(0) = 0, g(1) = 1 puis, pour x [0, 1], g(x) [0, 1].
Enfin,
(x, y) [0, 1]2 , |g(y) g(x)| = |f(y) f(x)| > |y x|.
Daprs ltude du premier cas, g = Id et donc f = 1 Id. Rciproquement, Id et 1 Id sont bien bien solutions du
problme.
Exercice no 13
Id est solution.
Rciproquement, soit f une bijection de [0, 1] sur lui-mme vrifiant x [0, 1], f(2x f(x)) = x. Ncessairement,
x [0, 1], 0 6 2x f(x) 6 1 et donc x [0, 1], 2x 1 6 f(x) 6 2x.
Soit f1 la rciproque de f.
x [0, 1], f(2x f(x)) = x x [0, 1], 2x f(x) = f1 (x)

y [0, 1], f(f(y)) 2f(y) + y = 0 (car x [0, 1], !y [0, 1]/ x = f(y))

Soit y [0, 1] et u0 = y. En posant n N, un+1 = f(un ), on dfinit une suite de rels de [0, 1] (car [0, 1] est
stable par f). La condition y [0, 1], f(f(y)) 2f(y) + y = 0 fournit n N, un+2 2un+1 + un = 0, ou encore
http ://www.maths-france.fr

c Jean-Louis Rouget, 2014. Tous droits rservs.


n N, un+2 un+1 = un+1 un . La suite (un+1 un )nN est constante ou encore u est arithmtique. Mais, u est
galement borne et donc u est constante.
En particulier, u1 = u0 ce qui fournit f(y) = y. On a montr que y [0, 1], f(y) = y et donc f = Id.
Exercice no 14




1
1
, posons g(x) = f x +
f(x).
1) Soit n un entier naturel non nul donn. Pour x lment de 0, 1
n
n


1
. De plus,
g est dfinie et continue sur 0, 1
n
 
n1
X  k + 1
X  k  n1
k
g
=
f
f
= f(1) f(0) = 0.
n
n
n
k=0
k=0
 
k
Maintenant, sil existe un entier k lment de J0, n 1K tel que g
= 0, on a trouv un rel x de [0, 1] tel que
n


1
k
f x+
= f(x) ( savoir x = ).
n
n
 
k
sont non nuls et, tant de somme nulle, il existe deux valeurs de la variable en lesquels g prend des
Sinon, tous les g
n


1
, le thorme des valeurs intermdiaires permet daffirmer
valeurs de signes contraires. Puisque g est continue sur 0, 1
n


1
que g sannule au moins une fois dans cet intervalle ce qui fournit de nouveau une solution lquation f x +
= f(x).
n



x
1


/ N . Pour x [0, 1], posons f(x) = sin
2) Soit a ]0, 1[ tel que
x sin . f est continue sur [0, 1], f(0) = f(1) = 0
a
a
a
mais pour tout rel x,






(x + a)
x




f(x + a) f(x) = sin

=
a

((x
+
a)

x)


6= 0.
sin
sin
sin

a
a
a
a

3) a) et b) Soit g(t) la distance, exprime en kilomtres, parcourue par le cycliste linstant t exprim en heures,
0 6 t 6 1, puis, pour t [0, 1], f(t) = g(t) 20t. f est continue sur [0, 1] (si le cycliste reste un tant soit peu cohrent) et
vrifie f(0) = f(1) = 0.








19
1
1
1
tels que f t1 +
= f (t1 ) et f t2 +
= f (t2 ) ce qui scrit encore
Daprs 1), t1 0, , t2 0,
2
20
2
20




1
1
g t1 +
g (t1 ) = 10 et g t2 +
g(t2 ) = 1.
2
20
1
1
De t1 t1 + , le cycliste a roul 10 km et de t2 t2 + , le cycliste a roul 1 km.
2
20

!






3
4t t 3
4t


c) Posons pour 0 6 t 6 1, f(t) = sin
t.
(de sorte que f(0) = f(1) = 0) et donc, g(t) = sin

+ 20


3
2
3
2
!

 




3
3
3
3
1
f(t) =
20
6= 0 ou encore g t +
g(t) 6= 15.
t 0, , f t +
4
4
4
2
4

http ://www.maths-france.fr

c Jean-Louis Rouget, 2014. Tous droits rservs.


Planche no 22. Drivation


* trs facile ** facile *** difficult moyenne **** difficile
I : Incontournable T : pour travailler et mmoriser le cours
Exercice no 1 (***)
Soit f C1 ([a, b], R) telle que

f(b) f(a)
= sup{f (x), x [a, b]}. Montrer que f est affine.
ba

Exercice no 2 (***) (Formule de Taylor-Lagrange)


Soient a et b deux rels tels que a < b et n un entier naturel. Soit f une fonction lment de Cn ([a, b], R)Dn+1 (]a, b[, R).
Montrer quil existe c ]a, b[ tel que
f(b) =

n
X
f(k) (a)
(b a)n+1 f(n+1) (c)
(b a)k +
.
k!
(n + 1)!

k=0

Indication. Appliquer le thorme de Rolle la fonction g(x) = f(b)

n
X
(b x)n+1
f(k) (x)
(b x)k A
o A est
k!
(n + 1)!

k=0

intelligemment choisi.
Exercice no 3 (***) (Formule des trapzes)
Soit f C2 ([a, b], R) D3 (]a, b[, R). Montrer quil existe c ]a, b[ tel que
ba
(f (a) + f (b)) f(3) (c).
2
xa
Indication. Appliquer le thorme de Rolle g puis g o g(x) = f(x) f(a)
(f (x) + f (a)) A(x a)3 o A
2
est intelligemment choisi.
f(b) = f(a) +

Que devient cette formule si on remplace f par F une primitive dune fonction f de classe C1 sur [a, b] et deux fois drivable
sur ]a, b[ ? Interprtez gomtriquement.
Exercice no 4 (***)
Dterminer dans chacun des cas suivants la drive n-me de la fonction propose :
1) x 7 xn1 ln(1 + x) 2) x 7 cos3 x sin(2x) 3) x 7

x2 + 1
(x 1)3

4) x 7 (x3 + 2x 7)ex .

Exercice no 5 (***I)
2

Montrer que la fonction dfinie sur R par f(x) = e1/x si x 6= 0 et 0 si x = 0 est de classe C sur R.
Exercice no 6 (**T)

x
x+1

1
1
Montrer que pour tout rel strictement positif x, on a : 1 +
<e< 1+
.
x
x
Exercice no 7 (***I)
Soit f une fonction drivable sur R valeurs dans R vrifiant f(0) = f(a) = f (0) = 0 pour un certain a non nul. Montrer
quil existe un point distinct de O de la courbe reprsentative de f en lequel la tangente passe par lorigine.
Exercice no 8 (****) (Toute fonction drive vrifie le thorme des valeurs intermdiaires)
Soit f une fonction drivable sur un intervalle ouvert I valeurs dans R. Soient a et b deux points distincts de I vrifiant
f (a) < f (b) et soit enfin un rel m tel que f (a) < m < f (b).
f(a + h) f(a)
f(b + h) f(b)
<m<
.
h
h
f(y + h) f(y)
puis quil existe x tel que f (x) = m.
2) Montrer quil existe y dans [a, b] tel que m =
h
Exercice no 9 (*IT)

Etudier la drivabilit droite en 0 de la fonction f : x 7 cos x.


1) Montrer quil existe h > 0 tel que

http ://www.maths-france.fr

c Jean-Louis Rouget, 2014. Tous droits rservs.


Exercice no 10 (**) (Gnralisation du thorme des accroissements finis)


Soient f et g deux fonctions continues sur [a, b] et drivables sur ]a, b[.
Soit : [a, b]
R
.
f(a) f(x) f(b) f(x)

x
7
g(a) g(x) g(b) g(x)

1) Montrer que est continue sur [a, b], drivable sur ]a, b[ et calculer sa drive.
2) En dduire quil existe c dans ]a, b[ tel que (g(b) g(a))f (c) = (f(b) f(a))g (c).

Exercice no 11 (**)
Soit f de classe C1 sur R+ telle que lim xf (x) = 1. Montrer que lim f(x) = +.
x+

x+

Exercice n 12 (***)
o

Soit f de classe C1 sur R vrifiant pour tout x rel, f f(x) =

que f est constante puis dterminer f.

x

f(x)
x
+ 3. En remarquant que f
+3 =
+ 3, montrer
2
2
2

Exercice no 13 (***I)
Soit f de classe C1 sur R vrifiant

lim (f(x) + f (x)) = 0. Montrer que

x+

lim f(x) =

x+

lim f (x) = 0. (Indication.

x+

Considrer g(x) = ex f(x)).


Exercice no 14 (***I)
Etudier la suite (un ) dans chacun des cas suivants :

1) u0 > 1 et n N, un+1 = 1 + un
3) u0 R et n N, un+1 = sin un
5) u0 R et n N, un+1 = sin(2un )

2) u0 > 1 et n N, un+1 = ln(1 + un )


4) u0 R et n N, un+1 = cos(un )
6) u0 R et n N, un+1 = u2n 2un + 2.

http ://www.maths-france.fr

c Jean-Louis Rouget, 2014. Tous droits rservs.


Planche no 22. Drivation : corrig


Exercice no 1
f est continue sur le segment [a, b] et donc est borne sur ce segment. Soit M = sup{f (x), x [a, b]}.
f(b) f(a)
(xa)+f(a))
Soit g la fonction affine qui prend les mmes valeurs que f en a et b (cest--dire x [a, b], g(x) =
ba
f(b) f(a)
puis h = f g. On va montrer que h = 0 sous lhypothse M =
.
ba
f(b) f(a)
= f (x) M 6 0. h est donc dcroissante sur
ba
[a, b]. Par suite, x [a, b], 0 = h(b) 6 h(x) 6 h(a) = 0. Ainsi, x [a, b], h(x) = 0, ou encore f = g. f est donc affine
sur [a, b].
h est drivable sur [a, b] et, pour x [a, b], h (x) = f (x)

Exercice no 2
On a dj g(b) = f(b) f(b) = 0. Puisque a 6= b, on peut
! choisir A tel que g(a) = 0
n
(k)
X
(n + 1)!
f (a)
( savoir A =
f(b)
(b a)k ).
(b a)n+1
k!
k=0

Avec les hypothses faites sur f, g est dautre part continue sur [a, b] et drivable sur ]a, b[. Le thorme de Rolle permet
alors daffirmer quil existe c ]a, b[ tel que g (c) = 0.
Pour x ]a, b[, on a
g (x) =

n
n
X
X
f(k+1) (x)
(b x)n
f(k) (x)
(b x)k +
(b x)k1 + A
k!
(k 1)!
n!

k=0

k=1

n1
n
X f(k+1) (x)
X
(b x)n
f(k+1) (x)
(b x)k +
(b x)k + A
=
k!
k!
n!
k=0
(n+1)

k=0

(b x)n
n!
n!

(b x)n 
A f(n+1) (x) .
=
n!
=

Ainsi, il existe c ]a, b[ tel que

(x)

(b x)n + A

(b c)n
(A f(n+1) (c)) = 0 et donc, puisque c 6= b, tel que A = f(n+1) (c).
n!

Lgalit g(a) = 0 scrit alors


f(b) =

n
X
f(k) (a)
(b a)n+1 f(n+1) (c)
(b a)k +
,
k!
(n + 1)!

k=0

pour un certain rel c de ]a, b[.


Exercice no 3
xa
(f (x) + f (a)) A(x a)3 o A est choisi de sorte que g(b) = g(a) = 0
Pour x [a, b], posons g(x) = f(x) f(a)
2


ba
1

f(b)

f(a)

(cest--dire A =
(f
(b)
+
f
(a))
).
(b a)3
2
f C2 ([a, b], R) D3 (]a, b[, R) et donc g C1 ([a, b], R) D2 (]a, b[, R). Pour x [a, b], on a
1
x a
g (x) = f (x) (f (x) + f (a))
f (x) 3A(x a)2 ,
2
2
puis pour x ]a, b[,
1
x a (3)
xa
1
f (x) 6A(x a) =
(12A f(3) (x)).
g (x) = f (x) f (x) f (x)
2
2
2
2
g est continue sur [a, b], drivable sur ]a, b[ et vrifie de plus g(a) = g(b). Donc, daprs le thorme de Rolle, il
existe d ]a, b[ tel que g (d) = 0. De mme, g est continue sur [a, d] [a, b], drivable sur ]a, d[(6= ) et vrifie de
plus g (a) = g (d)(= 0). Daprs le thorme de Rolle, il existe c ]a, d[]a, b[ tel que g (c) = 0 ou encore tel que
1
A = f(3) (c) (puisque c 6= a).
12
http ://www.maths-france.fr

c Jean-Louis Rouget, 2014. Tous droits rservs.


En crivant explicitement lgalit g(b) = 0, on a montr que :


1
ba
(f (b) + f (a)) f(3) (c)(b a)3 .
2
12

c ]a, b[/ f(b) = f(a) +

Si f C1 ([a, b], R) D2 (]a, b[, R) et si F est une primitive de f sur [a, b], la formule prcdente scrit :
Zb
a

1
ba
(F (b) + F (a)) F(3) (c)(b a)3
2
12
ba
1
=
(f(b) + f(a)) f (c)(b a)3 .
2
12

f(t) dt = F(b) F(a) =

Donc, si f C1 ([a, b], R) D2 (]a, b[, R),


c ]a, b[/

Zb

f(t) dt =

1
ba
(f(b) + f(a)) f (c)(b a)3 .
2
12

Interprtation gomtrique.
Zb
Si f est positive, A1 =
f(t) dt est laire du domaine D = {M(x, y) R2 / a 6 x 6 b et 0 6 y 6 f(x)} et A2 =
a





ba
a
b
b
a
(f(b) + f(a)) est laire du trapze
. Si M2 = sup{|f (x)|, x [a, b]} existe dans
0
0
f(b)
f(a)
2
R, on a :
|A1 A2 | 6 M2

(b a)3
.
12

Exercice no 4
1) Pour x > 1, posons fn (x) = xn1 ln(1 + x). Pour n > 1, fn est n fois drivable sur ] 1, +[ et pour x > 1, on a
daprs la formule de Leibniz :

f(n)
n (x) =

n  
X
n

(k)

(ln(1 + x))(nk)
k
k=0
n1
X n
(k)
=
xn1
(ln(1 + x))(nk) (car (xn1 )(n) ) = 0)
k
k=0
(nk1)

n1
X n

1
n1 (k)
=
x
1+x
k
k=0
n1
X n (n 1)!
(n 1 k)!
xn1k (1)n1k
=
(x + 1)nk
k (n 1 k)!
k=0
n1
X n (x)nk1
= (n 1)!
.
k (x + 1)nk
xn1

k=0

http ://www.maths-france.fr

c Jean-Louis Rouget, 2014. Tous droits rservs.


(n)

Puis, pour x = 0, fn (0) = n (n 1)! = n!, et pour x ] 1, 0[]0, +[, daprs la formule du binme de Newton,
f(n)
n (x)

nk

n

n1   
x
(n 1)!
(n 1)! X n
x
1
=
1
=

x
x+1
x
x+1
k
k=0

(n 1)! (x + 1)n 1
=
.
x
(x + 1)n
2) On sait driver facilement des sommes ou plus gnralement des combinaisons linaires. Donc, on linarise.
 




1
1
1 ix
ix 3
e +e
e2ix e2ix =
e3ix + 3eix + 3eix + e3ix e2ix e2ix
cos x sin(2x) =
8
2i
16i
1 5ix
1
=
(e
+ 3e3ix + 2eix 2eix 3e3ix e5ix ) = (sin(5x) + 3 sin(3x) + 2 sin(x))
16i
8
3

Puis, pour n naturel donn :





1 n



5 sin 5x + n
+ 3n+1 sin 3x + n
+ 2 sin x + n
,
8
2
2
2
expression que lon peut dtailler suivant la congruence de n modulo 4.
(cos3 x sin 2x)(n) =

3) On sait driver des objets simples et donc on dcompose en une somme de fractions plus simples. Pour tout rel x 6= 1,
x2 2x + 1 + 2x 2 + 2
1
2
2
x2 + 1
=
=
+
.
+
3
(x 1)
(x 1)3
x 1 (x 1)2
(x 1)3
Puis, pour n entier naturel donn et x 6= 1,


x2 + 1
(x 1)3

(n)

(1)n n!
(1)n (n + 1)! (1)n (n + 2)!
+
2
+
(x 1)n+1
(x 1)n+2
(x 1)n+3
n
(1) n!
=
((x 1)2 + 2(n + 1)(x 1) + (n + 2)(n + 1))
(x 1)n+3

(1)n n! x2 + 2nx + n2 + n + 1
.
=
(x 1)n+3

(x) =

4) La fonction propose est de classe C sur R en vertu de thormes gnraux. La formule de Leibniz fournit pour
n>3:
n  
3  
X
X
n
n
(x3 + 2x 7)(k) (ex )(nk) =
(x3 + 2x 7)(k) (ex )(nk)
k
k
k=0
k=0


n(n

1)
n(n 1)(n 2)
= (x3 + 2x 7) + n(3x2 + 2) +
(6x) +
6 ex
2
6
 x
3
2
2
3
2
= x + 3nx + (3n 3n + 2)x + n 3n + 4n 7 e .

((x3 + 2x 7)ex )(n) =

Enfin, non vrifie directement que cette formule reste valable pour n {0, 1, 2}.
Exercice no 5
f est de classe

sur R en vertu de thormes gnraux.

Montrons par rcurrence que n N, Pn R[X]/ x R , f(n) (x) =

Pn (x) 1/x2
e
.
x3n

Cest vrai pour n = 0 avec P0 = 1.

Pn (x)
2
Soit n > 0. Supposons que Pn R[X]/ x R , f(n) (x) = 3n e1/x . Alors, pour x R ,
x



2
2
Pn+1 (x)
1
1
2 Pn (x)
(n+1)

e1/x = 3(n+1) e1/x ,


f
(x) =
+ Pn (x) 3n 3nPn (x) 3n+1
x3 x3n
x
x
x
o, pour tout rel x, Pn+1 (x) = 2Pn (x)+ x3 Pn (x)3nx2 Pn (x). Puisque Pn+1 est un polynme, on a montr par rcurrence
que
http ://www.maths-france.fr

c Jean-Louis Rouget, 2014. Tous droits rservs.


Pn (x) 1/x2
e
.
x3n
Montrons alors par rcurrence que pour tout entier naturel n, f est de classe Cn sur R et que f(n) (0) = 0.
n N, Pn R[X]/ x R , f(n) (x) =

Pour n = 0, f est continue sur R et de plus,

lim

x0, x6=0

f(x) = lim eX = 0 = f(0). Donc, f est continue sur R.


X

sur R et que f(n) (0) = 0. Alors, dune part f est de classe Cn sur
Pn+1 (x) 1/x2
e
tend vers 0
R et Cn+1 sur R et de plus, daprs un thorme de croissances compares, f(n+1) (x) =
x3n+3
n+1
quand x tend vers 0, x 6= 0. Daprs un thorme classique danalyse, f est de classe C
sur R et en particulier,
f(n+1) (0) = lim f(n+1) (x) = 0.
n

Soit n > 0. Supposons que f est de classe C

x0, x6=0

On a montr par rcurrence que n N, f est de classe Cn sur R et que f(n) (0) = 0. f est donc de classe C sur R.
Exercice no 6
Montrons que : x > 0,


x
x+1

1
1
1+
< e < 1+
. Soit x > 0.
x
x

x
x+1






1
1
1
1
< 1 < (x + 1) ln 1 +
<e< 1+
x ln 1 +
1+
x
x
x
x
x(ln(x + 1) ln x) < 1 < (x + 1)(ln(x + 1) ln x)
1
1
< ln(x + 1) ln x < .

x+1
x

Soit x un rel strictement positif fix. Pour t [x, x + 1], posons f(t) = ln t. f est continue sur [x, x + 1] et drivable sur
]x, x+1[. Daprs le thorme des accroissements finis, il existe un rel c dans ]x, x+1[ tel que f(x+1)f(x) = (x+1x)f (c)
ou encore
c ]x, x + 1[/ ln(x + 1) ln x =
Ceci montre que x > 0,

1
,
c

1
1
< ln(x + 1) ln x < , et donc que
x+1
x
x
x+1


1
1
<e< 1+
.
x > 0, 1 +
x
x

Exercice no 7
Supposons par exemple a > 0 (si a < 0, on applique le travail ci-dessous la fonction x 7 f(x)).

a
Soit x0 un rel non nul. Une quation de la tangente (Tx0 ) la courbe reprsentative de f au point dabscisse x0 est
y = f (x0 )(x x0 ) + f(x0 ). (Tx0 ) passe par lorigine si et seulement si
x0 f (x0 ) f(x0 ) = 0.

f(x)
si x 6= 0
(g est la fonction pente lorigine ).
Pour x rel, on pose g(x) =
x

0 si x = 0
http ://www.maths-france.fr

c Jean-Louis Rouget, 2014. Tous droits rservs.


Puisque f est continue et drivable sur R, g est dj continue et drivable sur R .


Puisque f est drivable en 0 et que f(0) = f (0) = 0, g est de plus continue en 0.
Finalement, g est continue sur [0, a], drivable sur ]0, a[ et vrifie g(0) = g(a)(= 0). Daprs le thorme de Rolle, il existe
x0 f (x0 ) f(x0 )
. Lgalit g (x0 ) = 0
un rel x0 dans ]0, a[ tel que g (x0 ) = 0. Puisque x0 nest pas nul, on a g (x0 ) =
x20
scrit x0 f (x0 ) f(x0 ) = 0 et, daprs le dbut de lexercice, la tangente la courbe reprsentative de f au point dabscisse
x0 passe par lorigine.
Exercice no 8
f(a + h) f(a)
f(b + h) f(b)
= f (a) et que lim
= f (b), on a (en
h0
h
h
appliquant la dfinition de la limite avec = Min{m f (a), f (b) m} > 0)


f(a + h) f(a)
< m et
h1 > 0/ h ]0, h1 [, a + h I
h


f(b + h) f(b)
h2 > 0/ h ]0, h2 [ b + h I
>m .
h
1) Soit m un lment de ]f (a), f (b)[. Puisque lim

h0

Lensemble E = {h ]0, Min{h1 , h2 }[/ a + h et b + h sont dans I} nest pas vide (car I est ouvert) et pour tous les h de E,
f(b + h) f(b)
f(a + h) f(a)
<m<
.
on a :
h
h
h > 0 est ainsi dornavant fix.
f(x + h) f(x)
est continue sur [a, b]. Daprs le thorme
2) La fonction f est continue sur I et donc, la fonction g : x 7
h
f(y + h) f(y)
des valeurs intermdiaires, comme g(a) < m < g(b), y [a, b]/ g(y) = m ou encore y [a, b]/
= m.
h
f(y + h) f(y)
= f (x).
Maintenant, daprs le thorme des accroissements finis, x ]y, y + h[ I/ m =
h
On montr quune fonction drive (nest pas ncessairement continue mais) vrifie le thorme des valeurs intermdiaires
(Thorme de Darboux).
Exercice no 9
1re solution. Quand x tend vers 0 par valeurs suprieures,

1
1 cos( x) 1
cos( x) 1
2
.
=
x
2 ( x) /2
2
1
f est donc drivable droite en 0 et fd (0) = .
2

1
2me solution.
f est continue sur [0, +[ et de classe C sur ]0, +[ en vertu de thormes gnraux. Pour x > 0,
sin x
1

f (x) =
. Quand x tend vers 0, f tend vers .
2
2 x

1
En rsum, f est continue sur [0, +[, de classe C1 sur ]0, +[ et f a une limite relle quand x tend vers 0 savoir .
2
1
1

On en dduit que f est de classe C sur [0, +[ et en particulier, f est drivable en 0 et f (0) = .
2
Remarque. On a dmontr dans la deuxime solution un rsultat plus fort que celui dmontr dans la premire solution.
Exercice no 10
Pour tout rel x de [a, b], (x) = (f(a) f(x))(g(b) g(x)) (g(a) g(x))(f(b) f(x)). La fonction est donc continue
sur [a, b], drivable sur ]a, b[ et pour x ]a, b[,
(x) = f (x)(g(b) g(x)) g (x)(f(a) f(x)) + g (x))(f(b) f(x)) + f (x)(g(a) g(x))
= f (x)(g(a) g(b)) + g (x)(f(b) f(a)).
De plus, (a) = (b)(= 0). Donc, daprs le thorme de Rolle, c ]a, b[/ (c) = 0.

Lgalit (c) = 0 scrit : f (c)(g(b) g(a)) = g (c)(f(b) f(a)) ce quil fallait dmontrer.
Remarque. Ce rsultat gnralise le thorme des accroissements finis (g = Id est le thorme des accroissements
finis.)
http ://www.maths-france.fr

c Jean-Louis Rouget, 2014. Tous droits rservs.


Exercice no 11
Puisque lim xf (x) = 1, A > 0/ x > 0, (x > A xf (x) >
x+

Soit x un rel fix suprieur ou gal A. t [A, x], f (t) >

1
).
2

1
et donc, par croissance de lintgrale,
2x

ce qui fournit :

Zx

f (t) dt >

Zx

1
dt
2t

1
x A, f(x) > f(A) + (ln x ln A),
2
et montre que lim f(x) = +.
x+

Exercice n 12 On remarque tout dabord que


o

x


f(x)
+ 3 = f(f f(x)) = f f(f(x)) =
+ 3.
2
2
 1
1 x
+ 3 = f (x), et donc
Puisque f est drivable sur R, on obtient en drivant : x R, f
2
2
2
x

x R, f
+ 3 = f (x).
2
1
Soit alors x un rel donn et u la suite dfinie par u0 = x et n N, un+1 = un + 3.
2
Daprs ce qui prcde, n N, f (un ) = f (u0 ) = f (x). Maintenant, u est une suite arithmtico-gomtrique et on sait
que
x Rf

1
(u0 6)
2n
ce qui montre que la suite u converge vers 6. La suite (f (un ))n0 est constante, de valeur f (x). f tant continue sur R,
on en dduit que


lim un = f (6),
x R, f (x) = lim f (un ) = f
n N, un 6 =

n+

n+

ce qui montre que la fonction f est constante sur R et donc que f est affine.
Rciproquement, pour x rel, posons f(x) = ax + b o a et b sont deux rels.


x
1
2
x + ab + b 3 = 0
f solution x R, a(ax + b) + b = + 3 x R, a
2
2








1
1
1
2
a = et (a + 1)b = 3 a = et b = 3 2 2
ou a = et b = 3 2 + 2
.
2
2
2

On trouve deux fonctions solutions, les fonctions f1 et f2 dfinies par :





1
1
x R, f1 (x) = x + 3 2 2 et f2 (x) = x + 3 2 + 2 .
2
2
Exercice no 13
Montrons que lim f(x) = 0.
x+

Pour x rel, posons g(x) = ex f(x). g est drivable sur R et x R, g (x) = ex (f(x) + f (x)). Il sagit donc maintenant de
montrer que si lim ex g (x) = 0 alors lim ex g(x) = 0.
x+

x+

Soit un rel strictement positif.


< ex g (x) < ex 6 g (x) 6 ex .
2
2
2
2
Pour x rel donn suprieur ou gal A, on obtient en intgrant sur [A, x] (puisque g est continue sur [A, x])
Zx
Zx
t

x
A
e dt 6
g (t) dt = g(x) g(A) 6 (ex eA ),
(e e ) =
2
2
2
A
A
A > 0/ x R,

x>A

et donc

http ://www.maths-france.fr

c Jean-Louis Rouget, 2014. Tous droits rservs.





1 eAx 6 ex g(x) 6 g(A)ex +


1 eAx .
2
2

Maintenant, g(A)ex (1 eAx ) et g(A)ex + (1 eAx ) tendent respectivement vers et quand x tend vers
2
2
2
2
+. Donc,



B > A/ x R, x > B g(A)ex (1 eAx ) > et g(A)ex + (1 eAx ) < .


2
2
x
Pour x > B, on a donc < e g(x) < .
On a montr que > 0, B > 0/ x R, (x > B |ex g(x)| < ) et donc que lim ex g(x) = 0 ce quil fallait
x > A, g(A)ex

x+

dmontrer.

Exercice no 14
1) Pour x > 1, posons f(x) =

1 + x et g(x) = f(x) x.

Soit u0 I = [1, +[. f est dfinie sur I et de plus f(I) = [0, +[ [1, +[. On en dduit, par rcurrence, que la
suite u est dfinie et que n N, un [1, +[.
Si la suite u converge, puisque n N, un > 1, sa limite vrifie > 1. Puisque f est continue sur [1, +[ et donc
en ,


= lim un+1 = lim f(un ) = f
lim un = f().
n+

n+

n+

et est un point fixe de f. Or, pour x > 1,

!
1 5
1+ 5
x=
et x > 0
ou x =
2
2

1 + x = x 1 + x = x2 et x > 0

5+1
x=
.
2

Ainsi, si la suite (un ) converge, cest vers le nombre =

5+1
(le nombre dor).
2

Pour x > 1,

sgn(f(x) ) = sgn( 1 + x 1 + ) = sgn((1 + x) (1 + ))


= sgn(x ).

(par croissance de x 7 x2 sur [0, +[)

Ainsi, les intervalles [1, [ et ], +[ sont stables par f. Donc, si 1 6 u0 < , alors par rcurrence n N, 1 6 un <
et si u0 > , alors par rcurrence, n N, un > .

Soit x > 1. Si x [1, 0], 1 + x x > 0 et si x > 0,

sgn(g(x)) = sgn( 1 + x x)
= sgn((1 + x) x2 ) (par croissance de x 7 x2 sur [0, +[)
!
!!

1+ 5
51
x
= sgn( x) (car ici x > 0).
= sgn
x+
2
2
On en dduit que, si x [1, [, f(x) > x, et si x ], +[, f(x) < x. Mais alors, si 1 6 u0 < , puisque n N, 1 6
un < , pour n entier naturel donn, on a
un+1 = f (un ) > un .
La suite u est donc strictement croissante, majore par et donc convergente. On sait de plus que sa limite est ncessairement .
Si u0 > , puisque n N, un > , pour n entier naturel donn, on a
un+1 = f (un ) < un .

http ://www.maths-france.fr

c Jean-Louis Rouget, 2014. Tous droits rservs.


La suite u est donc strictement dcroissante, minore par et donc convergente. On sait de plus que sa limite est
ncessairement . Enfin, si u0 = , la suite u est constante.
En rsum,"

"

5+1
5+1
, la suite u est strictement croissante, convergente de limite
,
- si u0 1,
2
2
"
#

5+1
5+1
, + , la suite u est strictement dcroissante, convergente de limite
,
- si u0
2
2

5+1
5+1
- si u0 =
, la suite u est constante et en particulier convergente de limite
.
2
2

1+ 5
On note que dans tous les cas, la suite u est convergente et que lim un =
.
n+
2

y=x

y=

1+x

1
u0
u0
1

2) Si u0 > 0, alors puisque f est dfinie sur lintervalle I =]0, +[ et que I est stable par f (x > 0, ln(1 + x) > ln 1 = 0),
la suite u est dfinie et est strictement positive. Si la suite u converge, sa limite est un rel positif ou nul. Par continuit
de f sur [0, +[ et donc en ,


= lim un+1 = lim f (un ) = f
lim un = f().
n+

n+

n+

Pour x > 1, posons g(x) = ln(1 + x) x. g est dfinie et drivable sur ] 1, +[ et pour x > 1,
g (x) =

1
x
1=
.
1+x
1+x

g est strictement positive sur ] 1, 0[ et strictement ngative sur ]0, +[. g est donc strictement croissante sur ] 1, 0] et
strictement dcroissante sur [0, +[. Par suite, si x ] 1, 0[]0, +[, g(x) < 0. En particulier, pour x ] 1, 0[]0, +[,
f(x) 6= x. Puisque f(0) = 0, f admet dans ] 1, +[ un et un seul point fixe savoir 0.
En rsum, si u0 > 0, la suite u est dfinie, strictement positive, et de plus, si la suite u converge, alors

lim un = 0.

n+

Mais, pour n entier naturel donn,


un+1 un = ln(1 + un ) un < 0.
Par suite, la suite u est strictement dcroissante, minore par 0 et donc, daprs ce qui prcde, converge vers 0. Si u0 = 0,
la suite u est constante.
Il reste donc tudier le cas o u0 ] 1, 0[. Montrons par labsurde quil existe un rang n0 tel que un0 6 1. Dans le
cas contraire, n N, un > 1. Comme prcdemment, par rcurrence, la suite u est valeurs dans ]1, 0[ et strictement
dcroissante. Etant minore par 1, la suite u converge vers un certain rel .
Puisque n N, 1 < un 6 u0 < 0, on a 1 6 6 u0 < 0. Donc, ou bien = 1, ou bien f est continue en et est un
point fixe de f lment de ] 1, 0[.
On a vu que f nadmet pas de point fixe dans ] 1, 0[ et donc ce dernier cas est exclu. Ensuite, si = 1, il existe un rang
N tel que uN 6 0, 9. Mais alors, uN+1 6 ln(0, 9 + 1) = 2, 3... < 1 ce qui constitue de nouveau une contradiction.
Donc, il existe un rang n0 tel que un0 6 1 et la suite u nest plus dfinie partir dun certain rang.
http ://www.maths-france.fr

c Jean-Louis Rouget, 2014. Tous droits rservs.


En rsum,
- si u0 ]0, +[, la suite u est strictement dcroissante, convergente et

lim un = 0,

n+

- si u0 = 0, la suite u est constante,


- si u0 ] 1, 0[, la suite u nest pas dfinie partir dun certain rang.

y=x

y = ln(1 + x)

1
u0
1

u0
1

3) Pour tout choix de u0 , u1 [1, 1]. On supposera dornavant que u0 [1, 1]. Si u0 = 0, la suite u est constante.

Si u0 [1, 0[, considrons la suite u dfinie par u0 = u0 et n N, un+1


= sin(un ). La fonction x 7 sin x tant

impaire, il est clair par rcurrence que n N, un = un . On supposera dornavant que u0 ]0, 1].
i i
Puisque ]0, 1] 0, , on a sin]0, 1] ]0, 1] et lintervalle I =]0, 1] est stable par f. Ainsi, si u0 ]0, 1], alors par rcurrence
2
n N, un ]0, 1].
Pour x [0, 1], posons g(x) = sin x x. g est drivable sur [0, 1] et pour x [0, 1], g (x) = cos x 1. g est strictement
ngative sur ]0, 1] et donc g est strictement dcroissante sur [0, 1]. On en dduit que pour x ]0, 1], g(x) < g(0) = 0.

Mais alors, pour n entier naturel donn, un+1 = sin (un ) < un . La suite u est ainsi strictement dcroissante, minore
par 0 et donc converge vers [0, 1]. La fonction x 7 sin x est continue sur [0, 1] et donc, est un point fixe de f. Ltude
de g montre que f a un et un seul point fixe dans [0, 1] savoir 0. La suite u est donc convergente et lim un = 0.
n+

Ltude prliminaire montre la suite u converge vers 0 pour tout choix de u0 .

http ://www.maths-france.fr

c Jean-Louis Rouget, 2014. Tous droits rservs.


y=x
1

u0
2

y = sin x

h i
puis u2 [0, 1]. On supposera dornavant que u0 [0, 1].
4) Si u0 est un rel quelconque, u1 [1, 1] ,
2 2
On a cos([0, 1]) = [cos 1, cos 0] = [0, 504..., 1] [0, 1]. Donc, la fonction x 7 cos x laisse stable lintervalle I = [0, 1]. On
en dduit que n N, un [0, 1].
Pour x [0, 1], on pose g(x) = cos x x. g est somme de deux fonctions strictement dcroissantes sur [0, 1] et est donc
strictement dcroissante sur [0, 1]. De plus, g est continue sur [0, 1] et vrifie g(0) = cos 0 > 0 et g(1) = cos 1 1 < 0. g
sannule donc une et une seule fois sur [0, 1] en un certain rel . Ainsi, f admet sur [0, 1] un unique point fixe, savoir
. Puisque f est continue sur le segment [0, 1], on sait que si la suite u converge, cest vers .
La fonction f : x 7 cos x est drivable sur [0, 1] et pour x [0, 1],

|f (x)| = | sin x| sin 1 < 1.

Lingalit des accroissements finis montre alors que (x, y) [0, 1]2 , | cos x cos y| 6 sin(1)|x y|. Pour n entier naturel
donn, on a alors
|un+1 | = |f(un ) f()| 6 sin 1|un |,
et donc, pour tout entier naturel n,
|un | 6 (sin 1)n |u0 | 6 (sin 1)n .
Comme 0 6 sin 1 < 1, la suite (sin 1)n converge vers 0, et donc la suite (un )nN converge vers . On peut noter
que puisque la fonction x 7 cos x est strictement dcroissante sur [0, 1], les deux suites (u2n )nN et (u2n+1 )nN sont
strictement monotones, de sens de variations contraires (dans le cas o u0 [0, 1]. On peut noter galement que si
ln(102 )
n >
= 26, 6..., alors (sin 1)n < 102 . Par suite, u27 est une valeur approche de 102 prs. La machine
ln(sin 1)
fournit = 0, 73... (et mme = 0, 739087042.....).
y=x
1

u0

4
y = cos x

1
5) Si u0 est un rel quelconque, alors n N , un [1, 1]. On supposera sans perte de gnralit que u0 [1, 1].
Si u0 = 0, la suite u est constante et dautre part, ltude du cas u0 [1, 0[ se ramne, comme en 3), ltude du cas
u0 ]0, 1]. On supposera dornavant que u0 ]0, 1].
Si x ]0, 1], alors 2x ]0, 2] ]0, [ et donc sin(2x) ]0, 1]. Lintervalle I =]0, 1] est donc stable par la fonction
f : x 7 sin(2x). On en dduit que n N, un ]0, 1].
http ://www.maths-france.fr

10

c Jean-Louis Rouget, 2014. Tous droits rservs.



Pour x [0, 1], posons g(x)
1. g est donc
i pour x [0, 1], g (x) = 2icos(2x)
h 1] et
h =isin(2x) x. g est drivable sur [0,
i
, 1 . On en dduit que si x 0, , g(x) > g(0) = 0.
et strictement dcroissante sur
strictement croissante sur 0,
6
6
6
h i
  3
Dautre part, g est continue et strictement dcroissante sur
=
, 1 et vrifie g
> 0 et g(1) = sin 2 1 < 0.
6  2
6
i h
i6 h

= 1 > 0 et donc
, 1 . On note que g
,1 .
g sannule donc une et une seule fois en un certain rel
6
4
4
i h 4
En rsum, g sannule une et une seule fois sur ]0, 1] en un certain rel
, 1 , g est strictement positive sur ]0, [ et
4
strictement ngative sur ], 1].
h i
i h
et montrons par labsurde que n0 N/ un0
Supposons que u0 0,
, 1 . Dans le cas contraire, tous les un sont
4
4
i h
dans 0, . Mais alors, pour tout entier naturel n,
4

un+1 un = f(un ) un = g(un ) > 0.


h

i
La suite u est donc strictement croissante. Etant majore par , la suite u converge. Comme g est continue sur u0 ,
4
4
h
h
i
i
et que n N, un u0 , , on sait que la limite de u est un point fixe de f lment de u0 , . Mais ltude de g a
4
4
montr que f nadmet pas de point fixe dans cet intervalle (u0 tant strictement positif). On aboutit une contradiction.
h i
h i
i h
et dans ce cas, n0 N/ un0
Donc, ou bien u0
, 1 , ou bien u0 0,
,1 .
4
4
h i 4
h i h
i h i
Dans tous les cas, n0 N/ un0

, 1 . Mais alors, puisque f


, 1 = sin 2, sin
, 1 (car sin 2 = 0, 909... >
4
4
2
4

0, 785... = ), pour tout entier n > n0 , un [ , 1].


4
4
h i
, 1 , |g (x)| = |2 cos(2x)| 6 |2 cos 2|. Lingalit des accroissements finis montre alors que n > n0 , |un+1 | 6
Pour x
4
|2 cos 2| |un |, puis que
n > n0 , |un | 6 |2 cos 2|nn0 |un0 | .

Comme |2 cos 2| = 0, 83... < 1, on en dduit que la suite u converge vers . La machine donne par ailleurs = 0, 947....
y=x
y = sin(2x)

u0
2

u0

6) Pour x R,
x2 2x + 2 = x x2 3x + 2 = 0 (x 1)(x 2) = 0 x = 1 ou x = 2.

Donc, si la suite u converge, ce ne peut tre que vers 1 ou 2.


Pour n N,

un+1 un = (u2n 2un + 2) un = (un 1)(un 2)


un+1 1 = u2n 2un + 1 = (un 1)2 (II)
un+1 2 = u2n 2un = un (un 2) (III).

(I)

1er cas. Si u0 = 1 ou u0 = 2, la suite u est constante.


2me cas. Si u0 ]1, 2[, (II) et (III) permettent de montrer par rcurrence que n N, un ]1, 2[. (I) montre alors que
la suite u est strictement dcroissante. Etant minore par 1, elle converge vers un rel [1, u0 ] [1, 2[. Dans ce cas, la
suite (un ) converge vers 1.
http ://www.maths-france.fr

11

c Jean-Louis Rouget, 2014. Tous droits rservs.


3me cas. Si u0 ]2, +[, (III) permet de montrer par rcurrence que n N, un > 2. Mais alors, (I) montre que la
suite u est strictement croissante. Si u converge, cest vers un rel [u0 , +[]2, +[. f nayant pas de point fixe dans
cet intervalle, la suite u diverge et, u tant strictement croissante, on a lim un = +.
n+

4me cas. Si u0 ]0, 1[, alors u1 = (u0 1)2 + 1 ]1, 2[ ce qui ramne au deuxime cas. La suite u converge vers 1.
5me cas. Si u0 = 0, alors u1 = 2 et la suite u est constante partir du rang 1. Dans ce cas, la suite u converge vers 2.
6me cas. Si u0 < 0, alors u1 = u2n 2un + 2 > 2, ce qui ramne au troisime cas. La suite u tend vers +.
En rsum, si u0 ]0, 2[, la suite u converge vers 1, si u0 {0, 2}, la suite u converge vers 2 et si u0 ] , 0[]2, +[,
la suite u tend vers +.
y = x2 2x + 2
y=x
3

http ://www.maths-france.fr

u0

u0

u0 2 u0

12

c Jean-Louis Rouget, 2014. Tous droits rservs.


Planche no 23. Comparaison des fonctions en un point


* trs facile ** facile *** difficult moyenne **** difficile
I : Incontournable T : pour travailler et mmoriser le cours
Exercice no 1
Etudier lexistence et la valeur ventuelle des limites suivantes

1/(2x)

cos x

1) lim (sin x)

2) lim | tan x|

4) lim (cos x)ln |x|


x0
1/ sin x

1 + tan x
7) lim
x0
1 + th x

5) lim cos x e1/(1sin x)

x/2

x/2

x/2

8)

13)

lim
x1/ 2

(ln x)ln(ex)

(sin x)x xsin x


x0, x>0 ln(x x2 ) + x ln x
 !x
cos a + x1
(o cos a 6= 0)
14) lim
x+
cos a

x ln(ch x 1)
x+
x2 + 1

10) lim

(Arcsin x)2

lim

xe, x<e

11)

2
16

2x2 1

lim



n


n
n
+ sin
3) lim
cos
n+
3n + 1
6n + 1
2
2 cos x + cos x 1
6) lim
x/3 2 cos2 x 3 cos x + 1
xx 1


9) lim

x1, x>1
ln 1 x2 1
x

ln(x + 1)
12) lim
x+
ln x

Exercice no 2
Dterminer les dveloppements limits lordre demand au voisinage des points indiqus :
r
1
x
1
(ordre
7
en
0)
2)
(ordre
7
en
0)
3)
Arccos
(ordre 3 en 0)
1)
1 x2 x3
cos x
tan x



2
4) tan x (ordre 3 en )
5) (ch x)1/x (ordre 2 en 0)
6) tan3 x cos x2 1 (ordre 8 en 0)
4
r
x+1
ln(1 + x)
(ordre
3
en
1)
8)
Arctan(cos
x)
(ordre
5
en
0)
9)
Arctan
(ordre 2 en 0)
7)
x2
x +!2
2
Zx
99
X
1
xk
1
1

(ordre
5
en
0)
11)
dt
(ordre
10
en
0)
12)
ln
10) 2
(ordre 100 en 0)
x
k!
Arcsin2 x
1 + t4
x
k=0
p
13) tan 3 4 (3 + x3 ) (ordre 3 en )
Exercice no 3

Soit 0 < a < b. Etude complte de la fonction f(x) =


Exercice no 4
Etude au voisinage de + de

x2 3

a x + bx
2

1/x

3
8x3 + 7x2 + 1.

Exercice no 5
x
. Calculer f(n) (0) en moins de 10 secondes puis f(n) (x) pour |x| 6= 1 en peine plus de temps).
Soit f(x) =
1 x2
Exercice no 6

1) Equivalent simple en + et de x2 + 3x + 5 x + 1.
2) Equivalent simple en 0, 1, 2 et + de 3x2 6x
2

3) Equivalent simple en 0 de (sin x)xx (x x2 )sin x .


4) Equivalent simple en + de xth x .
5) Equivalent simple en 0 de tan(sin x) sin(tan x).
Exercice no 7
Dveloppement asymptotique la prcision

n
1
1 X
de
u
=
k!.
n
n3
n!
k=0

http ://www.maths-france.fr

c Jean-Louis Rouget, 2014. Tous droits rservs.


Exercice no 8
1) Dveloppement asymptotique la prcision x2 en 0 de
2) Dveloppement asymptotique la prcision

1
1
.
x(ex 1) x2

1
en + de x ln(x + 1) (x + 1) ln x.
x3

Exercice no 9

x n
Soient a > 0 et b > 0. Pour n N et x R, on pose fn (x) = 1 +
.
n
1) Equivalent simple quand n tend vers + de fn (a + b) fn (a)fn (b).
2) Mme question pour ea fn (a) 1 +

a2
.
2n

Exercice no 10
i i
Soit u0 0,
. Pour n N, on pose un+1 = sin(un ).
2
1) Montrer brivement que la suite u est strictement positive et converge vers 0.

2) a) Dterminer un rel tel que la suite u


n+1 un ait une limite finie non nulle.
b) En utilisant le lemme de Cesaro, dterminer un quivalent simple de un .

Exercice no 11
Soit u la suite dfinie par la donne de son premier terme u0 > 0 et la relation n N, un+1 = un eun . Equivalent
simple de un quand n tend vers +.
Exercice no 12
1) Montrer que lquation tan x = x a une unique solution dans lintervalle [n, (n + 1)] pour n entier naturel donn. On
note xn cette solution.
2) Trouver un dveloppement asymptotique de xn la prcision

1
.
n2

Exercice no 13
1) Montrer que lquation x + ln x = k admet, pour k rel donn, une unique solution dans ]0, +[, note xk .


ln k
ln k
o a, b et c sont des constantes
+o
2) Montrer que, quand k tend vers +, on a : xk = ak + b ln k + c
k
k
dterminer.
Exercice no 14
1
si x 6= 0 et 1 si x = 0.
x2
1) Montrer que f admet en 0 un dveloppement limit dordre 2.
Soit f(x) = 1 + x + x2 + x3 sin

2) Montrer que f est drivable sur R.


3) Montrer que f nadmet en 0 aucun dveloppement limit daucun ordre que ce soit.
Exercice no 15
Etude au voisinage de 0 de f(x) =

1
1

(existence dune tangente ?)


x Arcsin x

Exercice no 16
1) La fonction x 7 Arccos x admet-elle en 1 ( gauche) un dveloppement limit dordre 0 ? dordre 1 ?

2) Equivalent simple de Arccos x en 1.


Exercice no 17

1) Dveloppement limit lordre n en 0 de f(x) =

1
.
(1 x)2 (1 + x)

2) Soit ak le k-me coefficient. Montrer que ak est le nombre de solutions dans N2 de lquation p + 2q = k.

http ://www.maths-france.fr

c Jean-Louis Rouget, 2014. Tous droits rservs.


Planche no 23. Comparaison des fonctions en un point : corrig


Exercice no 1

(cest--dire un
1) Si x ]0, [, sin x > 0, de sorte que la fonction propose est bien dfinie sur un voisinage point de
2

voisinage de
auquel on a enlev le point ).
2
2
2




(2x )2
2 x
ln(sin x) sin x 1 = 1 cos
x

=
.
x 2
x 2
2
2
8
Donc,
2)

ln(sin x)
2x
ln(sin x)

. On en dduit que lim


= 0 puis que (sin x)1/(2x) = eln(sin x)/(2x) e0 = 1.
x 2 2x
x 2
2x
8
ln | tan x| = ln | sin x| ln | cos x| ln | cos x|,
x 2

puis cos x ln | tan x|

x
2
cos x ln | tan x|

| tan x|cos x = e
3) cos

cos x ln | cos x|. Par suite, lim cos x ln | tan x| lim cos x ln | cos x| = lim u ln u = 0 puis
x 2

u0

x 2

e0 = 1.

x 2

n
n
+ sin
3n + 1
6n + 1

n+

cos

1 1
+ sin = + = 1 (et on est en prsence dune indtermination du type 1 ).
3
6
2 2


1 !

 
1

1
1+
= cos

+o
3
3n
3 9n
n


 

 
 
1
1

1
1
3
3
= cos
+
= +
+o
sin
+o
+o
2
9n
n
2
9n
n
2
18n
n

n
cos
= cos
3n + 1

De mme,

1 !

 
1

1
1+
= sin

+o
6
6n
6 36n
n


 

 
 
3
3
1
1

1
1
= cos

=
.
+o
sin
+o
+o
2
36n
n
2
36n
n
2
72n
n

n
= sin
sin
6n + 1

Puis,


 !
3
n
n
1
= n ln 1 +
=n
+ sin
+o
3n + 1
6n + 1
24n
n
n


n
n
et donc cos
+ sin
e 3/24 .
n+
3n + 1
6n + 1

n ln cos

 !
3
3
1
=
+o
+ o(1),
24n
n
24

x2
x2
. Puis, ln |x|ln(cos x) ln |x| 0. Donc, (cos x)ln |x| = eln |x| ln(cos x) e0 = 1.
x0
x0
x0
x0
x0
2
2

5) Quand x tend vers ,


tend vers +. Posons h = x puis = sgn(h), de sorte que
2 1 sin x
2
4) ln(cos x) cos x1

(cos x)e1/(1sin x) = | sin h|e1/(1cos h) = eln | sin h|+ 1cos h .


Or,



h2

+ o h2 (ln |h| + o(ln |h|)) + 1


1
(1 cos h) ln | sin h| + 1
2
ln | sin h| +
=
=
x0
1 cos h
1 cos h
h2
+ o (h2 )
2
1 + o(1)
2
,
=

x0 h2
x0 h2
2
+ o(h )
2
http ://www.maths-france.fr

c Jean-Louis Rouget, 2014. Tous droits rservs.


et donc, ln | sin h| +
On en dduit que

1
2
+.

1 cos h x0 h2 x0
lim

x/2, x</2

cos x e1/(1sin x) = + et

lim

x/2, x>/2

cos x e1/(1sin x) = .

6) Pour x R, 2 cos2 x 3 cos x + 1 = (2 cos x 1)(cos x 1) et donc


h


i
x R, 2 cos2 x 3 cos x + 1 = 0 x + 2Z 2Z .
3


Pour x
/ + 2Z 2Z,
3
2 cos2 x + cos x 1
(2 cos x 1)(cos x + 1)
cos x + 1
=
=
,
2
2 cos x 3 cos x + 1
(2 cos x 1)(cos x 1)
cos x 1
1
+1
2 cos2 x + cos x 1
2
= 3.
=
et donc, lim
2
1
x/3 2 cos x 3 cos x + 1
1
2
7)
1 + tan x
1 + x + o(x)
=
= (1 + x + o(x)(1 x + o(x)) = 1 + o(x).
x0
1 + th x x0 1 + x + o(x) x0
Puis,


1
ln(1 + o(x))
o(x)
o(1)
1 + tan x
=
ln
=
=
0.
sin x
1 + th x x0 x + o(x) x0 x + o(x) x0 1 + o(1) x0
1/ sin x

1 + tan x
= e0 = 1.
Finalement, lim
x0
1 + th x
8)
ln(ln x)

ln x 1 = ln

xe

x
e

xe

x
1
1 = (e x),
e
e

puis,
1
(e x) ln(e x) 0,
xe
xe
e

ln(e x) ln(ln x)
et donc (ln x)ln(ex) = eln(ex) ln(ln x) 1.
xe

9) x ln x + 0, et donc
x1

xx 1 = ex ln x 1

x1+

x ln x

x1+

1 (x 1) = x 1.

Ensuite,

Finalement,



p
ln 1 x2 1

p
p
+ x2 1 = (x 1)(x + 1)

x1

xx 1

ln(1 x2 1)
10)
ln(ch x 1)

x+

x1

x1

p
2(x 1).

x1
1
p
= x 1 + 0.
x1
2(x 1)
2

ln(ch x)

x+

ln

ex
2

= x ln 2

x+

x,

et donc
x ln (ch x 1)
x2 + 1

x+

xx
= 1.
x2

11)

http ://www.maths-france.fr

c Jean-Louis Rouget, 2014. Tous droits rservs.



ln x x2 + x ln x = x + ln(1 x)

Ensuite,

sin x



3
x x6 +o(x3 ) ln x

x0+

x0+

= + ex ln x e

x0

3 ln x
+o(x3
6

x ln x x

x0

(sin x) x

sin x

= + xx e

x3
6

+o(x3 )

x0

ln x)

Donc


x2
x2
+ .
+ o x2
x0
2
2



2
x ln 1 x6 +o(x2 )

x ln x x6 +o(x3 )
(sin x)x = ex ln(sin x) = + e
x0



x3
3
x
+o x
= x 1
,
x0+
6

et

=+




x3 ln x
3
= x 1
+ o x ln x .
x0+
6
x



 3






x3
x3 ln x
x ln x
x
x
3
3
3
= x 1
x 1
= x
+o x
+ o x ln x
+ o x ln x
x0+
x0+
6
6
6
x

x3 ln x
x3 ln x
=
,
6
6

+1

x0

et finalement
(sin x)x xsin x
ln (x x2 ) + x ln x

x3 ln x/6
x ln x
=
x2 /2
3

x0

0.

x0+

12)


 

1
1
1
= ln x + + o
,
ln(x + 1) = ln x + ln 1 +
x x+
x
x
puis
ln(x + 1)
ln x

1
= 1+
+o
x+
x ln x

1
x ln x

Ensuite,


x ln
Donc,

ln(x + 1)
ln x

x

ln(x + 1)
ln x

x+

x+


x ln 1 +

1
+o
x ln x

e0 = 1.

1
x ln x



x+

1
+o
ln x

1
ln x

x+

0.

1
13) Quand x tend vers ,
2
2

Arcsin x +
Arcsin x
Arcsin x
+
Arcsin x 4
1
16 = 1
4
4
4
4
4

=
1
1
1
1
1
2x2 1
2
x 1 2
4 2 x 1
2
+
x+
x
x
2
2
2
2
2
2



1
1

(Arcsin)

= r
= .
4
x 1 4 2
2
4 2
1
2
1
2

(Arcsin x)2

14)

x ln

cos a +
cos a

1
x

!





 
 
1
tan a
tan a
1
1
1
= x ln cos tan a sin
= x ln 1
= x
+o
+o
x+
x
x x+
x
x
x
x
=

x+

http ://www.maths-france.fr

tan a + o(1),
3

c Jean-Louis Rouget, 2014. Tous droits rservs.


et donc lim

x+

cos a +
cos a

1
x

 !x

= e tan a .

Exercice no 2
1)
2
3


1
= 1 + x2 + x3 + x2 + x3 + x2 + x3 + o x7
1 x2 x3 x0

= 1 + x2 + x3 + x4 + 2x5 + 2x6 + 3x7 + o x7 .
x0

2)
1
=
cos x x0

1
2

= 1+

x6
x2 x4

+
2
24 720

x2 x4

2
24

2

x2
2

3

+ o x7

x0
x
x
x
+

+ o (x7 )
2
24 720






1
1
1
1
1
1
5
61 6
x2
6
4
+x
+ o x7 = 1 + x2 + x4 +
+x +

+
x + o x7 .
= 1+
x0
x0
2
24 4
720 24 8
2
24
720

3) Remarques initiales.
i h
i h
 x 
x
a) Pour x ,
\ {0}, on a 0 <
est dfinie sur ,
\ {0}
< 1 et donc la fonction x 7 Arccos
2 2
tan x
tan x
2 2
(qui est un voisinage point de 0).
 x 
x
b)
1 et donc Arccos
= o(1) (dveloppement limit lordre 0).
tan x x0
tan x x0
c) La fonction x 7 Arccos x nest pas drivable en 1 et nadmet donc pas en 1 de dveloppement limit dordre
suprieur ou gal 1 (et priori, cest mal parti).
d) La fonction propose est paire et, si elle admet en 0 un dveloppement limit dordre 3, sa partie rgulire
ne contient que des exposants pairs.

Recherche dun quivalent simple de Arccos x en 1 gauche.


Quand x tend vers 1 par valeurs infrieures, Arccos x 0 et donc,
p

p
2 1 x.
Arccos x sin(Arccos x) = 1 x2 = (1 + x)(1 x)
x0

x0

r

x
en 0. Daprs ce qui prcde,
tan x
s
r
r


r

2 r
r
|x|
x
x
x
tan x x
x3 /3
sin Arccos
= 1

= .
=
Arccos
tan x x0
tan x
tan x
tan x x0
x
3

r
x
nest pas drivable en 0 (mais est drivable droite et gauche) et nadmet
Ainsi, la fonction x 7 Arccos
tan x
donc pas de dveloppement limit dordre suprieur ou gal 1 (mais admet ventuellement des dveloppements limits
gauche et droite pour lesquels la remarque initiale sur la parit des exposants ne tient plus).
r

x
x
Dterminons un quivalent simple de f(x) = Arccos
quand x tend vers 0 par valeurs suprieures.
tan x
3
Dterminons alors un quivalent simple de Arccos

Arccos

r

x
tan x



r

x
x

sin Arccos
x0+
tan x
3







r

r
x
x
x
x
cos
sin
cos Arccos
= sin Arccos
tan x
tan x
3
3
r


 r

x
x
tan x x
x
=
cos
sin

= g(x)
tan x
tan
x
3
3

Maintenant,

http ://www.maths-france.fr

c Jean-Louis Rouget, 2014. Tous droits rservs.


1/2

x3 2x5
5
 3


3 + 15 + o x

 1/2
x
tan x x
1
x2
2x5
5
2

=
1

=+
+
+
o
x
+
o
x

tan x x0
3
15
3
x
x3
+ o (x3 )
x+
3



 3
 1/2
 1/2
x
x5
x2
x
1
=+
+
+ o x5
+ o x2
1+
=+
x0
x0
45
15
x 3
3

x
x3
= + + + o(x3 ),
x0
3 30 3

et donc,
r

tan x x
cos
tan x

Ensuite,
r

x
sin
tan x

et finalement,
g(x)
Ainsi,

x0+






x
x2
x
x3
2x3
2
3
+ +o x
= + + o x3 .
+o x
1
x0
6
3 30 3
3 15 3










 1/2 x

x2
x2
x
x3
x3
3
3
2
2
+o x
+o x
1
1+
=
=
+o x
+o x
x0+
x0+
3
6
3 18 3
3 18 3

x
2x3
= + o x3 ,
x0+
3 9 3
=+

x0

 




4x3
x
4x3
2x3
2x3
x
+ o x3 + o x3
= + + o x3
+ .
x0 45 3
x0 45 3
3 15 3
3 9 3
Arccos

f tant paire, on en dduit que

r

x
tan x

=+

x0


4x3
+ o x3 .
45 3



|x|
x
4|x|3
= + + o x3 .
Arccos
tan x x0 3 45 3

4) La fonction x 7 tan x est trois fois drivable en et admet donc en un dveloppement limit dordre 3 savoir son
4
4
dveloppement de Taylor-Young.


tan = 1 puis (tan)


= 1 + tan2 = 2. Ensuite, pour tout x non dans + Z, (tan) (x) = 2 tan x(1 + tan2 x) et
4
4
4
2

(tan) ( ) = 4. Enfin, pour tout x non dans + Z


4
2
r

(tan)(3) (x) = 2(1 + tan2 x)2 + 4 tan2 x(1 + tan2 x),


et (tan)(3)

5)


4

= 16. Finalement,



2 8 
3
3

+2 x
x
.
+
+o x
tan x = 1 + 2 x
4
4
3
4
4
x/4


x2 x4 1
+

2
24 2




1
1
1
x2 x4
4
=
ln(ch x) = 2 ln 1 +
+
+o x
x0 x2
x0 x
x2
2
24

x2
2

2

et donc

1/x2

(ch x)

= e

2
x2
1
2 12 +o(x )

2
1/2 x
12 +o(x )

= e

x0

x0

6)


tan3 x cos x2 1 =

x0

http ://www.maths-france.fr

x + o x2

3

e
=

x0

+o x

!

4

x0


1 x2

+ o x2 ,
2 12


e 2
x + o x2 .
12

 4
 4






x
x
x7
+ o x5
+ o x5
= x3 + o x4
= + o x8 .
x0
x0
2
2
2
5

c Jean-Louis Rouget, 2014. Tous droits rservs.


7) On pose h = x 1 ou encore x = 1 + h, de sorte que x tend vers 1 si et seulement si h tend vers 0.


ln(1 + x)
= ln(2 + h)(1 + h)2
x2
 




(2)(3) 2 (2)(3)(4) 3
h
1 2h +
ln 2 + ln 1 +
=
h +
h + o h3
h0
2
2
6


2
3


h h
h
ln 2 +
=
1 2h + 3h2 4h3 + o h3
+
+ o h3
h0
2
8
24







43
9
1
h2 + 4 ln 2 +
h3 + o h3 .
2 ln 2 h + 3 ln 2
= ln 2 +
h0
2
8
24







ln(1 + x)
43
9
1
2
Donc,
(x 1) + 4 ln 2 +
(x 1)3 + o (x 1)3 .
= ln 2 +
2 ln 2 (x 1) + 3 ln 2
x1
x2
2
8
24
8) Pour x rel, posons f(x) = Arctan(cos x). f est drivable sur R, et pour x rel, f (x) =

sin x
. Puis,
1 + cos2 x



x3
x3



x
+ o x4
+ o x4
x

 1
x3
x2
1
4
3
6
6
f (x) =
x

+
o
x
+
o
x
=


2 x0 2 x2 + o (x3 ) x0 2
x0
6
2
x2
1+ 1
+ o (x3 )
2



3



1
x
x2
x x3
=
x
1+
=
+ o x4
+ o x3
+ o x4 .
x0
x0
2
6
2
2
6

Donc, f admet un dveloppement limit dordre 4 en 0 et on sait que f admet en 0 un dveloppement limit dordre 5
obtenu par intgration.
x2 x4
x2 x4
Arctan(cos x) = Arctan(cos 0)

+ o(x5 ) =

+ o(x5 ).
x0
x0 4
4
24
4
24
r
x+1
. f est drivable sur ] 1, +[ et pour x > 1,
9) Pour x > 1, posons f(x) = Arctan
x+2
1
1
1
1
p

r
=
x
+
1
(x + 2)2
x+1
2(2x + 3) (1 + x)(2 + x)
1+
2
x+2
x+2
1


x 1/2
2x
1

(1 + x)1/2 1 +
.
1+
=
3
2
23 2

f (x) =

Par suite,




x
x
1
2x

+ o(x) 1 + o(x) 1 + o(x)


f (x) =
1
x0 6 2
3
2
4






1
1
17x
2 1 1

=
x + o(x) =
+ +
+ o(x) .
1
1
x0 6 2
x0 6 2
3 2 4
12

f admet donc en 0 un dveloppement limit dordre 1 et on sait alors que f admet en 0 un dveloppement limit dordre
2 obtenu par intgration.
r

x+1
1
1
17
x2 + o x2 .
Arctan
= Arctan + x
x + 2 x0
2 6 2
144 2
10)





1
1
2 1/2

= 1x
= 1+
x2 +
2
x0
2
1x

5 6
1 2 3 4
= 1 + x + x + x + o x7 .
x0
2
8
16
http ://www.maths-france.fr

 



 
3
3
5
1
1

2
3

2
2
2
2
2
x2 +
x2 + o x7
2
6

c Jean-Louis Rouget, 2014. Tous droits rservs.


Donc, Arcsin x = x +
x0

Finalement,

x3 3x5 5x7
+
+
+ o(x8 ). Ensuite,
6
40
112



 2
x2 3x4
1
5x6
1
1
1
7
1
+
=
+
+
+
o
x
=

2 x0 x2
6
40
112
Arcsin2 x x0 x2
x2 3x4 5x6
1+
+
+
+ o (x7 )
6
40
112
!

 2
2
 2
 2 3
4

x
1
x
x
3x
5x6
3x4
7
+3
12
4
+o x
=
+
+
+
x0 x2
6
40
112
6
40
6





1
5
1
3
1
3
1
2
=
x + +
x4 + o x5
+ +

x0 x2
3
20 12
56 40 54

1
1 x2 31x4
= 2

+ o x5 .
x0 x
3 15
945

11) Pour x rel, posons f(x) =


1
1
1 x2 31x4

+
+
+ o x5 .
=
2
2
x
945
Arcsin x x0 3 15

. f est continue sur R et admet donc des primitives sur R. Soit F la primitive de f
Z x2
1

qui sannule en 0 puis, pour x rel, soit g(x) =


dt.
1
+ t4
x

g est dfinie sur R et, pour x rel g(x) = F x2 F(x). g est drivable sur R et, pour tout rel x,
1 + x4



1
2x

,
g (x) = 2xF x2 F (x) = 2xf x2 f(x) =
8
1+x
1 + x4

puis,

 





1 4 3 8
1
3
1 8
8
9
1 x + x +o x
= 1 + 2x + x4 x8 x9 + o x9 .
g (x) = 2x 1 x + o x
x0
x0
2
2
8
2
8

Ainsi g admet un dveloppement limit dordre 9 en 0 et on sait que g admet un dveloppement limit dordre 10 en 0
obtenu par intgration. En tenant compte de g(0) = 0, on obtient
g(x) = x + x2 +
x0

12)
ln

99
X
xk
k!

k=0


1 5
1
1
x x9 x10 + o x10 .
10
24
10





100


x100
x
x
x x
100
100
= ln e
= ln (e ) + ln 1 e
+o x
+o x
x0
x0
(100)!
(100)!




x100
x100
100
= x + ln 1
= x
+o x
+ o x100 .
x0
x0
(100)!
(100)!

13) Posons h = x ou encore x = + h de sorte que x tend vers si et seulement si h tend vers 0.

puis,

q
q
p
3
3
4 (3 + x3 ) = 3 4 (3 + ( + h)3 ) = 83 + 122 h + 12h2 + 4h3

1/3
3h 3h2
h3
= 2 1 +
+ 2+ 3
2 2
2
!


 3

2

1 3h 3h2
5 3h
h3
1 3h 3h2
3

+
+o h
+
+
+
= 2 1 +
h0
3 2 22 23
9 2 22
81 2





1
1
1
5
1
2
3
= 2 + h + h
+h
+ o h3
2+

2
2
h0
2
3

12

h2
h3
= 2 + h +
+ o h3 ,

h0
2 42

http ://www.maths-france.fr

c Jean-Louis Rouget, 2014. Tous droits rservs.


q




h2
h3
3
4 (3 + x3 )
= tan h +
2 + o h3
h0
2 4




2


1
h2
h3
1
1
h
2 + h3 + o h3 = h +
+
2 h3 + o h3 .
h+
=
h0
2 4
3
2
3 4


p

1
1
1
Finalement, tan( 3 4(3 + x3 )) = (x ) +
(x )2 +
2 (x )3 + o (x )3 .
x
2
3 4
tan

Exercice no 3

Puisque a > 0, b > 0 et que pour tout rel x,

ax +bx
1
a x + bx
> 0, f est dfinie sur R , et pour x 6= 0, f(x) = e x ln( 2 ) .
2

Etude en 0.

ln









x
1 x
x2  2
1 x ln a
2
2
x ln b
x
= ln 1 + (ln a + ln b) +
ln a + ln b + o x
e
+e
(a + b ) = ln
x0
2
2
2
4
!

2



ln2 a + ln2 b
ln2 a + ln2 b 1 
= ln 1 + x ln ab + x2
x ln ab + o x2
= x ln( ab) + x2
+ o x2

x0
x0
4
4
2





1
x2 2 a
= x ln ab +
ln2 a 2 ln a ln b + ln2 b x2 + o x2 = x ln ab +
ln
+ o x2 .
x0
x0
8
8
b

Ensuite,

1/x





x
x
a
a
ab 1 + ln2 + o(x) .
= exp ln ab + ln2 + o(x) =
x0
x0
8
b
8
b

Ainsi,
f se prolonge par continuit en 0 en posant f(0) = ab. Le prolongement obtenu est drivable en 0 et f (0) =

ab 2 a
ln
(> 0).
8
b
Etude en +.



 a x 
1
1
a
1
1 x
x
ln (bx ) ln 2 + ln 1 +
=
ln
(a + b )
=
(x ln b + o(x)) (car 0 < < 1)
x+
x+
x
2
x
b
x
b


a x + bx
2

x+

ln b + o(1).

et lim f(x) = b (= Max{a, b}).


x+

Etude en . Pour tout rel x,


f(x) =

ax + bx
2

1/x

a x + bx
2ax bx

1/x

ab
,
f(x)

et donc,
lim f(x) = lim f(X) = lim

X+

X+

ab
ab
=
=a
f(X)
b

(= Min{a, b}).

Drive et variations. f est drivable sur ] , 0[]0, +[ en vertu de thormes gnraux (et aussi en 0 daprs ltude
faite plus haut), et pour x 6= 0 (puisque f > 0 sur R ),


 x

 x
1 ax ln a + bx ln b
1
a + bx
1
a + bx
f (x)
+
(x) = 2 ln
.
= (ln f) (x) =
ln
f(x)
x
2
x
2
x
a x + bx
f a le mme signe que (ln f) qui, elle-mme, a le mme signe que la fonction g dfinie sur R par

 x
ax ln a + bx ln b
a + bx
+x
.
x R, g(x) = ln
2
a x + bx
g est drivable sur R et, pour tout rel x,

http ://www.maths-france.fr

c Jean-Louis Rouget, 2014. Tous droits rservs.


(ax ln2 a + bx ln2 b)(ax + bx ) (ax ln a + bx ln b)2


ax ln a + bx ln b ax ln a + bx ln b
+
+
x
a x + bx
a x + bx
(ax + bx )2
(ab)x (ln a ln b)2
=x
.
(ax + bx )2

g (x) =

g est donc strictement ngative sur ] , 0[ et strictement positive sur ]0, +[. Par suite, g est strictement dcroissante
sur ] , 0] et strictement croissante sur [0, +[. g admet donc un minimum global strict en 0 gal 0 et on en dduit
que g est strictement positive sur R . De mme, f est strictement positive sur R . En tant compte de ltude en 0, on a
montr que f est drivable sur R et que f est strictement positive sur R. f est donc strictement croissante sur R.
Le graphe de f a lallure suivante :
b

ab

On peut noter que les ingalits f() < f(1) < f(0) < f(1) < f(+) fournissent :
a<

Exercice no 4

1
1
2

1
1
+
a b

<

a+b
< b.
ab <
2

Quand x tend vers +,



1/2
p
3
x2 3 = x 1 2
x

et,

p
3
8x3 + 7x2 + 1


 
 
3
3
1
1
,
=
x

x 1 2 +o
+
o
x+
x+
2x
x2
2x
x
=


 1/3
7
1
2x 1 +
+o
x+
8x
x2
=


 
 
7
7
49
49
1
1
.
=
2x+
2x 1 +
+
o

+o
x+
x+
24x 576x2
x2
12 288x
x
=

Donc,

7
389
f(x) = x

+o
x+
12 288x
Puisque f(x)
y = x

7
12

7
12

x+

 
1
.
x

o(1), a courbe reprsentative de f admet en + une droite asymptote dquation



 

389
389
7
7
1
=

est strictement ngative
+o
, lexpression f(x) x
Puisque f(x) x
12 x+ 288x
x x+ 288x
12
au voisinage de + et donc la courbe reprsentative de f est au-dessous de cette droite au voisinage de +.


Exercice no 5
f est de classe C sur son domaine R \ {1, 1} en tant que fraction rationnelle et en particulier admet un dveloppement
limit tout ordre en 0. Pour tout entier naturel n, on a
f(x) = x + x3 + ... + x2n+1 + o(x2n+1 ),
x0

http ://www.maths-france.fr

c Jean-Louis Rouget, 2014. Tous droits rservs.


et donc,
n N, f(2n) (0) = 0 et f(2n+1) (0) = (2n + 1)!.
Ensuite, pour x
/ {1, 1}, et n entier naturel donn,
f

(n)

1
(x) =
2

1
1

1x 1+x

(n)

n!
(x) =
2

1
(1)n

(1 x)n+1 (1 + x)n+1

Exercice no 6
1)
p
x2 + 3x + 5 x + 1

et,

x x = 2x,

p
(x2 + 3x + 5) (x 1)2
5x + 4
x2 + 3x + 5 x + 1 =
=
2
2
x + 3x + 5 + x 1
x + 3x + 5 + x 1

x+

5
5x
= .
x+x
2

2) 3x2 6x 6x.
x0

3x2 6x

x+

3x2 .

Quand x tend vers 1, 3x2 6x tend vers 3 6= 0 et donc, 3x2 6x 3.


x1

Enfin, 3x2 6x = 3x(x 2) 6(x 2).


x2

3)









x3
x2
x x2 ln x
= x x2 ln x + x x2 ln 1
+ o x3
+ o x2
x0
x0
6
6

2
2
= x ln x x ln x + o x ln x .


x x2 ln(sin x) =

x0

Ensuite,

sin x ln x x






x3
x3
3
3
(ln x x + o(x))
+ o(x ) (ln x + ln(1 x)) = x
+o x
= x
x0
x0
6
6

= x ln x + o x2 ln x .
x0

Donc,


 2

2
2
2
(sin x)xx (x x2 )sin x = ex ln x ex ln x+o(x ln x) eo(x ln x) = ex ln x 1 x2 ln x 1 + o x2 ln x

= (1 + o(1)) x2 ln x + o x2 ln x
x2 ln x.
x0

4) th x =

1 e2x
1 + e2x

= = 1 e2x

x+

1 e2x + o e2x



= 1 2e2x + o(e2x ), et donc

th x ln x = (1 2e2x + o(e2x )) ln x = ln x + o(1).


Par suite,
xth x

x+

eln x = x.

5) Tentative lordre 3.

et,








x3
1
x3
x3
+ o x3
+ o x3 ,
+ (x)3 + o x3 = x +
= x
tan(sin x) = tan x
x0
x0
x0
6
6
3
6







x3
1
x3
x3
3
(x)3 + o x3 = x +
= x+
+o x
+ o x3 .
sin(tan x) = sin x +
x0
x0
x0
3
3
6
6

Donc, tan(sin x) sin(tan x) = o(x3 ). Lordre 3 est insuffisant pour obtenir un quivalent.
x0

http ://www.maths-france.fr

10

c Jean-Louis Rouget, 2014. Tous droits rservs.


Tentative lordre 5.


3





1
x3
x3
2
x5
x5
x3
5
+
x
= x
+ (x)5 + o x5
+
+o x
+
tan(sin x) = tan x
x0
x0
6
120
6
120
3
6
15




x3 x5
1
2
1
x3
x5 + o x5 = x +
+
+

+ o x5 ,
= x+
x0
6
120 6 15
6
40

et,




3




x3 2x5
1
x3
1
x3 2x5

x+
= x+
+
sin(tan x) = sin x +
+
+ o x5
+
(x)5 + o x5
x0
x0
3
15
3
15
6
3
120


3
3
5
x
x
1
1
x
2
= x+
x5 + o(x5 ) = x +
+
+

+ o(x5 ).
x0
6
15 6 120
6
40
Donc, tan(sin x) sin(tan x) = o(x5 ). Lordre 5 est insuffisant pour obtenir un quivalent. Le contact entre les courbes
x0

reprsentatives des fonctions x 7 sin(tan x) et x 7 tan(sin x) est trs fort.


Tentative lordre 7.




x5
x7
x3
7
+

+o x
tan(sin x) = tan x
x0
6
120 5040


3

5


1
x3
x3
2
17 7
x5
x7
x5
x3
+
x
x
+
+
+

+
x + o x7
= x
x0
6
120 5040
3
6
120
15
6
315


 



x3 x5
1
2
5
17
1
1
1
= x+
3
+

+
x7 + o x7

+
+
+3
x0
6
40
5040 3
120
36
15
6
315


x3 x5
1
1
1
1
17
x7 + o(x7 ),
= x+

+
+
+
+
x0
6
40
5040 120 36 9 315
et,


17 7
x3 2x5
7
+
+
x + o(x )
sin(tan x) = sin x +
x0
3
15
315



3

5

x3 2x5 17x7
1
x3 2x5
x3
1
1
= x+

x+
x+
+

+
+
+
(x)7 + o x7
x0
3
15
315
6
3
15
120
3
5040





2
1
1
1
5
1
17
x3 x5
3
+
x7 + o x7

+3

= x+
x0
6
40
315 6
15
9
120 3 5040



x3 x5
1
1
1
1
17
= x+
x7 + o x7 .

x0
6
40
315 15 18 72 5040

Finalement,

1
1
1
1
1
1
+
+
+

tan(sin x) sin(tan x) =
x0
120 36 9 15 18 72

x7
=
+ o x7
x0 30
x7
.

x0 30

 (3 + 10 40 + 24 + 20 5)x7

x7 + o x7 =
+ o x7
360

Exercice no 7
Pour n > 5, on a
un = 1 +

n5
X
1
1
1
1
1
+
+
+
+
.
n n(n 1) n(n 1)(n 2) n(n 1)(n 2)(n 3)
n(n 1)...(k + 1)
k=0

Ensuite,

http ://www.maths-france.fr

11

c Jean-Louis Rouget, 2014. Tous droits rservs.


0 6 n3

n5
X
k=0

avec n3 (n 4)

1
1
6 n3 (n 4)
n(n 1)...(k + 1)
n(n 1)(n 2)(n 3)(n 4)

1
n(n 1)(n 2)(n 3)(n 4)

n+

1
n

n+

n+

1
n

0.

n5
X
1
Donc, daprs le thorme des gendarmes,
n(n 1)...(k + 1)
k=0
 
1
o
. Il reste
n3

n+

n+

0,


1
1
. De mme
3
n
n(n 1)(n 2)(n 3)

n+

1



 
 
1
1
1
1
1
1
1
1
1
1
un = 1 + + 2 1
+ 3 +o
= 1+ + 2 1+
+ 3 +o
n+
n n
n
n
n3 n+
n n
n
n
n3
 
1
2
1
1
= 1+ + 2 + 3 +o
.
n+
n n
n
n3
Exercice no 8
1)
!



1
2
3
4


x2 x3 x4
1
x
1
x5
x
x
x
x+
1+ +
1
+
+
+
+ o x5 2 = 2
+
+
+ o x4
2
6
24 120
x x0 x
2
6
24 120
!
 
2 
3  


1
x x2 x3
x x2 x3
x4
x x2
x 4
4
=
+

+
+o x
+
+
+
+
+
+
x0 x2
2
6
24 120
2
6
24
2
6
2











1
1
1
x
1 1
1 1
1
1
1
1
3
4
4
2
+x +
+x
+
+o x
+x +
+
+
=
x0 x2
2
6 4
24 6 8
120
36 24
8 16

1
x2
1

+ o x2 .
= +
x0
2x 12 720

1
1
1
=

x(ex 1) x2 x0 x

2)




1
(x + 1) ln x
x ln(x + 1) (x + 1) ln x = x ln x + ln 1 +
x

 
1
1
1
1
1
= ln x + x
2 + 3 4 +o
x+
x 2x
3x
4x
x4
 
1
1
1
1
= ln x + 1
.
+ 2 3 +o
x+
2x 3x
4x
x3
Exercice no 9
1)
fn (a)

a
n ln(1+ n
)

n+

n+

1
a a
2n +o( n )

En remplaant a par b ou a + b, on obtient

fn (a + b) fn (a)fn (b)

n+

a2
1
+o
2n

 
1
.
n




 

 
(a + b)2
a2
b2
1
ea+b 1
ea 1
eb 1
+o
n+
2n
2n
2n
n
=

= ea+b

1
abea+b
1
(a + b)2 + a2 + b2
+ o( ) =
+ o( ).
2n
n
n
n

abea+b
(puisque ab 6= 0).
n+
n


2

 


2
1
a3
a2
a2
1
1
a3
a+ a a
2n + 3n2 +o( n2 )
= 1+

+o
, et donc
2) ea fn (a) = e
+ 2 +
n+
n+
2n 3n
2
2n
n2

 3
a2
1
a4
a
ea fn (a) 1 +

+
(car a > 0).
2n n+ 3
8 n2
Donc, fn (a + b) fn (a)fn (b)

http ://www.maths-france.fr

12

c Jean-Louis Rouget, 2014. Tous droits rservs.


Exercice no 10
i i
i i
i i
h i
= ]0, 1] 0, . Donc, puisque u0 0, , on en dduit par
1) Pour x 0, , posons f(x) = sin x. On a f 0,
2
2
2
2
i i
rcurrence que n N, un 0, .
2
h i
h i
h i
, posons g(x) = f(x) x. g est drivable sur 0,
et pour x 0, , g (x) = cos x 1. g est strictement
Pour x 0,
2h
2
i i
i2 i
i
et g est donc strictement dcroissante sur 0, . Par suite, x 0, , sin x < x et de plus, pour
ngative sur 0,
2
2
2
h i
x 0, , sin x = x x = 0.
2

u est valeurs dans ]0, ] et donc n N, un+1 = sin(un ) < un . La suite u est donc strictement dcroissante et, tant
2
h i
h i
qui vrifie (f tant continue sur le segment 0,
et donc en ), f() =
minore par 0, converge vers un rel de 0,
2
2
ou encore = 0.
En rsum, la suite u est strictement positive, strictement dcroissante et converge vers 0.
2) Soit un rel quelconque. Puisque la suite u tend vers 0 , on a

u
n+1

u
n

= (sin un )




u3n
3
un
u
+ o un
n
6
!






u2n
u2n
2
2

1
= u

+ o un
+
o
u
= un
1
n
n+ n
n+
6
6

u
=
n n+

n+

Pour = 2, on a donc

Daprs le lemme de Csaro,

n1 
1 X
1
n
u2k+1
k=0

+2

un
+2
+ o un
6

1
1
1
=
+ o(1).
2
un n+ 3
u2n+1



1
1
1
1
1
1
1
2 = + o(1) ou encore
= + o(1) ou enfin, 2 =

3
n u2n
3
un
uk
u20

n
1
n
+ 2 + o(n)
. Par suite, puisque la suite u est strictement positive,
n+ 3
3
u0
r
3
un
.
n+
n
Exercice no 11
un+1
Il est immdiat par rcurrence que n N, un > 0. Donc, n N,
= eun < 1 et, puisque la suite u est
un
strictement positive, un+1 < un . La suite
u est strictement dcroissante, minore par 0 et donc converge vers un rel

vrifiant = e ou encore 1 e = 0 ou encore = 0.

u est strictement positive, strictement dcroissante et converge vers 0.


Soit un rel quelconque. Puisque la suite u tend vers 0,

un
1)
u
n+1 un = un (e

Pour = 1, on obtient en particulier


1
un

n+

n+

1
un+1

1
+ o(n) n et donc un
n+
u0

n+

1
un

u
n (un + o(un ))

n+

n+

n+

+1
+1
un
+ o(un
).

1 + o(1). Puis, comme dans lexercice prcdent,

1
.
n

Exercice n 12
o

h
i

Pour n entier naturel donn, posons In = + n, + n .


2
2
Pour x In , posons f(x) = tan x x. f est drivable sur In et pour x dans In , f (x) = tan2 x. f est donc continue et
strictement croissante sur In et ralise donc une bijection de In sur f(In ) = R. En particulier, n N, !xn In / f(xn ) = 0
(ou encore tel que tan xn = xn ).
h
i

On a bien sr x0 = 0 puis pour n N , f(n) = n < 0 et donc, n N , xn n, + n . En particulier,


2
http ://www.maths-france.fr

13

c Jean-Louis Rouget, 2014. Tous droits rservs.


xn = n + O(1).
n+
h
i h

Posons alors yn = xn n. n N , yn 0, . De plus, tan(yn ) yn n = 0 et donc, puisque yn 0, ,


2
2

.
yn = Arctan(tan(yn )) = Arctan(yn + n) > Arctan(n)
n+ 2

Daprs le thorme des gendarmes, yn =


+ o(1) ou encore
n+ 2

xn = n + + o(1).
n+
2
i h

Posons maintenant zn = yn
= xn n . Daprs ce qui prcde, n N , zn , 0 et dautre part
2 
2
2


= n + + zn et donc cotan(zn ) n. Puisque zn tend vers 0, on en dduit


zn = o(1). Ensuite, tan zn +
n+
2
2

 n+
1
1
1
que
. Ainsi,
cotan(zn n) n, ou encore zn =
+o
n+
n+
zn n+
n
n
 
1
1

.
+o
xn = n +
n+
2
n
n
 

1
1
1
et que
= xn n +
. On sait que tn = o
Posons enfin tn = zn +
n+
n
2 n 
n


1

1
1
cotan tn
= n +
. Par suite,
+o
n n+
2 n
n


 1
 

1
1
1
1
1
1
1
=
1+
,
=
+o
2 +o
tan tn
n+
n
n
2n
n
n 2n
n2
i

puis,

et donc tn

n+


 
 
1
1
1
1
1
1
1
tn = Arctan
2 +o

+
o
=
,
n+
n+ n
n
n 2n
n2
2n2
n2
 
1
1
. Finalement,
+
o
2n2
n2
 
1
1
1

.
+ 2 +o
xn = n +
n+
2
n 2n
n2

Exercice no 13
1) Pour x > 0, posons f(x) = x + ln x. f est continue sur ]0, +[, strictement croissante sur ]0, +[ en tant que somme
de deux fonctions continues et strictement croissantes sur
 ]0, +[.

f ralise donc une bijection de ]0, +[ sur f(]0, +[) =
lim f(x), lim f(x) =] , +[. En particulier,
x0, x>0

x+

k R, !xk ]0, +[/ f(xk ) = k.




 
k
k
k
k
k
k
k
= + ln < k pour k suffisamment grand (car k
= ln
+ ln
+ daprs un thorme de
2) f
2
2
2
2
2
2
2 k+
 
k
croissances compares). Donc, pour k suffisamment grand, f
< f (xk ). Puisque f est strictement croissante sur ]0, +[,
2
k
on en dduit que xk > pour k suffisamment grand et donc que lim xk = +. Mais alors, k = xk + ln xk xk .
k+
k+
2
Ainsi,
xk

k+

k + o(k).

Posons yk = xk k. On a yk = o(k) et de plus yk + ln (yk + k) = 0 ce qui scrit :


yk = ln (k + yk )
Donc, xk

k+

k+

ln(k + o(k)) = ln k + ln(1 + o(1)) = ln k + o(1).

k ln k + o(1).

Posons zk = yk + ln k = xk k + ln k. Alors, zk

http ://www.maths-france.fr

k+

o(1) et ln k + zk = ln (k ln k + zk ). Par suite,

14

c Jean-Louis Rouget, 2014. Tous droits rservs.


ln k
+o
zk = ln k ln(k ln k + o(1)) = ln 1
k+
k+
k

ln k
k



k+

ln k
+o
k

ln k
k

Finalement,
xk

k+

k ln k +

ln k
+o
k

ln k
k

Exercice no 14


1
1
= o x2 . Donc, en tenant compte de f(0) = 1, f(x) = 1 + x + x2 +
1) x3 sin 2 = O x3 et en particulier x3 sin
x0
x x0
 x x0
o x2 . f admet en 0 un dveloppement limit dordre 2.

2) f(x) = 1 + x + o(x). Donc, f admet en 0 un dveloppement limit dordre 1. On en dduit que f est continue et
x0

drivable en 0 avec f(0) = f (0) = 1. f est dautre part drivable sur R en vertu de thormes gnraux (et donc sur R)
1
1
et pour x 6= 0, f (x) = 1 + 2x + 3x2 sin 2 2 cos 2 .
x
x

1
1
1 + 2x + 3x2 sin 2 2 cos 2 si x 6= 0
.
x R, f (x) =
x
x
1 si x = 0




1
1
tendent vers 0
r
3) f est dfinie sur R mais na pas de limite en 0. En effet, les deux suites
et

2n
+ 2n
2

2n

1
tend vers 1.
tend vers 1 et f

r
+ 2n
2
f nadmet donc mme pas un dveloppement limit dordre 0 en 0.
quand n tend vers + mais f

Exercice no 15

x2 3x4
1

+
+ o x4 , et donc, en tenant compte de Arcsin(0) = 0, on obtient par intgration
= 1+
2
2
8
1 x x0

x3 3x5
Arcsin x = x +
+
+ o x5 .
x0
6
40
Puis,
1
1
=
Arcsin x x0 x

et donc,

1
x2 3x4
1+
+
+ o (x4 )
6
40


 2



 1


x2 3x4
x4
1 x 17x3
1
x
3x4
1
1+
1
+
=
=
+
+ o x4
+
+ o x4

+ o x3 ,
=
x0 x
x0 x
x0 x
6
40
6
40
36
6
360

1
1
x 17x3

=
+
+ o x3 .
x Arcsin x x0 6
360

La fonction f propose admet en 0 un dveloppement limit dordre 0 savoir f(x) = o(1). f se prolonge donc par
x0

continuit en 0 en posant f(0) = 0.


x
+ o(x). Donc f est
Le prolongement, encore not f, admet en 0 un dveloppement limit dordre 1 savoir f(x) =
x0 6
1
drivable en 0 et f (0) = .
6
x
La courbe reprsentative de f admet lorigine une tangente dquation y = .
6
x
17x3
Le signe de la diffrence f(x) est, au voisinage de 0, le signe de
. La courbe reprsentative de f admet donc
6
360
x
lorigine une tangente dinflexion dquation y = .
6

http ://www.maths-france.fr

15

c Jean-Louis Rouget, 2014. Tous droits rservs.


Exercice no 16
1) Arccos x

x1

o(1) (dveloppement limit lordre 0). Mais la fonction x 7 Arccos x nest pas drivable en 1 et

nadmet donc pas en 1 un dveloppement limit dordre 1.


2) Puisque Arccos x

x1

o(1),

Arccos x

sin(Arccos x) =

x1

Exercice no 17

p
p
1 x2 = (1 + x)(1 x)

x1


2 1 x.

1) Quand x tend vers 0,


1
1 1
1 1
1
1
+
=
+
(1 x)2 (1 + x)
4 1 x 2 (1 x)2
41+x
!
n
n
n
X
X
1 X k
k
k k
+ o (xn )
=
x +2
(k + 1)x +
(1) x
x0 4
k=0

k=0

k=0

n
X
2k + 3 + (1)k k
=
x + o (xn ) .
x0
4
k=0

2) On a aussi,
!
! n
n
X
X
1
1
=
=
x2q + o (xn )
xp
(1 x)2 (1 + x) x0 (1 x)(1 x2 )
k=0
k=0

n
n
X
X
X

=
1 xk + o (xn ) =
ak xk + o (xn ) .
x0

k=0

x0

p+2q=k

Par unicit des coefficients dun dveloppement limit, on a donc ak =


payer k euros en pices de 1 et 2 euros).

http ://www.maths-france.fr

16

k=0

2k + 3 + (1)k
(ak est le nombre de faons de
4

c Jean-Louis Rouget, 2014. Tous droits rservs.


Planche no 24. Structures


* trs facile ** facile *** difficult moyenne **** difficile
I : Incontournable T : pour travailler et mmoriser le cours
Exercice no 1 (***T)
Sur R2 , on dfinit une loi par
((x, y), (x , y )) R2

2




, (x, y) (x , y ) = x + x , yex + y ex .


1) Montrer que R2 , est un groupe non ablien.

2) Trouver les application f drivables sur R telles que {(x, f(x)), x R} soit un sous-groupe de R2 , .
Exercice no 2 (***T)
Sur ] 1, 1[, on dfinit une loi par
2

(x, y) (] 1, 1[) , (x, y) (x , y ) =

x+y
.
1 + xy

Montrer que (] 1, 1[, ) est un groupe commutatif.


Exercice no 3 (*IT)
1) Montrer que U est un sous-groupe de (C , ).
2) Montrer que pour tout entier naturel non nul n, Un est un sous-groupe de (U, ).
Exercice no 4 (**T)
Sur E un ensemble.
1) Montrer que (P(E), ) est un groupe commutatif.
2) Montrer que (P(E), , ) est un anneau commutatif.
Exercice no 5 (**T)


Montrer que
a + b 2, (a, b) Q2 , +, est un corps commutatif.
Exercice no 6 (***I) (Sous groupes de (Z, +))
1) Soient a un entier relatif puis G = aZ. Montrer que G est un sous-groupe de (Z, +).
2) Rciproquement, montrer que tous les sous-groupes de (Z, +) sont de la forme aZ o a Z (considrer, sil existe,
a = MinG Z+ ).
Exercice no 7 (***I) (Sous groupes de (R, +))
1) Montrer que les sous groupes du groupe (R, +) sont soit de la forme aZ, a rel donn, soit denses dans R.
Indication : pour G sous-groupe donn de (R, +), non rduit {0}, considrer a = Inf (G]0; +[) puis envisager les deux
cas a = 0 et a > 0.
(Rappel : G est dense dans R si et seulement si : (x R, > 0, y G/ |y x| < ).

2) Application 1. Montrer que a + b 2, (a, b) Z2 est dense dans R.


3) Application 2 (groupe des priodes dune fonction).
a) Soit f une fonction dfinie sur R valeurs dans R. Montrer que lensemble des priodes de f est un sous groupe
de (R, +) (ce sous-groupe est rduit {0} si f nest pas priodique).

b) Montrer quune fonction continue sur R qui admet 1 et 2 pour priodes, est constante sur R.
c) Trouver une fonction dont le groupe des priodes est dense dans R mais nest pas gal R.

http ://www.maths-france.fr

c Jean-Louis Rouget, 2014. Tous droits rservs.


Planche no 24. Structures : corrig


Exercice no 1
1) est une loi interne dans R2 .
Soit (x, x , y, y , z, z ) R6 .








((x, y) (x , y )) (x , y ) = x + x , yex + y ex (x , y ) = x + x + x , yex + y ex ex + y exx





= x + x + x , yex +x + y ex+x + y exx ,


et

 




(x, y) ((x , y ) (x , y )) = (x, y) x + x , y ex + y ex = x + x + x , yex +x + y ex + y ex ex





= x + x + x , yex +x + y ex+x + y exx ,


3
Donc, pour tout ((x, y), (x , y ), (x , y )) R2 , ((x, y) (x , y )) (x , y ) = (x, y) ((x , y ) (x , y )).
Pour tout (x, y) R2 ,

et


(x, y) (0, 0) = x + 0, ye0 + 0ex = (x, y),

(0, 0) (x, y) = 0 + x, 0ex + ye0 = (x, y).

Donc, admet un lment neutre savoir (0, 0).

Pour tout (x, y) R2 ,


et


(x, y) (x, y) = x x, yex yex = (0, 0),
(x, y) (x, y) = (x + x, yex + yex ) = (0, 0).

Donc, tout lment (x, y) de R2 admet un symtrique pour dans R2 savoir (x, y).
(1, 0) (0, 1) = (1, e1) et (0, 1) (1, 0) = (1, e). Donc (1, 0) (0, 1) 6= (0, 1) (1, 0). La loi nest donc pas commutative.
On a montr que

R2 , est un groupe non ablien.
2) Soit f une fonction drivable sur R puis G = {(x, f(x), x R}. Soit (x, x ) R2 .

(x, f(x)) (x , f(x )) = (x + x , f(x)ex + f(x )ex ).




Si G est un sous-groupe de R2 , , on doit avoir f(x + x ) = f(x)ex + f(x )ex . Pour x = x = 0, on obtient f(0) = 0.
Ensuite, pour tout rel x et tout rel non nul h


1
f(x + h) f(x)
f(x) eh 1 + f(h)ex .
=
h
h
Puisque f est drivable sur R, quand h tend vers 0, on obtient
f (x) = f(x) + f (0)ex .
f (0) x
e + Kex . En drivant, on obtient pour tout rel x, f (x) =
2
f (0)
f (0) x
f (0) x
e + Kex et pour x = 0 on obtient K =
. Finalement, pour tout rel x, f(x) =
(e ex ) = f (0) sh(x).
2
2
2
On a montr que si f est solution, alors il existe un rel tel que pour fout rel x, f(x) = sh(x).

Donc, il existe un rel K tel que pour tout rel x, f(x) =

Rciproquement, supposons quil existe un rel tel que pour tout rel x, f(x) = sh(x). Alors,
http ://www.maths-france.fr

c Jean-Louis Rouget, 2014. Tous droits rservs.


(0, 0) = (0, f(0)) G.

pour tout (x, x ) R2 ,




 

 

ex ex ex + ex ex ex
f(x)ex + f(x )ex = sh(x)ex + sh(x )ex =
2

 x+x
xx
= sh(x + x )
e
e
=
2
= f(x + x ),

et donc (x, f(x)) (x , f(x )) G.


pour tout x R,
(x, f(x)) = (x, sh(x)) = (x, sh(x)) = (x, f(x)),
et donc (x, f(x)) G.



Ceci montre que G est un sous-groupe de R2 , . Finalement, G est un sous-groupe de R2 , si et seulement si il existe
un rel tel que pour tout rel x, f(x) = x.
Exercice no 2
Pour tout (x, y) ] 1, 1[2 , 1 + xy 6 1 |xy| > 0 et en particulier 1 + xy 6= 0. Donc, pour tout (x, y) ] 1, 1[2 , x y
existe. Ensuite,
1

x+y
1 x y + xy
(1 x)(1 y)
=
=
>0
1 + xy
1 + xy
1 + xy

et
x+y
1 + x + y + xy
(1 + x)(1 + y)
(1) =
=
> 0.
1 + xy
1 + xy
1 + xy
Donc, pour tout (x, y) ] 1, 1[2 , x y ] 1, 1[. Ainsi, est une loi interne dans ] 1, 1[.
Soit (x, y) ] 1, 1[2 .

xy=

x+y
x+y
=
= y x.
1 + xy
1 + yx

Donc, la loi est commutative.


Soit (x, y, z) ] 1, 1[3 .

x+y
+z
x + y + z + xyz
x+y
1 + xy
z=
.
=
(x y) z =
x
+
y
1 + xy
1 + xy + xz + yz
z
1+
1 + xy
En changeant les rles de x, y et z, on obtient
x (y z) = (y z) x =

x + y + z + xyz
,
1 + xy + xz + yz

et finalement (x y) z = x (y z). Donc, la loi est associative.

x+0
= x. Donc admet un lment neutre savoir 0.
1+x0
xx
= 0. Donc tout lment de ] 1, 1[ admet un symtrique pour
Pour tout x de ] 1, 1[, x ] 1, 1[ et x (x) =
1 x2
dans ] 1, 1[.
Pour tout x de ] 1, 1[, x 0 =

On a montr que (] 1, 1[, ) est un groupe commutatif.


Exercice no 3
1) 1 U car |1| = 1 et U C car 0
/ U.
z
|z|
z

Soient (z, z ) U2 . Alors = = 1. Donc U.
z
|z |
z

On a montr que U est un sous-groupe de (C , ).


http ://www.maths-france.fr

c Jean-Louis Rouget, 2014. Tous droits rservs.


2) Soit n > 2.
1 Un car 1n et Un U car pour tout z C, zn = 1 |z|n = 1 |z| = 1.
 z n
zn
z
=
= 1 et donc Un .
Soit (z, z ) U2n .
z
z n
z
On a montr que Un est un sous-groupe de (U, ).
Exercice no 4
1) P(E) nest pas vide car P(E).
est une loi interne dans P(E).
est commutative et associative (voir planche no 3, exercice no 2, 3) et 4))
A P(E), A = A et donc possde un lment neutre dans P(E) savoir .
A P(E), AA = et donc tout lment de P(E) possde un symtrique pour dans P(E) savoir lui-mme.
On a montr que (P(E), ) est un groupe commutatif.
2) (P(E), ) est un groupe commutatif.
est une loi interne dans P(E).
est commutative et associative.
A P(E), A E = A et donc possde un lment neutre dans P(E) savoir E.
3

Soit (A, B, C) (P(E)) .


(AB) C =
et





AB AB C= ABC ABC




(A C) (B C) = (A C) B C A C (B C)



= (A C) B C A C (B C)




= ABC ACC ABC BCC


= ABC ABC

et donc (AB) C = (A C) (B C). Ainsi, est distributive sur .


On a montr que (P(E), , ) est un anneau (commutatif et unitaire).
Exercice no 5

Posons G = a + b 2, (a, b) Q2 .
Montrons que G est un sous-groupe de (R, +).

G R et 0 = 0 + 0 2 G.
Soit (a, a , b, b ) Q2 .

 
a + b 2 a + b 2 = (a a ) + (b b ) 2

avec a a Q et b b Q.
On a montr que G est un sous-groupe de (R, +) et en particulier, (G, +) est un groupe commutatif.
Montrons que G \ {0} est un sous-groupe de (R , ).

Vrifions tout dabord que si (a, b) 6= (0, 0) alors a + b 2 6= 0. Soit (a, b) Q2 tel que a + b 2 = 0. Si b 6= 0, on en

a
a
/ Q et Q. Donc, pour tous rationnels a et b, si a + b 2 = 0 alors
dduit que 2 = . Ceci est impossible car 2
b
b

a = b = 0. Par contraposition, pour tous rationnels a et b, si (a, b) 6= (0, 0) alors a + b 2 6= 0.

1 = 1 + 0 2 G.

Soit
(a, a , b, b ) Q4 tel que (a, b) 6= (0, 0) et (a , b ) 6= (0, 0). Daprs la remarque initiale, a + b 2 6= 0 et
a b 2 6= 0.
http ://www.maths-france.fr

c Jean-Louis Rouget, 2014. Tous droits rservs.






a
+
b
2
a

b
2
aa 2bb
ba ab
a+b 2

 = 2
= 
+ 2
2.

2
a 2b
a 2b 2
a + b 2
a + b 2 a b 2

a+b 2
ba ab
aa 2bb
G.
2
sont
des
rationnels,
et
Puisque 2
a 2b 2
a 2 2b 2
a + b 2
On a montr que G \ {0} est un sous-groupe de (R , ) et en particulier, (G \ {0}, ) est un groupe commutatif.
Finalement, (G, +, ) est un corps commutatif.
Exercice no 6
1) Soient a Z puis G = aZ.
0 = 0 a aZ et aZ Z.
Soit (n, m) Z2 . an am = a(n m) aZ.
Donc, aZ est un sous-groupe de (Z, +).
2) Soit G un sous-groupe de (Z, +). Si G = {0}, alors G = 0Z et G est de la forme voulue.
On suppose dornavant que G est un sous-groupe de (Z, +) non rduit {0}. Il existe dans G un entier relatif non nul n0 .
Puisque G est un sous groupe de (Z, +), lentier n0 est aussi dans G et lun des deux entiers n0 ou n0 est strictement
positif. Soit alors A = G Z+ . A est une partie non vide de N. A admet donc un plus petit lment que lon note a. a est
par dfinition le plus petit entier strictement positif de G.
Montrons alors que G = aZ. Puisque a est dans G, G contient encore a + a = 2a, puis a + a + a = 3a et plus gnralement
tous les na, n N . G contient aussi les opposs de ces nombres, et puisque G contient galement 0 = 0 a, G contient
finalement tous les na, n Z. On a montr que aZ G.
Rciproquement, soit n un lment de G. La division euclidienne de n par a scrit n = aq + r o q et r sont deux entiers
relatifs tels que 0 6 r 6 a 1 (puisque a est un entier strictement positif, a 1 est un entier positif).
Or, n est dans G et qa est dans G. Donc, n qa est dans G J0, a 1K = {0} (par dfinition de a) puis n = qa aZ.
On a ainsi montr linclusion contraire et donc G = aZ.
Les sous-groupe de (Z, +) sont donc de la forme aZ o a Z.
Exercice no 7
1) Soit G un sous groupe non nul de (R, +) ({0} = 0Z est du type voulu).
Il existe dans G un rel non nul x0 . Puisque G est un sous groupe de (R, +), le rel x0 est aussi dans G et lun des deux
rels x0 ou x0 est strictement positif. Soit alors A = G]0, +[.
Daprs ce qui prcde, A est une partie non vide et minore (par 0) de R. A admet donc une borne infrieure que lon
note a.
1er cas. Si a = 0. Montrons dans ce cas que G est dense dans R (cest par exemple le cas de (Q, +)).
Soient x un rel et un rel strictement positif.
Puisque inf A = inf(G]0, +[) = 0, il existe 
dans Gun lment g tel que 0 < g < . Puis il existe un entier relatif n
x
tel que ng 6 x < (n + 1)g savoir n = E
.
g
Soit y = (n + 1)g. Dune part, y est dans G (si n + 1 = 0, (n + 1)g = 0 G, si n + 1 > 0, (n + 1)g = g + g + ... + G G
et si n + 1 < 0, (n + 1)g = ((n + 1)g) G) et dautre part
x < (n + 1)g = ng + g < x + = x.
On a montr que x R, > 0, y G/ x < y < x et donc G est dense dans R.
2me cas. Supposons a > 0. Montrons dans ce cas que G = aZ.
Pour cela, montrons tout dabord que a est dans G.
Mais si a nest pas lment de G, par dfinition de a, il existe un rel x dans G]a, 2a[ puis il existe un rel y
dans G]a, x[. Le rel x y est alors dans G]0, a[ ce qui est impossible. Donc a est lment de G.
Montrons alors que G = aZ. Puisque a est dans G, G contient encore a + a = 2a, puis a + a + a = 3a et plus
gnralement tous les na, n N . G contient aussi les opposs de ces nombres, et puisque G contient galement
0 = 0 a, G contient finalement tous les na, n Z. On a montr que aZ G.
http ://www.maths-france.fr

c Jean-Louis Rouget, 2014. Tous droits rservs.


x

x
< n + 1 puis 0 6 x na < a.
a
a
Or, x est dans G et na est dans G. Donc, x na est dans G [0, a[= {0}, puis x = na aZ. On a ainsi montr
linclusion contraire et donc G = aZ.

2) Soit G = a + b 2, (a, b) Z2 . Montrons que G est un sous-groupe de (R, +).

G R et 0 = 0 + 0 2 G.
Soit (a, a , b, b ) Z4 .

 


a + b 2 a + b 2 = (a a ) + (b b ) 2,

 

avec a a Z et b b Z. Donc a + b 2 a + b 2 G.
Rciproquement, soit x un lment de G et n = E

( Z). Alors, n 6

On a montr que G est un sous-groupe


de (R, +). Maintenant, la formule du binme de Newton
montre que, pour
n
n

21
21
G]0, +[. Or, 0 < 2 1 < 1 et donc lim
= 0. Ceci montre que
chaque entier naturel n,
n+

inf(G]0; +[) = 0 et donc que G est dense dans R.

3) a) Soit f une application de R dans lui-mme et Gf = {T R/ x R, f(x + T ) = f(x)}. Montrons que Gf est un
sous-groupe de (R, +).
0 est lment de Gf (et cest mme le seul lment de Gf si f nest pas priodique) et donc G 6= .
De plus, si T et T sont deux lments de G alors, pour tout rel x,
f(x + (T T )) = f((x T ) + T ) = f(x T ) = f(x T + T ) = f(x),
et T T est encore un lment de G.
On a montr que Gf est un sous groupe de (R, +). Le groupe des priodes dune fonction est donc soit de la forme aZ,
a R, soit dense dans R.

b) Soit
de R dans R admettant 1 et 2 pour priodes. Gf contient encore tous les nombres de la forme
f une application
a + b 2, (a, b) Z2 et est donc dense dans R.
Montrons que si de plus f est continue sur R, f est constante.
Soit x un rel quelconque. Soit T une priode strictement positive de f.
x
Il existe un entier relatif n tel que nT 6 x < (n + 1)T savoir n = E
. Avec cet entier n, on a bien f(x) = f(x nT )
T
avec 0 6 x nT < T .
1
ce qui implique que
Soit alors N N . Puisque Gf est dense dans R, il existe dans Gf un rel TN tel que 0 < TN <
N
lim TN = 0.
N+
 
 
 
  
x
x
x
Mais alors, puisque 0 < xE
TN < TN , on a aussi lim xE
TN = 0. Or, la suite f x E
TN
N+
TN
TN
TN
N>0
est constante (gale (f(x))) et donc convergente. On en dduit que

f(x) =


 



 
x
x
xE
lim
.TN = f
lim f x E
N+
N+
TN
TN

(par continuit de f)

= f(0)
ce quil fallait dmontrer.
c) Soit r un nombre rationnel. Soit x un rel.
Si x est un rationnel, alors x + r est un rationnel et donc Q (x + r) = 1 = Q (x),
Si x est un irrationnel, alors x + r est un irrationnel (dans le cas contraire, x = (x + r) r serait un rationnel ce qui
nest pas) et donc Q (x + r) = 0 = Q (x).
Ainsi, pour tout rel x, Q (x + r) = Q (x). Tout rationnel est priode de Q .
Soit s un nombre irrationnel. Q (0 + s) = Q (s) = 0 et Q (0) = 1. Donc, s nest pas une priode de Q .
Finalement, le groupe des priodes de Q est Q. Ce groupe des priodes est dense dans R mais nest pas gal R.

http ://www.maths-france.fr

c Jean-Louis Rouget, 2014. Tous droits rservs.


Planche no 25. Arithmtique dans Z


* trs facile ** facile *** difficult moyenne **** difficile
I : Incontournable T : pour travailler et mmoriser le cours
Exercice no 1 (**)
Montrer que le produit de quatre entiers conscutifs, augment de 1, est un carr parfait.
Exercice no 2 (***T)
1) Montrer que n Z, 6 | 5n3 + n.
n

2) Montrer que n N, 7 | 42 + 22 + 1.
Exercice no 3 (***IT)
Montrer quun entier de la forme 8n + 7 ne peut pas tre la somme de trois carrs parfaits.
Exercice no 4 (**IT)
Pour n N , on pose (1 +

2) = an + bn 2 o (an , bn ) (N )2 . Montrer que PGCD (an , bn ) = 1.

Exercice no 5 (****)
Montrer que, pour tout entier naturel n, 2n+1 divise E



2n+1
.
1+ 3

Exercice no 6 (***IT)
Soient A la somme des chiffres de 44444444 et B la somme des chiffres de A. Trouver la somme des chiffres de B. (Commencer
par majorer la somme des chiffres de n = c0 + 10c1 + ... + 10p cp .)
Exercice no 7 (**)
Montrer que si p est premier et 8p2 + 1 est premier alors 8p2 1 est premier.
Exercice no 8 (**I)
 
n
1) Montrer que (k, n) (N ) , k n = 1 n |
.
k
 
2n
2) Montrer que n N , (n + 1) |
.
n
2

Exercice no 9 (**T)
Rsoudre dans (N )2 les quations ou systmes dquations suivants :


x + y = 56
PGCD(x, y) = x y
1)
2)
3) PPCM(x, y) PGCD(x, y) = 243.
PPCM(x, y) = 105
PPCM(x, y) = 72
Exercice no 10 (***)
Montrer que la somme de cinq carrs parfaits dentiers conscutifs nest jamais un carr parfait.
Exercice no 11 (***IT)
n

Pour n N, on pose Fn = 22 + 1 (nombres de Fermat). Montrer que les nombres de Fermat sont deux deux premiers
entre eux.
Exercice no 12 (***)
Soit (un )nN la suite dfinie par u0 = 0, u1 = 1 et n N, un+2 = un+1 + un (suite de Fibonacci).

1) Montrer que n N , un+1 un1 u2n = (1)n et en dduire que n N , PGCD (un , un+1 ) = 1.

2) Montrer que n N, m N , um+n = um un+1 + um1 un et en dduire que PGCD (um , un ) = uPGCD(m,n) pour
m et n non nuls.
Exercice no 13 (***I)
On veut rsoudre dans Z3 lquation x2 + y2 = z2 (de tels triplets dentiers relatifs sont appels triplets pythagoriciens,
comme par exemple (3, 4, 5)).

http ://www.maths-france.fr

c Jean-Louis Rouget, 2014. Tous droits rservs.


1) Montrer que lon peut se ramener au cas o PGCD(x, y, z) = 1. Montrer alors que dans ce cas, x, y et z sont de plus
deux deux premiers entre eux.
2) On suppose que x, y et z sont deux deux premiers entre eux. Montrer que deux des trois nombres x, y et z sont
impairs le troisime tant pair puis que z est impair.
zx
z+x
et Z =
.
On suppose dornavant que x et z sont impairs et y est pair. On pose y = 2y , X =
2
2
3) Montrer que PGCD(X, Z) = 1 et que X et Z sont des carrs parfaits.
4) En dduire que lensemble des triplets pythagoriciens est lensemble des triplets de la forme


d u2 v2 , 2duv, d u2 + v2

o d N, (u, v) Z2 , une permutation prs des deux premires composantes.


Exercice no 14 (**)
Rsoudre dans N2 lquation 3x3 + xy + 4y3 = 349.
Exercice no 15 (***)
Rsoudre dans (N )2 lquation dinconnue (x, y) :

x
X

k! = y2 .

k=1

Exercice no 16 (***)
Montrer que n = 4...48...89 (p chiffres 4 et p 1 chiffres 8 et donc 2p chiffres) (en base 10) est un carr parfait.
Exercice no 17 (***I)
Montrer que tout nombre impair non divisible par 5 admet un multiple qui ne scrit (en base 10) quavec des 1 (par
exemple, 37 1 = 37, 37 2 = 74, 37 3 = 111).
Exercice no 18 (***)
Soit un = 10...012 .(n chiffres gaux 0). Dterminer lcriture binaire de :
1)u2n 2) u3n 3)u3n u2n + un .
Exercice no 19 (**I)
1) Dterminer en fonction de n entier non nul, le nombre de chiffres de n en base 10.
2) Soit (n) la somme des chiffres de n en base 10.


(n + 1)
a) Montrer que la suite
est borne. Cette suite converge-t-elle ?
(n)
n1
b) Montrer que pour toutpnaturel non nul n, 1 (n) 9(1 + log n).
c) Montrer que la suite ( n (n))n1 converge et prciser sa limite.
Exercice no 20 (***I)

1) (Formule de Legendre) Soit n un entier naturel suprieur ou gal 2 et p un nombre premier. Etablir que lexposant
de p dans la dcomposition de n! en facteurs premiers est
 
 
 
n
n
n
+E
+E
+ ...
E
p
p2
p3
2) Par combien de 0 se termine lcriture en base 10 de 1000! ?
Exercice no 21 (***I) (Petit thorme de Fermat) Soit p un nombre premier.
 
p
1) Montrer que, pour tout entier k tel que 1 6 k 6 p 1, p divise
.
k
2) Montrer que a N , ap a (p) (par rcurrence sur a).
Exercice no 22 (***I) (Thorme de Wilson)
Soit p un entier suprieur ou gal 2. Montrer que : (p 1)! 1 (p) p est premier (en fait les deux phrases
sont quivalentes mais en Sup, on sait trop peu de choses en arithmtique pour pouvoir fournir une dmonstration
raisonnablement courte de la rciproque).

http ://www.maths-france.fr

c Jean-Louis Rouget, 2014. Tous droits rservs.


Planche no 25. Arithmtique : corrig


Exercice no 1
Soit n un entier naturel.
n(n + 1)(n + 2)(n + 3) + 1 = n4 + 6n3 + 11n2 + 6n + 1 = (n2 + 3n + 1)2 ,
avec n2 + 3n + 1 entier naturel.
Exercice no 2
1) Soit n un entier relatif.
Si n est pair, alors 5n3 + n 5 03 + 0 [2] ou encore 5n3 + n 0 [2]. Dans ce cas, 5n3 + n est divisible par 2.
Si n est impair,alors 5n3 + n 5 13 + 1 [2] ou encore 5n3 + n 6 [2] ou enfin 5n3 + n 0 [2]. Dans ce cas aussi, 5n3 + n
est divisible par 2. Finalement : n Z, 2 | (5n3 + n).

Si n est multiple de 3, alors 5n3 + n 5 03 + 0 [3] ou encore 5n3 + n 0 [3]. Dans ce cas, 5n3 + n est divisible par 3.
Si n est de la forme 3k + 1 o k Z, alors
5n3 + n 5 13 + 1 [3] puis 5n3 + n 6 [3] et donc 5n3 + n 0 [3].

Par suite, 5n3 + n est divisible par 3.


Si n est de la forme 3k + 2 o k Z, alors ,

5n3 + n 5 23 + 2 [3] puis 5n3 + n 42 [3] et donc 5n3 + n 0 [3].

Dans ce cas aussi, 5n3 + n est divisible par 3.


Finalement, n Z, 3 | (5n3 + n).

Enfin, pour tout n Z, 5n3 + n est divisible par les nombres premiers 2 et 3 et donc par 2 3 = 6. On a montr que
n Z, 6 | (5n3 + n).
n

2) 42 signifie (...((42 )2 )2 ...)2 . Etudions la suite de ces lvations au carr successives modulo 7.
0
0
1
1
2
4
42 = 4 et donc 42 4 [7]. Ensuite, 42 42 [7] ou encore 42 2 [7]. Ensuite, 42 22 [7] ou encore 42 4 [7] ...
2k

Montrons par rcurrence que : k N, 42

2k+1

4 [7] et 42

2 [7].

Cest vrai pour p = 0.


2k
2k+1
Soit k > 0. Supposons que 42 4 [7] et 42
2 [7].
 2k+1 2
 2(k+1) 2
2(k+1)
2(k+1)+1
22(k+1)
2
2
Alors, 4
= 4
2 [7] ou encore 42
4 [7] puis 42
= 42
42 [7] ou encore
2(k+1)+1

42

2 [7]

2k

On a montr par rcurrence que k N, 42


0

2k+1

4 [7] et 42
n

2 [7].

n1

n1

Ensuite 22 = 2 est dans 2+ 7Z puis, pour n > 1, 22 = 222


= 42
est dans 4+ 7Z si n1 est pair ou encore si n est
n
n
impair et est dans 2+ 7Z si n est pair. Ainsi, que n soit pair ou impair, 42 + 22 + 1 est dans (4+ 2)+ 1+ 7Z = 7+ 7Z = 7Z
et on a montr que :
n

n N, 7 | 42 + 22 + 1.
Exercice no 3
Soient m, n et p trois entiers naturels et r1 , r2 et r3 les restes des divisions euclidiennes de m, n et p par 8. Alors,
m2 + n2 + p2 r21 + r22 + r23 [8].

Donc m2 + n2 + p2 est dans 7 + 8Z si et seulement si r21 + r22 + r23 est dans 7 + 8Z.
Comme r1 , r2 et r3 sont des entiers entre 0 et 7, il suffit de vrifier que les sommes de trois carrs dentiers compris au
sens large entre 0 et 7 ne sont pas dans 7 + 8Z.
Or, 02 = 0 8Z, 12 = 1 1 + 8Z, 22 = 4 4 + 8Z, 32 = 9 1 + 8Z, 42 = 16 8Z, 52 = 25 1 + 8Z, 62 = 36 4 + 8Z et
72 = 49 1 + 8Z. Donc, les carrs des entiers de 0 7 sont dans 8Z ou 1 + 8Z ou 4 + 8Z. Enfin,
http ://www.maths-france.fr

c Jean-Louis Rouget, 2014. Tous droits rservs.


0 + 0 + 0 = 0 8Z,
0 + 1 + 4 = 5 5 + 8Z
1 + 4 + 4 = 9 1 + 8Z,

0 + 0 + 1 = 1 1 + 8Z,
0 + 4 + 4 = 8 8Z,
4 + 4 + 4 = 12 4 + 8Z.

0 + 0 + 4 = 4 4 + 8Z,
1 + 1 + 1 = 3 3 + 8Z,

0 + 1 + 1 = 2 2 + 8Z,
1 + 1 + 4 = 6 6 + 8Z,

Aucune de ces sommes nest dans 7 + 8Z et on a montr quun entier de la forme 8n + 7 nest pas la somme de trois carrs.
Exercice no 4

n

Soit n N . En dveloppant 1 + 2
par la formule du binme de Newton et en sparant les termes o 2 apparat
n

un exposant pair des termes o 2 apparat un exposant impair, on crit


la forme an + bn 2 o an
n(1 + 2) sous
et bn sont des entiers naturels non nuls. Un calcul conjugu fournit (1 2) = an bn 2 et donc

n 


n 
= an + bn 2 an bn 2 = a2n 2b2n
1 2
(1)n = 1 + 2
ou finalement,

((1)n an ) an + 2(1)n+1 bn bn = 1

o u = (1)n an et v = 2(1)n+1 bn sont des entiers relatifs. Le thorme de Bezout permet daffirmer que an et bn
sont premiers entre eux.
Exercice no 5


n
n

Posons 1 + 3 = an + bn 3 o an et bn sont des entiers naturels puis 1 3


= an bn 3 et donc

2n+1 
2n+1
3
+ 1 3
= 2a2n+1 N.

Mais de plus, 1 < 1 3 < 0 et donc, puisque 2n + 1 est impair, 1 < (1 3)2n+1 < 0. Par suite,

2a2n+1 < (1 + 3)2n+1 < 2a2n+1 + 1,







2n+1 
2n+1
2n+1
2n+1
= 2a2n+1 = 1 + 3
+ 1 3
et montre dj que E
1+ 3
ce qui montre que E
1+ 3


1+

est un entier pair. Mais on en veut plus :








2n+1 
2 n 
2 n
2n+1 
 
 
1+ 3
1+ 3
1 3
+ 1 3
= 1+ 3
+ 1 3







n
n

= 1+ 3 4+2 3 + 1 3 42 3
h

n 
n i

= 2n 1 + 3 2 + 3 + 1 3 2 3




n 
n
n


Montrons enfin que 1 + 3 2 + 3 + 1 3 2 3
est un entier, pair. Mais, 1 + 3 2 + 3
est de la


n

= A B 3, on a finalement
forme A + B 3 o A et B sont des entiers naturels et donc, puisque 1 3 2 3

n 
n


1 + 3 2 + 3 + 1 3 2 3 = 2A o A est un entier.





2n+1
n
n

2n+1
Donc, 1 + 3 2 + 3
+ 1 3
+ 1 3 2 3
est un entier pair, ou encore 1 + 3
=


2n+1
est un entier divisible par 2n+1 .
E
1+ 3
Exercice no 6
Soit n un entier naturel non nul. On note (n) la somme de ses chiffres en base 10. Si n = c0 + 10c1 + ... + 10k ck o
k N, 0 6 ci 6 9 pour 0 6 i 6 k et ck 6= 0, alors
(n) = c0 + ... + ck 6 9(k + 1) = 9(E(log n) + 1) 6 9(log n + 1).
Donc,
A = (44444444 ) 6 9(log(44444444 ) + 1) 6 9(4444 log(105 ) + 1) = 9(4444 5 + 1) = 9 22221 = 199989.
Puis, B = (A) 6 1 + 5 9 = 46, puis (B) 6 (39) = 12. Donc, 1 6 (B) 6 12.
http ://www.maths-france.fr

c Jean-Louis Rouget, 2014. Tous droits rservs.


Dautre part, on sait que modulo 9 : (B) B A 44444444 . Enfin, 44444444 = (9 493 + 7)4444 74444 (9). De
plus, 7 2 (9) puis 72 4 (9) puis 73 28 1 (9) et donc 74444 = (73 )1481 7 (13 )1481 7 7 (9). Finalement,
1 6 (B) 6 12 et (B) 7 (9) ce qui impose
(B) = 7.
Exercice no 7
On a trois possibilits : p 3Z, p 3Z + 1 ou p 3Z 1.
Dans les deux derniers cas, p2 1 + 3Z et 8p2 + 1 9 + 3Z = 3Z. Mais alors, 8p2 + 1 est premier et multiple de 3 ce qui
impose 8p2 + 1 = 3. Cette dernire galit est impossible.
Il ne reste donc que le cas o p est premier et multiple de 3, cest--dire p = 3 (en rsum, p et 8p2 + 1 premiers impliquent
p = 3). Dans ce cas, 8p2 + 1 = 73 et 8p2 1 = 71 sont effectivement premiers.
Exercice no 8
 


 
n
n1
n
1) Pour 1 6 k 6 n, k
=n
. Donc, si k et n sont premiers entre eux, puisque n divise k
, le thorme de
k
k1  
k
n
Gauss permet daffirmer que n divise
.
k


 
 
2n
2n
2n
2) De mme, (n + 1)
=n
montre que (n + 1) divise n
et, puisque n et (n + 1) sont premiers entre
n1
n
  n
2n
eux (daprs Bezout puisque (n + 1) n = 1), (n + 1) divise
daprs le thorme de Gauss.
n
Exercice no 9
1) Posons d = PGCD(x, y) et m = PPCM(x, y). d divise m = 105 = 3 5 7 mais, puisque d divise x et y, d divise aussi
x + y = 56 = 23 7. Donc, d divise PGCD(105, 56) = 7 et ncessairement d = 1 ou d = 7.
1er cas. d = 1 fournit, puisque m = 105, xy = md = 105. x et y sont donc les solutions de lquation X2 56X + 105 = 0
qui nadmet pas de solutions entires.

2me cas. d = 7 fournit xy = 7 105 = 735. x et y sont donc les solutions de lquation X2 56X + 735 = 0 qui admet
les solutions 21 et 35.
Rciproquement, 21 + 35 = 56 et PPCM(21, 35) = 3 5 7 = 105. Donc
S = {(21, 35), (35, 21)}.

x y = 1
2) On pose x = dx et y = dy avec x et y premiers entre eux et d = PGCD(x, y). Le systme scrit
dx y = 72

x = y + 1
. En particulier, y et y + 1 sont deux diviseurs conscutifs de 72. 72 = 23 32 admet
ou encore
d(y + 1)y = 72
4 3 = 12 diviseurs savoir 1, 2, 3, 4, 6, 8, 9, 12, 18, 24, 36 et 72. Donc y est lment de {1, 2, 3, 8}.

72
= 36 puis y = 36 1 = 36 et x = y + d = 72. Rciproquement, 72 36 = 36 =
12
PGCD(36, 72) et PPCM(36, 72) = 72.
2me cas. y = 2 fournit d = 12, y = 24, x = 36 qui rciproquement conviennent.
3me cas. y = 3 fournit d = 6, y = 18, x = 24 qui rciproquement conviennent.
4me cas. y = 8 fournit d = 1, y = 8, x = 9 qui rciproquement conviennent.

1er cas. y = 1 fournit d =

S = {(9, 8), (24, 18), (36, 24), (72, 36)}.


3) Posons d = PGCD(x, y) et m = PPCM(x, y). d divise m et donc d divise md = 243 = 35 puis d {1, 3, 9, 27, 81, 243}.
On pose alors x = dx , y = dy avec x et y premiers entre eux.
1er cas. Si d = 1 on a x y 1 = 243 ou encore x y = 244 = 22 61 ce qui fournit les possibilits (en noubliant pas que
x et y sont premiers entre eux) :
x = 1, y = 244 puis x = 1 et y = 244,
x = 4, y = 61 puis x = 4 et y = 61,
x = 61, y = 4 puis x = 61 et y = 4,
x = 244, y = 1 puis x = 244 et y = 1 qui rciproquement conviennent.
http ://www.maths-france.fr

c Jean-Louis Rouget, 2014. Tous droits rservs.


2me cas. Si d = 3, on a 3x y 3 = 243 puis x y = 81 + 1 = 82 = 2 41 ce qui fournit les possibilits :


x = 1, y = 82 puis x = 3 et y = 246,
x = 2, y = 41 puis x = 6 et y = 123,
x = 41, y = 2 puis x = 123 et y = 6,
x = 82, y = 1 puis x = 246 et y = 3 qui rciproquement conviennent.
3me cas. Si d = 9 on a x y = 27 + 1 = 28 = 22 7 ce qui fournit les possibilits :
x = 1, y = 28 puis x = 9 et y = 252,
x = 4, y = 7 puis x = 36 et y = 63,
x = 7, y = 4 puis x = 63 et y = 36,
x = 28, y = 1 puis x = 252 et y = 9 qui rciproquement conviennent.
4me cas. Si d = 27 on a x y = 9 + 1 = 10 = 2 5 ce qui fournit les possibilits :
x = 1, y = 10 puis x = 27 et y = 270,
x = 2, y = 5 puis x = 54 et y = 135,
x = 5, y = 2 puis x = 135 et y = 54,
x = 10, y = 1 puis x = 270 et y = 27 qui rciproquement conviennent.
5me cas. Si d = 81, on a x y = 3 + 1 = 4 = 22 ce qui fournit les possibilits :
x = 1, y = 4 puis x = 81 et y = 324,
x = 4, y = 1 puis x = 324 et y = 81 qui rciproquement conviennent.
6me cas. Si d = 243, on a x y = 1 + 1 = 2 ce qui fournit les possibilits :
x = 1, y = 2 puis x = 243 et y = 486,
x = 2, y = 1 puis x = 486 et y = 243 qui rciproquement conviennent.
Exercice no 10
Soit n un entier suprieur ou gal 2.
(n 2)2 + (n 1)2 + n2 + (n + 1)2 + (n + 2)2 = 5n2 + 10 = 5(n2 + 2).
5(n2 + 2) devant tre un carr parfait, n2 + 2 doit encore tre divisible par 5 mais si n est dans 5Z, n2 + 2 est dans 2 + 5Z,
si n est dans 1 + 5Z, n2 + 2 est dans 3 + 5Z et si n est dans 2 + 5Z, n2 + 2 est dans 1 + 5Z et n2 + 2 nest jamais
divisible par 5. Une somme de cinq carrs dentiers conscutifs nest donc pas un carr parfait.
Exercice no 11
Soient n et m deux entiers naturels tels que n < m. Posons m = n + k avec k > 0. On note que
n+k

Fm = 22

 n 2k
2k
+ 1 = 22
+ 1 = (Fn 1) + 1.

En dveloppant lexpression prcdente par la formule du binme de Newton et en tenant compte du fait que 2k est pair
puisque k est strictement positif, on obtient une expression de la forme
Fm = q Fn + 1 + 1 = q Fn + 2,
o q est un entier.
Le PGCD de Fn et Fm doit encore diviser Fm q Fn = 2 et vaut donc 1 ou 2. Enfin, puisque 2n et 2m sont strictement
positifs, Fn et Fm sont impairs et leur PGCD vaut donc 1 (ce rsultat redmontre aussi lexistence dune infinit de nombres
premiers).
Exercice no 12
1) Pour n entier naturel non nul donn, posons vn = un+1 un1 u2n . Alors,
vn+1 = un+2 un u2n+1 = (un + un+1 ) un un+1 (un1 + un ) = u2n un+1 un1 = vn .
La suite v est donc une suite gomtrique de raison 1 et on a :
n N , vn = (1)n1 v1 = (1)n .

Cette galit scrit encore ((1)n un1 ) un+1 + (1)n+1 un un = 1 et le thorme de Bezout permet daffirmer que
pour tout entier naturel n, les entiers un et un+1 sont premiers entre eux (il est clair par rcurrence que la suite u est
valeurs entires).
2) Pour m = 1 et n entier naturel quelconque :
un+m = un+1 = un+1 1 + un 0 = un+1 u1 + un u0 = un+1 um + um1 un .
Pour m = 2 et n entier naturel quelconque :
http ://www.maths-france.fr

c Jean-Louis Rouget, 2014. Tous droits rservs.


un+m = un+2 = un+1 + un = un+1 u2 + un u1 = un+1 um + um1 un .


Soit m > 1. Supposons que pour tout entier naturel n, on a un+m = un+1 um + um1 un et un+m+1 = un+1 um+1 +
um un . Alors, pour tout entier naturel n,
un+m+2 = un+m+1 + un+m = un+1 um+1 + um un + un+1 um + um1 un (par hypothse de rcurrence)
= un+1 (um+1 + um ) + un (um + um1 ) = un+1 um+2 + un um+1 .
ce qui dmontre lgalit propose par rcurrence.
Soient n et m deux entiers naturels tels que n > m. La division euclidienne de n par m scrit n = mq + r avec q et r
entiers tels que 0 6 r 6 m 1.
Or, um+r = um ur+1 + um1 ur . Par suite, un diviseur commun um et ur divise encore um et um+r et rciproquement
un diviseur commun um et um+r divise um1 ur . Mais, um et um1 sont premiers entre eux et, daprs le thorme
de Gauss, un diviseur commun um et um+r divise ur . Les diviseurs communs um et ur sont encore les diviseurs
communs um et um+r et donc :
PGCD (um , ur ) = PGCD (um , um+r ) .
Puis, par rcurrence
PGCD (um , ur ) = PGCD (um , um+r ) = PGCD (um , um+2r ) = ... = PGCD (um , uqm+r ) = PGCD (um , un ) .
Ainsi, les algorithmes dEuclide appliqus dune part um et un et dautre part m et n seffectuent en parallle et en
particulier, PGCD (um , un ) = uPGCD(m,n) .
Exercice no 13
1) Posons d = PGCD(x, y, z) puis x = dx , y = dy et z = dz o PGCD(x , y , z ) = 1.


2
2
2
2
2
x2 + y2 = z2 d2 x + d2 y = d2 z 2 x + y = z ,

avec PGCD(x , y , z ) = 1, ce qui montre que lon peut se ramener au cas o x, y et z sont premiers entre eux.
Supposons donc x, y et z premiers entre eux (dans leur ensemble). Soit p un nombre premier. Si p divise x et y alors p
divise x2 + y2 = z2 et donc p est galement un facteur premier de z contredisant le fait que x, y et z sont premiers entre
eux. Donc, x et y sont premiers entre eux.
Si p divise x et z alors p divise z2 x2 = y2 et donc p est galement un facteur premier de y, contredisant le fait que x, y
et z sont premiers entre eux. Donc, x et z sont premiers entre eux. De mme, y et z sont premiers entre eux. Finalement,
x, y et z sont premiers entre eux deux deux.
2) Puisque x, y et z sont deux deux premiers entre eux, parmi les nombres x, y et z, il y a au plus un nombre pair. Mais
si ces trois nombres sont impairs, x2 + y2 = z2 est pair en tant que somme de deux nombres impairs contredisant le fait
que z est impair. Ainsi, parmi les nombres x, y et z, il y a exactement un nombre pair et deux nombres impairs.
Si x et y sont impairs, alors dune part, z est pair et z2 est dans 4Z et dautre part x2 et y2 sont dans 1 + 4Z. Mais alors,
x2 + y2 est dans 2 + 4Z excluant ainsi lgalit x2 + y2 = z2 . Donc, z est impair et lun des deux nombres x ou y est pair.
Supposons, quite permuter les lettres x et y, que x est impair et y est pair.
zx
z+x
et Z =
(puisque x et z sont impairs, X et Z sont des entiers).
Posons alors y = 2y puis X =
2
2
3) On a
2

x2 + y2 = z2 4y = (z + x)(z x) y = XZ.

Un diviseur commun X et Z divise encore z = Z + X et x = Z X et est donc gal 1 puisque x et z sont premiers
entre eux. X et Z sont des entiers premiers entre eux.
Le produit des deux entiers X et Z est un carr parfait et ces entiers sont premiers entre eux. Donc, un facteur premier de
X napparat pas dans Z et apparat donc dans X un exposant pair ce qui montre que X est un carr parfait. De mme,
Z est un carr parfait.
4) Donc, il existe deux entiers relatifs u et v tels que X = u2 et Z = v2 . Mais alors, z = Z+X = u2 +v2 et x = ZX = u2 v2 .
Enfin, y2 = z2 x2 = (u2 + v2 )2 (u2 v2 )2 = 4u2 v2 et donc, y = 2uv quite remplacer
u par u.



2
2
2
2
En rsum, si x2 + y2 = z2 alors
il
existe
(d,
u,
v)

Z
tel
que
x
=
d
u

v
,
y
=
2duv
et
z
=
d
u
+
v
ou


2
2
2
2
bien x = 2duv, y = d u v et z = d u + v .
http ://www.maths-france.fr

c Jean-Louis Rouget, 2014. Tous droits rservs.


Rciproquement,
d u2 v2

2

2

,
+ (2duv)2 = d2 u4 + 2u2 v2 + v4 = d u2 + v2

et on a trouv tous les triplets Pythagoriciens. Par exemple, d = 1, u = 2 et v = 1 fournissent le triplet (3, 4, 5). d = 2,
u = 2 et v = 1 fournissent le triplet (6, 8, 10) et d = 1, u = 3 et v = 2 fournissent le triplet (5, 12, 13).
Exercice no 14
Soient x et y deux entiers naturels tels que 3x3 + xy + 4y3 = 349. On a 4y3 6 3x3 + xy + 4y3 = 349 et donc
r
3 349
= 4, 4...
y6
4
Donc, y {0, 1, 2, 3, 4}. De mme, 3x3 6 3x3 + xy + 4y3 = 349 et donc
r
3 349
x6
= 4, 8...
3
Donc, x {0, 1, 2, 3, 4} ce qui ne laisse plus que 5.5 = 25 couples candidats. Ensuite,
y = 0 donne 3x3 = 349 qui ne fournit pas de solutions.
y = 1 donne 3x3 + x 345 = 0, quation dont aucun des entiers de 0 4 nest solution.
y = 2 donne 3x3 + 2x 317 = 0, quation dont aucun des entiers de 0 4 nest solution.
y = 3 donne 3x3 + 3x 241 = 0, quation dont aucun des entiers de 0 4 nest solution.
y = 4 donne 3x3 + 4x 93 = 0 dont seul x = 3 est solution.
S = {(3, 4)}.
Exercice no 15
Si x > 5 et 5 6 k 6 x, alors k! est divisible par 2 5 = 10 puis
et le chiffre des units de

x
X

k=1

k! est 3.

x
X

x
X

k! est divisible par 10. Dautre part, 1! + 2! + 3! + 4! = 33

k=5

k! nest donc pas un carr parfait car le chiffre des units (en base 10) dun

k=1

carr parfait est choisir parmi 0, 1, 4, 5, 6, 9. Donc, x 6 4. Ensuite, 1! = 1 = 12 puis 1! + 2! = 1 + 2 = 3 nest pas un
carr parfait, puis 1! + 2! + 3! = 9 = 32 puis 1! + 2! + 3! + 4! = 33 nest pas un carr parfait.
S = {(1, 1), (3, 3)}.
Exercice no 16

10p 1
10p1 1
+ 4 10p
10 1
10 1
2


 1
1
2 10p + 1
p1
p
p
2p
p
=
,
81 + 80 10
1 + 4 10 (10 1) = (4 10 + 4 10 + 1) =
9
9
3

n = 9 + 8(10 + 102 + ... + 10p1 ) + 4(10p + ... + 102p1 ) = 9 + 80

(ce qui montre dj que n est le carr dun rationnel). Maintenant, modulo 3,
2 10p + 1 2 1p + 1 0,

2 10p + 1
et 2 10 + 1 est un entier divisible par 3 ou encore
est un entier. Finalement, n =
3
le carr dun entier.
p

2.10p + 1
3

2

est bien

Exercice no 17
Pour k N, posons ak = 11...1 (k + 1 chiffres 1 en base 10).
Soit n un entier naturel quelconque.
La division euclidienne de ak par n scrit : ak = n qk + rk o qk et rk sont des entiers naturels tels que 0 6 rk 6 n 1.
Les n + 1 entiers r0 , ... , rn sont choisir parmi les n entiers 0, 1, ... , n 1. Les n + 1 restes considrs ne peuvent donc
tre deux deux distincts (principe des tiroirs). Par suite,
http ://www.maths-france.fr

c Jean-Louis Rouget, 2014. Tous droits rservs.


(k, l) N2 / 0 6 k < l 6 n et rk = rl .
Mais alors, al ak = (ql qk )n est un multiple de n. Comme al ak = 11...10...0 (l k chiffres 1 et k + 1 chiffres 0),
on a montr que tout entier naturel admet un multiple de la forme 11...10...0 = 11...1 10k+1 . Si de plus n est impair,
non divisible par 5, alors n est premier 2 et 5 et donc 10k+1 . Daprs le thorme de Gauss, n divise 11...1.
Exercice no 18
2
1) u2n = 2n+1 + 1 = 22n+2 + 2n+2 + 1 = 10...010...012 (n 1 puis n + 1 chiffres 0)

2)

3
u3n = 2n+1 + 1 = 23n+3 + 3 22n+2 + 3 2n+1 + 1 = 23n+3 + (2 + 1) 22n+2 + (2 + 1) 2n+1 + 1
= 23n+3 + 22n+3 + 22n+2 + 2n+2 + 2n+1 + 1 = 10...0110...0110...012

(n 1 puis n 1 puis n chiffres 0)


3)
u3n u2n + un = 23n+3 + 3 22n+2 + 3 2n+1 + 1 22n+2 2n+2 1 + 2n+1 + 1 = 23n+3 + 22n+3 + 2n+2 + 1
= 10...010...010...01
(n 1 puis n puis n + 1 chiffres 0).
Exercice no 19
1) Soit n N . Posons n =

p
X

k=0

ck 10k , o p N, et k J0, pK, ck J0, 9K, et cp 6= 0. Le nombre de chiffres de n est

alors p + 1. Lentier p vrifie 10p 6 n < 10p+1 ou encore p 6 log n < p + 1. Par suite, p = E(log n). Ainsi,
le nombre de chiffres de n en base 10 est E(log n) + 1.
2) Pour n N , posons un =

(n + 1)
(n)

a) Soit n N . Posons n = cp 10p + ... + 10c1 + c0 = cp ...c1 c0 10 . Si au moins un des chiffres de n nest pas 9, on note k
le plus petit indice tel que ak 6= 9. Alors, 0 6 k 6 p 1 et n = ap ...ak 9...910 et n + 1 = ap ...ak+1 (ak + 1)0...010 . Dans
ce cas, si k = 0,
(n) + 1
1
(n + 1)
=
=1+
6 1 + 1 = 2.
(n)
(n)
(n)
et si 1 6 k 6 p 1,
(n + 1)
ap + ... + ak + 1
ap + ... + ak + 1
=
6
= 1 2.
(n)
ap + ... + ak + 9k
ap + ... + ak + 1
Sinon, tous les chiffres de n sont gaux 9, et dans ce cas,
1
(n + 1)
=
6 2.
(n)
9(p + 1)
Ainsi, pour tout entier naturel non nul n, on a un 6 2. La suite u est donc borne.
1
(10p )
=
. La suite extraite (u10p 1 )pN converge et a pour limite 0.
p
(10 1)
9p
(10p + 1)
2
Pour p N , u10p =
= = 2. La suite extraite (u10p )pN converge et a pour limite 2 6= 0.
(10p )
1
On en dduit que la suite u diverge.
Pour p N , u10p 1 =

La suite u est borne et diverge.

http ://www.maths-france.fr

c Jean-Louis Rouget, 2014. Tous droits rservs.


b) Avec les notations du a), 1 6 (n) 6 9(p + 1) = 9(E(log n) + 1) 6 9(log n + 1).


 


p
p
1
ln n
n
n

ln 9 + ln 1 +
. Les deux membres de cet encadrement
c) Soit n N . 1 6 (n) 6 9(log n + 1) = exp
n
ln 10
p

p
tendent vers 1 et donc la suite n (n)
converge et lim n (n) = 1.
n+

n>1

Exercice no 20

1) (Formule de Legendre) Soit n un entier naturel suprieur ou gal 2.


Si p est un nombre premier qui divise n! = 1 2... n, alors p est un facteur premier de lun des entiers 2,..., n et en
particulier, p 6 n. Rciproquement, il est clair que si p est un nombre premier tel que p 6 n, p divise n!. Les facteurs
premiers de n! sont donc les nombres premiers infrieurs ou gaux n.
Soit donc p un nombre premier tel que p 6 n. Pour trouver la valuation p-adique de n!, on compte 1 pour chaque multiple
de p infrieur ou gal n, on rajoute 1 pour chaque multiple de p2 infrieur ou gal n, on rajoute encore 1 pour chaque
multiple de p3 infrieur ou gal n... et on sarrte quand lexposant k vrifie pk > n. Or

(car ln p > 0). Donc, si k > E

ln n
ln p

n > pk ln n > k ln p k 6

ln n
,
ln p

+ 1, alors pk > n.

Dit autrement, lexposant de p est la somme du nombre de multiples de p infrieurs ou gaux n, du nombre de multiples
de p2 infrieurs ou gaux n, du nombre de multiple de p3 infrieurs ou gaux n... et du nombre de multiples de
pE(ln n/ ln p) .


ln n
Soit k un entier tel que 1 6 k 6 E
et K un entier naturel.
ln p
 
n
n
1
.
1 6 K pk 6 n k 6 K 6 k 1 6 K 6 E
p
p
pk
 
n
multiples de pk compris au sens large entre 1 et n. On a montr que la valuation p-adique de n! est
Il y a donc E
pk
n
n
n
E( ) + E( 2 ) + E( 3 ) + ...
p
p
p
2) Tout dabord 10 = 2 5. Lexposant de 5 dans la dcomposition primaire de 1000! est








1000
1000
1000
1000
E
+E
+
E
+
E
= 200 + 40 + 8 + 1 = 249.
5
52
53
54
Lexposant de 2 est videmment suprieur (il y a dj au moins 500 nombres pairs entre 1 et 1000). Donc, la plus grande
puissance de 10 divisant 1000! est encore la plus grande puissance de 5 divisant 1000!, savoir 249. Lcriture en base 10
de 1000! se termine par 249 zros.
Exercice no 21
Soit p un nombre premier.
 


 
p
p1
p
1) Soit p un nombre premier et k un entier tel que 1 6 k 6 p 1. On a k
=p
. Donc, p divise k
. Mais, p
k
k1
k
est premier
et donc p est premier tous les entiers compris entre 1 et p 1 au sens large. Daprs le thorme de Gauss,
 
p
p divise
.
k
2) Soit p un nombre premier. Montrons par rcurrence que a N , ap a (p).
Cest clair pour a = 1.

Soit a > 1. Supposons que ap a (p). On a alors


p  
p1
X
X
p k
p
(a + 1) =
a =a +1+
Ckp ak
k
p

k=0

k=1

ap + 1 (p) (daprs 1))


a + 1 (p) (par hypothse de rcurrence)
http ://www.maths-france.fr

c Jean-Louis Rouget, 2014. Tous droits rservs.


On a montr par rcurrence que


a N , ap a (p).
Exercice no 22
Soit p un entier naturel suprieur ou gal 2.
Supposons que (p 1)! 1 (p). Il existe doncun entier
relatif a tel que (p 1)! = 1 + ap ().
Y
Soit k J1, p 1K. Lgalit () scrit encore k
j + ap = 1. Le thorme de Bezout permet alors daffirmer que
j6=k

k et p sont premiers entre eux. Ainsi, p est premier avec tous les entiers naturels lments de J1, p 1K et donc, p est un
nombre premier.

http ://www.maths-france.fr

c Jean-Louis Rouget, 2014. Tous droits rservs.


Planche no 26. Polynmes


* trs facile ** facile *** difficult moyenne **** difficile
I : Incontournable T : pour travailler et mmoriser le cours
Exercice no 1 (***I)
Calculer an =

n1
Y

sin

k=1

k
pour n > 2.
n

Exercice no 2 (***)
Pour n N et k J0, n 1K, on pose k = e2ik/n . On note Q le polynme Q = 1 + 2X + ... + nXn1 .
n1
Y
Calculer
Q (k ).
k=0

+
X
1
2
Exercice n 3 (****I) (Calcul de
=
)
n2
6
o

n=1

1) Soient p un entier naturel et a un rel. Donner le dveloppement de (cos a + i sin a)2p+1 puis en choisissant astucieup
p
X
X
k
1
.
. En dduire alors
sement a, dterminer
cotan2
k
2p + 1
2
k=1 sin
k=1
2p + 1
2) Pour n entier naturel non nul, on pose un =

n
X
1
. Montrer que la suite (un )nN converge (pour majorer un , on
k2

k=1

1
1
remarquera que 2 6
).
k
k(k 1)

1
1
[, on a cotan x < <
.
2
x
sin x
4) En dduire un encadrement de un puis la limite de (un ).
3) Montrer que pour tout rel x de ]0,

Exercice no 4 (**IT)
Dterminer le PGCD de X6 7X4 + 8X3 7X + 7 et 3X5 7X3 + 3X2 7.
Exercice no 5 (**T)
Pour quelles valeurs de lentier naturel n le polynme (X + 1)n Xn 1 est-il divisible par X2 + X + 1 ?
Exercice no 6 (***)
Soit P un polynme coefficients rels tel que x R, P(x) > 0. Montrer quil existe deux polynmes R et S coefficients
rels tels que P = R2 + S2 .
Exercice no 7 (**)
Soit P un polynme diffrent de X. Montrer que P(X) X divise P(P(X)) X.
Exercice no 8 (***)
Soit P un polynme coefficients entiers relatifs de degr suprieur ou gal 1. Soit n un entier relatif et m = P(n).
1) Montrer que k Z, P(n + km) est un entier divisible par m.
2) Montrer quil nexiste pas de polynmes non constants coefficients entiers tels que P(n) soit premier pour tout entier
n.

http ://www.maths-france.fr

c Jean-Louis Rouget, 2014. Tous droits rservs.


Exercice no 9 (***) (Polynmes P vrifiant P(Z) Z)


Soit E la partie de C[X] forme des polynmes P vrifiant a Z, P(a) Z.
n
1 Y
(X + k) (on peut dfinir la notation Pn = Cn
1) On pose P0 = 1 et pour n entier naturel non nul, Pn =
X+n ). Montrer
n!
k=1
que n N, Pn E.

2) Montrer que toute combinaison linaire coefficients entiers relatifs des Pn est encore un lment de E.
3) Montrer que E est lensemble des combinaisons linaires coefficients entiers relatifs des Pn .
Exercice no 10 (***)
Division euclidienne de P = sin aXn sin(na)X + sin((n 1)a) par Q = X2 2X cos a + 1, a rel donn, n > 2.
Exercice no 11 (****I) (Thorme de Lucas.)
Soit P C[X] de degr suprieur ou gal 1. Montrer que pour toute racine z de P , il existe des rels positifs 1 , . . . , n ,
n
X
de somme gale 1 tels que z =
k zk o z1 , . . . , zn sont les n racines distinctes ou confondues de P dans C (on dit
k=1

que les racines de P sont des barycentres coefficients positifs des racines de P ou encore que les racines de P sont dans
P
lenveloppe convexe des racines de P). Indication : calculer
.
P
Exercice no 12 (***)
Dcomposer en produit de facteurs irrductibles dans R[X] le polynme X6 2X3 cos a + 1 o a est un rel donn dans
[0, ].
Exercice no 13 (***T)
Trouver un polynme de degr 5 tel que P(X) + 10 soit divisible par (X + 2)3 et P(X) 10 soit divisible par (X 2)3 .
Exercice no 14 (***I)
Trouver les polynmes P de R[X] vrifiant P(X2 ) = P(X)P(X + 1) (penser aux racines de P).
Exercice no 15 (**T)
Dterminer a C tel que P = X5 209X + a admette deux zros dont le produit vaut 1.
Exercice no 16 (***T)
Soit (ak )16k65 la famille des racines de P = X5 + 2X4 X 1. Calculer

5
X
ak + 2
.
ak 1

k=1

Exercice n 17 (**)
o

x+y+z=1

1
1 1
Rsoudre dans C3 le systme :
+ + =1 .

x
y
z

xyz = 4
Exercice no 18 (**T)

Trouver tous les polynmes P vrifiant P(2X) = P (X)P (X).


Exercice no 19 (**)
1) Soit P =

n
X

ak Xk un polynme non nul de degr n N, dont les coefficients a0 , . . . ,an , sont des entiers relatifs.

k=0

Soient p un entier relatif non nul et q un entier naturel non nul tels que PGCD(p, q) = 1 puis r =

p
.
q

Montrer que si P(r) = 0, alors p divise a0 et q divise an .


2) Factoriser dans C[X] le polynme 12X4 + X3 + 15X2 20X + 4.
Exercice no 20 (***)
Soit n N . Montrer que (X 1)2n X2n + 2X 1 est divisible par 2X3 3X2 + X puis dterminer le quotient.

http ://www.maths-france.fr

c Jean-Louis Rouget, 2014. Tous droits rservs.


Exercice no 21 (**I)
1) Soit n un entier naturel non nul. Dterminer deux polynmes U et V tels que UXn + V(1 X)n = 1 et deg(U) < n et
deg(V) < n..
2) Plus gnralement, si n et m sont deux entiers naturels non nuls, dterminer deux polynmes U et V vrifiant UXn +
V(1 X)m = 1 et deg(U) < m et deg(V) < n.
Exercice no 22 (**I)
Soit P un polynme rel de degr suprieur ou gal 2.
1) a) Montrer que si P na que des racines simples et relles, il en est de mme de P .
b) Le rsultat persiste-t-il si on suppose simplement que les racines de P sont simples mais pas ncessairement relles ?
2) Montrer que si P est scind sur R, il en est de mme de P .
Exercice no 23 (***)
k
Former une quation du sixime degr dont les racines sont les sin
o k {3, 2, 1, 1, 2, 3} puis montrer que ces six
7
nombres sont irrationnels.
Exercice no 24 (***)
2
y + yz + z2 = 7
3
z2 + zx + x2 = 13 .
Rsoudre dans C le systme
2
x + xy + y2 = 3

Exercice no 25 (***)

Dterminer et complexes tels que les zros de z4 4z3 36z2 + z + soient en progression arithmtique.
Exercice no 26 (***)
Rsoudre dans C lquation z4 21z + 8 = 0 sachant quil existe deux des solutions sont inverses lune de lautre.
Exercice no 27 (***)
Soient x1 , x2 , x3 les zros de X3 + 2X 1. Calculer x41 + x42 + x43 .
Exercice no 28 (***)
Soit P un polynme coefficients complexes de degr 4.
Montrer que les images dans le plan complexe des racines de P forment un paralllogramme si et seulement si P et P(3)
ont une racine commune
Exercice no 29 (**I)
Soit n un entier naturel suprieur ou gal 2. Pour k Z, on pose k = e2ik/n .

n1
Y
2
1) Calculer
1+
.
2 k
k=0

2) Montrer que, pour tout rel a,

n1
Y
k=0

http ://www.maths-france.fr


2k 2k cos a + 1 = 2(1 cos(na)) (questions indpendantes.)

c Jean-Louis Rouget, 2014. Tous droits rservs.


Planche no 26. Polynmes : corrig


Exercice no 1
Soit n > 2.
an =

n1
Y
k=1

Maintenant,
n1
Y
k=1

et donc

1
(2i)n1

n1
Y

eik/n =

k=1

n1
n1


Y
Y
1
1  ik/n
ik/n
2ik/n
e
eik/n =
.
e
1

e
2i
(2i)n1
k=1

k=1

n1

i
= in1 ,
eik/n = e n (1+2+...+(n1)) = ei(n1)/2 = ei/2
1

. Il reste calculer
2n1

n1
Y

(1 e2ik/n ).

k=1

1re solution. Les e2ik/n , 1 6 k 6 n 1,sont les n 1 racines n-mes de 1 distinctes de 1.


Puisque Xn 1 = (X 1) 1 + X + ... + Xn1 , ce sont donc les n 1 racines deux deux distinctes du polynme 1 + X +
... + Xn1 . Par suite,
1 + X + ... + Xn1 =

n1
Y


X e2ik/n = P.

k=1

En particulier

n1
Y

(1 e2ik/n ) = P(1) = 1 + 1... + 1 = n.

k=1

Finalement,
n > 2,

n1
Y

sin

k=1

k
n
= n1 .
n
2

Exercice no 2
Soit n > 2. Tout dabord
n

Q = (1 + X + ... + X ) =

Xn+1 1
X1


(n + 1)Xn (X 1) Xn+1 1
nXn+1 (n + 1)Xn + 1
=
=
.
(X 1)2
(X 1)2

Ensuite, 0 = 1 et donc, Q(0 ) = 1 + 2 + ... + n =


Q(k ) =

n(n + 1)
. Puis, pour 1 6 k 6 n 1, k 6= 1 et donc, puisque n
k = 1,
2

nkn+1 (n + 1)n
nk (n + 1) + 1
n
k +1
=
=
.
(k 1)2
(k 1)2
k 1

Par suite,
n1
Y

Q(k ) =

k=0

n1
n(n + 1) Y
n
=
2
k 1
k=1

nn (n + 1)
.
n1
Y
2
(k 1)
k=1

n1

Mais, X 1 = (X 1) 1 + X + ... + X
dduit que

n1
Y

et dautre part X 1 =

n1
Y

2ik/n

Xe

k=0

= (X 1)

n1
Y

(X k ). On en

k=1

(X e2ik/n ) = 1 + X + ... + Xn1 .

k=1

En particulier,

n1
Y

(1 k ) = 1 + 12 + ... + 1n1 = n ou encore

(k 1) = (1)n1 n. Donc,

k=1

k=1

n1
Y
k=0

http ://www.maths-france.fr

n1
Y

1
(1)n1 nn1 (n + 1)
n (n + 1)
=
.
n1
2
(1)
n
2
n

Q(k ) =

c Jean-Louis Rouget, 2014. Tous droits rservs.


n1
Y

n > 2,

Q(k ) =

k=0

(1)n1 nn1 (n + 1)
.
2

Exercice no 3
1) Soit p N. Pour tout rel a,
ei(2p+1)a = (cos a + i sin a)2p+1 =

2p+1
X 


2p + 1
cos2p+1j a(i sin a)j
j

j=0

puis


i(2p+1)a

sin((2p + 1)a) = Im e


p 
X
2p + 1
2j + 1

j=0

cos2(pj) a(1)j sin2j+1 a.

k
, on obtient :
2p + 1

p 
X
k
2p + 1
k
k J1, pK,
cos2(pj)
(1)j sin2j+1
= 0.
2j + 1
2p + 1
2p + 1

Pour 1 6 k 6 p, en posant a =

j=0

k
k
<
et donc sin2p+1
6= 0. En divisant les deux membres de () par
2p + 1
2
2p + 1

Ensuite, pour 1 6 k 6 p, 0 <


sin2p+1

k
, on obtient :
2p + 1

k J1, pK,

p
X

(1)j

j=0



k
2p + 1
= 0.
cotan2(pj)
2p + 1
2j + 1

k
sont deux deux distincts. En effet, pour 1 6 k 6 p, 0 <
< . Or, sur
Maintenant, les p nombres cotan2
2p + 1
2p + 1
2
i h
0, , la fonction x 7 cotan x est strictement dcroissante et strictement positive, de sorte que la fonction x 7 cotan2 x
2
est strictement dcroissante et en particulier injective.


p
X
2p + 1 pj
Ces p nombres deux deux distincts sont racines du polynme P =
X , qui est de degr p. Ce sont
(1)j
2j + 1
j=0

donc toutes les racines de P (ces racines sont par suite simples et relles). Daprs les relations entre les coefficients et les
racines dun polynme scind, on a :


2p + 1

k
p(2p 1)
3
 =
= 
.
cotan2
2p + 1
2p + 1
3
k=1
1
p
X

puis,
p
X

k
k=1 sin
2p + 1

p 
X
1 + cotan2

k=1

k
2p + 1

=p+

p(2p 1)
2p(p + 1)
=
.
3
3

2) Pour n entier naturel non nul donn, on a


un+1 un =

n+1
X
k=1

X 1
1
1

=
> 0,
2
2
k
k
(n + 1)2
k=1

et la suite (un )nN est strictement croissante. De plus, pour n > 2,


un =


n
n
n 
n
X
X
X
X
1
1
1
1
1
1
= 1 + 1 < 2.
=
1
+
<
1
+
=
1
+

k2
k2
k(k 1)
k1 k
n

k=1

k=2

k=2

k=2

La suite (un ) est croissante et est majore par 2. Par suite, la suite (un ) converge vers un rel infrieur ou gal 2.
http ://www.maths-france.fr

c Jean-Louis Rouget, 2014. Tous droits rservs.


h h
h h
3) Pour x lment de 0, , posons f(x) = x sin x et g(x) = tan x x. f et g sont drivables sur 0,
et pour x
2
2
h h
i h
lment de 0, ), f (x) = 1 cos x et g (x) = tan2 x). f et g sont strictement positives sur 0,
et donc f et g sont
2
2
i h
h h
strictement croissantes sur 0, . Comme f(0) = g(0) = 0, on en dduit que f et g sont strictement positives sur 0, .
2
2
i h
i h
1
1
Donc, x 0,
, 0 < sin x < x < tan x et par passage linverse x 0,
, 0 < cotan x <
<
puis
2
2
x
sin
x
i h
1
1
, cotan2 x < 2 <
x 0,
.
2
x
sin2 x


1

k
2p + 1
k
<
<
et donc cotan2
<
. En sommant ces ingalits, on
4) Pour 1 6 k 6 p, 0 <
k
2p + 1
2
2p + 1
k
sin2
2p + 1
obtient
p
p
p
X
X
p(2p 1) X
k
(2p + 1)2
<
=
<
cotan2
3
2p + 1
2 k2
k=1

k
2
k=1 sin
2p + 1

k=1

2p(p + 1)
,
3

puis
p
X
2 p(2p 1)
1
2p(p + 1)2
<
u
=
<
.
p
2
2
3(2p + 1)
k
3(2p + 1)2
k=1

Les membres de gauche et de droite tendent vers


gendarmes, la suite (up ) tend vers

2
.
6

quand p tend vers linfini et donc, daprs le thorme des


6

+
X
2
1
=
.
2
n
6

n=1

Exercice no 4
X6 7X4 + 8X3 7X + 7 = (X6 + 8X3 + 7) (7X4 + 7X) = (X3 + 1)(X3 + 7) 7X(X3 + 1) = (X3 + 1)(X3 7X + 7).
3X5 7X3 + 3X2 7 = 3X2 (X3 + 1) 7(X3 + 1) = (X3 + 1)(3X2 7). Donc,



PGCD X6 7X4 + 8X3 7X + 7, 3X5 7X3 + 3X2 7 = X3 + 1 PGCD X3 7X + 7, 3X2 7 .
r !3
r !
r
14 7
7
7
Maintenant, pour {1, 1},
+ 7 =
7
+ 7 6= 0.
3
3
3 3

Les polynmes (X3 7X + 7) et (3X2 7) nont pas de racines communes dans C et sont donc premiers entre eux. Donc,

PGCD X6 7X4 + 8X3 7X + 7, 3X5 7X3 + 3X2 7 = X3 + 1.
Exercice no 5
Soit n N.
(X + 1)n Xn 1 est divisible par X2 + X + 1 j et j2 sont racines de (X + 1)n Xn 1
j est racine de (X + 1)n Xn 1

(car (X + 1)n Xn1 est dans R[X])

Si
Si
Si
Si
Si

n 6Z, (j2 )n jn 1 = 3 6= 0.
n 1 + 6Z, (j2 )n jn 1 = j2 j 1 = 0.
n 2 + 6Z, (j2 )n jn 1 = j j2 1 = 2j 6= 0.
n 3 + 6Z, (j2 )n jn 1 = 3 6= 0.
n 4 + 6Z, (j2 )n jn 1 = j2 j 1 = 2j2 6= 0.

http ://www.maths-france.fr

(j + 1)n jn 1 = 0 (j2 )n jn 1 = 0.

c Jean-Louis Rouget, 2014. Tous droits rservs.


Si n 5 + 6Z, (j2 )n jn 1 = j j2 1 = 0.

En rsum, (X + 1)n Xn 1 est divisible par X2 + X + 1 si et seulement si n est dans (1 + 6Z) (5 + 6Z).
Exercice no 6
Soit P un polynme non nul coefficients rels. Pour tout rel x, on peut crire
P(x) =

k
l
Y
Y
(x ai )i
((x zj )(x zj ))j ,
i=1

j=1

o est un rel non nul, k et l sont des entiers naturels, les ai sont des rels deux deux distincts, les i et les i des
entiers naturels et les (x zj )(x zj ) des polynmes deux deux premiers entre eux racines non relles.
l
Y
((x zj )(x zj ))j > 0 (tous les trinmes du second degr considrs tant unitaires
Tout dabord, pour tout rel x,
j=1

sans racines relles.)


Donc, (x R, P(x) > 0) (x R,

k
Y
(x ai )i > 0).
i=1

Ensuite, si x R, P(x) 0, alors lim P(x) > 0 ce qui impose > 0. Puis, si un exposant i est impair, P change de
x+

signe en ai , ce qui contredit lhypothse faite sur P. Donc, > 0 et tous les i sont pairs. Rciproquement, si > 0 et si
tous les i sont pairs, alors bien sr, x R, P(x) > 0.

Posons A =

k
Y

(x ai )i /2 . A est un lment de R[X] car > 0 et car les i sont des entiers pairs. Posons ensuite
i=1

l
l
Y
Y
(x zj )j et Q2 =
(x zj )j . Q1 admet aprs dveloppement une criture de la forme Q1 = B + iC o B et
Q1 =
j=1

j=1

C sont des polynmes coefficients rels. Mais alors, Q2 = B iC puis


P = A2 Q1 Q2 = A2 (B + iC)(B iC) = A2 (B2 + C2 ) = (AB)2 + (AC)2 = R2 + S2 ,
o R et S sont des polynmes coefficients rels.
Exercice no 7
Si P est de degr infrieur ou gal 0, cest clair. Sinon, posons P =

n
X

k=0

P(P(X)) X = P(P(X)) P(X) + P(X) X =

n
X

k=0

n
X

k=1

ak

ak Xk avec n N .


ak (P(X))k Xk + (P(X) X)


(P(X))k Xk + (P(X) X).


Mais, pour 1 6 k 6 n, (P(X))k Xk ) = (P(X) X) (P(X))k1 + X(P(X))k2 + ... + Xk1 est divisible par P(X) X et il
en est donc de mme de P(P(X)) X.
Exercice no 8
1) Posons P =

l
X
i=0

ai Xi o l > 1 et o les ai sont des entiers relatifs avec al 6= 0. Daprs la formule du binme de

Newton, il existe des entiers relatifs Ki , 0 6 i 6 l, tels que


P(n + km) =

l
X
i=0

ai (n + km)i =

l
X

ai (ni + Ki m) =

i=0

l
X

ai ni + Km = m + Km = m(K + 1),

i=0

o K est un entier relatif. P(n + km) est donc un entier relatif multiple de m = P(n).
2) Soit P Z[X] tel que n N, P(n) est premier.
Soit n un entier naturel donn et m = P(n) (donc, m > 2 et en particulier m 6= 0). Pour tout entier relatif k, P(n+ km) est
divisible par m mais P(n + km) est un nombre premier ce qui impose P(n + km) = m. Par suite, le polynme Q = P m
admet une infinit de racines deux deux distinctes (puisque m 6= 0) et est donc le polynme nul ou encore P est constant.
http ://www.maths-france.fr

c Jean-Louis Rouget, 2014. Tous droits rservs.


Exercice no 9
1) Dj, P0 est dans E.
1
(X + 1)...(X + n) et donc, si k est lment de Jn, 1K, Pn (k) = 0 Z.
n!


1
n+k
Si k est un entier positif, Pn (k) = (k + 1)...(k + n) =
Z.
n!
n
Enfin, si k est un entier strictement plus petit que n,


1
k 1
1
Pn (k) = (k + 1)...(k + n) = (1)n (k 1)...(k n) = (1)n
Z.
n!
n!
n
Soit n un naturel non nul. Pn =

Finalement, k Z, Pn (k) Z, ou encore Pn (Z) Z.

2) Une combinaison linaire coefficients entiers relatifs des Pn est donc dans E.
3) Soit P C[X] \ {0} tel que k Z, P(k) Z (si P est nul, P est combinaison linaire coefficients entiers des Pk ).
Puisque k N, deg(Pk ) = k, on sait que pour tout entier naturel n, (Pk )06k6n est une base de Cn [X] (cest--dire que
tout polynme non nul de degr infrieur ou gal n scrit donc de manire unique comme combinaison linaire des Pk ,
0 6 k 6 n).et donc, (Pk )kN est une base de C[X] (cest--dire que tout polynme scrit donc manire unique comme
combinaison linaire des Pk , k N).
Soit n = degP. Il existe n + 1 nombres complexes a0 ,..., an tels que P = a0 P0 + ... + an Pn . Il reste montrer que les ai
sont des entiers relatifs.
La phrase P(1) est dans Z fournit : a0 est dans Z.
La phrase P(2) est dans Z fournit : a0 a1 est dans Z et donc a1 est dans Z.
La phrase P(3) est dans Z fournit : a0 2a1 + a2 est dans Z et donc a2 est dans Z...
La phrase P((k + 1)) est dans Z fournit : a0 a1 + ... + (1)k ak est dans Z et si par hypothse de rcurrence,
a0 ,..., ak1 sont des entiers relatifs alors ak lest encore.
Tous les coefficients ak sont des entiers relatifs et E est donc constitu des combinaisons linaires coefficients entiers
relatifs des Pk .
Exercice no 10
On prend n > 2 (sinon tout est clair).
Q = (X eia )(X eia ) est racines simples si et seulement si eia 6= eia ou encore e2ia 6= 1 ou enfin, a
/ Z.
1er cas. Si a Z alors, P = 0 = 0 Q.
2me cas. Si a
/ Z, daprs la formule de Moivre
P(eia ) = sin a(cos(na) + i sin(na)) sin(na)(cos a + i sin a) + sin((n 1)a)
= sin((n 1)a) (sin(na) cos a cos(na) sin a) = 0.
Donc, eia est racine de P et de mme, puisque P est dans R[X], eia est racine de P. P est donc divisible par Q.
!
n1
X



Xn1k eika sin(na)
P = P P(eia ) = sin a Xn eina sin(na) X eia = X eia sin a
k=0

Puis,


= X eia S.

ia

S = S S(e

) = sin a

n1
X
k=0


= sin a X eia

n2
X
k=0



eika Xn1k ei(n1k)a

eika

n2k
X
j=0

Xn2kj eija

n2
n2
X n2k
X
X
X



= sin a X eia
Xn2kj ei(kj)a = sin a X eia
ei(kj)a Xn2l
k=0

= sin a X eia

http ://www.maths-france.fr

n2
X
l=0

l=0

j=0

l
X

k=0

ei(2kl)a

k+j=l

Xn2l

c Jean-Louis Rouget, 2014. Tous droits rservs.


Maintenant,
l
X

ei(2kl) a = eila

k=0

sin((l + 1)a)
1 e2i(l+1)a
=
.
1 e2ia
sin a

Donc
S = sin a(X eia )

n2
X
l=0

n2

X
sin((l + 1)a) n2l
X
= (X eia )
sin((l + 1)a)Xn2l ,
sin a
l=0

et finalement
P = (X eia )(X eia )

n2
X

sin((k + 1)a)Xn2k = (X2 2X cos a + 1)

n2
X

sin((k + 1)a)Xn2k .

k=0

k=0

Exercice n 11
o

Soit P un polynme de degr n suprieur ou gal 2.


Posons P = (X z1 )(X z2 )...(X zn ) o est un complexe non nul et les zk des complexes pas ncessairement deux
deux distincts.

n
n
X
Y
X
P

P =
,
(X zj ) =
X zi
i=1

j6=i

i=1

et donc

P X 1
.
=
P
X zi
i=1

Soit alors z une racine de P dans C. Si z est racine de P (et donc racine de P dordre au moins 2) le rsultat est clair.
Sinon,
0=

n
n
n
X
X
z zk
z zk
P (z) X 1
=
=
=
2
2
P(z)
z zk
k=1 |z zk |
i=1 |z zk |
k=1

().

1
Pour k J1, nK, on pose k =

|z zk |
n
X

. Chaque k est un rel strictement positif et de plus

|z zj |2

j=1

n
X

k=1

k =

n
X

k=1 |z
n
X

zk |

|z zj |

= 1.

j=1

En conjuguant les deux membres de lgalit (), on obtient


z=

n
X

n
X
j=1

1
|z zj |

k zk .

z =

n
X

k=1

1
|z zk |

z
2 k

et donc,

k=1

Exercice no 12


P = X6 2X3 cos a + 1 = X3 eia X3 eia







= X eia/3 X jeia/3 X j2 eia/3 X eia/3 X jeia/3 X j2 eia/3








a 2
a 2
a
2
2
2
+1
X 2X cos
+1 .
X 2X cos
+

= X 2X cos + 1
3
3
3
3
3
Il reste se demander : 1) si les facteurs prcdents sont irrductibles sur R et 2) si ces facteurs sont deux deux distincts.
http ://www.maths-france.fr

c Jean-Louis Rouget, 2014. Tous droits rservs.


Les trois facteurs de degr 2 ont un discriminant rduit du type = cos2 1 = sin2 et est nul si et seulement
si est dans Z.
a + 2
a 2
a
est dans Z et donc a = ) et (
est dans
Les cas particuliers sont donc ( est dans Z et donc a = 0) et (
3
3
3
Z ce qui na pas de solution dans [0, ]).
1er cas. Si a = 0.
P = (X2 2X + 1)(X2 + X + 1)(X2 + X + 1) = (X 1)2 (X2 + X + 1)2 .
2me cas. Si a = , en remplaant X par X on obtient :
P = (X + 1)2 (X2 X + 1)2 .
3me cas. Si a est dans ]0, [, les trois facteurs de degr 2 sont irrductibles sur R. Dautre part, eia et eia sont distincts
et donc nont pas de racine cubique en commun. Les trois facteurs de degr 2 sont deux deux distincts. Dans ce cas,




a
a + 2
a 2
P = X2 2X cos + 1
X2 2X cos
+1
X2 2X cos
+1 .
3
3
3
Exercice no 13
Soit P un tel polynme. 2 est racine de P + 10 dordre au moins trois et donc racine de (P + 10) = P dordre au moins
deux.
De mme, 2 est racine de P dordre au moins deux et puisque P est de degr 4, il existe un complexe tel que
2

P = (X 2)2 (X + 2)2 = X2 4 = X4 8X2 + 16 et enfin, ncessairement,


1 5 8 3
(, ) C2 / P =
X X + 16X + avec 6= 0.
5
3


1 5 8 3
X X + 16X + avec 6= 0.
Rciproquement, soit P =
5
3
P solution P + 10 divisible par (X + 2)3 et P 10 est divisible par (X 2)3
P(2) + 10 = 0 = P (2) = P (2) et P(2) + 10 = 0 = P (2) = P (2)

P(2) = 10 et P(2) = 10


32 64

32 + = 10
+
3
 5


32
64

+ 32 + = 10

5
3


32 64

+ 32 = 10
= 0 et
5
3
75
= 0 et =
128


75 1 5 8 3
15 5 25 3 75
On trouve un et un seul polynme solution savoir P =
X X + 16X =
X X + X.
128 5
3
128
16
8

Exercice no 14

Les polynmes de degr infrieur ou gal 0 solutions sont 0 et 1 car = {0, 1}.

Soit P un polynme de degr suprieur ou gal 1 tel que P(X2 ) = P(X)P(X + 1).
Soit a une racine de P dans C. Alors, a2 , a4, a8 ..., sont encore racines de P. Mais, P tant non nul, P ne doit admettre
n
quun nombre fini de racines. La suite a(2 ) nN ne doit donc prendre quun nombre fini de valeurs ce qui impose a = 0
n 
n
ou |a| = 1 car si |a| ]0, 1[]1, +[, la suite a(2 ) est strictement monotone et en particulier les a(2 ) sont deux
deux distincts.
De mme, si a est racine de P alors (a 1)2 lest encore mais aussi (a 1)4 , (a 1)8 ..., ce qui impose a = 1 ou |a 1| = 1.
En rsum,
(a racine de P dans C) ((a = 0 ou |a| = 1) et (a = 1 ou |a 1| = 1))
(a = 0 ou a = 1 ou |a| = |a 1| = 1).

http ://www.maths-france.fr

c Jean-Louis Rouget, 2014. Tous droits rservs.


Maintenant, |a| = |a 1| = 1 |a| = 1 et |a| = |a 1| a C((0, 0), 1) med[(0, 0), (1, 0)] = {j, j2}.
Donc, si P R[X] est solution, il existe K, , , , K complexe non nul et , et entiers naturels tels que
P = KX (X 1) (X + j) (X + j2 ) = KX (X 1) (X2 X + 1)
(j et j2 devant avoir mme ordre de multiplicit puisque P est coefficients rels).
Rciproquement, si P = KX (X 1) (X2 X + 1) .
P(X2 ) = KX2 (X2 1) (X4 X2 + 1) = KX2 (X2 1) (X4 + 2X2 + 1 3X2 )

= KX2 (X 1) (X + 1) (X2 3X + 1) (X2 + 3X + 1) ,


et
P(X)P(X + 1) = KX (X 1) (X2 X + 1) K(X + 1) X (X2 + X + 1)
= K2 X+ (X 1) (X + 1) (X2 X + 1) (X2 + X + 1) .
Par unicit de la dcomposition en produit de facteurs irrductibles dun polynme non nul, P est solution si et seulement
si P = 0 ou K = 1 et = et = 0.
Les polynmes solutions sont 0 et les (X2 X) o est un entier naturel quelconque.
Exercice no 15
a est solution du problme si et seulement si X5 209X + a est divisible par un polynme de la forme X2 + X + 1. La
division euclidienne de X5 209X + a par X2 + X + 1 scrit
X5 209X + a = (X2 + X + 1)(X3 X2 + (2 1)X (3 2)) + (4 32 208)X + a + (3 2).
4
32 208 = 0
Donc a est solution C/
. Mais,
a = 3 + 2

4 32 208 = 0 2 {13, 16} 4, 4, i 13, i 13


et la deuxime quation fournit a 56, 56, 15i 13, 15i 13 .
Exercice no 16
On note que P(1) = 1 6= 0 et donc que lexpression propose a bien un sens.

5
5
5 
X
X
ak + 2 X
1
3
P (1)
12
=53
1+
=53
=
=53
= 31.
ak 1
ak 1
1 ak
P(1)
1

k=1

k=1

k=1

Exercice no 17
Notons (S) le systme propos.

x+y+z=1

xy + xz + yz
= 1 1 = 1, 2 = 3 = 4
(S)
xyz

xyz = 4

x, y et z sont les trois solutions de lquation X3 X2 4X + 4 = 0


x, y et z sont les trois solutions de lquation (X 1)(X 2)(X + 2) = 0

Exercice no 18

(x, y, z) {(1, 2, 2), (1, 2, 2), (2, 1, 2), (2, 2, 1), (2, 1, 2), (2, 2, 1)}

Le polynme nul est solution. Un polynme non nul de degr infrieur ou gal 1 ne convient pas.
Soit P un polynme non nul solution de degr n > 2. Alors n = n 1 + n 2 et donc n = 3. Posons P = aX3 + bX2 + cX + d
avec a 6= 0.

http ://www.maths-france.fr

c Jean-Louis Rouget, 2014. Tous droits rservs.


P(2X) = P (X)P (X) 8aX3 + 4bX2 + 2cX + d = (3aX2 + 2bX + c)(6aX + 2b)

(18a2 8a)X3 + (18ab 4b)X2 + (4b2 + 6ac 2c)X + 2bc d = 0

18a2 8a = 18ab 4b = 4b2 + 6ac 2c = 2bc d = 0


4
a = et b = c = d = 0.
9

Les polynmes solutions sont 0 et

4 3
X .
9

Exercice no 19
1)
pn
pn1
p
P(r) = 0 an n + an1 n1 + . . . + a1 + a0 = 0 an pn + an1 pn1 q + . . . + a1 pqn1 + a0 qn = 0
q
q
q


n
n1
an p = q an1 p
. . . a1 pqn2 a0 qn1 
.

a0 qn = p an pn1 an1 pn2 q . . . a1 qn2

La premire galit montre que q divise an pn . Mais q est premier p et donc q est premier pn . Daprs le thorme
de Gauss, q divise an .
De mme, la deuxime galit montre que p divise a0 .
2) 0 nest pas racine de P.
p
Soit r = , (p Z , q N , PGCD(p, q) = 1) une ventuelle racine racine rationnelle de P. Alors, p divise 4
q
et
q
divise
12 et donc, p est lment de {1,

2, 4} et q est lment de {1, 2, 3, 4, 6, 12} ou encore r est lment de
1 1 2 4 1 1
1
1, 2, 4, , , , , , ,
.
2 3 3 3 4 6 12
 
 
1
2
=P
= 0. P est donc divisible par
Rciproquement, on trouve P
3
4



1
2
X
= (3X 2)(4X 1) = 12X2 11X + 2.
12 X
3
4
!
!
 2

1 + i 7
1 i 7
2
Plus prcisment, P = 12X 11X + 2 X + X + 2 = (3X 2)(4X 1) X
X
.
2
2
Exercice no 20

 
1
1
= 0. Pn admet 0, 1 et pour racines et
Pour n > 0, posons Pn = (X 1) X + 2X 1. Pn (0) = Pn (1) = Pn
2
2
est donc divisible par X(X 1)(2X 1) = 2X3 3X2 + X.
2n

2n

Si n = 0 ou n = 1, le quotient est nul. Si n = 2, le quotient vaut 2.


Soit n > 3. On met successivement 2X 1 puis X 1 puis X en facteur :

http ://www.maths-france.fr

c Jean-Louis Rouget, 2014. Tous droits rservs.


Pn = (X 1)2

n

= (2X 1)

X2
n1
X

n

+ (2X 1) = (X 1)2 X2

X
 n1
(X 1)2k X2(n1k) + (2X 1)
k=0

(X 1)2k X2(n1k) + 1

k=0

= (2X 1) (X 1)

n1
X

= (2X 1)(X 1)

n1
X

= (2X 1)

(X 1)2k1 X2(n1k) (X 1)
(X 1)2k1 X2(n1k)

2n1
X

Xk

k=0

n2
X

2k1

(X 1)

2(n1k)

k=1

n2
X

2k1

(X 1)

2(n1k)

k=1

= X(2X 1)(X 1)

2n1
X

Xk

k=0

k=1

= (2X 1)(X 1)

(X 1)2k X2(n1k) + 1 X2n2

k=1

k=1

= (2X 1)(X 1)

n1
X

k=1

n2
X

2k1

(X 1)

2n2k3

k=1

k=1

2n3
X

2n3
X

2n1
X

2n3
X

2n3

1 (X 1)
2n3k

(1)

k=1

k1

k=1

2n3
X


 !
2n 3 k
X
k

2n3k

(1)

Ck2n3 Xk1

k=1

Exercice no 21
1)
2n1
X 


2n 1 k
1 = (X + (1 X))
=
X (1 X)2n1k
k
k=0
n1
2n1
X 2n 1
X 2n 1
=
Xk (1 X)n+m1k +
Xk (1 X)n+m1k
k
k
k=0
k=n


n1
2n1
X 2n 1
X 2n 1
= (1 X)n
Xk (1 X)n1k + Xn
Xkn (1 X)n+m1k
k
k
21

k=0

k=n

2n1
X 


n1
X 2n 1
2n 1 kn
2n1k
Soient U =
X
(1 X)
et V =
Xk (1 X)n1k . U et V sont des polynmes tels que
k
k
k=n
k=0

2n1k
UXn + V(1 X)n = 1. De plus, pour n 6 k 6 2n 1, deg Xkn (1
X)
= k n + 2n 1 k = n 1 < n et donc

k
n1k
deg(U) < n et de mme pour 0 6 k 6 n 1, deg X (1 X)
= k + n 1 k = n 1 < n et deg(V) < n.
2)

n+m1
X 


n+m1 k
X (1 X)n+m1k
k
k=0


n1
n+m1
X n+m1
X n + m 1
=
Xk (1 X)n+m1k +
Xk (1 X)n+m1k
k
k
k=0
k=n
n1
n+m1
X n + m 1
X n + m 1
m
k
n1k
n
= (1 X)
X (1 X)
+X
Xkn (1 X)n+m1k .
k
k

1 = (X + (1 X))n+m1 =

k=0

k=n

n+m1
X 


n1
X  n + m 1
n + m 1 kn
X
(1X)n+m1k et V =
Xk (1X)n1k . U et V sont des polynmes
k
k
k=n
k=0

= kn+n+m1k =
tels que UXn + V(1 X)m = 1. De plus, pour n 6 k 6 n + m 1, deg Xkn (1 X)n+m1k

m 1 < m et donc deg(U) < m et de mme pour 0 6 k 6 n 1, deg Xk (1 X)n1k = k + n 1 k = n 1 < n et
deg(V) < n.
Soient U =

Exercice no 22
http ://www.maths-france.fr

10

c Jean-Louis Rouget, 2014. Tous droits rservs.


Soit n > 2 le degr de P.


1) a) Si P admet n racines relles simples, le thorme de Rolle fournit au moins n 1 racines relles deux deux
distinctes pour P . Mais, puisque P est de degr n 1, ce sont toutes les racines de P , ncessairement toutes relles et
simples.
b) Le rsultat tombe en dfaut si les racines de P ne sont pas toutes relles. Par exemple, P = X3 1 = (X1)(Xj)(Xj2 )
est racines simples dans C mais P = 3X2 admet une racine double.
2) Sparons les racines simples et les racines multiples de P. Posons P = (X a1 )...(X ak )(X b1 )1 ...(X bl )l o les
ai et les bj sont k + l nombres rels deux deux distincts et les j des entiers suprieurs ou gaux 2 (ventuellement
k = 0 ou l = 0 et dans ce cas le produit vide vaut conventionnellement 1).
P sannule dj en k + l nombres rels deux deux distincts et le thorme de Rolle fournit k + l 1 racines relles
deux deux distinctes et distinctes des ai et des bj . Dautre part, les bj sont racines dordre j de P et donc dordre
j 1 de P . On a donc trouv
un nombre
de racines (comptes en nombre de fois gal leur ordre de multiplicit) gal
l
l
X
X
k+l1+
(j 1) = k +
j 1 = n 1 racines relles et cest fini.
j=1

j=1

Exercice no 23

k
(les xk sont deux deux opposs). Il faut calculer les coefficients
Pour k lment de {3, 2, 1, 1, 2, 3}, posons xk = sin
7
du polynme







2
3 

2
3

X sin
X sin
X + sin
X + sin
X + sin
P = X sin
7
7
7
7
7
7







2
3
= X2 sin2
X2 sin2
X2 sin2
7
7
7

 

 



2
1
4
1
6
1
2
2
2
1 cos
X
1 cos
X
1 cos
= X
2
7
2
7
2
7

1
= Q 2X2 + 1
8




4
8
2
Y
cos
Y
cos
Y .
o Q(Y) = cos
7
7
7
Posons = e2i/7 .

cos

Puis,

cos

 1



2
4
6
1
+ 6 2 + 5 3 + 4 =
6 + 7 + 9 + 10 + 11 + 12 + 14 + 15
cos
cos
=
7
7
7
8
8
 1
1
6 + 1 + 2 + 3 + 4 + 5 + 1 + = .
=
8
8







2
4
2
6
6
4
1
+ 6 2 + 5 + + 6 3 + 4 + 3 + 4 2 + 5
cos
+ cos
cos
+ cos
cos
=
7
7
7
7
7
7
4
 2
1
1
2 + 22 + 23 + 24 + 25 + 26 =
= .
=
4
4
2

Enfin,


1
4
6
1
2
+ 2 + 3 + 4 + 5 + 6 =
+ cos
+ cos
=
7
7
7
2
2


 

1
1
1
1
Donc, Q =
Y+
Y2 Y3 =
8Y 3 4Y 2 + 4Y + 1 puis,
8
2
2
8


3
2

1 
1
P=
64X6 112X4 + 56X2 7 .
8 2X2 + 1 4 2X2 + 1 + 4 2X2 + 1 + 1 =
64
64
k
, k {3, 2, 1, 1, 2, 3} est 64x6 112x4 + 56x2 7 = 0.
Une quation du 6me degr dont les solutions sont les sin
7
cos

http ://www.maths-france.fr

11

c Jean-Louis Rouget, 2014. Tous droits rservs.


p
(p entier relatif non nul, q entier naturel non nul, p et q premiers entre eux) est une racine
q
rationnelle de cette quation, alors p divise 7 et q divise 64 et donc p est lment de {1, 1, 7, 7} et q est lment de
{1, 2, 4, 8, 16, 32, 64}. Rciproquement, on vrifie quaucun des rationnels r obtenu nest racine de P et donc les racines de
P sont irrationnelles.
Maintenant, si r =

Exercice no 24
Si (x, y, z) est solution du systme propos not (S), alors x, y et z sont deux deux distincts. En effet, si par exemple
x = y alors 7 = y2 + yz + z2 = x2 + xz + z2 = 13 ce qui est impossible. Donc,
3
y z3 = 7(y z)
(S)
z3 x3 = 13(z x) .
3
x y3 = 3(x y)

En additionnant les trois quations, on obtient 10x + 4y + 6z = 0 ou encore 5x + 2y + 3z = 0. Donc,

y = (5x 3z)

 2
2
1

1
1
y = (5x 3z)

(5x 3z) + (5x 3z)z + z2 = 7


2
2
2
25x2 20xz + 7z2 = 28

(S)

z2 + zx + x2 = 13

z2 + zx + x2 = 13



2

39x2 36xz + 9z2 = 12

1
1

x + (5x 3z)x +
(5x 3z)
=3
2
2

1
y = (5x 3z)

y = (5x 3z)

2
2
xz = 13 x2 z2

xz = 13 x2 z2

2
2
2
2

25x

20(13

z
)
+
7z
=
28

5x2 + 3z2 = 32
39x2 36(13 x2 z2 ) + 9z2 = 12

Soit (S ) le systme form des deux dernires quations. On note que x = 0 ne fournit pas de solution et donc

2x2 + 7


1

2
2
z=
z =
32 5x
3x
3
(S )
 2x2 + 72
1

2
2
1

xz = 13 x
32 5x

= (32 5x2 )
3
2
9x
3

2

34 15
La deuxime quation scrit 2x2 + 7 = 3x2 32 5x2 puis 19x4 68x2 + 49 = 0 puis x2 =
Do les solu19
r
r
49
49
tions x = 1 ou x = 1 ou x =
ou x =
. Les quatre triplets solutions du systme : (1, 2, 3), (1, 2, 3),
19
19




7
1
11
1
11
7
, ,
et , ,
.
19
19
19
19
19
19
Exercice no 25
Posons P = X4 4X3 36X2 + X + .

http ://www.maths-france.fr

12

c Jean-Louis Rouget, 2014. Tous droits rservs.


(, ) solution (a, r) C2 / les

2
(a, r) C /

2
(a, r) C /

2
(a, r) C /

(a, r) C2 /

racines de P soient a, a + r, a + 2r, a + 3r


1
2
3
4

=4
= 36
=
=

4a + 6r = 4
a(3a + 6r) + (a + r)(2a + 5r) + (a + 2r)(a + 3r) = 36
3 =
4 =
2a + 3r = 2
6a2 + 18ra + 11r2 = 36
3 =
4 =
3
a=1 r
2 2



3
3
6 1 r + 18 1 r r + 11r2 = 36
2
2
3 =
4 =

r2 + 42 = 0

2
3
a=1 r
(a, r) C2 /
2

3 =

=
4

21
Do la solution (les deux valeurs opposes de r fournissent videmment la mme progression arithmtique) r = 2
5
r
r
r
r
r
21
21
21
21
21
puis les racines z1 = 1 3
, z2 = 1
, z3 = 1 +
et z4 = 1 + 3
, obtenues pour
puis a = 1 3
5
5
5
5
5
= z1 z2 z3 z4 =

13

21
5

21
5

1+

21
5

1+3

21
5




21
21
2944
19
1
=
,
5
5
25

et
!
r ! 
r ! 
r !
 



21
21
21
21
21
21
21
21
= 13
1
+ 19
1
+ 19
1+
+ 1
1+3
5
5
5
5
5
5
5
5






21
21
21
+2 19
= 2 2 10
= 80
=2 1
5
5
5
r

Exercice no 26
Lquation propose admet deux solutions inverses lune de lautre si et seulement si il existe deux complexes a et b tels
que
X4 21X + 8 = (X2 + aX + 1)(X2 + bX + 8) = X4 + (a + b)X3 + (9 + ab)X2 + (8a + b)X + 8 ()
() b = a et ab = 9 et 8a + b = 21 a = 3 et b = 3. Ainsi,


X4 21X + 8 = X2 3X + 1 X2 + 3X + 8 =

http ://www.maths-france.fr

!
!
!
!
3+ 5
3 5
3 + i 23
3 i 23
X
X
X
X
.
2
2
2
2

13

c Jean-Louis Rouget, 2014. Tous droits rservs.


Exercice no 27
n
n
Pour n N, posons Sn = xn
1 + x2 + x3 . On veut calculer S4 .

Pour i {1, 2, 3}, on a x3i + 2xi 1 = 0 et donc x4i + 2x2i xi = 0. En additionnant ces trois galits, on obtient
S4 + 2S2 S1 = 0 et donc


2
S4 = 2S2 + S1 = 2 (x1 + x2 + x3 ) 2 (x1 x2 + x1 x3 + x2 x3 ) + (x1 + x2 + x3 )

= 2 21 22 + 1 = 2(2 2) = 8.
x41 + x42 + x43 = 8.

Exercice no 28
Soit P un polynme coefficients complexes de degr 4. On suppose P unitaire sans perte de gnralit. On note z1 , z2 ,
z3 et z4 les racines de P dans C.
Si z1 , z2 , z3 et z4 forment un paralllogramme, notons a le centre de ce paralllogramme. Les racines de P scrivent alors
z1 , z2 , 2a z1 , 2a z2 et si Q = P(X + a) alors Q(a + z1 ) = Q(a z1 ) = Q(a + z2 ) = Q(a z2 ) = 0. Les racines
du polynme Q sont deux deux opposes, ce qui quivaut dire que le polynme Q est bicarr ou encore de la forme
X4 + X2 + ou enfin que
P = (X a)4 + (X a)2 + .
Mais alors a est racine de P = 4(X a)3 + 2(X a) et de P(3) = 24(X a).
Rciproquement, si P et P(3) ont une racine commune a. P(3) est de degr 1 et de coefficient dominant 24 et donc
P(3) = 24(X a) puis en intgrant P = 12(X a)2 + puis P = 4(X a)3 + (X a) + . La condition a est racine de P
fournit = 0 et donc P = (X a)4 + (X a)2 + . Donc, le polynme Q = P(X + a) est bicarr et ses racines sont deux
deux opposes et donc de la forme Z1 = a z1 , Z2 = z1 a, Z3 = a z2 , Z4 = z2 a et on a bien Z1 Z3 = Z4 Z2 .
Exercice no 29
n1
Y

Soit P =

(X k ) = Xn 1 (o k = e2ik/n )

k=0

1)

n1
Y
k=0

2
1+
2 k

n1
Y

k=0
n1
Y

(4 k )
=

P(4)
4n 1
= n
= 2n + 1.
P(2)
2 1

(2 k )

k=0

n N ,

n1
Y
k=0

2
1+
2 k

= 2n + 1.

2)
n1
Y
k=0

Y
 n1


2k 2k cos a + 1 =
eia k eia k = P(eia )P(eia ) = (eina 1)(eina 1)
k=0

= 2 eina eina = 2(1 cos na).

n N , a R,

http ://www.maths-france.fr

n1
Y
k=0


2k 2k cos a + 1 = 2(1 cos na).

14

c Jean-Louis Rouget, 2014. Tous droits rservs.


Planche no 27. Fractions rationnelles


* trs facile ** facile *** difficult moyenne **** difficile
I : Incontournable T : pour travailler et mmoriser le cours
Exercice no 1
Dcomposer en lments simples dans C(X) et R(X) les fractions rationnelles suivantes
X2 + 3X + 5
X2 3X + 2
X2 + 1
4)
(X 1)2 (X + 1)2
X6
7) 3
(X 1)2
X6 + 1
10) 5
X X4 + X3 X2 + X 1

1)

X2 + 1
(X 1)(X 2)(X 3)
1
5)
(X 2)3 (X + 2)3
1
8) 6
X +1
X7 + 1
11) 2
(X + X + 1)3
2)

1
X(X 1)2
X3
6) 3
X 1
3)

9)

X2 + 3
X5 3X4 + 5X3 7X2 + 6X 2

Exercice no 2
Dcomposer en lments simples les fractions rationnelles suivantes
1
dans C(X)
Xn 1
n!
dans R(X)
3)
(X 1)(X 2)...(X n)
1
5) 2n
dans C(X) et R(X).
X +1
1)

1
dans C(X)
(X 1)(Xn 1)
2
X
4) 4
dans C(X) et R(X)
X 2X2 cos(2a) + 1

2)

Exercice no 3
Soit Un lensemble des racines n-mes de lunit dans C. Ecrire sous forme dune fraction rationnelle (ou encore rduire
X
X + 1
.
au mme dnominateur) F =
2 X2 + X + 1
Un

Exercice no 4
P
o P et Q sont des polynmes tous deux non nuls et premiers entre eux. Montrer que F est paire si et seulement
Soit F =
Q
si P et Q sont pairs. Etablir un rsultat analogue pour F impaire.
Exercice no 5
Calculer la drive n-me de

X2

1
.
+1

Exercice no 6
Trouver tous les polynmes divisibles par leur drive.
Exercice no 7 (Equations rciproques)
Rsoudre dans C les quations suivantes :
1) z4 + 2z3 + 3z2 + 2z + 1 = 0 en posant Z = z +

1
(ou autrement).
z

2) z6 5z5 + 5z4 5z2 + 5z 1 = 0.


3) z7 z6 7z5 + 7z4 + 7z3 7z2 z + 1 = 0.
Exercice no 8
Soient x1 ,..., x8 les zros de X8 + X7 X + 3. Calculer

http ://www.maths-france.fr

X x1
(168 termes).
x2 x3

c Jean-Louis Rouget, 2014. Tous droits rservs.


Planche no 27. Fractions rationnelles : corrig


Exercice no 1
X2 + 3X + 5
X2 + 3X + 5
=
.
2
X 3X + 2
(X 1)(X 2)
1 et 2 ne sont pas racines du polynme X2 + 3X + 5 et donc F est bien sous forme irrductible. La dcomposition en lment
simples de F scrit
c
b
+
,
F=a+
X1 X2
o a, b et c sont deux rels.
1) Soit F =

a = lim F(x) = 1.
x+

1+3+5
= 9.
12
4+6+5
c = lim (x 2)F(x) =
= 15. Donc,
x2
21

b = lim (x 1)F(x) =
x1

9
15
X2 + 3X + 5
=1
+
.
X2 3X + 2
X1 X2
X2 + 1
. F est sous forme irrductible. La partie entire de F est nulle. La dcomposition en
(X 1)(X 2)(X 3)
lments simples de F scrit sous la forme :
2) Soit F =

F=

a
b
c
+
+
,
X1 X2 X3

o a, b et c sont trois rels.


1+1
= 1.
(1 2)(1 3)
4+1
= 5.
b = lim (x 2)F(x) =
x2
(2 1)(2 3)
9+1
c = lim (x 3)F(x) =
= 5. Donc,
x3
(3 1)(3 2)
a = lim (x 1)F(x) =
x1

1
5
5
X2 + 1
=

+
.
(X 1)(X 2)(X 3)
X1 X2 X3
3) Soit F =

1
. La dcomposition en lments simples de F scrit sous la forme :
X(X 1)2
b
c
a
,
+
F= +
X X 1 (X 1)2

a = lim xF(x) = 1.
x0

c = lim (x 1)2 F(x) = 1.


x1

Enfin, a + b = lim xF(x) = 0 et donc b = 1 (ou bien x = 1 fournit 1


x+

b 1
1
+ = et donc b = 1). Donc,
2 4
4

1
1
1
1
=
.
+
2
X(X 1)
X X 1 (X 1)2
Autre dmarche.
(X 1) + X
1
X1X
1
1
1
=
=
=
+
+
2
2
2
X(X 1)
X(X 1)
X(X 1) (X 1)
X(X 1)
(X 1)2
1
1
1
=
.
+
X X 1 (X 1)2

http ://www.maths-france.fr

c Jean-Louis Rouget, 2014. Tous droits rservs.


4) Soit F =

X2 + 1
X2 + 1
=
. Puisque F est paire, la dcomposition en lments simples de F est de la forme :
(X 1)2 (X + 1)2
(X2 1)2
a
b
b
a

.
+
+
F=
X 1 (X 1)2 X + 1 (X + 1)2

1
x1
2
x = 0 fournit 2a + 2b = 1 et donc a = 0.

b = lim (x 1)2 F(x) =

X2 + 1
1
=
(X 1)2 (X + 1)2
2
5) Soit F =

1
(X

2)3 (X


1
1
.
+
(X 1)2
(X + 1)2

1
. Puisque F est paire, la dcomposition en lments simples de F est de la forme :
4)3
a
c
a
c
b
b
F=
+

.
+
+
X 2 (X 2)2
(X 2)3 X + 2 (X + 2)2
(X + 2)3
=

2)3

c = lim (x 2)3 F(x) =


x2

1
F
64

(X2

1
puis,
64

1
1

(X 2)3
(X + 2)3

64 (X + 2)3 + (X 2)3
12X2 + 48
=
64(X 2)3 (X + 2)3
64(X 2)3 (X + 2)3

X2 4
1
3
3
=
16 (X 2)3 (X + 2)3
16 (X 2)2 (X + 2)2



1
1
3
1
1
3
=
b = lim (x 2)2 F(x)

=
.
x2
64 (x 2)3
(x + 2)3
16 (2 + 2)2
256
1
3
1
1
5
3
Enfin, x = 0 fournit
= a

et a =

=
. Donc,
64
512 256
64 512
512


3
1
6
8
3
6
8
1
=

.
(X 2)3 (X + 2)3
512 X 2 (X 2)2
(X 2)3
X + 2 (X + 2)2
(X + 2)3
=


X3
. On a dj X3 1 = (X 1)(X j) X j2 .
3
X 1
Puisque F est relle, la dcomposition en lments simples de F sur C scrit
6) Soit F =

F=1+

b
a
b
.
+
+
X 1 X j X j2

1
13
= .
3 12
3
13
j
b= 2 = .
3j
3
a=

La dcomposition en lments simples de F sur C est


1
X3
=1+
X3 1
3


1
j
j2
.
+
+
X 1 X j X j2

Dcomposition sur R.
1re mthode. On utilise la dcomposition sur C :
1
j
j2
+
+
X 1 X j X j2


1
1
X+2
=1+
.

3 X 1 X2 + X + 1

1
X3
=1+
X3 1
3

http ://www.maths-france.fr

1
=1+
3

!

j X j2 + j2 (X j)
1
+
X1
(X j) (X j2 )

c Jean-Louis Rouget, 2014. Tous droits rservs.


X3
1
=1+
3
X 1
3


1
X+2
.

X 1 X2 + X + 1

2me mthode. On trouve directement la dcomposition sur R sans passer par C. Puisque X3 1 = (X 1)(X2 + X + 1)
et que le trinme X2 + X + 1 na pas de racine relle, la dcomposition sur R scrit
F=1+
a=

bX + c
a
+
.
X 1 X2 + X + 1

13
1
= .
2
31
3

j3
j2 1
1 j 1
2 1
z3
=
=
= 3
= j.
2
2
zj
zj z 1
j1
(j 1)(j 1)
j jj +1
3 3
1
2
Puisque j nest pas rel, on en dduit que c = et b = et on retrouve le rsultat prcdent.
3
3
2
X6
7) Soit F = 3
. On a dj (X3 1)2 = (X 1)2 (X j)2 X j2 .
2
(X 1)
c + bj = lim (z2 + z + 1)F(z) = lim

Puisque F est relle, la dcomposition en lments simples de F sur C scrit


F = 1+
b = lim (z 1)2 F(z) =
z1

c
b
d
a
c
d
+
+
+
.
+
+
2
2
2
X 1 (X 1)
X j (X j)
Xj
(X j2 )2

16
1
=
(12 + 1 + 1)2
9

Ensuite
d = lim (z j)2 F(z) =
zj

1
j2 (3j)2

j6
(j

1)2 (j

j2 )2

j2 (j

1
1
=
2
4
1)
j2 (j2 2j + 1)

j
.
9

Puis,
1
j2
j
1
(j + j2 )X2 2(j + j2 )X + 2
+
+
=
+
2
2
2
2
2
(X 1)
(X j)
(X j )
(X 1)
(X j)2 (X j2 )
X2 + 2X + 2
(X2 + X + 1)2 + (X 1)2 (X2 + 2X + 2)
1
+
=
=
(X 1)2
(X j)2 (X j2 )
(X3 1)2



4
3
2
2
X + 2X + 3X + 2X + 1 + X 2X + 1 X2 + 2X + 2
=
(X3 1)2
3
6X + 3
.
= 3
(X 1)2
Par suite,
1
F1
9

1
j2
j
+
+
(X 1)2
(X j)2 (X j2 )2

X6
2X3 + 1

(X3 1)2
3(X3 1)2

4X3 4
3X6 3(X3 1)2 2X3 1
=
3(X3 1)2
3(X3 1)2
4
1
= 3
.
3 X 1
=

Mais alors, a =
Donc,

4
4j
1
4
1
4
= . De mme, c = 2 = .

3 3 12
9
3 3j
9
X6

1
=1+
2
3
9
(X 1)
http ://www.maths-france.fr


4
4j
4j2
j
1
j2
.
+
+
+
+
+
X 1 (X 1)2
X j (X j)2 X j2 (X j2 )2
3

c Jean-Louis Rouget, 2014. Tous droits rservs.


Dcomposition sur R. On regroupe les conjugus.


4
4j(X j2 ) + 4j2 (X j) j2 (X j2 )2 + j(X j)2
1
+
+
+
X 1 (X 1)2
X2 + X + 1
(X2 + X + 1)2


2
4
4X 8
1
X + 2X + 2
1
+ 2
+
+ 2
=1+
2
9 X 1 (X 1)
X + X + 1 (X + X + 1)2


1
4
4X 8
1
X2 X 1 + 3X + 3
=1+
+ 2
+
+
9 X 1 (X 1)2
X +X+1
(X2 + X + 1)2


4
4X + 9
1
3X + 3
1
.
2
+
+
=1+
9 X 1 (X 1)2
X + X + 1 (X2 + X + 1)2

1
F=1+
9

X6
(X3 1)

8) Soit F =

=1+

1
9


4
4X + 9
1
3X + 3
.

+
+
X 1 (X 1)2
X2 + X + 1 (X2 + X + 1)2

5
X
1
k
2k

o k = ei( 6 + 6 ) . De plus, pour k J0, 5K,


.
F
=
X6 + 1
X k
k=0

k =

1
k
k
.
=
=
5
6
6
6k
6k

Donc,
1
1
=
6
X +1
6


ei/6
ei/6
ei/6
i
i
ei/6

+
+
.
+

X i X + i X ei/6 X ei/6
X + ei/6
X + ei/6

Dcomposition sur R. On utilise la dcomposition sur C.


1
1
=
X6 + 1
6
=

1
6

1
6


i
i
ei/6
ei/6
ei/6
ei/6
+

+
+
X i X + i X ei/6
X ei/6 X + ei/6
X + ei/6

!


ei/6 X + ei/6 + ei/6 X + ei/6
ei/6 X ei/6 ei/6 X ei/6
2




+
+
X2 + 1
X ei/6 X ei/6
X + ei/6 X + ei/6
!

2
3X + 2
3X + 2

+
.
+
X2 + 1 X2 2 3X + 1 X2 + 2 3X + 1

1
1
=
6
X +1
6

9) Soit F =

2
3X + 2
3X + 2

+
.
+
X2 + 1 X2 2 3X + 1 X2 + 2 3X + 1

X2 + 3
.
X5 3X4 + 5X3 7X2 + 6X 2



X5 3X4 + 5X3 7X2 + 6X 2 = (X 1) X4 2X3 + 3X2 4X + 2 = (X 1)2 X3 X2 + 2X 2


= (X 1)2 X2 (X 1) + 2(X 1) = (X 1)3 X2 + 2 .

La dcomposition en lments simples de F sur C est donc de la forme


F=

c
d
b
d
a
+
.
+
+
+
X 1 (X 1)2
(X 1)3 X i 2 X + i 2

Puis,
 2

i 2 +3

1
1
 =

d = lim z i 2 F(z) = 
3 
 =  

4 + 10i 2
zi 2
2i 2 2i 2 + 6 + 3i 2 1
i 21
i 2+i 2

2 + 5i 2
.
=
108
http ://www.maths-france.fr

c Jean-Louis Rouget, 2014. Tous droits rservs.


Ensuite,

 



2
X
+
i
2
+
2

5i
2
X

i
2
2
+
5i
d
d
1 4X 20
X + 5
1
+
=
=
=
.
2
2
108
X +2
108 X + 2
27(X2 + 2)
Xi 2 X+i 2
Mais alors,
c
X2 + 3
b
X + 5
a
+
=
+

2
3
3
2
X 1 (X 1)
(X 1)
(X 1) (X + 2) 27 (X2 + 2)


27X2 + 81 + (X 5) X3 3X2 + 3X 1
27 X2 + 3 (X + 5)(X 1)3
=
=
27(X 1)3 (X2 + 2)
27(X 1)3 (X2 + 2)


X2 + 2 X2 8X + 43
X4 8X3 + 45X2 16X + 86
X2 8X + 43
=
=
=
27(X 1)3 (X2 + 2)
27(X 1)3 (X2 + 2)
27(X 1)3


2
1
1
36
6
X 2X + 1 6X + 6 + 36
.
=
+

=
27(X 1)3
27 X 1 (X 1)2
(X 1)3
Finalement, la dcomposition en lments simples de F sur C est
1
F=
27

1
36
6
+

2
X 1 (X 1)
(X 1)3

108

!
2 + 5i 2 2 5i 2
+

,
Xi 2
X+i 2

et la dcomposition en lments simples de F sur R est


F=

10) Soit F =

1
27


1
36
X + 5
6
.
+
+

X 1 (X 1)2
(X 1)3
X2 + 2

X6 + 1
.
X5 X4 + X3 X2 + X 1


X5 X4 + X3 X2 + X 1 = X4 (X 1) + X2 (X 1) + (X 1) = (X 1) X4 + X2 + 1




2
= (X 1) X4 + 2X2 + 1 X2 = (X 1) X2 + 1 X2


= (X 1)(X2 + X + 1)(X2 X + 1) = (X 1)(X j) X j2 (X + j) X + j2 .

Puisque F est relle, la dcomposition en lments simples de F sur C est de la forme


F = aX + b +

d
e
c
e
d
+
.
+
+
+
X 1 X j X j2 X + j X + j2

x5 ...
F(x)
= 1, puis b = lim (F(x) x) = lim 5 = 1.
x+
x+ x ...
x+ x
16 + 1
2
c=
= ,
5 14 4 13 + 3 12 2 1 + 1
3
2
2
1
j6 + 1
= 2
=
=
d= 4
5j 4j3 + 3j2 2j + 1
3j + 3j 3
6
3
2
1
2j2 + 1
(j)6 + 1
= 2
=
=
. Donc,
e=
4
3
2
5(j) 4(j) + 3(j) 2(j) + 1
3j + 7j + 5
2j + 1
3
a = lim

X6 + 1
1
= X+1+
5
4
X X + X3 X2 + X 1
3

11) Soit F =


2
2j2 + 1
1
1
2j + 1
.
+

+
X 1 X j X j2
X+j
X j2

X7 + 1
. La dcomposition sur R sobtient de la faon suivante
(X2 + X + 1)3

http ://www.maths-france.fr

c Jean-Louis Rouget, 2014. Tous droits rservs.


X7 + 1
3

(X2 + X + 1)

=
=
=

X7 + X6 + X5 X6 X5 X4 + X4 + X3 + X2 X3 X2 X + X + 1
3

(X2 + X + 1)
 2

5
4
2
X X +X X X +X+1 +X+1
(X2 + X + 1)

X5 X4 + X2 X
2

(X2 + X + 1)

X+1
(X2 + X + 1)

X5 + X4 + X3 2X4 2X3 2X2 + X3 + X2 + X + 2X2 + 2X + 2 4X 2


(X2

+ X + 1)

X+1
(X2

+ X + 1)

4X + 2
X3 2X2 + X + 2
X+1

+
2
3
2
2
X2 + X + 1
(X + X + 1)
(X + X + 1)
4X + 2
X+1
X3 + X2 + X 3X2 3X 3 + 3X + 5

+
=
2
3
2
X +X+1
(X2 + X + 1)
(X2 + X + 1)
4X + 2
3X + 5
X+1

= X3+ 2
+
.
X + X + 1 (X2 + X + 1)2 (X2 + X + 1)3

Donc,

(X2

X+1
3X + 5
4X + 2
X7 + 1
+
.
=X3+ 2

+ X + 1)3
X + X + 1 (X2 + X + 1)2 (X2 + X + 1)3

Exercice no 2
1
. La partie entire de F est nulle et les ples de F sont simples (car P = Xn 1 et P = nXn1
P
n1
n1
Y
X k
nont pas de racines communes dans C). De plus, P =
(X k ) o k = e2ik/n . Donc, F =
. Pour tout
X k
k=0
k=0
k J0, n 1K,
1) Soient P = Xn 1 puis F =

k =

1
k
k
1
=
=
.
=
P (k )
nn
n
nkn1
k

Ainsi,
n N ,

n1
1
1 X e2ik/n
=
.
Xn 1
n
X e2ik/n
k=0

n1
Y

1
2) Soit P = (X 1)(Xn 1) = (X 1)2 Xn1 + . . . + X + 1 = (X 1)2
k o k = e2ik/n . Soit F = . La partie
P
k=1

entire de F est nulle. Dautre part, F admet un ple double, savoir 1 et n 1 ples simples savoir les k = e2ik/n ,
1 6 k 6 n 1. Donc,
F=

n1
X k
a
b
+
.
+
2
X 1 (X 1)
X k
k=1

1
k
1
=
=
.
n1
n1
n(
(n + 1)n

1
n(1

)
k 1)
k
k
k
1
1
= .
b = lim (x 1)2 F(x) = n1
x1
1
+ ... + 11 + 1
n
Il reste calculer a.
Pour k J1, n 1K, k =


n Xn1 + Xn2 + ... + X + 1
Xn2 2Xn3 ... (n 2)X (n 1)
1
=
=
.
F
2
2
n1
n(X 1)
n(X 1) (X
+ ... + X + 1)
n(X 1)(Xn1 + ... + X + 1)


[(n 1) + (n 2) + ... + 2 + 1]
1
n1
=
Donc, a = lim (x 1) F(x)
=
. Finalement,
2
x1
n(x 1)
n(1 + 1... + 1)
2n
http ://www.maths-france.fr

c Jean-Louis Rouget, 2014. Tous droits rservs.


1
1
n N ,
=
(X 1) (Xn 1)
n

!
n1
X k
n1
1
1

.
+
+
2n(X 1) (X 1)2
k 1 X k
k=1

n
X
n!
k
3)
=
avec
(X 1)...(X n)
Xk
k=1

n!

k = lim (x k)F(x) = Y
xk
(j k)



n!
nk n 1
=
.
= n(1)
(1)nk (k 1)!(n k)!
k1

j6=k

Donc,


n1
n
X
n!
k1
n N ,
=n
(1)nk
.
(X 1)...(X n)
Xk
k=1

4) Posons P = X4 2X2 cos(2a) + 1 puis F =

X2
.
P






X2 e2ia = X eia X eia X + eia X + eia


= X2 2X cos a + 1 X2 + 2X cos a + 1 .

X4 2X2 cos(2a) + 1 = X2 e2ia

P est racines simples si et seulement si eia 6= eia ce qui quivaut a


/

Z.
2

1er cas. Si a Z, puisque F est paire


F=

12
1
1
= puis x = 0 fournit 0 = 2A + 2B et donc A = B = .
(1 + 1)2
4
4

B = lim (x 1)2 F(x) =


x1

Si a Z,
2me cas. Si a

1
X2
X2
=
= 2
4
2
X 2X cos(2a) + 1
(X 1)2
4


1
1
1
1
.

+
+
X 1 (X 1)2
X + 1 (X + 1)2

+ Z,
2
F=

B = lim (x i)2 F(x) =


xi

A
A
B
B
X2
=

.
+
+
2
2
2
(X 1)
X 1 (X 1)
X + 1 (X + 1)2

A
B
A
B
X2
=
+

+
.
2
2
+ 1)
X i (X i)
X + i (X + i)2

(X2

1
i
i2
= puis x = 0 fournit 0 = 2iA 2A et donc A = iB = .
2
(i + i)
4
4

1
X2
X2
Si a + Z, 4
=
=
2
X 2X2 cos(2a) + 1
(X2 + 1)2
4


i
i
1
1
.
+
+
+
X i (X i)2 X + i (X + i)2

Pour obtenir la dcomposition sur R, on crit directement

F=

Si a

http ://www.maths-france.fr

X2 + 1 1
1
1
.
= 2

2
2
(X + 1)
X + 1 (X2 + 1)2

X2
X2
1
1

.
+ Z, 4
= 2
= 2

2
2
X 2X cos(2a) + 1
(X + 1)2
X + 1 (X2 + 1)2

c Jean-Louis Rouget, 2014. Tous droits rservs.


3me cas. Si a
/

Z, puisque F est relle et paire,


2
F=

A
A
A
A
+

,
ia
ia
ia
Xe
Xe
X+e
X + eia

avec
e2ia
ieia
e2ia
=
=
.
(eia eia )(eia + eia )(eia + eia )
8i sin a cos a eia
4 sin(2a)

A=
Donc,
Si a
/

X2
1
Z, 4
=
2
2
X 2X cos(2a) + 1
4 sin(2a)



ieia
ieia
ieia
ieia

.
+
+
+
X eia X eia X + eia X + eia

Pour obtenir la dcomposition sur R, on regroupe les conjugus





!
ieia X eia + ieia X eia
ieia X + eia + ieia X + eia
+
(X eia ) (X eia )
(X + eia ) (X + eia )


2X sin(a)
2X sin(a)
1
2
=
2
4 sin(2a) X 2X cos(a) + 1 X + 2X cos(a) + 1


1
X
X
=
.

2 cos(a) X2 2X cos(a) + 1 X2 + 2X cos(a) + 1

1
F=
4 sin(2a)

X2
1
Si a
/ Z, 4
=
2
X 2X2 cos(2a) + 1
2 cos(a)

5) Le polynme X2n + 1 =

2n1
Y 

X ei( 2n +

k=0

+ 2k
i( 2n
2n )

drive. En posant k = e

2k
2n

= ei( 2n +

k
n


X
X
.

X2 2X cos(a) + 1 X2 + 2X cos(a) + 1

est racines simples car na pas de racine commune dans C avec sa

) , on a

2n1
X
k
1
,
=
2n
X +1
X k
k=0

o
k =

1
k
k
.
=
=
2n
2n2n
2nk2n1
k

Finalement,
n N ,

Pour dcomposer

k
2n1
1
1 X
ei( 2n + n )
.
=

+ k
i( 2n
X2n + 1
2n
n )
k=0 X e

1
sur R, on regroupe les conjugus. Pour k J0, 2n 1K,
X2n + 1
(2n1k)

k
n
+ k
) = ei( 2n
n
n ) = ei( 2n + n ) = .
2n1k = ei( 2n +
k

Donc

n1 
n1
1
1 X k (X k ) + k (X k )
1 X
k
k
=

+
=

X2n + 1
2n
X k X k
2n
(X k ) (X k )
k=0
k=0


k

X+1
+
cos
n1
X
1
2n
n


=
.

k
n
k=0 X2 2 cos
X+1
+
2n
n
http ://www.maths-france.fr

c Jean-Louis Rouget, 2014. Tous droits rservs.



k

X+1
+
cos
n1
1
1 X
2n
n


n N , 2n
=
.

k
X +1
n
k=0 X2 2 cos
X+1
+
2n
n


Exercice no 3
Pour k J0, n 1K, posons k = e2ik/n . Dcomposons F en lments simples (sur C).
X + 1
X + 1
X + 1
a
b
,
=
=
=
+
2
2
+ X + 1
(X) + X + 1
(X j)(X j )
X j X j2

2 X2

j
+1
j+1
j2
j
j2 + 1
1

avec a =
=
=
=
et de mme b = 2
=
. Donc,
2
2
j
j

j
j

j
j

1
j

j
j

j2



n1 
n1 
1 X
k
1
j
jk
1 X
=

F=
j1
k X j k X j 2
j1
X jk X j2 k
k=0
k=0

n1 
k
1 X
jk
.

=
j1
X jk X j2 k
k=0

Maintenant les n nombres jk sont deux deux distincts et vrifient (jk )n = jn et donc,
n1
Y

(X jk ) = Xn jn .

k=0
n1
X

jk
P
est donc la dcomposition en lments simples dune fraction du type n
avec degP 6 n 1. De plus,
X jk
X jn
k=0
P (jk )
par unicit de la dcomposition dune fraction rationnelle en lments simples sur C, on sait que jk =
et
n1
n (jk )
donc, k {0, ..., n 1}, P(jk ) = njn . Le polynme P njn est de degr infrieur ou gal n 1, admet les n racines
deux deux distinctes jk et est donc le polynme nul. Par suite
n1
X
k=0

De mme,

n1
X
k=0

njn
jk
= n
.
X jk
X jn

k
nj2n2
=
, puis
X j 2 k
Xn j2n
F=

n
j1

jn
j2n2
n
n
n
X j
X j2n

Si n 3N, posons n = 3p o p N. Dans ce cas,




3p
1
j
3p
=
.

F=
3p
3p
j1 X 1 X 1
1 X3p
Si n 3N + 1, posons n = 3p + 1 o p N. Dans ce cas,



(3p + 1) X3p+1 + 1
j
1
3p + 1
= 6p+2

.
F=
j1
X3p+1 j X3p+1 j2
X
+ X3p+1 + 1
Si n 3N + 2, posons n = 3p + 2 o p N. Dans ce cas,


3p + 2
3p + 2
j2
j2
F=
= 6p+4

.
3p+2
2
3p+2
j1
X
j
X
j
X
+ X3p+2 + 1
Exercice no 4
Soient P et Q deux polynmes non nuls et premiers entre eux, puis soit F =

P(X)
P(X)
P
. Si F est paire, alors
=
, ou
Q
Q(X)
Q(X)

encore P(X)Q(X) = P(X)Q(X) ().


http ://www.maths-france.fr

c Jean-Louis Rouget, 2014. Tous droits rservs.


Par suite, P(X) divise P(X)Q(X) = Q(X)P(X) et P(X) est premier Q(X). Daprs le thorme de Gauss, P(X) divise
P(X). Donc, il existe C tel que P(X) = P(X) (car deg(P(X)) = deg(P)). Lanalyse des coefficients dominants
des deux membres fournit = (1)n o n = degP. Ceci scrit P(X) = (1)n P(X). En reportant dans (), on obtient
encore Q(X) = (1)n Q(X). Ainsi, si F est paire, alors P et Q sont ou bien tous deux pairs, ou bien tous deux impairs.
Ce dernier cas est exclu, car alors P et Q admettraient tous deux 0 pour racine contredisant le fait quils sont premiers
entre eux. Finalement, si F est paire, alors P et Q sont pairs. La rciproque est claire.
F paire (P et Q sont pairs.)

Je vous laisse tablir que

F impaire (P est impair et Q est pair) ou(P est pair et Q est impair.)

Exercice no 5

1
1
=
Soit n N . 2
X +1
2i

1
2
X +1

(n)


1
1
. Donc,

Xi X+i

(n) #


1 (1)(2)...(n) (1)(2)...(n)
1
=

X+i
2i
(X i)n+1
(X + i)n+1




(X + i)n+1
1
n
=
(1)
n!
Im
= (1)n n! Im
(X i)n+1
(X2 + 1)n+1


X
n+1
X2n2k
(1)k
2k + 1
06k6n
.
= (1)n n!
(X2 + 1)n+1
1
=
2i

"

1
Xi

(n)

Exercice no 6
n
Y
Si P =
(X zk ), on sait que
k=1
n
X
P
1
.
=
P
X xk
k=1

Si maintenant, on regroupe les ples identiques ou encore si on pose P = (X z1 )1 ...(X zk )k o cette fois-ci les zj
sont deux deux distincts. La formule ci-dessus scrit alors
k

P X j
=
P
X zj

().

j=1

Dterminons alors les polynmes divisibles par leur drive. Soit P un tel polynme. Ncessairement degP > 1 puis, il
1
P
=
. () montre que P a une et une seule
existe deux complexes a et b, a 6= 0 tel que P = (aX + b)P ou encore
P
aX + b
n
racine. Par suite, P est de la forme (X a) , 6= 0, n > 1 et a quelconque.


1
a
1
n1
Rciproquement, on a dans ce cas P = (X a) n(X a)
P et P divise effectivement P.
=
X
n
n
n

Les polynmes divisibles par leur drive sont les polynmes de la forme (X a)n , C , n N et a C.
Exercice no 7
1)

!




2



1
1
1
1
2
2
X + 2X + 3X + 2X + 1 = X X + 2 + 2 X +
+3 =X
X+
+1
+2 X+
X
X
X
X
2

2
2
1
= X2 + X + 1 = (X j)2 X j2 .
= X2 X + + 1
X
4

2) Soit P = X6 5X5 + 5X4 5X2 + 5X 1. 1 et 1 sont racines de P. On crit donc P = (X2 1)(X4 5X3 + 6X2 5X + 1)
puis

http ://www.maths-france.fr

10

c Jean-Louis Rouget, 2014. Tous droits rservs.


!




2




1
1
1
1
2
2
X + 2 5 X+
X 5X + 6X 5X + 1 = X
+6 =X
X+
+4
5 X+
X
X
X
X





1
1
X + 4 = X2 X + 1 X2 4X + 1
= X2 X + 1
X
X



et donc, P = (X 1)(X + 1) (X + j) X + j2 X 2 + 3 X 2 3 .
4

3)



P = X7 X6 7X5 + 7X4 + 7X3 7X2 X + 1 = X2 1 X5 X4 6X3 + 6X2 + X 1

= (X 1)2 (X + 1) X4 6X2 + 1


  2 

= (X 1)2 (X + 1) X2 3 + 2 2
X 32 2
Les racines de P dans C sont 1 (dordre 2), 1,
Exercice no 7

p
p
p

p
3 + 2 2, 3 + 2 2, 3 2 2 et 3 2 2.

 
7
Pour chacun des 8 numrateurs possibles, il y a
= 21 dnominateurs et donc au total, 8 21 = 168 termes.
2
X x2 x4 x5 x6 x7 x8
X x1
1X 2
1 X 2
1
x1 x2 x3 x4 x5 x6 =
=
=
x1 x2 x3 x4 x5 x6 .
x2 x3
x1 x2 ...x8
8
3
Ensuite,
1 6 =

x1

x1 x2 x3 x4 x5 x6

x21 x2 x3 x4 x5 x6 +

x1 x2 x3 x4 x5 x6 x7 ,

et donc,
X
Donc,

x21 x2 x3 x4 x5 x6 = 1 6 7 = (1) 0 1 = 1.

X x1
1
= .
x2 x3
3

http ://www.maths-france.fr

11

c Jean-Louis Rouget, 2014. Tous droits rservs.


Planche no 28. Espaces vectoriels


* trs facile ** facile *** difficult moyenne **** difficile
I : Incontournable T : pour travailler et mmoriser le cours
Exercice no 1 : (*T)
Soit E le R-espace vectoriel des applications de [0, 1] dans R muni des oprations usuelles. Soit F lensemble des applications
de [0, 1] dans R vrifiant lune des conditions suivantes :
 
1
1
4) x [0, 1], f(x) + f(1 x) = 0
=
1) f(0) + f(1) = 0
2) f(0) = 0
3) f
2
4
5) x [0, 1], f(x) > 0 6) 2f(0) = f(1) + 3
Dans quel cas F est-il un sous-espace vectoriel de E ?
Exercice no 2 : (**T)
On munit Rn des lois usuelles. Parmi les sous-ensembles suivants F de Rn , lesquels sont des sous-espaces vectoriels ?
1) F = {(x1 , ..., xn ) Rn / x1 = 0}
3) F = {(x1 , ..., xn ) Rn / x1 = x2 }
5) F = {(x1 , ..., xn ) Rn / x1 x2 = 0}

2) F = {(x1 , ..., xn ) Rn / x1 = 1}
4) F = {(x1 , ..., xn ) Rn / x1 + ... + xn = 0}

Exercice no 3 : (**)
Soit E un K-espace vectoriel. Soient A, B et C trois sous-espaces vectoriels de E vrifiant A B = A C, A + B = A + C
et B C. Montrer que B = C.
Exercice no 4 : (**T)
Soit E = RN le R-espace vectoriel des suites relles (muni des oprations usuelles). On considre les trois lments de E
suivants : u = (cos(n))nN , v = (cos(n + a))nN et w = (cos(n + b))nN o , a et b sont des rels donns. Montrer
que (u, v, w) est une famille lie.
Exercice no 5 : (**T)
Soit F le sous-espace vectoriel de R4 engendr par u = (1, 2, 5, 3) et v = (2, 1, 4, 7). Dterminer et rels tels que
(, , 37, 3) appartienne F.
Exercice no 6 : (**T)
Montrer que a = (1, 2, 3) et b = (2, 1, 1) engendrent le mme sous-espace de R3 que c = (1, 0, 1) et d = (0, 1, 1).
Exercice no 7 : (**T)
1) Vrifier quil existe une unique application linaire de R3 dans R2 vrifiant f((1, 0, 0)) = (1, 1) puis f((0, 1, 0)) = (0, 1)
et f((0, 0, 1)) = (1, 1). Calculer f((3, 1, 4)) et f((x, y, z)) en gnral.
2) Dterminer Kerf. En fournir une base. Dterminer Imf.
Exercice no 8 : (**I)
Soit E un K-espace vectoriel et f un lment de L (E).
1) Montrer que [Kerf = Kerf2 Kerf Imf = {0}] et [Imf = Imf2 E = Kerf + Imf] (o f2 = f f).
2) Par dfinition, un endomorphisme p de E est un projecteur si et seulement si p2 = p. Montrer que
[p projecteur Id p projecteur]

puis que

[p projecteur Imp = Ker(Id p) et Kerp = Im(Id p) et E = Kerp Imp].

3) Soient p et q deux projecteurs, montrer que : [Kerp = Kerq p = p q et q = q p].

4) p et q tant deux projecteurs vrifiant p q + q p = 0, montrer que p q = q p = 0. Donner une condition ncessaire
et suffisante pour que p + q soit un projecteur lorsque p et q le sont. Dans ce cas, dterminer Im(p + q) et Ker(p + q) en
fonction de Kerp, Kerq, Imp et Imq.

http ://www.maths-france.fr

c Jean-Louis Rouget, 2014. Tous droits rservs.


Exercice no 9 : (**)
Soient E un K-espace vectoriel et A, B et C trois sous-espaces de E.
1) Montrer que : (A B) + (A C) A (B + C).
2) A-t-on toujours lgalit ?
3) Montrer que : (A B) + (A C) = A (B + (A C)).
Exercice no 10 : (**T)
Dans E = R4 , on considre V = {(x, y, z, t) E/ x 2y = 0 et y 2z = 0} et W = {(x, y, z, t) E/ x + z = y + t}.
1) Montrer que V et W sont des sous espaces vectoriels de E.
2) Donner une base de V, une base de W et une base de V W.
3) Montrer que E = V + W.
Exercice no 11 : (***)
Soit C lensemble des applications de R dans R, croissantes sur R.
1) C est-il un espace vectoriel (pour les oprations usuelles) ?
2) Montrer que V = {f RR / (g, h) C2 tel que f = g h} est un R-espace vectoriel.
Exercice no 12 : (**)
Montrer que la commutativit de la loi + est une consquence des autres axiomes de la structure despace vectoriel.
Exercice no 13 : (***)
Soient E un K-espace vectoriel et A, B et C trois sous-espaces vectoriels de E. Montrer que
(A B) + (B C) + (C A) (A + B) (B + C) (C + A).
Exercice no 14 : (**IT)
Soient F = {(, , . . . , ), R} puis G = {(x1 , ..., xn ) Rn / x1 + ... + xn = 0}. Montrer que F et G sont des sous-espaces
vectoriels de Rn et que Rn = F G.
Exercice no 15 : (****)

1) Soit n un entier naturel. Montrer que si n nest pas un carr parfait alors n
/ Q.

4
2) Soit E = {a + b 2 + c 3 + d 6, (a, b, c, d) Q }. Vrifier que E est un Q-espace vectoriel puis dterminer une base
de E.
Exercice no 16 : (***T)
Dans E = RR , tudier la libert des familles suivantes A de vecteurs de E :
1) a, b et c tant trois rels deux deux distincts donns, A = (fa , fb , fc ) o, pour tout rel x, fu (x) = sin(x + u).
2) A = (fn )nZ o, pour tout rel x, fn (x) = nx + n2 + 1.
3) A = (x 7 x )R (ici E = (]0; +[)2 ).
4) A = (x 7 |x a|)aR .

Exercice no 17 : (****)
Soit E un K-espace vectoriel et soit (u, v) (L (E))2 .
1) Montrer que [Kerv Keru w L (E)/ u = w v].

2) En dduire que [v injectif w L (E)/ w v = IdE ].

Exercice no 18 : (***)

Soit E = R[X] le R-espace vectoriel des polynmes coefficients rels.


1) Soit f :

E E . f est-elle linaire, injective, surjective ? Fournir un supplmentaire de Kerf.


P 7 P

2) Mmes questions avec g :

E Z E
.
x
P 7
P(t) dt
0

http ://www.maths-france.fr

c Jean-Louis Rouget, 2014. Tous droits rservs.


Exercice no 19 : (**IT)
1) Soit E = CN . Soient a, b et c trois nombres complexes tels que a 6= 0.
a) Soit F lensemble des suites u vrifiant : n N, aun+2 + bun+1 + cun = 0. Montrer que F est un sous-espace
vectoriel de E.
b) Soit lapplication de E dans E qui un lment u de E associe llment (u) de E dfini par
n N, ((u))n = aun+2 + bun+1 + cun .
En utilisant lapplication , retrouver le fait que F est un sous-espace vectoriel de E.
2) Soit E = C (I, C) o I est un intervalle de R. Soient a, b et c trois nombres complexes tels que a 6= 0.
a) Soit F lensemble des fonctions f vrifiant : x I, af (x) + bf (x) + cf(x) = 0. Montrer que F est un sous-espace
vectoriel de E.
b) Soit lapplication de E dans E qui un lment f de E associe llment de E dfini par
x I, ((f))(x) = af (x) + bf (x) + cf(x).
En utilisant lapplication , retrouver le fait que F est un sous-espace vectoriel de E.
Exercice no 20 : (**IT)
Soient F et G deux sous-espaces vectoriels dun espace vectoriel E.
1) CE F est-il un sous-espace vectoriel de E ?
2) a) Montrer que F G est un sous-espace vectoriel de E si et seulement si F G ou G F.
b) Quel est lespace vectoriel engendr par F G ?
Exercice no 21 : (*IT)
Soient E un espace vectoriel puis f et g deux lments de L (E). Montrer que

Exercice no 22 : (*)

g f = 0 Imf Kerg.

Soient E un espace vectoriel puis f et g deux lments de L (E). Montrer que Ker (g f) = f1 (Kerg).
Exercice no 23 : (**I)
Soit z un nombre complexe non rel. Montrer que (1, z) est une base du R-espace vectoriel C.
Exercice no 24 : (***I)
Pour a R et x R, on pose fa (x) = eax . Montrer que la famille (fa )aR est libre.
Exercice no 25 : (**I)
Soient et deux formes linaires sur un K-espace vectoriel E. Montrer que

Exercice no 26 : (**IT)

= 0 = 0 ou = 0.

Soit a R et n N. Montrer que la famille (X a)k

http ://www.maths-france.fr

06k6n

est une base de Rn [X].

c Jean-Louis Rouget, 2014. Tous droits rservs.


Planche no 28. Espaces vectoriels : corrig


Exercice no 1
1) La fonction nulle est dans F et en particulier, F 6= . Soient alors (f, g) F2 et (, ) R2 .
(f + g)(0) + (f + g)(1) = (f(0) + f(1)) + (g(0) + g(1)) = 0.
Par suite, f + g est dans F. On a montr que
0 F et (f, g) F2 , (, ) R2 , f + g F.
F est donc un sous-espace vectoriel de E.
2) La fonction nulle est dans F et en particulier, F 6= . Soient alors (f, g) F2 et (, ) R2 .
(f + g)(0) = f(0) + g(0) = 0.
Par suite, f + g est dans F. On a montr que
0 F et (f, g) F2 , (, ) R2 , f + g F.
F est donc un sous-espace vectoriel de E.
3) F ne contient pas la fonction nulle et nest donc pas un sous-espace vectoriel de E.
4) La fonction nulle est dans F et en particulier, F 6= . Soient alors (f, g) F2 et (, ) R2 . Pour tout rel x de [0, 1],
(f + g)(x) + (f + g)(1 x) = (f(x) + f(1 x)) + (g(x) + g(1 x)) = 0
et donc f + g est dans F. F est un sous-espace vectoriel de E.
1
Remarque. Les graphes des fonctions considrs sont symtriques par rapport au point ( , 0).
2
5) F contient la fonction constante 1 mais pas son oppos la fonction constante 1 et nest donc pas un sous-espace
vectoriel de E.
6) F ne contient pas la fonction nulle et nest donc pas un sous-espace vectoriel de E.
Exercice no 2
Dans les cas o F est un sous-espace, on a chaque fois trois dmarches possibles pour le vrifier :
- Utiliser la caractrisation dun sous-espace vectoriel.
- Obtenir F comme noyau dune forme linaire ou plus gnralement, comme noyau dune application linaire.
- Obtenir F comme sous-espace engendr par une famille de vecteurs.
Je dtaille une seule fois les trois dmarches.
1) 1re dmarche. F contient le vecteur nul (0, ..., 0) et donc F 6= . Soient alors ((x1 , ..., xn ), (x1 , ..., xn )) F2 et
(, ) R2 . On a
(x1 , . . . , xn ) + (x1 , . . . , xn ) = (x1 + x1 , . . . , xn + xn )
avec x1 + x1 = 0. Donc, (x1 , . . . , xn ) + (x1 , . . . , xn ) F. F est un sous-espace vectoriel de Rn .

2me dmarche. Lapplication (x1 , . . . , xn ) 7 x1 est une forme linaire sur Rn et F en est le noyau. F est donc un
sous-espace vectoriel de Rn .
3me dmarche.




F = (0, x2 , ..., xn ), (x2 , ..., xn ) Rn1 = x2 (0, 1, 0, . . . , 0) + . . . + xn (0, . . . , 0, 1), (x2 , . . . , xn ) Rn1
= Vect((0, 1, 0, . . . , 0), . . . , (0, . . . , 0, 1)).

F est donc un sous-espace vectoriel de Rn .


2) F ne contient pas le vecteur nul et nest donc pas un sous-espace vectoriel de Rn .
3) (Ici, n > 2). Lapplication (x1 , ..., xn ) 7 x1 x2 est une forme linaire sur Rn et F en est le noyau. F est donc un
sous-espace vectoriel de Rn .
http ://www.maths-france.fr

c Jean-Louis Rouget, 2014. Tous droits rservs.


4) Lapplication (x1 , ..., xn ) 7 x1 + ... + xn est une forme linaire sur Rn et F en est le noyau. F est donc un sous-espace
vectoriel de Rn .

5) (Ici, n > 2). Les vecteurs e1 = (1, 0, ..., 0) et e2 = (0, 1, 0..., 0) sont dans F mais e1 + e2 = (1, 1, 0...0) ny est pas. F nest
donc pas un sous espace vectoriel de E.
Remarque. F est la runion des sous-espaces {(x1 , ..., xn ) Rn / x1 = 0} et {(x1 , ..., xn ) Rn / x2 = 0}.
Exercice no 3
Il suffit de montrer que C B.
Soit x un lment de C. Alors x A + C = A + B et il existe (y, z) A B tel que x = y + z. Mais z B C et donc,
puisque C est un sous-espace vectoriel de E, y = x z est dans C. Donc, y A C = A B et en particulier y est dans
B. Finalement, x = y + z est dans B. On a montr que tout lment de C est dans B et donc que, C B. Puisque dautre
part B C, on a B = C.
Exercice no 4

Soit u = (sin(n))nN . On a u = 1.u + 0.u , puis v = cos a.u sin a.u , puis w = cos b.u sin b.u . Les trois suites u, v
et w sont donc combinaisons linaires des deux suites u et u et constituent par suite une famille lie (p + 1 combinaisons
linaires de p vecteurs constituent une famille lie).
Exercice no 5
Soit (, ) R2 .

a + 2b =

2a b =
(, , 37, 3) F (a, b) R2 / (, , 37, 3) = au + bv (a, b) R2 /
5a
+ 4b = 37

3a + 7b = 3

a + 2b =




247
126
5
2a b =

=
+
2

47
47
47
247

.
(a, b) R2 /

a=

47

= 620

= 2 247 + 126

47

47
47
b = 126
47
Exercice no 6
Posons F = Vect(a, b) et G = Vect(c, d). On a immdiatement c + 2d = a et 2c d = b et donc a et b sont dans G.
Puisque G est un sous-espace vectoriel de R4 , on en dduit que Vect(a, b) G ou encore F G.
1
2
2
1
En inversant les galits prcdentes, on obtient c = a + b et d = a b. Par suite, {c, d} G et donc Vect(c, d) F
5
5
5
5
ou encore G F. Finalement F = G.

Exercice no 7
1) Si f existe alors ncessairement, pour tout (x, y, z) R3
f((x, y, z)) = xf((1, 0, 0)) + yf((0, 1, 0)) + zf((0, 0, 1)) = x(1, 1) + y(0, 1) + z(1, 1) = (x z, x + y + z).
On en dduit lunicit de f.
Rciproquement, f ainsi dfinie vrifie bien les trois galits de lnonc. Il reste donc se convaincre que f est linaire.
2
Soient ((x, y, z), (x , y , z )) R3 et (, ) R2 .
f((x, y, z) + (x , y , z )) = f((x + x , y + y , z + z ))
= ((x + x ) (z + z ), (x + x ) + (y + y ) + (z + z ))
= ((x z) + (x z ), (x + y + z) + (x + y + z ))
= (x z, x + y + z) + (x z , x + y + z )
= f((x, y, z)) + f((x , y , z )).
f est donc linaire et convient. On en dduit lexistence de f. On a alors f((3, 1, 4)) = (3 4, 3 1 + 4) = (1, 6).

http ://www.maths-france.fr

c Jean-Louis Rouget, 2014. Tous droits rservs.


Remarque. La dmonstration de la linarit de f ci-dessus est en fait superflue car le cours donne lexpression gnrale
dune application linaire de Rn dans Rp .
2) Dtermination de Kerf. Soit (x, y, z) R3 .
(x, y, z) Kerf f((x, y, z)) = (0, 0) (x z, x + y + z) = (0, 0)

xz=0

x+y+z=0

z=x
.
y = 2x

Donc, Kerf = {(x, 2x, x), x R} = {x(1, 2, 1), x R} = Vect((1, 2, 1)). La famille ((1, 2, 1)) engendre Kerf et est
libre. Donc, la famille ((1, 2, 1)) est une base de Kerf.
Dtermination de Imf. Soit (x , y ) R2 .
(x , y ) Imf (x, y, z) R3 / f((x, y, z)) = (x , y )


x z = x
z = x x
3
3
(x, y, z) R /
(x, y, z) R /
x+y+z=y
y = 2x + x + y

z = x x
le systme dinconnue (x, y, z) :
a au moins une solution.
y = 2x + x + y

Or, le triplet (0, x + y , x ) est solution et le systme propos admet une solution. Par suite, tout (x , y ) de R2 est dans
Imf et finalement, Imf = R2 .
Exercice no 8
1) On a toujours Kerf Kerf2 . En effet, si x est un vecteur de Kerf, alors f2 (x) = f(f(x)) = f(0) = 0 (car f est linaire)
et x est dans Kerf2 .
Montrons alors que : [Kerf = Kerf2 Kerf Imf = {0}].

Supposons que Kerf = Kerf2 et montrons que Kerf Imf = {0}.


Soit x Kerf Imf. Alors, dune part f(x) = 0 et dautre part, il existe y lment de E tel que x = f(y). Mais alors,
f2 (y) = f(x) = 0 et y Kerf2 = Kerf. Donc, x = f(y) = 0.
Ceci montre que Kerf Imf {0}. Dautre part, puisque f est linaire, Kerf et Imf sont des sous-espaces vectoriels de E
et donc Kerf Imf est un sous-espace vectoriel de E. On en dduit que {0} Kerf Imf puis que Kerf Imf = {0}.
On a montr que Kerf = Kerf2 Kerf Imf = {0}.

Supposons que Kerf Imf = {0} et montrons que Kerf = Kerf2 .


Soit x Kerf2 . Alors f(f(x)) = 0 et donc f(x) Kerf Imf = {0}. Donc, f(x) = 0 et x est dans Kerf. On a ainsi
montr que Kerf2 Kerf et, puisque lon a toujours Kerf Kerf2 , on a finalement Kerf = Kerf2 . On a montr que
Kerf Imf = {0} Kerf = Kerf2 et finalement que
Kerf = Kerf2 Kerf Imf = {0}.

On a toujours Imf2 Imf. En effet : y Imf2 x E/ y = f(f(x)) y Imf.

Montrons alors que : [Imf = Imf2 E = Kerf + Imf].

Supposons que Imf = Imf2 et montrons que Kerf + Imf = E.


Soit x E. Puisque f(x) Imf = Imf2 , il existe t E tel que f(x) = f2 (t). Soit alors z = f(t) et y = x f(t). On a
bien x = y + z et z Imf. De plus, f(y) = f(x) f(f(t)) = 0 et y est bien lment de Kerf. On a donc montr que
E = Kerf + Imf.
Supposons que Kerf + Imf = E et montrons que Imf = Imf2 .
Soit x E. Il existe (y, z) Kerf Imf tel que x = y + z. Mais alors f(x) = f(z) Imf2 car z est dans Imf. Ainsi, pour
tout x de E, f(x) est dans Imf2 ce qui montre que Imf Imf2 et comme on a toujours Imf2 Imf, on a montr que
Imf = Imf2 . Finalement
Imf = Imf2 E = Kerf + Imf.

2) Id p projecteur (Id p)2 = Id p Id 2p + p2 = Id p p2 = p p projecteur.

Soit x un lment de E. x Imp y E/ x = p(y). Mais alors p(x) = p2 (y) = p(y) = x.


Donc, x E, (x Imp p(x) = x).

Rciproquement, si p(x) = x alors bien sr, x est dans Imp.


Finalement, pour tout vecteur x de E, x Imp p(x) = x (Id p)(x) = 0 x Ker(Id p). On a montr que

http ://www.maths-france.fr

c Jean-Louis Rouget, 2014. Tous droits rservs.


Imp = Ker(Id p).


En appliquant ce qui prcde Id p qui est galement un projecteur, on obtient Im(Id p) = Ker(Id (Id p)) = Kerp.
Enfin, puisque p2 = p et donc en particulier que Kerp = Kerp2 et Imp = Imp2 , le 1) montre que E = Kerp Imp.
3)
p = p q et q = q p p (Id q) = 0 et q (Id p) = 0 Im(Id q) Kerp et Im(Id p) Kerq
Kerq Kerp et Kerp Kerq (daprs 2))
Kerp = Kerq.

4) Supposons que pq+qp = 0 Alors, pq = (pp)q = p(pq) = p(qp) et de mme, qp = qpp = pqp.
En particulier, p q = q p et donc 0 = p q + q p = 2p q = 2q p puis p q = q p = 0.
La rciproque est immdiate.
p + q projecteur (p + q)2 = p + q p2 + pq + qp + q2 = p + q pq + qp = 0 pq = qp = 0 (daprs ci-dessus).

Ensuite, Im(p + q) = {p(x) + q(x), x E} {p(x) + q(y), (x, y) E2 } = Imp + Imq.

Rciproquement, soit z un lment de Imp + Imq. Il existe deux vecteurs x et y de E tels que z = p(x) + q(y). Mais alors,
p(z) = p2 (x) + pq(y) = p(x) et q(z) = qp(x) + q2 (y) = q(y) et donc
z = p(x) + p(y) = p(z) + q(z) = (p + q)(z) Im(p + q).
Donc, Imp + Imq Im(p + q) et finalement,
Im(p + q) = Imp + Imq.
Kerp Kerq = {x E/ p(x) = q(x) = 0} {x E/ p(x) + q(x) = 0} = Ker(p + q).
Rciproquement, si x est lment de Ker(p + q) alors p(x) + q(x) = 0.
Par suite, p(x) = p2 (x) + pq(x) = p(p(x) + q(x)) = p(0) = 0 et q(x) = qp(x) + q2 (x) = q(0) = 0. Donc, p(x) = q(x) = 0
et x Kerp Kerq. Finalement,
Ker(p + q) = Kerp Kerq.
Exercice no 9
1) Soit x E. x (A B) + (A C) y A B, z A C/ x = y + z.
y et z sont dans A et donc x = y + z est dans A car A est un sous-espace vectoriel de E.
Puis y est dans B et z est dans C et donc x = y + z est dans B + C. Finalement,
x E, [x (A B) + (A C) x A (B + C)].

Autre dmarche.
(A B B et A C C) (A B) + (A C) B + C puis (A B A et A C A (A B) + (A C) A + A = A),
et finalement (A B) + (A C) A (B + C).

2) Si on essaie de dmontrer linclusion contraire, le raisonnement coince car la somme y + z peut tre dans A sans que
ni y, ni z ne soient dans A.
Contre-exemple. Dans R2 , on considre A = R.(1, 0) = {(x, 0), x R}, B = R.(0, 1) et C = R.(1, 1).
B + C = R2 et A (B + C) = A mais A B = {0} et A C = {0} et donc (A B) + (A C) = {0} 6= A (B + C).
3) A B B (A B) + (A C) B + (A C) mais aussi (A B) + (A C) A + A = A.
Donc, (A B) + (A C) A (B + (A C)).

Inversement, soit x A (B + (A C)) alors il existe y B et z A C tel que x = y + z. Mais alors, x et z sont dans A
et donc y = x z est dans A et mme plus prcisment dans A B. Donc, x (A B) + (A C).
Ceci montre que A (B + (A C)) (A B) + (A C) et finalement,
A (B + (A C)) = (A B) + (A C).

http ://www.maths-france.fr

c Jean-Louis Rouget, 2014. Tous droits rservs.


Exercice no 10
1) Pour (x, y, z, t) R4 , on pose f((x, y, z, t)) = x 2y, g((x, y, z, t)) = y 2z et h((x, y, z, t)) = x y + z t. f, g et
h sont des formes linaires sur R4 . Donc, V = Kerf Kerg est un sous-espace vectoriel de R4 en tant quintersection de
sous-espaces vectoriels de R4 et W = Kerh est un sous-espace vectoriel de R4 .
2) Soit (x, y, z, t) R4 .
(x, y, z, t) V

x = 2y

y = 2z

x = 4z
.
y = 2z

Donc, V = {(4z, 2z, z, t), (z, t) R2 } = Vect(e1 , e2 ) o e1 = (4, 2, 1, 0) et e2 = (0, 0, 0, 1). Montrons alors que (e1 , e2 ) est
libre. Soit (z, t) R2 .
Donc, (e1 , e2 ) est une base de V.

ze1 + te2 = 0 (4z, 2z, z, t) = (0, 0, 0, 0) z = t = 0.

Pour (x, y, z, t) R4 , (x, y, z, t) W t = x y + z. Donc, W = {(x, y, z, x y + z), (x, y, z) R3 } = Vect (e1 , e2 , e3 ) o


e1 = (1, 0, 0, 1), e2 = (0, 1, 0, 1) et e3 = (0, 0, 1, 1).

Montrons alors que (e1 , e2 , e3 ) est libre. Soit (x, y, z) R3 .

xe1 + ye2 + ze3 = 0 (x, y, z, x y + z) = (0, 0, 0, 0) x = y = z = 0.

Donc, (e1 , e2 , e3 ) est une base de W.


Soit (x, y, z, t) R4 .

x = 4z
x = 4z
y = 2z
y = 2z .
(x, y, z, t) V W

xy+zt=0
t = 3z

Donc, V W = {(4z, 2z, z, 3z), z R} = Vect(e) o e = (4, 2, 1, 3). De plus, e tant non nul, la famille (e) est libre et est
donc une base de V W.

3) Soit u = (x, y, z, t) un vecteur de R4 . On cherche v = (4, 2, , ) V et w = (a, b, c, a b + c) W tels que


u = v + w.

a = x 4

4 + a = x

2 + b = y
b = y 2
u=v+w

.
+c=z
c=z

+ab+c=t
= x + y z + t + 3

et = 0, = x + y z + t, a = x, b = y et c = z conviennent. Donc, u R4 , (v, w) V W/ u = v + w. On a


montr que
R4 = V + W.
Exercice no 11
1) C contient lidentit de R, mais ne contient pas son oppos. Donc, C nest pas un espace vectoriel.

2) Montrons que V est un sous-espace vectoriel de lespace vectoriel des applications de R dans R. V est dj non vide
car contient la fonction nulle (0 = 0 0).
Soit (f1 , f2 ) V 2 . Il existe (g1 , g2 , h1 , h2 ) C4 tel que f1 = g1 h1 et f2 = g2 h2 . Mais alors,
f1 + f2 = (g1 + g2 ) (h1 + h2 ).
Or, une somme de fonctions croissantes sur R est croissante sur R, et donc, g1 + g2 et h1 + h2 sont des lments de C ou
encore f1 + f2 est dans V.
Soit f V et R. Il existe (g, h) V 2 tel que f = g h et donc f = g h.
Si > 0, g et h sont croissantes sur R et f est dans V.
Si < 0, on crit f = (h) (g), et puisque g et h sont croissantes sur R, f est encore dans V.
En rsum, V nest pas vide et est stable pour + et . et on a donc montr que V est un sous-espace vectoriel de lespace
vectoriel des applications de R dans R.
http ://www.maths-france.fr

c Jean-Louis Rouget, 2014. Tous droits rservs.


Exercice no 12
Soit (x, y) E2 .
(1 + 1).(x + y) = 1.(x + y) + 1.(x + y) = (x + y) + (x + y) = x + y + x + y
mais aussi
(1 + 1).(x + y) = (1 + 1).x + (1 + 1).y = x + x + y + y.
Enfin, (E, +) tant un groupe, tout lment est rgulier et en particulier x est rgulier gauche et y est rgulier droite.
Aprs simplification, on obtient y + x = x + y. On a montr que pour tout couple (x, y) lment de E2 , x + y = y + x.
Exercice no 13
Soit F = (A B) + (A C) + (B C).
F A+ A+ B = A+ B puis F A+ C+ C = A+ C puis F B+ C+ C = B+ C et finalement F (A+ B)(A+ C)(B+ C).
Exercice no 14
Soit u = (1, 1, . . . , 1). F = Vect(u) et donc F est un sous espace vectoriel de Rn . G est un sous espace vectoriel de Rn , car
G est le noyau de la forme linaire (x1 , ..., xn ) 7 x1 + ... + xn .

Soit x = (x1 , ..., xn ) Rn et soit R.

x u G (x1 , ..., xn ) G

Donc,

n
X

(xk ) = 0 =

k=1

n
1 X
xk .
n
k=1

x Rn , ! R/ x u G,

et donc,

Rn = F G.

Le projet sur F paralllement G dun vecteur x = (x1 , ..., xn ) est


!
!
n
n
n
1 X
1 X
1 X
xk .u =
xk , ...,
xk
n
n
n
k=1

k=1

k=1

et le projet du mme vecteur sur G paralllement F est


!
!
n
n
n
1 X
1 X
1 X
x
xk .u = x1
xk , ..., xn
xk .
n
n
n
k=1

k=1

k=1

Exercice no 15

a
n Q, il existe (a, b) (N )2 tel que n =
ou encore tel que
b
2
2
n b = a . Mais alors, par unicit de la dcomposition dun entier naturel suprieur ou gal 2 en facteurs premiers,
tous les facteurs premiers
de n ont un exposant pair ce qui signifie exactement que n est un carr parfait.

Si n = 0 ou n = 1, n Q et n est dautre part un carr parfait. On a montr que

1) Soit n un entier naturel suprieur ou gal 2. Si

ou encore par contraposition

n N, ( n Q n est un carr parfait)

n N, (n nest pas un carr parfait

n
/ Q).

2) Daprs 1), 2, 3 et 6 sont irrationnels.


 

E = VectQ (1, 2, 3, 6) et donc, E est un Q-espace vectoriel et 1, 2, 3, 6 en est une famille gnratrice.

Montrons que cette famille est Q-libre. Soit (a, b, c, d) Q4 .

2 
2

a + b 2 + c 3 + d 6 = 0 a + d 6 = b 2 c 3 a2 + 2ad 6 + 6d2 = 2b2 + 2bc 6 + 3c2

a2 2b2 3c2 + 6d2 = 2(ad + bc) 6.

http ://www.maths-france.fr

c Jean-Louis Rouget, 2014. Tous droits rservs.


a2 2b2 3c2 + 6d2


Puisque 6
/ Q, on obtient a2 2b2 3c2 + 6d2 = 2(ad + bc) = 0 (car si bc ad 6= 0, 6 =
Q)
2(ad + bc)
ou encore,
2
a 3c2 = 2b2 6d2 (1)
.
ad = bc (2)
De mme,


2 
2

a + b 2 + c 3 + d 6 = 0 a + c 3 = b 2 d 6 a2 + 2ac 3 + 3c2 = 2b2 + 4bd 3 + 6d2


2
a + 3c2 = 2b2 + 6d2 (3)

.
ac = 2bd (4)

(puisque 3 est irrationnel). En additionnant et en retranchant (1) et (3), on obtient a2 = 2b2 et c2 = 2d2 . Puisque 2

a
est irrationnel, on ne peut avoir b 6= 0 (car alors 2 = Q) ou d 6= 0. Donc, b = d = 0 puis a = c = 0. Finalement,
b
 
la famille 1, 2, 3, 6 est Q-libre et est donc une base de E.
Exercice no 16

1) Notons respectivement s et c, les fonctions sinus et cosinus.


fa = cos a.s + sin a.c, fb = cos b.s + sin b.c et fc = cos c.s + sin c.c. Donc, fa , fb et fc sont trois combinaisons linaires des
deux fonctions s et c et constituent donc une famille lie (p + 1 combinaisons linaires de p vecteurs donns constituent
une famille lie).
2) f0 , f1 et f2 sont trois combinaisons linaires des deux fonctions x 7 1 et x 7 x. Donc, la famille (f0 , f1 , f2 ) est une
famille lie puis la famille (fn )nZ est lie en tant que sur-famille dune famille lie.

3) Pour rel donn et x > 0, posons f (x) = x .


Soient n un entier naturel suprieur ou gal 2 puis (1 , ..., n ) Rn tel que 1 < ... < n . Soit encore (1 , ..., n ) Rn .
n
X

k=1

k fk = 0 x ]0; +[,

n
X

k=1

k xk = 0 x ]0; +[,

n
X

k xk n = 0,

k=1

(en divisant les deux membres par x ). Dans cette dernire galit, on fait tendre x vers + et on obtient n = 0. Puis,
par rcurrence descendante, n1 = ... = 1 = 0. On a montr que toute sous-famille finie de la famille (f )R est libre
et donc, la famille (f )R est libre.
4) Pour a rel donn et x rel, posons fa (x) = |x a|. Soient n un entier naturel suprieur ou gal 2, puis a1 ,...,an , n
n
X
n
rels deux deux distincts. Soit (1 , ..., n ) R tel que
k fak = 0.
k=1

Sil existe i J1, nK tel que i 6= 0 alors,

fa i =

1 X
k f a k .
i
k6=i

Mais cette dernire galit est impossible car fai nest pas drivable en ai alors que

1 X
k fak lest. Donc, tous les i
i
k6=i

sont nuls. Ceci montre que la famille (fa )aR est libre.
Exercice no 17
1) / Soit (u, v)((L (E))2 . On suppose quil existe w L (E) tel que u = w v. Soit x un lment de Kerv. Alors
v(x) = 0 et donc u(x) = w(v(x)) = w(0) = 0. Mais alors, x est dans Keru. Donc Kerv Keru.

/ Supposons que Kerv Keru. On cherche dfinir w, lment de L (E) tel que w v = u. Il faut dfinir prcisment
w sur Imv car sur E \ Imv, on a aucune autre contrainte que la linarit.

Soit y un lment de Imv. (Il existe x lment de E tel que y = v(x). On a alors envie de poser w(y) = u(x) mais le
problme est que y, lment de Imv donn peut avoir plusieurs antcdents x, x ... et on peut avoir u(x) 6= u(x ) de sorte
que lon naurait mme pas dfini une application w.)
Soient x et x deux lments de E tels que v(x) = v(x ) = y alors v(x x ) = 0 et donc x x Kerv Keru. Par suite,
u(x x ) = 0 ou encore u(x) = u(x ). En rsum, pour y lment donn de Imv, il existe x lment de E tel que v(x) = y.
On pose alors w(y) = u(x) en notant que w(y) est bien uniquement dfini, car ne dpend pas du choix de lantcdent x
http ://www.maths-france.fr

c Jean-Louis Rouget, 2014. Tous droits rservs.


de y par v. w nest pas encore dfini sur E tout entier. Notons F un supplmentaire quelconque de Imv dans E (lexistence
de F est admise).
Soit X un lment de E. Il existe deux vecteurs y et z, de Imv et F respectivement, tels que X = y + z. On pose alors
w(X) = u(x) o x est un antcdent quelconque de y par v (on a pris pour restriction de w F lapplication nulle). w
ainsi dfinie est une application de E dans E car, pour X donn, y est uniquement dfini puis u(x) est uniquement dfini
(mais pas ncessairement x).
Soit x un lment de E et y = v(x). w(v(x)) = w(y) = w(y + 0) = u(x) (car 1) y est dans Imv 2) 0 est dans F 3) x est un
antcdent de y par v) et donc w v = u.
Montrons que w est linaire. Soient, avec les notations prcdentes, X1 = y1 + z1 et X2 = y2 + z2 . . .
w(X1 + X2 ) = w((y1 + y2 ) + (z1 + z2 )) = u(x1 + x2 ) (car y1 + y2 = v(x1 ) + v(x2 ) = v(x1 + x2 ) et car z1 + z2 F)
= u(x1 ) + u(x2 ) = w(X1 ) + w(X2 )

et
w(X) = w(y + z) = u(x) = u(x) = w(X).
2) On applique 1) u = Id.
v injective Kerv = {0} Kerv KerId w L (E)/ w v = Id.

Exercice no 18

1) P E, f(P) = P est un polynme et donc f est une application de E vers E.


(P, Q) E2 , (, ) R2 , f(P + Q) = (P + Q) = P + Q = f(P) + f(Q) et f est un endomorphisme de E.

Soit P E. P Kerf P = 0 P est constant. Kerf nest pas


Z x nul et f nest pas injective.
Soient Q E puis P le polynme dfini par : x R, P(x) =
Q(t) dt. P est bien un polynme tel que f(P) = Q. f est
0

surjective.

Soit F = {P E/ P(0) = 0}. F est un sous espace de E en tant que noyau de la forme linaire P 7 P(0). KerfF = {0} car si un
polynme est constant et sannule en 0, ce polynme est nul. Enfin, si P est un polynme quelconque, P = P(0) + (P P(0))
et P scrit bien comme la somme dun polynme constant et dun polynme sannulant en 0. Finalement E = Kerf F.
2) On montre facilement
Z x que g est un endomorphisme de E.
P Kerg x R,
P(t) dt = 0 x R, P(x) = 0 (en drivant les deux membres de lgalit). Donc, Kerg = {0} et
0

donc g est injective.


Si P est dans Img alors P(0) = 0 (ce qui montre que g nest pas surjective car par exemple, le polynme 1 na pas
dantcdent par g).
Zx
Rciproquement, si P(0) = 0 alors P (t) dt = P(x) P(0) = P(x) ce qui montre que P = g(P ) est dans Img. Finalement,
0

Img = {P E/ P(0) = 0}.


Exercice no 19
1) a) La suite nulle est dans F.
Soient (u, v) F2 et (, ) C2 . Pour tout entier naturel n,
a (u + v)n+2 + b (u + v)n+1 + c (u + v)n = a (un+2 + vn+2 ) + b (un+1 + vn+1 ) + c (un + vn )
= (aun+2 + bun+1 + cun ) + (avn+2 + bvn+1 + cvn )
= 0 + 0 = 0,
et donc la suite u+v est dans F. En rsum, F contient 0 et est stable par combinaison linaire. Donc, F est un sous-espace
vectoriel de E.
b) est bien une application de E dans E. Soient (u, v) F2 et (, ) C2 . Pour tout entier naturel n,

http ://www.maths-france.fr

c Jean-Louis Rouget, 2014. Tous droits rservs.


(u + v)n = a(u + v)n+2 + b(u + v)n+1 + c(u + v)n


= a (un+2 + vn+2 ) + b (un+1 + vn+1 ) + c (un + vn )
= (aun+2 + bun+1 + cun ) + (avn+2 + bvn+1 + cvn ) = (u)n + (v)n
= ((u) + (v))n ,
et donc (u + v) = (u) + (v). Ainsi, est un endomorphisme de E. Puisque F = Ker, F est un sous-espace
vectoriel de E.
2) a) La fonction nulle est dans F.
Soient (f, g) F2 et (, ) C2 . Pour tout rel x de I,
a (f + g) (x) + b (f + g) (x) + c (f + g) (x) = a (f (x) + g (x)) + b (f (x) + g (x)) + c (f(x) + g(x))
= (af (x) + bf (x) + cf(x)) + (ag (x) + bg (x) + cg(x))
= 0 + 0 = 0,
et donc la fonction f + g est dans F. En rsum, F contient 0 et est stable par combinaison linaire. Donc, F est un
sous-espace vectoriel de E.
b) est application de E dans E car si f E, alors (f) = af + bf + cf est dfinie et de classe C sur I ou encore (f)
est un lment de E.
Soient (f, g) F2 et (, ) C2 . Pour tout rel x de I,
(f + g)(x) = a (f + g) (x) + b (f + g) (x) + c (f + g) (x)
= a (f (x) + g (x)) + b (f (x) + g (x)) + c (f(x) + g(x))
= (af (x) + bf (x) + cf(x)) + (ag (x) + bg (x) + cg(x))
= (f)(x) + (g)(x) = ((f) + (g))(x),
et donc (f + g) = (f) + (g). Ainsi, est un endomorphisme de E. Puisque F = Ker, F est un sous-espace
vectoriel de E.
Exercice no 20
1) F contient 0 et donc CE F ne contient pas 0. Par suite, CE F nest pas un sous-espace vectoriel de E.
2) a) Si F G ou G F, alors F G = G ou F G = F. Dans tous les cas, F G est un sous-espace vectoriel de E.
Rciproquement, supposons que F G soit un sous-espace vectoriel de E. Si F G, cest fini. Sinon, F 6 G et il existe
un vecteur x0 E tel que x0 F et x0
/ G. Montrons alors que G F.
Soit x G. Alors, x F G et x0 F G. Puisque F G est un sous-espace vectoriel de E, x + x0 F G.
Si x + x0 G, alors (x + x0 ) x G ou encore x0 G ce qui nest pas. Donc, x + x0 F. Mais alors, (x + x0 ) x0 F
ou encore x F. On a montr que
et donc que G F.

x E, (x G x F),

b) F + G est un sous-espace vectoriel de E contenant F et G et donc contenant F G. Dautre part, si H est un sous-espace
vectoriel contenant F G, H contient F et G et donc aussi lensemble des sommes dun lment de F et dun lment de G
cest--dire F + G. Finalement,
Vect(F G) = F + G.
Exercice no 21
g f = 0 x E, g(f(x)) = 0 x E, f(x) Kerg
Imf Kerg.

http ://www.maths-france.fr

c Jean-Louis Rouget, 2014. Tous droits rservs.


Exercice no 22
Soit x E.
x Ker(g f) g(f(x)) = 0 f(x) Kerg
x f1 (Kerg).

Ceci montre que Ker (g f) = f1 (Kerg).


Exercice no 23

Montrons que la famille (1, z) est une famille libre du R-espace vectoriel C.

Soit (, ) R2 tel que 1 + z = 0. Si 6= 0, alors z = . En particulier, z est rel ce qui nest pas. Donc = 0

puis = 0.
Ainsi, (, ) R2 , (.1 + .z = 0 = = 0) et donc la famille (1, z) est une famille libre du R-espace vectoriel C.

Montrons que la famille (1, z) est une famille gnratrice du R-espace vectoriel C. Posons z = + i o et sont
deux rels. Puisque z
/ R, on a 6= 0.
Soit Z C. Posons Z = a + ib o a et b sont deux rels. Alors
Z = a + ib =

b
a b
b
b
( + i)
+a=
.1 + .z,

a b
b
et
rels. Donc Z est combinaison linaire coefficients rels de 1 et z. Par suite, la famille (1, z) est une

famille gnratrice du R-espace vectoriel C.


avec

En rsum, la famille (1, z) est une famille libre et gnratrice du R-espace vectoriel C. Donc la famille (1, z) est une base
du R-espace vectoriel C.
Exercice no 24
Soit n un entier naturel suprieur ou gal 2 puis a1 , . . . , an n rels tels que a1 < a2 < . . . < an .
Supposons par labsurde la famille (fa1 , . . . , fan ) lie. Il existe (1 , . . . , n ) 6= (0, . . . , 0) tel que 1 fa1 + . . . + n fan = 0.
{k J1, nK/ k 6= 0} est une partie non vide (par hypothse) de N et majore par n. Donc {k J1, nK/ k 6= 0} admet un
plus grand lment.
Soit p = Max {k J1, nK/ k 6= 0}. Par dfinition de p, on a p 6= 0 et pour tout rel x, 1 ea1 x + . . . + p eap x = 0. On
divise les deux membres de cette galit par eap x qui nest pas nul et on obtient pour tout rel x,
1 e(ap a1 )x + . . . + p1 e(ap ap1 )x + p = 0.
On fait tendre x vers + et on obtient p = 0 (car k < p, ap ak > 0). Ceci contredit le fait que p 6= 0. Donc, il
nexiste pas (1 , . . . , n ) 6= (0, . . . , 0) tel que 1 fa1 + . . . + n fan = 0. Ceci montre que la famille (fak )16k6n est libre.
On a montr que toute sous-famille finie de la famille (fa )aR est libre et donc la famille (fa )aR est libre.
Exercice no 25
Soient et deux formes linaires telles que = 0. Pour tout lment x de E, on a (x) (x) = 0. Supposons
par labsurde que 6= 0 et 6= 0. Donc il existe x0 et x1 deux lments de E tels que (x0 ) 6= 0 et (x1 ) 6= 0. Puisque
(x0 ) (x0 ) = 0 et que (x0 ) 6= 0, on en dduit que (x0 ) = 0. De mme, (x1 ) = 0. Mais alors
(x0 + x1 ) (x0 + x1 ) = ( (x0 ) + (x1 )) ( (x0 ) + (x1 ))
= (x0 ) (x0 ) + (x0 ) (x1 ) + (x1 ) (x0 ) + (x1 ) (x1 )
= (x0 ) (x1 ) 6= 0.
Ceci contredit le fait que pour tout lment x de E, on a (x) (x) = 0. Donc, = 0 ou = 0.
Exercice no 26

Montrons que la famille (X a)k 06k6n est gnratrice de Rn [X].
Soit P Rn [X]. Daprs la formule de Taylor,
n
X
P(k) (a)
P=
(X a)k .
k!
k=0

http ://www.maths-france.fr

10

c Jean-Louis Rouget, 2014. Tous droits rservs.



Donc P est combinaison linaire des polynmes 1, X a, . . . , (X a)n . Ceci montre que la famille (X a)k 06k6n est
gnratrice de Rn [X].

Montrons que la famille (X a)k 06k6n est libre. Supposons par labsurde quil existe (0 , . . . , n ) 6= (0, . . . , 0) tel que
n
X
k (X a)k = 0.
k=0

{k J0, nK/ k 6= 0} est une partie non vide (par hypothse) de N et majore par n. Donc {k J0, nK/ k 6= 0} admet un
plus grand lment.
Soit p = Max {k J0, nK/ k 6= 0}. Par dfinition de p, on a p 6= 0 et pour tout rel x,
p
X

p
X

k (X a)k = 0. Mais cette

k=0

dernire galit est impossible car, puisque p 6= 0, le polynme


k (Xa)k est de degr p et nest donc pas le polynme
k=0

nul. Donc, la famille (X a)k 06k6n est libre.


En rsum, la famille (X a)k 06k6n est libre et gnratrice de Rn [X]. Donc la famille (X a)k 06k6n est une base
de Rn [X].

http ://www.maths-france.fr

11

c Jean-Louis Rouget, 2014. Tous droits rservs.


Planche no 29. Dimensions des espaces vectoriels


* trs facile ** facile *** difficult moyenne **** difficile
I : Incontournable T : pour travailler et mmoriser le cours
Dans cette planche, la lettre K dsigne toujours un sous-corps de C, comme Q, R ou C.
Exercice no 1 : (**T)
E dsigne lespace vectoriel R4 (muni des oprations usuelles). On considre les vecteurs e1 = (1, 2, 3, 4), e2 = (1, 1, 1, 3),
e3 = (2, 1, 1, 1), e4 = (1, 0, 1, 2) et e5 = (2, 3, 0, 1). Soient alors F = Vect(e1 , e2 , e3 ) et G = Vect(e4 , e5 ). Quelles sont
les dimensions de F, G, F G et F + G ?
Exercice no 2 : (**IT)
Soit E un K-espace vectoriel de dimension finie n > 2. Soient H1 et H2 deux hyperplans de E. Dterminer dimK (H1 H2 ).
Interprtez le rsultat quand n = 2 ou n = 3.
Exercice no 3 : (**)
Soit E un K-espace vectoriel de dimension finie.
Soient f et g deux endomorphismes de E vrifiant E = Kerf + Kerg = Imf + Img. Montrer que ces sommes sont directes.
Exercice no 4 : (***I)
Soit E = Rn [X], le R-espace vectoriel des polynmes coefficients rels de degr infrieur ou gal n (n entier naturel
donn). Soit lapplication dfinie par : P E, (P) = P(X + 1) P(X).
1) Vrifier que est un endomorphisme de E.
2) Dterminer Ker et Im.
Exercice no 5 : (**T)
Soient (ei )16i64 la base canonique de R4 et f lendomorphisme de R4 dfini par : f(e1 ) = 2e1 + e3 , f(e2 ) = e2 + e4 ,
f(e3 ) = e1 + 2e3 et f(e4 ) = e2 e4 . Dterminer Kerf et Imf.
Exercice no 6 : (**)
Soit f :

C
C
o a est un nombre complexe donn non nul.
z 7 z + az
Montrer que f est un endomorphisme du R-espace vectoriel C. f est-il un endomorphisme du C-espace vectoriel C ? Dterminer le noyau et limage de f.
Exercice no 7 : (**)
Soit f L (R2 ). Pour (x, y) R2 , on pose f((x, y)) = (x , y ).

1) Rappeler lcriture gnrale de (x , y ) en fonction de (x, y).


2) Si on pose z = x + iy et z = x + iy (o i2 = 1), montrer que : (a, b) C2 / z C, z = az + bz.
3) Rciproquement, montrer que lexpression ci-dessus dfinit un unique endomorphisme de R2 (en clair, lexpression
complexe dun endomorphisme de R2 est z = az + bz).
Exercice no 8 : (**I)
Soient E et F deux K-espaces vectoriels de dimensions finies sur K et u et v deux applications linaires de E dans F. Montrer
que : |rgu rgv| 6 rg(u + v) 6 rgu + rgv.
Exercice no 9 : (***)
Soit E un K-espace vectoriel de dimension finie n.
1) Soit f L (E) tel que Kerf = Imf. Montrer quil existe une base (u1 , ..., up , v1 , ..., vp ) de E telle que :
i J1, pK, f(ui ) = 0 et f(vi ) = ui .
2) Montrer que, pour tout endomorphisme f de R2 , on a :
(Kerf = Imf) (f2 = 0 et n = 2rgf) (f2 = 0 et g L (E)/ f g + g f = IdE ).

http ://www.maths-france.fr

c Jean-Louis Rouget, 2014. Tous droits rservs.


Exercice no 10 : (***I) (Le thorme des noyaux itrs)


Soient E un K-espace vectoriel et f un endomorphisme de E. Pour k entier naturel donn, on pose Nk = Kerfk et Ik = Imfk
(avec la convention f0 = IdE ).
1) Montrer que : k N, (Nk Nk+1 et Ik+1 Ik ).
2) Montrer que : (k N, (Nk = Nk+1 Nk+1 = Nk+2 ).
A partir dici, on suppose de plus que E est de dimension finie n.


3) a) Montrer que : p N/ k N, k < p Nk Nk+1 et k > p Nk = Nk+1 .
6=

b) Montrer que p 6 n.
4) Montrer que si k < p, Ik Ik+1 et si k > p, Ik = Ik+1 .
6=

5) Soit dk = dimIk . Montrer que la suite (dk dk+1 )kN est dcroissante (en dautres termes la suite des images itres
dcrot de moins en moins vite ou aussi la suite des noyaux itrs crot de moins en moins vite).
Exercice no 11 : (***I)
Soit E un K-espace vectoriel de dimension finie note n. Soit u un endomorphisme de E.
On dit que u est nilpotent si et seulement si k N / uk = 0 et on appelle alors indice de nilpotence de u le plus petit
de ces entiers k (par exemple, le seul endomorphisme u, nilpotent dindice 1 est 0).

1) Soit u un endomorphisme nilpotent dindice p. Montrer quil existe un vecteur x0 de E tel que la famille x, u(x0 ), ..., up1 (x0 )
soit libre.
2) Soit u un endomorphisme nilpotent. Montrer que un = 0.
3) On suppose dans cette question que u est nilpotent dindice n exactement. Dterminer rgu (utiliser le no 10).
Exercice no 12 : (***I)
Soit E un K-espace vectoriel de dimension quelconque sur K et f un endomorphisme de E vrifiant f2 5f + 6IdE = 0.
Montrer que E = Ker(f-2Id) Ker(f 3Id).
Exercice no 13 : (**IT)
Soit E = CN . Soient a, b et c trois nombres complexes tels que a 6= 0. Soit F lensemble des suites u vrifiant : n N,
aun+2 + bun+1 + cun = 0. Montrer que F est un sous-espace vectoriel de E de dimension 2.
Exercice no 14 : (**I)
Soit z un nombre complexe non rel. Montrer que (1, z) est une base du R-espace vectoriel C.
Exercice no 15 : (**I)
Soit n N. Soit (Pk )06k6n une famille de n + 1 polynmes tels que k J0, nK, deg (Pk ) = k. Montrer que la famille
(Pk )06k6n est une base de Rn [X].
Exercice no 16 : (**I)
Soit E un K-espace vectoriel de dimension finie n. Soit f L (E) tel que
x E, px N / fpx (x) = 0.
Montrer que f est nilpotent.
Exercice no 17 : (***I)
1) Soit E un K-espace vectoriel. Soit f L (E) tel que
x E, x K/ f(x) = x x.
Montrer que f est une homothtie.
2) Application. On suppose de plus que E est de dimension finie. Dterminer les endomorphismes f de E qui commutent
avec tous les endomorphismes g de E.

http ://www.maths-france.fr

c Jean-Louis Rouget, 2014. Tous droits rservs.


Planche no 29. Dimensions des espaces vectoriels : corrig


Exercice no 1
e4 et e5 ne sont pas colinaires. Donc (e4 , e5 ) est une famille libre et dim G = rg (e4 , e5 ) = 2.
Ensuite, puisque e1 et e2 ne sont pas colinaires, on a 2 6 dim F 6 3. Soit alors (, , ) R3 .

+ + 2 = 0 (1)

= 0 ((3) (2))
2 + + = 0 (2)
= 0 ((1) (2)) = = = 0.
e1 + e2 + e3 = 0

3 + + = 0 (3)

+ + 2 = 0 (1)

4 + 3 + = 0 (4)

On a montr que : (, , ) R3 , (e1 + e2 + e3 = 0 = = = 0).


La famille (e1 , e2 , e3 ) est donc libre et dim F = rg (e1 , e2 , e3 ) = 3.

Comme F F + G, dim (F + G) > 3 ou encore dim (F + G) = 3 ou 4. De plus :


dim (F + G) = 3 F = F + G G F {e4 , e5 } F.

On cherche alors (, , ) lment de R3 tel que e4 = e1 + e2 + e3 ce qui quivaut au systme :

+ + 2 = 1 (1)

2 + + = 0 (2)
.
3
+ + = 1 (3)

4 + 3 + = 2 (4)

(3) (2) fournit = 1 puis (1) (2) fournit = 2 puis (2) fournit = 4.
Avec ces valeurs, (4) nest pas vrifie car 4 (1) + 3 4 2 = 6 6= 2. Le systme propos nadmet pas de solution ou
encore e4
/ Vect(e1 , e2 , e3 ) = F. Par suite, dim (F + G) = 4.
Enfin,
dim (F G) = dim F + dim G dim (F + G) = 3 + 2 4 = 1.
Exercice no 2
On a H1 H1 + H2 et donc dim (H1 + H2 ) > n 1 ou encore dim (H1 + H2 ) {n 1, n}. Donc

(n 1) + (n 1) (n 1) = n 1
ou
dim(H1 H2 ) = dim H1 + dim H2 dim(H1 + H2 ) =
.

(n 1) + (n 1) n = n 2

Maintenant, si dim(H1 + H2 ) = n 1 = dim H1 = dim H2 , alors H1 = H1 + H2 = H2 et donc en particulier, H1 = H2 .


Rciproquement, si H1 = H2 alors H1 + H2 = H1 et dim (H1 + H2 ) = n 1.
En rsum, si H1 et H2 sont deux hyperplans distincts, dim(H1 H2 ) = n2 et bien sr, si H1 = H2 , alors dim (H1 H2 ) =
n 1.
Si n = 2, les hyperplans sont des droites vectorielles et lintersection de deux droites vectorielles distinctes du plan vectoriel
est de dimension 0, cest--dire rduite au vecteur nul.
Si n = 3, les hyperplans sont des plans vectoriels et lintersection de deux plans vectoriels distincts de lespace de dimension
3 est une droite vectorielle.
Exercice no 3
On a
n = dim E = dim(Ker f + Ker g) = dim(Ker f) + dim (Ker g) dim(Ker f Ker g),
mais aussi,
n = dim (Im f) + dim (Im g) dim (Im f Im g) = n dim Ker f + n dim (Ker g) dim (Im f Im g).
Par suite,
dim (Ker f) + dim Ker g = n + dim (Ker f Ker g) = n dim (Im f Im g)
et donc dim (Ker f Ker g) + dim (Im f Im g) = 0 ou encore dim (Ker f Ker g) = dim (Im f Im g) = 0, et finalement,
Ker f Ker g = Im f Im g = {0}, ce qui montre que les sommes proposes sont directes.
http ://www.maths-france.fr

c Jean-Louis Rouget, 2014. Tous droits rservs.


Exercice no 4
1) Si P est un polynme de degr infrieur ou gal n, alors P(X + 1) P(X) est encore un polynme de degr infrieur
ou gal n. Par suite, est bien une application de E dans lui-mme.
Soient alors (P, Q) E2 et (, ) R2 .
(P + Q) = (P + Q)(X + 1) (P + Q)(X) = (P(X + 1) P(X)) + (Q(X + 1) Q(X))
= (P) + (Q).
est linaire de E vers lui-mme et donc un endomorphisme de E.
2) Soit P E. P Ker x R, P(x + 1) = P(x). Montrons alors que P est constant.
Soit Q = P P(0). Q est un polynme de degr infrieur ou gal n sannulant en les entiers naturels 0, 1, 2, . . . (car
P(0) = P(1) = P(2) = ...) et a ainsi une infinit de racines deux deux distinctes. Q est donc le polynme nul ou encore
x R, P(x) = P(0). Par suite, P est un polynme constant.
Rciproquement, les polynmes constants sont clairement dans Ker et donc
Ker = {polynmes constants} = R0 [X].
Pour dterminer Im , on note tout dabord que si P est un polynme de degr infrieur ou gal n, alors
n1
X
(P) = P(X + 1) P(X) est un polynme de degr infrieur ou gal n 1. En effet, si P = an Xn +
ak Xk (avec an
k=0

quelconque, ventuellement nul) alors

(P) = an ((X + 1)n Xn ) + termes de degr infrieur on gal n 1


= an (Xn Xn ) + termes de degr infrieur on gal n 1
= termes de degr infrieur on gal n 1
Donc, Im () Rn1 [X]. Mais daprs le thorme du rang,
dim Im () = dim Rn [X] dim Ker () = (n + 1) 1 = n = dim Rn1 [X] < +,
et donc Im = Rn1 [X]. (On peut noter que le problme difficile soit Q Rn1 [X]. Existe-t-il P Rn [X] tel que
P(X + 1) P(X) = Q ? a t rsolu simplement par le thorme du rang.)
Exercice no 5
Soit u = (x, y, z, t) = xe1 + ye2 + ze3 + te4 R4 . Alors,
f(u) = xf(e1 ) + yf(e2 ) + zf(e3 ) + tf(e4 ) = x(2e1 + e3 ) + y(e2 + e4 ) + z(e1 + 2e3 ) + t(e2 e4 )
= (2x + z)e1 + (y + t)e2 + (x + 2z)e3 + (y t)e4 .
Par suite,

2x + z = 0

y + t = 0
x=z=0

u Ker f
.
x
+
2z
=
0
y
=t

yt=0

Donc, Ker f = {(0, y, 0, y), y R} = Vect((0, 1, 0, 1)) = Vect (e2 + e4 ). En particulier, Ker f est de dimension 1. Le
thorme du rang permet daffirmer que dim (Im(f)) = 4 dim (Kerf) = 3. Ensuite,
Imf = Vect (f (e1 ) , f (e2 ) , f (e3 ) , f (e4 )) = Vect (2e1 + e3 , e2 + e4 , e1 + 2e3 , e2 e4 )
= Vect (2e1 + e3 , (e2 e4 ) , e1 + 2e3 , e2 e4 ) = Vect (2e1 + e3 , e1 + 2e3 , e2 e4 )
= Vect (2 (2e1 + e3 ) (e1 + 2e3 ) , e1 + 2e3 , e2 e4 ) = Vect (3e1 , e1 + 2e3 , e2 e4 )
= Vect (e1 , e1 + 2e3 , e2 e4 ) = Vect (e1 , e1 + 2e3 e1 , e2 e4 ) = Vect (e1 , 2e3 , e2 e4 )
= Vect (e1 , e3 , e2 e4 ) .
Ainsi, la famille (e1 , e3 , e2 e4 ) est une famille gnratrice de Imf.
http ://www.maths-france.fr

c Jean-Louis Rouget, 2014. Tous droits rservs.


Dautre part, card (e1 , e3 , e2 e4 ) = 3 = dim (Imf) < +. On en dduit que la famille (e1 , e3 , e2 e4 ) est une base de
Imf.
On peut aussi dterminer directement Imf de la faon suivante : soit u = (x , y , z , t ) R4 .

2x + z = x

y + t = y
(x, y, z, t) R4 /
u Imf (x, y, z, t) R4 /

x
+
2z
=
z

y t = t
y + t = 0.

x = (2x z )

1
z = (x + 2z )
3

t
=
y
+ y


y + t = 0

(si y + t 6= 0, le systme ci-dessus, dinconnues x, y, z et t, 


na pas de solution et si y + t =
 0, le systme ci-dessus
1
1
admet au moins une solution comme par exemple (x, y, z, t) =
(2x z ), 0, (x + 2z ), y ) .
3
3
Donc, Im f = {(x, y, z, t) R4 / y + t = 0} = {(x, y, z, y)/(x, y, z) R3 } = {xe1 + y(e2 e4 ) + ze3 , (x, y, z) R4 } =
Vect((e1 , e2 e4 , e3 )).
Exercice no 6
Soient (z, z ) C2 et (, ) R2 .


f(z + z ) = (z + z ) + a z + z = (z + az) + z + az = f(z) + f(z ).


f est donc R-linaire. On note que f(ia) = i a |a|2 et que if(a) = i a + |a|2 . Donc, if(a) f(ia) = 2i|a|2 . Comme
a 6= 0, on a f(ia) 6= if(a). f nest pas C-linaire.
Soit z C \ {0}. Posons z = rei o r R+ et R.

z Ker f z + az = 0 ei + aei = 0 e2i = a.

1er cas. Si |a| 6= 1, alors, pour tout rel , e2i 6= a. Dans ce cas, Ker f = {0} et daprs le thorme du rang,
Im f = C.
2me cas. Si |a| = 1, posons a = ei .
e2i = a e2i = ei(+) 2 + + 2Z

+
+ Z.
2

Dans ce cas, Ker f = Vect(ei(+)/2 ). Daprs le thorme du rang, Im f est une droite vectorielle et pour dterminer
Im f, il suffit den fournir un vecteur non nul. Donc, si a 6= 1, Im f = Vect(f(1)) = Vect(1 + a). Si a = 1,
z C, f(z) = z z = 2iIm (z) et Im f = iR.
Exercice no 7
1) Pour (x, y) R2 , posons f((x, y)) = (x , y ).
f L (R2 ) (, , , ) R4 / (x, y) R2 ,

x = x + y
.
y = x + y

2) Avec les notations prcdentes,





z+z
z+z
zz
zz
+i
z = x + iy = (x + y) + i(x + y) =
+
+
2
2i
2
2i





+
+
z+
+i
+i
=
z = az + bz
2
2
2
2

o a =

+
+
+i
et b =
+i
.
2
2
2
2

3) Rciproquement, si z = az + bz, en posant a = a1 + ia2 et b = b1 + ib2 o (a1 , a2 , b1 , b2 ) R4 , on obtient :


x + iy = (a1 + ia2 )(x + iy) + (b1 + ib2 )(x iy) = (a1 + b1 )x + (a2 + b2 )y + i((a2 + b2 )x + (a1 b1 )y)
et donc,
http ://www.maths-france.fr

c Jean-Louis Rouget, 2014. Tous droits rservs.


x = (a1 + b1 )x + (b2 a2 )y
.
y = (a2 + b2 )x + (a1 b1 )y

Ceci montre que lapplication de R2 dans lui-mme dexpression complexe z = az + bz est R-linaire.
Exercice no 8
Par dfinition, rg (u + v) = dim (Im (u + v)).
Mais, Im (u + v) = {u(x) + v(x), x E} {u(x) + v(x ), (x, x ) E2 } = Im u + Im v. Donc,
rg (u + v) = dim (Im u + Im v) = dim (Im u) + dim (Im v) dim (Im u Imv) 6 dim (Im u) + dim (Im v)
= rg u + rg v.
On a montr que :
(u, v) (L (E, F))2 , rg (u + v) = rg u + rg v.
Ensuite,
rg u = rg (u + v v) 6 rg (u + v) + rg (v) = rg (u + v) + rg v,
(puisque Im (v) = Im v) et donc rg u rg v = rg (u + v). En changeant les rles de u et v, on a aussi rg v rg u =
rg (u + v) et finalement
(u, v) (L (E, F))2 , |rg u rg v| 6 rg (u + v).
Exercice no 9
1) Posons F = Kerf = Imf puis r = dimF. Daprs le thorme du rang,
r = dim (Imf) = n dim (Kerf) = n r,
n
et donc n = 2r. Donc, n est pair et r = .
2
Soit G un supplmentaire de F dans E (dim G = n r = r). Soit (v1 , ..., vr ) une base de G. Pour i J1, rK, on pose
ui = f (vi ). Montrons que la famille (u1 , ..., ur ) est libre.
Soit (1 , ..., r ) Rr .
r
X
i=1

i ui = 0 f

r
X
i=1

i vi

=0

r
X
i=1

i vi Ker f G = {0} i J1, rK, i = 0,

car (vi )16i6r est une famille libre. Ainsi, (u1 , ..., ur ) est une famille libre de Im f = F de cardinal r et donc une base de
F = Ker f = Im f.
Puisque E = F G, (u1 , ..., ur , v1 , ..., vr ) est une base de E. Puisque u1 , . . . , ur sont dans Imf = Kerf, iJ1, rK, f (ui ) = 0.
Dautre part, par construction, iJ1, rK, f (vi ) = ui .
2) (1) (2). Si Ker f = Im f, alors pour tout lment x de E, f(x) est dans Im f = Ker f et donc f(f(x)) = 0. Par suite,
f2 = 0. De plus, daprs le thorme du rang, n = dim (Ker f) + rg f = 2r ce qui montre que n est ncessairement pair et
n
que rg f = .
2
(2) (1). Si f2 = 0, alors pour tout lment x de E, f(f(x)) = 0 ou encore pour tout lment x de E, f(x) est dans Kerf.
Ceci montre que Imf Kerf. De plus, daprs le thorme du rang
dim (Kerf) = n r = 2r r = r = dim (Imf) < +.
Par suite, Ker f = Im f.
(1) (3). Supposons Kerf = Imf. Daprs ce qui prcde, f2 = 0. Daprs 1), il existe une base (u1 , ..., ur , v1 , ..., vr ) de E
telle que iJ1, rK, f (ui ) = 0 et f (vi ) = ui .

Soit alors g lendomorphisme de E dfini par les galits : i J1, rK, g(ui ) = vi et g(vi ) = vi (g est entirement dtermin
par les images des vecteurs dune base de E). Pour i lment de J1, rK, on a alors :
(f g + g f) (ui ) = f (vi ) + g(0) = ui + 0 = ui ,
et
http ://www.maths-france.fr

c Jean-Louis Rouget, 2014. Tous droits rservs.


(f g + g f) (vi ) = f (ui ) + g (ui ) = 0 + vi = vi .


Les endomorphismes f g + g f et IdE concident sur une base de E, et donc f g + g f = IdE .
(3) (1). Supposons que f2 = 0 et quil existe g L (E) tel que f g + g f = IdE . Comme f2 = 0, on a dj
Im f Ker f. Dautre part, si x est un lment de Ker f, alors x = f(g(x)) + g(f(x)) = f(g(x)) Im f et on a aussi
Ker f Im f. Finalement, Ker f = Im f.
Exercice no 10
1) Soient k un entier naturel et x un lment de E.

x Nk fk (x) = 0 f fk (x) = f(0) fk+1 (x) = 0 x Nk+1 .

On a montr que : k N, Nk Nk+1 . Ensuite,

x Ik+1 y E/ x = fk+1 (y) z(= f(y)) E/ x = fk (z) x Ik .

On a montr que : k N, Ik+1 Ik .

2) Soit k un entier naturel. Supposons que Nk = Nk+1 . On a dj Nk+1 Nk+2 . Montrons que Nk+2 Nk+1 .
Soit x un lment de E.
x Nk+2 fk+2 (x) = 0 fk+1 (f(x)) = 0 f(x) Nk+1 = Nk fk (f(x)) = 0
fk+1 (x) = 0 x Nk+1 .

3) a) On a {0} = N0 N1 N2 ... Supposons que chacune de ces inclusions soient strictes. Alors,
0 = dim N0 < dim N1 < dim N2 ...
Donc dim N1 > 1, dim N2 > 2 et par rcurrence , k N, dim Nk > k. En particulier, dim Nn+1 > n + 1 > n = dim E,
ce qui est impossible. Donc, il existe k entier naturel tel que Nk = Nk+1 .
Ainsi, {k N/ Nk = Nk+1 }, K est une partie non vide de N. {k N/ Nk = Nk+1 } admet donc un plus petit lment. Soit
donc p le plus petit des entiers k tels que Nk = Nk+1 .
Par dfinition de p (et mme si p = 0), pour k < p, Nk Nk+1 . Dautre part, daprs 2) et puisque Np = Np+1 , on
6=

montre par rcurrence que pour k > p, on a Nk = Np .


b) Si p = 0 (ou encore si f est injectif), on a p 6 n. Sinon
0 < dim N1 < ... < dim Np
et donc, par rcurrence, pour k 6 p, on a dim Nk > k. En particulier
p 6 dim Np 6 n.
4) Puisque Nk Nk+1 , Ik+1 Ik et que dim E < +, on a :


Nk = Nk+1 dim Nk = dim Nk+1 n rg fk = n rg fk+1 dim (Ik ) = dim (Ik+1 ) Ik = Ik+1 .

Donc, pour k < p, Ik Ik+1 et pour k > p, Ik = Ik+1 .


6=

5) Soient k un entier naturel puis gk la restriction de f Ik . Daprs le thorme du rang,


dk = dim (Ik ) = dim (Ker gk ) + dim (Im gk ).
Maintenant, Im (gk ) = gk (Ik ) = f (Ik ) = Ik+1 et donc dim (Im (gk )) = dk+1 . Dautre part, Ker gk = Ker f/Ik = Ker fIk .
Ainsi, pour tout entier naturel k,
dk dk+1 = dim(Kerf Ik ).
Puisque la suite (Ik )kN est dcroissante pour linclusion, la suite dentiers naturels (dim (Ker fIk ))kN = (dk dk+1 )kN
est dcroissante.
http ://www.maths-france.fr

c Jean-Louis Rouget, 2014. Tous droits rservs.


Exercice no 11
1) Soit p( N ) lindice de nilpotence de u.
Par dfinition, up1 6= 0 et plus gnralement, pour 1 6 k 6 p 1, uk 6= 0 car si uk = 0 alors up1 = uk up1k = 0
ce qui nest pas.
Puisque up1 6= 0, il existe au moins un vecteur x0 tel que up1 (x0 ) 6= 0 (et en particulier x0 6= 0).
Montrons que la famille (uk (x))06k6p1 est libre.
p

Soit (k )06k6p1 K tel que

p1
X

k uk (x) = 0. Supposons par labsurde quau moins un des coefficients k ne soit pas

k=0

nul. Soit i = Min {k J0, p 1K/ k 6= 0}.


p1
X
k=0

k u (x) = 0

p1
X
k=i

p1i

k u (x) = 0 u

p1

i u

(x) = 0

p1
X

k u (x)

k=i

=0

p1
X

k up1i+k (x) = 0

k=i

(car pour k > i + 1, p 1 i + k > p et donc up1i+k = 0)

i = 0 (car up1 (x) 6= 0)


ce qui contredit la dfinition de i. Donc tous les coefficients k sont nuls et on a montr que la famille uk (x) 06k6p1
est libre.
2) Le cardinal dune famille libre est infrieur ou gal la dimension de lespace et donc p 6 n. Par suite,
un = up unp = 0.
3) On applique le no 10. Puisque un1 6= 0, on a Nn1 Nn .
6=

Par suite (daprs le no 10, 3)a)), les inclusions N0 N1 ... Nn = E sont toutes strictes et donc
0 < dimN1 < dimN2 ... < dimNn = n.
Pour k J0, nK, notons dk est la dimension de Nk . Par rcurrence, pour k J0, n 1K, on a dk > k.
Mais si de plus, pour un certain indice i lment de J1, n 1K, on a di = dimNi > i, alors, par rcurrence, pour i 6 k 6 n,
on a dk > k et en particulier dn > n ce qui nest pas. Donc,
k J0, nK, dim (Nk ) = k.

Daprs le thorme du rang, k J0, nK, rg uk = n k, et en particulier rg(u) = n 1.
Exercice no 12

Montrons que Ker (f 2Id) Ker (f 3Id) = {0}. Soit x E.


x Ker (f 2Id) Ker (f 3Id) f(x) = 2x et f(x) = 3x 3x 2x = f(x) f(x) = 0
x = 0.

Donc, Ker (f 2Id) Ker (f 3Id) = {0} (mme si f2 5f + 6Id 6= 0).


Montrons que E = Ker (f 2Id) + Ker (f 3Id). Soit x E. On cherche y et z tels que y Ker (f 2Id), z Ker (f 3Id)
et x = y + z.


y+z= x
y = 3x f(x)
Si y et z existent, ncessairement y et z sont solution du systme
et donc
.
2y + 3z = f(x)
z = f(x) 2x
Rciproquement . Soient x E puis y = 3x f(x) et z = f(x) 2x. On a bien y + z = x puis
f(y) = 3f(x) f2 (x) = 3f(x) (5f(x) 6x) (car f2 = 5f 6Id)
= 6x 2f(x) = 2(3x f(x)) = 2y
et donc y Ker (f 2Id). De mme,
f(z) = f2 (x) 2f(x) = (5f(x) 6x) 2f(x) = 3(f(x) 2x) = 3z,
et donc z Ker (f 3Id). On a montr que E = Ker (f 2Id) + Ker (f 3d) et finalement que
http ://www.maths-france.fr

c Jean-Louis Rouget, 2014. Tous droits rservs.


E = Ker (f 2Id) Ker (f 3d).


Exercice no 13
On sait dj que F est un sous-espace vectoriel de E (voir exercice no 19, planche no 28). Soit

F
u

est bien une application de F dans C2 .

C2
7

(u0 , u1 )

Soient (u, v) F2 et (, ) C2 .
(u + v) = (u0 + v0 , u1 + v1 ) = (u0 , u1 ) + (v0 , v1 )
= (u) + (v).
est une application linaire de F dans C2 .
c
b
Soit u Ker. Alors u0 = u1 = 0 et n N, aun+2 + bun+1 + cun = 0 ou encore n N, un+2 = un+1 un
a
a
(puisque a 6= 0). Mais alors, par rcurrence double, n N, un = 0 ou encore u = 0. Ainsi, Ker est le sous-espace nul
et donc est injectif.
Soit (a, b) C2 . Soit u la suite dfinie par u0 = a, u1 = b et n N, aun+2 + bun+1 + cun = 0. u est un lment de
F tel que (u) = (a, b). Ceci montre que est surjectif.

Finalement, est un isomorphisme de F sur C2 . En particulier, dimF = dim C2 = 2.

On a montr que F est un sous-espace vectoriel de E de dimension 2.


Exercice no 14

On a dj montr que la famille (1, z) est une famille libre du R-espace vectoriel C (voir exercice no 23, planche 28). De
plus, card(1, z) = 2 = dimR (C) < +. Donc (1, z) est une base du R-espace vectoriel C.
Exercice no 15
Pour tout k J0, nK, deg (Pk ) 6 n. Donc chaque Pk , 0 6 k 6 n, est un lment de Rn [X]. De plus,
card (Pk )06k6n = n + 1 = dimRn [X] < +.
Pour montrer que la famille (Pk )06k6n est une base de Rn [X], il suffit de vrifier que la famille (Pk )06k6n est libre.
Soit (k )06k6n Rn+1 tel que

n
X

k Pk = 0. Supposons par labsurde que lun au moins des k ne soit pas nul.

k=0

Soit p = Max {k J0, nK/ k 6= 0} ({k J0, nK/ k 6= 0} est une partie non vide et majore (par n) de N et donc {k J0, nK/ k 6= 0}
admet un plus grand lment). Par dfinition de p,
p
X

k Pk = 0.

k=0

Cette dernire galit est impossible car

p
X

k Pk est un polynme de degr p (puisque p 6= 0) et donc

k=0

pas le polynme nul. Donc

(k )06k6n R

k Pk nest

k=0

n+1

n
X

k=0

et la famille (Pk )06k6n est libre.

p
X

k Pk = 0 k J0, nK, k = 0 ,

On a montr que la famille (Pk )06k6n est une base de Rn [X].


Exercice no 16
Soit B = (ek )16k6n une base de E. Par hypothse, k J1, nK, pk N / fpk (ek ) = 0. Soit p = Max {p1 , p2 , . . . , pn }.
p est un entier naturel non nul et pour tout k J1, nK, p pk > 0. On a donc
fp (ek ) = fppk (fpk (ek )) = fppk (0) = 0.
Lendomorphisme fp sannule en chacun des vecteurs dune base de E et donc fp = 0. On a montr que f est nilpotent.
http ://www.maths-france.fr

c Jean-Louis Rouget, 2014. Tous droits rservs.


Exercice no 17
1) Si E = {0}, alors f = 0 et en particulier f est une homothtie. Dornavant, on supposera que E 6= {0}. Soit x0 un lment
non nul de E. Par hypothse, il existe K tel que f (x0 ) = x0 . Vrifions alors que pour tout x de E, f(x) = x. Soit
donc x un lment de E.
1er cas. Supposons la famille (x, x0 ) libre.
Il existe x K tel que f(x) = x x et il existe x+x0 K tel que f (x + x0 ) = x+x0 (x + x0 ) = x+x0 + x+x0 x0 . Puisque
f est linaire,
x+x0 + x+x0 x0 = f (x + x0 ) = f (x) + f (x0 ) = x x + x0 .
Puisque la famille (x, x0 ) est libre, on peut identifier les coefficients et on obtient x = x+x0 = . Par suite, f(x) = x.
2me cas. Supposons la famille (x, x0 ) lie. Puisque x0 nest pas nul, il existe K tel que x = x0 . Mais alors
f(x) = f (x0 ) = f (x0 ) = .x0 = .x0 = x.
Ainsi, on a trouv K tel que, pour tout x de E, f(x) = x ou encore on a trouv K tel que f = Id. On a montr
que f est une homothtie.
2) Soit f un endomorphisme de E tel que g L (E), f g = g f. Vrifions que x E, x K/ f(x) = x x ou encore
vrifions que x E, f(x) Vect(x). Cest immdiat si x = 0.
Soit x un lment non nul de E. Soit D la droite vectorielle engendre par x, soit H un supplmentaire de D dans E puis
s la symtrie par rapport D paralllement H.
s f = f s s(f(x)) = f(s(x)) s(f(x)) = f(x) f(x) D f(x) Vect(x).

Ainsi, x E, f(x) Vect(x). Daprs 1), f est ncessairement une homothtie.

Rciproquement, soient K puis f = Id. Pour tout g L (E), f g = Id g = g, g f = g Id = g Id = g et


donc f g = g f.
On a montr que les endomorphismes qui commutent avec tous les endomorphismes sont les homothties vectorielles.

http ://www.maths-france.fr

c Jean-Louis Rouget, 2014. Tous droits rservs.


Planche no 30. Matrices


* trs facile ** facile *** difficult moyenne **** difficile
I : Incontournable T : pour travailler et mmoriser le cours
Exercice no 1 : (**T)
Soit u lendomorphisme de R3 dont la matrice dans la base canonique (i, j, k) de R3 est

2
1
0
M = 3 1 1 .
1
0 1
1) Dterminer u(2i 3j + 5k).
2) Dterminer Keru et Imu.
3) Calculer M2 et M3 .
4) Dterminer Keru2 et Imu2 .

5) Calculer (I M) I + M + M2 et en dduire que I M est inversible. Prciser (I M)1 .
Exercice no 2 : (**)

Pour x rel, on pose


A(x) =

cos x
sin x

sin x
cos x

Dterminer (A(x))n pour x rel et n entier relatif.


Exercice no 3 : (***T)
Soit u lendomorphisme de R3 dont la matrice dans la base canonique (i, j, k) de R3 est

0 1 0
M = 0 0 1 .
1 3 3
1) Montrer que u est un automorphisme de R3 et dterminer u1 .
2) Dterminer une base (e1 , e2 , e3 ) de R3 telle que u(e1 ) = e1 , u(e2 ) = e1 + e2 et u(e3 ) = e2 + e3 .
3) Dterminer P la matrice de passage de (i, j, k) (e1 , e2 , e3 ) ainsi que P1 .
4) En dduire un (i), un (j) et un (k) pour n entier relatif.
Exercice no 4 : (**)
Soit f :

Rn [X]
P

Rn+1 [X]
X2
X2
)
7

Q = e (Pe

1) Vrifier que f (L (Rn [X], Rn+1 [X]).

2) Dterminer la matrice de f relativement aux bases canoniques de Rn [X] et Rn+1 [X].


3) Dterminer Kerf et rgf.
Exercice no 5 : (***I)
Soit f un
de R3 , nilpotent dindice 2. Montrer quil existe une base de R3 dans laquelle la matrice de f
endomorphisme

0 0 0
scrit 1 0 0 .
0 0 0

http ://www.maths-france.fr

c Jean-Louis Rouget, 2014. Tous droits rservs.


Exercice no 6

0
0

Soit A = ...

0
1

: (*)
0

1
0

...

0
1

1
0
..
.

Mp (R). Calculer An pour n entier relatif.

0
0
... ... 0

Exercice no 7 : (**)


1
1
Montrer que
2
x
1x

x
1

, x ] 1, 1[

est un groupe pour la multiplication des matrices.

Exercice no 8 : (***)
1) Montrer quune matrice triangulaire suprieure est inversible si et seulement si ses coefficients diagonaux sont tous non
nuls.
2) Montrer que toute matrice triangulaire suprieure est semblable une matirce triangulaire infrieure.
Exercice no 9 : (***)






1 0
1 1
Soient I =
et J =
puis E = M(x, y) = xI + yJ, (x, y) R2 .
0 1
0 1
1) Montrer que (E, +, .) est un sous-espace vectoriel de M2 (R). Dterminer une base de E et sa dimension.
2) Montrer que (E, +, ) est un anneau commutatif.
3) Quels sont les inversibles de cet anneau ?
4) Rsoudre dans E les quations suivantes :
a) X2 = I

b) X2 = 0 c) X2 = X.

5) Calculer (M(x, y))n pour n entier naturel non nul.


Exercice no 10 : (***)
Soient A M3,2 (R) et B M2,3 (R) telles que

0 1 1
AB = 1 0 1 .
1
1
2
Montrer lexistence dau moins un couple (A, B) vrifiant les conditions de lnonc puis calculer BA. (Indication. Calculer
(AB)2 et utiliser le rang.)
Exercice no 11 : (***)
Soit A = (ai,j )16i,j6n (n > 2) dfinie par
i J1, nK, ai,j
Montrer que A est inversible et calculer son inverse.

i si i = j
1 si i > j .
=

0 si i < j

Exercice no 12 : (***I)
Dterminer lensemble des lments de Mn (K) qui commutent avec tous les lments de Mn (K) (utiliser les matrices
lmentaires).
Exercice no 13 : (***T)
Dterminer le rang des matrices suivantes :

http ://www.maths-france.fr

c Jean-Louis Rouget, 2014. Tous droits rservs.


1
1) 1/2
1/3

1/2
1/3
1/4

1/3
1/4
m

1
2) b + c
bc

1
1
c+a a+b
ca
ab

3)

4) (i + j + ij)16i,j6n

6)

5) (sin(i + j))16i,j6n

1 a
a 1
1 b
b 1
a b
0
..
.
0
b

a
..
.

1
b
1
a

b
1

a
1
0 ...
..
.
. ..
..

..

.
...

..
..

.
0

0
..
.

, (a, b) C2 .
0

b
a

Exercice no 14 : (****)
Montrer que tout hyperplan de Mn (K) (n > 2) contient au moins une matrice inversible.
Exercice no 15 : (***I) (Thorme de Hadamard).
X
Soit A Mn (C) telle que : i J1, nK, |ai,i | >
|ai,j |. Montrer que A est inversible.
j6=i

Exercice n 16 : (***I) (Matrice de Vandermonde des racines n-imes de lunit).



Soit = e2i/n , (n > 2). Soit A = (j1)(k1) 16j,k6n . Montrer que A est inversible et calculer A1 (calculer dabord
o

AA).

Exercice no 17 : (***I)


i1
Soit A = (ai,j )16i,j6n+1 dfinie par ai,j = 0 si i > j et ai,j =
si i 6 j.
j1
Montrer que A est inversible et dterminer son inverse. (Indication : considrer lendomorphisme de Rn [X] qui un
polynme P associe le polynme P(X + 1)).
Exercice no 18 : (**I)
On pose u0 = 1, v0 = 0, puis, pour n N, un+1 = 2un + vn et vn+1 = un + 2vn .


2 1
1) Soit A =
. Pour n N, calculer An . En dduire un et vn en fonction de n.
1 2
2) En utilisant deux combinaisons linaires intressantes des suites u et v, calculer directement un et vn en fonction
de n.
Exercice no 19 : (**)

Soient A Mn (C) puis B llment de Mnp (C) dfini par B =

du rang de A.

http ://www.maths-france.fr

A
0
..
.
0

... 0
.
..
. ..
A
..
..
.
. 0
... 0 A

. Dterminer le rang de B en fonction

c Jean-Louis Rouget, 2014. Tous droits rservs.


Planche no 30. Matrices : corrig


Exercice no 1

2
2
1
0
2
1
1) Soit X = 3 . MX = 3 1 1 3 = 2 et u(2i 3j + 5k) = i + 2j 3k.
5
1
0 1
5
3

x
2) Soit X = y M3,1 (R) .
z



2
1
0
x
0
2x + y = 0
y = 2x
3x y + z = 0
MX = 0 3 1 1 y = 0
.
z=x

1
0 1
z
0
xz =0

Donc, Ker(u) = Vect(i2j+k). En particulier, dim(Ker(u)) = 1 et, daprs le thorme du rang, rg(u) = 2. Or, u(j) = ij
et u(k) = j k sont deux vecteurs non colinaires de Im(u) qui est un plan vectoriel et donc Im(u) = Vect(i j, j k).
On peut noter que i 2j + k = (i j) (j k) Im(u) et donc Ker(u) Im(u).
3)

2
1
M2 = 3 1
1
0

et

0
2
1
1 3 1
1
1
0


0
1
1
1 = 2 2
1
1
1

1
2
1

1
1
1
2
1
0
M3 = M2 M = 2 2 2 3 1 1 = 0.
1
1
1
1
0 1




4) Ker u2 est le plan dquation x + y + z = 0. Une base de Ker u2 est (i j, j k) et donc Ker u2 = Im(u) =
Vect(i j, j k).

Daprs le thorme du rang, Imu2 est une droite vectorielle. Mais u3 = 0 scrit encore u u2 = 0, et donc Im u2 est
contenu dans Ker(u) qui est une droite vectorielle. Donc, Imu2 = Keru = Vect(i 2j + k).

5) (I M) I + M + M2 = I M3 = I. Par suite, I M est inversible droite et donc inversible et
(I M)1
Exercice no 2

1 0
= I + M + M2 = 0 1
0 0


0
2
0 + 3
1
1


1
0
1
1
1 1 + 2 2
0 1
1
1


1
4
2 = 5
1
2

2
1
2 1 .
1
1

Soient x et y deux rels.

A(x)A(y) =
=

cos(x + y)
sin(x + y)



 

sin y
cos x cos y sin x sin y (sin x cos y + cos x sin y)
=
cos y
sin x cos y + cos x sin y
cos x cos y sin x sin y

sin(x + y)
= A(x + y).
cos(x + y)

cos x sin x
sin x cos x

cos y
sin y

En particulier,
A(x)A(x) = A(x)A(x) = A(0) =

1 0
0 1

= I2 ,

et A(x) est inversible dinverse A(x).


On a aussi, pour n entier naturel non nul donn :
(A(x))n = A(x)A(x)...A(x) = A(x + x... + x) = A(nx),
ce qui reste clair pour n = 0 car A(x)0 = I2 = A(0). Enfin, (A(x))n = (A(x)1 )n = A(x)n = A(nx). Finalement,
n Z, (A(x))n = A(nx) =
http ://www.maths-france.fr

cos(nx) sin(nx)
sin(nx) cos(nx)


.

c Jean-Louis Rouget, 2014. Tous droits rservs.


Exercice no 3
1) rg(u) = rg(u(i),
u(j), u(k)) = rg(u(j), u(k), u(i)). La matrice de cette dernire famille dans la base B = (i, j, k) est

1 0 0
0 1 0 . Cette dernire famille est de rang 3. Donc, rg(u) = 3 et u est bien un automorphisme de R3 . Posons
3 3 1
i = u(i), j = u(j) et k = u(k) de sorte que u1 (i ) = i, u1 (j ) = j et u1 (k ) = k.

1
i =k
k = i
u (k) = i
j = i 3k
i = 3i + j

u1 (i) = 3i + j

k = j + 3k
j = 3i + k
u (j) = 3i + k

et

A1 = MatB
2) et 3) Posons e1 = xi + yj + zk o (x, y, z) R3 .

3

u1 = 1
0

3 1
0 0 .
1 0


1 1 0
x
0
x + y = 0
y + z = 0
u(e1 ) = e1 (u Id)(e1 ) = 0 0 1 1 y = 0
x = y = z.

1 3 2
z
0
x 3y + 2z = 0

On prend e1 = i + j + k.

Posons e2 = xi + yj + zk.

x + y = 1
y + z = 1
u(e2 ) = e1 + e2 (u Id)(e2 ) = e1
y = x + 1 et z = x + 2.

x 3y + 2z = 1

On prend e2 = j + 2k.

Posons e3 = xi + yj + zk.

On prend e3 = k.

x + y = 0
y + z = 1
u(e3 ) = e2 + e3 (u Id)(e3 ) = e2
y = x et z = x + 1.

x 3y + 2z = 2

1 0 0
La matrice de la famille (e1 , e2 , e3 ) dans la base (i, j, k) est P = 1 1 0 . Cette matrice est de rang 3 et est donc
1 2 1
3
inversible. Par suite (e1 , e2 , e3 ) est une base de R . Enfin,

k = e3
e1 = i + j + k
e2 = j + 2k
j = e2 2e3

e3 = k
i = e1 e2 + e3
et

1
0 0
P1 = 1 1 0 .
1 2 1

1 1 0
4) Soit T est la matrice de u dans la base (e1 , e2 , e3 ). T = 0 1 1 . Les formules de changement de bases scrivent
0 0 1
T = P1 AP ou encore A = PTP1 . Par suite, pour tout relatif n, An = PT n P1 .

0 1 0
0 0 1
Posons N = 0 0 1 . On a N2 = 0 0 0 puis N3 = 0.
0 0 0
0 0 0

Donc, pour n entier naturel suprieur ou gal 2 donn, puisque I et N commutent, la formule du binme de Newton
fournit
http ://www.maths-france.fr

c Jean-Louis Rouget, 2014. Tous droits rservs.


Tn

1
n(n

1)
= (I + N)n = I + nN +
N2 = 0
2
0

n n(n 1)/2
.
1
n
0
1

Cette formule reste vraie pour n = 0 et n = 1. Pour n = 1, (I + N)(I N + N2 ) = I + N3 = I et donc


(1)(1 1)
1 1 1
1 1

2
T 1 = (I + N)1 = I N + N2 = 0 1 1 = 0 1
,
1
0 0
1
0 0
1


1
n(n 1) 2
et la formule reste vraie pour n = 1. Enfin, pour n entier naturel non nul donn, T
= I + nN +
N
2



n(n 1) 2
n(n 1) 2
n(n 1) 2
N
I nN +
N = I et donc T n = I nN +
N . Finalement,
mais I + nN +
2
2
2

1 n n(n 1)/2
n(n 1) 2
.
0 1
n
n Z, T n = I + nN +
N =
2
0 0
1
n

Puis

An = PT n P1

0
1
0 0
1
0

n n(n 1)/2
1
n
0
1

n(n 1)/2
1
0
1 1
n(n + 1)/2
(n + 1)(n + 2)/2
1 2

1 0
= 1 1
1 2

1
n

1
n
+1
=
1 n+2

(n 1)(n 2)/2
n(n 1)/2
=
n(n + 1)/2

ce qui fournit un (i), un (j) et un (k).

1
1
1

0
0
1

0 0
1 0
2 1

n(n 2)
n(n 1)/2

(n 1)(n + 1)
n(n + 1)/2
n(n + 2)
(n + 1)(n + 2)/2

Exercice no 4
1) Pour P lment de Rn [X],
2

f(P) = eX (PeX ) = eX (P eX 2XPeX ) = P 2XP.


Ainsi, si P est un polynme de degr infrieur ou gal n, f(P) = P 2XP est un polynme de degr infrieur ou gal
n + 1, et f est bien une application de Rn [X] dans Rn+1 [X].
De plus, pour (, ) R2 et (P, Q) Rn [X], on a :
f(P + Q) = (P + Q) 2X(P + Q) = (P 2XP) + (Q 2XQ) = f(P) + f(Q).
f est lment de L (Rn [X], Rn+1 [X]).
2) La matrice A cherche est lment de Mn+1,n (R).

Pour k = 0, f(Xk ) = f(1) = 2X et pour 1 6 k 6 n, f(Xk ) = kXk1 2Xk+1 . On a donc :

A=

...

0
..
.

2
..
.

0
..
.

0
..
.

..

..

..

..

..
..
.
0

..

...

. 2
... 0

...

0
..
.
..
.

.
0

0
2

3) Soit P Rn [X] tel que f(P) = 0. Si P nest pas nul, 2XP a un degr strictement plus grand que P et donc f(P) nest
pas nul. Par suite, Kerf = {0} (f est donc injective) et daprs le thorme du rang, rgf = dim(Rn [X]) 0 = n + 1, ce qui
montre que Imf nest pas Rn+1 [X] (f nest pas surjective).
http ://www.maths-france.fr

c Jean-Louis Rouget, 2014. Tous droits rservs.


Exercice no 5
f nest pas nul et donc dim(Kerf) 6 2. Puisque f2 = 0, Imf Kerf. En particulier, dim(Kerf) > rgf = 3 dim(Kerf) et
3
dim(Kerf) > .
2
Finalement, dim(Kerf) = 2. Kerf est un plan vectoriel et, daprs le thorme du rang, Imf est une droite vectorielle,
contenue dans Kerf.
f nest pas nul et donc il existe e1 tel que f(e1 ) 6= 0 (et en particulier e1 6= 0). Posons e2 = f(e1 ). Puisque f2 = 0,
f(e2 ) = f2 (e1 ) = 0 et e2 est un vecteur non nul de Kerf. Daprs le thorme de la base incomplte, il existe un vecteur
e3 de Kerf tel que (e2 , e3 ) soit une base de Kerf.
Montrons que (e1 , e2 , e3 ) est une base de R3 . Soit (, , ) R3 .
e1 + e2 + e3 = 0 f(e1 + e2 + e3 ) = 0 e2 = 0 = 0 (car e2 6= 0).

Puis, comme e2 + e3 = 0, on obtient = = 0 (car la famille (e2 , e3 ) est libre).


Finalement, = = = 0 et on a montr que (e1 , e2 , e
Puisque cette famille est de cardinal 3, cest une base
3 ) est libre.
0 0 0
de R3 . Dans cette base, la matrice A de f scrit : A = 1 0 0 .
0 0 0
Exercice no 6

Soit f lendomorphisme de Rp de matrice A dans la base canonique B de Rp . Pour 1 6 k 6 p, on a f(ek ) = ep+1k et


donc f2 (ek ) = ek . Ainsi, A2 = Ip . Mais alors, pour n entier relatif donn, An = Ip si n est pair et An = A si n est impair.
Exercice no 7


1
1 x
Pour x ] 1, 1[, posons M(x) =
. Posons ensuite G = {M(x), x ] 1, 1[}.
x 1
1 x2
Soit alors x ] 1, 1[. Il existe un rel a (et un seul) tel que x = tha. On a



 

1
1 x
1
th a
ch a sh a
= ch a
=
.
M(x) =
x 1
th a
1
sh a ch a
1 x2


ch a sh a
Posons, pour a R, N(a) =
. On a ainsi x ] 1, 1[, M(x) = N(a) o a est le rel tel que x = th a ou
sh a ch a
aussi, a R, N(a) = M(th a). Par suite, G = {N(a), a R}.
Soit alors (a, b) R2 .



 

sh b
ch a ch b + sh a sh b sh a ch b + sh b ch a
N(a)N(b) =
=
ch b
sh a ch b + sh b ch a ch a ch b + sh a sh b


ch(a + b) sh(a + b)
=
= N(a + b).
sh(a + b) ch(a + b)
ch a sh a
sh a ch a

ch b
sh b

Montrons alors que G est un sous-groupe de (G L 2 (R), ).


N(0) = I2 G et donc G est non vide.
a R, det(N(a)) = ch2 a sh2 a = 1 6= 0 et donc G G L 2 (R).
1

(a, b) R , N(a)N(b) = N(a + b) G et a R, (N(a))

ch a
sh a

sh a
ch a

= N(a) G.

On a montr que G est un sous-groupe de (G L 2 (R), ).


Exercice no 8
1) La dmonstration la plus simple apparatra dans le chapitre suivant : le dterminant dune matrice triangulaire est le
produit de ses coefficients diagonaux. Cette matrice est inversible si et seulement si son dterminant est non nul ou encore
si et seulement si aucun des coefficients diagonaux nest nul.
Pour linstant, le plus simple est dutiliser le rang dune matrice. Si aucun des coefficients diagonaux nest nul, on sait que
le rang de la matrice est son format et donc que cette matrice est inversible.
Rciproquement, notons (e1 , ..., en ) la base canonique de Mn,1 (K). Supposons que A soit une matrice triangulaire infrieure dont le coefficient ligne i, colonne i, est nul. Si i = n, la dernire colonne de A est nulle et A nest pas de rang n et
donc nest pas inversible. Si i < n, alors les n i + 1 dernires colonnes sont dans Vect(ei+1 , ..., en ) qui est de dimension
au plus n i(< n i + 1), et encore une fois, la famille des colonnes de A est lie et donc la matrice A nest pas inversible.
http ://www.maths-france.fr

c Jean-Louis Rouget, 2014. Tous droits rservs.


2) Soit A = (ai,j )16i,j6n une matrice triangulaire suprieure et f lendomorphisme de Kn de matrice A dans la base
canonique B = (e1 , ..., en ) de Kn . Soit B = (en , ..., e1 ). B est encore une base de Kn . Soit alors P la matrice de
passage de B B puis A la matrice de f dans la base B . Les formules de changement de bases permettent daffirmer
que A = P1 AP et donc que A et A sont semblables.
Vrifions alors que A est une matrice triangulaire infrieure. Pour i J1, nK, posons ei = en+1i .
A est triangulaire suprieure. Donc, pour tout i J1, nK, f(ei ) Vect(e1 , ..., ei ). Mais alors, pour tout i J1, nK,

f(en+1i
) Vect(en , ..., en+1i
) ou encore, pour tout i J1, nK, f(ei ) Vect(en , ..., ei ). Ceci montre que A est une
matrice triangulaire infrieure.
Exercice no 9
1) E = Vect(I, J). Donc, E est un sous-espace vectoriel de M2 (R). La famille (I, J) est libre car la matrice J nest pas une
matrice scalaire et donc est une base de E. Par suite, dimE = 2.


 

1 1
1 1
1 2
2
2) J =
=
= 2J I. Plus gnralement, pour (x, y, x , y ) R4 ,
0 1
0 1
0 1
M(x, y)M(x , y ) = (xI + yJ)(x I + y J) = xx I + (xy + yx )J + yy J2 = (xx yy )I + (xy + yx + 2yy )J ().
Montrons alors que (E, +, ) est un anneau.

(E, +, .) est un sous-espace vectoriel de M2 (R). En particulier, (E, +) est un groupe commutatif.
Daprs (), la restriction de E est une loi interne dans E.
est associative et distributive sur laddition.
I2 E est lment neutre pour .
Daprs (), est commutative dans E.

Donc, (E, +, ) est un anneau.

3) Soit ((x, y), (x , y )) (R2 )2 .


M(x, y) M(x , y ) = I (xx yy )I + (xy + yx + 2yy )J = I

xx yy = 1

(car la famille (I, J) est libre).


yx + (x + 2y)y = 0

Le dterminant de ce dernier systme dinconnues x et y vaut x(x + 2y) + y2 = x2 + 2xy + y2 = (x + y)2 . Si y 6= x, ce


systme admet un et seule couple solution.
Par suite, si y 6= x, il existe (x , y ) R2 tel que M(x, y)M(x , y ) = I. Dans ce cas, la matrice M(x, y) est inversible
dans E.

x(x + y ) = 1
Si y = x, le systme scrit
et na pas de solution.
x(x + y ) = 0
4) a) Soit (x, y) R2 .
2

M(x, y) = I

x2 y2 = 1

2y(x + y) = 0

y=0
x2 = 1

ou

x2 y2 = 1

x+y=0

y=0
x=1

ou

y=0
.
x = 1

Dans E, lquation X2 = I admet exactement deux solutions savoir I et I.


b) Soit (x, y) R2 .

x2 y2 = 0

2y(x + y) = 0

y=0
x2 = 0

y = x
y = x.
0=0


0
2
Dans E, lquation X = 0 admet pour solutions les matrices de la forme (J I) =
, R.
0 0
M(x, y)2 = 0

ou

c) Soit (x, y) R2 .

http ://www.maths-france.fr

c Jean-Louis Rouget, 2014. Tous droits rservs.


2
x2 y2 = x
x y2 = x

2y(x + y) = y
y(2x + 2y 1) = 0


y = x +
y=0
2 2


ou
1
x2 = x

=x
x x +
2




1 =0
y=0
y=0
y=0
4

ou
ou
1
x=0
x
=0
x=1

y = x +
2

M(x, y)2 = M(x, y)

ou

y=0
.
x=1

Dans E, lquation X2 = X admet exactement deux solutions savoir 0 et I.




0 1
5) Posons N = J I =
. On a N2 = 0 et plus gnralement Nk = 0 pour tout k > 2.
0 0
Soient (x, y) R2 et n N .
M(x, y) = xI + yJ = xI + y(I + N) = (x + y)I + yN.
Puisque les matrices (x + y)I et yN commutent, la formule du binme de Newton permet dcrire

(M(x, y)) = ((x + y)I + yN) =

n  
X
n

k=0
n

n1

= (x + y) I + ny(x + y)

(x + y)nk Ink yk Nk

N=

(x + y)n
0

ny(x + y)n1
(x + y)n

Exercice no 10
Soit (i, j) la base canonique de R2 et (e1 , e2 , e3 ) la base canonique de R3 . On cherche f L (R2 , R3 ) et g L (R3 , R2 )
tels que
f g(e1 ) = e2 + e3 , f g(e2 ) = e1 + e3 et f g(e3 ) = e1 e2 + 2e3 = f g(e1 + e2 ).
On pose g(e1 ) = i, g(e2 ) = j et g(e3 ) = i + j, puis f(i) = e2 + e3 et f(j) = e1 + e3 . Les applications linaires f et g
conviennent, ou encore si on pose



0 1
1 0 1

A=
1 0
et B =
,
0 1 1
1
1



0 1
0 1 1
1 0 1
alors AB = 1 0
= 1 0 1 .
0 1 1
1
1
1
1
2

A et Bdsignent maintenant
deux matrices quelconques, lments de M3,2 (R) et M2,3 (R) respectivement, telles que

0 1 1
AB = 1 0 1 . Calculons (AB)2 . On obtient
1
1
2

0 1 1
0 1 1
0 1 1
(AB)2 = 1 0 1 1 0 1 = 1 0 1 = AB.
1
1
2
1
1
2
1
1
2

Mais alors, en multipliant les deux membres de cette galit par B gauche et A droite, on obtient
(BA)3 = (BA)2 ().
Notons alors que
rg(BA) > rg(ABAB) = rg((AB)2 ) = rg(AB) = 2,

et donc, BA tant une matrice carre de format 2, rg(BA) = 2. BA est donc une matrice inversible. Par suite, on peut
simplifier les deux membres de lgalit () par (BA)2 et on obtient BA = I2 .
http ://www.maths-france.fr

c Jean-Louis Rouget, 2014. Tous droits rservs.


Exercice no 11
Soit B = (ei )16i6n la base canonique de Cn et (ei )16i6n la famille dlments de Cn de matrice A dans la base B.
Par dfinition de A, on a
i J1, n 1K, ei = iei +

n
X

ej et en = nen .

j=i+1

En retranchant membre membre ces galits, on obtient

i J1, n 1K, ei ei+1


= i (ei ei+1 ) et en = nen ,

ou encore
i J1, n 1K, ei ei+1 =

1
1

(ei ei+1
) et en = en .
i
n

Mais alors, pour i J1, n 1K, on a


ei =

n1
X

(ej ej+1 ) + en =

j=i

1
e +
i i
1
e
i i

n1
X
j=i

n
X

1
e
j j

n
X

1
e
j(j 1) j

j=i+1

j=i+1

n
X

j=i+1

n1
n
X1
X
1
1
1 1

(ej ej+1
) + en =
ej
e + e
j
n
j
j1 j n n
j=i

j=i+1

1
e
j1 j

On en dduit que Cn = Vect(e1 , ..., en ) Vect(e1 , ..., en ), ce qui montre que la famille B = (e1 , ..., en ) est gnratrice
de Cn et donc une base de Cn . Par suite, A est inversible et

si i = j

1
1
A = MatB B = (ai,j )16i,j6n o ai,j =

si i > j .

i(i

1)

0 si i < j
Exercice no 12

Soit A = (ak,l )16k,l6n Mn (K).


Si A commute avec toute matrice, en particulier : (i, j) {1, ..., n}2 , AEi,j = Ei,j A. Maintenant,
AEi,j =

X
k,l

ak,l Ek,l Ei,j =

n
X

ak,i Ek,j et Ei,j A =

k=1

X
k,l

ak,l Ei,j Ek,l =

n
X

aj,l Ei,l .

l=1

On note que si k 6= i ou l 6= j, Ek,j 6= Ei,l . Puisque la famille (Ei,j ) est libre, on peut identifier les coefficients et on
obtient : si k 6= i, ak,i = 0. Dautre part, le coefficient de Ei,j est ai,i dans la premire somme et aj,j dans la deuxime.
Ces coefficients doivent tre gaux.
Finalement, si A commute avec toute matrice, ses coefficients non diagonaux sont nuls et ses coefficients diagonaux sont
gaux. Par suite, il existe un scalaire K tel que A = In . Rciproquement, si A est une matrice scalaire, A commute
avec toute matrice.
Exercice no 13
1)

1
1/2
rg 1/2 1/3
1/3 1/4

1
0
1/3

1/2
1/12
1/4 = rg
m
1/3 1/12

1
0
1/2 1/12
= rg
1/3 1/12

http ://www.maths-france.fr

0
1/12 (rg(C , C , C ) = rg(C , C 1 C , C 1 C ))

1
3
1
1
2
3
1
2
1
2
3
m
9

0
(rg(C1 , C2 , C3 ) = rg(C1 , C2 , C3 C2 ))
7
m
36
7

c Jean-Louis Rouget, 2014. Tous droits rservs.


Si m =

7
7
, rgA = 2 (on note alors que C1 = 6(C2 C3 )) et si m 6=
, rgA = 3 et A est inversible.
36
36

2)

1
rg b + c
bc

1
c+a
ca

1
1
a + b = rg b + c
ab
bc

0
0
ab
ac
c(a b) b(a c)

1er cas. Si a, b et c sont deux deux distincts.

1
0 0
1
0
rg b + c 1 1 = rg b + c 1
bc
c b
bc
c

Donc, si a, b et c sont deux deux distincts alors rgA = 3.

(rg(C1 , C2 , C3 ) = rg(C1 , C2 C1 , C3 C1 ))

0
1
0 = rg b + c
bc
bc

1
2me cas. Si b = c 6= a (ou a = c 6= b ou a = b 6= c). A a mme rang que 2b
b2
Donc, si b = c 6= a ou a = c 6= b ou a = b 6= c, rgA = 2.

0
1
b

0 0
1 0 .
c 1

0
1
1 puis que 2b
b
b2

0
1
b

0
0 .
0

3me cas. Si a = b = c, il est clair ds le dpart que A est de rang 1.


3) Puisque rg(C1 , C2 , C3 , C4 ) = rg(C1 , C2 aC1 , C3 C1 , C4 bC1 ),

1 a
a 1
rg
1 b
b 1

1er cas. Si a 6= b.

1 b

b 1
= rg

1 a
a 1

1 a2
rgA = 1 + rg b a
1 ab

1
= 1 + rg 1 a2
1 ab

1
= 1 + rg 1 a2
1 ab

1
= 1 + rg 1 a2
1 ab

1
0
0
0
1 a2
2

a 1a
b a 1 ab
= 1 + rg b a
1 ba
0
ab
1 ab
b 1 ab a b 1 b2

ba
0
ab

1 ab
ab
1 b2

1 a2 1 1 ab
1 ab
1
0
1
a b = 1 + rg
1 ab 1 1 b2
1 b2

0
1
1 1 ab (rg(L1 , L2 , L3 ) = rg(L2 , L1 , L3 )).
1 1 b2

0
0
1
0
0

1 2 a2 ab = 1 + rg 1 a2 1
0
1 2 b2 ab
1 ab 1 (2 b2 ab) + (2 a2 ab)

0
0

1
0
1 4 (a + b)2
ba
0
ab

Si a 6= b et a + b 6= 2, rgA = 4 et si a 6= b et a + b = 2, rgA = 3.
2me cas. Si a = b.
1 a2

0
rgA = 1 + rg
1 a2

0 1 a2
1 a2

0
0
0
= 1 + rg
0 1 a2
1 a2

Si a = b = 1, rgA = 1 et si a = b 6= 1, rgA = 2.


1 a2
1 a2

0
= 1 + rg
1 a2
1 a2

1 a2
1 a2

4) Pour n > 2 et j J1, nK, notons Cj la j-me colonne de la matrice propose.

avec U =

Cj = (i + j + ij)16i6n = (i)16i6n + j(i + 1)16i6n = jU + V,

1
2
2
3

..
..

.
.

et V =
i .
i+1

.
..
.

.
.
n
n+1

http ://www.maths-france.fr

c Jean-Louis Rouget, 2014. Tous droits rservs.


Ainsi, j J1, nK, Cj Vect(U, V) ce qui montre que rgA 6 2. De plus, la matrice extraite
1 et 2) est inversible. Donc rg(A) > 2 et finalement rgA = 2.

3
5

5
8

(lignes et colonnes

5) On suppose n > 2. La j-me colonne de la matrice scrit


Cj = (sin i cos j + sin j cos i)1in = (sin j) C + (cos j) S avec C = (cos i)16i6n et S = (sin i)16i6n .
Par suite, j J1, nK, Cj Vect(C, S) ce qui montre que rgA 6 2. De plus, la matrice extraite forme des termes lignes et
colonnes 1 et 2 est inversible car son dterminant vaut sin 2 sin 4 sin2 3 = 0, 7... 6= 0 et finalement rgA = 2.
6) Dterminons KerA. Soit (xi )16i6n Mn,1 (C).
(xi )16i6n KerA i J1, n 1K, axi + bxi+1 = 0 etbx1 + axn = 0 (S).

1er cas. Si a = b = 0, alors clairement rgA = 0.


2me cas. Si a = 0 et b 6= 0, alors (S) i {1, ..., n} xi = 0. Dans ce cas, KerA = {0} et donc rgA = n.
b
3me cas. Si a 6= 0. Posons = .
a
(S) k J1, n 1K, xk = xk+1 et xn = x1

k J1, nK, xk = (k1) x1 et xn = x1

k J1, nK, xk = (k1) x1 et n x1 = x1

Mais alors, si n 6= 1, le systme (S) admet lunique solution (0, ..., 0) et rgA = n, et si n = 1,
KerA = Vect((1, n1 , ..., 2 , )) est de dimension 1 et rgA = n 1.
b
En rsum, si a = b = 0, rgA = 0 et si a = 0 et b 6= 0, rgA = n. Si a 6= 0 et Un , rgA = n 1 et si a 6= 0 et
a
b
/ Un , rgA = n.

a
Exercice no 14
Soit H un hyperplan de Mn (K) et f une forme linaire non nulle sur Mn (K) telle que H = Kerf.
X
Pour A = (ai,j )16i,j6n , posons f(A) =
i,j ai,j .
16i,j6n

1er cas. Supposons (i, j) J1, nK2 / i 6= j et i,j 6= 0. On pose alors S =


A=

n
X

Ek,k

k=1

n
X

k,k et on considre

k=1

S
Ei,j .
i,j

A est triangulaire coefficients diagonaux tous non nuls et est donc inversible.
n
X
S
De plus, f(A) =
k,k
i,j = S S = 0 et A est lment de H.
i,j
k=1

2me cas. Supposons (i, j) J1, nK2 , (i 6= j i,j = 0). Alors, A Mn (K), f(A) =

n
X

i,i ai,i .

i=1

Soit A = En,1 + E2,1 + E3,2 + ... + En1,n . A est inversible car par exemple gale la matrice de passage de la base
canonique (e1 , e2 , ..., en ) de Kn la base (en , e1 , ..., en1 ). De plus, f(A) = 0 et donc A H.

Exercice no 15

x1

Soit X = ... un vecteur du noyau de A. Supposons X 6= 0. Soit i0 est un indice tel que |xi0 | = Max{|xi |, i J1, nK}.

xn
On a |xi0 | > 0. Mais alors,

AX = 0 i J1, nK,

http ://www.maths-france.fr

n
X

ai,j xj = 0

j=1





X
X
X

|ai0 ,j | |xj | 6 |xi0 |
|ai0 ,j |
|ai0 ,i0 xi0 | =
ai0 ,j xj 6
j6=i0
j6=i0
j6=i0
9

c Jean-Louis Rouget, 2014. Tous droits rservs.


Puisque |xi0 | > 0, on obtient |ai0 ,i0 6

|ai0 ,j | contredisant les hypothses de lnonc. Donc, il est absurde de supposer

j6=i0

que KerA contient un vecteur non nul et A est bien inversible.


Exercice no 16
Soient k et l deux entiers tels que 1 6 k 6 n et 1 6 l 6 n. Le coefficient ligne k, colonne l de AA vaut :
n
X

(k1)(j1) (j1)(l1) =

j=1

n
X

kl

j=1

1er cas. Si k = l, kl = 1, et le coefficient vaut

n
X

j1

1 = n.

j=1

2me cas. Si k 6= l. On a (n 1) 6 k l 6 n 1 avec k l 6= 0 et donc, k l nest pas multiple de n.


Par suite, kl 6= 1 et
n
n
X

1 kl
1 1kl
kl j1
=
= 0.

=
1
1
j=1

En rsum, AA = nIn . Donc A est inversible gauche et donc inversible et A1 =

1
A.
n

Exercice no 17
Soit f lendomorphisme de Rn [X] qui, un polynme P de degr infrieur ou gal n, associe le polynme P(X + 1).
Par la formule du binme de Newton, on voit que A est la matrice de f dans la base canonique (1, X, ..., Xn ) de Rn [X].
f est clairement un automorphisme de Rn [X], sa rciproque tant lapplication qui, un polynme P associe le polynme
P(X 1).

i
A est donc inversible et A1 = (bi,j )06i,j6n o bi,j = 0 si i > j et bi,j = (1)i+j
si i 6 j.
j
Exercice no 18


1 1
1) Posons J =
de sorte que A = I + J. On a J2 = 2J et donc, plus gnralement : k > 1, Jk = 2k1 J. Mais
1 1
alors, puisque I et J commutent, la formule du binme de Newton fournit pour n entier naturel non nul donn :
!
!
!
n  
n  
X
X
n k
1
n k1
2
1 J
2
J=I+
2
k
k
k=0
k=1
k=1
 n

1
1
1
3 + 1 3n 1
= I + ((1 + 2)n 1) J = I + (3n 1)J =
3n 1 3n + 1
2
2
2

n  
X
n k
n
n
A = (I + J) = I +
J =I+
k

ce qui reste vrai pour n = 0. Donc,


n N, An =
Pour n entier naturel donn, posons Xn =

Xn = An X0 =

un
vn

1
2

1
2

3n + 1 3n 1
3n 1 3n + 1


. Pour tout entier naturel n, on a alors Xn+1 = A Xn et donc,
3n + 1 3n 1
3n 1 3n + 1



1
0

Donc,

3n + 1

=
3n 1 .
2

3n + 1
3n 1
et vn =
.
2
2
2) Soit n N. un+1 + vn+1 = 3(un + vn ). Donc, la suite u + v est une suite gomtrique de raison 3 et de premier terme
u0 + v0 = 1. On en dduit que
n N, un =

n N, un + vn = 3n (I).
http ://www.maths-france.fr

10

c Jean-Louis Rouget, 2014. Tous droits rservs.


De mme, pour tout entier naturel n un+1 vn+1 = un vn . Donc, la suite u + v est une suite constante. Puisque
u0 v0 = 1, on en dduit que
n N, un vn = 1 (II).
En additionnant et en retranchant (I) et (II), on obtient
n N, un =

3n 1
3n + 1
et vn =
.
2
2

Exercice no 19
On note r le rang de A. Si r = 0, A est nulle et donc B est nulle.
Sinon, il existe deux matrices carres inversibles

P 0 ... 0

.
0 . . . . . . ..

P = .
Mnp (C) et Q =
.
.
.. .. 0
..

0 ... 0 P
Un calcul par blocs fournit

P
0
..
.
0

0
..
.

... 0
.
..
. ..
.. ..
.
. 0
... 0 P

P1

Donc P est inversible dinverse

0
..
.
0

P1
0
..
.

0
..
.

...
0
..
..
.
.
..
..
.
.
0
. . . 0 P1

0
..
.

...
0
..
..
.
.
..
..
.
.
0
. . . 0 P1

0
De plus, un calcul par blocs montre que

PJr Q 0 . . .
0

..
.
.
.
.
0
.
.
.

B= .
.
.
.. ..
..
0
0
. . . 0 PJr Q

P1

P
...
0

..
.
.

0
.
.
0
=
.
.
.
.
..
..
..
0 ..
0
0
. . . 0 P1

In 0 . . . 0

.
0 . . . . . . ..
= Inp .
=
. .

.. ... 0
..
0 . . . 0 In

0
..
.

0
0

... 0
.
..
. ..
..
.
. .. 0
... 0 P

0
..
.

Q1

. De mme, Q est inversible dinverse

Jr

= P Jr Q o Jr = 0

..
0

0
..
.

... 0
.
..
. ..
..
.
. .. 0
. . . 0 Jr

Ir
0

P et Q de format n telles que A = PJr Q o Jr =

Q 0 ... 0
.
.. ..
.
. ..
0
Mnp (C).

.. . . . .
.
. 0
.
0 ... 0 Q

0
..
.
0

0
..
.

. Soient

...
0
..
..
.
.
..
..
.
.
0
. . . 0 Q1

La matrice B est quivalente a la matrice Jr et a donc mme rang que Jr . Enfin, en supprimant les lignes nulles et les
colonnes nulles, on voit que la matrice Jr a mme rang que la matrice Ipr savoir pr. Dans tous les cas, on a montr que
rg(B) = p rg(A).

http ://www.maths-france.fr

11

c Jean-Louis Rouget, 2014. Tous droits rservs.


Planche no 31. Intgration


* trs facile ** facile *** difficult moyenne **** difficile
I : Incontournable T : pour travailler et mmoriser le cours
Toutes les fonctions considres dans cette planche sont valeurs relles .
Exercice no 1 : (****)
Soient f et g deux fonctions continues et strictement positives sur [a, b]. Pour n entier naturel non nul donn, on pose
!1/n
Zb
un =
(f(x))n g(x) dx
.
a

Montrer que la suite (un ) converge et dterminer sa limite (commencer par le cas g = 1).
Exercice no 2 : (**I)
1) Soit f une application de classe C1 sur [0, 1] telle que f(1) 6= 0.
Z1
Pour n N, on pose un = tn f(t) dt. Montrer que lim un = 0 puis dterminer un quivalent simple de un quand n
tend vers + (tudier

n+

lim nun ).

n+

2) Mmes questions en supposant que f est de classe C2 sur [0, 1] et que f(1) = 0 et f (1) 6= 0.
Exercice no 3 : (***IT)
Limites quand n tend vers + de
n
k
1 X 2
k sin
1) 3
n
n
k=1
n
1 X  
E
k
5)
n n k=1

!1/n
n
1 Y
(a + k)
(a > 0 donn)
n!

2)

k=1

6)

n
X

k=1

k2
3
8k + n3

n
X
n+k
3)
n2 + k

7)

k=1
2n1
X
k=n

1
2k + 1

n
X

2
n k2
k=1
n
X en/k
.
8) n
k2
4)

k=1

Exercice no 4 : (***I)
Soit f une fonction de classe C2 sur [0, 1]. Dterminer le rel a tel que :
Z1

f(t) dt

 
n1  
a
1 X
1
k
=
.
+o
f
n
n n+ n
n
k=1

Exercice n 5 : (**I) (Le lemme de Lebesgue).


o

1) On suppose que f est une fonction de classe C sur [a, b]. Montrer que lim

Zb

+ a

sin(t)f(t) dt = 0.

2) (***) Redmontrer le mme rsultat en supposant simplement que f est continue par morceaux sur [a, b] (commencer
par le cas des fonctions en escaliers).
Exercice no 6 : (***T)
Soit E lensemble des fonctions continues strictement positives sur [a, b]. Soit :

E
f

1) Montrer que (E) nest pas major.

Zb
a

f(t) dt

!R Z
b
a

1
dt
f(t)

! .

2) Montrer que (E) est minor. Trouver m = Inf{(f), f E}. Montrer que cette borne infrieure est atteinte et trouver
toutes les f de E telles que (f) = m.
Exercice no 7 : (****)
Etude complte de la fonction f(x) =

http ://www.maths-france.fr

1
x1

Zx
1

t2
dt.
1 + t8

c Jean-Louis Rouget, 2014. Tous droits rservs.


Exercice no 8 : (***)
x2

Pour x rel, on pose f(x) = e

Zx

et dt.

1) Montrer que f est impaire et de classe C sur R.


2) Montrer que f est solution de lquation diffrentielle y + 2xy = 1.
3) Montrer que lim 2xf(x) = 1.
x+
2

ex
f (x). Montrer que g est strictement dcroissante sur ]0, +[ et que g admet sur ]0, +[ un unique
4) Soit g(x) =
2x
zro not x0 vrifiant de plus 0 < x0 < 1.
5) Dresser le tableau de variations de f.
Exercice no 9 : (***)
1

Soit f une fonction de classe C sur [0, 1] telle que f(0) = 0. Montrer que 2

Z1

f (t) dt 6

Z1

f (t) dt.

Exercice no 10 : (***)
Soit f une fonction continue sur [a, b]. Pour x rel, on pose F(x) =

Zb

|t x|f(t) dt. Etudier la drivabilit de F sur R.

Exercice no 11 : (***)
Soit a un rel strictement positif et f une application
de Zclasse C1 et strictement croissante sur [0, a] telle que f(0) = 0.
Zx
y
Montrer que x [0, a], y [0, f(a)], xy 6 f(t) dt +
f1 (t) dt.
0

Exercice no 12 : (**T)
Soit f continue sur [0, 1] telle que

Z1

f(t) dt =

1
. Montrer que f admet un point fixe.
2

Exercice no 13 : (**T)
Soient f et g deux fonctions
par morceaux et positives sur [0, 1] telles que x [0, 1], f(x)g(x) > 1.
! Zcontinues !
Z1
1
Montrer que
f(t) dt
g(t) dt > 1.
0

Exercice no 14 : (***)
Partie principale quand n tend vers + de un =

n
X

sin

k=1

1
.
(n + k)2

Exercice n 15 : (**)
o

Montrer que

n
X

k=1

sin

k
n2

n+

1
1
+
+o
2 2n

 
1
.
n

Exercice no 16 : (**)
Dterminer les limites quand n tend vers + de
1
1) un =
n!

Z1

Arcsin x dx

2)

Z1
0

xn
dx
1+x

3)

Z
0

n sin x
dx.
x+n

Exercice no 17 : (***)
Etude complte de F(x) =

Z 2x
x

dt

.
4
t + t2 + 1

Exercice n 18 : (***)
o

Trouver toutes les applications continues sur R vrifiant : (x, y) R , f(x)f(y) =

http ://www.maths-france.fr

Z x+y

f(t) dt.

xy

c Jean-Louis Rouget, 2014. Tous droits rservs.


Exercice no 19 : (***)
Soit f une fonction
de classe
C1 sur [a, b] telle que f(a) = f(b) = 0 et soit M = sup{|f (x)|, x [a, b]}.
Z

b

(b a)2


.
Montrer que f(x) dx 6 M
a

4
Exercice no 20 : (**T)

Z
Z
1

1


Dterminer les fonctions f continues sur [0, 1] valeurs dans R vrifiant f(t) dt = |f(t)| dt.
0

0

Exercice no 21 : (***I)
Z x2
dt
.
1) Dterminer lim
x1 x ln t

2) Etude complte de F(x) =

Z x2
x

dt
.
ln t

Exercice no 22 : (****)
Soit f(t) =

t2
si t 6= 0 et 0 si t = 0.
1

et

1) Vrifier que f est continue sur R.


Zx
n
X
1
f(t) dt. Montrer que F a une limite relle quand x tend vers + puis que = 2 lim
2) Soit F(x) =
.
n+
k3
0
k=1

http ://www.maths-france.fr

c Jean-Louis Rouget, 2014. Tous droits rservs.


Planche no 31. Intgration : corrig


Exercice no 1
f est continue sur le segment [a, b] et admet donc un maximum M sur ce segment. Puisque f est strictement positive sur
[a, b], ce maximum est strictement positif.
Soit ]0, 2M[.
Zb

Pour n N , posons un =

(f(x)) dx

!1/n

. Par croissance de lintgrale, on a dj


Zb

un 6

Mn dx

!1/n

= M(b a)1/n ,

(car x [a, b], 0 6 f(x) 6 M x [a, b], (f(x))n Mn par croissance de la fonction t 7 tn sur [0, +[).

Dautre part, par continuit de f en x0 tel que f(x0 ) = M, [, ] [a, b]/ < et x [, ], f(x) > M .
2
Pour n lment de N , on a alors
Z

un >

(f(x)) dx

!1/n

Z 

>

n
dx
M
2

!1/n



( )1/n .
= M
2

En rsum,


( )1/n 6 un 6 M(b a)1/n .
]0, 2M[, (, ) [a, b]2 / < et n N , M
2



Mais, lim M(b a)1/n = M et lim M


( )1/n = M .
n+
n+
2
2

Par suite, n1 N / n > n1 , M(b a)1/n 6 M + et n2 N / n > n2 , (M )( )1/n > M .


2
Soit n0 = Max{n1 , n2 }. Pour n > n0 , on a M 6 un 6 M + . On a montr que

et donc que

> 0, n0 N / n N, (n > n0 |un M| 6 ) ,

lim un = M.

n+

Zb

lim

n+

(f(x))n dx

! n1

= Max f.
[a,b]

Plus gnralement, si g continue sur [a, b], g admet un minimum m1 et un maximum M1 sur cet intervalle, tous deux
strictement positifs puisque g est strictement positive. Pour n dans N , on a
1/n
m1

Zb
a

(f(x)) dx

!1/n

et comme daprs ltude du cas g = 1, on a

Zb

le thorme des gendarmes permet daffirmer que

lim

(f(x)) g(x) dx

1/n
lim m1
n+

n+

lim

n+

Zb

Zb

!1/n
n

(f(x)) dx

Zb

!1/n

(f(x)) g(x) dx

(f(x))n g(x) dx

! n1

1/n
M1

!1/n

Zb

(f(x)) dx ,

1/n
lim M1
n+

Zb

(f(x)) dx

!1/n

= M,

= M. On a montr que

= Max f.
[a,b]

Exercice no 2
1) f est continue sur le segment [0, 1] et est donc borne sur ce segment. Soit M un majorant de |f| sur [0, 1]. Pour n N,

http ://www.maths-france.fr

c Jean-Louis Rouget, 2014. Tous droits rservs.


|un | 6

Z1

t |f(t)| dt 6 M

et comme

lim

n+

M
= 0, on a montr que
n+1

Soit n N. Les deux fonctions t 7


par parties qui fournit

Z1

M
,
n+1

tn dt =

lim un = 0.

n+

tn+1
et f sont de classe C1 sur le segment [0, 1]. On peut effectuer une intgration
n+1

1
Z1
Z1
tn+1
1
f(1)
1
f(t)

tn+1 f (t) dt =
tn+1 f (t) dt.
n+1
n
+
1
n
+
1
n
+
1
0
0
0
Z1
Puisque f est continue sur [0, 1], daprs le dbut de la question lim
tn+1 f (t) dt = 0 ou encore
n+ 0
 
Z1
1
1
n+1
.
t
f (t) dt = o

n+
n+1 0
n
 
f(1)
1
f(1)
f(1)
f(1)
Dautre part, puisque f(1) 6= 0,

ou encore
=
+o
.
n+ n
n+ n
n
+
1
n
+
1
n
 
1
f(1)
ou encore
+o
Finalement, un =
n+ n
n
un =

un

n+

f(1)
.
n

2) Puisque f est de classe C1 sur [0, 1] et que f(1) = 0, une intgration par parties fournit
1
un =
n+1
1

Z1

tn+1 f (t) dt.

Puisque f est de classe C sur [0, 1] et que f (1) 6= 0, le 1) appliqu f fournit

Z1
f (1)
1 f (1)
1
= 2 .
tn+1 f (t) dt
un =
n+
n+1 0
n n
n
Z1
Z1
t
t
1

Par exemple,
tn sin
.
dt
et
tn cos
dt
n+
n+
2
n
2
2n2
0
0

Exercice no 3
1) Pour n > 1,
un =

 
n  2
n
n
k
1 X k
1 X
k
k
1 X 2
,
sin
=
=
f
k
sin
n3
n
n
n
n
n
n
k=1

k=1

k=1

o f(x) = x2 sin(x). un est donc une somme de Riemann pas constant associe la fonction f sur [0, 1]. Puisque la
1
fonction f est continue sur le segment [0, 1] et que le pas
tend vers 0 quand n tend vers +, on sait que un tend vers
n
Z1
0

1

Z
2 1
2
1 2
1
x sin(x) dx = x cos(x) +
x cos(x) dx = +

0

0



1
1
1
1
2
2 1
1
= 2 cos(x) = 2
+



0
1
4
= 3.

2

!
1
Z
1
1 1
x sin(x)
sin(x) dx

0
0

2) On peut avoir envie dcrire :


1
ln(un ) =
n
http ://www.maths-france.fr

n
X

(ln(a + k) ln k)

k=1

n
1 X 
a
.
ln 1 +
n
k
k=1

c Jean-Louis Rouget, 2014. Tous droits rservs.


a
a
La suite de nombres a, ,...,
est une subdivision ( pas non constant) de [0, a] mais malheureusement son pas
2
n
a
a
a = ne tend pas vers 0 quand n tend vers +. On nest pas dans la mme situation que prcdemment.
2
2


vn+1
tend vers un rel positif , alors la
Rappel. (exo classique) Soit v une suite strictement positive telle que la suite
vn

suite ( n vn ) tend encore vers .


n
1 Y

Posons vn =
(a + k) puis un = n vn .
n!
k=1

et donc

vn+1
a+n+1
=
1,
vn
n+1

lim un = 1.

n+

3) Encore une fois, ce nest pas une somme de Riemann. On tente un encadrement assez large : pour 1 6 k 6 n,
n+k
n+k
n+k
.
6 2
6
2
n +n
n +k
n2
En sommant ces ingalits, il vient
n
n
n
X
X
1
1 X
n+k
6
(n
+
k)
6
(n + k),
n2 + n
n2 + k
n2
k=1

k=1

k=1

et donc ((premier terme + dernier terme)nombre de termes/2),


((n + 1) + 2n)n
1 ((n + 1) + 2n)n
1
6 un 6 2
,
n2 + n
2
n
2
3n + 1
3n + 1
3n + 1
3n + 1
3
3
et finalement,
6 un 6
. Or,
et
tendent tous deux vers . Donc, un tend vers .
2(n + 1)
2n
2(n + 1)
2n
2
2
4) Tout dabord
 
n
n
1
1 X
k
1 X
1

s
,
f
=
un =
=


2
2
2
n
n
n
n k
k
k=1
k=1
k=1
1
n
n
X

1
o f(x) =
pour x [0, 1[. un est donc effectivement une somme de Riemann pas constant associe la fonction
1 x2
f mais malheureusement, cette fonction nest pas continue sur le segment [0, 1], ou mme prolongeable par continuit en
1. On sen sort nanmoins en profitant du fait que f est croissante sur [0, 1[.
Puisque f est croissante sur [0, 1[, pour 1 6 k 6 n 2, on a
1
1
s
 2 6
n
k
1
n

Z (k+1)/n

dx,
1 x2

Z k/n

k/n

et pour 1 6 k 6 n 1,
1
1
s
 2 >
n
k
1
n

(k1)/n

1
dx.
1 x2

En sommant ces ingalits, on obtient


un =



Z 1 n1
n1
n1
X Z k/n
1
1
1 X
1
1
s

dx
=
Arcsin
1

>
,
dx
=
 2
n
n
1 x2
1 x2
0
k
k=1 (k1)/n
k=1
1
n

et

http ://www.maths-france.fr

c Jean-Louis Rouget, 2014. Tous droits rservs.


Z 1 n1
n2
1
1
1
1 X
1
s
p

dx +
6
+


2
2
2
1
2
n
2n 1
1x
n (n 1)
n
k
k=1
1
n


1
1
1
Arcsin +
= Arcsin 1
.
n
n
2n 1

un =

Quand n tend vers +, les deux membres de cet encadrement tendent vers Arcsin 1 = , et donc un tend vers .
2
2

5) Pour 1 6 k 6 n, k 1 6 E( k) 6 k, et en sommant,
!
!
n
n
X
X
1
1
1

k 6 un 6
k .
n n
n
n n
k=1
k=1
Z1
n r
n

1 X
1
1 X k
3
Quand n tend vers +, tend vers 0 et la somme de Riemann
k=
x dx = .
tend vers
n
n
2
n
n n k=1
0
k=1
3
Donc, un tend vers .
2
6) L, on a une somme de Riemann pas constant associe une fonction continue sur un segment :

1
Z1
n
x2
1
ln 3
1 X (k/n)2
3
tend
vers
dx
=
ln
|8x
+
1|
.
=
un =
3+1
n
1 + 8(k/n)3
8x
24
12
0
0
k=1

7) un =

n1
X
k=0

n1
1
1
2 X
=
2(k + n) + 1
2 n
k=0

1
1
tend vers
2k + 1
2
2+
n

Z2
0

1
ln 2
1
dx = (ln 4 ln 2) =
.
2+x
2
2

1
8) Soit f(x) = 2 e1/x si x > 0 et 0 si x = 0. f est continue sur [0, 1] (thorme de croissances compares). Donc,
 x
Z1
Z1
n
k
1 X
tend vers
f(x) dx. Pour x [0, 1], posons F(x) = f(t) dt. Puisque f est continue sur [0, 1], F lest
f
un =
n
n
0
x
k=1
et
Z1

f(x) dx = F(0) =

Donc, un tend vers

lim

x0, x>0

F(x) =

lim

x0, x>0

1
quand n tend vers +.
e

i1
h
e1/t =
x

lim

x0, x>0

(e1 e1/x ) =

1
.
e

Exercice no 4
Supposons f de classe C2 sur [0, 1]. Soit F une primitive de f sur [0, 1]. Soit n un entier naturel non nul.

un =

Z1
0

 
 
 ! n1
n1
n1  
X Z (k+1)/n
X  k + 1
1
k
k
1
k
k
1 X
=
=
F
F
F
.
f(t) dt f
f
f(t) dt
n
n
n
n
n
n
n
n
k/n
k=0

k=0

k=0

f est de classe C2 sur le segment [0, 1]. Par suite, F(3) = f est dfinie et borne sur ce segment. En notant M2 la borne
suprieure de |f | sur [0, 1], lingalit de Taylor-Lagrange lordre 3 applique F sur le segment [0, 1] fournit


 
 
 
3


F k + 1 F k 1 F k 1 F k 6 (1/n) M2 ,

n
n
n
n
2n2
n
6
et donc,

n1  

 

 
  n1 
 
 
 
X
1 k
1 k X
1 k
1 k
k
k
k+1
k+1

F
F
2F
F
F
2F
F
6

F

n
n
n
n
2n
n
n
n
n
n
2n
n
k=0

k=0

n1
X
k=0

http ://www.maths-france.fr

(1/n)3 M2
M2
=
.
6
6n2

c Jean-Louis Rouget, 2014. Tous droits rservs.


n1
X


 
 
 
 
1 k
1 k
k
1
k+1
F
F
2F
= O
,
Ainsi,
F
n+
n
n
n
n
2n
n
n2
k=0
 
 
 
n1
X  k + 1
1 k
1 k
1
k
F
ou encore
F
F
2F
= o( ), ou enfin,
n+
n
n
n
n
2n
n
n
k=0


n1
X 1
1
k
.
F
(
)
+
o
un =
n+
2n2
n
n


k=0

Maintenant,
n1
X
k=0

 
n1
1
1 X k
1 k
.
F
(
)
=

f
2n2
n
2n n
n
k=0

 
Z1
n1
1 X k
Or, la fonction f est continue sur le segment [0, 1]. Par suite, la somme de Riemann
tend vers f (t) dt =
f
n
n
0
k=0
f(1) f(0) et donc

 
 
n1
1
1 X k
f(1) f(0)
1
1
=
.

(f(1) f(0) + o(1)) =


+o
f
n+
2n n
n n+ 2n
2n
n
k=0

Finalement,
Z1
0

 
n1  
f(1) f(0)
1 X
1
k
=
.
+o
f
f(t) dt
n
n
2n
n
k=0

Exercice no 5
1) Puisque f est de classe C1 sur [a, b], on peut effectuer une intgration par parties qui fournit pour > 0 :
Z


!
Zb
Zb
b
1
1



b

|f(a)| + |f(b)| +
|f (t)| dt .
f(t) sin(t) dt = ( [cos(t)f(t)]a + f (t) cos(t) dt) 6
a


a
a
Zb
Cette dernire expression tend vers 0 quand tend vers +, et donc
f(t) sin(t) dt tend vers 0 quand tend vers +.
a

2) Si f est simplement suppose continue par morceaux, on ne peut donc plus effectuer une intgration par parties.
Z b
cos(a) cos(b) 2


6 .
Le rsultat est clair si f = 1, car pour > 0, sin(t) dt =

a

Le rsultat stend aux fonctions constantes par linarit de lintgrale puis aux fonctions constantes par morceaux par
additivit par rapport lintervalle dintgration, cest--dire aux fonctions en escaliers.
Soit alors f une fonction continue par morceaux sur [a, b].
Soit > 0. On sait quil existe une fonction en escaliers g sur [a, b] telle que x [a, b], |f(x) g(x)| 6

.
2(b a)

Pour > 0, on a alors


Z
Z

Zb
b
b




g(t) sin(t) dt
f(t) sin(t) dt = (f(t) g(t)) sin(t) dt +
a
a

a
Z

Z

Zb
b

b





+ g(t) sin(t) dt
6
|f(t) g(t)| dt + g(t) sin(t) dt 6 (b a)




2(b

a)
a
a
a
Z


b

= + g(t) sin(t) dt .

2 a

Maintenant, le rsultat tant dj tabli pour les fonctions en escaliers,


Z

!
b



.
A > 0/ R, > A g(t) sin(t) dt 6
a
2
Z

b



Pour > A, on a alors f(t) sin(t) dt 6 + = . On a montr que
a
2 2
http ://www.maths-france.fr

c Jean-Louis Rouget, 2014. Tous droits rservs.


et donc que

Zb

Z

b



> 0, A > 0/ R, ( > A f(t) sin(t) dt 6 ),
a

f(t) sin(t) dt tend vers 0 quand tend vers +.

Exercice no 6
1) Soient m un rel strictement positif et, pour t R, fm (t) = emt . fm est bien un lment de E et de plus,
1 mb
(e
ema )(ema emb )
m2




1
= 2 em(a+b)/2 em(ba)/2 em(ba)/2 em(a+b)/2 em(ba)/2 em(ba)/2
m
4 sh2 (m(b a)/2)
.
=
m2

(fm ) =

Cette expression tend vers + quand m tend vers + et (E) nest pas major.
2) Soit f continue et strictement positive sur [a, b]. Lingalit de Cauchy-Schwarz montre que :

!2
!2
! Z
Z b p
Zb p
2
b
1
1
p
(f) =
dt
dt >
= (b a)2 ,
f(t)
f(t) p
dt
f(t)
f(t)
a
a
a

p
1
ou encore si et seulement si
f(t) = p
f(t)
R+ / t [a, b], f(t) = , cest--dire que f est une constante strictement positive.

avec galit si et seulement si R+ / t [a, b],

Ceci montre que (E) admet un minimum gal (b a)2 et obtenu si et seulement si f est une constante strictement
positive.
Exercice no 7

Zx
t2
Pour t rel, posons g(t) =
puis, pour x rel, G(x) =
g(t) dt. Puisque g est dfinie et continue sur R, G est
1 + t8
1
dfinie sur R et de classe C1 et G = g (G est la primitive de g sur R qui sannule en 1). Plus prcisment, g est de classe
C sur R et donc G est de classe C sur R. Finalement, f est dfinie et de classe C sur ] , 1[]1, +[.
Etude en 1.

f(x) =

G(x)
=
x 1 x1

G(1) + G (1)(x 1) +

G (1)
(x 1)2 + o((x 1)2 )
g (1)
2
= g(1) +
(x 1) + o((x 1)).
x1
x1
2

Donc, f admet en 1 un dveloppement limit dordre 1. Par suite, f se prolonge par continuit en 1 en posant
1
g (1)
f(1) = g(1) = puis le prolongement est drivable en 1 et f (1) =
. Or, pour tout rel x,
2
2


1
4x7
1 x8

g (x) = 2x
= 2x
+ x2
8
8
8
1+x
(1 + x ) 1 + x
(1 + x8 ) 1 + x8
et g (1) = 0. Donc, f (1) = 0.
G (x)(x 1) G(x)
.
(x 1)2
Sur R \ {1}, f (x) est du signe de h(x) = G (x)(x 1) G(x) dont la drive est
Drive. Variations Pour x 6= 1, f (x) =


2x(x 1) 1 x8

.
h (x) = G (x)(x 1) + G (x) G (x) = (x 1)g (x) =
(1 + x8 ) 1 + x8

Pour tout rel x, h (x) est du signe de 2x(x 1)(1 x8 ) ou encore du signe de 2x(1 + x)(x 1)2 . h est donc dcroissante
sur ] , 1] et sur [0, +[ et croissante sur [1, 0].
1
1
Maintenant, quand x tend vers + (ou ), G (x)(x 1) = g(x)(x 1) x 2 = et donc G (x)(x 1) tend vers 0.
x
x
Ensuite, pour x > 1
http ://www.maths-france.fr

c Jean-Louis Rouget, 2014. Tous droits rservs.


Zx

1
t2
dt = 1 6 1,
8
x
t
1
et G est borne au voisinage de + (ou de ). Comme G est croissante sur R, G a une limite relle en + et en .
Cette limite est strictement positive en + et strictement ngative en . Par suite, h a une limite strictement positive
en et une limite strictement ngative en +. Sur [0, +[, h est dcroissante et sannule en 1. Donc, h est positive
sur [0, 1] et ngative sur [1, +[. Ensuite,
0 6 G(x) 6

h(1) =

Z1

t2

dt 2 = 2
8
1+t

Z1
0

t2

dt 2 < 2
8
1+t

Z1
0

1
dt 2 = 0,
2

et h(1) < 0. h sannule donc, une et une seule fois sur ] , 1[ en un certain rel et une et une seule fois sur ] 1, 0[
en un certain rel . De plus, h est strictement positive sur ] , [, strictement ngative sur ], [, strictement positive
sur ], 1[ et strictement ngative sur ]1, +[.
f est strictement croissante sur ] , ], strictement dcroissante sur [, ], strictement croissante sur [, 1] et strictement
dcroissante sur [1, +[.
Etude en linfini. En + ou , G a une limite relle et donc f tend vers 0.
Exercice no 8
t2

1) La fonction t 7 e

est de classe C

mme de f.

sur R. Donc, la fonction x 7

La fonction t 7 et est paire et donc la fonction x 7

impaire.

x2

2) Pour x rel, f (x) = 2xe

Zx

Zx
0

Zx

et dt est de classe C sur R et il en est de

et dt est impaire. Comme la fonction x 7 ex est paire, f est

et dt + ex ex = 2xf(x) + 1.

3) Pour x > 1, une intgration par parties fournit :


Zx

t2

dt =

Zx
1

x

Z
Z
2
2
2
ex
e 1 x et
1 t2
1 x et
1
t2
dt
=
dt,
2te dt =
e

+
+
2t
2t
2 1 t2
2x
2 2 1 t2
1

et donc,


Zx
Z1


2
2
2
2


|1 2xf(x)| = 1 2xex
et dt 2xex
et dt


1
0


Z1
Z x t2


e


x2
t2
x2
x2
dt

2xe
e
dt
= 1 1 + exe
+ xe

2


t
0
1
Z1
Z x t2
2
2
2
2
e
dt + exex + 2xex
et dt.
6 xex
2
t
0
1
Les deux derniers termes tendent vers 0 quand x tend vers + daprs un thorme de croissances compares. Il reste le
premier. Pour x > 2,

0 6 xex

Zx
1

Z x1

2
et
dt + xex
2
t

Zx

et
dt
2
1
x1 t
Zx
2
1
e(x1)
x2 x2
x2
+ xe
e
dt
6 xe
(x 2)
2
12
t
x1


1
1
1
= x(x 1)e2x+1 +

.
= x(x 2)e2x+1 + x
x1 x
x1

2
et
dt = xex
2
t

Zx

et
dt tend vers 0 quand x tend
2
1 t
1
vers +. Finalement, 1 2xf(x) tend vers 0 quand x tend vers +, ou encore, f(x)
.
x+ 2x
Zx
2
2
2
ex
ex
(1 2xf(x)) =
et dt puis,
4) Pour x > 0, g(x) =
2x
2x
0
x2

Cette dernire expression tend vers 0 quand x tend vers +. On en dduit que xe

http ://www.maths-france.fr

c Jean-Louis Rouget, 2014. Tous droits rservs.


2
ex
ex
ex = 2 < 0.
2
2x
2x
g est donc strictement dcroissante sur ]0, +[ et donc, g sannule au plus une fois sur ]0, +[. Ensuite, f (1) = 12f(1) =
Z1
Z1
2
2
e
1 2e1 et dt. Or, la mthode des rectangles fournit et dt = 1, 44... > 1, 35... = , et donc f (1) < 0 puis g(1) < 0.
2
0
0
1
1
f (0) =
, g(0+ ) = +.
Enfin, comme en 0+ , g(x)
2x
2x
Donc, g sannule exactement une fois sur ]0, +[ en un certain rel x0 de ]0, 1[.
2

g (x) = ex

5) g est strictement positive sur ]0, x0 [ et strictement ngative sur ]x0 , +[. Il en de mme de f . f est ainsi strictement
croissante sur [0, x0 ] et strictement dcroissante sur [x0 , +[. Par parit, f est strictement croissante sur [x0 , 0] et
strictement croissante sur ], x0 ].
Exercice no 9
Pour t [0, 1], puisque f(0) = 0,
f2 (t) =

Z t

=t

0
Zt

2 Z t
 Z t

2
f (u) du
6
f (u) du
1 du
2

f (u) du 6 t

Z1

(daprs lingalit de Cauchy-Schwarz)

f (u) du,

et donc, par croissance de lintgrale,


Z1

f (t) dt 6

Z1

Z1

f (u) du

Z1

dt =

!Z

f (u) du

t dt =
0

1
2

Z1

f (u) du.

Exercice no 10
Pour x rel donn, la fonction t 7 |t x|f(t) est continue sur [a, b] et donc F(x) existe.
Zb
Zb
Zb
Pour x 6 a, F(x) = (t x)f(t) dt = x f(t) dt + tf(t) dt. F est donc de classe C1 sur ] , a] en tant que fonction
a
a
aZ
Zb
b

affine et, pour x < a, F (x) = f(t) dt et Fg (a) = f(t) dt.


a

Zb

Zb

De mme, pour x > b, F(x) = x f(t)dt tf(t) dt. F est donc de classe C1 sur [b, +[ en tant que fonction affine et,
a
Z ba
Zb

f(t) dt.
pour x > b, F (x) =
f(t) dt et Fd (b) =
a

Enfin, si a 6 x 6 b,
F(x) =

Zx

(x t)f(t) dt +

Zb

(t x)f(t) dt = x

Zx
a

f(t) dt

Zb

f(t) dt

Zx

tf(t) dt +

Zb

tf(t) dt.

Puisque les fonctions t 7 f(t) et t 7 tf(t) sont continues sur [a, b], F est de classe C1 sur [a, b] et, pour a 6 x 6 b,
F (x) =
=

Zx

a
Zx
a

et en particulier, Fd (a) =

Zb
a

f(t) dt
f(t)dt

Zb

x
Zb

f(t) dt + x(f(x) (f(x))) xf(x) xf(x)

f(t) dt.

f(t) dt = Fg (a) et Fg (b) =

Zb
a

f(t) dt = Fd (b).

F est continue ] , a], [a, b] et [b, +[ et donc sur R. F est de classe C1 sur ] , a], [a, b] et [b, +[. De plus,
Fg (a) = Fd (a) et Fg (b) = Fd (b). F est donc de classe C1 sur R.
Exercice no 11
Puisque f est continue et strictement croissante sur [0, a], f ralise une bijection de [0, a] sur f([0, a]) = [0, f(a)].
http ://www.maths-france.fr

c Jean-Louis Rouget, 2014. Tous droits rservs.


Zy

f(t) dt +
f1 (t) dt xy. Puisque f est continue sur [0, a], on sait
Zy 0
est continue sur [0, f(a)] et donc la fonction y 7
f1 (t) dt est dfinie et de classe C1 sur [0, f(a)]. Donc g est

Soit x [0, a]. Pour y [0, f(a)], posons g(y) =


que f1

Zx
0

de classe C1 sur [0, f(a)] et pour y [0, f(a)], g (y) = f1 (y) x.

f tant strictement croissante sur [0, a], g (y) > 0 f1 (y) > x y > f(x). Par suite, g est strictement ngative
sur [0, f(x)[ et strictement positive sur ]f(x), f(a)]. Par suite, g est strictement dcroissante sur [0, f(x)] et strictement
Zx
Z f(x)
croissante sur [f(x), f(a)]. g admet en y = f(x) un minimum global gal g(f(x)) =
f(t) dt +
f1 (t) dt xf(x).
0

Notons h(x) cette expression.


f est continue sur [0, a]. Donc, la fonction x 7

classe C1 sur [0, f(a)]. On en dduit que x 7


x [0, a],

Zx

f(t) dt est de classe C1 sur [0, a]. Dautre part, y 7

Z 0f(x)

Zy

f1 (t) dt est de

f1 (t) dt est de classe C1 sur [0, a]. Il en est de mme de h et pour

h (x) = f(x) + f (x)f1 (f(x)) f(x) xf (x) = 0.


h est donc constante sur [0, a] et pour x [0, a], h(x) = h(0) = 0.
La fonction h admet donc un minimum global gal 0 et on a montr que
Zx

(x, y) [0, a] [0, f(a)],

Zy

f(t) dt +

f1 (t) dt xy >

Zx

f(t) dt +

Z f(x)

f1 (t) dt xf(x) = 0.

Exercice no 12
Pour x [0, 1], posons g(x) = f(x) x. g est continue sur [0, 1] et
Z1

g(x) dx =

Z1

f(x) dx

Z1

x dx =

1 1
= 0.
2 2

Si g est de signe constant, g tant de plus continue sur [0, 1] et dintgrale nulle sur [0, 1], on sait que g est nulle. Sinon,
g change de signe sur [0, 1] et le thorme des valeurs intermdiaires montre que g sannule au moins une fois. Dans tous
les cas, g sannule au moins une fois sur [0, 1] ou encore, f admet au moins un point fixe dans [0, 1].
Exercice no 13
Daprs lingalit de Cauchy-Schwarz,
Z1
0

f(t) dt

! Z
1

g(t) dt

! Z
!
Z 1 p
1 p
2
2
f(t)
g(t)
dt
dt
0

>

!2
Z1 p
p
f(t) g(t) dt
>
0

Z1
0

1 dt

!2

= 1.

Exercice no 14
h i
Soit x [0, 1] 0, . La formule de Taylor-Laplace lordre 1 fournit
2
Zx
sin x = x (x t) sin t dt 6 x,
0

i
car pour t [0, x], (x t) > 0 et pour t [0, x] 0, , sin t > 0.
2
De mme, la formule de Taylor-Laplace lordre 3 fournit
Zx
x3
x3
(x t)3
sin x = x
+
sin t dt > x .
6
6
6
0
h

Donc, x [0, 1], x

x3
6 sin x 6 x.
6

Soient alors n > 1 puis k J1, nK. On a 0 6


http ://www.maths-france.fr

1
6 1 et donc
(n + k)2
9

c Jean-Louis Rouget, 2014. Tous droits rservs.


1
1

6 sin
(n + k)2
6(n + k)6

1
(n + k)2

1
,
(n + k)2

puis en sommant
n
X

k=1

n
n
n
X
X
1
1X
1
1
1

6
sin
6
.
(n + k)2
6
(n + k)6
(n + k)2
(n + k)2
k=1

k=1

k=1

Maintenant,
n
1
1 X
1
1
=

2

2
(n + k)
n n
k
k=1
k=1
1+
n
n
X

n+

Z1

1
n

!
 
1
1
1
.
dx + o(1)
=
+o
2
n+
(1 + x)
2n
n

Dautre part,
06

n
X

k=1

et donc,

n
X

k=1

1
1
1
6n 6 =
,
6
6(n + k)
6n
6n5

 
1
1
.
=
o
6(n + k)6 n+
n

1
1
1
1
1

) = 2n(
= 1 + o(1). Mais alors,
+ o( )) = 1 + o(1) et que 2n
(n + k)2
6(n + k)6
2n
n
(n + k)2
n
X
1
daprs le thorme des gendarmes, 2n
sin
tend vers 1 quand n tend vers +, ou encore
(n + k)2

On en dduit que 2n(

k=1

n
X

sin

k=1

1
(n + k)2

n+

1
.
2n

Exercice n 15
o

1re solution. Soit n N . Comme dans lexercice prcdent, la formule de Taylor-Laplace fournit pour tout rel x
x3
de [0, 1], x
6 sin x 6 x. Donc,
6
n
n
n
n
X
X
X
1 X k3
k
k
k

6
sin
6
.
n2 6
n6
n2
n2
k=1

k=1

n
X

k=1

k=1

k=1

k
n(n + 1)
1
1
=
= +
et dautre part,
n2
2n2
2 2n
06

n
X
n3
1
k3
6 n 6 = 2.
6
n
n
n

k=1

Donc,

1
1
1
6
+

2 2n n2

n
X

sin

k=1

k
1
1
6 +
puis
n2
2 2n

!
n
X
1
1
1
k
6 0.
6n
sin 2
n
n
2 2n
k=1
!
n
X
k
1
1
Le thorme des gendarmes montre que lim n
sin 2
= 0 ou encore
n+
n
2 2n
k=1
 
n
X
1
1
1
k
.
+
=o
sin 2 =
n+
n
2 2n
n
k=1

2me solution. Soit n N .

http ://www.maths-france.fr

10

c Jean-Louis Rouget, 2014. Tous droits rservs.


n
X

k=1

sin

k
= Im
n2

n
X

eik/n

k=1

= Im ei/n

ni/n2

1e
1 ei/n2

n+1
1
1
sin
sin
2

i(1+ n2 21 )/n2
2n
2n
2n
=
= Im e
1
1
sin 2
sin 2
2n
2n


sin

  

1
1
1
1
1
+
+o
+o
2n 2n2
n2
2n
n2
 
=
n+
1
1
+
o
2
2n
n3
 1
  
  

1
1
1
1
1
1+o
+o
=
1+ +o
n+
n
n
2
n
n
 
1
1
1
=
.
+
+o
n+ 2
2n
n


n+

1
1
+ +o
2 n

 
  
1
1
1+o
n
n

Exercice no 16
h i
 n
1) Soit n N. Pour x 0, , 0 6 Arcsinn x 6
et donc, par croissance de lintgrale,
2
2
Z
1  n
1 1  n
dx =
.
0 6 un 6
n! 0 2
n! 2
1  n
tend vers 0 quand n tend vers +. Daprs le thorme des
Daprs un thorme de croissances compares,
n! 2
gendarmes, un tend vers 0 quand n tend vers +.
Z1 n
Z1 n
x
1
1
x
2) Pour tout n N , 0 6
dx 6
dx =
. Comme
tend vers 0 quand n tend vers +,
n+1
n+1
0 1+x
0 1+0
Z1 n
x
dx tend vers 0 quand n tend vers +.
1
+x
0
3) Soit n N .
Z
Z
Z
Z
Z
x sin x


x sin x
n sin x
2

=
6
6


dx

sin
x
dx
dx
dx
dx =
.






x+n
n
0
0
0 0+n
0 x+n
0 x+n
Z
Z
2
n sin x
Or,
tend vers 0 quand n tend vers +, et donc
dx tend vers
sin x dx = 2 quand n tend vers +.
n
0 x+n
0
Exercice no 17
1
. g est continue sur R (car pour tout rel t, t4 + t2 + 1 > 0) et admet donc des
Pour t R, posons g(t) =
4
t + t2 + 1
primitives sur R. Soit G une primitive de g sur R.
Dfinition, drivabilit, drive.
Puisque g est continue sur R, F est dfinie sur R et pour tout rel x, F(x) = G(2x) G(x). G est de classe C1 sur R et
donc F est de classe C1 sur R et pour tout rel x,
F (x) = 2G (2x) G (x) = 2g(2x) g(x) =

16x4

2
1

.
2
4
+ 4x + 1
x + x2 + 1

Parit. Soit x R. En posant t = u et donc dt = du, on obtient, en notant que g est paire
F(x) =

Z 2x
x

g(t) dt =

Z 2x
x

g(u) (du) =

Z 2x

g(u) du = F(x).

F est donc impaire.


Variations. Pour tout x rel,



 p

p
2
1
sgn(F (x)) = sgn
= sgn 2 x4 + x2 + 1 16x4 + 4x2 + 1

16x4 + 4x2 + 1
x4 + x2 + 1

= sgn 4(x4 + x2 + 1) (16x4 + 4x2 + 1) (par croissance de t 7 t2 sur R+ )

= sgn(12x4 + 3) = sgn(1 4x4 ) = sgn(1 2x2 ).

http ://www.maths-france.fr

11

c Jean-Louis Rouget, 2014. Tous droits rservs.







1
1
1
1
F est donc strictement croissante sur , et strictement dcroissante sur , et sur , + .
2
2
2
2
Etude en +. Pour tout x > 0,
0 6 F(x) 6

Z 2x
x

1
dt 6
t4

Z 2x
x

2x x
1
1
dt =
= .
2
4
x
x
x

1
tend vers 0 quand x tend vers +, le thorme des gendarmes permet daffirmer que lim F(x) = 0.
x+
x
Graphe. Le graphe de F a lallure suivante

Comme

1
Exercice no 18
f est continue sur R et admet donc des primitives sur R. Soit F une primitive donne de f sur R. Notons () la relation :
(x, y) R2 , f(x)f(y) = F(x + y) F(x y).
Pour x = y = 0, on obtient : x R, f(0) = 0. Puis x = 0 fournit y R, F(y) F(y) = 0. F est donc ncessairement
paire et sa drive f est ncessairement impaire.
La fonction nulle est solution du problme. Soit f une ventuelle solution non nulle. Il existe alors un rel y0 tel que
f(y0 ) 6= 0. Pour tout rel x, on a alors
Z x+y0
1
1
f(x) =
f(t) dt =
(F(x + y0 ) F(x y0 )).
f(y0 ) xy0
f(y0 )
1
(F(x + y0 ) F(x y0 ))
f(y0 )
et donc de f. Mais alors, F est de classe C2 sur R et donc f lest aussi (f est en fait de classe C par rcurrence).
f est continue sur R et donc F est de classe C1 sur R. Il en est de mme de la fonction x 7

En drivant () y fix, on obtient f (x)f(y) = f(x + y) f(x y) (), mais en drivant x fix, on obtient aussi
f(x)f (y) = f(x + y) + f(x y) ( ). En redrivant () y fix, on obtient f (x)f(y) = f (x + y) f (x y) et en
drivant ( ) x fix, on obtient f(x)f (y) = f (x + y) f (x y). Mais alors,
(x, y) R2 , f (x)f(y) = f(x)f (y),
et en particulier,
x R, f (x)

f (y0 )
f(x) = 0.
f(y0 )

On a montr que si f est solution du problme, il existe un rel tel que f est solution de lquation diffrentielle y y = 0
(E).

si > 0, en posant k = , (E) scrit y k2 y = 0. Les solutions de (E) sont les fonctions de la forme
x 7 A sh(kx) + B ch(kx), (A, B) R2 et les solutions impaires de (E) sont les fonctions de la forme x 7 A sh(kx), A R.
Rciproquement, soit k un rel strictement positif. Pour A R (on sait que la fonction nulle est solution) et x R,
posons f(x) = A sh(kx). Alors
Z x+y
A
2A
2
f(t) dt = (ch(k(x + y)) ch(k(x y))) =
sh(kx) sh(ky) =
f(x)f(y).
k
k
kA
xy
2
2
= 1 ou encore A = .
f est solution si et seulement si
kA
k

si < 0, en posant k = , (E) scrit y + k2 y = 0. Les solutions de (E) sont les fonctions de la forme
x 7 A sin(kx) + B cos(kx), (A, B) R2 et les solutions impaires de (E) sont les fonctions de la forme x 7 A sin(kx), A R.
Rciproquement, soit k un rel strictement positif. Pour A R et x R, posons f(x) = A sin(kx). Alors
http ://www.maths-france.fr

12

c Jean-Louis Rouget, 2014. Tous droits rservs.


Z x+y

f(t) dt =

xy

2A
2
A
(cos(k(x y)) cos(k(x + y))) =
sin(kx) sin(ky) =
f(x)f(y).
k
k
kA

2
2
= 1 ou encore A = .
kA
k
si = 0, (E) scrit y = 0. Les solutions de (E) sont les fonctions affines et les solutions impaires de (E) sont les
fonctions de la forme x 7 Ax, A R. Rciproquement, si f(x) = Ax
Z x+y
A
2
f(t) dt = ((x + y)2 (x y)2 ) = 2Axy = f(x)f(y),
2
A
xy

f est solution si et seulement si

et f est solution si et seulement si A = 2.


2
2
Les solutions sont la fonction nulle, la fonction x 7 2x, les fonctions x 7 sin(kx), k > 0, et les fonctions x 7 sh(kx),
k
k
k > 0.
Exercice no 19
Soit F une primitive de f sur [a, b]. F est de classe C2 sur le segment [a, b] et lingalit de Taylor-Lagrange permet
dcrire



2


F a + b F(a) b a F (a) 6 1 (b a) sup{|F (x)|, x [a, b]}.

2
2
2
4
Mais F (a) = f(a) = 0 et F = f . Donc,



a+b
1 M(b a)2
F(
6
)

F(a)
.

2
2
4

De mme, puisque F (b) = f(b) = 0,



a+b
1 (b a)2
F(
) F(b) 6 M
.

2
2
4

Mais alors,

Z



 


b



a+b
a + b


F(a)
+
F
f(t) dt = |F(b) F(a)| 6 F(b) F



a

2
2
6

1 (b a)2 1 (b a)2
(b a)2
M
+ M
=M
.
2
4
2
4
4

Exercice no 20
Z1
Si
f(t) dt > 0,
0

Z
Z
Z1
Z1
Z1
1

1


f(t) dt = |f(t)| dt f(t) dt = |f(t)| dt (|f(t)| f(t)) dt = 0
0

0
0
0
0

Si

Z1

f(t) dt 6 0, alors

f 6 0.

Z1

|f| f = 0 (fonction continue positive dintgrale nulle)


f = |f| f > 0.

f(t) dt > 0 et daprs ce qui prcde, f est solution si et seulement si f = | f| ou encore

En rsum, f est solution si et seulement si f est de signe constant sur [0, 1].
Exercice no 21
1) Si x > 1, la fonction t 7

Z x2
1
dt
est continue sur ]1, +[ et [x, x2 ] ]1, +[. Par suite,
existe. De plus,
ln t
ln
t
x
Z x2
Z x2
Z x2
Z x2
1
dt
1
1
2
t
dt 6
=
dt 6 x
dt.
x
t ln t
x ln t
x
x t ln t
x t ln t

Mais,
http ://www.maths-france.fr

13

c Jean-Louis Rouget, 2014. Tous droits rservs.


Z x2
x



2 ln x
1
x2
= ln 2.
dt = [ln | ln t|]x = ln | ln(x2 )| ln | ln(x)| = ln
t ln t
ln x

Donc, x > 1, x ln 2 6 F(x) 6 x2 ln 2. Daprs le thorme des gendarmes,

lim

x1, x>1

F(x) = ln 2.

1
Si 0 < x < 1, on a x < x puis [x , x] ]0, 1[. Donc, t 7
est continue sur [x2 , x] et F(x) =
ln t
2

Zx

x2

1
dt existe.
ln t

Pour t [x2 , x], on a t ln t < 0 et x2 6 t 6 x. Par suite,


x
puis,

Zx

x2

1
dt 6
t ln t

On obtient alors

Zx

x2

lim

1
dt, et finalement,
t ln t
Z x2
Z x2
Z x2
1
1
2 1
2
x
dt 6 F(x) =
dt 6
dt = x ln 2.
x
x ln 2 =
t ln t
t ln t
x ln t
x
x

1
dt 6
ln t

x1, x<1

Zx

1
1
1
1
6t
=
6 x2
,
t ln t
t ln t
ln t
t ln t

x2

x2

F(x) = ln 2 et finalement, lim F(x) = ln 2. On en dduit que F se prolonge par continuit en 1 en


x1

posant F(1) = ln 2 (on note encore F le prolongement obtenu).


2) Domaine de dfinition. On a dj vu que F est dfinie (au moins) sur ]0, +[ (F dsignant le prolongement). Il ne
parait pas encore possible de donner un sens F(0) et encore moins F(x) quand x < 0, car alors [x, 0] est un intervalle
1
nest mme pas dfinie.
de longueur non nulle contenu dans [x, x2 ], sur lequel la fonction t 7
ln t
DF =]0, +[.

1
et notons G une primitive de g sur cet ensemble.
ln t
Alors, pour x dans ]0, 1[]1, +[, F(x) = G(x2 ) G(x). On en dduit que F est de classe C1 sur ]0, 1[]1, +[ et que pour
x dans ]0, 1[]1, +[,
Drivabilit et drive. Pour t ]0, 1[]1, +[, posons g(t) =

F (x) = 2xg(x2 ) g(x) =

1
x1
2x

=
.
2
ln(x ) ln x
ln x

x1
tend vers 1. Ainsi, F est continue sur ]0, +[, de classe C1 sur ]0, 1[]1, +[ et F
ln x
a une limite relle en 1. Un thorme classique danalyse permet daffirmer que F est de classe C1 sur DF et en particulier,
drivable en 1 avec F (1) = 1.

x 1 si x 6= 1
x ]0, +[, F (x) =
.
ln x

1 si x = 1.
Maintenant, quand x tend vers 1,

Variations. Si x > 1, x 1 > 0 et ln x > 0 et si 0 < x < 1, x 1 < 0 et ln x < 0. Dans tous les cas (0 < x < 1, x = 1,
x > 1) F (x) > 0. F est strictement croissante sur ]0, +[.
Etude en +. On a vu que x > 1, F(x) > x ln 2 et donc lim F(x) = +. Plus prcisment, pour x > 1,
x+

1
F(x)
=
x
x

Z x2
x

x2 x
x1
1
dt >
=
.
ln t
x ln x
ln x

x1
F(x)
Comme
tend vers + quand x tend vers + daprs un thorme de croissances compares, on en dduit que
ln x
x
tend vers + quand x tend vers + et donc que la courbe reprsentative de F admet en + une branche parabolique
de direction (Oy).
1
1
1
6
6
, puis
Etude en 0. Pour x ]0, 1[ et t [x2 , x], on a 2 ln x = ln(x2 ) 6 ln t 6 ln x < 0 et donc
ln x
ln t
2 ln x
Zx
1
1
1
(x x2 )
6
dt 6 (x x2 )
et finalement,
ln x
2 ln x
x2 ln t
x x2
x x2
x ]0, 1[,
F(x)
.
2 ln x
ln x

http ://www.maths-france.fr

14

c Jean-Louis Rouget, 2014. Tous droits rservs.


On obtient dj lim F(x) = 0. On peut prolonger F par continuit en 0 en posant F(0) = 0. Ensuite,
x0

est compris entre

F(x)
F(x) F(0)
=
x0
x

1x
1x
et
. Comme ces deux expressions tendent vers 0 quand x tend vers 0, on en dduit que
2 ln x
ln x

F(x) F(0)
tend vers 0 quand x tend vers 0. F est donc drivable en 0 et F (0) = 0.
x0
Graphe.
6
5
4
3
2
1

Exercice no 22
1) f est continue sur R en tant que quotient de fonctions continues sur R dont le dnominateur ne sannule pas sur R .
t2
= t et lim f(t) = 0 = f(0). Ainsi, f est continue en 0 et donc sur R.
Dautre part, f(t)
t0, t6=0
x0 t
2) f est continue sur R et donc F est dfinie et de classe C1 sur R. De plus, F = f est positive sur [0, +[, de sorte que F
est croissante sur [0, +[. On en dduit que F admet en + une limite dans ] , +].

t2
tend vers 0 quand t tend vers +, daprs un
1
t2
thorme de croissances compares. Par suite, il existe un rel A > 0 tel que pour t > A, 0 6 t2 t
6 1 ou encore
e 1
1
0 6 f(t) 6 2 . Pour x > A, on a alors
t
Vrifions alors que F est majore sur R. On constate que t2

et

Zx

Zx
ZA
t2
1
f(t)
dt
+
dt
dt
6
t
2
A e 1
A t
0
0
ZA
ZA
1
1
1
f(t) dt + 6
=
f(t) dt + .
A
x
A
0
0

F(x) =

F est croissante et majore par

ZA

ZA

f(t) dt +

f(t) dt +

1
et donc a une limite relle quand n tend vers +.
A

Soit n N . Pour t ]0, +[,


2 t

f(t) = t e

n1
X

1
= t2 et
1 et

t2 e(k+1)t +

k=0

n1
X
k=0


t k

(et )
+
1 et

n
t2 et nt X 2 kt
e
=
t e
+ fn (t) (),
1 et
k=1

t2 et nt
e
pour t > 0. En posant de plus fn (0) = 0, dune part, fn est continue sur [0, +[ et dautre part,
1 et
lgalit () reste vraie quand t = 0. En intgrant, on obtient
o fn (t) =

x [0, +[, n N , F(x) =


http ://www.maths-france.fr

n Zx
X

k=1 0

15

2 kt

t e

dt +

Zx

fn (t) dt ().

c Jean-Louis Rouget, 2014. Tous droits rservs.


Soient alors k N et x [0, +[. Deux intgrations par parties fournissent :




x
x

Z
Z
2
1
1
2 x kt
1 x kt
1
te
dt = x2 ekx +
tekt +
e
dt
t2 ekt dt = t2 ekt +
k
k 0
k
k
k
k 0
0
0
0
1 2 kx
2 kx
2
2 kx
= x e
3e
+ 3.
2 xe
k
k
k
k
Zx
2
Puisque k > 0, quand x tend vers +, on obtient lim
t2 ekt dt = 3 . On fait alors tendre x vers + dans () et
x+ 0
k
on obtient
Zx

Zx
n
X
1
n N , 2
= lim
fn (t) dt ( ).
k3 x+ 0
k=1


Zx
t2 et
est continue sur ]0, +[, se prolonge par
Vrifions enfin que lim
lim
fn (t) dt = 0. La fonction t 7
n+ x+ 0
1 et
continuit en 0 et a une limite relle en +. On en dduit que cette fonction est borne sur ]0, +[. Soit M un majorant
de cette fonction sur ]0, +[. Pour x [0, +[ et n N , on a alors
Zx
Zx
M
06
fn (t) dt 6 M ent dt =
(1 enx ).
n
0
0

n N tant fix, on passe la limite quand n tend vers + et on obtient


Zx
M
fn (t) dt 6
0 6 lim
,
x+ 0
n
puis on passe la limite quand n tend vers + et on obtient


Zx
lim
lim
fn (t) dt = 0.
n+

x+ 0

Par passage la limite quand x tend vers + puis quand n tend vers + dans ( ), on obtient enfin
!

Z x
n
X
1
t2
.
dt = 2 lim
lim
t
n+
x+
k3
0 e 1
k=1

http ://www.maths-france.fr

16

c Jean-Louis Rouget, 2014. Tous droits rservs.


Planche no 32. Sries numriques


* trs facile ** facile *** difficult moyenne **** difficile
I : Incontournable T : pour travailler et mmoriser le cours
Exercice no 1 (***I) Un calcul de

+
X
1
.
n2

n=1

1) Dterminer deux rels a et b tels que, pour tout entier naturel non nul n,


(2n + 1)t
sin
n
X
2
 
2) Montrer que t ]0, ],
cos(kt) =
t
k=1
2 sin
2
3) En utilisant le lemme de Lebesgue, dterminer

1
=
n2

Z
0


at2 + bt cos(nt) dt.

1
.
2

+
X
1
.
n2

n=1

Exercice no 2 (**I) Un calcul de

+
X

n=1

1) En remarquant que k > 1,

1
=
k

+
X
(1)n
dterminer la valeur de
.
n

+
X
(1)n1
(1)n
et de
.
n
2n + 1
n=0

Z1

tk1 dt, montrer que la srie de terme gnral

(1)n1
, n > 1, converge et
n

n=1

2) En adaptant lide prcdente, montrer que la srie de terme gnral


de

+
X
(1)n
.
2n + 1

(1)n
, n > 0, converge et dterminer la valeur
2n + 1

n=0

Exercice no 3 (***I) Sries de Bertrand.


1
.
ln n
ln n
1
1) Deux exemples : montrer que la srie de terme gnral 2 , n > 1, converge et que la srie de terme gnral
,
n
n ln2 n
n > 2, diverge.
Soient et deux rels. Pour n > 2, on pose un =

2) Montrer que si < 0, la srie de terme gnral un diverge grossirement.


3) Montrer que si 0 6 < 1, la srie de terme gnral un diverge.
4) Montrer que si > 1, la srie de terme gnral un converge.
5) Dans cette question, = 1.
a) Montrer que si 6 0, la srie de terme gnral un diverge.
b) En comparant un une intgrale, montrer que la srie de terme gnral un converge si et seulement si > 1.
Exercice no 4
Nature de la srie de terme gnral
1) (*) ln

n2 + n + 1
n2 +r
n1

1
5) (**) Arccos 3 1 2
n
Z /2
cos2 x
dx
9) (*)
n2 + cos2 x
0

http ://www.maths-france.fr

2) (*)
n + (1)n n
n2
6) (*)
(n 1)!
10) (**) n

1
2 sin(
4+n)

ln n
n+3
3) (**)
2n+ 1

n
1
1

7) cos
n 
e
n
1
11) (**) e 1 +
n


1
ln(n)
ln(ch
n)


2
n2 + 1
8) (**) ln
Arctan

4) (**)

c Jean-Louis Rouget, 2014. Tous droits rservs.


Exercice no 5
Nature de la srie de terme gnral
1) (***)

n4

2n2

3) (**) un o n N , un =
5) (***)

+
X
1
1
, n > 2.
2) (***) S(n), R, o S(n) =
n
pn
p=2
a 

a 

1
1
Arctan
1
, a R.
4) (**) Arctan
1+
n
n

p
3 P(n) o P est un polynme.
1 un1
e
.
n

n
1 X 3/2
k .
n
k=1

Exercice n 6
o

Calculer les sommes des sries suivantes aprs avoir vrifi leur convergence.
+
X
n+1
3n
n=0


+
X
1
1
2

4) (**)
+

n
n1
n+1
n=2
a
+ th
X
2n , a R .
7)
2n

1) (**)

+
X
2n 1
n3 4n
n=3

+
X 
(1)n
5) (***)
ln 1 +
n

2) (**)

n=2

+
X

n2
(n 1)!
n=1
+

i h
X
a
6) (***)
ln cos n a 0,
2
2
3) (**)

n=0

n=0

Exercice no 7 (*** I)
Soit (un )nN une suite dcroissante
de nombres rels strictement positifs telle que la srie de terme gnral un converge.
 
1
. Trouver un exemple de suite (un )nN de rels strictement positifs telle que la srie de
Montrer que un = o
n+
n
terme gnral un converge mais telle que la suite de terme gnral nun ne tende pas vers 0.
Exercice no 8 (***)
Trouver un dveloppement limit lordre 4 quand n tend vers linfini de

n
X
1
e
k!
k=0

(n + 1)!.

Exercice no 9 (***)
 
n 
Nature de la srie de terme gnral un = sin 2 + 3
.
Exercice no 10 (**)

Soit (un )nN une suite positive telle que la srie de terme gnral un converge. Etudier la nature de la srie de terme

un
.
gnral
n
Exercice no 11 (***)
un
Soit (un )nN une suite de rels positifs. Trouver la nature de la srie de terme gnral vn =
, n > 1,
(1 + u1 ) . . . (1 + un )
connaissant la nature de la srie de terme gnral un puis en calculer la somme en cas de convergence.
Exercice no 12
Convergence et somme ventuelle de la srie de terme gnral
1) (**) un =

2n3 3n2 + 1
(n + 3)!

2) (***) un =

n!
, n > 1, a R+ donn.
(a + 1)(a + 2) . . . (a + n)

Exercice no 13 (*)
Nature de la srie de terme gnral un =

n
X

k=1

1
, p ]0, +[.
(n + k)p

Exercice n 14 (** I)
o

Calculer

+
+
X
X
1
(1)n1
et
.
2
n
(2n + 1)2

n=1

n=0

http ://www.maths-france.fr

c Jean-Louis Rouget, 2014. Tous droits rservs.


Exercice no 15 (*** I)
Pour n N , on pose Rn =

+
X

k=n+1

1
.
k2

1) Justifier lexistence de Rn . Quelle est la limite de Rn ?


1
1
par des termes gnraux de sommes tlescopiques, montrer que Rn
.
2
n+
k
n

+ 
X
1
1
1
, dterminer le dveloppement limit lordre 2 de Rn
=

3) En commenant par remarquer que


n
k1 k
k=n+1
quand n tend vers linfini.

2) En encadrant

Exercice no 16 (***)

X
X

Calculer
pN

nN , n6=p

Exercice no 17 (***)

X
1
et

n2 p2

nN

pN , p6=n

1
. Que constatez-vous ?
n2 p2
n

Convergence et somme de la srie de terme gnral un =

X (1)k

, n > 0.
4
2k + 1
k=0

Exercice n 18 (****)
o

Pour n > 1, on note pn le n-me nombre premier. On veut montrer que

+
X
1
= +.
pn

n=1

1
est de mme nature que la srie de terme gnral ln
1) Montrer que la srie de terme gnral
pn

1 !

1
.
1
pn

2) En utilisant la dcomposition dun entier naturel suprieur ou gal 2, montrer que


1 !

+
N
X
X
1
1

.
6
ln
1
N N ,
k
pn
k=1

n=1

Conclure.

http ://www.maths-france.fr

c Jean-Louis Rouget, 2014. Tous droits rservs.


Planche no 32. Sries numriques : corrig


Exercice no 1 :
1) Soient a et b deux rels. Pour tout entier naturel n non nul, deux intgrations par parties fournit
Z
0

Z
sin(nt)
1
(2at + b)
=
(2at + b)( sin(nt)) dt
at + bt cos(nt) dt = at + bt
n
n
n 0
0
0

 Z



1
cos(nt)
1
cos(nt)
=
(2at + b)
2a cos(nt) dt = 2 (2at + b)

n
n
n
n
0
0
0
1
n
= 2 ((2a + b)(1) b) .
n
2

 sin(nt)

puis

n N ,

Z
0


1
1
1
at2 + bt cos(nt) dt = 2 n > 1, 2 ((2a + b)(1)n b) 2
n
n
n
n > 1, (2a + b)(1)n b = 1
1
b = 1 et a =
.
2
n > 1,

2) Pour n > 1,
fn (t) =

n
X

k=1

1
=
n2

Z 
0


t2
t cos(nt) dt.
2


X
Z  2
n Z  2
n
n
X
X
t
t
1
=

t
cos(kt)
dt
=

t
cos(kt) dt. Pour t [0, ], posons
k2
2
2
0
0
k=1

k=1

k=1

cos(kt). Pour t [0, ],

 
 

 

 
n
n 
X
X
t
1
1
t
fn (t) =
2 sin
cos(kt) =
sin
k+
t sin
k
t
2
2
2
2
k=1
k=1

 
  
1
1
= sin
n+
t sin
t (somme tlescopique)
2
2

 

t
(2n + 1)t
sin
.
= sin
2
2


(2n + 1)t
 
sin
t
1
2
 
Par suite, si t ]0, ] de sorte que 2 sin
6= 0, fn (t) =
et dautre part, fn (0) = n.
t
2
2
2 sin
2
2 sin

3) Pour t ]0, ],


t2
t fn (t) =
2

t2


 2

t
2   sin (2n + 1)t 1 t t . Pour t ]0, ], on pose alors
t
2
2 2
2 sin
2

t2
t
g(t) = 2   . La fonction g est continue sur ]0, ] et se prolonge par continuit en 0 en posant g(0) = 1. En notant
t
2 sin
2
encore g le prolongement obtenu, pour tout entier naturel n non nul,



Z
Z 
n
X
1
1 t2
(2n + 1)t
.
=

g(t)
sin

t
dt
+
k2
2 0 2
2
0

k=1

Puisque
sur le segment [0, ], le lemme de Lebesgue permet daffirmer que

 g est continue
Z la fonction
(2n + 1)t
lim
= 0 et donc
g(t) sin
n+ 0
2
http ://www.maths-france.fr

c Jean-Louis Rouget, 2014. Tous droits rservs.







Z 
+
X
2
1 t2
1 3
1 t3
t2
2
1
=
=

t
dt
=

.
2
n
2 0 2
2 6
2 0
2 6
2
6

n=1

+
X
1
2
=
.
2
n
6

n=1

Exercice no 2 :
1) Soit n N . Soit k J1, nK.

Z1

tk1 dt =

tk
k

1

1
. Donc,
k

!
Z1 X
Z1
n
n
n
X
X
(1)k1
k1
k1
k1
dt
(t)
=
(1)
t
dt =
k
0
0
k=1
k=1
k=1
Z1
1 (t)n
dt (car t [0, 1], t 6= 1)
=
0 1 (t)
Z1 n
Z1 n
Z1
t
t
1
n
n
dt (1)
dt = ln 2 (1)
dt.
=
1
+
t
1
+
t
1
+t
0
0
0

Z
Z1 n
Z1


1
t
1
tn


n
De plus, (1)
dt =
dt 6 tn dt =
.


n+1
0 1+t
0
0 1+t
Z1 n
t
1
= 0, on en dduit que lim (1)n
dt = 0. Mais alors, la suite des sommes partielles
Puisque lim
n+
n+ n + 1
1
+t
0
!
n
X
(1)n1
(1)k1
, n > 1, converge et
converge ou encore la srie de terme gnral
k
n
k=1

n>1

+
X
(1)n1
= ln 2.
n

n=1

2) Soit n N . Soit k J1, nK.

Z1
0

t2k dt =

1
. Donc,
2k + 1

!
Z1
Z1 X
n
n
n
X
X
(1)k
k
2k
k
=
(1)
t dt =
(t) dt
2k + 1
0
0
k=0
k=0
k=0
Z1
1 (t2 )n+1
dt (car t [0, 1], t2 6= 1)
=
1 (t2 )
0
Z 1 2n
Z 1 2n
Z1

t
t
1
n
n
dt
+
(1)
dt
=
dt.
+
(1)
=
2
2
1
+
t
1
+
t
4
1
+ t2
0
0
0
Z

Z1
Z 1 2n


1
1
t2n
t


n
dt =
dt 6 t2n dt =
.
De plus, (1)
2
2


1
+
t
1
+
t
2n
+1
0
0
0
Z 1 2n
1
t
dt = 0. Mais alors, la suite des sommes partielles
Puisque lim
= 0, on en dduit que lim (1)n
2
n+ 2n + 1
n+
0 1+t
!
n
X
(1)k
(1)n
, n > 0, converge et
converge ou encore la srie de terme gnral
2k + 1
2n + 1
k=0

n>1

+
X

(1)n
= .
2n + 1
4

n=0

http ://www.maths-france.fr

c Jean-Louis Rouget, 2014. Tous droits rservs.


Exercice no 3 : La suite (un ) est strictement positive.


ln n
ln n
ln n
= . Daprs un thorme de croissances compares, lim = 0 et donc
n+
n2
n 
n

1
ln
n
1
ln
n
3
= o(1) ou encore 2
= o
n3/2 2
. Puisque > 1, la srie de terme gnral 3 converge et donc la
3
n n+
n n+
2
n2
n2
ln n
srie de terme gnral 2 converge.
n

n
n
1

Pour tout n N , n
= 2 . Daprs un thorme de croissances compares, lim
= 0 et donc
2
2
n+
n ln n
ln n
ln n
1
1
1
est prpondrant devant
quand n tend vers +. Puisque la srie de terme gnral
diverge, la srie de

2
n
n
n ln n
1
diverge.
terme gnral
n ln2 n
1) Pour tout n N , n3/2

n
avec > 0. Daprs un thorme de croissance compares, un tend vers +
ln n
quand n tend vers + et donc la srie de terme gnral un , n > 2, diverge grossirement.
2) Si < 0, on peut crire un =

1
n1
=
avec 1 > 0. Daprs un thorme de croissances compares,
n ln n
ln n
1
1
1
quand n tend vers + et donc
= +. Par suite,
est prpondrant devant
pour tout rel , lim

n+ n ln n
n
n ln n
la srie de terme gnral un , n > 2, diverge.

3) On suppose que 0 6 < 1. n

1
1
1
> 0. Daprs un thorme de croissances
=
avec
2
n ln n
n(1)/2 ln n


1
1+1
+1
1
1
compares, pour tout rel , lim
>
= 1,
. Puisque
=
0.
Donc,
=
o
+1

(1)/2

n+ n
2
2
ln n
n ln n n+
n 2
1
la srie de terme gnral +1 converge et donc la srie de terme gnral un converge.
n 2
1
.
5) Dans cette question, pour tout n > 2, un =
n ln n
4) On suppose que > 1. n(+1)/2

ln n
1
1
a) Si < 0, un =
avec > 0. Dans ce cas, un est prpondrant devant
en +. Si = 0, un = . Dans
n
n
n
tous les cas, si 6 0, la srie de terme gnral un diverge.
b) Soit > 1. Vrifions que la srie de terme gnral un converge. Puisque la suite (un )nN est positive,
on sait que la
!
n
X
srie de terme gnral un , n > 2, converge si et seulement si la suite des sommes partielles
uk
est majore.
k=2

n>2

La fonction t 7 t ln t est strictement croissante sur ]1, +[ en tant que produit de fonctions strictement positives et
1
strictement croissantes sur ]1, +[. Donc, la fonction t 7
est strictement dcroissante sur ]1, +[ en tant quinverse
t ln t
de fonction strictement positive et strictement croissante sur ]1, +[.
On en dduit que pour n > 3,
n
X

X
1
1
1
=
+

k ln k
2 ln 2 k=3 k ln k
k=2
Zn
n Zk
X
1
1
1
1
6
+
dt
=
+
dt

2 ln 2 k=3 k1 t ln t
2 ln 2
t
ln
t
2
n

1
1
1
1
1
=
+

=
+

2 ln 2
( 1) ln1 t 2
2 ln 2 ( 1) ln1 2 ( 1) ln1 n
1
1
+
(car 1 > 0).
6

2 ln 2 ( 1) ln1 2
!
n
X
Ainsi, si > 1, la suite des sommes partielles
est majore et donc la srie de terme gnral un converge.
uk
k=2

http ://www.maths-france.fr

n>2

c Jean-Louis Rouget, 2014. Tous droits rservs.


Vrifions que la srie de terme gnral


n
X

k=2

1
1
diverge. Par dcroissance de la fonction t 7
sur ]1, +[, pour n > 2,
n ln n
t ln t

Z n+1
n Z k+1
X
1
1
1
>
dt =
dt
k ln k
t
ln
t
t
ln
t
k
2
k=2

= [ln | ln t||2n+1 = ln(ln(n + 1)) ln(ln 2).


Puisque

lim (ln(ln(n + 1)) ln(ln 2)) = +, on en dduit que

n+

lim

n+

n
X

k=2

1
= + ou encore que la srie de terme
k ln k

1
, n > 2, diverge.
gnral
n ln n
1/n ln n
1/n ln n
1
1
1
en
+
car
=
ln
n
et
donc
tend vers +
est
prpondrant
devant
n ln n
1/n ln n
1/n ln n
n ln n
1
quand n tend vers + car 1 > 0. On en dduit que la srie de terme gnral
, n > 2, diverge.
n ln n
1
, n > 2, converge si et seulement si > 1.
En rsum, la srie de terme gnral
n ln n
Exercice no 4 :
 2

n +n+1
1) Pour n > 1, on pose un = ln
. Pour tout entier n > 1, un existe. De plus
n2 + n 1
Enfin, si < 1,





1
1
1
1
un = ln 1 + + 2 ln 1 + 2
n n
n n
 
  
 

1
1
1
1
1

= O
.
+O
+O
=
n+
n+
n
n2
n
n2
n2
1
Comme la srie de terme gnral 2 , n > 1, converge (srie de Riemann dexposant > 1), la srie de terme gnral un
n
converge.
2) Pour n > 2, on pose un =

1
. n > 2, un existe et de plus un
n + (1)n n

n+

1
. Comme la srie de terme gnral
n

1
, n > 2, diverge et est positive, la srie de terme gnral un diverge.
n
ln n

n+3
. Pour n > 1, un existe et un > 0.
3) Pour n > 1, on pose un =
2n + 1



  




n+3
3
1
1
ln(un ) = ln(n) ln
= ln(n) ln
+ ln 1 +
ln 1 +
2n + 1
2
n
2n

 


1
ln n
= ln(n) ln 2 + O
= ln 2 ln(n) + O
n+
n+
n
n
= ln 2 ln(n) + o(1).
n+

Donc un = eln(un )

n+

e ln 2 ln n =

1
nln 2

. Comme la srie de terme gnral

dexposant 6 1) et est termes positifs, la srie de terme gnral un diverge.

1
nln 2

, n > 1, diverge (srie de Riemann

 n
1
e
= n ln 2 n puis
4) Pour n > 2, on pose un =
. un existe pour n > 2. ln(ch n) ln
n+
n+
ln(n) ln(ch n)
2
1
un
> 0.
n+ n ln(n)
1
Vrifions alors que la srie de terme gnral
, n > 2, diverge. La fonction x x ln x est continue, croissante et
n ln n
strictement positive sur ]1, +[ (produit de deux fonctions positives et croissantes sur ]1, +[). Par suite, la fonction
1
est continue et dcroissante sur ]1, +[ et pour tout entier k suprieur ou gal 2,
x
x ln x
Z k+1
1
1
>
dx
k ln k
x
ln
x
k
http ://www.maths-france.fr

c Jean-Louis Rouget, 2014. Tous droits rservs.


Par suite, pour n > 2,


n
X

k=2

Z n+1
n Z k+1
X
1
1
1
>
dx =
dx = ln(ln(n + 1)) ln(ln(2) +.
n+
k ln k
x ln x
x ln x
2
k
k=2

On en dduit que la srie de terme gnral un diverge.


r
r
1
1
3
5) Pour n > 1, on pose un = Arccos 1 2 . un existe pour n > 1 car pour n > 1, 3 1 2 [1, 1]. De plus
n
n
un 0. On en dduit que
n+

un

sin(un ) = sin Arccos

n+

n+

r
3

1
1 2
n


2/3
1
1 1 2
n

n+

11+

2
+o
3n2

1
n2

2
1
>0
3 n

qui est le terme gnral dune srie de Riemann divergente. Donc la srie de terme gnral un diverge.
6) Pour n > 1, on pose un =

n2
. Pour n > 1
(n 1)!
n2 un = n2

n5
n3
=
.
n!
n!

Daprs un thorme de croissances compares, n un tend vers 0 quand n tend vers + ou encore un

n+


1
.
n2

On en dduit que la srie de terme gnral un converge.


n

i h
1
1
1
. un est dfini pour n > 1 car pour n > 1, 0,
7) Pour n > 1, on pose un = cos
et donc
2
n
e
n
1
cos > 0. Ensuite
n

 
 


1
1
1
1
1
1
1
1
ln cos
=

+
o

+
o
+
+
= ln 1
n+
2n 24n2
n2
2n 24n2
8n2
n2
n n+
 
1
1
1
.
+
o

=
n+
2n 12n2
n2


 
1
1
1
1
Puis n ln cos
=
et donc
+o
n+
2
12n
n
n

un = en ln(cos(1/
La srie de terme gnral

n))

n+


1
1
1
1 
< 0.
e 12n +o( n ) 1

n+
e
12n e

1
est divergente et donc la srie de terme gnral un diverge.
12n e

8)

ln

2
Arctan

n2 + 1
n






2
n
= ln 1 Arctan

n2 + 1


2
2 n
n
Arctan

n+

n2 + 1 n+ n2 + 1

n+

2
< 0.
n

Donc, la srie de terme gnral un diverge.


Z /2
cos2 x
9) Pour n > 1, on pose un =
dx.
2
n + cos2 x
0
h i
cos2 x
et positive et donc, un existe et est positif. De plus,
dx
est
continue
sur
0,
Pour n > 1, la fonction x 7 2
n + cos2 x
2
pour n > 1,
0 6 un 6

Z /2
0

http ://www.maths-france.fr

1
.
dx =
n2 + 0
2n2
5

c Jean-Louis Rouget, 2014. Tous droits rservs.



La srie de terme gnral
converge et donc la srie de terme gnral un converge.
2n2

 
 
 


1
1
1
1

= sin
cos
= 1 + O
puis
+
10) 2 sin
n+
4 n
n
n
n





ln n
1

2 sin
ln n = ln(n) + O
= ln(n) + o(1).
+
n+
n+
4 n
n
Par suite,
0 < un = e

1
2 sin(
4 + n ) ln n

n+

e ln n =

1
.
n

1
diverge et la srie de terme gnral un diverge.
La srie de terme gnral
n


 
1
1
1
11) n ln 1 +
= 1
et donc
+o
n n+
2n
n

 
1
1
1
e
1
+o( n
1 2n
)
= e 11+

un = e e
+o
> 0.
n+
n+
n+
2n
n
2n
La srie de terme gnral

e
diverge et la srie de terme gnral un diverge.
2n

Exercice no 5
1) Si P nest pas un polynme unitaire de degr 3, un ne tend pas vers 0 et la srie de terme gnral un diverge
grossirement.
Soit P un polynme unitaire de degr 3. Posons P = X3 + aX2 + bX + c.

1/4 
1/3 !
c
2
b
a
un = n
1+ 2
1+ + 2 + 3
n
n n
n
  
 

a
a2
b
1
1
1
1+
+O
2 +O
+
= n
1+
n+
2n2
n3
3n 3n2
9n
n3


 
1 b a2 1
1
a
.
+
+O
= +
n+
3
2 3
9 n
n2
Si a 6= 0, un ne tend pas vers 0 et 
la srie 
de terme gnral un diverge grossirement.
1 b
1 b 1
Si a = 0 et 6= 0, un

. un est donc de signe constant pour n grand et est quivalent au terme


n+
2
3
2 3 n
gnral dune srie divergente. Donc lasriede terme gnral un diverge.
1
1 b
. Dans ce cas, la srie de terme gnral un converge (absolument).
Si a = 0 et = 0, un = O
n+
2 3
n2
3
En rsum, la srie de terme gnral un converge si et seulement si a = 0 et b = ou encore la srie de terme gnral un
2
3
3
converge si et seulement si P est de la forme X + X + c, c R.
2
1
2) Pour n > 2, posons un = S(n). Pour n > 2,
n
0 < S(n + 1) =

+
+
X
1
1X 1
1
1
= S(n)
n 6
n
p p
2
p
2

p=2

et donc n > 2, S(n) 6

p=2

S(2)
. Par suite,
2n2
0 6 un 6

http ://www.maths-france.fr

1 S(2)
n 2n2

n+


1
.
n2

c Jean-Louis Rouget, 2014. Tous droits rservs.


Pour tout rel , la srie de terme gnral un converge.


1
3) u0 R, n N , un > 0. Par suite, n > 2, 0 < un < .
n
1
> 0. La srie de terme gnral un diverge.
On en dduit que lim un = 0 et par suite un
n+
n+ n

4) lim un = = 0. Donc
n+
4
4
tan(un )

a 
a
1
1
1+
1
n
n

a
=
1
1+ 1 2
n

un

n+

n+

 
2a
1
+O
n
n2
 
1
2+O
n2

a
+O
n

Zk

x3/2 dx 6 k3/2 6

n+

1
n2

Par suite, la srie de terme gnral un converge si et seulement si a = 0.


5) La fonction x 7 x3/2 est continue et croissante sur R+ . Donc pour k > 1,

pour n N :

Zn

3/2

dx =

n Zk
X

3/2

dx 6

k=1 k1

n
X

3/2

n Z k+1
X

3/2

k1

x3/2 dx puis

dx =

k=1 k

k=1

Z k+1

Z n+1

x3/2 dx

ce qui fournit
n
n
X
2
2 5/2 X 3/2
n
6
k
6 ((n + 1)5/2 1) et donc
k3/2
5
5
k=1

Donc un

n+

k=1

n+

2n5/2
.
5

1
7
2

.
5 > 0. La srie de terme gnral un converge si et seulement si >

5 n 2
2

Exercice no 6
 
n+1
n+1
1
. Par suite, la srie de terme gnral
= o
converge.
1)
n
2
n+
3
n
3n
1er calcul. Soit S =

+
X
n+1
. Alors
3n

n=0

+
+
+
+
X
X
X
1
n
n+1 X 1
n+1
=
=

S=
3
3n+1
3n
3n
3n
n=1

n=0

1
= (S 1)
3

On en dduit que S =

n=1

n=1

3
=S .
1
2
1
3
1

9
.
4
+
X
n+1
9
= .
3n
4

n=0

2me calcul. Pour x R et n N, on pose fn (x) =


Soit n N . fn est drivable sur R et pour x R,
fn (x) =

n
X

xk .

k=0

n
X

kxk1 =

k=1

n1
X

(k + 1)xk .

k=0

Par suite, pour n N et x R \ {1}


http ://www.maths-france.fr

c Jean-Louis Rouget, 2014. Tous droits rservs.


n1
X

(k + 1)x =

fn (x)

k=0

n1
X k+1
1
Pour x = , on obtient
=
3
3k
k=0

2) Pour k > 3,

xn 1
x1

(x) =

nxn1 (x 1) (xn 1)
(n 1)xn nxn1 + 1
=
.
(x 1)2
(x 1)2

n
n1
n1 + 1
9
3n
3
et quand n tend vers linfini, on obtient de nouveau S = .
2

4
1
1
3

3
1
5
2k 1
=
+

. Puis
k3 4k
8(k 2) 4k 8(k + 2)

n
n
n
n
n2
n
n+2
X
2k 1
3X 1
1X1 5X 1
3X1 1X1 5X1
=
+

=
+

k3 4k
8
k2 4
k 8
k+2
8
k 4
k 8
k
k=3
k=3
k=3
k=3
k=1
k=3
k=5
!
!
!
n
n
n
1
1 X1
5
1 1 1 X1
3 X1
+
1 +

1 +
+ o(1)
=
n+ 8
k
4
2
k
8
2 3 4
k
k=1
k=1
k=1


3 5
1 1 1
3 125
= +
1+ + +
+ o(1) = +
+ o(1)
n+
n+
8 8
2 3 4
8
96
89
=
+ o(1).
n+ 96

La srie propose est donc convergente de somme

89
.
96
+
X
89
2n 1
=
.
n3 4n
96

n=3

n2
. Pour n > 3
(n 1)!
1
1
1
(n 1)(n 2) + 3n 3 + 1
=
+3
+
.
un =
(n 1)!
(n 3)!
(n 2)! (n 1)!
1
1
1
Les sries de termes gnraux respectifs
et
converge et donc la srie de terme gnral un
(n 3)! (n 2)!
(n 1)!
converge. De plus,
3) Pour n > 1, on pose un =

+
X

n=1

+
+
+
+
X
X
X
X
n2
n2
1
1
1
=1+4+
=5+
+3
+
(n 1)!
(n 1)!
(n 3)!
(n 2)!
(n 1)!
n=3

n=3

n=3

n=3

+
+
+
X
X
X
1
1
1
+3
+
= 5 + e + 3(e 1) + e 2 = 5e.
=5+
n!
n!
n!
n=1

n=0

+
X

n=1

n=2

n2
= 5e.
(n 1)!

4)
n 
X

k=2

1
1
2
+

k1
k+1
k

+ 
X

n=2

http ://www.maths-france.fr

n 
X

 

1
1
1
1

k1
k+1
k
k
k=2
 


1
1
1

= 1
(somme tlescopique)
n
n+1
2
1
= 1 + o(1)
n+
2
=

1
2
1
+

n
n1
n+1

1
=1 .
2

c Jean-Louis Rouget, 2014. Tous droits rservs.


5) Pour n N , posons Sn =

n
X

k=2

S2p+1 =



(1)k
ln 1 +
. Soit p N .
k

2p+1
X
k=2

p
X


 X



p  
(1)k
1
1
ln 1 +
=
+ ln 1
ln 1 +
k
2k
2k + 1
k=1

(ln(2k + 1) ln(2k) + ln(2k) ln(2k + 1)) = 0.

k=1





1
(1)2p+1
= ln 1
. Mais alors les suites (S2p )pN et (S2p+1 )pN
Dautre part, S2p = S2p+1 ln 1 +
2p + 1
2p + 1
convergentet ont mmes
 limites, savoir 0. On en dduit que la suite (Sn )nN converge ou encore la srie de terme
(1)n
gnral ln 1 +
, n > 2, converge et
n
+
X

n=2



(1)n
ln 1 +
= 0.
n

i h
a
i h
a
6) Si a 0,
alors, pour tout entier naturel n, n 0,
et donc cos n > 0.
2
2


 2


 2  a 
1
1
Ensuite, ln cos n
= ln 1 + O
= O
et la srie converge. Ensuite,
n+
n+
2
22n
22n
 a 


a 
!
n
n
n
n





sin
sin
2

X
Y
Y
k
k1
a
a
1 Y
 a2  = ln n+1
2 a 
= ln
ln cos k
= ln
cos k
2
2
2
k=0
k=0 sin
k=0 2 sin
k=0
2k
2k

sin(2a)
 a  (produit tlescopique)
sin n
2



sin(2a)
sin(2a)

.
= ln
ln
a
n+
2a
2n+1 n
2


+

 a 
i h X
sin(2a)
ln cos n
a 0, ,
.
= ln
2
2
2a

= ln

2n+1

n=0

2 th x
. Soit x R.
1 + th2 x
2  1 2x
2

1 x
=
e + ex + ex ex
e + e2x = ch(2x)
ch2 x + sh2 x =
4
2

7) Vrifions que pour tout rel x on a th(2x) =

et

 1 2x


1 x
e ex ex + ex =
e e2x = sh(2x)
2
2
puis, en multipliant numrateur et dnominateur par le rel non nul ch2 x,
2 sh x ch x =

2 th x
2 sh x ch x
sh(2x)
= th(2x).
= 2
=
ch(2x)
1 + th2 x
ch x + sh2 x
2
1 + th2 x
2
1
=
puis th x =

. Mais alors, pour a R et n N


th(2x)
th x
th(2x) th x

n
n
n
a X
X
1 X
1
1
2
1
1

th k =
=
a
a k
a
2k
2
2k th a
k1 th
th
2
2
th
k=0
k=0
k=0
2k1
2k
2k1
2k
1
2
(somme tlescopique)

=
th(2a) 2n th a
n
2
2
1

,
n+ th(2a)
a

Par suite, pour x R ,

http ://www.maths-france.fr

c Jean-Louis Rouget, 2014. Tous droits rservs.


a R ,

+
a
X
1
2
1
th n =
.
n
2
2
th(2a) a

n=0

Exercice no 7 :
Il faut vrifier que nun

n+

n
X

0. Pour n N, posons Sn =

0 < (2n)u2n = 2(u2n + . . . + u2n ) 6 2


|
{z
}
n

k=0

uk . Pour n N, on a

2n
X

uk (car la suite u est dcroissante)

k=n+1

= 2(S2n Sn ).

Puisque la srie de terme gnral un converge, lim 2(S2n Sn ) = 0 et donc


n+

Ensuite, 0 < (2n + 1)u2n+1 6 (2n + 1)u2n = (2n)u2n + u2n

n+

lim (2n)u2n = 0.

n+

0. Donc les suites des termes de rangs pairs et impairs

extraites de la suite (nun )nN convergent et ont mme limite savoir 0. On en dduit que lim nun = 0 ou encore que
n+
 
1
.
un = o
n+
n

0 si n = 0

1
Contre exemple avec u non monotone. Pour n N, on pose un =
si n est un carr parfait non nul .

0 sinon
+
+
X
X
1
< +. Pourtant, p2 up2 = 1 1 et donc la suite (nun ) admet une suite
La suite u est positive et
un =
p+
p2
n=0

p=1

extraite convergeant vers 1. On a donc pas

lim nun = 0.

n+

Exercice no 8 :
!
n
X
1
Pour n N, posons un = (n + 1)! e
. Soit n N .
k!
k=0

un =

+
X

k=n+1

=1+

(n + 1)!
k!

+
X
1
1
1
1
1
+
+
+
+
.
n + 2 (n + 2)(n + 3) (n + 2)(n + 3)(n + 4) (n + 2)(n + 3)(n + 4)(n + 5)
(n + 2)(n + 3) . . . k
k=n+6

On a 0 <

+
X

k=n+6

1
6
(n + 2)(n + 3) . . . k

On en dduit que

+
X

k=n+6

un

+
X

1
1
=
k(n+1)
(n + 2)5
(n
+
2)
k=n+6

1
(n + 2)(n + 3) . . . k

n+

1
1
1
n+2

1
1
6 5.
(n + 2)4 (n + 1)
n


1
. Donc
n4

 
1
1
1
1
1
+
+
+
+o
n+
n + 2 (n + 2)(n + 3) (n + 2)(n + 3)(n + 4) (n + 2)(n + 3)(n + 4)(n + 5)
n4

1
1 
1
1 
1 
1


 
2
3
3
4
2
2
1
1
1
1
1
1+
1+
1+
1+
+ 2 1+
+ 3 1+
+ 4 +o
= 1+
n+
n
n
n
n
n
n
n
n
n
n
n4









1
2
3
4
1
2
3
1
2
8
4
4
9
= 1+
1 + 2 3 + 2 1 + 2
1
1
1 + 2 + 3 1
n+
n
n n
n
n
n n
n n
n
n
n
n
 
1
1
+ 4 +o
n
n4






 
2
1
1
5
1
9
1
8
4
19
1
1 + 2 3 + 2 1 + 2 + 3 1
+ 4 +o
= 1+
n+
n
n n
n
n
n n
n
n
n
n4
 
3
1
1
1
.
= 1+ 2 + 4 +o
n+
n n
n
n4
=

1+

Finalement
http ://www.maths-france.fr

10

c Jean-Louis Rouget, 2014. Tous droits rservs.


n
X
1
(n + 1)! e
k!
k=0

n+

1+

1
3
1
+
+o

n n2 n4


1
.
n4

Exercice no 9 :
 

n 
n

Pour n N, posons un = sin 2 + 3


. Daprs la formule du binme de Newton, 2 + 3
= An + Bn 3 o


n

n
= An Bn 3. Par suite, 2 + 3 +
An et Bn sont des entiers naturels. Un calcul conjugu fournit aussi 2 3

n
= 2An est un entier pair. Par suite, pour n N,
2 3


 
n 
n 
un = sin 2An 2 3
= sin 2 3
.


n

3 < 1 et donc 2 3

Mais 0 < 2

n+

0. On en dduit que |un |

n+

gomtrique convergente. Donc la srie de terme gnral un converge.


n
2 3
terme gnral dune srie

Exercice no 10 :

2





un
1
1
1
1
1

Pour n N , on a
un
un + 2 . Comme la srie terme gnral
un + 2
> 0 et donc 0 6
6
n
n
2
n
2
n

un
converge.
converge, la srie de terme gnral
n
Exercice no 11 :
un + 1 1
1
1
1
Pour n > 2, vn =
.
=

et dautre part v1 = 1
(1 + u1 ) . . . (1 + un )
(1 + u1 ) . . . (1 + un1 ) (1 + u1 ) . . . (1 + un )
1 + u1
Donc, pour n > 2

n
X

vk = 1

k=1

1
(somme tlescopique).
(1 + u1 ) . . . (1 + un )

Si la srie de terme gnral un converge alors


gnral ln(1 + un ) converge ou encore la suite

lim un = 0 et donc 0 < un


!!

n+
n
Y

ln

n>1

converge vers le rel strictement positif P = e . Dans ce cas, la suite

(1 + uk )

k=1

ln(1 + un ). Donc la srie de terme

converge vers un certain rel . Mais alors la suite

(1 + uk )

k=1

n
Y

n+

n
X

k=1

n>1

vk

n>1

1
converge vers 1 .
P

Si la srie de terme gnral


! un diverge alors la srie de terme
!gnral ln(1 + un ) diverge vers + et il en est de mme
n
n
Y
X
que la suite
(1 + uk )
. Dans ce cas, la suite
converge vers 1.
vk
k=1

k=1

n>1

n>1

Exercice n 12 :
o

1) Soit n N.
2n3 3n2 + 1 = 2(n + 3)(n + 2)(n + 1) 15n2 22n 11 = 2(n + 3)(n + 2)(n + 1) 15(n + 3)(n + 2) + 53n + 79
= 2(n + 3)(n + 2)(n + 1) 15(n + 3)(n + 2) + 53(n + 3) 80
Donc



+ 
+
X
X
5
15
53
80
2
2n3 3n2 + 1
= 2e 15(e 1) + 53(e 2) 80 e
=

(n + 3)!
n! (n + 1)! (n + 2)! (n + 3)!
2

n=0

n=0

= 40e + 109.
+
X
2n3 3n2 + 1
= 40e + 109.
(n + 3)!

n=0

http ://www.maths-france.fr

11

c Jean-Louis Rouget, 2014. Tous droits rservs.


2) Pour n N, on a un+1 =
(1 a)

n
X

uk =

k=1

n
X

n+1
un . Par suite (n + a + 1)un+1 = (n + 1)un = (n + a)un + (1 a)un puis
a+n+1

((k + a + 1)uk+1 (k + a)uk ) = (n + a + 1)un+1 (a + 1)u1 = (n + a + 1)un+1 1.

k=1

1
. Dans ce cas, la srie diverge.
n+1
n
X
1
1
1
uk =
Si a 6= 1, n N ,
((n + a + 1)un+1 1) =

(a + n + 1)un+1 .
1a
a1 a1

Si a = 1, n N , un =
k=1

1
. Donc la srie de
a1
terme gnral un converge. Il en est de mme de la suite ((a + n + 1)un+1 ). Soit = lim (a + n + 1)un+1 .

Si a > 1, la suite u est strictement positive et la suite des sommes partielles (Sn ) est majore par
n+

Si 6= 0, un+1

n+

contredisant la convergence de la srie de terme gnral un . Donc = 0 et


n+a+1
a > 1,

Si 0 < a < 1, pour tout n N , un >


Exercice no 13 :

+
X

un =

n=1

1
.
a1

1
1 2 ... n
=
. Dans ce cas, la srie diverge.
2 3 . . . (n + 1)
n+1

n
n
n
X
X
X
1
1
1
1
1
=
6
6
= p1 et la srie de terme
2p np1
(2n)p
(n + k)p
np
n
k=1
k=1
k=1
gnral un converge si et seulement si p > 2.

Pour tout entier naturel non nul n, 0 <

Exercice no 14 :
La srie de terme gnral

(1)n1
, n > 1, est absolument convergente et donc convergente.
n2





+
X
(1)n1
1
1
1
1
1
1
1
1
=
1

+
.
.
.
=
1
+
+
+
+
.
.
.

2
+
+
.
.
.
n2
22
32
42
22
32 42
22
42
n=1



2
1
1
1
2
= 1 2
1+ 2 + 2 + 2 + ... =
2
2
3
4
12
et
+
X


 

1
1
1
1
1
1
1
1
=
1
+
+
+
.
.
.
=
1
+
+
+
+
.
.
.

+
+
.
.
.
(2n + 1)2
32 52
22 32
42
22
42
n=0



1
1
1
1
2
= 1 2
1 + 2 + 2 + 2 + ... =
.
2
2
3
4
8
+
+
X
X
2
2
(1)n1
1
=
=
et
.
n2
12
(2n + 1)2
8

n=1

n=0

Exercice no 15 :
+
X

1
1
. Puisque la srie de terme gnral 2 , k > 1, converge, la suite (Rn ) est dfinie
2
k
k
k=n+1
et tend vers 0 quand n tend vers +.
1) Pour n N , posons Rn =
2) Pour tout entier k > 2,
1
1
1
6 2 6
.
k(k + 1)
k
(k 1)k
http ://www.maths-france.fr

12

c Jean-Louis Rouget, 2014. Tous droits rservs.


Soient n un entier naturel non nul puis N un entier suprieur ou gal n + 1. En sommant membre membre les ingalits
prcdentes, on obtient




N
N
N
X
X
X
1
1
1
1
1
6
,
6

k k+1
k2
k1 k
k=n+1

k=n+1

ou encore

k=n+1

N
X
1
1
1
1
1
6 . Quand N tend vers +, n tant fix, on obtient

6
2
n+1 N+1
k
n N
k=n+1

1
1
6 Rn 6 .
n+1
n
Le thorme des gendarmes montre alors que nRn tend vers 1 quand n tend vers + ou encore Rn
Rn

n+

1
+o
n

n+

1
ou enfin
n

 
1
.
n

3) Soient n un entier naturel non nul puis N un entier naturel suprieur ou gal n + 1.


N
X
1
1
1
1
=

(somme tlescopique).
k1 k
n N

k=n+1


+ 
X
1
1
1

. Par suite, pour tout entier naturel non nul n,


Quand N tend vers +, n tant fix, on obtient =
n
k1 k
k=n+1



+ 
+ 
+
X
X
X
1
1
1
1
1
1
Rn =
=

n
k2
k1 k
k2 k(k 1)
=

k=n+1

k=n+1

k=n+1

+
X

k=n+1

1
.
k2 (k 1)

Pour tout k > 2, on a


1
1
1
6 2
6
(k 1)k(k + 1)
k (k 1)
(k 2)(k 1)k
et donc
N
X

k=n+1
N
X

N
N
X
X
1
1
1
6
6
(k 1)k(k + 1)
k2 (k 1)
(k 2)(k 1)k
k=n+1

k=n+1


N
X
1
1
1
1
1
1
=
et

=
(k 1)k k(k + 1)
2 n(n + 1) N(N + 1)
(k 2)(k 1)k
k=n+1
k=n+1
k=n+1




N
1
1
1
1
1
1 X
=
et donc

2
(k 2)(k 1) (k 1)k
2 (n 1)n (N 1)N
k=n+1




N
X
1
1
1
1
1
1
1
6
.

2 n(n + 1) N(N + 1)
k2 (k 1)
2 (n 1)n (N 1)N

Ensuite,

1
1
=
(k 1)k(k + 1)
2

N
X

k=n+1

Quand N tend vers +, n tant fix, on obtient

+
X
1
1
1
6
6
puis
2
2n(n + 1)
k (k 1)
2(n 1)n
k=n+1

1
1
1
6 Rn 6
.

2(n 1)n
n
2n(n + 1)


1
2
1 + o(1) ou encore
Le thorme des gendarmes montre que 2n Rn
n n+
+
X

k=n+1

http ://www.maths-france.fr

1
k2

n+

1
1
+o

n 2n2

13


1
.
n2
c Jean-Louis Rouget, 2014. Tous droits rservs.

Exercice no 16 :
1
1
=
Soit p N . Pour n N \ {p}, 2
2
n p
2p

16n6N, n6=p

1
1
=
n2 p2
2p
1
=
2p

Maintenant,

N+p
X

k=Np+1

16n6N, n6=p

p1
X
k=1


1
1
. Donc pour N > p,

np n+p

1
1

np n+p

1
=
2p

1p6k6Np, k6=0

Np
p
X 1 N+p
X 1
X
1
1
1
+

+
+
k
k
k 2p
k
k=1

k=1

k=1

1 3
=

2p 2p

p+16k6N+p, k6=2p
N+p
X

k=Np+1

1
k

1
k

1
1
1
=
+...+
est une somme de 2p termes tendant vers 0 quand N tend vers +.
k
Np+1
N+p

Puisque 2p est constant quand N varie,

nN , n6=p

N+p
X

lim

N+

k=Np+1

1
= 0 et donc
k

X
1
1
3
3

= 2 puis
n2 p2
2p 2p
4p

Pour n N donn, on a aussi

pN

pN , p6=n

n2

1
=
p2

nN

pN , p6=n

pN , p6=n

n2

nN , n6=p

p2

+
X
2
3
1
=
=
.
n2 p2
4p2
8
p=1

3
1
= 2 et donc
2
n
4n

2
1
= .
2
p
8

Ainsi, les deux sommes existent et ne sont pas gales ou encore

XX
n

6=

XX
p

Exercice no 17 :
Soit n N.
Z1
Z1
Z1
Z1
n
n
X
1 (t2 )n+1
1
1
X (1)k
k
2k
dt

(1)
dt

=
dt
t
dt
=
2
2
4
2k + 1
1 (t2 )
0
0 1+t
0
0 1+t
k=0
k=0
n+1
Z1
t2
=
dt.
1 + t2
0

un =

Par suite, pour N N,


N
X

n=0

un =

Z1
Z1
Z 1 2N+2
Z1 X
N
2 N+1
t2
t
(t2 )n+1
2 1 (t )
N+1
dt
=
(t
)
dt
=

dt
+
(1)
dt.
2
2 )2
2 )2
1
+
t
(1
+
t
(1
+
t
(1
+ t2 )2
0
0
0
0
n=0


Z
Z1
Z 1 2N+2


1
1
1
t2N+2
t


N+1
Or (1)
dt =
dt 6 t2N+2 dt =
. Comme
tend vers 0 quand N tend vers
2 )2
2 )2


(1
+
t
(1
+
t
2N
+
3
2N
+3
0
0
0
Z 1 2N+2
t
dt. On en dduit que la srie de terme gnral un , n N, converge et de
+, il en est de mme de (1)N+1
2 2
0 (1 + t )
plus
+
X

n=0

http ://www.maths-france.fr

Z1
2t
t2
t
dt
=
dt

2 )2
(1
+
t
2
(1
+ t2 )2
0
0
1 Z 1

1
1
1
t
1
=
dt = .

2
2
2 1+t 0
1+t
4
8
0 2

un =

Z1

14

c Jean-Louis Rouget, 2014. Tous droits rservs.


+
X

n=0

X (1)k

4
2k + 1
k=0

1
.
4
8

Exercice no 18 :
1) On sait quil existe une infinit de nombres premiers. Notons (pn )nN la suite strictement croissante des nombres
1
premiers. La suite (pn )nN est une suite strictement croissante dentiers et donc lim pn = + ou encore lim
= 0.
n+
n+ pn
1 !
1 !


1
1
1
1
Par suite, 0 <
et les sries de termes gnraux
sont de mme
ln
1
et ln
1
n+
pn
pn
pn
pn
nature.

1 !
1
Il reste donc tudier la nature de la srie de terme gnral ln
1
.
pn
!
1 !

N
+
X
X
1
1
.
> ln
2) Montrons que N N ,
ln
1
pn
k
k=1
n=1
1
1
Soit n > 1. Alors
< 1 et la srie de terme gnral k , k N, est une srie gomtrique convergente de somme :
pn
pn
1

+
X
1
1
=
1

.
pkn
pn
k=0

Soit alors N un entier naturel suprieur ou gal 2 et p1 < p2 ... < pn la liste des nombres premiers infrieurs ou gaux
N.


ln(N)
1
k
et deux entiers
Tout entier entre 1 et N scrit de manire unique p1 . . . pk o i J1, nK, 0 6 i 6 i = E
ln(pi )
distincts ont des dcompositions distinctes. Donc
+
X



1 ! X
1 !
1

n
1
1
1
ln
ln
>
(car k N , 1
1
> 1)
1
pk
pk
pk
k=1
k=1
!
!
k
n
n
+
X
X
X
X
1
1
>
ln
=
ln
pi
pi
k=1
k=1
i=0 k
i=0 k

!!
k
n
Y
X
X
1
1

= ln
= ln
n
1
pi
k=1 i=0 k
061 61 ,...,...06n 6n p1 . . . , pn
!
N
X
1
> ln
.
k
k=1

!
N
X
1
= +, quand N tend vers +,
Cette ingalit est vraie pour tout entier naturel non nul N. Puisque lim ln
N+
k
k=1

1 !

1 !
+
+
X
X
1
1
on obtient
ln
1
> + et donc
ln
1
= +.
pk
pk
k=1
k=1
1

1
1
.
diverge et il en est de mme de la srie de terme gnral
La srie de terme gnral ln 1
pk
pn
(Ceci montre quil y a beaucoup de nombres premiers et en tout cas beaucoup plus de nombres premiers que de carrs
parfaits par exemple).

http ://www.maths-france.fr

15

c Jean-Louis Rouget, 2014. Tous droits rservs.


Planche no 33. Le groupe symtrique


* trs facile ** facile *** difficult moyenne **** difficile
I : Incontournable T : pour travailler et mmoriser le cours
Exercice no 1 : (**IT)
Soit llment de S12 : = (3 10 7 1 2 6 4 5 12 8 9 11).
1) Combien possde-t-elle dinversions ? Que vaut sa signature ?
2) Dcomposer en produit de transpositions. Retrouvez sa signature.
3) Dterminer les orbites de .
4) Dterminer 2015 .
Exercice no 2 : (**T)
Dmontrer que Sn est engendr par 1,2 , 1,3 ,...,1,n .
Exercice no 3 : (***T)
Dmontrer que An est engendr par les cycles de longueur 3 (pour n > 3).
Exercice no 4 : (***T)
Dmontrer que Sn est engendr par 1,2 et le cycle (2 3 ... n 1).
Exercice no 5 : (***)
Dfinition : Soient (G, ) et (G , ) deux groupes. On dit que ces groupes sont isomorphes si et seulement si il existe une
application bijective f de G sur G telle que : (x, y) G2 , f(x y) = f(x) f(y) (f est alors un isomorphisme du groupe
(G, ) sur le groupe (G , ).
Soit (G, ) un groupe. Montrer que (G, ) est isomorphe un sous-groupe de (S(G), ) et que, en particulier, tout groupe
fini dordre n est isomorphe un sous-groupe de Sn (thorme de Cayley). (Indication : montrer que pour chaque x de
G, lapplication y 7 xy est une permutation de G.)

Exercice no 6 : (***)

Soit une permutation de J1, nK et k le nombre dorbites de . Montrer que () = (1)nk .


Exercice no 7 : (***I)
tant une permutation de J1, nK donne, on dfinit la matrice note P , carre dordre n dont le terme ligne i colonne j
est i,(j) (o i,j est le symbole de Kronecker). On note G lensemble des P o dcrit Sn .
1) a) et tant deux lments de Sn , calculer P P .
b) En dduire que (G, ) est un sous-groupe de (GLn (R), ), isomorphe (Sn , ) (la dfinition de deux groupes
isomorphes a t donne dans lexercice no 5). Les matrices P sont appeles matrices de permutation .
2) (Une utilisation des P ) A tant une matrice carre donne, calculer AP et P A. Que constate-t-on ?
Exercice no 8 : (***I)
A1 , A2 ,...,Ap sont p matrices carres dordre n, deux deux distinctes et inversibles. On suppose que {A1 , ..., Ap } est
stable pour . Montrer que {A1 , ..., Ap } est un sous groupe de (GLn (R), ) (on utilisera le rsultat suivant dmontr dans
le chapitre dnombrement : si f est une application injective dun ensemble E dans un ensemble F et si E et F sont deux
ensembles finis ayant le mme nombre dlments, alors f est bijective).
Exercice no 9 : (***)
Dans E = Rn , on considre lhyperplan H dquation x1 + ... + xn = 0 dans la base canonique (ei )16i6n de E. Pour Sn
donne, on considre lendomorphisme f de E dfini par : i E, f (ei ) = e(i) .
1 X
On pose alors p =
f . Montrer que p est une projection dont on dterminera limage et la direction.
n!
Sn

http ://www.maths-france.fr

c Jean-Louis Rouget, 2014. Tous droits rservs.


Planche no 33. Le groupe symtrique : corrig


Exercice no 1 :

1 2 3 4
=
3 10 7 1

5 6
2 6

7 8
4 5

9
12

10
8

11
9

12
11


.

1) Les inversions de sont :


{1, 4}, {1, 5}, {2, 3}, {2, 4}, {2, 5}, {2, 6}, {2, 7}, {2, 8}, {2, 10}, {2, 11}, {3, 4}, {3, 5},
{3, 6}, {3, 7}, {3, 8}, {6, 7}, {6, 8}, {9, 10}, {9, 11}, {9, 12}.
Au total, il y a 2 + 8 + 5 + 2 + 3 = 20 inversions. est donc une permutation paire (de signature 1).
2) 11,12 = (3 10 7 1 2 6 4 5 11 8 9 12).
Puis, 9,11 11,12 = (3 10 7 1 2 6 4 5 9 8 11 12).
Puis, 10,8 9,11 11,12 = (3 8 7 1 2 6 4 5 9 10 11 12).
Puis, 8,5 10,8 9,11 11,12 = (3 5 7 1 2 6 4 8 9 10 11 12).
Puis, 7,4 8,5 10,8 9,11 11,12 = (3 5 4 1 2 6 7 8 9 10 11 12).
Puis, 5,2 7,4 8,5 10,8 9,11 11,12 = (3 2 4 1 5 6 7 8 9 10 11 12).
Puis, 1,4 5,2 7,4 8,5 10,8 9,11 11,12 = (3 2 1 4 5 6 7 8 9 10 11 12) = 1,3 .
Par suite,
1
1
1
1
1
1
= 1
11,12 9,11 10,8 8,5 7,4 5,2 1,4 1,3
= 11,12 9,11 10,8 8,5 7,4 5,2 1,4 1,3 .

est le produit de 8 transpositions et on retrouve le fait que est une permutation paire.
3) O(1) = {1, 3, 4, 7} = O(3) = O(4) = O(7), puis O(2) = {2, 5, 8, 10} = O(5) = O(8) = O(10) puis O(6) = {6} et
O(9) = {9, 11, 12} = O(11) = O(12). a 4 orbites, deux de cardinal 4, une de cardinal 3 et un singleton (correspondant
un point fixe).




1 3 4 7
2 5 8 10
4) est donc le produit commutatif des cycles c1 =
, c2 =
et
3 7 1 4
10 2 5 8


9 11 12
c3 =
.
12 9 11
On a c41 = c42 = Id et c33 = Id. Or, 2015 = 4 1002 + 3. Donc,
c2015
= c31 (c41 )1001 = c31 = c1
1
1 =
et de mme c2015
= c1
2
2 =
commutent,

2015

c2015
c2015
c2015
1
2
3

2
5

5 8
8 10

10
2

1 1
c1
1 c2 c3

1
4

3 4
1 7


671 2
. Puis, c2015
= c33
c3 = c1
3
3 =
1

= (c1 c2 c3 )

1 2
4 5

3 4
1 7

7
3


9
11

5 6
8 6

11
12

12
9

7 8
3 10


. Puisque c1 , c2 et c3

9
11

10
2

11
12

12
9

Exercice no 2 :
(Sn , ) est engendr par les transpositions. Il suffit donc de montrer que pour 2 6 i < j 6 n, la transposition i,j est
produit des 1,k , 2 6 k 6 n. Mais



 

1 i j
1 i j
1 i j
1 i j
1,i 1,j 1,i =
=
= i,j
i 1 j
j i 1
i 1 j
1 j i
ce quil fallait dmontrer.
Exercice no 3 :
Les lments de An sont les produits pairs de transpositions. Il suffit donc de vrifier quun produit de deux transpositions
est un produit de cycles de longueur 3.
Soient i, j et k trois lments deux deux distincts de J1, nK. i,k i,j est le 3-cycle : i j j k k i, ce qui montre
quun 3-cycle est pair et que le produit de deux transpositions dont les supports ont en commun un singleton est un
3-cycle.
http ://www.maths-france.fr

c Jean-Louis Rouget, 2014. Tous droits rservs.


Le cas i,j i,j = Id = (231)(312) est immdiat. Il reste tudier le produit de deux transpositions supports disjoints.
Soient i, j, k et l quatre lments de deux deux distincts de J1, nK.
i,j k,l = (jikl) (ijlk) = (jilk) = (ljik) (jkil).
Donc, i,j k,l est un bien un produit de 3-cycles ce qui achve la dmonstration.
Exercice no 4 :
Daprs le no 2, il suffit de montrer que pour 2 6 i 6 n, 1,i peut scrire en utilisant uniquement = 1,2 et c = (2 3 ... n 1).
On note que cn = Id.
Tout dabord, pour 1 6 i 6 n 1, tudions = ci1 cni+1 . Soit k J1, nK.
cni+1 (k) 6= cni+1 (k) cni+1 (k) {1, 2} k {cn+i1 (1), cn+i1 (2)} k {ci1 (1), ci1 (2)}
k {i, i + 1}.
Donc, si k
/ {i, i + 1},
(k) = ci1 (k)( cni+1 (k)) = ci1 (cni+1 (k)) = cn (k) = k,
et la restriction de {1, ..., n} \ {i, i + 1} est lidentit de cet ensemble. Comme nest pas lidentit (car sinon =
ci+1n+i1 = cn = Id ce qui est faux), est donc ncessairement la transposition i,i+1 .
On a montr que i J1, n 1K, ci1 cni+1 = i,i+1 .
Vrifions maintenant que les 1,i scrivent laide des j,j+1 . Daprs le no 2, i,j = 1,i 1,j 1,i , et donc bien sr,
plus gnralement, i,j = k,i k,j k,i .
Par suite, 1,i = 1,2 2,i 1,2 puis, 2,i = 2,3 3,i 2,3 , puis, 3,i = 3,4 4,i 3,4 ... et i2,i = i2,i1 i1,i
i2,i1 . Finalement,
1,i = 1,2 2,3 ... i2,i1 i1,i i2,i1 ... 2,3 1,2 ,
ce qui achve la dmonstration.
Exercice no 5 :
Soit (G, ) un groupe. Pour x lment de G, on considre fx :
plus, clairement fx fx1 = fx1

G G . fx est une application de G vers G et de


y 7 xy
fx = IdG . Donc, pour tout lment x de G, fx est une permutation de G.

Soit alors :

(G, ) (SG , ) . Daprs ce qui prcde, est une application. De plus, est de plus un morphisme
x
7
fx
de groupes. En effet, pour (x, x , y) G3 , on a :
((xx ))(y) = fxx (y) = xx y = fx (fx (y)) = fx fx (y) = ((x) (x ))(y),
et donc (x, x ) G2 , (xx ) = (x)o(x ).
Enfin, est injectif car, pour (x, x ) lment de G2 ,
(x) = (x ) y G, xy = x y xe = x e x = x ,

(e dsignant llment neutre de G).

est ainsi un isomorphisme de groupes de (G, ) sur (f(G), ) qui est un sous groupe de (SG , ). (G, ) est bien isomorphe
un sous groupe de (SG , ).
Exercice no 6 :
Montrons dabord par rcurrence sur l > 2 que la signature dun cycle de longueur l est (1)l1 .
Cest connu pour l = 2 (signature dune transposition).
Soit l > 2. Supposons que tout cycle de longueur l ait pour signature (1)l1 . Soit c un cycle de longueur l + 1.
On note {x1 , x2 , ..., xl+1 } le support de c et on suppose que, pour 1 6 i 6 l, c (xi ) = xi+1 et que c (xl+1 ) = x1 .
Montrons alors que x1 ,xl+1 c est un cycle de longueur l. x1 ,xl+1 c fixe dj xl+1 puis, si 1 6 i 6 l 1,
x1 ,xl+1 c(xi ) = x1 ,xl+1 (xi+1 ) = xi+1
(car xi+1 nest ni x1 , ni xl+1 ), et enfin x1 ,xl+1 c(xl ) = x1 ,xl+1 (xl+1 ) = x1 . x1 ,xl+1 c est donc bien un cycle
http ://www.maths-france.fr

c Jean-Louis Rouget, 2014. Tous droits rservs.


de longueur l. Par hypothse de rcurrence, x1 ,xl+1 c a pour signature (1)l1 et donc, c a pour signature (1)(l+1)1 .
Le rsultat est dmontr par rcurrence.
Montrons maintenant que si est une permutation quelconque de J1, nK ayant k orbites la signature de est (1)nk .
Si est lidentit, a n orbites et le rsultat est clair.
Si nest pas lidentit, on dcompose en produit de cycles supports disjoints.
Posons = c1 ...cp o p dsigne le nombre dorbites de non rduites un singleton et donc k p est le nombre de points
fixes de . Si li est la longueur de ci , on a donc n = l1 + ... + lp + (k p) ou encore n k = l1 + ... + lp p. Mais alors,
() =

p
Y

(ci ) =

p
Y
(1)li 1 = (1)l1 +...+lp p = (1)nk .
i=1

i=1

Exercice no 7 :
1) a) Soient et deux lments de Sn . Soit (i, j) J1, nK2 . Le coefficient ligne i, colonne j de P P vaut
n
X

i,(k) k, (j) = i,( (j)) (obtenu quand k = (j)),

k=1

et est donc aussi le coefficient ligne i, colonne j de la matrice P . Par suite,


(, ) (Sn )2 , P P = P .
b) Soit Sn . Daprs a), P P1 = P1 = PId = In = P1 P . On en dduit que toute matrice P est inversible,
dinverse P1 G. Par suite, G GLn (R) (et clairement, G 6= ).
Soit alors (, ) (Sn )2 .
P (P )

= P P 1 = P 1 G.

On a montr que G est un sous-groupe de (GLn (R), ).


Soit :
est injectif.

Sn

G . Daprs a), est un morphisme de groupes. est clairement surjectif. Il reste vrifier que
7 P

Soit (, ) S2n .
() = ( ) P = P (i, j) J1, nK2 , i,(j) = i, (j)

i J1, nK, i,(i) = i, (i) i J1, nK, (i) = (i)


= .

Donc est injectif.


Finalement, est un isomorphisme du groupe (Sn , ) sur le groupe (G, ) et on a montr que (G, ) est un sous-groupe
de (GLn (R), ), isomorphe (Sn , ).
2) Soit (i, j) J1, nK2 . Le coefficient ligne i, colonne j de AP vaut :
n
X

ai,k k,(j) = ai,(j) (obtenu quand k = (j)).

k=1

Ainsi, llment ligne i, colonne j, de AP est llment ligne i, colonne (j), de A, ou encore, si j est un lment donn
de J1, nK, la j-me colonne de AP est la (j)-me colonne de A. Ainsi, si on note C1 ,...,Cn les colonnes de A (et donc
A = (C1 , ..., Cn )), alors AP = C(1) , ..., C(n) . En clair, multiplier A par P droite a pour effet dappliquer la
permutation aux colonnes de A (puisque P est inversible, on retrouve le fait que permuter les colonnes de A ne modifie
pas le rang de A).
De mme, le coefficient ligne i, colonne j, de P A vaut
n
X
k=1

i,(k) ak,j =

n
X

1 (i),k ak,j = a1 (i),j ,

k=1

(on a utilis (k) = i k = 1 (i)) et multiplier A par P gauche a pour effet dappliquer la permutation 1 aux
lignes de A.
http ://www.maths-france.fr

c Jean-Louis Rouget, 2014. Tous droits rservs.


Exercice no 8 :
G = {A1 , ..., Ap } est dj une partie non vide de GLn (R), stable pour . Il reste vrifier que G est stable pour le passage
linverse.
Soient i J1, nK, puis i :

G
A

G . Puisque G est stable pour le produit, i est une application de G dans G.


7 Ai A

Montrons que i est injective. Soit (A, B) G2 .

1
( A) = i (B) Ai A = Ai B A1
i Ai A = Ai Ai B A = B.

Donc, i est une application injective de lensemble fini G dans lui-mme. On sait alors que i est une permutation de
G.
1
Par i , Ai a un antcdent A dans G. Lgalit Ai A = Ai fournit A1
i Ai A = Ai Ai puis A = I G. Ainsi, G contient
la matrice I. Ensuite, I a un antcdent par i dans G. Donc, il existe B G telle que Ai B = I. Mais alors A1
i = B G.
G est bien stable pour le passage linverse et est donc un sous-groupe de (GLn (R), ).

Exercice no 9 :
Pour (x1 , ..., xn ) E, on pose ((x1 , ..., xn )) = x1 + ... + xn . est une forme linaire non nulle sur E et H est le noyau de
. H est donc bien un hyperplan de E.
Il est clair que, pour (, ) S2n , f f = f .
(L (E), +, .) est un espace vectoriel et donc p est bien un endomorphisme de E.
1
p2 = 2
n!

Sn

!2

f f =

f .

(, )(Sn )2

(, )(Sn )2

(Sn , ) est un groupe fini. Par suite, lapplication Sn

Sn , injective (mme dmarche que dans le no 8), est une


7
X
X
permutation de Sn . On en dduit que, pour donne,
f =
f . Ainsi,
p2 =

1
n!2

Sn

Sn

!Sn
=

Sn

1
n!2

n!p =

Sn

1
n!.n!p = p.
n!2

p est donc une projection. Dterminons alors limage et le noyau de p. Soit i J1, nK.
p(ei ) =

1 X
1 X
f (ei ) =
e(i) .
n!
n!
Sn

Sn

Maintenant, il y a autant de permutations telles que (i) = 1, que de permutations telles que (i) = 2,... ou de
n!
permutations telles que (i) = n, savoir
= (n 1)!. Donc,
n
n
n
1 X
1 n! X
ek =
ek .
i J1, nK, p(ei ) =
n! n
n
k=1

Posons u =

1
n

n
X

k=1

ek . Daprs ce qui prcde,

k=1

Imp = Vect(p(e1 ), ..., p(en )) = Vect(u).


Ensuite, si x = x1 e1 + ... + xn en est un lment de E,
p(x) = 0

n
X
k=1

xk p(ek ) = 0

n
X
k=1

xk

u=0

n
X
k=1

xk = 0 x H.

Ainsi, p est la projection sur Vect(u) paralllement H.

http ://www.maths-france.fr

c Jean-Louis Rouget, 2014. Tous droits rservs.


Planche no 34. Dterminants


* trs facile ** facile *** difficult moyenne **** difficile
I : Incontournable T : pour travailler et mmoriser le cours
Exercice no 1 : (**)

2a a + b a + c

Montrer que b + a 2b b + c
c + a c + b 2c

Exercice no 2 : (**)




= 4(b + c)(c + a)(a + b).





Pour a, b et c donns, dont les valeurs absolues sont deux deux distinctes, factoriser

X
a
b
c

a b c
X c b
c X a
b a X

Exercice no 3 : (***)

Calculer :


a 0 ...


0 a ...

..
..

.
1) det(|i j|)16i,j6n 2) det(sin(ai + aj ))16i,j6n (a1 ,...,an tant n rels donns) 3) . 0
.
.. 0

0 b

b 0 ...


X 1

0 ...
...
0





1 1 ...
1
.


.


.
.


.
0 X 1
.
1 1 0 ... 0






..
..
..
.. ..
..
..
..
.
.


j1
.
.
.
.
.
.
.
.
. . 5) det(Cn+i1 )16i,j6p+1 6) .
4) . 0



.
.
.
.

.. .. . .
.
.
.
.


.
.
.

. .
0
.
.


1
0




1 0 ... 0 1
... ... 0
X
1
0

a0 . . .
. . . an2 an1 X


b

b 0
..
0
.
..
0
.
a 0
... a

...

..

Exercice no 4 : (****) (Dterminant de Cauchy et dterminant de Hilbert)




1
o a1 ,..., an , b1 ,...,bn sont 2n rels tels que toutes les sommes ai + bj soient non nulles. CalSoit A =
ai + bj 16i,j6n
culer detA (en gnralisant lide du calcul dun dterminant de Vandermonde par lutilisation dune fraction rationnelle)
et en donner une criture condense dans le cas ai = bi = i.
Exercice no 5 : (****)
Soit A = (ai,j )16i,j6n o, pour tout i et tout j, ai,j {1, 1}. Montrer que detA est un entier divisible par 2n1 .
Exercice no 6 : (***)

Rsoudre le systme MX = U o M =

1
1
1
..
.

1
2
22
..
.

1 2n1

... ...
... ...
... ...

1
n
n2
..
.

. . . . . . nn1

et U =

1
0
0
..
.
0

Exercice n 7 : (***)
o

Soit (A, B) (Mn (R))2 et C =

A B
B A

M2n (R). Montrer que det(C) > 0.

Exercice no 8 : (**)
Soit A = (ai,j )16i,j6n et B = (bi,j )16i,j6n avec bi,j = (1)i+j ai,j . Montrer que detB = detA.
Exercice no 9 : (***)
Dterminer les matrices A, carres dordre n, telles que pour toute matrice carre B dordre n on a det(A+B) = detA+detB.
http ://www.maths-france.fr

c Jean-Louis Rouget, 2014. Tous droits rservs.


Exercice no 10 : (****) (Dterminant

a1
a2 . . .
...
an
a
a
1
2

an1 an a1

.
..
.
Soit A =
..
. ..

..
..

.
.
a2
a3 . . .
an
dduire detA.

circulant)

an
an1

an2

et P = ((k1)(l1) )16k,l6n o = e2i/n . Calculer P2 et PA. En


..

..

.
a1

Exercice no 11 : (***)
Soit A Mn (C). Calculer det(com(A)) en fonction de detA puis tudier le rang de com(A) en fonction du rang de A.
Exercice no 12 : (***) (Drive dun dterminant)
Soient ai,j ((i, j) lment de J1, nK2 ) n2 fonctions de R dans R, drivables sur R et A = (ai, j)16i,j6n . Calculer la drive
de la fonction x 7 det(A(x)).





x + a1
x+1
x
...
x
1
...
1







.
.
.
.
.
.
.
.



1
.
.
.
x
x + a2
.
x+1






..
.
.
.
..
..
..
..
..
..
..
..
.. 2)
Applications. Calculer : 1) .

.
.
.
.
.
.







.
.
.
.
.
.
.
.
.
.. ..



..
.
.
x
.
1





1
x
...
. . . x x + an
...
... 1 x + 1
Exercice no 13 : (***)

Calculer :

0 1


1 0


1) ... . . .

.
..

1 ...

a1 + x


b+x


..
4)
.


.
..


b+x


...
1 1 1
..
..
.
. 1 0
.. et .. . .
.. ..
.
. . .
.
.
..
. 0 1 ..
... 1 0 1 ...
c + x ...
...
..
.
a2 + x
..
..
..
.
.
.
..
. an1 + x
...
...
b+x

http ://www.maths-france.fr

...
1
..
..
.
.
..
..
..
.
. .
..
. 0 1
... 1 0

c + x
..


.


..
b, c
.


c + x
an + x









2) det (i + j 1)2 16i,j6n 3)






2 1


1 2


complexes distincts 5) 0 . . .

. .
..
..

0 ...

b a
.. . .
.
.
..
.
b ...
0 ...
.. ..
.
.
.. ..
.
.
..
. 2
0
1

...
..
.
..
.
..
.
...

0
...


0 .

1
2

..

a
b











b
a
b
..
.
..
.

c Jean-Louis Rouget, 2014. Tous droits rservs.


Planche no 34. Dterminants : corrig


Exercice no 1 :

2x

3
Soit (a, b, c) R . Notons le dterminant de lnonc. Pour x rel, on pose D(x) = b + x
c+x
2
que = D(a)). D est un polynme de degr infrieur ou gal 2. Le coefficient de x vaut

x+b
2b
c+b

x+c
b+c
2c




(de sorte

(2c) + (b + c) + (b + c) (2b) = 4(b + c).


Puis,

2b

D(b) = 0
cb

b + c
b+c
2c

0
2b
c+b




= 2b(4bc (b + c)2 ) + 2b(c b)2 = 0,

et par symtrie des roles D(c) = 0. De ce qui prcde, on dduit que si b 6= c, D(x) = 4(b + c)(x + b)(x + c) (mme si
b + c = 0 car alors D est un polynme de degr infrieur ou gal 1 admettant au moins deux racines distinctes et est
donc le polynme nul).
Ainsi, si b 6= c (ou par symtrie des roles, si a 6= b ou a =
6 c, on a
encore tudi savoir le cas o a = b = c. Dans ce cas,



2a 2a
1 1
2a
1


3


D(a) = 2a 2a 2a = 8a 1 1 1
2a
1
2a 2a
1 1

: = 4(b + c)(a + b)(a + c). Un seul cas nest pas





= 32a3 = 4(a + a)(a + a)(a + a),

ce qui dmontre lidentit propose dans tous les cas (on pouvait aussi conclure en constatant que, pour a et b fixs,
la fonction est une fonction continue de c et on obtient la valeur de pour c = b en faisant tendre c vers b dans
lexpression de dj connue pour c 6= b).
Exercice no 2 :


X a b c


a X c b
. P est un polynme unitaire de degr 4.

Soit P =

b c X a
c b a X
En remplaant C1 par C1 + C2 + C3 + C4 et par linarit par rapport la premire colonne, on voit que P est divisible
par (X + a + b + c). Mais aussi, en remplaant C1 par C1 C2 C3 + C4 ou C1 C2 + C3 C4 ou C1 + C2 C3 C4 ,
on voit que P est divisible par (X a b + c) ou (X a + b c) ou (X + a b c).
Si les quatre nombres a b c, a + b + c, a b + c et a + b c sont deux deux distincts, P est unitaire de degr
4 et divisible par les quatre facteurs de degr 1 prcdents, ceux-ci tant deux deux premiers entre eux. Dans ce cas,
P = (X + a + b + c)(X + a + b c)(X + a b + c)(X a + b + c).
Notons alors que a b c = a + b c b = a et que a + b + c = a b + c a = b. Par symtrie des rles, deux
des quatre nombres a b c, a + b + c, a b + c et a + b c sont gaux si et seulement si deux des trois nombres
a, b ou c sont gaux en valeur absolue ce qui nest pas le cas.
Exercice no 3 :

0

1


1) Pour n > 2, posons n = 2

..

.

n1







. Tout dabord, on fait apparatre beaucoup de 1.
1
0


.. ..
.
.
1
n 2 ... 1
0


1 1 . . . 1 n 1



..

1 1

.
n

2




.
.
.
.
n = det(C1 C2 , C2 C3 , ..., Cn1 Cn , Cn ) = 1
.
1 1 .
.


.

..
..

.
1
1


1
1 ... 1
0
1
0

2
1

... n 1
n2
..
..
.
.

Puis, on fait apparatre un dterminant triangulaire en constatant que det(L1 , L2 , ..., Ln ) = det(L1 , L2 + L1 , ..., Ln1 +
L1 , Ln + L1 ), ce qui fournit :
http ://www.maths-france.fr

c Jean-Louis Rouget, 2014. Tous droits rservs.








n =



1 1 . . . 1 n 1
0 2

..
.
0
0
..
..
. 2

.
0 ... 0
0 n1







= (1 n)(2)n2 .




2) (i, j) J1, nK2 , sin(ai + aj ) = sin ai cos aj + cos ai sin aj et donc si on pose C =

j J1, nK, Cj = cos aj S + sin aj C.

cos a1
cos a2
..
.

cos an

et S =

sin a1
sin a2
..
.
sin an

En particulier, Vect(C1 , ..., Cn ) Vect(C, S) et le rang de la matrice propose est infrieur ou gal 2. Donc,
n > 3, det (sin (ai + aj ))16i,j6n = 0.
Si n = 2, det (sin (ai + aj ))16i,j62 = sin (2a1 ) sin (2a2 ) sin2 (a1 + a2 ).
3) Lexercice na de sens que si le format n est pair. Posons n = 2p o p est un entier naturel non nul.








n =





a
0
..
.
..
.
0
b

0
..
.

... ...
0

0
0
... ...

0
..
.
0









= (a + b)p














p
= (a + b)





1
0
..
.
..
.
0
1
1
0
..
.
..
.
0

0
..
.

...

0
...

0
..
.

...

1
..
.

...



...
. . . 0 b
b a + b 0


..
.
0
0
0
0 0
..
..
..
.
.
0 a+b b
0
.
=


.. (pour 1 6 j 6 p, Cj Cj + C2p+1j )
..
..
.
.
0 b+a a
0
.


..


. 0
0
0
0 0
...
... 0 a
a b+a 0

. . . 0 b

0
0
..
b
0
.
.. (par linarit par rapport aux colonnes C1 , C2 , ..., Cp )
a
0
.

..
. 0
0
... 0 a

...
0
b

0
0

..

b
0
.
(pour p + 1 6 i 6 2p, Li Li L2p+1i ).

..

ab 0
.


..
..
.
.
0
...
0 ab

et n = (a + b)p (a b)p = (a2 b2 )p .

4) On retranche la premire colonne la somme de toutes les autres et




1 1 ...
1 (n 2) 1

1 1 0 ... 0
0
1



..

.
.
.
.
.
..
..
. . .. =
Dn = . 0 . .

. . .

.
..
.. 1 0
.. ..


1 0 ... 0 1
0
...

5) Pour 1 6 i 6 p,

http ://www.maths-france.fr

on obtient

...
1
0 . . . 0
.
..
..
.
. .. = (n 2).

..
. 1 0
... 0 1

c Jean-Louis Rouget, 2014. Tous droits rservs.


p
p1
0
1
Li+1 Li = (C0n+i C0n+i1 , C1n+i C1n+i1 , ..., Cp
n+i Cn+i1 ) = (0, Cn+i1 , Cn+i1 , ..., Cn+i1 ).

On remplace alors dans cet ordre Lp par Lp Lp1 puis Lp1 par Lp1 Lp2 puis ... puis L2 par L2 L1 pour obtenir,
avec des notations videntes


1


det (Ap ) =
= det(Ap1 ).
0 Ap1

Par suite, det(Ap ) = det(Ap1 ) = ... = det(A2 ) = 1.

6) En dveloppant suivant la dernire ligne, on obtient :


Dn = (an1 X)(X)n1 +

n2
X

(1)n+k+1 ak k ,

k=0








o k =




X
0
0
0
0

1
..
.
0








= (1)k Xk et donc
1
0 0

..
. 0
X
0 X 1

0
1
X
0
0

Dn = (1)n

Xn

n1
X

ak Xk

k=0

Exercice no 4 :
Si deux des bj sont gaux, det(A) est nul car deux de ses colonnes sont gales. On suppose dornavant que les bj sont
deux deux distincts.
Soient 1 ,..., n , n nombres complexes tels que n =
6 0.

n
X
1
det C1 , ..., Cn1 ,
j Cj = det(B),
det(A) =
n
j=1

o la dernire colonne de B est de la forme (R (ai ))16i6n avec R =

n
X
j=1

j
.
X + bj

(X a1 )...(X an1 )
. R ainsi dfinie est irrductible (car (i, j) {1, ..., n}2 , ai 6= bj ). Les ples de R
(X + b1 )...(X + bn )
sont simples et la partie entire de R est nulle. La dcomposition en lments simples de R a bien la forme espre.
On prend R =

Pour ce choix de R, puisque R(a1 ) = ... = R(an1 ) = 0, on obtient en dveloppant suivant la dernire colonne
n =

1
R(an )n1 ,
n

avec
n =

lim (z + bn )R(z) =

zbn

(bn a1 )...(bn an1 )


(a1 + bn )...(an1 + bn )
=
.
(bn + b1 )...(bn + bn1 )
(bn b1 )...(bn bn1 )

Donc
n > 2, n =

(an a1 ) ... (an an1 ) (bn b1 ) ... (bn bn1 )


n1 .
(an + b1 ) (an + b2 ) ... (an + bn ) .. (a2 + bn ) (a1 + bn )

En ritrant et compte tenu de 1 = 1, on obtient

n =

(aj ai )

16i<j6n

(bj bi )

16i<j6n

(ai + bj )

16i,j6n

Van (a1 , ..., an ) Van (b1 , ..., bn )


.
Y
(ai + bj )
16i,j6n

Dans le cas particulier o i J1, nK, ai = bi = i, en notant Hn le dterminant (de Hilbert) calculer :
http ://www.maths-france.fr

c Jean-Louis Rouget, 2014. Tous droits rservs.


Hn =

Van(1, 2, ..., n)2


.
Y
(i + j)
16i,j6n

Mais,

(i + j) =

16i,j6n

n
Y
i=1

2n
Y

n
n
Y
Y
(n + i)!
(i + j) =
=
i!
j=1

k!

k=1
n
Y

i=1

k=1

k!

!2 ,

et dautre part,
Van(1, 2, ..., n) =

16i<j6n

Donc, n > 1, Hn =

n
Y

k!

k=1
2n
Y
n!2

!3

(j i) =

n1
Y
i=1

n
Y

(j i) =

j=i+1

n
1 Y
k!.
(n i)! =
n!

n1
Y
i=1

k=1

k!

k=1

Exercice no 5 :
On procde par rcurrence sur n > 1.
Pour n = 1, cest clair.
Soit n > 1. Supposons que tout dterminant n de format n et du type de lnonc soit un entier divisible par 2n1 .
Soit n+1 un dterminant de format n + 1, du type de lnonc.
Si tous les coefficients ai,j de n+1 sont gaux 1, puisque n + 1 > 2, n+1 a deux colonnes gales et est donc nul.
Dans ce cas, n+1 est bien divisible par 2n .
Sinon, on va changer petit petit tous les 1 en 1.

Soit (i, j) un couple dindices tel que ai,j = 1 et n+1


le dterminant dont tous les coefficients sont gaux ceux
de n+1 sauf le coefficient ligne i et colonne j qui est gal 1.

n+1 n+1
= det(C1 , ..., Cj , ..., Cn ) det(C1 , ..., Cj , ..., Cn ) = det(C1 , ..., Cj Cj , ..., Cn ),

0
..
.

o Cj Cj =
2 (2 en ligne i). En dveloppant ce dernier dterminant suivant sa j-me colonne, on obtient :
0

.
..
0

n+1 n+1
= 2n ,

o n est un dterminant de format n et du type de lnonc. Par hypothse de rcurrence, n est divisible

n
par 2n1 et donc n+1
n+1 est divisible par 2 . Ainsi, en changeant les 1 en 1 les uns aprs les autres, on obtient
1 ... 1



.. , (mod 2n ). Ce dernier dterminant tant nul,
n
n+1 ...

n+1 est un entier divisible par 2 .
.


1 ... 1

Le rsultat est dmontr par rcurrence.


Exercice no 6 :

det(M) = Van(1, 2, ..., n) 6= 0 et le systme est de Cramer. Posons = detM. Les formules de Cramer fournissent alors
k
o
pour k J1, nK, xk =

http ://www.maths-france.fr

c Jean-Louis Rouget, 2014. Tous droits rservs.









k = Van(1, ..., k 1, 0, k + 1, ..., n) = (1)k+1




1 ...
1
..
.
..
.
1

k1
(k 1)2
..
.

k+1
(k + 1)2
..
.

..
.
(k 1)n1

..
.
(k + 1)n1

...

n
n2
..
.
..
.
nn1

(en dveloppant par rapport la k-me colonne). Par linarit par rapport chaque colonne, on a

k = (1)k+1 1 2... (k 1) (k + 1)... n Van(1, 2, ..., k 1, k + 1, ..., n)


n!
Van(1, 2, ..., n)
n!

= (1)k+1
.
k
(k (k 1))...(k 1)((k + 1) k)....(n k)
k!(n k)!
 
n
Donc, k J1, nK, xk = (1)k+1
.
k
= (1)k+1

Exercice no 7 :
En remplaant les colonnes C1 ,..., Cn par respectivement C1 + iCn+1 ,..., Cn + iC2n , on obtient :


A + iB B
detC = det
,
B + iA A
puis en remplaant les lignes Ln+1 ,..., L2n de la nouvelle matrice par respectivement Ln+1 iL1 ,..., L2n iLn , on obtient :


A + iB
B
2
det(C) = det
= det(A + iB)det(A iB) = |det(A + iB)| R+ .
0
A iB
Exercice no 8 :
1re solution.

det(B) =

()(1)1+(1)+2+(2)+...+n+(n) a(1),1 a(2),2 ...a(n),n

()a(1),1 a(2),2 ...a(n),n (car 1 + (1) + 2 + (2) + ... + n + (n) = 2(1 + 2 + ... + n) 2N)

Sn

Sn

= detA.
2me solution. On multiplie par 1 les lignes 2, 4, 6... puis les colonnes 2, 4, 6...On obtient detB = (1)2p detA = detA
(o p est le nombre de lignes ou de colonnes portant un numro pair).
Exercice no 9 :
On suppose n > 2. La matrice nulle est solution du problme.
Soit A un lment de Mn (C) tel que B Mn (C), det(A+B) = detA+detB. En particulier, 2det(A) = det(2A) = 2n detA
puis detA = 0 car n > 2. Donc, A
/ GLn (C).
Si A 6= 0, il existe une certaine colonne Cj qui nest pas nulle. Puisque la colonne Cj nest pas nulle, on peut complter la famille libre (Cj ) en une base (C1 , ..., Cj , ..., Cn ) de Mn,1 (C). La matrice B dont les colonnes sont justement
C1 ,...,Cj ,...,Cn est alors inversible de sorte que detA + detB = detB 6= 0. Mais, A + B a une colonne nulle et donc
det(A + B) = 0 6= detA + detB.
Ainsi, seule la matrice nulle peut donc tre solution du problme. Rciproquement A = 0 est solution.
Exercice no 10 :
Le coefficient ligne k, colonne l de P2 vaut :
k,l =

n
X

(k1)(u1) (u1)(l1) =

u=1

n
X

u=1

(k+l2)(u1) =

n
X

(k+l2 )u .

u=1

Or, k+l2 = 1 k + l 2 nZ. Mais, 0 6 k + l 2 6 2n 2 < 2n et donc, k + l 2 nZ k + l 2 {0, n}


k + l = 2 ou k + l = n + 2. Dans ce cas, k,l = n. Sinon,
http ://www.maths-france.fr

c Jean-Louis Rouget, 2014. Tous droits rservs.


k,l

Ainsi, P2 = n

1 0
0
..
.
0 1

n
1 k+l2
11
=
=
= 0.
k+l2
1
1 k+l2

... ... 0
0 1

.
0
1 0

0 ... 0

Le coefficient ligne k, colonne l de PP vaut :


k,l =

n
X

(k1)(u1)

(u1)(l1)

u=1

Or,

n
X

(kl )u1 .

u=1

kl

= 1 k l nZ. Mais, n < (n 1) 6 k l 6 n 1 < n et donc k l nZ k = l. Dans ce cas, k,l = n.


1
Sinon, k,l = 0. Ainsi, PP = nIn (ce qui montre que P GLn (C) et P1 = P).
n
Calculons enfin PA. Il faut dabord crire proprement les coefficients de A. Le coefficient ligne k, colonne l de A peut
scrire alk+1 si lon adopte la convention commode an+1 = a1 , an+2 = a2 et plus gnralement pour tout entier relatif
k, an+k = ak .
Avec cette convention dcriture, le coefficient ligne k, colonne l de PA vaut
n
X

(k1)(u1) alu+1 =

u=1

l
X

(k1)(lv) av .

v=ln+1

Puis on rordonne cette somme pour quelle commence par a1 .


l
X

(k1)(lv) av =

v=ln+1

l
X

(k1)(lv) av +

l
X

(k1)(lv) av +

l
X

(k1)(lv) av +

v=1

v=1
n
X

(k1)(lv) av

v=ln+1

v=1

0
X

n
X

(k1)(lw+n)aw+n (en posant w = v + n)

n
X

(k1)(lw)aw

w=l+1

w=l+1

(k1)(lv) av = (k1)(l1)

v=1

n
X

(k1)(1v) av

v=1

(le point cl du calcul prcdent est que les suites ak et k ont mme priode n ce qui sest traduit par (k1)(lw+n) aw+n =
(k1)(lv) av ).
Posons alors Sk =

n
X

(k1)(1v) av pour k lment de J1, nK. On a montr que PA = ((k1)(l1) Sk )16k,l6n .

v=1
n
Y

=
Sk det (k1)(l1)
det(PA) = det (k1)(l1) Sk 16k,l6n =
16k,l6n
k=1
!
n
Y
Donc detP detA =
Sk detP. Finalement, puisque detP 6= 0,


n
Y

Sk

k=1

detP.

k=1

detA =

n X
n
Y
(
(k1)(1v) av ).

k=1 v=1

Par exemple, pour n = 3, detA = (a1 + a2 + a3 )(a1 + ja2 + j2 a3 )(a1 + j2 a2 + ja3 ).


Exercice no 11 :
On a toujours A t comA = (detA)In et donc
(detA)(det(comA)) = (detA)(det(t comA)) = det((detA)In ) = (detA)n .
http ://www.maths-france.fr

c Jean-Louis Rouget, 2014. Tous droits rservs.


Si detA 6= 0, on obtient det(comA) = (detA)n1 .


Si detA = 0, alors At comA = 0 et comA nest pas inversible car sinon, A = 0 puis comA = 0 ce qui est absurde.
Donc, det(comA) = 0. Ainsi, dans tous les cas,
det(comA) = (detA)n1 .
Si rgA = n, alors comA GLn (K) (car det(comA) 6= 0) et rg(comA) = n.
Si rgA 6 n 2, soit A une matrice de format n 1 extraite de A cest--dire obtenue en supprimant dans A une ligne
et une colonne. Alors rg(A ) 6 rg(A) 6 n 2 < n 1. Puisque A est de format n 1, A nest pas inversible et donc
det(A ) = 0. Ainsi, tous les coefficients de la matrice com(A) sont nuls et donc comA = 0. Dans ce cas, rg(comA) = 0.
Si rgA = n 1, il existe n 1 colonnes de A constituant une famille libre. Soit A la matrice de format (n, n 1) constitue
de ces colonnes. On a rg(A ) = n 1. Donc, il existe n 1 lignes de A constituant une famille libre. Soit A la matrice
de format (n 1, n 1) constitue de ces lignes. On a rg(A ) = n 1. Donc A est une matrice carre inversible. On en
dduit que det(A ) 6= 0. Mais det(A ) est, au signe prs lun des coefficients de la matrice com(A). Donc, com(A) 6= 0.
Par suite, rg(comA) > 1. Dautre part,

A t (comA) = 0 comA t A = 0 Im t A Ker(comA) dim(Ker(comA)) > rg(t A) = rgA = n 1
n rg(comA) > n 1
rg(comA) 6 1,

et finalement si rgA = n 1, rg(comA) = 1.


Exercice no 12 :
X
()a(1),1 a(2),2 ...a(n),n et donc la fonction x 7 det(A(x)) est drivable sur R en tant que combinaison
detA =
Sn

linaire de produits de fonctions drivables sur R. De plus, en notant C1 , . . . , Cn , les colonnes de A,


X

(detA) =

()a(1),1 a(2),2 ...a(n),n

Sn

n X
X

Sn
n
X

...a(n),n =
()a(1),1 ...a(k),k

()

n
X

...a(n),n
a(1),1 ...a(k),k

k=1

det(C1 , ..., Ck , ..., Cn )

k=1

k=1 Sn

Applications.

x+1
1
...


.
1
x + 1 ..


..
..
1) Soit n (x) = ...
.
.

.
.
..
..

1
...
...
n
X
n =
k o k est le dterminant

..

..








. n est un polynme dont la drive est daprs ce qui prcde,


1
x+1
1
..
.
..
.

dduit de n en remplaant sa k-me colonne par le k-me vecteur de la base

k=1

canonique de Mn,1 (K). En dveloppant k par rapport sa k-me colonne, on obtient k = n1 et donc n = nn1 .

On a dj 1 = X + 1 puis 2 = (X + 1)2 1 = X2 + 2X ...


Montrons par rcurrence que pour n > 1, n = Xn + nXn1 .

= (n + 1)Xn + (n + 1)nXn1 et, par


Cest vrai pour n = 1 puis, si pour n > 1, n = Xn + nXn1 alors n+1
n+1
n
intgration, n+1 = X
+ (n + 1)X + n+1 (0). Mais, puisque n > 1, on a n + 1 > 2 et n+1 (0) est un dterminant
ayant au moins deux colonnes identiques. Par suite, n+1 (0) = 0 ce qui montre que n+1 = Xn+1 + (n + 1)Xn .

Le rsultat est dmontr par rcurrence.




x + a1

x
...
x




.
..
..


.
x
x
+
a
2




.
.
.
.
.
..
..
..
..
..
2) Soit n (x) =
. n = det(a1 e1 + xC, ..., an en + xC) o ek est le k-me vecteur de




..
..
..


.
.
x
.



x
...
. . . x x + an
la base canonique de Mn,1 (K) et C est la colonne dont toutes les composantes sont gales 1. Par linarit par rapport
http ://www.maths-france.fr

c Jean-Louis Rouget, 2014. Tous droits rservs.


chaque colonne, n est somme de 2n dterminants mais ds que C apparait deux fois, le dterminant correspondant est
nul. Donc, n = det(a1 e1 , ..., an en ) + det(a1 e1 , ..., xC, ..., an en ). Ceci montre que n est un polynme de degr infrieur
ou gal 1.
n
Y

La formule de Taylor fournit alors : n = n (0) + Xn (0). Immdiatement, n (0) =


ak = n puis
k=1

n (0)

n
X

det(a1 e1 , ..., C, ..., an en ) =

n Y
X

ai = n1 .

k=1 i6=k

k=1

Donc, n = n + Xn1 .
Exercice no 13 :
1) Pour le deuxime dterminant, on retranche la premire colonne chacune des autres et on obtient un dterminant
triangulaire infrieur dont la valeur est (1)n1 . Pour le premier, on ajoute la premire colonne la somme de toutes
les autres, puis on met (n 1) en facteurs de la premire colonne et on tombe sur le deuxime dterminant. Le premier
dterminant vaut donc (1)n1 (n 1).
2) Pour (i, j) lment de J1, nK2 , (i + j 1)2 = j2 + 2(i 1)j + (i 1)2 . Donc,
j J1, nK, Cj = j2 (1)16i6n + 2j(i 1)16i6n + ((i 1)2 )16i6n .



Les colonnes de la matrice sont donc lments de Vect (1)16i6n , (i 1)16i6n , (i 1)2 16i6n qui est de dimension
infrieure ou gale 3 et la matrice propose est de rang infrieur
ou gal
3. Donc, si n > 4, n = 0. Il reste ensuite


1 4

9

1 4
= 7 puis 3 = 4 9 16 = (225 256) 4(100 144) + 9(64 81) =
calculer 1 = 1 puis 2 =


4 9
9 16 25
31 + 176 153 = 8.
3)








n = det(C1 , ..., Cn ) = det(C1 + ... + Cn , C2 , ..., Cn ) = (a + (n 1)b)




1
1
..
.
..
.
1

... ... b
..
.
a ..
.
..
..
..
.
. .
b
.. . .
..
.
. b
.
b ... b a

par linarit par rapport la premire colonne. Puis, aux lignes numros 2,..., n, on retranche la premire ligne pour
obtenir :


1
b
... ...
b

0 a b 0 ...
0

..

..
.
.
.. ..
= (a + (n 1)b)(a b)n1 .
0
.
n = (a + (n 1)b) .

.

..
..
..
..
.
.
0
.

0
0
... 0 a b

4) Par n linarit, Dn est somme de 2n dterminants. Mais dans cette somme, un dterminant
est

nul ds quil contient


b
..
.

au moins deux colonnes de x. Ainsi, en posant n = det(C1 + xC, ..., Cn + xC) o Ck =


ak et C = (1)16i6n , on
b

.
..
b
obtient :
n = det(C1 , ..., Cn ) +

n
X

det (C1 , ..., Ck1 , xC, Ck+1 , ..., Cn ) ,

k=1

http ://www.maths-france.fr

c Jean-Louis Rouget, 2014. Tous droits rservs.


ce qui montre que n est un polynme de degr infrieur ou gal 1. Posons n = AX + B et P =

n
Y

(ak X). Quand

k=1

x = b ou x = c, le dterminant propos est triangulaire et se calcule donc immdiatement. Donc :



P(c) P(b)
bA + B = P(b)
et donc A =
1er cas. Si b 6= c. n (b) = P(b) et n (c) = P(c) fournit le systme
cA + B = P(c)
cb
cP(b) bP(c)
et B =
. Ainsi, si b 6= c,
cb
n
Y
cP(b) bP(c)
P(c) P(b)
x+
o P =
(ak X).
n =
cb
cb
k=1

2me cas. Si b = c, lexpression obtenue en fixant x et b est clairement une fonction continue de c car polynomiale en c.
On obtient donc la valeur de n quand b = c en faisant tendre c vers b dans lexpression dj connue de n pour b 6= c.
P(c) P(b)
Maintenant, quand b tend vers c,
tend vers P (b) et
cb
cP(b) bP(c)
c(P(b) P(c)) + (c b)P(c)
=
,
cb
cb
tend vers bP (b) + P(b). Si b = c,

n = xP (b) + P(b) bP (b) o P =

n
Y

(ak X).

k=1


2 1

5) 2 = 3 et 3 = 1 2
0 1

0
1
2




= 2 3 2 = 4. Puis, pour n > 4, on obtient en dveloppant suivant la premire colonne :


n = 2n1 n2 .

Do, pour n > 4, n n1 = n1 n2 et la suite (n n1 )n>3 est constante. Par suite, pour n > 3,
n n1 = 3 2 = 1 et donc la suite (n )n>2 est arithmtique de raison 1. On en dduit que, pour n > 2,
n = 2 + (n 2) 1 = n + 1 (on pouvait aussi rsoudre lquation caractristique de la rcurrence double).
n > 2, n = n + 1.

http ://www.maths-france.fr

c Jean-Louis Rouget, 2014. Tous droits rservs.


Planche no 35. Systmes dquations linaires


* trs facile ** facile *** difficult moyenne **** difficile
I : Incontournable T : pour travailler et mmoriser le cours
Exercice no 1
Rsoudre (en discutant en fonction des diffrents paramtres) les systmes suivants :

x+y+z+t=3
2x + my + z = 3m
2x + 3y + z = 4
x + my + z mt = m + 2
x (2m + 1)y + 2z = 4
x + my + 2z = 5
3)
2)
1)

mx y mz t = 1
5x y + 4z = 3m 2
7x + 3y + (m 5)z = 7

x + 2y + 3z + mt = m 1

2x + y + mz + 3t = 1
4)
3x + my + z + 2t = 0

mx + 3y + 2z + t = 0

mx + y + z = m + 2

x y + mz = m 2
5)
mx + y + mz = m

x y mz = m 4

(b + c)2 x + b2 y + c2 z = 1
7)
a2 x + (c + a)2 y + c2 z = 1
2
a x + b2 y + (a + b)2 z = 1

ax + by + cz = p
cx + ay + bz = q
8)

bx + cy + az = r

x+y+z=1
ax + by + cz = m
6)

x+y+z = 1
a b c
m

Exercice no 2
Donner une base du sous-espace vectoriel de R5 dfini par :

x1 + 2x2 x3 + 3x4 + x5 = 0
x2 + x3 2x4 + 2x5 = 0
.

2x1 + x2 5x3 4x5 = 0


Exercice no 3

Dans le plan, on donne n points A1 , ... , An . Existe-t-il n points M1 ,..., Mn tels que A1 soit le milieu de [M1 , M2 ], A2
soit le milieu de [M2 , M3 ],..., An1 soit le milieu de [Mn1 , Mn ] et An soit le milieu de [Mn , M1 ].
Exercice no 4
Rsoudre le systme : x1 + x2 = 0, xk1 + xk + xk+1 = 0 pour k = 2, ..., n 1, xn1 + xn = 0.
Exercice no 5
Soit E un ensemble contenant au moins n lments et (f1 , f2 ..., fn ) un n-uplet de fonctions de E dans C. Montrer que les
propositions suivantes sont quivalentes :
1) la famille (f1 , ..., fn ) est libre ;
2) il existe n lments a1 , a2 ,..., an dans E tels que det(fi (aj ))16i,j6n 6= 0.
Exercice no 6
Dterminer linverse de A = (ai,j ) telle que ai,i+1 = ai,i1 = 1 et ai,j = 0 sinon.
Exercice no 7
Soient a1 ,..., an , b1 ,..., bn 2n nombres complexes deux deux distincts tels que les sommes ai + bj soient toutes non
n
X
xj
nulles. Rsoudre le systme
, pour tout i = 1, ..., n (en utilisant la dcomposition en lments simples de
a i + bj
j=1

R=

n
X
j=1

xj
).
X + bj

Exercice no 8
1) Pour a C, on pose x R, fa (x) = eax . Soient a1 , . . . , an n nombres complexes deux deux distincts.
Montrer que la famille de fonctions (fak )16k6n est libre.
2) Soient q1 , . . . , qp p nombres complexes
 deux deux distincts.
Montrer que la famille de suites (qn
)
k nN 16k6p est libre.

http ://www.maths-france.fr

c Jean-Louis Rouget, 2014. Tous droits rservs.


Planche no 35. Systmes dquations linaires : corrig


Exercice no 1
m, a, b, . . . sont des paramtres rels. On note (S) le systme propos et S lensemble des solutions de (S).
1) det(S) = 2(m(m 5) 6) + (3(m 5) 3) + 7(6 m) = 2m2 14m + 12 = 2(m 1)(m 6). Le systme est de Cramer
si et seulement si m {1, 6}.
Si m
/ {1, 6}, les formules de Cramer fournissent alors :



1

x=
2(m 1)(m 6)



1

y=
2(m 1)(m 6)



1

z=
2(m 1)(m 6)
Si m
/ {1, 6}, S =



4 3
1
4(m2 5m 6) 5(3m 18) 7(m 6)
2(m 6)(2m 9)
2m 9
=
5 m
2
=
=

2(m 1)(m 6)
2(m 1)(m 6)
m1
7 3 m 5

2 4
1
2(5m 39)) + (4m 27) + 21
14(m 6)
7
=
1 5
2
=
=

2(m 1)(m 6)
2(m 1)(m 6)
m1

7 7 m
5

2
3 4
14(m 6)
7
2(7m 15) + 9 + 7(4m + 15)
=
=
1 m 5 =
2(m 1)(m 6)
2(m 1)(m 6)
m1
7
3 7

2m 9
7
7
,
,
m1 m1 m1


.



2 1

= 5 6= 0. On peut choisir les deux premires quations
Si m {1, 6}, det(S) = 0. Un dterminant principal est
1 2
comme quations principales et x etz comme inconnues principales. Le systme des deux premires quations scrit


x = 3 + (m 6)y
2x + z = 4 3y
5
et quivaut
.
x + 2z = 5 my

z = 14 (2m + 3)y
5
La dernire quation fournit alors une condition ncessaire et suffisante de compatibilit (les termes en y disparaissent si
m {1, 6}).
14 (2m + 3)y
3 + (m 6)y
+ 3y + (m 5)
= 7 21 + 14(m 5) 35 = 0
5
5
14(m 6) = 0 m = 6.

7x + 3y + (m 5)z = 7 7

Si m = 1, le systme na pas de solution et si m = 6, lensemble des solutions est S =

3
y
, y,
5
5


, yR .

2) det(S) = 2(8m


4 + 2) (4m + 1) + 5(2m + 2m + 1) = 0. Le systme nest jamais de Cramer. Un dterminant
2 1

= 3 6= 0. On peut choisir les deux premires quations comme quations principales et x et z comme
principal est
1 2

x = 6m 4 (4m + 1)y
3
inconnues principales. Le systme des deux premires quations quivaut
. La dernire

z = 3m + 8 + (5m + 2)y
3
quation fournit alors une condition ncessaire et suffisante de compatibilit.
3m + 8 + (5m + 2)y
6m 4 (4m + 1)y
y+4
= 3m 2
3
3
5(6m 4) + 4(3m + 8) 3(3m 2) = 0 9(m + 2) = 0 m = 2.



16 + 7y
14 8y
Si m 6= 2, le systme na pas de solution. Si m = 2, lensemble des solutions est S =
, yR .
, y,
3
3


1
1
1


1 = 2m2 + 2m = 2m(m 1). Le systme est de Cramer en x, y et z si et seulement si m
/ {0, 1}.
3) 1 m
m 1 m
5x y + 4z = 3m 2 5

Si m
/ {0, 1}, les formules de Cramer fournissent :

http ://www.maths-france.fr

c Jean-Louis Rouget, 2014. Tous droits rservs.



3t
1
1
(2m2 2m)t + (2m2 + 2m)
= t + 1
m + 2 + mt m
1 =
2m(m 1)
1 + t
1 m
1
3t
1
m+1
(2m2 2m)t + (2m2 + 2m)
=
t+1
1 m + 2 + mt
1 =
2m(m 1)
m1
m
1 + t
m

1
1
3t
(2m2 + 2m)t + (2m2 + 2m)
m+1
1 m m + 2 + mt =
=
t + 1.
2m(m 1)
m1

m 1
1 + t



m+1
m+1
Dans ce cas, lensemble des solutions est S =
t + 1,
t + 1,
t + 1, t , t R .
m1
m1

x+y+z+t=3
z=2x
x+z=2
t = 1 y . Dans ce cas, le systme na pas de solution.
Si m = 0, le systme scrit

y + t = 1
1=3

x+y+z+t=3
t=0
t=0
Si m = 1, le systme scrit
x+y+zt=3

x+y+z=3

x=1
. Dans ce cas, lensemble de

x y z t = 1
x y z = 1
z=2y
solutions est {(1, y, 2 y, 0), y R}.



1

x=
2m(m 1)



1

y=
2m(m 1)



1

z=
2m(m 1)

4)

m + 6 2 3 m
m + 6 1 m 3
(C1 C1 + C2 + C3 + C4 )
m + 6 m 1 2
m+6 3 2 1



1
2
3
m
1 2 3 m

0
1
m 3 3 m
1 1 m 3
(i J2, 4K, Li Li L1 )
= (m + 6)

2
2 m
1 m 1 2
0 m2
0
1
1
1m
1 3 2 1



1
m3
0
1
m 3 3 m

2
m (C3 C3 + C2 )
m2
2
2 m = (m + 6) m 2


1
1
m
1
1
1m


1
m 3 0


2
1
= m(m + 6) m 2

1
1
1


1
m 3 0

1
0 (L2 L2 L3 )
= m(m + 6) m 3

1
1
1


1
m 3
= m(m + 6)
= m(m 2)(m 4)(m + 6).
m3
1





det(S) =

1
2
3
m

2
1
m
3




= (m + 6)





= (m + 6)

3
m
1
2

m
3
2
1

Le systme est de Cramer si et seulement si m


/ {0, 2, 4, 6}. Dans ce cas :


m1 2 3 m



1
1 m 3
=
m(m 2)(m 4)(m + 6)x =
0
m 1 2


0
3 2 1

3 m m2 + m + 3

1
= m

3
2


0 2 (m 1) 3 m(m 1) m 3(m 1)

1
1
m
3


0
m
1
2


0
3
2
1


5m 6 m2 + 5m 3 2m + 3
2m + 3



3
2
2
= m 6


0
0
1
1

= [3(5m 6) (m 6)(m2 + 5m 3)]

= m3 + 11m2 18m = m(m 2)(m 9).


et x =

m9
.
(m 4)(m + 6)

http ://www.maths-france.fr

c Jean-Louis Rouget, 2014. Tous droits rservs.






m(m 2)(m 4)(m + 6)y =





=




=

1
2
3
m

m1 3
1
m
0
1
0
2

m
3
2
1


2m + 3


2
=

3


m

m2 + m + 3
1
2

2m + 3
3
m

0
1
0
0

2m + 3

2


1

3m2 5m 6 m2 + m + 3
0
1
m6
2

m2 + m + 3
m
1
2

2m2 4m 3
0
3

= 3(3m2 5m 6) (m 6)(2m2 4m 3)

2m + 3
3
2
1

= 2m3 + 7m2 6m = m(2m 3)(m 2)


et y =

2m 3
.
(m 4)(m 6)

1 2 m 1 m 2m + 3
2
2 1
1
3
=
3
3 m
0
2
m
m 3
0
1


2m + 3 m + 3 2m + 3



3
m
2
=



m
3
1





m(m 2)(m 4)(m + 6)z =

m + 3
1
m
3

0 2m + 3
1
3
0
2
0
1

= (2m + 3)(m 6) + 3(5m 6) m(2m2 5m + 6) = 2m3 + 7m2 6m


= m(2m 3)(m 2),

et z =

2m 3
.
(m 4)(m 6)




m(m 2)(m 4)(m + 6)t =





=

1
2
3
m

2
1
m
3

3
m
1
2

2m + 3
3
m

m1
1
0
0
m + 3
m
3


2m + 3


2
=

3


m

m2 + m + 3
1
2

m + 3
1
m
3

m2 + m + 3
m
1
2

0
1
0
0

= (2m + 3)(2m 3) 3(3m2 5m 3) + m(m3 m2 4m + 3)

= m4 m3 17m2 + 30m = m(m 2)(m2 + m 15)


et t =

m2 + m 15
.
(m 4)(m 6)

Si m = 0, le systme scrit

x + 2y + 3z = 1
x + y + z + t = 0 (E1 + E2 )

t = x y z
2x + y + 3t = 1
2x + y + 3t = 1
x 2y 3z = 1

3x + z + 2t = 0
x + y + z + t = 0(E3 + E4 )

x + 2y + z = 0

3y + 2z + t = 0
3y + 2z + t = 0

y=x+

4
z = x 2y

1
x 2y 3(x 2y) = 1
.

z = x

t = x y z

t = x + 1
4



1
1
1
Do lensemble de solutions : S =
x, x + , x , x +
, xR .
4
2
4
http ://www.maths-france.fr

c Jean-Louis Rouget, 2014. Tous droits rservs.





5
1
1
Si m = 2, on obtient pour ensemble de solutions :
x, x , x + ; x
, xR .
8
2
8
Si m = 4 ou m = 6, on voit en rsolvant que le systme est incompatible.


m
1
1

5) 1 1 m = m(2m) + (m + 1) + (m + 1) = 2(m2 + 1) 6= 0.
1 1 m
Le systme form des quations 1, 2 et 4 est donc de Cramer. Les formules de Cramer fournissent alors :
3m 1
2m2 m 1
, y = 3 m et z = 2
.
m2 + 1
m +1
La troisime quation fournit alors une condition ncessaire et suffisante de compatibilit :
x=

2m2 m 1
3m 1
+3m+m 2
= m
m2 + 1
m +1
m(2m2 m 1) + (3 m)(m2 + 1) + m(3m 1) = m(m2 + 1)
2m3 + 7m2 m + 3 = 0

Le systme est compatible si et seulement si m est lune des trois racines de lquation 2z3 + 7z2 z + 3 = 0.




a b c
1
1
1


1 2
1
a b c = Van(a, b, c) .
6) detS =
a b2 c2 =


abc
abc 2
abc
1
1
1
a b2 c2
Si a, b et c sont deux deux distincts, le systme est de Cramer. On obtient :
x=
puis, par symtrie des rles, y =

a(b m)(c m)
abc Van(m, b, c)
=
,
mbc Van(a, b, c)
m(b a)(c a)

b(a m)(c m)
c(a m)(b m)
et z =
.
m(a b)(c b)
m(a c)(b c)

Si a = b 6= c (ou a = c 6= b ou b = c 6= a), le systme scrit :

x+y=1z
ax + ay + cz = m

1 x + 1 y + 1z = 1
a
a
c
m

x+y=1z
a(1 z) + cz = m

1 (1 z) + 1 z = 1
a
c
m

x+y=1z

ma

z=

.
c a


1
1
1 1 ma

c a ca
m a

Le systme est compatible


si et seulementsi (m a)(m
c) = 0 ou encore (m = a ou m = c). Dans ce cas, lensemble


ma
mc
, xR .
x,
des solutions est :
x,
ac
ca
m
a
Si a = b = c, le systme scrit : x + y + z = 1 =
= . Le systme est compatible si et seulement si m = a = b = c et
a
m
dans ce cas lensemble des solutions est : {(x, y, 1 x y), (x, y) R2 }.
7)



(b + c)2
b2
c2


2
2
2


a
(a + c)
c
detS =

2
2
2

a
b
(a + b)


2


b2
c2

2 (b + c) 2
2
2
(L3 L3 L2 puis L2 L2 L1 )

0
= a (b + c) (a + c) b

2
2
2
2

0
b (a + c) (a + b) c





(b + c)2
2bc
b2
b2
c2



2
2

2c
a+cb
= (a + b + c) a b c a + c b
0
= (a + b + c)
2(b c) b a c

0
bac a+bc

= 2(a + b + c)2 (c2 (c(b a c) + (b c)(a + c b)) + (a + b c)(bc(a + c b) + b2 c))

c2
0
a+bc

= 2(a + b + c)2 (c2 b(a b + c) + (a + b c)bc(a + c))


= 2bc(a + b + c)2 (a2 + ab + ac) = 2abc(a + b + c)3 .
http ://www.maths-france.fr

c Jean-Louis Rouget, 2014. Tous droits rservs.


Si abc(a + b + c) 6= 0, le systme est de Cramer et on obtient aprs calcul :


x=

(a b c)(a + b c)
(a b + c)(a b c)
(a b + c)(a + b c)
, y=
et z =
.
2abc(a + b + c)
2abc(a + b + c)
2abc(a + b + c)

Si a = 0 (ou b = 0 ou c = 0), le systme scrit :

(b + c)2 x + b2 y + c2 z = 1
c2 (y + z) = 1
.
2
b (y + z) = 1

Donc,
Si ((a = 0 et b2 6= c2 ) ou (b = 0 et a2 6= c2 ) ou (c = 0 et a2 6= b2 )), le systme na pas de solution.
y
Si a = 0 et b = c 6= 0, lensemble des solutions est {(0, y, 2 ), y R} (rsultats analogues pour les cas
b
(b = 0 et a = c 6= 0) et (c = 0 et a = b 6= 0)).
Si a = b = c = 0, il ny a pas de solution.
y
Si a = 0 et c = b 6= 0, lensemble des solutions est {(x, y 2 ), (x, y) R2 } (rsultats analogues pour
b
(b = 0 et c = a 6= 0) et (c = 0 et b = a 6= 0).
2
2
2
Si abc 6= 0 et a + b +c = 0, le systme
quivaut lquation a x + b y + c z = 1. Lensemble des solutions est

2
2
1a xb y
, (x, y) R2 .
x, y,
c2
8)

a b

detS = c a
b c

c
b
a




1


= (a + b + c) 1



1

b c
a b
c a



1

b
c


= (a + b + c) 0 a b b c



0 cb ac

= (a + b + c)((a b)(a c) + (b c)2 ) = (a + b + c)(a2 + b2 + c2 ab ac bc)

= (a + b + c)(a + jb + j2 c)(a + j2 b + jc)


Si detS 6= 0, les formules de Cramer fournissent :


p b c


xdetS = q a b = p(a2 bc) + q(c2 ab) + r(b2 ac) . . .
r c a

Exercice no 2


1 2 3


0 1 2 = 2 + 2(7) = 12 6= 0 et le systme est de Cramer en x1 , x2 et x4 . On note aussi que le systme est


2 1 0
homogne de rang 3 et donc que lensemble des solutions F est un sous-espace vectoriel de R5 de dimension 5 3 = 2.

x1 + 2x2 + 3x4 = x3 x5
x1 + 2x2 x3 + 3x4 + x5 = 0
x2 = 2x1 + 5x3 + 4x5
1
x2 + x3 2x4 + 2x5 = 0
x2 2x4 = x3 2x5

x4 = ((2x1 + 5x3 + 4x5 ) + x3 + 2x5 )

2x1 + x2 5x3 4x5 = 0


2x1 + x2 = 5x3 + 4x5
x1 + 2x2 + 3x4 = x3 x5

x2 = 2x1 + 5x3 + 4x5


x4 = x1 + 3x3 + 3x5

x1 + 2(2x1 + 5x3 + 4x5 ) + 3(x1 + 3x3 + 3x5 ) = x3 x5

x1 = 3x3 + 3x5

x2 = x3 2x5

x4 = 0



Ainsi, F = (3x3 + 3x5 , x3 2x5 , x3 , 0, x5 ) , (x3 , x5 ) R2 = Vect (e1 , e2 ) o e1 = (3, 1, 1, 0, 0) et e2 = (3, 2, 0, 0, 1)
et, puisque dimF = 2, une base de F est (e1 , e2 ).
Exercice no 3
Si zk est laffixe complexe de Mk et ak est laffixe complexe de Ak , le problme pos quivaut au systme :
k J1, n 1K, zk + zk+1 = 2ak et zn + z1 = 2an .
http ://www.maths-france.fr

c Jean-Louis Rouget, 2014. Tous droits rservs.


Le dterminant de ce systme vaut :














1
..
.

0
..
.

..

0
1

0
..
.

...
..
.
..
.
..
..
.
.
... 0








n1
+ (1)n+1 1n1 (en dveloppant suivant la premire colonne)
0 = 1 1

1
1
0
..
.

= 1 (1)n .

Si n est impair, detS = 2 6= 0 et le systme admet une et une seule solution.


On obtient z2 = 2a1 z1 , z3 = 2a2 2a1 + z1 ,..., zn = 2an1 2an2 + ... + 2a2 2a1 + z1 et enfin :
2an1 2an2 + ... + 2a2 2a1 + z1 + z1 = 2an ,
et donc z1 = a1 a2 + ... an1 + an puis z2 = a1 + a2 a3 + ... + an1 an puis z3 = a1 + a2 + a3 a4 ... + an ...
puis zn = a1 + a2 a3 + ... + an1 + an .
Si n est pair, detS = 0 mais le mineur form des n 1 premires lignes et n 1 dernires colonnes est non nul. Donc, le
systme est de rang n 1, les n 1 premires quations et n 1 dernires inconnues peuvent tre choisies pour quations
et inconnues principales.
On rsout les n 1 premires quations constituant un systme de Cramer en z2 ,...,zn . On obtient
z2 = 2a1 z1 , z3 = 2a2 2a1 + z1 , ..., zn = 2an1 2an2 + ... 2a2 + 2a1 z1 .
La dernire quation fournit alors une condition ncessaire et suffisante de compatibilit :
2an1 2an2 + ... 2a2 + 2a1 z1 + z1 = 2an a1 + a3 ... = a2 + a4 + ...

Cette dernire condition se traduit gomtriquement par le fait que les systmes de points (A1 , A3 , ...) et (A2 , A4 , ...) ont
mme isobarycentre.
En rsum, si n est pair et si les systmes de points (A1 , A3 , ...) et (A2 , A4 , ...) nont pas mme isobarycentre, le problme
na pas de solutions.
Si n est pair et si les systmes de points (A1 , A3 , ...) et (A2 , A4 , ...) ont mme isobarycentre, le problme a une infinit de
solutions : M1 est un point quelconque puis on construit les symtriques successifs par rapport aux points A1 , A2 ...
Exercice no 4
Soit Dn le dterminant du systme pour n > 3. En dveloppant ce dterminant suivant sa premire colonne, on obtient
la relation de rcurrence :
n > 5, Dn = Dn1 Dn2 ,
ce qui fournit aisment par rcurrence, en tenant compte de D3 = D4 = 1 :
k > 1, D3k = D3k+1 = (1)k et D3k+2 = 0.
Pour n lment de 3N (1 + 3N ), le systme est de Cramer et homogne et admet donc une et une seule solution
savoir la solution nulle.
Pour n = 3k + 2, puisque Dn = 0 mais que le mineur de format n 1 constitu des n 1 premires lignes et colonnes est
Dn1 et est donc non nul, le systme est homogne de rang n 1 et lensemble des solutions est un sous-espace vectoriel
de Rn de dimension 1. On trouve aisment S = {(1, 1, 0, 1, 1, 0..., 1, 1), ; R}.
Exercice no 5
(1) (2). Montrons par rcurrence sur n > 1 que : ((a1 , ..., an ) En / (det(fi (aj ))16i,j6n = 0) ((f1 , ..., fn ) lie).
Pour n = 1,

(a1 E/ det(fi (aj ))16i,j61 = 0) (a1 / f1 (a1 ) = 0) (f1 = 0) (f1 ) lie.

Soit n > 1. Supposons que ((a1 , ..., an ) En / det(fi (aj ))16i,j6n = 0) (f1 , ..., fn ) lie.
Soient f1 ,..., fn+1 n + 1 fonctions telles que (a1 , ..., an+1 ) En+1 / det(fi (aj ))16i,j6n+1 = 0.
Si (f1 , ..., fn ) est lie alors (f1 , ..., fn+1 ) est lie en tant que sur-famille dune famille lie. Si (f1 , ..., fn ) est libre, par
hypothse de rcurrence, il existe a1 ,...,an n lments de E tels que det(fi (aj ))16i,j6n 6= 0. Mais, par hypothse, on a :
http ://www.maths-france.fr

c Jean-Louis Rouget, 2014. Tous droits rservs.


x E, det(fi (a1 ), ..., fi (an ), fi (x))16i6n+1 = 0.


En dveloppant ce dterminant suivant sa dernire colonne, on obtient une galit du type

n+1
X

i fi (x) = 0 o les i sont

i=1

indpendants de x ou encore une galit du type

n+1
X

i fi = 0 avec n+1 = det(fi (aj ))16i,j6n 6= 0 ce qui montre encore

i=1

que (f1 , ..., fn+1 ) est lie.

(2) (1). On suppose que (a1 , ..., an ) En / det(fi (aj ))16i,j6n 6= 0). Montrons que (f1 , ..., fn ) est libre.
n
n
X
X
n
Soit (1 , ..., n ) C tel que
i fi = 0. En particulier : j J1, nK,
i fi (aj ) = 0. Les n galits prcdentes
i=1

i=1

fournissent un systme dquations linaires en les i n inconnues, n quations, de dterminant non nul et homogne
ou encore un systme de Cramer homogne dont on sait quil admet pour unique solution (1 , ..., n ) = (0, ..., 0). On a
montr que (f1 , ..., fn ) est libre.
Exercice no 6

Soit An la matrice de lnonc. En dveloppant det (An ) suivant sa premire colonne puis en dveloppant le dterminant
de format n 1 obtenu suivant sa premire ligne, on obtient det (An ) = det (An2 ) pour n > 3.
Par suite, pour p > 1, det (A2p ) = (1)p1 det (A2 ) = (1)p 6= 0 et pour p > 1, A2p est inversible.
On a aussi, pour p > 1, det (A2p+1 ) = (1)p1 det (A3 ) = 0 et, pour p > 1, A2p+1 nest pas inversible. Finalement, An
est inversible si et seulement si n est pair.
Dornavant, on pose n = 2p (p > 1). Pour X = (xi )16i6n et Y = (yi )16i6n vecteurs colonnes donns, on a :

x2 = y1
AX = Y
i J2, 2p 1K, xi1 + xi+1 = yi .

x2p1 = y2p

Ce systme se rsout en x2 = y1 puis, par rcurrence, pour k 6 p, x2k = y2k1 y2k3 + ... + (1)k1 y1 et aussi
x2p1 = y2p , puis, par rcurrence, pour k 6 p, x2k1 = y2k y2k+2 + ... + (1)pk y2p . Do linverse de A quand n = 8
par exemple :

0 1 0 1 0
1 0 1
1 0 0
0
0
0 0 0

0 0 0
1
0
1
0 1

1 0 1
0
0
0 0 0
1

.
A =
0
0
1 0 1
0 0 0

1 0 1 0
1
0 0 0

0 0 0
0
0
0 0 1
1 0 1
0 1 0 1 0
Exercice no 7
Soit (x1 , ..., xn ) Rn et F =

n
X

k=1

F=

xk
. La fraction rationnelle F scrit, aprs rduction au mme dnominateur :
X + bk

n
Y
P
o Q =
(X + bk ) et P est un polynme de degr infrieur ou gal n 1.
Q
k=1

Maintenant,
(x1 , ..., xn ) solution de (S) k J1, nK, F(ak ) = 1 k J1, nK, (Q P)(ak ) = 0.

Par suite, puisque les ak sont deux deux distincts, Q P est divisible par

n
Y

(X ak ). Mais, Q est unitaire de degr n

k=1

et P est de degr infrieur ou gal n 1, et donc Q P est unitaire de degr n ce qui montre que Q P =

(X ak )

k=1

ou encore que
P=

n
Y

(X + bk )

k=1

http ://www.maths-france.fr

n
Y

n
Y

(X ak ).

k=1

c Jean-Louis Rouget, 2014. Tous droits rservs.


Rciproquement, si F =

n
Y

(X + bk )

k=1

n
Y

(X ak )

k=1
n
Y

, alors k {1, ..., n}, F(ak ) = 1.

(X + bk )

k=1

En rsum,

(x1 , ..., xn ) solution de (S)

n
X

k=1

xk
=
X + bk

n
Y

(X + bk )

n
Y

(X ak )

k=1

k=1
n
Y

(X + bk )

k=1

i J1, nK, xi = lim (x + bi )


xbi

n
Y

n
Y

(x + bk )

n
Y

(x ak )

k=1

k=1
n
Y

(x + bk )

k=1

(bi + ak )

k=1

i J1, nK, xi = Y

(bi bk )

k6=i

Exercice no 8
1) Soit (k )16k6n Cn .
n
X

k=1

k fak = 0 p J0, n 1K,


p J0, n 1K,

n
X

k=1
n
X

k f(p)
ak

= 0 p J0, n 1K, x R,

n
X

ak x
k a p
=0
ke

k=1

k a p
k (galits obtenues pour x = 0).

k=1

Les n dernires galits crites constituent un systme (S) de n quations linaires n inconnues 1 , . . . , n . Le dterminant
de ce systme est Van (a1 , . . . , an ). Ce dterminant nest pas nul car les ak sont deux deux distincts. Par suite, (S) est
un systme de Cramer homogne. (S) admet donc lunique solution (1 , . . . , n ) = (0, . . . , 0). On a montr que la famille
(fak )16k6n est libre.
2) Soit (k )16k6p Cp .
p
X

k=1

k (qn
k )nN = 0 n J0, p 1K,

n
X

k qn
k = 0.

k=1

Les n dernires galits crites constituent un systme (S) de p quations linaires p inconnues 1 , . . . , p . Le dterminant
de ce systme est Van (q1 , . . . , qp ). Ce dterminant nest pas nul car les qk sont deux deux distincts. Par suite, (S) est
un systme
 de Cramer homogne. (S) admet donc lunique solution (1 , . . . , p ) = (0, . . . , 0). On a montr que la famille
(qn
)
k nN 16k6n est libre.

http ://www.maths-france.fr

c Jean-Louis Rouget, 2014. Tous droits rservs.


Planche no 36. Produit scalaire


* trs facile ** facile *** difficult moyenne **** difficile
I : Incontournable T : pour travailler et mmoriser le cours
Exercice no 1 : (***)
Pour A = (ai,j )16i,j6n Mn (R), N(A) = Tr(t AA). Montrer que N est une norme vrifiant de plus N(AB) 6 N(A)N(B)
pour toutes matrices carres A et B. N est-elle associe un produit scalaire ?
Exercice no 2 : (***)
Soit E un R espace vectoriel de dimension finie. Soit k k une
 norme sur E vrifiant lidentit du paralllogramme, cest-dire : (x, y) E2 , kx + yk2 + kx yk2 = 2 kxk2 + kyk2 . On se propose de dmontrer que k k est associe un produit
scalaire.

1
kx + yk2 kx yk2 .
On dfinit sur E2 une application f par : (x, y) E2 , f(x, y) =
4
1) Montrer que pour tout (x, y, z) de E3 , on a : f(x + z, y) + f(x z, y) = 2f(x, y).
2) Montrer que pour tout (x, y) de E2 , on a : f(2x, y) = 2f(x, y).
3) Montrer que pour tout (x, y) de E2 et tout rationnel r, on a : f(rx, y) = rf(x, y).

On admettra que pour tout rel et tout (x, y) de E2 on a : f(x, y) = f(x, y) (ce rsultat provient de la continuit de f).
4) Montrer que pour tout (u, v, w) de E3 , f(u, w) + f(v, w) = f(u + v, w).
5) Montrer que f est bilinaire.
6) Montrer que k k est une norme euclidienne.
Exercice no 3 : (**IT)
Dans R4 muni du produit scalaire usuel, on pose : V1 = (1, 2, 1, 1) et V2 = (0, 3, 1, 1).
On pose F = Vect(V1 , V2 ). Dterminer une base orthonormale de F et un systme dquations de F .
Exercice no 4 : (***)
1) Soit (E, | ) un espace euclidien.
a) Soit u E. Montrer que x 7 u|x est une forme linaire sur E.
b) Soit une forme linaire sur E. Montrer quil existe un vecteur u de E et un seul tel que x E, (x) = u|x.
2) a) Existe-t-il A lment de Rn [X] tel que P Rn [X], P|A = P(0) ?
b) Existe-t-il A lment de R[X] tel que P R[X], P|A = P(0) ?
Exercice no 5 : (***I) (Matrices et dterminants de Gram)
Soit E un espace vectoriel euclidien de dimension p sur R (p > 2).
Pour (x1 , ..., xn ) donn dans En , on pose G(x1 , ..., xn ) = (xi |xj )16i,j6n (matrice de Gram) et (x1 , ..., xn ) = det(G(x1 , ..., xn ))
(dterminant de Gram).
1) Montrer que rg(G(x1 , ..., xn )) = rg(x1 , ..., xn ).
2) Montrer que (x1 , ..., xn ) est lie si et seulement si (x1 , ..., xn ) = 0 et que (x1 , ..., xn ) est libre si et seulement si
(x1 , ..., xn ) > 0.
3) On suppose que (x1 , ..., xn ) est libre dans E (et donc n 6 p). On pose F = Vect(x1 , ..., xn ).
Pour x E, on note pF (x) la projection orthogonale
de x sur F puis dF (x) la distance de x F (cest--dire
s
(x, x1 , ..., xn )
dF (x) = kx pF (x)k). Montrer que dF (x) =
.
(x1 , ..., xn )
Exercice no 6 : (**I)
Matrice de la projection orthogonale sur la droite dquation 3x = 6y = 2z dans la base canonique orthonorme de R3
ainsi que de la symtrie orthogonale par rapport cette mme droite.
De manire gnrale, matrice de la projection orthogonale sur le vecteur unitaire u = (a, b, c) et de la projection orthogonale sur le plan dquation ax + by + cz = 0 dans la base canonique orthonorme de R3 .
Exercice no 7 : (***I) (Ingalit de Hadamard)
Soit B une base orthonorme de E, espace euclidien de dimension n.
Montrer que : (x1 , ..., xn ) En , |detB (x1 , ..., xn )| 6 kx1 k . . . kxn k en prcisant les cas dgalit.

http ://www.maths-france.fr

c Jean-Louis Rouget, 2014. Tous droits rservs.


Exercice no 8 : (***I)
Existence, unicit et calcul de a et b tels que

Z1

(x4 ax b)2 dx soit minimum (trouver deux dmonstrations, une dans

la mentalit du lyce et une dans la mentalit maths sup).


Exercice no 9 : (***)
Soit (e1 , ..., en ) une base quelconque de E euclidien. Soient a1 ,..., an n rels donns.
Montrer quil existe un unique vecteur x tel que i J1, nK, x|ei = ai .
Exercice no 10 : (****I)
Soit E un espace vectoriel euclidien de dimension n > 1.
Une famille de p vecteurs (x1 , ..., xp ) est dite obtusangle si et seulement si pour tout (i, j) tel que i 6= j, xi |xj < 0. Montrer
que lon a ncessairement p 6 n + 1.
Exercice no 11 : (****)
Z1
Soit P R3 [X] tel que
P2 (t) dt = 1. Montrer que sup{|P(x)|, |x| 6 1} = 2. Cas dgalit ?
1

Exercice n 12 : (**)
o

Soit f continue strictement positive sur [0, 1]. Pour n N, on pose In =


Montrer que la suite un =

Z1

fn (t) dt.

In+1
est dfinie et croissante.
In

Exercice no 13 : (****I)
Sur E = Rn [X], on pose P|Q =

Z1

P(t)Q(t) dt.

1) Montrer que (E, | ) est un espace euclidien.


2) Pour p entier naturel compris entre 0 et n, on pose Lp = ((X2 1)p )(p) . Montrer que
lorthonormalise de Schmidt de la base canonique de E. Dterminer kLp k.

Lp
kLp k

est

06p6n

Exercice no 14 : (**I)
Soit f un automorphisme orthogonal dun espace euclidien (E,
 | ). Soit F un sous-espace vectoriel de E, stable par f
(cest--dire vrifiant f(F) F). Montrer que f(F) = F et f F = F .
Exercice no 15 : (***I)

Soit f un endomorphisme dun espace euclidien (E, | ) de dimension n > 2 vrifiant


(x, y) E2 , (x|y = 0 f(x)|f(y) = 0)

(f est donc un endomorphisme qui conserve lorthogonalit). Montrer quil existe R+ et g O(E) tels que f = g (f
est alors la compose dune homothtie vectorielle et dune isomtrie vectorielle et sappelle une similitude vectorielle).
Exercice no 16 : (***I)
Soit (E, | ) un espace euclidien et B = (ei )16i6n une base orthonorme de E.
1) Soit p une projection. Montrer que : p est une projection orthogonale x E, kp(x)k 6 kxk.

2) Soit p un endomorphisme de E. Soit P = MatB (p).


Montrer que : p est une projection orthogonale P2 = P et t P = P
(pour , vrifier dabord que (x, y) E2 , (p(x)|y) = (x|p(y))).

3) Soit s une symtrie dun espace euclidien (E, | ). Montrer que s est une symtrie orthogonale x E, ks(x)k = kxk.

4) Soit s un endomorphisme de E. Soit S = MatB (s).


Montrer que : s est une symtrie orthogonale S2 = In et t S = S.

http ://www.maths-france.fr

c Jean-Louis Rouget, 2014. Tous droits rservs.


Planche no 36. Produit scalaire : corrig


Exercice no 1
Montrons que : (A, B) 7 Tr (t A B) est un produit scalaire sur Mn (R).
- est symtrique. En effet, pour (A, B) (Mn (R))2 ,
t

(A, B) = Tr

t t


A B = Tr


A B = Tr


B A = (B, A).

- est bilinaire par linarit de la trace et de la transposition.


- Si A = (ai,j )16i,j6n Mn (R) \ {0}, alors

n
n
X
X
X

(A, A) =
ai,j ai,j =
a2i,j > 0
i=1

j=1

i,j

car au moins un des rels de cette somme est strictement positif. est donc dfinie, positive.
Finalement, est un produit scalaire sur Mn (R). N nest autre que la norme associe au produit scalaire (et en
particulier, N est une norme).
Soit (A, B) (Mn (R))2 .
2

N(AB) =

X
i,j

k=1

n
X

i,j

n
X

ai,k bk,j
a2i,k

k=1

a2i,k b2l,j =

i,j,k,l

!2
n
X
l=1

X
i,k

et donc, (A, B) (Mn (R))2 , N(AB) 6 N(A)N(B).

b2l,j

(daprs lingalit de Cauchy-Schwarz)

!
X
a2i,k
b2l,j = N(A)2 N(B)2 ,
l,j

Exercice no 2
1) Soit (x, y, z) E3 .

1
kx + z + yk2 + kx z + yk2 kx + z yk2 kx z yk2
4


1
2 kx + yk2 + kzk2 2 kx yk2 + kzk2 (puisque N vrifie lidentit du paralllogramme)
=
4

1
kx + yk2 kx yk2 = 2f(x, y).
=
2

f(x + z, y) + f(x z, y) =

2) Soit (x, y) E2 .

2f(x, y) = f(x + x, y) + f(x x, y) = f(2x, y) + f(0, y)


mais f(0, y) =

1
(||y||2 || y||2 ) = 0 (dfinition dune norme) et donc f(2x, y) = 2f(x, y).
4

3) Soit (x, y) E2 . Montrons par rcurrence que n N, f(nx, y) = nf(x, y).


Lgalit est vraie pour n = 0 et n = 1.
Soit n > 0. Si lgalit est vraie pour n et n + 1 alors daprs 1),
f((n + 2)x, y) + f(nx, y) = f((n + 1)x + x, y) + f((n + 1)x x, y) = 2f((n + 1)x, y),
et donc, par hypothse de rcurrence,
f((n + 2)x, y) = 2f((n + 1)x, y) f(nx, y) = 2(n + 1)f(x, y) nf(x, y) = (n + 2)f(x, y).
On a montr par rcurrence que n N, f(nx, y) = nf(x, y).
http ://www.maths-france.fr

c Jean-Louis Rouget, 2014. Tous droits rservs.




1
1
1
1
Soit n N , f(x, y) = f n x, y = nf( x, y) et donc f( x, y) = f(x, y).
n
n
n
n
p
Puis, si r = , p N, q N ,
q
1
1
f(rx, y) = f(px, y) = p f(x, y) = rf(x, y)
q
q

et pour tout rationnel positif r, f(rx, y) = rf(x, y).


Enfin, si r 6 0, f(rx, y) + f(rx, y) = 2f(0, y) = 0 (daprs 1)) et donc= f(rx, y) = f(rx, y) = rf(x, y).
4) On pose x =

1
1
(u + v) et y = (u v).
2
2

f(u, w) + f(v, w) = f(x + y, w) + f(x y, w) = 2f(x, w) = 2f


1
(u + v), w = f(u + v, w).
2

5) f est symtrique (dfinition dune norme) et linaire par rapport sa premire variable. Donc f est bilinaire.
1
1
(kx + xk2 + kx xk2 ) = ||2x||2 = ||x||2 (dfinition dune
4
4
norme) ce qui montre tout la fois que f est dfinie positive et donc un produit scalaire, et que k k est la norme associe.
k k est donc une norme euclidienne.
6) f est une forme bilinaire symtrique. Pour x E, f(x, x) =

Exercice no 3
La famille (V1 , V2 ) est libre et donc est une base de F. Son orthonormalise (e1 , e2 ) est une base orthonorme de F.

kV1 k = 1 + 4 + 1 + 1 = 7 et
1
1
e1 = V1 = (1, 2, 1, 1).
7
7
1
1
4
4
(V2 |e1 ) = (0 + 6 1 1) = puis V2 (V2 |e1 )e1 = (0, 3, 1, 1) (1, 2, 1, 1) = (4, 13, 11, 11) puis
7
7
7
7
1
(4, 13, 11, 11).
e2 =
427
1
1
Une base orthonorme de F est (e1 , e2 ) o e1 = (1, 2, 1, 1) et e2 =
(4, 13, 11, 11).
7
427
Soit u = (x, y, z, t) R4 .

u F u (Vect (V1 , V2 )) u (V1 , V2 )

Exercice no 4

u|V1 = 0

u|V2 = 0

x + 2y z + t = 0
.
3y + z t = 0

1) a) Soit u E. Lapplication x 7 u|x est une forme linaire sur E par linarit du produit scalaire par rapport sa
deuxime variable.
b) Soit une forme linaire sur E.

Existence. Soit B = (e1 , . . . , en ) une base orthonorme de E. Pour i J1, nK, posons ai = (ei ) puis u =
Soit x =

n
X
i=1

n
X

ai ei .

i=1

xi ei E.
(x) =

n
X
i=1

xi (ei ) =

n
X

a i xi

i=1

= u|x (car la base B est orthonorme.)


Ainsi, il existe un vecteur u E (indpendant de x E) tel que x E, (x) = u|x.
Unicit. Soit v E tel que x E, (x) = v|x. Par suite, x E, u|x = v|x puis, x E, (u v)|x = 0. Mais alors
u v E = {0} puis u = v. Ceci montre lunicit du vecteur u.
Z1
2) a) Lapplication (P, Q) 7 P|Q =
P(t)Q(t) dt est un produit scalaire sur lespace E = Rn [X] qui est de dimension
0

finie sur R. Lapplication : P 7 P(0) est une forme linaire sur E. Daprs la question 1), il existe un lment A de
http ://www.maths-france.fr

c Jean-Louis Rouget, 2014. Tous droits rservs.


R [X] tel que pour tout P Rn [X], (P) = A|P ou encore il existe un lment A de Rn [X] tel que pour tout P Rn [X],
Z 1n
A(t)P(t) dt = P(0).
0

b) Soit A un ventuel polynme solution cest dire tel que P R[X],

Z1

P(t)A(t) dt = P(0).
Z1
Le choix de P = 1 montre que A 6= 0. Le choix P = XA fournit : 0 = P(0) = tA2 (t) dt.
0

Mais alors, t [0, 1], tA2 (t) = 0 (fonction continue positive dintgrale nulle) puis t ]0, 1], A(t) = 0 et donc A = 0
(polynme ayant une infinit de racines deux deux distinctes). Ceci est une contradiction et donc il nexiste pas de
Z1
A(t)P(t) dt = P(0).
polynme A tel que pour tout P R[X],
0

Exercice no 5

1) Soit B une base orthonorme de E et M = MatB (x1 , ..., xn ) (M est une matrice de format (p, n)).
Puisque B est orthonorme, le produit scalaire usuel des colonnes Ci et Cj est encore xi |xj .
Donc, (i, j) J1, nK2 , t Ci Cj = xi |xj ou encore G = t M M.

Il sagit alors de montrer que rg(M) = rg(t MM). Ceci provient du fait que M et t M M ont mme noyau. En effet, pour
X Mn,1 (R),
X KerM MX = 0 t M MX = 0 X Ker(t MM)

et

X Ker(t MM) t MMX = 0 t Xt MMX = 0 t (MX)MX = 0 kMXk2 = 0 MX = 0


X KerM.

On a montr que Ker(t MM) = Ker(M). Mais alors, daprs le thorme du rang, rg(t MM) = rg(M).

2) Si la famille (x1 , ..., xn ) est lie, rg(G) = rg(x1 , ..., xn ) < n, et donc, puisque G est une matrice carre de format n,
(x1 , ..., xn ) = det(G) = 0.
Si la famille (x1 , ..., xn ) est libre, (x1 , ..., xn ) engendre un espace F de dimension n. Soient B une base orthonorme de F
et M la matrice de la famille (x1 , ..., xn ) dans B. Daprs 1), on a G = t MM et dautre part, M est une matrice carre,
inversible car matrice dune base de F dans une base de F. Par suite,
(x1 , ..., xn ) = det(t MM) = det(t M)det(M) = (detM)2 > 0.
3) On crit x = x pF (x) + pF (x).


kxk2
x|x1


x|x2

=
..
.
x|xn

La premire colonne de (x, x1 , ..., xn ) scrit



kx pF (x) + pF (x)k2
kx pF (x)k2

(x pF (x) + pF (x)) |x1


0

(x pF (x) + pF (x)) |x2


0
=


..
..

.
.
(x pF (x) + pF (x)) |xn

kpF (x)k2
pF (x)|x1
pF (x)|x2
..
.
pF (x)|xn

(en 1re ligne, cest le thorme de Pythagore et dans les suivantes, x pF (x) F ). Par linarit par rapport la
premire colonne, (x, x1 , ..., xn ) est somme de deux dterminants. Le deuxime est (pF (x), x1 , ..., xn ) et est nul car la
famille (pF (x), x1 , ..., xn ) est lie. On dveloppe le premier suivant sa premire colonne et on obtient :
(x, x1 , ..., xn ) = ||x pF (x)||2 (x1 , ..., xn ),
ce qui fournit la formule dsire.
Exercice no 6

Un vecteur engendrant D est


u = (2, 1, 3). Pour
v = (x, y, z) R3 ,

 (x, y, z)|(2, 1, 3)
2x + y + 3z

(2, 1, 3) =
(2, 1, 3)
pD
v =
||(2, 1, 3)||2
14


4x + 2y + 6z 2x + y + 3z 6x + 3y + 9z
.
=
,
,
14
14
14

http ://www.maths-france.fr

c Jean-Louis Rouget, 2014. Tous droits rservs.


4 2 6
3 2 6
1
1
2 1 3 puis MatB (s) = 2P I = 2 6 3 .
On en dduit que MatB (p) = P =
14
7
6 3 9
6
3 2
Plus gralement,
la
matrice
de
la
projection
orthogonale
sur
le
vecteur
unitaire
(a,
b,
c)
dans
la base canonique orthonorme
2

a
ab ac
est P = ab b2 bc et la matrice de la projection orthogonale sur le plan ax + by + cz = 0 dans la base canonique
ac bc c2

1 a2 ab
ac
orthonorme est I P = ab 1 b2 bc .
ac
bc 1 c2
Exercice no 7

Si la famille (xi )16i6n est une famille lie, lingalit est claire et de plus, on a lgalit si et seulement si lun des vecteurs
est nuls.
Si la famille (xi )16i6n est une famille libre et donc une base de E, considrons B = (e1 , ..., en ) son orthonormalise de
Schmidt. On a
|detB (xi )16i6n | = |detB (xi )16i6n detB B | = |detB (xi )16i6n | ,

car detB B est le dterminant dune base orthonorme dans une autre et vaut donc 1 ou 1.

Maintenant, la matrice de la famille (xi )16i6n dans B est triangulaire suprieure et son dterminant est le produit des
coefficients diagonaux savoir les nombres xi |ei (puisque B est orthonorme). Donc :


n
n
n
Y
Y
Y


|detB (xi )16i6n | = |detB (xi )16i6n | = (xi |ei ) 6
kxi k,
kxi k kei k =


i=1

i=1

i=1

daprs lingalit de Cauchy-Schwarz. De plus, on a lgalit si et seulement si, pour tout i, |xi |ei | = kxi k kei k ou
encore si et seulement si, pour tout i, xi est colinaire ei ou enfin si et seulement si la famille (xi )16i6n est orthogonale.
Exercice no 8
1re solution.
Z1


2
1
2
1 2
1
1 1 2
a + a(3b 1) + b2 b +
+ a + b2 a b + ab =
9 3
3
5
3
5
9

2
1
2
1
1
1
a + (3b 1) (3b 1)2 + b2 b +
=
3
2
12
5
9

2
1
1
1
1
1
=
a + (3b 1) + b2 + b +
3
2
4
10
36

2
1
1
4
1
1
4
2
= (a + (3b 1)) +
b+
+
>
,
3
2
4
5
225
225

(x4 ax b)2 dx =

1
1
4
1
avec galit si et seulement si a + (3b 1) = b + = 0 ou encore b = et a = .
2
5
5
5
Z1
1
4
4
4
2
.
(x ax b) dx est minimum pour a = et b = et ce minimum vaut
5
5
225
0

2me solution.
Z1
Z1
(P, Q) 7 P(t)Q(t) dt est un produit scalaire sur R4 [X] et (x4 ax b)2 dx est, pour ce produit scalaire, le carr de
0

4
la distance du polynme
Z X au polynme de degr infrieur
 ou gal 1, aX + b.
1
On doit calculer Inf
(x4 ax b)2 dx, (a, b) R2 qui est le carr de la distance de X4 F = R1 [X]. On sait que
0

cette borne infrieure est un minimum, atteint une et une seule fois quand aX + b est la projection orthogonale de X4
sur F.
Trouvons une base orthonormale de F. Lorthonormalise (P0 , P1 ) de (1, X) convient.
Z1
Z1
1
2
||1|| = 1 dt = 1 et P0 = 1. Puis X (X|P0 )P0 = X t dt = X , et comme
2
0
0
http ://www.maths-france.fr

c Jean-Louis Rouget, 2014. Tous droits rservs.


kX (X|P0 )P0 k2 =



1
= 3(2X 1).
on a P1 = 2 3 X
2

2
Z1 
1
1
1 1 1
t
,
dt = + =
2
3 2 4
12
0

Z1

1
La projection orthogonale de X sur F est alors (X |P0 )P0 + (X |P1 )P1 avec (X |P0 ) = t4 dt = et
5
0



Z

1 4
1 1
2 3
4
4
=

. Donc, la projection orthogonale de X sur F est


(X |P1 ) = 3 t (2t 1) dt = 3
3 5
15
0

1 2 3
1
3(2X 1) = (4X 1).
+
5
15
5
4

2
Z1 
4
1
4
.
dt = ... =
t (4t 1)
Le minimum cherch est alors
5
225
0

Exercice no 9

. est clairement linaire et Ker est (e1 , ..., en ) = E = {0}.


E
Rn
x 7 (x|e1 , ..., x|en )
Comme E et Rn ont mmes dimensions finies, est un isomorphisme despaces vectoriels. En particulier, pour tout n-uplet
(a1 , ..., an ) de rels, il existe un unique vecteur x tel que i J1, nK, x|ei = ai .

Soit :

Exercice no 10
1re solution. Montrons par rcurrence sur n = dimE que, si (xi )16i6p est obtusangle, p 6 n + 1.
Pour n = 1, une famille obtusangle ne peut contenir au moins trois vecteurs car si elle contient les vecteurs x1 et x2
verifiant x1 |x2 < 0, un vecteur x3 quelconque est soit nul (auquel cas x3 |x1 = 0), soit de mme sens que x1 (auquel cas
x1 |x3 > 0) soit de mme sens que x2 (auquel cas x2 |x3 > 0). Donc p 6 2.
Soit n > 1. Supposons que toute famille obtusangle dun espace de dimension n a un cardinal infrieur ou gal n + 1.
Soit (xi )16i6p une famille obtusangle dun espace E de dimension n + 1. Si p = 1, il ny a plus rien dire. Supposons
p > 2. xp nest pas nul et H = x
p est un hyperplan de E et donc est de dimension n.
(xi |xp )
Pour 1 6 i 6 p 1, notons yi = xi
xp le projet orthogonal de xi sur H.
kxp k2
Vrifions que la famille (yi )16i6p1 est une famille obtusangle de H. Soit (i, j) J1, p 1K tel que i 6= j.
yi |yj = xi |xj

(xi |xp )(xj |xp ) (xj |xp )(xi |xp ) (xi |xp )(xj |xp )(xp |xp )
(xi |xp )(xj |xp )

+
= xi |xj
< 0.
||xp ||2
||xp ||2
||xp ||4
||xp ||2

Mais alors, par hypothse de rcurrence, p 1 6 1 + dimH = n + 1 et donc p 6 n + 2.


2me solution. Montrons que si la famille (xi )16i6p est obtusangle, la famille (xi )16i6p1 est libre. Supposons par
p1
X
i xi = 0 ().
labsurde, quil existe une famille de scalaires (i )16i6p1 non tous nuls tels que
i=1

Quite multiplier les deux membres de () par 1, on peut supposer quil existe au moins un rel i > 0. Soit I lensemble
des indices i tels que i > 0 et J lensemble des indices i tels que i 6 0 (ventuellement J est vide). I et J sont disjoints.
X
X
(*) scrit
i x i =
i xi (si J est vide, le second membre est nul). On a
iI

iJ


2
X



i x i =
06


iI


2
X

X


Donc, |
i xi = 0 puis
i xi = 0.


iI

X
iI

i x i |

X
iJ

i x i

i (j )xi |xj 6 0.

(i,j)IJ

iI

X
X
Mais, en faisant le produit scalaire avec xp , on obtient (
i xi )|xp =
i (xi .xp ) < 0 ce qui est une contradiction.
iI

iI

Puisque la famille (xi )16i6p1 est libre, son cardinal p 1 est infrieur ou gal la dimension n et donc p 6 n + 1.

http ://www.maths-france.fr

c Jean-Louis Rouget, 2014. Tous droits rservs.


Exercice no 11
Lapplication (P, Q) 7

Z1

P(t)Q(t) dt est un produit scalaire sur E = R3 [X]. Dterminons une base orthonorme de E.

Pour cela, dterminons (Q0 , Q1 , Q2 , Q3 ) lorthonormalise de la base canonique (P0 , P1 , P2 , P3 ) = (1, X, X2 , X3 ).


Z1
1
||P0 ||2 =
12 dt = 2 et on prend Q0 = .
2
1
r
Z
Z1
2
3
1 1
2
2
X.
t dt = 0 puis P1 (P1 |Q0 )Q0 = X puis kP1 (P1 |Q0 )Q0 k =
t dt = et Q1 =
P1 |Q0 =
3
2
2 Z1
1

2
1 1 2
1
P2 |Q0 =
et P2 |Q1 = 0. Donc, P2 (P2 |Q0 )Q0 (P2 |Q1 )Q1 = X2 ,
t dt =
3
3
2 1

2


Z1 

8
1 2 1
1
5
=
dt = 2
+
et Q2 = 3X2 1 .
t2
puis kP2 (P2 |Q0 )Q0 (P2 |Q1 )Q1 k2 =
3
5 9 9
45
2 2
r1 Z 1

3
3
6
et P3 (P3 |Q0 )Q0 (P3 |Q1 )Q1 (P3 |Q2 )Q2 = X3 X,
t4 dt =
Enfin, P3 |Q0 = P3 |Q2 = 0 et P3 |Q1 =
2 1
5
5




2

Z1 
3 3 2

7
3
25

21
6
3
8
1

puis
t3 t
=2

+
=
, et P3 = 5X3 3X .
dt = 2
X 5 X =
5
7 25 25
175
175
2 2
1

3
5
7
1
2
Une base orthonorme de E est (Q0 , Q1 , Q2 , Q3 ) o Q0 = , Q1 = X, Q2 = (3X 1) et Q3 = (5X3 3X).
2
2
2 2
2 2
Z1
P2 (t) dt = 1. Posons P = aQ0 + bQ1 + cQ2 + dQ3 .
Soit alors P un lment quelconque de E = R3 [X] tel que
1
Z1
Puisque (Q0 , Q1 , Q2 , Q3 ) est une base orthonorme de E,
P2 (t) dt = ||P||2 = a2 + b2 + c2 + d2 = 1. Maintenant, pour
1

x [1, 1], en posant Mi = Max{|Qi (x)|, x [1, 1]}, on a :

|P(x)| 6 |a| |Q0 (x)| + |b| |Q1 (x)| + |c| |Q2 (x)| + |d| |Q3 (x)| 6 |a|M0 + |b|M1 + |c|M2 + |d|M3
q
p
6 a2 + b2 + c2 + d2 M20 + M21 + M22 + M23 (daprs lingalit de Cauchy-Schwarz)
q
= M20 + M21 + M22 + M23 .

Une tude brve montre alors que chaque |Pi | atteint son maximum sur [1, 1] en 1 (et 1) et donc
r
q

1 3 5 7
2
2
2
2
+ + + = 2 2.
M0 + M1 + M2 + M3 =
2 2 2 2

Ainsi, x [1, 1], |P(x)| 6 2 2 et donc Max{|P(x)|, x [1, 1]} 6 2 2.

Etudions les cas dgalit. Soit P R3 [X] un polynme ventuel tel que Max{|P(x)|, x [1, 1]} 6 2 2.
Soit x0 [1, 1] tel que Max{|P(x)|, x [1, 1]} = |P(x0 |. Alors :

2 2 = |P(x0 )| 6 |a| |Q0 (x0 )| + |b| |Q1 (x0 )| + |c| |Q2 (x0 )| + |d| |Q3 (x0 )| 6 |a| M0 + |b| M1 + |c| M2 + |d| M3
q

6 M20 + M21 + M22 + M23 = 2 2.

Chacune de ces ingalits est donc une galit. La dernire (Cauchy-Schwarz) est une galit si et seulement si 


(|a|, |b|, |c|, |d|) est colinaire (1, 3, 5, 7) ou encore si et seulement si P est de la forme Q0 3Q1 5Q2 7Q3
1
o 2 (1 + 3 + 5 + 7) = 1 et donc = , ce qui ne laisse plus que 16 polynmes possibles. Lavant-dernire ingalit est
4
une galit si et seulement si x0 {1, 1} (clair). La premire ingalit est une galit si et seulement si
|aQ0 (1) + bQ1 (1) + cQ2 (1) + dQ3 (1)| = |a|Q0 (1) + |b|Q1 (1) + |c|Q2 (1) + |d|Q3 (1),
ce qui quivaut au fait que a, b, c et d aient mme signe et P est lun des deux polynmes





1
5
7
1
Q0 + 3Q1 + 5Q2 + 7Q3 =
1 + 3X + (3X2 1) + (5X3 3X)
4
2
2
4 2

1
= 35X3 + 15X2 15X 3
8 2

http ://www.maths-france.fr

c Jean-Louis Rouget, 2014. Tous droits rservs.


Exercice no 12
Lapplication (f, g) 7

Z1

f(t)g(t) dt est un produit scalaire sur C0 ([0, 1], R). Daprs lingalit de Cauchy-Schwarz,

In In+2 =

Z1

fn (t) dt

Z1

fn+2 (t) dt =

Z 1 p
Z 1 p


n 2
n+2 2
f(t)
f(t)
dt
dt

>

!2
Z1 p
np
n+2
f(t)
f(t)
dt
=
0

Z1

n+1

(t) dt

!2

= I2n+1

Comme f est continue et strictement


positive
sur [0, 1], In est strictement positif pour tout entier naturel n. Donc,


In+2
In+1
In+1
est dfinie et croissante.
6
. La suite
n N,
In
In+1
In
Exercice no 13

1) La symtrie, la bilinarit et la positivit sont claires. Soit alors P Rn [X].


P|P = 0

Z1

P2 (t) dt = 0

t [0, 1], P2 (t) = 0 (fonction continue, positive, dintgrale nulle)


P = 0 (polynme ayant une infinit de racines).
Z1
Ainsi, lapplication (P, Q) 7 P(t)Q(t) dt est un produit scalaire sur Rn [X].
0

2) Pour vrifier que la famille

vrifier que

Lp
kLp k

est lorthonormalise de Schmidt de la base canonique de E, nous allons

06p6n

p
a) p
 J0,nK, Vect(L0 , L1 , ..., Lp ) = Vect(1, X, ..., X ),
Lp
est orthonormale,
b)
kLp k 06p6n
c) p J0, nK, Lp |Xp > 0.

Pour a), on note que Lp est un polynme de degr p (et de coefficient dominant
base de Rp [X], ou encore, p J0, nK, Vect(L0 , L1 , ..., Lp ) = Vect(1, X, ..., Xp ).

(2p)!
). Par suite, (L0 , L1 , ..., Lp ) est une
p!

Soit p J0, nK. Soit P un polynme de degr infrieur ou gal p. Si p > 1, une intgration par parties fournit :
Lp |P =

Z1


p (p)

(t 1)

1
Z1

Z1
h
i1
2
p (p1)
P(t) dt = ((t 1) )
P(t)

((t2 1)p )(p1) P (t) dt


1

((t2 1)p )(p1) P (t) dt.

(k)
En effet, 1 et 1 sont racines dordre p de (t2 1)p et donc dordre pk de (t2 1)p
pour 0 6 k 6 p et en particulier,

2
p (k)
racines de chaque (t 1)
pour 0 6 k 6 p 1.
Z1
En ritrant, on obtient pour tout k J0, pK, Lp |P = (1)k
((t2 1)p )(pk) P(k) (t) dt et pour k = p, on obtient enfin
1
Z1
2
p (p)
p
Lp |P = (1)
(t 1) P (t) dt, cette formule restant vraie pour p = 0.
1

Soient p et q deux entiers tels que 0 6 q < p 6 n. Daprs ce qui prcde, Lp |Lq = (1)p

Z1

(t2 1)p L(p)


q (t) dt = 0 car

q = deg (Lq ) < p. Ainsi, la famille (Lp )06p6n est une famille
de n + 1 polynmes tous non nuls et est par

 orthogonale
Lp
est une base orthonormale de Rn [X].
suite est une base orthogonale de Rn [X]. On en dduit que
||Lp || 06p6n


Z1
Z1
Lp
2
p p (p)
2 p
p
p
Enfin, Lp |X = (1)
est
(t 1) (t )
dt = p!
(1 t ) dt > 0. On a montr que la famille
kLp k 06p6n
1
1
lorthonormalise de la base canonique de Rn [X].
http ://www.maths-france.fr

c Jean-Louis Rouget, 2014. Tous droits rservs.


Calculons kLp k. On note que Lp (L0 , ..., Lp1 ) = (Rp1 [X]) . Par suite,
kLp k2 = Lp |Lp = Lp |dom(Lp )Xp (car Lp (Rp1 [X]) )
Z1
Z1
(2p)!
(2p)!
p
2 p
Lp |X =
p!
(1 t ) dt = 2(2p)! (1 t2 )p dt
=
p!
p!
1
0
Z0
Z /2
= 2(2p)!
(1 cos2 u)p ( sin u) du = 2.(2p)!
sin2p+1 u du
/2

= 2(2p)!W2p+1 (Intgrales de Wallis)

(2p)(2p 2)....2
( revoir).
(2p + 1)(2p 1)...3
22p (p!)2
2
= 2(2p)!
=
22p (p!)2 .
(2p + 1)!
2p + 1

= 2(2p)!

Donc, p J0, nK, kLp k =

2
2p p!. On en dduit que la famille
2p + 1

2p + 1 1
((X2 1)p )(p)
2 2p p!

est une base


06p6n

orthonormale de Rn [X] (pour le produit scalaire considr).


Exercice no 14
Par hypothse, f(F) est un sous-espace vectoriel de F. De plus, f est un automorphisme et donc dim (f(F)) = dim(F) < +.
On en dduit que f(F) = F.

Vrifions que f F F . Soit x F .
Soit y F. Puisque F = f(F), il existe x F tel que f(x ) = y. Puisque f est un automorphisme orthogonal,
f(x)|y = f(x)|f(x ) = x|x = 0.


Donc, y F, f(x)|y = 0. On en dduit que f(x) F . Ainsi, x E, x F f(x) F et donc f F F .

Enfin, F est un sous-espace vectoriel de E, stable par f et le dbut de lexercice permet daffirmer que f F = F .

Exercice no 15

Soit B = (e1 , . . . , en ) une base orthonorme de E. Si et g existent, ncessairement on doit avoir


kf (e1 )k = kg (e1 )k = || kg (e1 )k = .
Soit donc = kf (e1 )k. Soit i J2, nK.
2

(ei + e1 |ei e1 ) = (ei |ei ) (e1 |e1 ) = 1 1 = 0.


Par hypothse, on en dduit que (f (ei + e1 ) |f (ei e1 )) = 0 ou encore kf (ei )k2 = kf (e1 )k2 = 2 puis kf (ei )k = . En
rsum, i J1, nK, kf (ei )k = .
1er cas. Si = 0, alors i J1, nK, kf (ei )k = 0 puis i J1, nK, f (ei ) = 0. Lendomorphisme f sannule sur une base de
E et donc f = 0. Dans ce cas, on peut prendre = 0 R+ et g = Id O(E). On a bien f = g.

1
1
f de sorte que f = g. Pour tout i J1, nK, kg (ei )k = kf (ei )k = 1.

Ainsi, les vecteurs g (e1 ), . . . , g (en ) sont unitaires et deux deux orthogonaux (car f conserve lorthogonalit). Limage
par g de la base orthonorme B est donc une base orthonorme de E. On en dduit que g est un automorphisme orthogonal.
Encore une fois, on a fourni un rel positif et un automorphisme orthogonal g tel que f = g.
2me cas. Sinon > 0. On pose alors g =

Exercice no 16
1) Supposons que p soit une projection orthogonale. Posons F = Im(p) de sorte que p = pF . On sait que Ker(p) = F .
Soit x E.
kxk2 = kpF (x) + (x pF (x))k2

= kpF (x)k2 + k(x pF (x))k2 (car x pF (x) Ker(p) = F et daprs le thorme de Pythagore)

> kpF (x)k2


et donc kxk > kpF (x)k.

http ://www.maths-france.fr

c Jean-Louis Rouget, 2014. Tous droits rservs.


Supposons que x E, kp(x)k 6 kxk. Montrons que p est une projection orthogonale. Posons F = im(p) et G = Ker(p)
(de sorte que p est la projection sur F paralllement G) et montrons que G = F . Soient x F et y G \ {0}. Pour tout
rel , on a
2

kxk2 = kp(x + y)k 6 kx + yk2 = kxk2 + 2(x|y) + 2 kyk2 ,

et donc R, 2(x|y) + 2 kyk2 > 0. Le polynme 7 2(x|y) + 2 kyk2 est un trinme du second degr (car kyk2 > 0)
et est de signe constant sur R. Son discriminant est rduit est donc ngatif ou nul.
On en dduit que (x|y)2 6 0 puis que x|y = 0. On a montr que tout vecteur de G est orthogonal tout vecteur de F et
donc que G F . Dautre part, G et F sont deux supplmentaires de F. F et G ont donc mmes dimensions finies. On en
dduit que G = F et donc que p est une projection orthogonale.
2) Supposons que p soit une projection orthogonale. Puisque p est une projection, on a p2 = p et donc P2 = P. Vrifions
alors que P est une matrice symtrique.
Puisque B est orthonorme, le coefficient ligne i, colonne j, 1 6 i, j 6 n de P qui est la i-me coordonne de p (ej ) dans B
est encore (p (ei ) |ej ). Pour montrer que P est symtrique, on doit donc vrifier que (i, j) J1, nK2 , (p (ei ) |ej ) = (ei |p (ej )).
Soit (i, j) J1, nK2 .

(p (ei ) |ej ) = (p (ei ) |p (ej ) + ej p (ej )) = (p (ei ) |p (ej )) + (p (ei ) |ej p (ej ))

= (p (ei ) |p (ej )) (car p (ei ) Im(p) et ej p (ej ) Ker(p) = (Im(p)) ).


Par symtrie des rles, on a aussi (ei |p (ej )) = (p (ei ) |p (ej )) et finalement (p (ei ) |ej ) = (ei |p (ej )). On a montr que la
matrice P est symtrique.
Supposons que P2 = P et t P = P et montrons que p est une projection orthogonale. Puisque P2 = P, on a p2 = p et
donc p est une projection.
Vrifions que (x, y) E2 , p(x)|y = x|p(y). Soient x =

p(x)|y =

n
X
i=1

n
X

xi ei et y =

i=1

n
X

yi ei deux lments de E.

i=1

! n
X
xi p (ei ) |
yj ej =
j=1

xi yj (p (ei ) |ej )

16i,j6n

xi yj (ei |p (ej )) (daprs plus haut)

16i,j6n

n
X
i=1

! n
X
xi ei |
yj p (ej )
j=1

= x|p(y).

Montrons alors que tout lment de Im(p) = F est orthogonal tout lment de Ker(p) = G. Soient x E et y Ker(p).
p(x)|y = x|p(y) = x|0 = 0.
Ainsi, Im(p) (Ker(p)) puis Im(p) = (Ker(p)) comme prcdemment car Im(p) et (Ker(p)) ont mmes dimensions
finies. On a montr que p est une projection orthogonale.
3)
x E, ks(x)k = kxk (y, z) Ker(s Is) Ker(s + Id), ky zk2 = ky + zk2

(y, z) Ker(s Is) Ker(s + Id), kyk2 2(y|z) + kzk2 = kyk2 + 2(y|z) + kzk2
(y, z) Ker(s Is) Ker(s + Id), y|z = 0

Ker(s + Id) (Ker(s Id)) .

Enfin, puisque Ker(s + Id) et Ker(s Id) sont supplmentaires, comme la question 1),

4)

Ker(s + Id) (Ker(s Id)) Ker(s + Id) = (Ker(s Id)) s symtrie orthogonale.

http ://www.maths-france.fr

c Jean-Louis Rouget, 2014. Tous droits rservs.


1
s symtrie orthogonale p = (Id + s) projection orthogonale
2


2

1
1
1
1
t

(In + S)
(In + S) = (In + S) (daprs 2))
= (In + S) et
2
2
2
2
In + 2S + S2 = 2In + 2S et In + t S = In + S

S2 = In et t S = S.

http ://www.maths-france.fr

10

c Jean-Louis Rouget, 2014. Tous droits rservs.


Planche no 37. Dnombrements


* trs facile ** facile *** difficult moyenne **** difficile
I : Incontournable T : pour travailler et mmoriser le cours
Exercice no 1 : (IT) (le poker)
On dispose dun jeu de 32 cartes. On distribue 5 cartes un joueur. Lordre des cartes est As, Roi, Dame, Valet, 10, 9, 8,
7 soit un total de 8 hauteurs. Il y a quatre couleurs : (carreau), (cur), (pique) et (trfle).
1) Combien de mains diffrentes peut-il recevoir (nombre de distributions possibles) ?
2) Combien contiennent une quinte floche (5 cartes conscutives dans une mme couleur) ?
3) Combien contiennent un carr (4 cartes dune mme hauteur et une autre carte) ?
4) Combien contiennent une couleur (5 cartes dune mme couleur ne constituant pas une quinte floche) ?
5) Combien contiennent un full (3 cartes dune mme hauteur et deux dune mme autre hauteur) ?
6) Combien contiennent une suite (5 cartes conscutives ne constituant pas une quinte floche) ?
7) Combien contiennent un brelan (3 cartes dune mme hauteur et deux autres ne constituant pas un full) ?
8) Combien contiennent une double paire (2 cartes dune mme hauteur, deux autres dune mme hauteur ne
constituant pas un carr ou un full) ?
9) Combien contiennent une paire (2 cartes dune mme hauteur et rien de mieux) ?
Exercice no 2 : (IT)
On dispose dun jeu de 32 cartes. On distribue 5 cartes un joueur.
1)
2)
3)
4)
5)

Combien
Combien
Combien
Combien
Combien

de
de
de
de
de

mains
mains
mains
mains
mains

contiennent
contiennent
contiennent
contiennent
contiennent

exactement un roi ?
exactement trois piques ?
exactement deux piques et deux curs ?
au moins deux carreaux ?
exactement un roi et deux trfles ?

Exercice no 3 : (IT) (le loto)


On joue au loto en cochant dans une grille 6 numros parmi les numros 1, 2, . . . , 49. On place ensuite 49 boules numrotes
de 1 49 dans une urne et on en extrait 6. On obtient ainsi les numros gagnants.
1) Combien y-a-t-il de tirages possibles ?
2) Combien de tirages nous fournissent exactement 1 numro gagnant ?
3) Combien de tirages nous fournissent exactement 2, 3, 4, 5, 6 numros gagnants ?
Exercice no 4 : (IT)
Combien y-a-t-il danagrammes du mot LVE ? Mme question en effaant les accents ?
Combien y-a-t-il danagrammes du mot MATHMATIQUES ?
Exercice no 5 : (IT)
1) (***) Trouver une dmonstration combinatoire de lidentit

X
06k6n/2

n
2k

X
06k6(n1)/2


n
ou encore
2k + 1

dmontrer directement quun ensemble n lments contient autant de parties de cardinal pair que de parties
de cardinal impair.
 


n
n1
2) (****) Trouver une dmonstration combinatoire de lidentit k
=n
.
k
k1
  X
n  2
2n
n
3) (****) Trouver une dmonstration combinatoire de lidentit
=
.
n
k
k=0
Exercice no 6 : (***)
Combien y a-t-il de partitions dun ensemble pq lments en p classes ayant chacune q lments ?
Exercice no 7 : (***) (Combinaisons avec rptitions)
Montrer que
 le nombre de solutions en nombres entiers xi > 0 de lquation x1 + x2 + ... + xn = k (k entier naturel donn)
k
est n+k1
. (Noter an,k le nombre de solutions et procder par rcurrence.)
Exercice no 8 : (*)

Combien y a-t-il de nombres de 5 chiffres o 0 figure une fois et une seule ?


http ://www.maths-france.fr

c Jean-Louis Rouget, 2014. Tous droits rservs.


Exercice no 9 : (***I)
Quelle est la probabilit pn pour que dans un groupe de n personnes choisies au hasard, deux personnes au moins aient
le mme anniversaire (on considrera que lanne a toujours 365 jours, tous quiprobables). Montrer que pour n > 23, on
1
a pn > .
2
Exercice no 10 : (***)
Montrer que le premier de lan tombe plus souvent un dimanche quun samedi.
Exercice no 11 : (**I)
On part du point de coordonnes (0, 0) pour rejoindre le point de coordonnes (p, q) (p et q entiers naturels donns) en
se dplaant chaque tape dune unit vers la droite ou vers le haut. Combien y a-t-il de chemins possibles ?
Exercice no 12 : (***)
De combien de faons peut-on payer 100 euros avec des pices de 10, 20 et 50 centimes ?
Exercice no 13 : (****)
1) Soit E un ensemble fini et non vide. Soient n un entier naturel non nul et A1 ,..., An , n parties de E.
Montrer la formule du crible :

card(A1 ... An ) =

n
X

card(Ai )

i=1

card(Ai1 Ai2 )

16i1 <i2 6n

+ ... + (1)k1

card(Ai1 Ai2 ... Aik )

16i1 <i2 <...<ik 6n

+ ... + (1)n1 card(A1 ... An ).


2) Combien y a-t-il de permutations de J1, nK vrifiant i J1, nK, (i) 6= i ? (Ces permutations sont appeles
drangements (permutations sans point fixe)). Indication : noter Ai lensemble des permutations qui fixent i et
utiliser 1).
On peut alors rsoudre un clbre problme de probabilit, le problme des chapeaux. n personnes laissent leur chapeau
un vestiaire. En repartant, chaque personne reprend un chapeau au hasard. Montrer que la probabilit quaucune de ces
1
personnes nait repris son propre chapeau est environ quand n est grand.
e
o
Exercice n 14 : (**)
Combien y a-t-il de surjections de J1, n + 1K sur J1, nK ?
Exercice no 15 : (***)
Soit (P) un polygone convexe n sommets. Combien ce polygone a-t-il de diagonales ? En combien de points distincts des
sommets se coupent-elles au maximum ?
Exercice no 16 : (***)
1) On donne n droites du plan. On suppose quil nen existe pas deux qui soient parallles, ni trois qui soient concourantes.
Dterminer le nombre P(n) de rgions dlimites par ces droites.
2) On donne n plans de lespace. On suppose quil nen existe pas deux qui soient parallles, ni trois qui soient concourants
en une droite, ni quatre qui soient concourants en un point. Dterminer le nombre Q(n) de rgions dlimites par ces
plans.
Exercice no 17 : (***)
k
Soit Pn
le nombre de partitions dun ensemble n lments en k classes.
k1
k
k
Montrer que Pn
= Pn1
+ kPn1
pour 2 6 k 6 n 1.
Dresser un tableau pour 1 6 k, n 6 5.
k
Calculer en fonction de Pn
le nombre de surjections dun ensemble n lments sur un ensemble p lments.

http ://www.maths-france.fr

c Jean-Louis Rouget, 2014. Tous droits rservs.


Exercice no 18 : (*** I)
Soit E un ensemble fini n lments (n N ). Combien y-a-t-il de couples (X, Y) (P(E))2 tels que X Y ?
Exercice no 19 : (*** I)
X

Soit E un ensemble fini n lments (n N ). Calculer S1 =

card (X Y) et S2 =
2

(X,Y)(P(E))

card (X Y).
2

(X,Y)(P(E))

Exercice n 20 : (***)
o

Soit E un ensemble fini n lments (n N ). Combien y-a-t-il de :


1) lois de composition interne sur E ;
2) lois de composition interne commutatives sur E ;
3) lois de composition interne sur E possdant un lment neutre ?

http ://www.maths-france.fr

c Jean-Louis Rouget, 2014. Tous droits rservs.


Planche no 37. Dnombrements : corrig


Exercice no 1
1) Le nombre de mains est le nombre de parties 5 lments dun ensemble 32 lments. Il y en a
 
32 31 30 29 28
32
= 8 31 29 28 = 201 376.
=
5432
5
Avec un jeu de trente-deux cartes, il y a 201 376 mains de cinq cartes.
2) Il y a quatre couleurs (carreau, cur, pique trfle) et quatre hauteurs ( las, au roi, la dame et au valet). Au total,
il y a 4 4 = 16 quintes floches.
Avec un jeu de trente-deux cartes, il y a 16 quintes floches (dont quatre royales).
16
1
=
= 0, 000079 . . .
201 376
12 586
3) Chaque carr das peut tre accompagn dune carte choisie parmi les 28 qui ne sont pas des as. Il y a donc 28 carrs
das. Comme il y a 8 hauteurs possibles, il y a 28 8 = 224 mains contenant un carr.
La probabilit davoir une quinte floche est donc

Avec un jeu de trente-deux cartes, il y a 224 mains contenant un carr.


224
1
=
= 0, 0011 . . .
201 376
899
4) Il y a 4 couleurs (carreau, cur, pique ou trfle). Le nombre de mains contenant 5 carreaux est le nombre de parties
5 lments dun ensemble 8 lments. Il y en a
   
8
876
8
= 8 7 = 56,
=
=
32
5
3
La probabilit davoir un carr est donc

et donc au total, 4 56 = 224 mains contenant 5 cartes de couleurs identiques. On doit enlever ces mains les quintes
floches au nombre de 16 (qui sont ranger dans la catgorie des quintes floches et pas la catgorie des couleurs). Il reste
224 16 = 208 couleurs.
Avec un jeu de trente-deux cartes, il y a 208 couleurs.
13
208
=
= 0, 00103 . . .
201 376
12 586
5)
Comptons le nombre de mains contenant un full aux as par les rois (brelan das et paire de roi). Il y a 4 brelans das et
 
4
= 6 paires de rois. Il y a donc 4 6 = 24 mains contenant un full aux as par les rois. Maintenant, il y a 8 hauteurs
2
possibles pour le full et pour chacune de ces hauteurs, il y a 7 hauteurs pour la paire soit au total 8 7 = 56 hauteurs
pour le full. Il y a donc 24 56 = 1344 mains contenant un full.
La probabilit davoir une couleur est donc

Avec un jeu de trente-deux cartes, il y a 1 344 mains contenant un full.


La probabilit davoir un full est donc

1 344
6
=
= 0, 0066 . . .
201 376
899

6) Comptons le nombre de suites las. Il y a 4 as, 4 rois, 4 dames, 4 valets et 4 dix et donc 45 mains contenant la
squence As, roi, dame, valet, dix. Une suite peut commencer las, au roi, la dame ou au valet et donc il y a 4 45 = 46
mains contenant 5 cartes qui se suivent. On doit retirer de ces mains celles qui fournissent une quinte floche et il reste
46 16 = 4080 mains contenant une suite.
Avec un jeu de trente-deux cartes, il y a 4 080 mains contenant une suite.

http ://www.maths-france.fr

c Jean-Louis Rouget, 2014. Tous droits rservs.


4 080
255
=
= 0, 019 . . .
201 376
12856
 
4
7) Il y a 8 hauteurs possibles du brelan et pour chaque hauteur
= 4 brelans possibles. Donc, 8 4 = 32 brelans
3
possibles. On a 28 possibilits de complter ce brelan par une quatrime carte puis 24 possibilits de complter par une
28 24
cinquime carte. Lordre dans lequel on a reu les deux dernires cartes na pas dimportance et donc il y a 32
=
2
10 752 mains contenant un brelan.
La probabilit davoir une suite est donc

Avec un jeu de trente-deux cartes, il y a 10 752 mains contenant un brelan.


10 752
La probabilit davoir un brelan est donc
= 0, 053 . . .
201 376
 
 
8
4
8) Il y a
= 28 hauteurs possibles des deux paires et pour chaque hauteur
= 6 doubles paires possibles. Donc,
2
2
     
8
4
4

= 28 6 6 = 1 008 doubles paires possibles. On a ensuite 24 possibilits de complter ces deux paires
2
2
2
par une cinquime carte. Il y a 1 008 24 = 24 192 mains contenant deux paires.
Avec un jeu de trente-deux cartes, il y a 24 192 mains contenant deux paires.
24 192
La probabilit davoir deux paires est donc
= 0, 12 . . .
201 376
 
4
9) Il y a 8
= 48 paires. On complte par une 3me, 4me et 5me carte soit 28 24 20 possibilits. Lordre dans
2
28 24 20
= 107 520.
lequel on reoit ces cartes na pas dimportance et il y a donc 48
32
Avec un jeu de trente-deux cartes, il y a 107 520 mains contenant une paire.
La probabilit davoir une paire est donc

107 520
= 0, 53 . . .
201 376

Exercice no 2
Le nombre de mains est le nombre de parties 5 lments dun ensemble 32 lments. Il y en a
 
32
32 31 30 29 28
= 8 31 29 28 = 201 376.
=
5432
5

1) On prend une carte parmi les 4 rois et 4 cartes parmi les 28 qui ne sont pas des rois. Au total,
   
28 27 26 25
4
28

=4
= 4 7 9 13 25 = 81 900
1
4
432
mains contenant exactement un roi.

2) On prend 2 cartes parmi les 8 piques et 3 cartes parmi les 24 qui ne sont pas des piques. Au total,
   
8
8 7 24 23 22
24

= 4 7 4 23 22 = 56 672

=
2
32
2
3

mains contenant exactement deux piques.

3) On prend 2 cartes parmi les 8 piques et 2 cartes parmi les 8 crus et 1 cartes parmi les 16 qui ne sont ni des piques,
ni des curs. Au total,
     
8
8
16

= 28 28 16 = 12 544
2
2
1
mains contenant exactement deux piques et deux curs.
 
24
24 23 22 21 20
4) Le nombre de mains contenant 0 carreau est
=
= 4 23 22 21 = 42 504 et le nombre
5
5432
   
24 23 22 21
8
24
= 8 23 22 21 = 85 008. Le nombre de mains
de mains contenant 1 carreau est

= 8
432
1
4
http ://www.maths-france.fr

c Jean-Louis Rouget, 2014. Tous droits rservs.


contenant aux moins deux carreaux est le nombre total de mains auquel on retranche le nombre de mains contenant au
plus un carreau. Au total, il y a
       
32
24
8
24

= 201 376 42 504 85 008 = 73 864


8
5
1
4
mains contenant au moins deux carreaux.
5) Il y a 32 11 = 21 cartes qui ne sont ni des rois, ni des trfles. Le nombre de mains contenant deux trfles et un roi
qui nest pas le roi de trfles est
     
3
7
21

= 3 21 21 10 = 13 230
1
2
2
et le nombre de mains contenant exactement deux trfles et un roi qui est le roi de trfle est
   
21
7
21 20 19
1

=7
= 7 7 10 19 = 9 310.
1
1
3
32
Au total, il y a 13 230 + 9 310 = 22 540 mains contenant exactement un roi et deux trfles.

Exercice no 3
 
49
1) Il y a
tirages possibles avec
6
 
49 48 47 46 45 44
49
=
= 49 47 46 3 44 = 13 983 816.
6
65432

2) On choisit un numro parmi les six bons numros et les cinq autres parmi les 49 6 = 43 mauvais numros. Au total,
il y a
   
6
43 42 41 40 39
43

=6
= 6 43 14 41 39 = 5 775 588
1
5
5432

5 775 588
tirages avec exactement 1 bon numro. La probabilit davoir exactement un bon numro est donc
= 0, 41 . . .
13 983 816
   
6
1 851 150
43
= 0, 13 . . ..
3) Deux bons numros :

= 1 851 150. Probabilit :


13 983 816
  2  4
6
246 820
43
Trois bons numros :

= 246 820. Probabilit :


= 0, 01 . . ..
3  
3 
13 983 816
13 545
6
43
= 0, 0009 . . ..
Quatre bons numros :

= 13 545. Probabilit :
13
983 816
4
2
   
6
258
43
Cinq bons numros :

= 258. Probabilit :
1, 8 105 .
5
1
13
983
816
   
6
1
43
7 108 .
Six bons numros :

= 1. Probabilit :
13 983 816
6
0
Exercice no 4
Avec les accents : il y a 5! = 120 anagrammes du mot LVE.

Sans les accents : le mot ELEVE est lun des anagrammes. Il fournit 3! = 6 anagrammes deux deux distincts en tenant
compte des accents, 3! tant le nombre de permutations des 3 E dans leurs trois positions savoir LVE, LEV,
LEV, LVE, ELV et ELV.
Plus gnralement, chacun des n anagrammes du mot ELEVE fournit 6 anagrammes deux deux distincts du mot LVE.
120
Donc 6n = 120 ou encore n =
= 20. Il y a 20 anagrammes du mot ELEVE.
6

Le mot MATHMATIQUES est constitu de 13 lettres savoir 2 M, 2 A, 2 T, 1 H, 1 , 1 I, 1 Q, 1 U, 1 E et 1S.


On compte dabord les anagrammes en diffrenciant les deux M, les deux A et les deux T, par exemple en les notant M1
et M2 . Il y en a 13!. Maintenant, lanagramme MATHMATIQUES fournit 2! 2! 2! = 8 anagrammes deux deux
distincts o les M, les A et les T sont diffrencis savoir M1 A1 T1 HM2 A2 T2 IQUES, M2 A1 T1 HM1 A2 T2 IQUES, ...
Le nombre danagrammes cherch est donc
13!
= 13 12 11 10 9 7 6 5 4 3 2 = 778 377 600.
2! 2! 2!
Exercice no 5
http ://www.maths-france.fr

c Jean-Louis Rouget, 2014. Tous droits rservs.


1) Soit E un ensemble n lments, n > 1, et a un lment fix de E. Soit f :

P(E)
A

Montrons que f est involutive (et donc bijective). Soit A un lment de P(E).


P(E)
A \ {a} si a A
7
A {a} si a
/A

Si a
/ A, f(A) = A {a} et donc, puisque a A {a}, f(f(A)) = (A {a}) \ {a} = A.
Si a A, f(A) = A \ {a} et f(f(A)) = (A \ {a}) {a} = A.

Ainsi, A P(E), f f(A) = A ou encore, f f = IdP(E) .

Maintenant clairement, en notant Pp (E) (resp. Pi (E)) lensemble des parties de E de cardinal pair (resp. impair),
f (Pp (E)) Pi (E) et f(Pi (E)) Pp (E). Donc, puisque f est bijective
card (Pp (E)) = card (f (Pp (E))) 6 card (Pi (E))
et de mme card (Pi (E)) 6 card (Pp (E)). Finalement, card (Pi (E)) = card (Pp (E)).
2) Soient
 E ={a1 , ..., an } un ensemble n lments et a un lment
 fix de E.Soit k J1, n 1K.

n1
n1
n1
n1
Il y a
parties k lments qui contiennent a. Donc, n
=
+ ... +
est la somme du
k1
k1
k1
k1
nombre de parties k lments qui contiennent a1 et du nombre de parties k lments qui contiennent a2 ... et du
nombre de parties k lments qui contiennent an .
Dans cette derniresomme,
chaque partie k lments de E a t compte plusieurs fois et toutes les parties k lments

n
(en nombre gal
) ont t compts un mme nombre de fois. Combien de fois a t compte {a1 , a2 ...ak } ? Cette
k
partie a t compte une fois en tant que partie contenant a1 , une fois en tant que partie contenant a2 ... et une fois comme
partie contenant ak et donc a t compte k fois.
 


n
n1
Conclusion : k
=n
.
k
k1
 
2n
3) Soit E = {a1 , ..., an , b1 , ..., bn } un ensemble 2n lments. Il y a
parties n lments de E. Une telle partie a k
n
 
n
lments dans {a1 , ..., an } et n k dans {b1 , ..., bn } pour un certain k de {0, ..., n}. Il y a
choix possibles de k lments
k


n
dans {a1 , ..., an } et
choix possibles de n k lments dans {b1 , ..., bn } pour k donn dans {0, ..., n}. Quand k varie
nk
de 0 n, on obtient :
  X
 X
n  
n  2
2n
n
n
n
=
=
.
n
k
nk
k
k=0

k=0

Exercice n 6
 

 

pq
pq q
(p 1)q
Il y a
choix possibles dune premire classe. Cette premire classe tant choisie, il y a
=
choix
q
q 
q   
 
q
pq (p 1)q
q
possibles de la deuxime classe... et
choix possibles de la p-me classe. Au total, il y a
...
q
q
q
q
choix possibles dune premire classe, puis dune deuxime ...puis dune p-me.
 

 
pq (p 1)q
q
Maintenant dans le nombre
...
, on a compt plusieurs fois chaque partition, chacune ayant t
q
q
q
compt un nombre gal de fois.
On a compt chaque partition autant de fois quil y a de permutations des p classes savoir p!. Le nombre cherch est
donc :
 
  
(pq)!
((p 1)q)!
1
(2q)! q!
(pq)!
q
1 pq (p 1)q
...
.
=
...
=
p! q
p! q!((p 1)q)! q!((p 2)q)! q!q! q!0!
p!(q!)p
q
q
o

Exercice no 7
n N , an,0 = 1 (unique solution : 0 + 0 + ... + 0 = 0) et k N, a1,k = 1 (unique solution : k = k).

Soient n > 1 et k > 0. an+1,k est le nombre de solutions en nombre entiers positifs xi de lquation x1 +...+xn +xn+1 = k. Il
y a an,k solutions telles que xn+1 = 0 puis an,k1 solutions telles que xn+1 = 1 ... puis an,0 solutions telles que xn+1 = k.
Donc, n N , k N, an+1,k = an,k + an,k1 + ... + an,0 (et on rappelle an,0 = a1,k = 1).
http ://www.maths-france.fr

c Jean-Louis Rouget, 2014. Tous droits rservs.


Montrons alors par rcurrence sur n, entier naturel non nul, que : n N , k N, an,k



n+k1
=
.
k



1+k1
.
k
= Ckn+k1 . Soit k > 1.

Pour n = 1, on a pour tout naturel k, a1,k = 1 =


Soit n > 1, supposons que k N, an,k
an+1,k =

k
X

an,i =

i=0


k 
X
n+i1
i=0

=1+


k 
X
n+i
i=1

i+1







n+i1
n+k
n+k
=1+
1=
,
i
k+1
k+1

ce qui reste vrai pour k = 0.


On a montr par rcurrence que n N , k N, an,k



n+k1
=
.
k

Exercice no 8
On place le 0 soit au chiffre des units, soit au chiffre des dizaines, soit au chiffre des centaines, soit au chiffre des milliers
(mais pas au chiffre des dizaines de milliers) et le 0 tant plac, on ny a plus droit.
Rponse : 4 9 9 9 9 = 4 94 = 4 (80 + 1)2 = 4 6561 = 26244.
Exercice no 9
Si n > 366, on a pn = 1 (Principe des tiroirs : si 366 personnes sont associer 365 dates danniversaire, alors 2 personnes
au moins sont associer la mme date danniversaire).
Si 2 6 n 6 365, on a pn = 1 qn o qn est la probabilit que les dates danniversaire soient deux deux distinctes. Il y
a (365)n rpartitions possibles des dates danniversaires (cas possibles) et parmi ces rpartitions, il y en a
365 364 363 . . . (365 (n 1)) telles que les dates danniversaire soient deux deux distinctes. Finalement
pn = 1


n1
n1
Y 365 k
Y
365 364 363 . . . (365 (n 1))
k
=
1

=
1

.
1

(365)n
365
365
k=1

k=1

Ensuite,



n1
n1
n1
Y
X 
X
1
k
1
k
k
1
6 ln
> ln 2.
(1
)6
ln 1
ln 1
pn >
2
365
2
365
2
365
k=1

k=1

Maintenant, soit x [0, 1[. On a

ln(1 x) =

k=1

Zx
0

1
dt >
1t

Zx
0

1
dt = x.
10

k
Pour k lment de J1, n 1K J1, 364K,
est un rel lment de [0, 1[. En appliquant lingalit prcdente, on obtient
365
 n1

n1
X k
X
k
n(n 1)
>
ln 1
=
.
365
365
730
k=1

k=1

Ainsi,

n(n 1)
1 + 1 + 2920 ln 2
1
2
> ln 2 n n 730 ln 2 > 0 n >
= 22, 9 . . . n > 23.
pn >
2
730
2
Finalement, dans un groupe dau moins 23 personnes, il y a plus dune chance sur deux que deux personnes au moins
aient la mme date danniversaire.
Exercice no 10 (Cest long lire et inutile pour Sup et Sp)
1) Notre calendrier est 400 ans priodique (et presque 4 7 = 28 ans priodique). En effet,

a) la rpartition des annes bissextiles est 400 ans priodique (1600 et 2000 sont bissextiles mais 1700, 1800 et 1900 ne le
sont pas (entre autre pour regagner 3 jours tous les 400 ans et coller le plus possible au rythme du soleil))
b) il y a un nombre entier de semaines dans une priode de 400 ans. En effet, sur 400 ans, le quart des annes, soit 100
ans, moins 3 annes sont bissextiles et donc sur toute priode de 400 ans il y a 97 annes bissextiles et 303 annes non
bissextiles.
http ://www.maths-france.fr

c Jean-Louis Rouget, 2014. Tous droits rservs.


Une anne non bissextile de 365 jours est constitue de 52 7 + 1 jours ou encore dun nombre entier de semaines plus un
jour et une anne bissextile est constitue dun nombre entier de semaine plus deux jours.
Une priode de 400 ans est donc constitue dun nombre entier de semaines auquel on ajoute
97 2 + 303 1 = 194 + 303 = 497 = 7 71

jours qui fournit encore un nombre entier de semaines.

2) Deux priodes conscutives de 28 ans ne contenant pas dexception (sicles non bissextiles) reproduisent le mme
calendrier. En effet, les 7 annes bissextiles fournissent un nombre entiers de semaines plus 2 7 jours = 2 semaines et les
21 annes non bissextiles fournissent un nombre entier de semaines plus 21 1 jours = 3 semaines.

3) Daprs ce qui prcde, il suffit de compter les 1ers de lan qui tombe un dimanche ou un samedi sur une priode de
400 ans donne, par exemple de 1900 2299 (inclus).
On dcompose cette priode comme suit :
1900 , 1901 1928, 1929 1956, 1957 1984, 1985 2012, 2013 2040, 2041 2068, 2069 2096,
2097 2100 , 2101 2128, 2129 2156, 2157 2184, 2185 2200 , 2201 2228, 2229 2256,
2257 2284, 2285 2299 .

4) On montre ensuite que sur toute priode de 28 ans sans sicle non bissextile, le premier de lan tombe un mme nombre
de fois chaque jour de la semaine (lundi, mardi,..). Quand on passe dune anne non bissextile lanne suivante, comme
une telle anne contient un nombre entier de semaines plus un jour, le 1er de lan tombe un jour plus tard lanne qui suit
et deux jours plus tard si lanne est bissextile. Par exemple,
1er janvier 1998 : jeudi, 1er janvier 1999 : vendredi, 1er janvier 2000 : samedi, 1er janvier 2001 : Lundi, 1er janvier
2002 : Mardi, 1er janvier 2003 : Mercredi, 1er janvier 2004 : Jeudi, 1er janvier 2005 : samedi...
Notons A,B,C,D,E,F,G les jours de la semaine. Sur une priode de 28 ans sans sicle non bissextile finissant par exemple
une anne bissextile, on trouve la squence suivante :
ABCD FGAB DEFG BCDE GABC EFGA CDEF (puis redmarre ABCD...) soit 4A, 4B, 4C, 4D, 4E, 4F, et 4G.
5) Il reste tudier les priodes exception (encadres dans le 3)).
Dtermination du 1er janvier 1900. Le 1er janvier 2015 tait un jeudi . Il en est donc de mme du 1er janvier 2015-28 =
1987 et des premiers janvier 1959, 1931 et 1903 puis on remonte : 1903 Jeudi 1902 Mercredi 1901 Mardi 1900 Lundi (1900
nest pas bissextile). Le premier de lan 1900 tait un Lundi.
Les premiers de lan 2000, 2028 , 2056 et 2084 sont des samedis, 2088 un jeudi, 2092 un mardi, 2096 un dimanche et donc
2097 mardi 2098 mercredi 2099 jeudi 2100 vendredi.
2101 est un samedi de mme que 2129, 2157, 2185 ce qui donne de 2185 2200 inclus la squence :
S D L Ma J V S D Ma Me J V D L Ma Me
2201 est un jeudi de mme que 2285 ce qui donne de 2285 2299 inclus la squence : J V S D Ma Me J V D L Ma Me V
SD
Le dcompte des Lundis , mardis ... des priodes exceptions est : 6D 4L 6Ma 5Me 5J 6V 4S. Dans toute priode de 400
ans, le 1er de lan tombe 2 fois de plus le dimanche que le samedi et donc plus souvent le dimanche que le samedi.
Exercice no 11
On pose H = vers le haut et D = vers la droite . Un exemple de chemin de (0, 0) (p, q) est le mot DD...DHH...H
o D est 
crit p 
fois et H est crit q fois. Le nombre de chemins cherch est le nombre danagrammes du mot prcdent.
p+q
Il y en a
(nombre de choix de lemplacement du D).
q
Exercice no 12
On note respectivement x, y et z le nombre de pices de 10, 20 et 50 centimes. Il sagit de rsoudre dans N3 lquation
10x + 20y + 50z = 10000 ou encore x + 2y + 5z = 1000.


k
Soit k N. x+2y = k x = k2y et donc lensemble des solutions dans N2 de cette quation est (k 2y, y), 0 6 y 6
2

 
 
k
k
ou encore (k 2y, y), 0 6 y 6 E
. Le nombre de solutions de cette quation est E
+ 1.
2
2


1000 5z
Pour 0 6 z 6 200 donn, le nombre de solutions de lquation x + 2y = 1000 5z est E
+ 1. Le nombre de
2
solutions en nombres entiers de lquation x + 2y + 5z = 1000 est donc
http ://www.maths-france.fr

c Jean-Louis Rouget, 2014. Tous droits rservs.



 X



200  
200 
200
200  
X
X
X
5z
5z
5z
1000 5z
+1 =
E
+ 501 = 201 501 +
E
= 100701 +
E(
).
E
2
2
2
2
z=0

z=0

z=0

z=0

Enfin
 X


 X

 X
200 
100  
100  
100
X
5z
5
5(2k 1)
5(2k)
E
E
E 5k +
=
+E
=
5k =
(10k + 2)
2
2
2
2

z=0

k=1

k=1

k=1

100 101
= 200 10
.
2
Le nombre de solutions cherch est donc 100701 50300 = 50401. Il y a 50401 faons de payer 100 euros avec des pices
de 10, 20 et 50 centimes.
Exercice no 13
1)
A1 ...An = 1 A1 ...An = 1 A1 ...An = 1 A1 ... An

n
X
X
= 1 (1 A1 ) ... (1 An ) = 1 1 +
(1)k
k=1

n
X

(1)k1

k=1

16i1 <...<ik 6n

16i1 <...<ik 6n

Ai1 ...Aik

Ai1 ...Aik ,

et en sommant sur les x lments de E, on obtient le rsultat.


2) Pour 1 6 k 6 n, posons Ak = { Sn / (k) = k}. Lensemble des permutations ayant au moins un point fixe est
A1 A2 ... An . Lensemble des permutations sans point fixe est le complmentaire dans Sn de A1 A2 ... An .
Daprs 1), leur nombre est

card(Sn ) card(A1 A2 ... An ) = card(Sn )

n
X

card(Ai ) +

i=1

... + (1)k

X
i<j

i1 <i2 <...<ik

card(Ai Aj )

card(Ai1 ...Aik ) + ...

+ (1)n card(A1 ... An ).


Ai est lensemble des permutations qui fixent i. Il y en a (n 1)! (nombre de permutations de {1, ..., n} \ {i}). Ai Aj est
lensemble des permutations qui fixent i et j. Il y en a (n 2)!. Plus gnralement, card(Ai1 ... Aik ) = (n k)!.
 
 
n
n
Dautre part, il y a n =
entiers i dans {1, . . . , n} puis
couples (i, j) tels que i < j et plus gnralement, il y a
1
2
 
n
k-uplets (i1 , ..., ik ) tels que i1 < i2 < ... < ik . Le nombre de drangements est
k
n
n
X
X
n!
(1)k
(1)k
n! +
(n k)! = n!
.
k!(n k)!
k!
k=1

k=0

Ainsi le problme des chapeaux admet pour rponse


pn =

n
X
(1)k
.
k!

k=0

1
= 0, 36... quand n tend vers linfini.
e
Z1
n
X
(1)k
(1 t)n t
n+1
x
1
La formule de Taylor-Laplace applique la fonction x 7 e fournit n N, e =
+(1)
e dt.
k!
n!
0

Montrons que cette suite tend trs rapidement vers

k=0

http ://www.maths-france.fr

c Jean-Louis Rouget, 2014. Tous droits rservs.


On en dduit que


pn

Z

Z1
Z1
1
1
(1 t)n t
(1 t)n
1
(1 t)n t
n+1
e
dt
=
e
dt
6
dt =
.
=
(1)





e
n!
n!
n!
(n
+
1)!
0
0
0

Ceci montre que pn tend trs rapidement vers

1
.
e

Exercice no 14
Soit n un naturel non nul. Dire que f est une surjection de J1, n + 1K sur J1, nK quivaut dire que deux des entiers de
{1, ..., n + 1} ont mme
 image
 k par f et que les autres ont des images deux deux distinctes etdistinctes
 de k. On choisit
n+1
n+1
ces deux entiers :
choix et leur image commune : n images possibles ce qui fournit n
choix dune paire
2
2
de {1, ..., n + 1} et de leur image commune. Puis il y a (n 1)! choix des images des n 1 lments restants. Au total, il y
n(n + 1)
n (n + 1)!
a (n 1)! n
=
surjections de J1, n + 1K sur J1, nK.
2
2
Exercice no 15
Soit n > 5. De chaque sommet part n 1 droites (vers les n 1 autres sommets) dont 2 sont des cots et n 3 des
n(n 3)
diagonales. Comme chaque diagonale passe par 2 sommets , il y a
diagonales.
2


n(n 3)/2
Ces diagonales se recoupent en
points distincts ou confondus. Dans ce dcompte, chaque sommet a t
2


n3
compt autant de fois que lon a choisi une paire de deux diagonales passant par ce sommet savoir
. Maintenant,
2
il y a n sommets.
Rponse :






1 n(n 3) n(n 3)
n(n 3)/2
n3
(n 3)(n 4)
n(n 3)
n
=
1 n
=
(n(n 3) 2 4(n 4))
2
2
2
2
2
2
8
n(n 3) 2
=
(n 7n + 14)
8
Les diagonales se recoupent en

n(n 3)(n2 7n + 14)


points distincts ou confondus et distincts des sommets ou encore
8

n(n 3)(n2 7n + 14)


points au maximum.
8
Exercice no 16

en

1) On a bien sr P(1) = 2. Soit n > 1. On trace n droites vrifiant les conditions de lnonc. Elles partagent le plan
en P(n) rgions. On trace ensuite Dn+1 , une (n + 1)-me droite. Par hypothse, elle coupe chacune des n premires
droites en n points deux deux distincts. Ces n points dfinissent (n + 1) intervalles sur la droite Dn+1 . Chacun de ces
(n + 1) intervalles partage une des P(n) rgions dj existantes en deux rgions et rajoute donc une nouvelle rgion. Ainsi,
P(n + 1) = P(n) + (n + 1).
Soit n > 2.

P(n) = P(1) +

n1
X

(P(k + 1) P(k)) = 2 +

k=1

n1
X
k=1

(k + 1) = 1 +

n
X

k=1

k=1+

n(n + 1)
2

n2 + n + 2
=
2
ce qui reste vrai pour n = 1.
n > 1, P(n) = 1 +

n2 + n + 2
n(n + 1)
=
.
2
2

2) On a bien sr Q(1) = 2. Soit n > 1. On trace n plans vrifiant les conditions de lnonc. Ils partagent lespace en
Q(n) rgions. On trace ensuite Pn+1 , un (n + 1)-me plan. Par hypothse, il recoupe chacun des n premiers plans en n
n(n + 1)
rgions sur le plan Pn+1 . Chacune
droites vrifiant les conditions du 1). Ces n droites dlimitent P(n) = 1 +
2
http ://www.maths-france.fr

c Jean-Louis Rouget, 2014. Tous droits rservs.


de ces rgions partage une des Q(n) rgions dj existantes en deux rgions et rajoute donc une nouvelle rgion. Ainsi,
n2 + n + 2
.
Q(n + 1) = Q(n) + P(n) = Q(n) +
2
Soit n > 2.

Q(n) = Q(1) +

n1
X

(Q(k + 1) Q(k)) = 2 +

k=1

n1
X
k=1

n1
n1
1X 2 1X
k2 + k + 2
) = 2 + (n 1) +
k +
k
2
2
2
k=1

k=1

(n 1)n(2n 1) n(n 1)
n3 + 5n + 6
= (n + 1) +
+
=
12
4
6

n n2 + 5
n3 + 5n + 6
=1+
.
n > 1, Q(n) =
6
6
Exercice no 17
Soient n et k des entiers naturels tels que 2 6 k 6 n 1.
Soit E un ensemble n lments et a un lment fix de E.
k
Il y a Pn
partitions de E en k classes. Parmi ces partitions, il y a celles dans lesquelles a est dans un singleton. Elles
k1
sidentifient aux partitions en k 1 classes de E \ {a} et sont au nombre de Pn1
. Il y a ensuite les partitions dans lesquelles
a est lment dune partie de cardinal au moins 2. Une telle partition est obtenue en partitionnant E \ {a} en k classes puis
k1
k
k
k
en adjoignant lune de ces k classes au choix llment a. Il y a k Pn1
telles partitions. Au total, Pn
= Pn1
+ kPn1
.
k
Valeurs de Pn
pour 1 6 k, n 6 5.

15

25

10

n
1

k
Exprimons maintenant en fonction des Pn
le nombre de surjections dun ensemble n lments sur un ensemble p
lments. Soient En et Ep dsignent des ensembles n et p lments respectivement.
Si p > n, il ny a pas de surjections de En dans Ep .
On suppose dornavant p 6 n. La donne dune surjection f de En sur Ep quivaut la donne dune partition de
lensemble En en p classes (chaque lment dune mme classe ayant mme image par f) puis dune bijection de lensemble
des parties de la partition vers Ep .
p
Au total, il y a donc p!Pn
surjections dun ensemble n lments dans un ensemble p lments pour 1 6 p 6 n.

Exercice no 18
Soit Y0 une partie fixe de E. Notons k le cardinal de Y0 . k est un lment de J0, nK. Le nombre de couples (X, Y0 ) (P(E))2
tels que X Y0 est encore le nombre de parties X de Y0 savoir 2k .
 
n
Soit k J0, nK fix. Il y a
parties Y de E de cardinal k et pour chaque partie Y de E de cardinal k, il y a 2k couples
k
 
n
2
2
(X, Y) (P(E)) tels que X Y. Au total, il y a
2k couples (X, Y) (P(E)) tels que X Y et card(Y) = k.
k
2

Le nombre de couples (X, Y) (P(E)) tels que X Y est donc


n  
X
n k
2 = (1 + 2)n = 3n .
k

k=0

Il y a 3n couples (X, Y) (P(E))2 tels que X Y.

http ://www.maths-france.fr

c Jean-Louis Rouget, 2014. Tous droits rservs.


Exercice no 19
Lapplication : X 7 CE (X) = X est une permutation de P(E) car est involutive. Donc,
X

2S1 =

(X,Y)(P(E))2

card(X Y) +

(X,Y)(P(E))2

card(X Y) + card X Y

card(X) +

(X,Y)(P(E))2

card X Y



card(X),

(X,Y)(P(E))2

puis

2 2S1 =

(X,Y)(P(E))2

(X,Y)(P(E))2


card X =

card(E)

(X,Y)(P(E))2


2
= card(E) card P(E)2 = n (2n ) ,

et donc S1 =

n22n
= n22n2 . Dautre part,
4
S2 =

card(X Y) =

(n card (X Y)) = n22n S2 = n22n n22n2 = 3n22n2 .

(X,Y)(P(E))2

(X,Y)(P(E))2

(X,Y)(P(E))2


card X Y =

(X,Y)(P(E))2

card X Y

S1 = n22n2 et S2 = 3n22n2 .
Exercice no 20
1) Une loi de composition interne sur E est une application de E E dans E (et rciproquement). Le nombre de lois de
composition interne sur E est donc

2
card EEE = n(n ) .

2) Posons E = {a1 , . . . , an }. La donne dun ldci commutative quivaut la donne des ai aj , 1 6 i 6 j 6 n. Il y a n


n(n + 1)
valeurs possibles pour chaque ai aj et
couples (i, j) tels que 1 6 i 6 j 6 n. Le nombre de lois de composition
2
interne sur E, commutatives est donc
n(n(n+1)/2) .
3) Il y a n choix possibles de llment neutre. Pour chaque choix i tel que ai soit lment neutre, les valeurs des ai aj ,
1 6 j 6 n et des aj ai , 1 6 j 6 n, sont imposes. Sinon, le nombre de choix pour les aj ak , j 6= i, k 6= i, est encore le
2
nombre dapplications de ({a1 , . . . , an } \ {ai }) vers {a1 , . . . , an }. Le nombre de lois de composition interne sur E possdant
un lment neutre est donc
2
2
n n((n1) ) = nn 2n+2 .

http ://www.maths-france.fr

10

c Jean-Louis Rouget, 2014. Tous droits rservs.


Planche no 38. Probabilits


* trs facile ** facile *** difficult moyenne **** difficile
I : Incontournable T : pour travailler et mmoriser le cours
Exercice no 1 : (*T) (extrait de la revue Scientific American de 1959)
Mr. Jones has two children. The older child is a girl. What is the probability that both children are girls ? (cest--dire :
Mr Jones a deux enfants. Laine est une fille. Quelle est la probabilit que les deux enfants soient des filles ?)
Mr. Smith has two children. At least one of them is a boy. What is the probability that both children are boys ? (cest--dire
Mr Smith a deux enfants. Lun des deux est un garon. Quelle est la probabilit que les deux enfants soient des garons ?
Exercice no 2 : (*T)
Soient A et B deux vnements dun espace probabilis tels que p(A) = 0, 4 et p(B) = 0, 6 et pA (B) = 0, 4.
Calculer pA (B), p(A B) et pB (A).
Exercice no 3 : (**T)
1) On tire 5 cartes simultanment dans un jeu de 32 cartes. Quelle est la probabilit davoir exactement 3 curs ?
2) On tire 5 cartes successivement et sans remise dans un jeu de 32 cartes. Quelle est la probabilit davoir exactement
3 curs ?
3) On tire 5 cartes successivement et avec remise dans un jeu de 32 cartes. Quelle est la probabilit davoir exactement
3 curs ?
Exercice no 4 : (**T)
Une urne contient 4 boules blanches et 3 noires. On tire 3 boules une par une sans remise.
Quelle est la probabilit que la premire soit blanche, la seconde blanche et la troisime noire ?
Exercice no 5 : (**T)
Dans une loterie, il y a 30 billets dont n sont gagnants. On suppose que tous les billets ont la mme probabilit dtre
achets. On achte 2 billets au hasard. Dterminer la probabilit de ne rien gagner et en dduire la valeur de n partir
de laquelle on a 90% de chance de gagner.
Exercice no 6 : (**T)
On jette deux ds non pips, un d noir et un d blanc.
Soient A lvnement : le chiffre du d noir est pair , B lvnement : le chiffre du d blanc est impair , C
lvnement : les deux chiffres ont mme parit .
Montrer que A et B, A et C, B et C sont indpendants mais que les trois vnements A, B et C ne le sont pas.
Exercice no 7 : (**)
On dispose de 2n cartons numrotes de 1 2n. On les tire un par un au hasard. Quelle est la probabilit que les numros
impairs soient tous avant les numros pairs ?
Exercice no 8 : (**)
Un jeu de 32 cartes est truqu. Une des cartes, autre que las de cur, a t remplace par un second as de cur.
1) On tire 3 cartes simultanment. Quelle est la probabilit que lon se rende compte de la supercherie ?
2) Et si lon tire 4 cartes ? 5 cartes ?
3) A partir de combien de cartes a-t-on au moins une chance sur deux de voir la supercherie ?
Exercice no 9 : (**)
Soit N un entier strictement positif, et p un rel strictement compris entre 0 et 1.
Une particule se dplace sur une droite en faisant des sauts dune unit vers la gauche ou vers la droite. A chaque instant,
la probabilit quelle aille vers la droite est p et celle quelle aille vers la gauche q = 1 p, tous ces dplacements tant
supposs indpendants.
Initialement, la particule est en n J0, NK, et elle sarrte ds quelle atteint lune des extrmits de cet intervalle : 0 ou
N.
On note qn la probabilit que la particule sarrte en 0.
1) a) Justifier que q0 = 1 et qN = 0.
b) Montrer que pour tout n tel que 1 6 n 6 N 1, on a
qn = pqn+1 + qqn1
http ://www.maths-france.fr

c Jean-Louis Rouget, 2014. Tous droits rservs.


c) En dduire une expression de qn en fonction de n, N, p et q.


On pensera distinguer les cas p = 1/2 et p 6= 1/2.
2) Calculer de mme la probabilit pn que la particule sarrte en N (mmes cas).
3) Calculer pn + qn , et en dduire la probabilit pour que la particule ne sarrte jamais.
Exercice no 10 : (**Oral CCP)
1) Enoncer et dmontrer la formule de Bayes.
2) On dispose de 100 ds dont 25 sont pips.
1
.
2
a) On tire un d au hasard parmi les 100 ds. On lance ce d et on obtient le chiffre 6.
Quelle est la probabilit que ce d soit pip ?
Pour chaque d pip, la probabilit dobtenir le chiffre 6 lors dun lancer vaut

b) Soit n N .
On tire un d au hasard parmi les 100 ds. On lance ce d n fois et on obtient n fois le chiffre 6.
Quelle est la probabilit pn que ce d soit pip ?
c) Dterminer

lim pn . Interprter ce rsultat.

n+

Exercice no 11 : (**Oral CCP)


On dispose de deux urnes U1 et U2 .
Lurne U1 contient deux boules blanches et trois boules noires.
Lurne U2 contient quatre boules blanches et trois boules noires.
On effectue des tirages successifs dans les conditions suivantes :
on choisit une urne au hasard et on tire une boule dans lurne choisie.
On note sa couleur et on la remet dans lurne do elle provient.
Si la boule tire tait blanche, le tirage suivant se fait dans lurne U1 .
Sinon, le tirage suivant se fait dans lurne U2 .
n N , on note Bn lvnement la boule tire au n-me tirage est blanche .
On pose galement n N , pn = P (Bn ).
1) Calculer p1 .
4
6
pn + .
35
7
3) En dduire, pour tout entier naturel n non nul, la valeur de pn .

2) Prouver que : n N , pn+1 =

Exercice no 12 : (***) (le problme du scrutin)


Au cours dune lection, deux candidats A et B saffrontent. Le candidat A lemporte sur le candidat B par a voix contre
b (a > b). On veut calculer la probabilit quau cours du dpouillement le candidat A ait t constamment en tte.
1) Dans le plan rapport un repre orthonorm, on considre les points (, ) et ( + m, + m) o (, , m, n) N4 .
On va de (, ) ( + m, + m) par dplacements successifs de une unit vers la droite ou une unit vers le haut chaque
tape. On appelle chemin de (, ) et ( + m, + m) un tel trajet.
Combien y-a-t-il de chemins de (, ) et ( + m, + m) ?
2) a) Montrer quun chemin de (0, 1) (a, b), a au moins un point commun avec la droite dquation y = x.
b) Montrer quil y a autant de chemins de (0, 0) (a, b), passant par (1, 0) et rencontrant la droite dquation y = x
en au moins un point distinct de (0, 0) que de chemins de (0, 0) (a, b) passant par (0, 1).
c) En dduire le nombre de chemins de (0, 0) (a, b) situs en dessous de et ne rencontrant quen (0, 0).
3) Dterminer la probabilit quau cours du dpouillement le candidat A ait t constamment en tte.
Exercice no 13 : (***) (le problme des allumettes de Banach)
Une personne porte tout moment deux botes dallumettes, une dans la poche gauche, lautre dans la poche droite.
Chaque fois quelle a besoin dune allumette, elle choisit au hasard dans une de ses botes. Elle dcouvre subitement que
la bote tire est vide. Les deux botes contenaient initialement n allumettes chacune.
Quelle est la probabilit quil lui reste k allumettes dans lautre bote ?
Quand n = 6, combien reste-t-il dallumettes dans la poche non vide en moyenne ?

http ://www.maths-france.fr

c Jean-Louis Rouget, 2014. Tous droits rservs.


Exercice no 14 : (***) (le problme de la ruine du joueur)


Deux joueurs A et B qui disposent au dpart dun capital de n et s n respectivement (0 6 n 6 s) jouent pile ou face.
Le joueur A parie toujours sur pile, la probabilit de pile est gale p (p ]0, 1[). A chaque coup, le joueur perdant donne
1 euro lautre. Le jeu se poursuit jusqu la ruine de lun des deux joueurs.
Quelles sont les chances de chacun des joueurs de ruiner son adversaire ?

http ://www.maths-france.fr

c Jean-Louis Rouget, 2014. Tous droits rservs.


Planche no 38. Probabilits : corrig


Exercice no 1
Lunivers est dans les deux cas = {(F, F), (F, G), (G, F), (GG)}. Les vnements lmentaires sont quiprobables.
2
1
Famille Jones. Lvnement laine est une fille est lvnement A = {(F, F), (F, G)} avec p(A) = = .
4
2
1
Lvnement les deux enfants sont des filles est lvnement B = {(F, F)} avec p(B) = . La probabilit demande est
4
p(B A)
1/4
1
pA (B) =
=
= .
p(A)
2/4
2
Famille Smith. Lvnement lun des deux enfants est un garon est lvnement C = {(G, G), (G, F), (F, G)} avec
3
p(C) = .
4
1
Lvnement les deux enfants sont des garons est lvnement D = {(G, G)} avec p(D) = . La probabilit demande
4
est
pC (D) =

p(D C)
1/4
1
=
= .
p(C)
3/4
3

Exercice no 2
Reprsentons la situation par un arbre de probabilits.
B
A
0, 4

0, 4

0, 6

A
B

p(B) = p(A) pA (B) + p A pA (B) et donc 0, 6 = 0, 4 pA (B) + 0, 6 0, 4 puis
pA (B) =

0, 6 0, 6 0, 4
= 0, 9.
0, 4

p(A B) = p(A) pA (B) = 0, 4 0, 9 = 0, 36 puis


p(A B) = P(A) + p(B) p(A B) = 0, 4 + 0, 6 0, 36 = 0, 64.
pB (A) =

0, 4(1 0, 9)
p(A) pA (B)
= 0, 1.
=
1 0, 6
p(B)

Exercice no 3
1) Lunivers est ici lensemble des parties 5 lments dun ensemble 32 lments. Les vnements lmentaires sont
quiprobables.
 
32
32 31 30 29 28
card() =
=
= 8 31 29 28 = 201 376.
5
5432

Il y a ainsi 201 376 cas possibles. Pour les cas favorables, on tire 3 cartes parmi les 8 curs et 2 cartes parmi les 24 qui
ne sont pas des curs. Au total, il y a
   
8
8 7 6 24 23
24

= 56 12 23 = 15 456.

=
32
2
3
2

Il y a 15 456 mains de 5 cartes contenant exactement 3 curs. La probabilit dobtenir exactement 3 curs est donc
p=
http ://www.maths-france.fr

15 456
56 12 23
3 23
69
=
=
=
= 0, 076 . . .
201 376
8 31 29 28
31 29
899
1

c Jean-Louis Rouget, 2014. Tous droits rservs.


2) Lunivers est ici lensemble des tirages successifs sans remise de 5 lments dans un ensemble 32 lments. Les
vnements lmentaires sont quiprobables.
card() = 32 31 30 29 28 = 24 165 120.

 
5
Il y a ainsi 24 165 120 cas possibles. Pour les cas favorables, on choisit dabord lemplacement des curs. Il y a
= 10
3
emplacements des 3 curs. Pour chacun des ces emplacements, il y a 8 7 6 choix des curs et 24 23 choix des cartes
qui ne sont pas des curs. Le nombre de cas favorables est donc
 
5
(8 7 6) (24 23) = 1 854 720.
3
Il y a 1 854 720 tirages successifs sans remise de 5 cartes contenant exactement 3 curs. La probabilit dobtenir exactement
3 curs est donc
10 (8 7 6) (24 23)
3 23
69
1 854 720
=
=
=
= 0, 076 . . .
24 165 120
32 31 30 29 28
31 29
899
On note que la probabilit ne change pas.
p=

3) Lunivers est ici lensemble des tirages successifs avec remise de 5 lments dans un ensemble 32 lments. Les
vnements lmentaires sont quiprobables.
card() = 325 = 33 554 432.
 
5
Il y a ainsi 33 554 432 cas possibles. Pour les cas favorables, on choisit dabord lemplacement des curs. Il y a
= 10
3
emplacements des 3 curs. Pour chacun des ces emplacements, il y a 83 choix des curs et 242 choix des cartes qui ne
sont pas des curs. Le nombre de cas favorables est donc
 
5
83 242 = 2 949 120.
3
Il y a 2 949 120 tirages successifs avec remise de 5 cartes contenant exactement 3 curs. La probabilit dobtenir exactement
3 curs est donc
2 949 120
= 0, 087 . . .
33 554 432
On note que la situation du 3) est en fait un schma de Bernoulli.
p=

Exercice no 4
Pour avoir les ides claires, on diffrencient les boules de mme couleur en crivant des numros sur ces boules, ce qui ne
change rien aux probabilits calculer. Les sept boules de lurne sont alors B1, B2, B3, B4, N1, N2 et N3.
On peut prendre pour univers lensemble des tirages successifs sans remise de 3 lments dans un ensemble 7 lments.
Les vnements lmentaires sont quiprobables.
car() = A37 = 7 6 5 = 210. Pour les cas favorables, il y a 4 3 = 12 tirages des deux premires boules tels que ces
deux premires boules soient blanches et pour chacun de ces tirages, il y a 3 possibilits que la troisime boule soit noire.
Au total, 4 3 3 = 36 cas favorables. La probabilit demande est

Exercice no 5

433
23
6
=
=
= 0, 17 . . .
765
75
35



30 n
n2 59n + 870
(30 n)(29 n)
2
=
.
La probabilit de ne rien gagner est pn =   =
30
30 29
870
2

59 349
59 + 349
n2 59n + 870
2
6 0, 1 n 59n + 783 6 0
6n6
1 pn > 0, 9
870
2
2
20, 1 . . . 6 n 6 38, 8 . . . n > 21.
A partir de 21 billets gagnants, on a au moins 9 chances sur 10 de gagner.

Exercice no 6
http ://www.maths-france.fr

c Jean-Louis Rouget, 2014. Tous droits rservs.


Les rsultats de lexprience sont des couples o la premire composante est le numro obtenu sur le d noir et la deuxime
composante est le numro obtenu sur le d blanc. On peut donc prendre = J1, 6K2 puis card() = 36.
1
63
1
63
36
= , p(B) =
= et p(C) =
=
66
2
66
2
66
33
1
33
p(A B) =
= = p(A) p(B), p(A C) =
66
4
66
Les vnements A, B et C sont deux deux indpendants.
p(A) =

A B C = et donc p(A B C) = 0 6=

1
.
2
=

1
33
1
= p(A) p(C) et p(B C) =
= = p(B) p(C).
4
66
4

1
= p(A) p(B) p(C). A, B et C ne sont pas mutuellement indpendants.
8

Exercice no 7
est lensemble des permutations de lensemble des entiers de 1 2n. card() = (2n)! et les vnements lmentaires
sont quiprobables.
Pour les cas favorables, il y a n! possibilits pour les n premiers cartons et pour chacune de ces possibilits, il y a n!
possibilits pour les n derniers. Il y a donc n! n! = n!2 cas favorables. La probabilit demande est
p=

n! n!
1
= n .
(2n)!
C2n

Exercice no 8
1) p =

30
C22 C130
3
3
=
=
=
= 0, 006 . . .
32 31 5
16 31
496
C332

30 29
43
3
3
C22 C230
2
=
=
=
= 0, 01 . . .
=
2) Avec 4 cartes, p =
32 31 30 29
32 31
8 31
248
C432
432
30 29 28
C2 C3
54
5
5
32
Avec 5 cartes, p = 2 5 30 =
=
=
=
= 0, 02 . . .
32 31 30 29 28
32 31
8 31
248
C32
5432
30!
n2
C22 C30
n(n 1)
(n 2)!(32 n)!
=
=
puis
3) Avec n cartes, 3 6 n 6 32, p =
32!
Cn
32 31
32
n!(32 n)!

1 + 1985
2
pn > 0, 5 n(n 1) > 496 n n 496 > 0 n >
n > 22, 7 . . . n > 23.
2
A partir de 23 cartes tires, on a au moins une chance sur deux de se rendre compte de la supercherie.
Exercice no 9
1) a) Quand n = 0, la particule est initialement en 0 et sarrte donc immdiatement en 0 avec certitude. Donc q0 = 1.
Quand n = N, la particule sarrte immdiatement en N avec certitude et donc ne sarrte pas en 0 avec certitude. Donc
qN = 0.
b) Si on note An lvnement la particule est en n, An est la runion disjointe des vnements la particule va en n + 1 et
sarrte en 0 et de lvnement la particule va en n 1 et sarrte en 0 de probabilits respectives p qn+1 (produit
de la probabilit daller vers la droite et donc darriver en n + 1 par la probabilit de sarrter en 0 sachant quon est en
n + 1) et q qn1 . Donc, qn = p qn+1 + q qn1 .
c) Pour tout n J1, N 1K, on a pqn+1 qn + qqn1 = 0. Lquation caractristique associe est pz2 z + (1 p) = 0.
Le discriminant de cette quation est = 1 4p(1 p) = 4p2 4p + 1 = (2p 1)2 .

1 |2p 1|
1 (2p 1)
1
, lquation caractristique admet deux solutions relles distinctes savoir
=
2
2p
n


 2p
n
1
1
q
ou encore 1 et 1. On sait quil existe et tels que pour tout n J0, NK, qn = +
.
1
=+
p
p
p

=1
+ 
N
pN
qN
et = N
puis
et donc = N
Les conditions q0 = 1 et qN = 0 fournissent
q
N
+
p q
p qN
=0
p
 n
qN
q
pN
qn = N
.
+ N
N
N
p q
p q
p
1er cas. Si p 6=

http ://www.maths-france.fr

c Jean-Louis Rouget, 2014. Tous droits rservs.


1
, lquation caractristique admet une solution relle double savoir 1. On sait quil existe et tels
2
que pour tout n J0, NK, qn = + n. Les conditions q0 = 1 et qN = 0 fournissent

2me cas. Si p =

qn =

nN
n
=1 .
N
N

2) De nouveau,
 n
q
1
.
, il existe deux rels et tels que pour tout n J0, NK, pn = +
2
p

=0
+ 
N
pN
pN
Les conditions p0 = 0 et pN = 1 fournissent
et donc = N
et = N
puis
q
N
+
p q
p qN
=1
p
n

pN
pN
1p
pn = N
.

p qN pN qN
p

1er cas. Si p 6=

1
2me cas. Si p = , il existe et tels que pour tout n J0, NK, pn = + n. Les conditions p0 = 0 et pN = 1
2
fournissent
n
.
N

pn =

n
n
1

1
+
si p =
N
N
2
 n
 n
3) pn + qn =
pN
1
pN
q
pN
q
(1 p)N


+
si p 6=
+

N
N
N
N
N
N
N
N
p q
p q
p
p q
p q
p
2
La probabilit pour que la particule ne sarrte jamais est 1 (pn + qn ) = 0.

= 1.

Exercice no 10
1) Formule de Bayes.
Soit (, P) un espace probabilis.
Soit (Ai )16i6n un systme complet dvnements de cet espace tel que pour tout i J1, nK, P (Ai ) 6= 0.
Soit B un vnement tel que P(B) 6= 0. Alors,
i J1, nK, PB (Ai ) =

P (Ai ) PAi (B)


.
n
X
P (Aj ) PAj (B)
j=1

Dmonstration. Soit i J1, nK. Puisque P(B) 6= 0,


PB (Ai ) =

P (Ai B)
P (Ai ) PAi (B)
=
.
P(B)
P(B)

Puisque (Aj )16j6n un systme complet dvnements de cet espace tel que pour tout j J1, nK, P (Aj ) 6= 0, on a
P(B) =

n
X
j=1

P (Aj B) =

n
X
j=1

P (Aj ) PAj (B).

Donc,
PB (Ai ) =

P (Ai ) PAi (B)


.
n
X
P (Aj ) PAj (B)
j=1

2) a) Notons A lvnement le d est pip et B lvnement on obtient le chiffre 6 . La


PB (A).


25
1
1
= 6= 0 et P A = 1 =
A, A est un systme complet dvnements. On a P(A) =
100
4
4
1
PA (B) = . Donc,
6

1 1 3 1
1
P(B) = P(A) PA (B) + P A PA (B) = + = 6= 0.
4 2 4 6
4
http ://www.maths-france.fr

probabilit demande est


3
1
. Ensuite PA (B) = et
4
2

c Jean-Louis Rouget, 2014. Tous droits rservs.


Daprs la formule de Bayes,


1 1

1
P(A) PA (B)

= 4 2 = .
PB (A) =
1
2
P(A) PA (B) + P A PA (B)
4
1
La probabilit que ce d soit pip est .
2
b) Notons A lvnement le d est pip et B lvnement on obtient n fois le chiffre 6 . La probabilit demande
est PB (A).


1
3
1
A, A est un systme complet dvnements. On a toujours P(A) =
6= 0 et P A = . Ensuite PA (B) = n et
4
4
2
1
PA (B) = n . Donc,
6

1
3
1
1
P(B) = P(A) PA (B) + P A PA (B) = n + n 6= 0.
4 2
4 6
Daprs la formule de Bayes,
1
1
n
1
P(A) PA (B)
4
2

=
.
PB (A) =
=
3
1
1
1
1
P(A) PA (B) + P A PA (B)
+

1 + n1
4 2n 4 6n
3
1
La probabilit que ce d soit pip est
.
1
1 + n1
3
1
c) lim n1 = 0 et donc lim pn = 1. Ceci signifie que si au bout dun grand nombre de lancers, on a obtenu
n+
n+ 3
chaque fois le 6, il est quasiment sr que le d est pip.
Exercice no 11
Pour n N , notons An lvnement au n-me tirage, la boule provient de lurne U1 (lvnement An est donc
lvnement au n-me tirage, la boule provient de lurne U2 ).

 1
1) A1 , A1 est un systme complet dvnements et P (A1 ) = P A1 = 6= 0. Daprs la formule des probabilits totales,
2

1 2 1 4
17
p1 = P (B1 ) = P (A1 ) PA1 (B1 ) + P A1 PA1 (B1 ) = + =
.
2 5 2 7
35
17
.
La probabilit p1 que la premire boule tire soit blanche est
35

2) Soit n N . Bn , Bn est un systme complet dvnements. Daprs la formule des probabilits totales,

P (Bn+1 ) = P (Bn ) PBn (Bn+1 ) + P Bn PBn (Bn+1 )
4
6
4
2
= pn + (1 pn ) = pn + .
5
7
35
7
3) La suite (pn )nN est arithmtico-gomtrique.
6
4
La fonction affine x 7 x + admet un point fixe et un seul :
35
7
4
41
4
20
6
x= x=
.
x= x+
35
7
35
7
41


20
20
6
pn
puis que pour tout entier naturel non
On sait alors que pour tout entier naturel non nul n, pn+1
=
41
35
41
nul n,
20
pn
=
41

n1 
 
n1 

n1


20
6
3
17 20
6
6
p1
=
=
,


35
41
35
35 41
1435
35

et donc

http ://www.maths-france.fr

c Jean-Louis Rouget, 2014. Tous droits rservs.


n1

20
3
6


.
41 1435
35

n1
20
3
6
Pour tout entier naturel non nul n, pn =
.


41 1435
35
pn =

Exercice no 12
1) Un chemin de (, ) ( + m, + n) est un mot de m + n lettres comportant m lettres D (pour droite) et n
lettes H (pour haut) du type DHHDDDH. . . H. Le nombre
 de ces chemins est le nombre de choix des emplacements des
(m + n)!
m+n
.
m lettres D dans les m + n positions. Il y en a
=
m!n!
m
2) a) Soit C un chemin allant de (1, 0) (a, b). Soit E lensemble des abscisses x des points (x, y) de C tels que x 6 y.
E est une partie non vide de N (car 0 E ) et majore (par a). Donc E admet un plus grand lment. Notons le c.
De mme, on peut dfinir d la plus grande ordonne dun point de C dabscisse c. Par construction, c 6 d. c nest pas a
car a > b > d et donc c 6 a 1. Par dfinition de c et d, le point de C qui suit (c, d) est (c + 1, d) avec c + 1 > d. Ainsi,
c 6 d et c > d 1. On en dduit que c = d. Par construction le point (c, c) est un point de C et de .

b) Notons E lensemble des chemins de (0, 0) (a, b), passant par (1, 0) et rencontrant la droite dquation y = x en
au moins un point distinct de (0, 0) et F lensemble des chemins de (0, 0) (a, b) passant par (0, 1). On va construire une
bijection de E sur F.

(a, b)

C
b

C
b

(0, 0)

Soit C un lment de E. On note C1 la partie de C qui va de (0, 0) au premier point de C autre que (0, 0) qui se trouve
sur puis C2 = C \ C1 . C ne rencontre pas en (a, b) (car a > b) et donc C rencontre en un point distinct de (0, 0)
strictement avant (a, b). Donc (a, b) C2 et en particulier, C2 6= . De mme, (0, 0) et (1, 0) appartiennent C1 et donc
C1 6= .
Notons alors C1 le symtrique de C1 par rapport et enfin C = C1 C2 . Puisque (1, 0) C1 , (0, 1) C1 et donc
C F.
On considre f :

E F . Daprs ce qui prcde, f est une application de E vers F.


C 7 C
De mme, on peut considrer g : F E . Daprs a), un chemin C de F a ncessairement un point commun avec
C 7 C
autre que (0, 0). On peut lui appliquer la transformation prcdente et on obtient un chemin C de E.
Pour tout chemin C de E, on a g (f (C )) = C et pour tout chemin C de F, on a f (g (C )) = C . Donc g f = IdE et
f g = IdF .
On sait alors que f est bijective. En particulier, card(F) = card(E).
c) Un chemin de (0, 0) (a, b) situ en dessous de passe par (1, 0). Le nombre
total de

 chemins de (0, 0) (a, b) passant
a+b1
par (1, 0) est encore le nombre total de chemins de (1, 0) (a, b). Il y en a
daprs la question 1). Le nombre
a 1

a+b1
de chemins de (0, 0) (a, b) situs en dessous de est la diffrence entre
et le nombre de chemin passant par
a1
http ://www.maths-france.fr

c Jean-Louis Rouget, 2014. Tous droits rservs.


(1, 0) et rencontrant en au moins un point distinct de (0, 0) savoir card(E). Daprs la question b), card(E) = card(F)
et donc le nombre cherch est





 

a+b1
a+b1
a+b1
a+b1
card(E) =
card(F) =

.
a1
a1
a1
a
3) En reprsentant chaque tape du dpouillement par un couple (x, y) o x est le nombre de voix de A et y est le nombre
de voix de B, un dpouillement est un chemin de (0, 0) (a, b) et un dpouillement o A est constamment en tte
est est un chemin de (0, 0) (a, b) situ en dessous de et ne rencontrant quen (0, 0). La probabilit demande
est donc

 

a+b1
a+b1



a!b!
a
b
ab
(a + b 1)! (a + b 1)!
a1
a



=
.
=
p=
a+b
(a 1)!b!
a!(b 1)!
(a + b)!
a+b a+b
a+b
a
Exercice no 13
Soient n N puis k J1, nK.
Supposons par exemple que la personne se rende compte que la bote dallumettes qui est dans sa poche gauche est vide.
A chaque tape, on peut reprsenter le nombre dallumettes de chaque bote par un couple (x, y) o x est le nombre
dallumettes prises dans la poche gauche et y est le nombre dallumettes de la poche droite.
Prendre une allumette la fois dans lune ou lautre poche consiste ajouter 1 x ou y et donc faire un dplacement
dune unit vers la droite ou vers le haut. On veut prendre n allumettes dans la poche gauche et n k dans la poche droite
(pour quil en reste k) et donc on veut atteindre le point (n, n k). Tout ceci permet didentifier une succession de prises
dallumettes nous laissant une bote vide gauche et une bote contenant k allumettes droite un chemin joignant le
point (0, 0) au point (n, n k) comme dans lexercice prcdent.
On se rend compte que la bote de la poche gauche est vide en effectuant un dplacement supplmentaire vers la droite.
Le nombre de prises dallumettes cherch est donc le nombre de chemins joignant (0, 0) (n + 1, n 
k) et se 
terminant par
2n k
un dplacement vers la droite. Il y en a autant que de chemins joignant (0, 0) (n, n k) savoir
. Un chemin
n
joignant (0, 0) (n + 1, n k) est de longueur 2n k + 1. Comme chaque tape, la probabilit de choisir une des deux
1
1
botes est et que les choix successifs sont mutuellement indpendants, la probabilit dun tel chemin est 2nk+1 . La
2
2

2n k
n
probabilit quon se rende compte que la bote gauche est vide et que la bote droite contient k allumettes est 2nk+1 .
2
Cette probabilit est la mme si cest la bote droite qui est vide et la bote gauche contient k allumettes. La probabilit
demande est




2n k
2n k
n
n
p = 2 2nk+1 =
.
2
22nk


2n k
n
X
n
k 2nk ce qui scrit quand n = 6,
En moyenne, il reste dans la poche pas vide
2
k=1


12 k
6
X
462 420 252 112 35
6
3958
6
k 12k = 11 + 10 + 9 + 8 + 7 + 6 =
= 1, 9 . . .
2
2
2
2
2
2
2
2048
k=1

Exercice no 14
La situation est la mme que celle de lexercice no 9 o on remplace la particule et sa position initiale par la somme que
possde le joueur A initialement savoir n. Celui-ci est ruin si son capital arrive 0 et son adversaire est ruin si le
capital du joueur A arrive n + s n = s.
On reprend les notations du no 9 en remplaant simplement N par s. qn est alors la probabilit que A soit ruin et pn est
la probabilit que B soit ruin.
n

1
ps
1p
n
1
(1 p)s
si p 6= et 1 si p = .
+
La probabilit que A soit ruin est s
p (1 p)s ps (1 p)s
p
2
s
2
http ://www.maths-france.fr

c Jean-Louis Rouget, 2014. Tous droits rservs.


ps
ps
La probabilit que B soit ruin est s

p (1 p)s
ps (1 p)s

1p
p

n

si p 6=

1
n
1
et
si p = .
2
s
2

On peut noter que la probabilit quun des deux joueurs soit ruin en un temps fini est pn + qn = 1 dans tous les cas.

http ://www.maths-france.fr

c Jean-Louis Rouget, 2014. Tous droits rservs.

You might also like